Class 11

HBSE 11th Class Political Science Important Questions Chapter 10 संविधान का राजनीतिक दर्शन

Haryana State Board HBSE 11th Class Political Science Important Questions Chapter 10 संविधान का राजनीतिक दर्शन Important Questions and Answers.

Haryana Board 11th Class Political Science Important Questions Chapter 10 संविधान का राजनीतिक दर्शन

अति लघूत्तरात्मक प्रश्न

प्रश्न 1.
42वें संशोधन द्वारा संविधान की प्रस्तावना में कौन-से दो शब्द जोड़े गए थे? यह संशोधन कब पारित हुआ था ?
उत्तर:
संविधान का 42वां संशोधन सन् 1976 में पारित हुआ। इस द्वारा प्रस्तावना में ‘समाजवादी’ (Socialist) तथा धर्म-निरपेक्ष (Secular) शब्द जोड़े गए थे।

प्रश्न 2.
भारत एक प्रभुसत्ता सम्पन्न राज्य है, व्याख्या करें।
उत्तर:
इसका अर्थ यह है कि भारत अपने आन्तरिक तथा विदेशी मामलों में पूर्ण रूप से स्वतन्त्र है और यह किसी अन्य विदेशी शक्ति के नियन्त्रण में नहीं है। भारत अन्तर्राष्ट्रीय क्षेत्रों में अपनी इच्छानुसार आचरण कर सकता है और वह किसी भी अन्तर्राष्ट्रीय सन्धि अथवा समझौते को मानने के लिए बाध्य नहीं है। भारत अपनी आन्तरिक तथा बाहरी नीति के निर्माण में पूर्ण रूप से स्वतन्त्र है।

प्रश्न 3.
किसी देश के संविधान में अंकित प्रस्तावना का महत्त्व क्यों होता है? कोई दो कारण लिखिए।
उत्तर:

  • प्रस्तावना उस देश के संविधान के संचालन के पथ-प्रदर्शन का कार्य करती है।
  • प्रस्तावना संविधान में निहित उद्देश्यों, आदर्शों एवं मूल्यों की ओर संकेत करती है।

प्रश्न 4.
संविधान के दार्शनिक पहलू से क्या तात्पर्य है?
अथवा
संविधान के राजनीतिक दर्शन का क्या अर्थ है?
उत्तर:
संविधान के दार्शनिक पहलू से तात्पर्य संविधान के सार से होता है जो एक देश के सामाजिक एवं राष्ट्रीय मूल्यों को प्रतिबिम्बित करता है।

प्रश्न 5.
किसी देश के संविधान के दर्शन में निहित किन्हीं दो बातों का उल्लेख कीजिए।
उत्तर:

  • संविधान के दर्शन में 3G देश के संविधान के आदर्श एवं सिद्धान्त निहित होते हैं।
  • संविधान के दर्शन में 3G देश के संविधान द्वारा अपनाई जाने वाली कुछ अवधारणाएँ; जैसे न्याय, समानता, विकास एवं स्थिरता आदि निहित होती हैं।

प्रश्न 6.
किसी भी लोकतान्त्रिक देश में संविधान के मूल्य या महत्त्व के कोई दो कारण लिखिए।
उत्तर:

  • संविधान, सरकार पर नियन्त्रण रखने और ‘जन स्वतन्त्रता’ की रक्षा का साधन है।
  • संविधान, लोकतन्त्रीय पद्धति के अनुरूप देश में शान्तिपूर्ण तरीके से बदलाव लाने का साधन है।

HBSE 11th Class Political Science Important Questions Chapter 10 संविधान का राजनीतिक दर्शन

प्रश्न 7.
भारतीय संविधान के उद्देश्य को संक्षेप में लिखिए।
उत्तर:
भारतीय संविधान का उद्देश्य है-धर्मनिरपेक्ष लोकतान्त्रिक गणतन्त्र स्थापित करना जो भारतीय नागरिकों को न्याय, स्वतन्त्रता और समानता का आश्वासन दे। इसके अतिरिक्त, संविधान भातृत्व को बढ़ावा देता है और व्यक्ति की गरिमा तथा राष्ट्र की अखण्डता को आश्वासन देता है।

प्रश्न 8.
भारतीय संविधान की प्रस्तावना से शासन के किन उद्देश्यों की स्पष्टता होती है?
उत्तर:

  • न्याय,
  • स्वतन्त्रता,
  • समानता,
  • बन्धुता एवं
  • राष्ट्र की एकता एवं अखण्डता।

प्रश्न 9.
भारतीय संविधान की प्रस्तावना की आलोचना के कोई दो आधार लिखिए।
उत्तर:

  • प्रस्तावना की न्यायिक मान्यता का अभाव होना।
  • समाजवाद एवं धर्मनिरपेक्षता; जैसे शब्दों की अस्पष्टता का होना।

प्रश्न 10.
भारतीय संविधान पर विदेशी संविधानों के प्रभाव के कारण इसे क्या कहकर आलोचना की जाती है?
उत्तर:
भारतीय संविधान पर विदेशी संविधानों के प्रभाव के कारण इसे ‘उधार ली गई वस्तुओं का थैला’ एवं विविध संविधानों की खिचड़ी’ कहकर आलोचना की जाती है।

प्रश्न 11.
भारतीय संविधान की आलोचना के किन्हीं दो आधारों का उल्लेख कीजिए।
उत्तर:

  • भारतीय संविधान अत्यधिक लम्बा और विस्तृत संविधान है।
  • भारतीय संविधान में अनेक प्रावधान ऐसे हैं जिनके पीछे कानूनी शक्ति का अभाव है।

प्रश्न 12.
भारतीय संविधान निर्माण पर किन देशों की छाप या प्रभाव अधिक दिखाई देता है? कोई पाँच देशों के नाम लिखिए।
उत्तर:

  • इंग्लैण्ड,
  • अमेरिका,
  • ऑस्ट्रेलिया,
  • जापान,
  • जर्मनी आदि।

प्रश्न 13.
भारतीय संविधान ‘उधार लिया गया थैला नहीं है। इस कथन के पक्ष में तर्क दीजिए।
उत्तर:
भारतीय संविधान निर्माताओं ने विदेशी संविधानों की आँखें मूंद कर नकल नहीं की बल्कि प्रत्येक व्यवस्था को भारतीय परिस्थितियों को ध्यान में रखते हुए ही संविधान में स्थान दिया जिससे हमारे संविधान में एक मौलिकता आ गई। ऐसे में हमारे संविधान को उधार का थैला कहना गलत है।

प्रश्न 14.
भारतीय संविधान की प्रस्तावना का क्या महत्त्व है?
उत्तर:
भारतीय संविधान की प्रस्तावना का बहुत महत्त्व है। संविधान सभा के सदस्य पं० ठाकुरदास भार्गव का कहना है कि प्रस्तावना संविधान का मुख्य तथा मूल्यवान भाग है। यह संविधान की आत्मा है। इसमें संवैधानिक शक्ति के स्रोत, भारतीय राज्य के स्वरूप, संविधान द्वारा प्राप्त किए जाने वाले उद्देश्यों और संविधान को स्वीकार किए जाने वाली तिथि का वर्णन किया गया है। इसी कारण से कई लोगों द्वारा इसे ‘संविधान की कुंजी’ या ‘संविधान का दर्पण’ कहा जाता है।

लघूत्तरात्मक प्रश्न

प्रश्न 1.
भारत एक गणराज्य (Republic) कैसे है?
उत्तर:
गणराज्य का अर्थ यह है कि देश का अध्यक्ष प्रत्यक्ष या अप्रत्यक्ष रूप से निर्वाचित पदाधिकारी होगा। हमारे देश के मुखिया अर्थात् राष्ट्रपति का पद पैतृक सिद्धान्त पर आधारित नहीं है। वह एक निश्चित काल (5 वर्ष) के लिए जनता के प्रतिनिधियों (संसद एवं राज्यों की विधानसभा के सदस्यों द्वारा) द्वारा निर्वाचित किया जाता है। अतः भारत गणराज्य होने की शर्त को पूर्ण करता है। यदि राष्ट्रपति की मृत्यु हो जाए तो 6 मास के अन्दर नए राष्ट्रपति का चुनाव करवाना संविधान के अनुसार आवश्यक है।

प्रश्न 2.
भारत एक धर्म-निरपेक्ष राज्य (Secular State) है, व्याख्या करें।
उत्तर:
यह शब्द संविधान की प्रस्तावना में संविधान के 42वें संशोधन के द्वारा जोड़ा गया है। इसका अर्थ है कि भारत में रहने वाले सभी लोगों को अपनी इच्छानुसार किसी भी धर्म को मानने तथा उसका प्रचार करने का अधिकार है। राज्य की दृष्टि में सभी धर्म समान हैं और धर्म के आधार पर राज्य किसी प्रकार के भेदभाव का प्रयोग नहीं करेगा।

सरकार किसी भी धर्म को कोई विशेष संरक्षण प्रदान नहीं करेगी। सरकार के द्वारा अपने नागरिकों को कोई ऐसा चन्दा या करं देने के लिए मजबूर नहीं किया जा सकता, जिससे प्राप्त आय किसी एक धर्म के प्रचार के लिए खर्च की जाने वाली हो । यद्यपि हमारे संविधान में धार्मिक स्वतन्त्रता के अधिकार का पहले से ही संकेत था, परन्तु अब यह शब्द जोड़ने से संविधान का उद्देश्य और भी स्पष्ट हो गया है।

प्रश्न 3.
भारत को समाजवादी (Socialist) राज्य क्यों कहा जाता है?
उत्तर:
यह शब्द संविधान की प्रस्तावना में संविधान के 42वें संशोधन के द्वारा जोड़ा गया है। इस शब्द का उद्देश्य है कि भारत में इस प्रकार की शासन व्यवस्था स्थापित की जाए जिससे समाज के सभी वर्गों को, विशेष रूप से पिछडे लिए उचित तथा समान अवसर प्राप्त हों, मनुष्य के द्वारा शोषण समाप्त हो और आर्थिक विषमता को कम किया जाए। काँग्रेस ने बहुत पहले से ही भारत में लोकतान्त्रिक समाजवाद की स्थापना करना अपना उद्देश्य घोषित किया था और उसी दल की सरकार द्वारा यह शब्द संविधान में सन् 1976 (42वें संशोधन द्वारा) में जोड़ दिया गया।

प्रश्न 4.
भारत एक लोकतान्त्रिक (Democratic) राज्य है, स्पष्ट करें।
उत्तर:
भारत एक लोकतान्त्रिक राज्य है जिसमें अन्तिम शक्ति जनता के हाथों में है। जनता वयस्क मताधिकार के आधार पर प्रत्येक पाँच वर्ष के लिए अपने प्रतिनिधियों का चुनाव करती है जो उस कार्यकाल में देश के शासन को चलाते हैं। मन्त्रिमण्डल जनता के चुने हुए प्रतिनिधियों अर्थात् संसद के द्वारा हटाया जा सकता है। अतः अन्तिम शक्ति जनता में निहित है। देश के सभी नागरिकों को समान अधिकार दिए गए हैं और उनकी रक्षा के लिए स्वतन्त्र न्यायपालिका की स्थापना की गई है।

प्रश्न 5.
‘प्रस्तावना’ (Preamble) का क्या अर्थ है?
उत्तर:
‘प्रस्तावना’ उस लेख (Document) को कहते हैं जिसे किसी संविधान के आरम्भ होने से पूर्व अंकित किया जाता है। इससे संविधान के मख्य उद्देश्यों, मौलिक सिद्धान्तों तथा उसके आदर्शों का सांकेतिक रूप में वर्णन किया जाता है। प्रस्तावना में उस समाज के सामाजिक, राजनीतिक तथा संवैधानिक ढाँचे का चित्र देखा जा सकता है। संविधान की प्रस्तावना एक ऐसा झरोखा होती है, जिसमें संविधान-निर्माताओं की भावनाओं और आशाओं का दृश्य देखा जा सकता है। यही कारण है कि प्रस्तावना को संविधान-निर्माताओं के ‘हृदय की कुंजी’ और ‘संविधान की आत्मा’ कहा जाता है।

प्रश्न 6.
भारतीय संविधान की प्रस्तावना (Preamble) लिखें।
उत्तर:
भारतीय संविधान की प्रस्तावना इस प्रकार है “हम, भारत के लोग, भारत को एक सम्पूर्ण प्रभुत्व सम्पन्न, समाजवादी, धर्म-निरपेक्ष, लोकतन्त्रात्मक गणराज्य बनाने के लिए। तथा उसके समस्त नागरिकों को सामाजिक, आर्थिक और राजनीतिक न्याय विचार, अभिव्यक्ति, विश्वास, धर्म और उपासना की स्वतन्त्रता प्रतिष्ठा और अवसर की समता प्राप्त कराने के लिए, तथा उन सबमें व्यक्ति की गरिमा और राष्ट्र की एकता और अखण्डता सुनिश्चित करने वाली बन्धुता बढ़ाने के लिए दृढ़ संकल्प होकर अपनी इस संविधान सभा में आज तारीख 26 नवम्बर, 1949 (मिति मार्गशीर्ष शुक्ल पक्ष सप्तमी, सम्वत् दो हजार छह विक्रमी) को एतद् द्वारा इस संविधान को अंगीकृत, अधिनियमित और आत्म-समर्पित करते हैं।

HBSE 11th Class Political Science Important Questions Chapter 10 संविधान का राजनीतिक दर्शन

प्रश्न 7.
संविधान की प्रस्तावना में वर्णित सामाजिक, आर्थिक तथा राजनीतिक न्याय पर संक्षिप्त नोट लिखें।
उत्तर:
न्याय शब्द से अभिप्राय यह है कि व्यक्ति के निजी हित और व्यवहार समाज के सामान्य हितों के अनुकूल हों। इस प्रकार न्याय का उद्देश्य वास्तव में सार्वजनिक भलाई ही है। इसमें सामाजिक, आर्थिक तथा राजनीतिक क्रियाएँ सम्मिलित हैं। सामाजिक न्याय से भाव यह है कि समाज का सदस्य होने के नाते एक व्यक्ति और दूसरे व्यक्ति में धर्म, जाति, नस्ल, रंग आदि के आधार पर भेद नहीं किया जा सकता। आर्थिक न्याय का अर्थ है कि प्रत्येक व्यक्ति को अपनी जीविका कमाने का समान अवसर प्राप्त है और उसे अपने काम की उचित मजदूरी मिलेगी।

राजनीतिक न्याय से अभिप्राय है कि राजनीति से सम्बन्धित क्रियाओं में प्रत्येक व्यक्ति बिना किसी भेदभाव के कोई भी पदवी प्राप्त कर सकता है तथा राजनीति में भाग ले सकता है। किसी भी व्यक्ति के साथ किसी भी आधार पर अर्थात् जाति, जन्म, वर्ग, वंश, कुल आदि के आधार पर भेदभाव नहीं किया जाएगा। सभी व्यक्तियों को इनमें से किसी भी आधार पर भेदभाव किए बिना वोट डालने, चुनाव लड़ने, सरकारी पद प्राप्त करने, राजनीतिक दल बनाने तथा सरकार की आलोचना करने के अधिकार प्रदान किए गए हैं।

प्रश्न 8.
भारतीय संविधान की प्रस्तावना की चार विशेषताएँ बताइए।
उत्तर:
भारतीय संविधान की प्रस्तावना का बहुत महत्त्व है। इसकी मुख्य विशेषताएँ इस प्रकार हैं-

  • भारत एक पूर्ण प्रभुसत्ता-सम्पन्न राज्य होगा, जिसका अर्थ है कि भारत अब व्यावहारिक या कानूनी रूप से किसी भी बाहरी शक्ति के अधीन नहीं है,
  • संविधान में 42वें संशोधन के पश्चात् समाजवाद की स्थापना की गई, जिससे राष्ट्र की उन्नति का लाभ समाज के सब लोगों को प्राप्त हो सके,
  • प्रस्तावना में लोकतन्त्र की स्थापना के लिए कहा गया है। संविधान ने प्रभुसत्ता किसी एक व्यक्ति अथवा वर्ग को नहीं, अपितु जनता को सौंपी है,
  • गणराज्य की स्थापना की गई, जिसके अन्तर्गत राज्याध्यक्ष पैतृक न होकर जनता द्वारा एक निश्चित अवधि के लिए निर्वाचित होगा।

प्रश्न 9.
भारतीय संविधान की प्रस्तावना में प्रयुक्त ‘राष्ट्र की एकता’ तथा ‘अखण्डता’ शब्दों का क्या अर्थ है?
उत्तर:
भारतीय संविधान के निर्माता अंग्रेज़ों की ‘फूट डालो’ और शासन करो’ (Divide and Rule) की नीति से भली-भाँति परिचित थे और उसका परिणाम भुगत चुके थे। अतः वे भारत की एकता को बनाए रखने के बहुत इच्छुक थे। अतः संविधान की प्रस्तावना में भारत की एकता की घोषणा की गई। इस लक्ष्य की प्राप्ति के लिए भारत के सभी नागरिकों को भारत की (इकहरी) नागरिकता प्रदान की गई है।

भारत को धर्म-निरपेक्ष राज्य घोषित किया गया है तथा सभी नागरिकों को बिना किसी भेदभाव के समान राजनीतिक, सामाजिक तथा आर्थिक अधिकार प्रदान किए गए हैं। भारतीय संविधान में 22 भाषाओं को मान्यता दी गई है। हिन्दी को देश की राष्ट्रभाषा घोषित किया गया है। संविधान के 42वें संशोधन द्वारा ‘एकता’ के साथ ‘अखण्डता’ शब्द को भी जोड़ दिया गया है।

प्रश्न 10.
भारतीय संविधान की त्रुटियाँ या कमियाँ (Weaknesses) बताएँ।
अथवा
किन बातों के आधार पर भारतीय संविधान की आलोचना की जाती है?
य संविधान निर्माताओं ने देश की वर्तमान एवं भावी परिस्थितियों को ध्यान में रखते हए एक अच्छा और स्थायी संविधान बनाने का प्रयास किया, परन्तु फिर भी कोई संविधान ऐसा नहीं हो सकता जो कमियों या दोषों से पूर्णतः मुक्त हो। भारतीय संविधान भी इसका अपवाद नहीं है। भारतीय संविधान की भी अनेक आधारों पर आलोचना हुई है। ऐसे कुछ प्रमुख आधार इस प्रकार हैं-

(1) भारतीय संविधान अत्यधिक लम्बा एवं विस्तृत संविधान है। इसके परिणामस्वरूप भारतीय संविधान न्यायपालिका के सामने वकीलों की वाक्कुशलता से एक खिलौना बनकर रह गया है। अनेक संवैधानिक विवाद खड़े हुए हैं, जिसमें स्वयं न्यायपालिका ने भी अलग-अलग समय में अलग-अलग निर्णय दिए हैं। जैसे मौलिक अधिकारों के सम्बन्ध में न्यायपालिका ने अपने निर्णय को बार-बार बदला है,

(2) भारतीय संविधान में अनेक प्रावधान ऐसे भी हैं जिनके पीछे कानूनी मान्यता का अभाव । है। ऐसे प्रावधान व्यर्थ में ही संविधान के प्रावधानों को अनावश्यक रूप दे रहे हैं। जैसे राज्य नीति के निदेशक सिद्धान्त एक अलग अध्याय IV के द्वारा संविधान में रखे गए हैं, परन्तु कानूनी शक्ति के अभाव में न्यायपालिका भी इन्हें लागू करवाने में असमर्थ है। अतः संविधान में केवल कानूनी स्वरूप के प्रावधान रखना ही अधिक अच्छा है,

(3) भारतीय संविधान-निर्मात्री सभा भी वास्तव में सच्चे अर्थों में जनता का प्रतिनिधित्व नहीं कर रही थी। आलोचकों का कहना है कि अधिकांश सदस्य विभिन्न सम्पन्न वर्गों से सम्बन्धित थे। अतः वे भारतीय समाज का समुचित प्रतिनिधित्व करने वाली संविधान सभा नहीं कही जा सकती। ऐसे में उनके द्वारा निर्मित संविधान समस्त जनता का संविधान कैसे कहा जा सकता है।

प्रश्न 11.
भारतीय संविधान की ‘प्रस्तावना’ की आलोचना किन बातों के आधार पर की गई है?
उत्तर:
भारतीय संविधान की प्रस्तावना की आलोचना दी गई बातों के आधार पर की गई है-
(1) संविधान की प्रस्तावना के आरम्भ में यह बताया जा चुका है कि संविधान सभा के सदस्य वयस्क मताधिकार के आधार पर सभी नागरिकों द्वारा नहीं चुने गए हैं। उनको चुनने वाला देश की आबादी का एक बहुत छोटा भाग था, क्योंकि सन् 1935 के भारत सरकार के अधिनियम ने सम्पत्ति के आधार पर लोगों को मताधिकार दिया था।

इसलिए आलोचकों का कहना है कि ऐसे सीमित मतदाताओं के प्रतिनिधियों द्वारा बनाया गया संविधान जनता का संविधान कैसे कहा जा सकता है, परन्तु हमें यह नहीं भूलना चाहिए कि संविधान सभा में 82 प्रतिशत सदस्य काँग्रेस दल के थे तथा काँग्रेस उस समय सम्पूर्ण राष्ट्र की प्रतिनिधि संस्था थी,

(2) संविधान की प्रस्तावना को न्यायिक मान्यता प्राप्त न होने से न्यायालय उसमें दिए गए सिद्धान्तों को लागू करने के लिए सरकार को बाध्य नहीं कर सकते। अतः उसमें दिए गए सिद्धान्त पवित्र घोषणा से अधिक कुछ भी नहीं हैं,

(3) संविधान के 42वें संशोधन ने प्रस्तावना में दो शब्द जोड़ तो अवश्य दिए हैं, लेकिन इन शब्दों का कोई सुनिश्चित अर्थ उसमें नहीं दिया है। अतः देश के विभिन्न वर्गों, संगठनों, राजनीतिक दलों तथा उनके नेताओं द्वारा इन शब्दों का मनमाने अर्थ लगाने से उन्हें व्यावहारिक रूप अभी तक नहीं दिया गया है। समाज में धन की प्रधानता तथा धर्म और जातियों के आधार पर प्रतिदिन होने वाले हिंसक दंगे इस विश्लेषण की पुष्टि करते हैं।

निबंधात्मक प्रश्न

प्रश्न 1.
संविधान के राजनीतिक दर्शन के क्या अर्थ हैं? इसमें प्रायः कौन-कौन सी बातें शामिल होती हैं?
उत्तर:
संविधान के राजनीतिक दर्शन का अर्थ (Meaning of Political Philosophy of the Constitution) वास्तव में संविधान का दर्शन संविधान में अंकित विभिन्न शब्दों एवं धाराओं के वास्तविक अर्थ एवं लक्ष्य को प्रकट करने वाला होता है। दूसरे शब्दों में किसी देश के संविधान का मूल्यांकन जिन आधारों पर किया जाता है, उन्हें ही संविधान के दर्शन का नाम दिया जाता है। संविधान का मूल्यांकन वास्तव में संविधान में निहित उद्देश्यों, आदर्शों एवं मूल्यों की सफलता के आधार पर किया जाता है कि हम इन्हें कहाँ तक प्राप्त करने में सफल हुए और कहाँ तक असफल हुए।

इस प्रकार किसी संविधान का वास्तविक ज्ञान केवल उस देश के संविधान की धाराओं या प्रावधानों के अध्ययन से ही नहीं होता, बल्कि उस दर्शन के अध्ययन से होता है जिस पर उस देश के संविधान की धाराएँ आधारित हैं और जिन उद्देश्यों की पूर्ति के लिए शासन या सरकार का स्वरूप निश्चित किया गया है।

जैसे हमारे भारतीय संविधान निर्माताओं ने प्रत्येक भारतीय नागरिक को न्याय, स्वतन्त्रता एवं समानता प्रदान करने के उद्देश्य से संविधान की विभिन्न धाराओं में कानूनी प्रावधान किए हैं। ऐसा इसीलिए क्योंकि हमारा राजनीतिक दर्शन लोकतान्त्रिक व्यवस्था पर आधारित है। एक लोकतान्त्रिक व्यवस्था में न्याय, समानता एवं स्वतन्त्रता के तत्त्व अपरिहार्य होते हैं। इस प्रकार स्पष्ट है कि संविधान के राजनीतिक दर्शन के आधार पर ही संविधान की सफलताओं एवं कमियों को इंगित किया जा सकता है। संक्षेप में, संविधान का दर्शन संविधान का सार होता है जो एक देश के सामाजिक एवं राष्ट्रीय मूल्यों को प्रतिबिम्बित करता है।

किसी भी देश के संविधान के दर्शन में जो बातें निहित होती हैं वे इस प्रकार हैं-

(1) किसी देश के संविधान का निर्माण जिन आदर्शों की बुनियाद पर हुआ है, वे संविधान के दर्शन की ओर संकेत करते हैं, जैसे भारत के संविधान का निर्माण लोकतन्त्र रूपी आदर्श की बुनियाद पर निर्मित हुआ है,

(2) किसी भी देश का संविधान कुछ अवधारणाओं को भी अपने में, अपने देश की परिस्थितियों के अनुरूप निहित रखता है जो उस देश के संविधान के दर्शन की ओर एक संकेत होता है; जैसे भारत में अधिकार, कर्तव्य, न्यायपालिका, नागरिकता आदि का भारतीय लोकतन्त्र के अनुरूप अर्थ एवं उद्देश्य है। जबकि चीन जैसे साम्यवादी देश में वहाँ के साम्यवादी दर्शन के अनुरूप इन्हीं अवधारणाओं के अर्थ बदल जाते हैं,

(3) एक देश के संविधान का दर्शन वास्तव में उस देश के संविधान निर्माण के समय संविधान निर्माताओं के सम्मुख तत्कालिक परिस्थितियों एवं उसके परिणामस्वरूप उनके बीच हुए वाद-विवाद एवं निर्णयों में भी देखा जा सकता है। जैसे द्वितीय विश्वयुद्ध के बाद जापान में निर्मित होने वाला संविधान एक तरह से ‘शान्ति संविधान’ रूपी दर्शन पर क्यों आधारित हुआ?

इसका उत्तर एवं कारण स्पष्ट है कि द्वितीय विश्वयुद्ध में जापान की हार एवं हिरोशिमा और नागासाकी के ऊपर हुई विजित राष्ट्रों की बमबारी से व्यवस्थित तत्कालिक चुनौतियों ने संविधान निर्माताओं को यह निर्णय जापानी जनता के लिए लेना पड़ा कि वे युद्ध एवं अन्तर्राष्ट्रीय विवादों के समाधान हेतु बल प्रयोग एवं धमकी के साधनों से सर्वथा दूर रहेंगे और न ही सेना व युद्ध सामग्री इत्यादि राष्ट्र के लिए रखेंगे।

जबकि दूसरी तरफ चीनी संविधान का निर्माण भी दूसरे विश्वयुद्ध के बाद हुआ, लेकिन उनकी साम्यवादी विचारधारा के प्रसार की चुनौती ने उन्हें युद्ध एवं राष्ट्रीय सीमाओं के विस्तार को अपने संविधान का निर्माण करते समय अपने राजनीतिक दर्शन में प्रमुख स्थान दिया। इसके अतिरिक्त यहाँ हम एक उदाहरण भारतीय संविधान का भी ले सकते हैं जो दूसरे विश्वयुद्ध के बाद निर्मित हुआ।

भारतीय संविधान निर्माताओं ने शोषणयुक्त एवं अन्याय एवं अत्याचार पर आधारित जीवन ब्रिटिश शासकों के अधीन व्यतीत किया था। इसलिए भारतीय संविधान निर्माताओं की चुनौतियाँ विशेषकर भारतीय नागरिक को गौरवमय जीवन के साथ-साथ न्याय, समानता एवं स्वतन्त्रता पर आधारित जीवन प्रदान करना था। अतः भारतीय संविधान एवं उसकी शासन-प्रणाली वास्तव में ऐसे ही राजनीतिक दर्शन पर आधारित है,

(4) किसी देश के संविधान का दर्शन वास्तव में देश की सरकार के विभिन्न अंगों की शक्ति एवं उसके सम्प्रभु सम्पन्न स्वरूप के साथ-साथ उस देश के नागरिकों के साथ उनके सम्बन्धों को भी स्पष्ट करता है। जैसे भारत का संविधान न्यायपालिका की सर्वोच्चता, स्वतन्त्रता एवं निष्पक्षता के साथ व्यक्तियों के अधिकारों के संरक्षण में महत्त्वपूर्ण भूमिका निभाता है।

जबकि चीन में न्यायपालिका एक अधीनस्थ संस्था या अंग के रूप में कार्य करती हुई विधानपालिका एवं कार्यपालिका के कार्यों की ही पुष्टि करने का कार्य करती है। अतः उपर्युक्त विवरण से स्पष्ट होता है कि किसी देश के संविधान के वास्तविक स्वरूप को समझने के लिए उसके वैधानिक स्वरूप के साथ-साथ संविधान में निहित आदर्शों एवं मूल्यों के अनुरूप उसका मूल्यांकन किया जाए। यहाँ हम संविधान के दर्शन की स्पष्टता हेतु विशेषतः भारतीय संविधान में निहित मूल्यों एवं आदर्शों की स्पष्टता हेतु संविधान के मूल प्रावधानों का भी अध्ययन करेंगे।

प्रश्न 2.
भारतीय संविधान के मूल प्रावधानों (Core Provisions) का उल्लेख करें।
उत्तर:
किसी भी देश का संविधान न केवल शासक एवं शासित सम्बन्धों को निश्चित करता है, बल्कि एक वैधानिक एवं राजनीतिक व्यवस्था के अधीन नागरिकों एवं समाज के विकास सम्बन्धी सिद्धान्तों का विवेचन करते हुए शासन शक्ति या सरकार के प्रभुसत्ता सम्पन्न अंगों के कार्यों एवं उनकी शक्तियों की भी स्पष्ट व्याख्या करता है। यही वास्तव में उस देश के संविधान के सारभूत या मूल प्रावधान होते हैं। यहाँ हम भारतीय संविधान के मूल प्रावधानों का विवेचन करेंगे जो निम्नलिखित हैं

1. सम्पूर्ण प्रभुसत्ता सम्पन्न, समाजवादी, धर्म-निरपेक्ष, लोकतन्त्रात्मक, गणराज्य की स्थापना (Establishment of a Sovereign, Socialist, Secular, Democratic, Republic)-भारत में संविधान के द्वारा भारत को एक सम्पूर्ण प्रभुसत्ता सम्पन्न, लोकतान्त्रिक गणराज्य घोषित किया गया है और संविधान के 42वें संशोधन के द्वारा इसमें समाजवादी (Socialist) तथा धर्म-निरपेक्ष (Secular) शब्दों को जोड़कर इसे सम्पूर्ण प्रभुसत्ता सम्पन्न, समाजवादी, धर्म-निरपेक्ष लोकतान्त्रिक गणराज्य बना दिया गया है। इन विभिन्न अवधारणाओं को संक्षेप में निम्नलिखित प्रकार से स्पष्ट किया जा रहा है

(1) सम्पूर्ण प्रभुसत्ता सम्पन्न (Sovereign):
सम्पूर्ण प्रभुसत्ता सम्पन्न का अर्थ यह है कि भारत अपने आन्तरिक तथा बाहरी दोनों ही प्रकार के मामलों में पूर्ण रूप से स्वतन्त्र तथा सर्वोच्च सत्ताधारी है। आन्तरिक प्रभुसत्ता का अर्थ है कि भारत क्षेत्र में रहने वाले लोगों और भारत राज्य में स्थित सभी समुदायों पर भारत राज्य को अधिकार प्राप्त है और बाहरी प्रभुसत्ता का अभिप्राय यह है कि भारत किसी विदेशी राज्य के अधीन नहीं है

तथा दूसरे राज्यों से वह अपनी इच्छानुसार सम्बन्ध स्थापित कर सकता है। 15 अगस्त, 1947 से पूर्व भारत को यह स्थिति प्राप्त नहीं थी। लेकिन 15 अगस्त, 1947 के पश्चात् भारत अन्तर्राष्ट्रीय क्षेत्रों में अपनी इच्छानुसार आचरण कर सकता है और यह किसी भी अन्तर्राष्ट्रीय संधि अथवा समझौते को मानने के लिए बाध्य नहीं है।

(2) समाजवादी (Socialist):
समाजवादी शब्द संविधान की प्रस्तावना में संविधान के 42वें संशोधन द्वारा जोड़ा गया है। इस शब्द का उद्देश्य है कि भारत में इस प्रकार की शासन-व्यवस्था स्थापित की जाए, जिससे समाज के सभी वर्गों को, विशेष रूप से पिछड़े वर्गों को, विकास के लिए उचित तथा समान अवसर प्राप्त हों, मनुष्य का मनुष्य के द्वारा शोषण समाप्त हो और आर्थिक विषमता को कम किया जाए।

कांग्रेस ने बहुत पहले से ही भारत में लोकतान्त्रिक समाजवाद की स्थापना करना अपना उद्देश्य घोषित किया था और उसी दल की सरकार द्वारा यह शब्द संविधान में 1976 (42वें संशोधन द्वारा) में जोड़ दिया गया।

(3) धर्म-निरपेक्ष (Secular):
धर्म-निरपेक्ष शब्द भी संविधान की प्रस्तावना में संविधान के 42वें संशोधन द्वारा जोड़ा गया है। इसका अर्थ यह है कि भारत में रहने वाले सभी लोगों को अपनी इच्छानुसार किसी भी धर्म को मानने तथा उसका प्रचार करने का अधिकार है। राज्य की दृष्टि में सभी धर्म समान हैं और धर्म के आधार पर राज्य किसी प्रकार के भेदभाव का प्रयोग नहीं करेगा।

सरकार किसी भी धर्म को कोई विशेष संरक्षण प्रदान नहीं करेगी। सरकार के द्वारा अपने नागरिकों को कोई ऐसा चंदा या कर देने के लिए मजबूर नहीं किया जा सकता, जिससे प्राप्त आय किसी धर्म विशेष के प्रचार के लिए खर्च की जाने वाली हो। यद्यपि हमारे संविधान में धर्म-निरपेक्षता की ओर पहले से ही संकेत था, परन्तु अब यह शब्द जोड़ने से संविधान का उद्देश्य और भी स्पष्ट हो गया है।

(4) लोकतन्त्रात्मक (Democratic):
लोकतन्त्रात्मक का अर्थ है कि भारत एक लोकतन्त्रात्मक राज्य है, जिसमें अन्तिम शक्ति जनता के हाथों में है। जनता वयस्क मताधिकार के आधार पर प्रत्येक पांच वर्ष के लिए अपने प्रतिनिधियों का चुनाव करती है जो उस काल के लिए देश के शासन को चलाते हैं। अब तक देश में हुए 17 लोकसभा चुनाव इसका प्रत्यक्ष प्रमाण हैं।

इसके अतिरिक्त यह भी स्पष्ट है कि यदि सरकार अपने उत्तरदायित्व के आधार पर कार्य ठीक तरह से नहीं करती है तो उस सरकार या मन्त्रिमण्डल को जनता के चुने हुए प्रतिनिधियों अर्थात् संसद के द्वारा हटाया जा सकता है। देश के सभी नागरिकों को समान अधिकार दिए गए हैं और उनकी रक्षा के लिए स्वतन्त्र न्यायपालिका की स्थापना की गई है।

(5) गणराज्य (Republic):
गणराज्य का अर्थ यह है कि देश का अध्यक्ष प्रत्यक्ष या अप्रत्यक्ष रूप से निर्वाचित पदाधिकारी होगा। हमारे देश के मुखिया अर्थात् राष्ट्रपति का पद पैतृक सिद्धान्त पर आधारित नहीं है, वह एक निश्चित काल (5 वर्ष) के लिए जनता के प्रतिनिधियों द्वारा निर्वाचित किया जाता है। यदि उसकी मृत्यु हो जाए तो 6 मास के अन्दर नए राष्ट्रपति का चुनाव करवाना आवश्यक है।

संघात्मक संविधान परन्तु एकात्मक प्रणाली की ओर झुकाव (Federal Constitution with a Unitary Bias) यद्यपि भारतीय संविधान में ‘संघ’ शब्द का प्रयोग नहीं किया गया है, परन्तु फिर भी यह सत्य है कि भारत का वर्तमान संविधान देश में संघीय शासन-प्रणाली की स्थापना करता है। इसमें वे सभी लक्षण मौजूद हैं जो संघीय सरकार की स्थापना के लिए आवश्यक हैं; जैसे

(1) संविधान के अनुसार शासन की शक्तियों का केन्द्रीय सरकार तथा राज्य सरकारों में संघ सूची, राज्य सूची एवं समवर्ती सूची के द्वारा बँटवारा किया गया है।

(2) भारत का संविधान लिखित है तथा इसका अधिकांश भाग कठोर है। इसका कारण यह है कि संविधान का एक बहुत बड़ा. भाग ऐसा है, जिसमें संशोधन करने के लिए संसद के दोनों सदनों का विशेष बहुमत तथा राज्यों की स्वीकृति लेना भी आवश्यक है।

(3) संविधान देश का सर्वोच्च कानून है और इसकी रक्षा करने के लिए एक सर्वोच्च न्यायालय की स्थापना की गई है। सर्वोच्च न्यायालय को संविधान के उल्लंघन में पास किए गए किसी भी कानून को अवैध घोषित करके रद्द करने का अधिकार प्राप्त है।

(4) इसके अतिरिक्त केन्द्रीय संसद का संगठन द्वि-सदनीय विधानमण्डल प्रणाली के आधार पर किया गया है। लोकसभा, जो कि संसद का निचला सदन है, देश की जनता का प्रतिनिधित्व करता है और राज्यसभा में संघ की इकाइयों के प्रतिनिधि बैठते हैं। इस प्रकार हम देखते हैं कि इस संविधान में संघीय शासन-प्रणाली के सभी तत्त्व मौजूद हैं,

अतः यह संघीय संविधान है। कुछ आलोचकों का विचार है कि भारतीय संविधान एक पूर्ण संघीय संविधान नहीं है, क्योंकि इसका झुकाव एकात्मकता की ओर है। इसका कारण यह है कि केन्द्रीय सरकार को इतना अधिक शक्तिशाली बनाया गया है कि यह संविधान बनावट में संघात्मक है, परन्तु भाव में एकात्मक है। पी०एस० देशमुख का मत था कि, “जो संविधान बना है वह संघात्मक की अपेक्षा एकात्मक अधिक है।” भारतीय संविधान के एकात्मक लक्षण संक्षेप में निम्नलिखित हैं-

(1) भारतीय संविधान ने एक अत्यन्त शक्तिशाली केन्द्र का निर्माण किया है। डॉ० कश्यप के अनुसार, “संघ सूची में 97 विषय हैं और वह तीनों सूचियों में सबसे लम्बी है। समवर्ती सूची में 52 विषयों पर भी केन्द्र सरकार जब चाहे कानून बना सकती है। इसके अतिरिक्त अपशिष्ट शक्तियाँ भी केन्द्रीय सरकार में ही निहित हैं।”
(2) संघ एवं राज्यों के लिए एक ही संविधान का होना भी एकात्मक शासन-प्रणाली का लक्षण है।
(3) राष्ट्रीय आपातकाल के दौरान संविधान के संघात्मक स्वरूप का एकात्मक स्वरूप में परिवर्तित हो जाना एकात्मक शासन का लक्षण है।
(4) देश में एकीकृत न्याय व्यवस्था का होना भी एकात्मक शासन का लक्षण है।
(5) समूचे देश के लिए अखिल भारतीय प्रशासनिक सेवा व पुलिस सेवाएँ एवं एक ही चुनाव आयोग की व्यवस्था भी देश को एकात्मक शासन की ओर ले जाता है।
(6) देश के नागरिकों को केवल इकहरी नागरिकता प्रदान करना भी एकात्मक शासन का ही लक्षण है।
अतः संविधान में एकात्मक लक्षणों को देखने के पश्चात् यह स्वतः ही स्पष्ट हो जाता है कि भारतीय संविधान का स्वरूप संघात्मक होने के पश्चात् भी यह कहा जा सकता है कि यह एकात्मक स्वरूप की ओर झुका हुआ है।

अंशतः लचीला तथा अंशतः कठोर (Partly Flexible and Partly Rigid)-भारत का संविधान इतना कठोर नहीं है जितना कि संयुक्त राज्य अमेरिका का संविधान और न ही इतना लचीला है जितना कि इंग्लैण्ड का संविधान। यह अंशतः लचीला (Flexible) और अंशतः कठोर (Rigid) है। वास्तव में हमारे संविधान-निर्माताओं का उद्देश्य यह था कि संविधान इतना लचीला न हो कि यह बहु-संख्यक दल के हाथों में खिलौना बन जाए। दूसरी ओर यह इतना कठोर भी न हो कि इसे देश की बदलती हुई परिस्थितियों के अनुसार बदला न जा सके, जिससे इसकी प्रगति के मार्ग में बाधा आए।

भारतीय संविधान में संशोधन के लिए तीन विभिन्न प्रणालियों को अपनाया गया है। सर्वप्रथम, संविधान में कुछ विषय ऐसे हैं जिनके सम्बन्ध में संशोधन संसद के दोनों सदनों के साधारण बहुमत से किया जा सकता है। इस श्रेणी में नए राज्यों का निर्माण, राज्यों की सीमाओं में परिवर्तन तथा उनका पुनर्गठन, राज्य विधानमण्डलों का संगठन एक-सदनीय अथवा द्वि-सदनीय आधार पर करना तथा भारतीय नागरिकता से सम्बन्धित विषय शामिल हैं। दूसरे स्थान पर कुछ विषय ऐसे हैं जिनमें संशोधन करने के लिए दोनों सदनों के कुल सदस्य संख्या के बहुमत द्वारा तथा उपस्थित और मत देने वाले सदस्यों के 2/3 बहुमत द्वारा स्वीकार होने के पश्चात् इसका कम-से-कम आधे राज्यों के विधानमण्डलों द्वारा स्वीकृत होना आवश्यक है।

इस श्रेणी में राष्ट्रपति के चुनाव की ‘पद्धति, सर्वोच्च न्यायालय तथा उच्च न्यायालयों से सम्बन्धित विषय तथा राज्यों के संसद में प्रतिनिधित्व आदि विषय आते हैं। संविधान में दिए गए शेष विषयों के सम्बन्ध में संशोधन करने के लिए संसद के प्रत्येक सदन में कुल सदस्य संख्या का बहुमत तथा उपस्थित और मत देने वाले सदस्यों के 2/3 बहुमत द्वारा प्रस्ताव स्वीकृत होना आवश्यक है। इस प्रकार हम देखते हैं कि हमारा संविधान न तो पूर्ण रूप से लचीला है और न ही पूर्ण रूप से कठोर, बल्कि इन दोनों के बीच का मार्ग अपनाता है। अब तक भारतीय संविधान में 104 (दिसम्बर, 2019 तक) संशोधन हो चुके हैं।

4. मौलिक अधिकार (Fundamental Rights)-भारतीय संविधान की एक अन्य विशेषता यह है कि इसके अनुसार, नागरिकों के मौलिक अधिकारों को सुरक्षित किया गया है। ये वे अधिकार हैं जो एक मनुष्य को शारीरिक, मानसिक तथा नैतिक विकास के लिए बहुत आवश्यक समझे जाते हैं। इस सम्बन्ध में एक महत्त्वपूर्ण बात यह है कि इन अधिकारों की न केवल संविधान में घोषणा ही की गई है, वरन उन्हें लागू करने के लिए भी उचित साधन जुटाए गए हैं। ये अधिकार न्याययोग्य (Justiciable) हैं और सर्वोच्च न्यायालय तथा उच्च न्यायालयों को उनकी रक्षा का कार्य सौंपा गया है।

उन्हें विधानमण्डल द्वारा पास किए गए किसी भी कानून को अथवा कार्यपालिका द्वारा जारी किए गए किसी भी ऐसे अध्यादेश को अवैध घोषित करने तथा रद्द करने का अधिकार दिया गया है, जो इन अधिकारों का उल्लंघन करते हों। प्रत्येक नागरिक अपने इन अधिकारों को प्राप्त करने के लिए न्यायालयों की सहायता ले सकता है और उसका यह अधिकार, केवल संकटकालीन स्थिति को छोड़कर, कभी भी स्थगित नहीं किया जा सकता। .. संविधान के भाग तीन, अनुच्छेद 14 से 32 तक नागरिकों के निम्नलिखित छः अधिकारों का वर्णन किया गया है

  • समानता का अधिकार (Right to Equality)
  • स्वतन्त्रता का अधिकार (Right to Freedom)
  • शोषण के विरुद्ध अधिकार (Right against Exploitation)
  • धार्मिक स्वतन्त्रता का अधिकार (Right to Freedom of Religion)
  • सांस्कृतिक तथा शिक्षा सम्बन्धी अधिकार (Cultural and Educational Rights)
  • सवैधानिक उपचारों का अधिकार (Right to Constitutional Remedies)

मौलिक अधिकारों के सम्बन्ध में यह बात ध्यान देने योग्य है कि ये अधिकार असीमित नहीं हैं। इनके प्रयोग पर अनेक प्रकार के प्रतिबन्ध लगे हुए हैं। इसके अतिरिक्त संसद इनमें संशोधन करके इन पर रोक लगा सकती है।

5. मौलिक कर्त्तव्य (Fundamental Duties)-संविधान के 42वें संशोधन द्वारा अनुच्छेद 51-A में नागरिकों के दस मौलिक कर्तव्य निश्चित किए गए हैं। यहाँ यह उल्लेखनीय है कि भारतीय संविधान में मूल कर्त्तव्यों का समावेश पूर्व सोवियत संघ के संविधान से प्रेरित होते हुए किए गए हैं। स्वर्ण सिंह समिति की अनुशंसा के पश्चात् भारतीय संविधान में निम्नलिखित दस मूल कर्तव्यों को समाविष्ट किया गया

  • संविधान का पालन करना तथा उसके आदर्शों, संस्थाओं, राष्ट्रीय ध्वज तथा राष्ट्रीय गान का सम्मान करना।
  • स्वतन्त्रता संग्राम को प्रेरित करने वाले उच्च आदर्शों को बनाए रखना तथा उनका पालन करना।
  • भारत की प्रभुसत्ता, एकता तथा अखण्डता का समर्थन एवं रक्षा करना।
  • देश की रक्षा करना और आवश्यकता पड़ने पर राष्ट्रीय सेवा करना।
  • धार्मिक, भाषायी तथा प्रादेशिक विभिन्नताओं को त्यागकर भारत के सभी लोगों में मेल-मिलाप तथा बन्धुत्व की भावना विकसित करना, स्त्रियों की प्रतिष्ठा के विरुद्ध प्रथाओं का त्याग करना।
  • अपनी मिली-जुली संस्कृति की सम्पन्न परम्परा का सम्मान करना तथा उसे सुरक्षित रखना।
  • वनों, झीलों, नदियों तथा अन्य जीवों सहित प्राकृतिक वातावरण की रक्षा और सुधार करना तथा जीव-जन्तुओं के प्रति दया की भावना रखना।।
  • वैज्ञानिक दृष्टिकोण, मानवता, अन्वेषण और सुधार की भावना विकसित करना।
  • सार्वजनिक सम्पत्ति की रक्षा करना तथा हिंसा का त्याग करना।
  • व्यक्तिगत तथा सामाजिक कार्य-कलापों के क्षेत्र में कुशलता लाने का प्रयत्न करना, ताकि राष्ट्र उपलब्धि के उच्च

शिखरों तक पहुँच सके। यहाँ यह उल्लेखनीय है कि सन् 2002 में 86वें संवैधानिक संशोधन के द्वारा 14 वर्ष तक की आयु के बच्चे के माता-पिता या संरक्षकों को अपने बच्चे को शिक्षा दिलाने के अवसर उपलब्ध कराने संबंधी कर्त्तव्य को निश्चित किया गया है। इस प्रकार अब मूल कर्त्तव्यों की संख्या 11 हो गई है।

6. राज्य-नीति के निदेशक सिद्धान्त (Directive Principles of State Policy):
भारतीय संविधान में नागरिकों के मौलिक अधिकारों के साथ-साथ राज्य-नीति के निदेशक सिद्धान्तों का वर्णन संविधान के भाग IV, अनुच्छेद 36 से 51 तक किया गया है। इन सिद्धान्तों को अपनाने में हमारे संविधान निर्माताओं ने आयरलैण्ड के संविधान की नकल की है।

ये सिद्धान्त केन्द्रीय सरकार तथा राज्य सरकारों के लिए पथ-प्रदर्शक के रूप में हैं और इनके द्वारा उन्हें यह आदेश दिया गया है कि वे अपनी नीति का निर्माण करते समय इनको ध्यान में रखें, क्योंकि इन सिद्धान्तों के पूर्ण रूप से अपनाए जाने पर ही भारत में एक कल्याणकारी राज्य की स्थापना हो सकती है।

इन सिद्धान्तों का उद्देश्य भारत में सामाजिक, आर्थिक तथा राजनीतिक न्याय की स्थापना करना है। यह बात ध्यान देने योग्य है कि राज्य-नीति के निदेशक सिद्धान्त न्यायसंगत (Justiciable) नहीं हैं। इसका अर्थ यह है कि ये सिद्धान्त न्यायालयों द्वारा कानूनी तौर पर लागू नहीं किए जा सकते और यदि केन्द्र सरकार अथवा राज्य सरकारें उन्हें लागू नहीं करती अथवा उन्हें ध्यान में रखकर कार्य नहीं करतीं, तो नागरिकों को न्यायालय में जाकर उनके विरुद्ध न्याय माँगने का अधिकार नहीं है।

7. संयुक्त चुनाव-प्रणाली तथा वयस्क मताधिकार (Joint Electorate and Universal Adult Franchise):
भारत के नए संविधान के अनुसार पृथक् तथा साम्प्रदायिक प्रणाली (Separate and Communal Electorate), जिसका आरम्भ भारत में सन् 1909 के अधिनियम के द्वारा किया गया था और जिसका विस्तार सन् 1919 तथा 1935 के अधिनियमों के द्वारा किया गया था, को समाप्त कर दिया गया है और उसके स्थान पर संयुक्त चुनाव-प्रणाली की व्यवस्था की गई है। इसके द्वारा देश में साम्प्रदायिकता को कम करने के लिए एक बहुत ही महत्त्वपूर्ण कदम उठाया गया है।

इसके अतिरिक्त संविधान द्वारा वयस्क मताधिकार की व्यवस्था की गई है। स्वतन्त्रता से पूर्व भारत में मताधिकार बहुत ही सीमित था, जो सम्पत्ति, शिक्षा तथा कर देने आदि की योग्यताओं पर आधारित था, परन्तु नए संविधान के अनुसार, इस प्रकार के सभी भेदभावों को समाप्त कर दिया गया है।

अब मताधिकार के लिए व्यक्ति की आयु ही एकमात्र योग्यता है कि भारत के प्रत्येक नागरिक को, जिसकी आयु 18 वर्ष अथवा उससे अधिक है, बिना किसी प्रकार के भेदभाव के यह अधिकार दिया गया है। यहाँ यह उल्लेखनीय है कि मूल संविधान में मताधिकार की आयु 21 वर्ष रखी गई थी जोकि 61वें संवैधानिक संशोधन (1989) के द्वारा घटाकर 18 वर्ष कर दी गई है।

8. इकहरी नागरिकता (Single Citizenship):
भारत में संघीय शासन-व्यवस्था की स्थापना के बावजूद प्रत्येक नागरिक को इकहरी नागरिकता प्रदान की गई है। भारत का प्रत्येक नागरिक चाहे वह देश के किसी भी भाग में (किसी भी राज्य में) रहता हो, भारत का ही नागरिक है। संयुक्त राज्य अमेरिका तथा कुछ अन्य संघ-राज्यों में दोहरी नागरिकता के सिद्धान्त को अपनाया गया है।

संयुक्त राज्य अमेरिका में रहने वाला प्रत्येक नागरिक संयुक्त राज्य अमेरिका का नागरिक होने के साथ-साथ उस राज्य-विशेष का भी नागरिक है, जिसमें वह निवास करता है, परन्तु भारतीय संविधान के निर्माताओं का यह भी विचार था कि दोहरी नागरिकता देश की एकता को बनाए रखने में बाधक सिद्ध हो सकती है। अतः उन्होंने संघीय व्यवस्था की स्थापना करते हुए भी इकहरी नागरिकता के सिद्धान्त को अपनाया है।

9. स्वतन्त्र न्यायपालिका (Independent Judiciary):
भारतीय संविधान की एक विशेषता यह है कि इसके द्वारा न्यायपालिका की स्वतन्त्रता को सुरक्षित किया गया है। संघात्मक शासन-व्यवस्था में संविधान की रक्षा के लिए न्यायपालिका का स्वतन्त्र होना आवश्यक है। इसके अतिरिक्त संविधान द्वारा नागरिकों को प्रदान किए गए मौलिक अधिकारों की रक्षा हेतु भी । न्यायपालिका की स्वतन्त्रता आवश्यक हो जाती है। इस कारण से हमारे संविधान में न्यायपालिका की स्वतन्त्रता के लिए अनेक व्यवस्थाएँ की गई हैं।

10. द्वि-सदनीय विधानमण्डल (Bicameral Legislature):
संसार के अन्य संघीय संविधानों की भान्ति भारतीय संविधान के द्वारा भी संघीय स्तर पर द्वि-सदनीय विधानमण्डल की स्थापना की गई है। संसद के दो सदन हैं लोकसभा (House of the People) तथा राज्यसभा (Council of States)। लोकसभा संसद का निचला सदन है, जिसके सदस्यों का चुनाव लोगों द्वारा प्रत्यक्ष चुनाव-प्रणाली के अनुसार किया जाता है।

इसके निर्वाचित सदस्यों की अधिकतम संख्या 550 तथा कार्यकाल 5 वर्ष निश्चित किया गया है। राज्यसभा में अधिक-से-अधिक 250 सदस्य हो सकते हैं। इनमें से.12 सदस्य राष्ट्रपति के द्वारा मनोनीत (Nominate) किए जाते हैं और शेष सदस्य राज्यों की विधानसभाओं के सदस्यों द्वारा निर्वाचित किए जाते हैं। यह एक स्थायी सदन है। इसका प्रत्येक सदस्य 6 वर्ष के लिए चुना जाता है और प्रत्येक दो वर्ष के पश्चात् इसके 1/3 सदस्य सेवा-निवृत्त हो जाते हैं। इस प्रकार लोकसभा में देश की जनता तथा राज्यसभा में भारतीय संघ की इकाइयों (राज्यों) को प्रतिनिधित्व दिया गया है।

11. अल्पसंख्यकों तथा पिछड़े वर्गों के कल्याण के लिए विशेष व्यवस्था (Special Provisions for the Welfare of Minorities and Backward Classes):
संविधान के द्वारा अनुसूचित जातियों तथा अन्य पिछड़े वर्गों के कल्याण के लिए विशेष व्यवस्था की गई है। उनके लिए सरकारी नौकरियों, विधानसभाओं तथा स्थानीय संस्थाओं में स्थान आरक्षित रखे गए हैं।

79वें सवैधानिक संशोधन द्वारा यह अवधि 2010 तक तथा 95वें संशोधन द्वारा यह प्रावधान सन् 2020 तक बढ़ा दिया गया था जो 104वें संवैधानिक द्वारा आरक्षण सम्बन्धी प्रावधान जनवरी, 2030 तक बढ़ा दिया गया। पिछड़े वर्गों को यह सुविधा इसलिए दी गई है कि इनकी बहुत पिछड़ी स्थिति के कारण इन्हें विकास के लिए विशेष सुविधाओं की आवश्यकता है।

12. एक राष्ट्र भाषा (One National Language):
भारत एक विशाल देश है, जिसमें अनेक भाषाएँ बोली जाती हैं। इसलिए संविधान के द्वारा 22 भाषाओं (92वें संशोधन के पश्चात) को मान्यता दी गई है, परन्तु देश की एकता को बनाए रखने के लिए तथा विभिन्न भाषायी प्रान्तों में राष्ट्रीय भावनाओं का विकास करने के लिए एक राष्ट्रीय भाषा के महत्त्व को आवश्यक समझा गया, इसलिए हिन्दी को देवनागरी लिपि में राष्ट्र भाषा घोषित किया गया है। हिन्दी के विकास के लिए केन्द्र तथा राज्यों को विशेष उत्तरदायित्व सौंपे गए हैं।

13. कानून का शासन (Rule of Law):
भारत के संविधान की यह भी विशेषता है कि इसके द्वारा कानून के शासन की व्यवस्था की गई है। कानून के समक्ष सभी नागरिक बराबर हैं, कोई भी व्यक्ति कानून से ऊपर नहीं है। अमीर-गरीब, पढ़े-लिखे तथा अनपढ़, कमजोर और शक्तिशाली सभी देश के कानून के समक्ष बराबर हैं। कानून से ऊँचा कोई नहीं है। जो व्यक्ति कानून का उल्लंघन करता है तो उसे कानून के द्वारा ही सजा दी जाती है। कानूनी कार्रवाई किए बिना किसी व्यक्ति को बन्दी नहीं बनाया जा सकता।

14. पंचायतों तथा नगरपालिकाओं की संवैधानिक व्यवस्था (Constitutional Provision of Panchayats and Municipalities):
संविधान के 73वें संशोधन द्वारा पंचायतों की तथा 74वें संशोधन द्वारा नगरपालिकाओं की संवैधानिक व्यवस्था की गई है। ये दोनों संशोधन सन 1992 में पारित किए गए। इन संशोधनों द्वारा पहली बार ग्रामीण तथा शहरी क्षेत्रों की स्थानीय संस्थाओं को संविधान द्वारा मान्यता दी गई है।

15. विश्व-शान्ति का समर्थक (It Supports World Peace):
भारतीय संविधान विश्व-शान्ति तथा अन्तर्राष्ट्रीयता का प्रबल समर्थक है। राज्य-नीति के निदेशक सिद्धान्तों में यह कहा गया है कि राज्य अन्तर्राष्ट्रीयता का प्रबल समर्थक है तथा राज्य अन्तर्राष्ट्रीय शान्ति तथा सुरक्षा की उन्नति और राष्ट्रों के बीच न्याय एवं सम्मानपूर्ण सम्बन्धों को बनाए रखने का प्रयत्न करेगा।

व्यवहार में भी भारत द्वारा विश्व-शान्ति बनाए रखने का हर सम्भव प्रयत्न किया गया है और युद्ध केवल उसी समय किए गए, जब आत्मरक्षा हेतु ऐसा करना आवश्यक हो गया था।

16. संकटकालीन शक्तियाँ (Emergency Powers) भारतीय संविधान में हमारे संवैधानिक अध्यक्ष राष्ट्रपति को विशेष रूप से आपात्कालीन शक्तियाँ प्रदान की गई हैं, जो निम्नलिखित हैं

(1) संविधान के अनुच्छेद 352 के अनुसार यदि युद्ध, बाहरी आक्रमण अथवा सशस्त्र विद्रोह के कारण या इनमें से किसी भी एक के होने की सम्भावना हो तो राष्ट्रपति पूरे देश में या देश के किसी भाग विशेष में संकटकालीन घोषणा कर सकता है। ऐसी घोषणा होने पर उसका नाममात्र का स्वरूप वास्तविक शासक के स्वरूप में बदल जाता है।

(2) किसी राज्य की संवैधानिक मशीनरी असफल होने या ऐसी सम्भावना होने पर भी उस राज्य में संविधान के अनुच्छेद 356 का प्रयोग करते हुए राज्य की शासन-व्यवस्था को राज्यपाल के माध्यम से राष्ट्रपति सीधे अपने हाथों में ले लेता है।

(3) संविधान के अनुच्छेद 360 के अन्तर्गत यदि देश में वित्तीय संकट का भय हो तो राष्ट्रपति आर्थिक संकट की घोषणा कर सकता है। अतः दिए गए सारभूत प्रावधान भारतीय संविधान के वैधानिक स्वरूप को स्पष्ट करने के साथ भारतीय समाज में नागरिकों हेतु शासन-व्यवस्था के स्वरूप एवं उद्देश्यों को भी स्पष्ट करते हैं। परन्तु यहाँ हम संविधान के मूल प्रावधानों के अतिरिक्त संविधान में की गई उन परिकल्पनाओं को जो संविधान-निर्माता भावी भारत के निर्माण के सन्दर्भ में रखते थे, का विवेचन करना भी बहुत उपयुक्त होगा। यहाँ हम उन्हीं परिकल्पनाओं का विवेचन कर रहे हैं।

HBSE 11th Class Political Science Important Questions Chapter 10 संविधान का राजनीतिक दर्शन

प्रश्न 3.
प्रस्तावना (Preamble) से आप क्या समझते हैं? भारतीय संविधान की प्रस्तावना की व्याख्या करें।
उत्तर:
प्रस्तावना का अर्थ (Meaning of Preamble)-किसी भाषण अथवा लेख में प्रारम्भिक अथवा परिचयात्मक कथन को प्रस्तावना कहते हैं। प्रस्तावना किसी संविधान अथवा अधिनियम का वह प्रारम्भिक कथन है जिसमें उसके निर्माण के कारणों का उल्लेख किया जाता है तथा उन उद्देश्यों एवं आकांक्षाओं का संक्षिप्त वर्णन किया जाता है जिन्हें वह प्राप्त करना चाहता है।

उसमें संविधान के स्रोत को इंगित किया जाता है तथा उस तिथि का उल्लेख किया जाता है जब संविधान को अंगीकृत, अधिनियमित एवं आत्मार्पित किया जाता है। प्रस्तावना का स्थान प्रस्तावना का स्थान किसी संविधान अथवा अधिनियम के शीर्षक के बाद और उसके मुख्य भाग से पहले होता है। अतः संविधान की प्रस्तावना एक ऐसा झरोखा होती है, जिसमें संविधान के निर्माताओं की भावनाओं तथा उनकी आशाओं का दृश्य देखा जा सकता है। इसी कारण से भारतीय संविधान की प्रास्तावना को संविधान-निर्माताओं के दिलों की कुंजी’ कहा जाता है।

प्रस्तावना में संशोधन-प्रायः यह विचार प्रकट किया जाता था कि प्रस्तावना संविधान का भाग नहीं है और संसद को इसमें संशोधन करने का अधिकार नहीं था, परन्तु सन् 1973 में सर्वोच्च न्यायालय के केशवानन्द भारती बनाम भारत सरकार, नामक मुकद्दमे का फैसला देते हुए इसका स्पष्टीकरण कर दिया। जिसके अनुसार, प्रस्तावना संविधान का भाग है और संसद को, इसके मूल ढाँचे (Basic Structure) को छोड़कर अन्य भागों में संशोधन करने का अधिकार है।

इसके परिणामस्वरूप सन् 1976 में भारतीय संसद ने संविधान की प्रस्तावना में संशोधन करके इसमें समाजवादी (Socialist) तथा धर्म-निरपेक्ष (Secular) शब्द जोड़ दिए थे। इसके अतिरिक्त जहाँ ‘राष्ट्र की एकता’ के शब्द का प्रयोग किया गया, वहाँ ‘एकता तथा अखण्डता’ (Unity and Integrity) का भी प्रयोग किया गया है। इस प्रकार प्रस्तावना में यह शब्द जोड़कर यह सिद्ध किया गया है कि हम देश में समाजवाद लाने और राज्य को धर्म-निरपेक्ष बनाए रखने के लिए पूरी तरह उत्सुक हैं। भारतीय संविधान की प्रस्तावना भारतीय संविधान की प्रस्तावना निम्नलिखित है

“हम, भारत के लोग, भारत को एक सम्पूर्ण प्रभुत्व सम्पन्न, समाजवादी, धर्म-निरपेक्ष, लोकतन्त्रात्मक गणराज्य बनाने के लिए तथा उसके समस्त नागरिकों को सामाजिक, आर्थिक और राजनीतिक न्याय विचार, अभिव्यक्ति, विश्वास, धर्म और उपासना की स्वतन्त्रता प्रतिष्ठा और अवसर की समता प्राप्त कराने के लिए,

तथा उन सभी में व्यक्ति की गरिमा और राष्ट्र की एकता और अखण्डता सुनिश्चित करने वाली बन्धुता बढ़ाने के लिए दृढ़ संकल्प होकर अपनी इस संविधान सभा में आज तारीख 26 नवम्बर, 1949 (मिति मार्गशीर्ष शुक्ल पक्ष सप्तमी, सम्वत दो हजार छह विक्रमी) को एतद् द्वारा इस संविधान को अंगीकृत, अधिनियमित और आत्म-समर्पित करते हैं।” प्रस्तावना की विशेषताएँ या प्रस्तावना की व्याख्या-भारतीय संविधान की प्रस्तावना की विशेषताओं या इसकी व्याख्या करने के लिए हम उपर्युक्त प्रस्तावना को निम्नलिखित तीन शीर्षकों के अन्तर्गत विभाजित कर सकते हैं

  • संवैधानिक शक्ति के स्त्रोत (Source of Constitutional Authority),
  • भारतीय शासन का स्वरूप (Nature of Indian Government)
  • संविधान के उद्देश्य (Aims of the Constitution)। उपर्युक्त तीनों शीर्षकों का वर्णन भारतीय संविधान की प्रस्तावना के अनुसार निम्नलिखित है

1. संवैधानिक शक्ति के स्रोत भारतीय संविधान की प्रस्तावना में प्रयुक्त प्रारम्भिक एवं अन्तिम शब्द, “हम भारत के लोग ……… संविधान को अंगीकृत, अधिनियमित एवं आत्मार्पित करते हैं”, शब्दों से तीन बातें स्पष्ट होती हैं। प्रथम, संविधान के द्वारा अन्तिम प्रभुसत्ता भारत की जनता में निहित की गई है; द्वितीय, संविधान निर्माता भारतीय जनता के प्रतिनिधि हैं तथा तृतीय, संविधान निर्माता भारतीय जनता की इच्छा का परिणाम है और जनता द्वारा ही

इसे राष्ट्र को समर्पित किया गया है। इस सम्बन्ध में डॉ०बी०आर० अम्बेडकर (Dr. B.R. Ambedkar) ने संविधान सभा की बहस में ठीक ही कहा था, “मैं समझता हूँ कि प्रस्तावना इस सदन के प्रत्येक सदस्य की इच्छानुसार यह प्रकट करती है कि इस संविधान का आधार, इसकी शक्ति तथा प्रभुसत्ता इसे लोगों से ही प्राप्त हुई है।”

2. भारतीय शासन का स्वरूप संविधान की प्रस्तावना में भारत को सम्पूर्ण प्रभुसत्ता सम्पन्न, समाजवादी, धर्म-निरपेक्ष, लोकतन्त्रीय गणराज्य घोषित किया गया है, जो भारतीय शासन-व्यवस्था के स्वरूप को स्पष्ट करते हैं। प्रस्तावना में प्रयुक्त भारतीय शासन-व्यवस्था के स्वरूप को स्पष्ट करने वाले शब्दों का वर्णन निम्नलिखित प्रकार से है

(1) सम्पूर्ण प्रभुसत्ता सम्पन्न का अर्थ है कि भारत पर आन्तरिक अथवा बाहरी दृष्टि से किसी विदेशी सत्ता का अधिकार नहीं है। इंग्लैण्ड, अमेरिका तथा रूस आदि देशों की भाँति भारत भी अन्तर्राष्ट्रीय सन्धि अथवा समझौते को मानने के लिए बाध्य नहीं है। भारत का संयुक्त राष्ट्र संघ तथा राष्ट्रमण्डल का सदस्य होना इसकी प्रभुसत्ता पर कोई प्रभाव नहीं डालता।

(2) समाजवादी शब्द प्रस्तावना में 42वें संशोधन के द्वारा जोड़ा गया है। इसका अर्थ है कि भारत में शासन-व्यवस्था इस प्रकार चलाई जाए कि सभी वर्गों को विशेष रूप से पिछड़े हुए वर्गों को अपने विकास के लिए उचित वातावरण तथा परिस्थितियाँ मिलें, आर्थिक असमानता कम और देश के विकास का फल थोड़े से लोगों के हाथों में न होकर समाज के सभी लोगों को मिले।

(3) धर्म-निरपेक्ष शब्द भी प्रस्तावना में सन् 1976 में 42वें संशोधन द्वारा जोड़ा गया है। इसका अर्थ यह है कि देश के सभी नागरिकों को अपनी इच्छानुसार, किसी भी धर्म को अपनाने तथा उसका प्रचार करने की स्वतन्त्रता है। राज्य धर्म के आधार पर नागरिकों में किसी प्रकार का भेदभाव नहीं कर सकता और राज्य किसी विशेष धर्म की किसी विशेष रूप से सहायता नहीं कर सकता। धर्म के आधार पर किसी सरकारी शिक्षा-संस्था में किसी को दाखिला देने से इन्कार नहीं किया जा सकता।

(4) लोकतन्त्रीय शब्द का अर्थ है कि शासन शक्ति किसी एक व्यक्ति या वर्ग के हाथों में न होकर समस्त जनता के हाथों में है। शासन चलाने के लिए जनता अपने प्रतिनिधियों का चुनाव करती है जो अपने कार्यों के लिए जनता के प्रति उत्तरदायी हैं। देश के प्रत्येक नागरिक को, चाहे वह किसी धर्म, जाति अथवा स्थान से सम्बन्ध रखता हो, राजनीतिक अधिकार समान रूप से प्रदान किए गए हैं। साम्प्रदायिक चुनाव-प्रणाली को समाप्त कर दिया गया है।

(5) गणराज्य का अर्थ यह है कि भारत में राज्य के अध्यक्ष का पद वंश-क्रमानुगत (Hereditary) नहीं है, बल्कि राज्य का अध्यक्ष, राष्ट्रपति एक निश्चित काल के लिए जनता के प्रतिनिधियों द्वारा निर्वाचित किया जाता है। इंग्लैण्ड तथा जापान लोकतन्त्रीय राज्य होते हुए भी, गणराज्य नहीं हैं क्योंकि इन देशों में राजा का पद पैतृक आधार पर चलता है और वह जनता अथवा उनके प्रतिनिधियों द्वारा निश्चित काल के लिए निर्वाचित नहीं किया जाता।

3. संविधान के उद्देश्य भारतीय संविधान की प्रस्तावना से यह बात भी स्पष्ट होती है कि संविधान द्वारा कुछ विशिष्ट उद्देश्यों की पूर्ति की आशा की गई है। ये उद्देश्य निम्नलिखित हैं

(1) न्याय-संविधान का उद्देश्य यह है कि देश के सभी नागरिकों को जीवन के प्रत्येक क्षेत्र- सामाजिक, आर्थिक व राजनीतिक क्षेत्रों में न्याय मिले। इस बहुमुखी न्याय से ही नागरिक अपने जीवन का पूर्ण विकास कर सकता है। .

(क) सामाजिक न्याय का अर्थ है किसी व्यक्ति के साथ धर्म, जाति, जन्म-स्थान, रंग तथा लिंग आदि के आधार पर कोई भेदभाव नहीं किया जाएगा। इसी उद्देश्य की प्राप्ति के लिए देश के सभी नागरिकों के लिए समानता का अधिकार संविधान के द्वारा सुरक्षित किया गया है। देश के सभी नागरिक कानून के सामने समान हैं। सार्वजनिक स्थानों पर जाने के लिए नागरिकों में रंग, जाति व धर्म आदि के आधार पर भेदभाव नहीं किया जा सकता। सरकारी नौकरी पाने के क्षेत्र में सभी के लिए समानता स्थापित करने का प्रयत्न किया गया है।

(ख) आर्थिक न्याय का यह अर्थ है कि सभी व्यक्तियों को अपनी आजीविका कमाने के समान तथा उचित अवसर प्राप्त हों तथा उन्हें अपने कार्य के लिए उचित वेतन मिले। आर्थिक न्याय के लिए यह आवश्यक है कि उत्पादन तथा वितरण के साधन थोड़े-से व्यक्तियों के हाथों में न होकर समाज के हाथों में हों और उनका प्रयोग समस्त समाज के हितों को ध्यान में रखकर किया जाए। आर्थिक न्याय के इस लक्ष्य की प्राप्ति समाजवादी ढाँचे के समाज की स्थापना के आधार पर ही की जा सकती है।

(ग) राजनीतिक न्याय का अर्थ है कि राज्य के सभी नागरिकों को बिना किसी भेदभाव के सभी राजनीतिक अधिकार प्राप्त हों। भारत में वयस्क मताधिकार प्रणाली को अपनाकर इस व्यवस्था को लागू किया गया है। धर्म, जाति, रंग तथा लिंग आदि के आधार पर नागरिकों को राजनीतिक अधिकार देने में कोई भेदभाव नहीं किया गया है और साम्प्रदायिक चुनाव-प्रणाली का अन्त कर दिया गया है।

(2) स्वतन्त्रता भारतीय संविधान का उद्देश्य नागरिकों को केवल न्याय दिलाना ही नहीं बल्कि स्वतन्त्रता को भी निश्चित करना है, जो व्यक्ति के जीवन के विकास के लिए आवश्यक मानी जाती है। संविधान की प्रस्तावना में नागरिकों को विचार, अभिव्यक्ति, विश्वास तथा उपासना (Thought, Expression, Faith and Worship) की स्वतन्त्रता का आश्वासन दिया गया है। इसकी पूर्ति संविधान में दिए गए मौलिक अधिकारों के द्वारा की गई है।

(3) समानता प्रस्तावना में सामाजिक स्तर तथा अवसर (Social Status and Opportunity) का उल्लेख किया गया है। स्तर . की समानता का अर्थ यह है कि कानून की दृष्टि में देश के सभी नागरिक समान हैं तथा किसी को कोई विशेषाधिकार प्राप्त नही हैं

संविधान का राजनीतिक दर्शन और धर्म, रंग, लिंग आदि के आधार पर नागरिकों में कोई भेदभाव नहीं किया जा सकता। इन बातों का स्पष्टीकरण संविधान की धाराओं – 14-18 के द्वारा किया गया है। धारा 14 के अन्तर्गत सभी नागरिकों को कानून के सामने समानता तथा सुरक्षा प्रदान की गई है। धारा 15 में यह कहा गया है कि राज्य किसी नागरिक के साथ धर्म, रंग, जाति तथा लिंग के आधार पर कोई भेदभाव नहीं करेगा।

धारा 16 के द्वारा सभी नागरिकों को अवसर की समानता प्रदान की गई है। धारा 17 के द्वारा छुआछूत को समाप्त कर दिया गया है तथा धारा 18 के द्वारा शिक्षा तथा सेना की उपाधियों को छोड़ अन्य सभी प्रकार की उपाधियों का अन्त कर दिया गया है।

(4) बन्धुता-संविधान की प्रस्तावना में बन्धुता की भावना को विकसित करने पर भी बल दिया गया है। भारत जैसे देश के लिए, जिसमें गुलामी के लम्बे काल के कारण धर्म, जाति व भाषा आदि के आधार पर भेदभाव उत्पन्न हो गए थे, बन्धुता की भावना के विकास का विशेष महत्त्व है। इसी उद्देश्य से ही साम्प्रदायिक चुनाव-प्रणाली तथा छुआछूत को समाप्त कर दिया गया है।

इसके साथ ही प्रस्तावना में ‘व्यक्ति की गरिमा’ (Dignity of Individual) शब्दों का रखा जाना इस बात का प्रतीक है कि व्यक्ति का व्यक्तित्व बहुत पवित्र है। इसी धारणा से देश के सभी नागरिकों को समान मौलिक अधिकार दिए गए हैं।

(5) राष्ट्र की एकता व अखण्डता भारतीय संविधान के निर्माता अंग्रेज़ों की ‘फूट डालो व शासन करो’ (Divide and Rule) की नीति से अच्छी प्रकार परिचित थे, इसलिए उनके मन और मस्तिष्क में राष्ट्र की एकता का विचार बहुत प्रबल था। इसी कारण से संविधान की प्रस्तावना में राष्ट्र की एकता को बनाए रखने की घोषणा की गई।

इस उद्देश्य की प्राप्ति के लिए ही भारत को एक धर्म-निरपेक्ष राज्य घोषित किया गया है तथा इकहरी नागरिकता (Single Citizenship) के सिद्धान्त को अपनाया गया है। समस्त देश का एक ही संविधान है तथा संविधान में अनेक (22) भाषाओं को मान्यता दी गई है। 42वें संशोधन के द्वारा राष्ट्र की एकता के साथ अखण्डता शब्द जोड़ दिया गया है।

प्रश्न 4.
भारत के भावी संविधान (शासक-व्यवस्था) के बारे में संविधान सभा की परिकल्पनाएँ क्या थीं?
उत्तर:
भारत के संविधान का निर्माण करने वाली संविधान सभा के सदस्यों की स्वतन्त्र भारत के भावी राज्य-प्रबन्ध के बारे में क्या परिकल्पनाएँ थीं। इनकी पृष्ठभूमि को सन् 1928 में प्रकाशित हुई नेहरू रिपोर्ट (Nehru Report, 1928) में देखा जा सकता है। डॉ० सरकारिया ने नेहरू रिपोर्ट को ‘वर्तमान संविधान की रूप-रेखा’ (Blue print of the Present (Indian) Constitution) कहा है।

यह रिपोर्ट भारतीयों द्वारा भारत के संविधान के निर्माण के लिए किया गया पहला प्रयत्न था। उसके पश्चात् संविधान सभा में पं० जवाहरलाल नेहरू थे जिन्होंने ‘उद्देश्य प्रस्ताव’ (Objective Resolution) रखा। वह भी संविधान सभा के सदस्यों के विचारों का दर्पण था।

इसी तरह संविधान की प्रस्तावना में भी संविधान सभा के भावी भारत के राज्य-प्रबन्ध से सम्बन्धित धारणाओं के दर्शन किए जा सकते हैं। अतः उद्देश्य, प्रस्ताव और संविधान की प्रस्तावना उन उद्देश्यों और लक्ष्यों की ओर इंगित करते हैं जिनके आधार पर संविधान सभा के सदस्य भावी भारत की शासन-व्यवस्था का निर्माण करना चाहते थे। संविधान सभा के सदस्यों की भारत की भावी शासन-व्यवस्था से सम्बन्धित परिकल्पनाओं का वर्णन निम्नलिखित है-

1. सम्पूर्ण प्रभुत्व सम्पन्न राज्य की परिकल्पना-भारतीय नेता मुख्य रूप से उस वर्ग से सम्बन्ध रखते थे जिन्होंने अंग्रेज़ों की गुलामी देखी थी, इसलिए उनकी इच्छा भारत को एक सम्पूर्ण प्रभुत्व सम्पन्न राज्य देखने की थी। इस उद्देश्य की प्राप्ति हेतु उन्होंने प्रस्तावना में भारत को प्रभुत्व सम्पन्न राज्य घोषित किया। इसका अभिप्राय यह है कि भारत पूर्ण रूप से स्वतन्त्र है। वह किसी भी बाहरी शक्ति के अधीन नहीं है। प्रभुसत्ता जनता के पास है। जनता अपने प्रतिनिधियों का चुनाव स्वयं करे।

2. प्रजातन्त्र की परिकल्पना भारतीय संविधान सभा की दूसरी परिकल्पना यह थी कि भारत में प्रजातन्त्रात्मक शासन-व्यवस्था की स्थापना की जाए। संविधान सभा के अधिकांश सदस्य प्रजातन्त्र के समर्थक थे। अतः उन्होंने भारत के लिए प्रजातन्त्रात्मक व्यवस्था को अपनाया। इसका अर्थ यह है कि शासन की अन्तिम शक्ति जनता के हाथ में ही रहेगी।

लोग शासन चलाने के लिए अपने प्रतिनिधियों को चुनते हैं तथा निर्वाचित प्रतिनिधि जनता के प्रति उत्तरदायी हैं। भारत के प्रत्येक नागरिक को बिना किसी भेदभाव के समान अधिकार प्रदान किए गए हैं। प्रत्येक 18 वर्ष के नागरिक को बिना किसी भेदभाव के मताधिकार दिया गया है और निर्वाचित होने का अधिकार भी दिया गया है। कानून के शासन की व्यवस्था तथा स्वतन्त्र न्यायपालिका की व्यवस्था की गई है।

3. समाजवाद की परिकल्पना-भारतीय संविधान सभा के सदस्यों पर समाजवाद की अवधारणा का प्रभाव था, इसलिए वे भारत में समाजवाद की स्थापना करना चाहते थे। नेहरू जी भारत में समाजवाद स्थापित करने के पक्ष में थे। उनका कहना था कि भारत के विकास का मार्ग केवल समाजवाद में ही निहित है। यद्यपि संविधान की मूल प्रस्तावना में समाजवाद शब्द को नहीं रखा गया था, लेकिन सन् 1976 में संविधान की प्रस्तावना में यह शब्द जोड़ दिया गया है। वास्तविकता में कांग्रेस पहले से ही भारत में प्रजातन्त्रीय समाजवाद की स्थापना का लक्ष्य घोषित कर चुकी थी। समाजवाद का अर्थ है कि भारत में इस प्रकार से

सन-व्यवस्था चलाई जाए जिसमें आर्थिक असमानता न हो तथा इस प्रकार का वातावरण उत्पन्न किया जाए जिसके द्वारा समाज के सभी वर्गों को अपना विकास करने के अवसर प्राप्त हों। देश के विकास का फल कुछ थोड़े-से लोगों के हाथ में न रहकर समाज के सभी वर्गों को बिना भेदभाव के प्राप्त हो। इसी उद्देश्य की प्राप्ति हेतु, संविधान में राज्य-नीति के निदेशक सिद्धान्तों को अंकित किया गया है। यही नहीं समाजवाद के लक्ष्य को प्राप्त करने के लिए 42वां संशोधन पास किया गया जिसके अन्तर्गत निदेशक सिद्धान्तों को मौलिक अधिकारों पर श्रेष्ठता प्रदान की गई है।

4. गणराज्य बनाने की परिकल्पना भारतीय संविधान निर्माताओं के समक्ष, किस प्रकार की शासन-व्यवस्था अपनाई जाए, इस सम्बन्ध में कई विकल्प थे। कुछ सदस्य अमेरिका की भाँति अध्यक्षात्मक शासन-व्यवस्था अपनाने के पक्ष में थे। लगभग 8 सदस्यों ने शक्ति पृथक्करण के आधार पर अध्यक्षात्मक शासन-प्रणाली अपनाए जाने का समर्थन किया।

इसके विपरीत कुछ सदस्य संसदीय शासन-व्यवस्था को अंगीकार करने के पक्ष में थे। अन्त में संसदीय सरकार अपनाए जाने का निर्णय किया गया, परन्तु संसदीय सरकार जरूरी नहीं है कि वह गणराज्य भी हो; जैसे इंग्लैण्ड और जापान में संसदीय सरकार तो है, परन्तु गणराज्य नहीं है। इसलिए संविधान सभा के सदस्यों ने भारत में गणराज्य स्थापित करने का निर्णय लिया।

गणराज्य की प्रमुख विशेषता यह होती है कि उसमें राज्य का मुखिया नाममात्र का अधिकारी होता है, जिसे जनता द्वारा प्रत्यक्ष या अप्रत्यक्ष ढंग से निर्वाचित किया जाता है। भारत के राष्ट्रपति का निर्वाचन एक निर्वाचन-मण्डल द्वारा अप्रत्यक्ष विधि द्वारा किया जाता है।

5. धर्म-निरपेक्ष राज्य व्यवस्था की परिकल्पना-संविधान सभा के सदस्य इस बात से भली-भाँति परिचित थे कि भारत में अनेक प्रकार के धर्म, वर्ग व जाति के लोग रहते हैं। इन सभी लोगों में मधुर सम्बन्ध स्थापित करने का एकमात्र साधन धर्म-निरपेक्षता है। इसलिए उनकी यह हार्दिक इच्छा थी कि भारत की भावी शासन-व्यवस्था धर्म-निरपेक्षता की नींव पर आधारित हो।

अतः भारतीय संविधान के अनुच्छेद 25 के अन्तर्गत धार्मिक स्वतन्त्रता का अधिकार प्रदान किया गया है। यही नहीं धर्म-निरपेक्ष शब्द को 42वें संशोधन द्वारा प्रस्तावना में भी जोड़ा गया है। इससे यह स्पष्ट हो जाता है कि भारत एक धर्म-निरपेक्ष राज्य है। भारत का अपना कोई धर्म नहीं है। भारत में सभी लोगों को बिना किसी भेदभाव के अपनी इच्छानुसार किसी भी धर्म को मानने की स्वतन्त्रता है। राज्य के लिए सभी धर्म समान हैं। राज्य किसी विशेष धर्म को संरक्षण प्रदान नहीं करेगा। सरकारी शिक्षा संस्थाओं में धार्मिक शिक्षा और धार्मिक प्रचार पर रोक लगाई गई है।

6. कल्याणकारी राज्य की परिकल्पना-राष्ट्रीय आन्दोलन राजनीति आन्दोलन होने के साथ-साथ सामाजिक व आर्थिक आन्दोलन भी था। राष्ट्रीय आन्दोलन के अन्तर्गत नेताओं द्वारा भारतीयों के सामाजिक व आर्थिक स्तर को ऊँचा उठाने के लिए अनेक प्रयत्न किए गए। इन सभी प्रयत्नों का संविधान सभा के सदस्यों पर गहरा प्रभाव पड़ा और उन्होंने कामना की कि भारत एक ऐसा कल्याणकारी राज्य होना चाहिए जिसमें सभी को सामाजिक, आर्थिक व राजनीतिक न्याय मिले।

कल्याणकारी राज्य की धारणा का उल्लेख संविधान के अनुच्छेद 38 में किया गया है। सामाजिक, आर्थिक व राजनीतिक न्याय को उद्देश्य, प्रस्ताव और संविधान की प्रस्तावना में भी देखा जा सकता है। सामाजिक न्याय का अर्थ है किसी भी व्यक्ति के साथ जाति, धर्म, लिंग, रंग आदि पर किसी प्रकार का भेदभाव नहीं किया जाएगा।

इस लक्ष्य की प्राप्ति हेतु अनुच्छेद 14 के अनुसार कानून के समक्ष समानता का अधिकार प्राप्त है। संविधान के अनुच्छेद 15 के अनुसार किसी भी नागरिक के विरुद्ध जाति, धर्म, वंश, लिंग व भाषा के आधार पर भेदभाव नहीं किया जाएगा। अनुच्छेद 17 के अनुसार छुआछूत के कलंक को समाप्त किया गया है। आर्थिक न्याय का अर्थ है कि प्रत्येक व्यक्ति को अपनी आजीविका कमाने के लिए समान अवसर प्राप्त हों एवं उसके कार्य के लिए उसे उचित वेतन मिले तथा उसका आर्थिक शोषण न हो।

इन्हीं उद्देश्यों की प्राप्ति हेतु संविधान के अनुच्छेद 38 एवं 39 में राज्य-न की व्यवस्था की गई है। अनुच्छेद 41 के अनुसार बेकारी, बीमारी, बुढ़ापा तथा अंगहीन होने की अवस्था में राज्य की ओर से सहायता की व्यवस्था की गई है। राजनीतिक न्याय का तात्पर्य है कि सभी व्यक्तियों को जाति, धर्म, रंग, लिंग आदि के भेदभाव के बिना समान राजनीतिक अधिकार प्रदान किए जाएंगे।

7. शक्तिशाली केन्द्र की परिकल्पना-राष्ट्रीय आन्दोलन के नेतागण तत्कालीन साम्प्रदायिक तत्त्वों तथा उनकी गतिविधियों से भली-भाँति अवगत थे और इसी साम्प्रदायिकता के कारण भारत का दो भागों में विभाजन हुआ। संविधान निर्माता ने ऐसा सोचा था कि विभाजन के बाद साम्प्रदायिकता का दानव समाप्त हो जाएगा, परन्तु ऐसा नहीं हुआ। विभाजन शक्तियों ने अपना सिर फिर से उठाना आरम्भ कर दिया। उन्होंने अनुभव किया कि इसका एकमात्र उपचार शक्तिशाली केन्द्रीय सरकार की व्यवस्था है।

इस प्रकार के वाद-विवाद के उपरान्त भारत में शक्तिशाली केन्द्र के अधीन संघात्मक शासन-व्यवस्था की स्थापना की गई। भारतीय संविधान में कुछ ऐसे तत्त्व पाए जाते हैं जिनके आधार पर भारतीय शासन-व्यवस्था संघात्मक है। इसके साथ-साथ कुछ ऐसे तत्त्व भी पाए जाते हैं जिनके आधार पर भारतीय शासन-व्यवस्था एकात्मक है। इसी सन्दर्भ में के०सी० वीयर (K.C. Weare) ने कहा है, “भारत एक एकात्मक राज्य है, जिसमें संघात्मक लक्षण हैं।” इसी प्रकार जैनिंग्स (Jennings) ने विचार व्यक्त किए हैं, “भारत शक्तिशाली केन्द्रीयकरण की प्रवृत्तियों वाला संघात्मक राज्य है।” पण्डित नेहरू ने भी संविधान सभा में बोलते हुए शक्तिशाली केन्द्रीय सरकार की स्थापना का समर्थन किया था।

8. स्वतन्त्रता, समानता व बन्धुत्व की परिकल्पना मनुष्य की उन्नति व विकास के लिए केवल राजनीतिक स्वतन्त्रता ही काफी नहीं है, बल्कि अन्य स्वतन्त्रताएँ, समानता व बन्धुत्व भी आवश्यक हैं। संविधान सभा के सदस्य स्वतन्त्रता, समानता व बन्धुत्व की भावना के महत्त्व से भली-भाँति अवगत. थे। अतः वे इनको भारत के संविधान में समावेश करने के पक्ष में थे। उद्देश्य, प्रस्ताव व संविधान की प्रस्तावना में स्वतन्त्रता, समानता व बन्धुत्व के महत्त्व पर बल दिया गया है। संविधान में नागरिकों को कई प्रकार की स्वतन्त्रताएँ प्रदान की गई हैं; जैसे विचार व्यक्त करने की स्वतन्त्रता, किसी भी धर्म को मानने की स्वतन्त्रता आदि। संविधान के अनुच्छेद 14 से 18 तक में विभिन्न समानताओं का समावेश किया गया है;

जैसे कानून के समक्ष समानता, जाति, रंग, लिंग व भाषा के आधार पर कोई भेदभाव न करना, सरकारी नौकरी प्राप्त करने में समानता, शिक्षा व सैनिक उपाधियों को छोड़कर शेष उपाधियों की समाप्ति आदि । संविधान के निर्माता फ्रांस की क्रान्ति के नारे, स्वतन्त्रता, समानता के साथ-साथ बन्धुत्व के नारे से भी प्रभावित थे और उन्होंने भारतीय संविधान की प्रस्तावना में बन्धुत्व की भावना को विकसित करने पर बल दिया।

डॉ०.अम्बेडकर (Dr. Ambedkar) के अनुसार, “बन्धुता एक ऐसा सिद्धान्त है जो सामाजिक जीवन को एकता और मजबूती प्रदान करता है।” इसी सन्दर्भ में एम०वी० पायली (M.V. Pylee) का कथन है, “न्याय, स्वतन्त्रता और समानता के आधार पर नए राष्ट्र का निर्माण करने का उद्देश्य था कि सब यह समझें कि वे एक ही धरती तथा एक ही जन्मभूमि की सन्तान हैं और आपस में बन्धु हैं।”

9. राष्ट्र की एकता तथा अखण्डता की परिकल्पना संविधान को बनाने वाले अंग्रेज़ों की इस ‘फूट डालो और शासन करो’ (Divide and Rule) की नीति से भली-भाँति परिचित थे। अंग्रेजों की इसी नीति के आधार पर भारत का विभाजन हुआ था। इसलिए संविधान निर्माता भारत की एकता को बनाए रखने के पक्ष में थे। वस्तुतः संविधान की प्रस्तावना में राष्ट्र की एकता को बनाए रखने की घोषणा की गई तथा इस उद्देश्य की प्राप्ति के लिए भारत को धर्म-निरपेक्ष राज्य बनाया गया। सभी नागरिकों को भारत की नागरिकता प्रदान की गई।

समस्त भारत के लिए एक ही संविधान को अपनाया गया तथा 18 भारतीय भाषाओं को मान्यता दी गई, किन्तु अब भारतीय में संविधान में चार भाषाओं (मैथिली, डोगरी, बोडो और सन्थाली) को और जोड़ दिया गया है। अब भारतीय संविधान में 22 भाषाओं को मान्यता मिल गई है। नवम्बर, 1976 में हुए 42वें संशोधन के द्वारा प्रस्तावना में राष्ट्र की एकता के साथ अखण्डता (Integrity) शब्द जोड़ा गया है। यहाँ यह उल्लेखनीय है कि यद्यपि संविधान में नागरिकों के लिए मौलिक अधिकारों की व्यवस्था की गई है तथापि इन्हें राष्ट्र की एकता और अखण्डता को बनाए रखने के लिए निलम्बित भी किया जा सकता है।

10. व्यक्तिगत गौरव विषयक परिकल्पना संविधान सभा के सदस्य अंग्रेजों द्वारा भारतीयों के प्रति किए जाने वाले व्यवहार से भली-भाँति परिचित थे। स्वाधीनता से पहले अंग्रेजों ने भारतीयों के गौरव को मान्यता प्रदान नहीं की थी। विदेशों में भारतीयों के साथ अच्छा व्यवहार नहीं किया जाता था। स्वाधीनता के बाद भारतीयों में गौरव को बनाए रखने के लिए प्रस्तावना में इस बात को अंकित किया गया है कि बिना गौरव अनुभव किए कोई भी राष्ट्र उन्नति नहीं कर सकता।

इस तरह से संविधान की प्रस्तावना के द्वारा भारत में व्यक्तिगत गौरव को स्थापित किया जाएगा। भारतीय संविधान द्वारा सब व्यक्तियों को मौलिक अधिकार समान रूप से प्रदान करना, भारतीयों के व्यक्तिगत गौरव को ऊँचा उठाने के लिए एक महत्त्वपूर्ण कदम है। मौलिक अधिकारों के अलावा मताधिकार तथा चुनाव लड़ने का अधिकार भारतीयों में व्यक्तिगत गौरव एवं स्वाभिमान को प्रोत्साहित करने का प्रयत्न करता है।

11. अनुसूचित तथा पिछड़ी जातियों के हितों की रक्षा सम्बन्धी परिकल्पना स्वतन्त्रता से पूर्व भारत में पिछड़ी जातियों व वर्गों .. की संख्या भारत की कुल जनसंख्या का लगभग 1/5 भाग थी जिन्हें अपमान व तिरस्कार की नज़रों से देखा जाता था। श्री अम्बेडकर स्वयं इसी वर्ग से सम्बन्धित थे और उन्हें अपमान व तिरस्कार का कटु अनुभव था।

इसलिए वे पिछड़े वर्गों व जातियों के लिए कुछ करने के लिए कटिबद्ध थे। यही नहीं संविधान सभा के अन्य सदस्य भी पिछड़ी जातियों व वर्गों के उत्थान के लिए किए जाने वाली व्यवस्था के पक्ष में थे। भारतीय संविधान में अनुसूचित तथा पिछड़ी जातियों के हितों की सुरक्षा के विशेष अनुच्छेद लिखे गए हैं। इन श्रेणियों के लोगों के लिए संविधान में यह लिखा गया है कि केन्द्रीय सरकार तथा प्रान्तीय सरकारें उनके लिए कुछ सरकारी नौकरियाँ आरक्षित रखेंगी।

इसी तरह उनके लिए संसद तथा प्रान्तीय विधानपालिकाओं की कुछ सीटें भी आरक्षित की गई हैं। राष्ट्रपति को लोकसभा में दो ऐंग्लो-इण्डियन सदस्य मनोनीत करने का अधिकार प्राप्त है। शुरू में अल्पसंख्यकों, अनुसूचित व पिछड़ी जातियों तथा कबीलों के लिए स्थान सुरक्षित रखने की व्यवस्था सन् 1960 तक की गई थी। बाद में दस-दस वर्ष करके इस अवधि को कई बार बढ़ा दिया गया।

अब 104वें संशोधन द्वारा इस अवधि को वर्ष 2030 तक बढ़ा दिया गया है। कुछ लोगों का मत है कि इनके लिए स्थान आरक्षित रखना समानता के विपरीत है। मण्डल आयोग की रिपोर्ट पर वी०पी० सिंह की सरकार सामाजिक एवं शिक्षा के आधार पर पिछड़े वर्ग के लोगों के लिए केन्द्रीय सेवाओं के लिए नौकरियों में 28% स्थान सुरक्षित रखना चाहती थी, जिसके विरोध में भारत के अनेक युवकों को आत्मदाह करना पड़ा।

यह सरकार के लिए बहुत शर्म का विषय था। पी०वी० नरसिम्हा राव की सरकार ने सामाजिक एवं शिक्षा के आधार पर पिछड़े वर्ग के लोगों के लिए केन्द्रीय सेवाओं के लिए आर्थिक स्थिति और जोड़ने की घोषणा की। इसके साथ ही 10 प्रतिशत स्थान उनके लिए भी आरक्षित किए जाएँगे जो आर्थिक कारण से तो कमज़ोर हैं, लेकिन उच्च जातीय वर्ग से सम्बन्ध रखते हैं। अभी तक इस जटिल समस्या का समाधान नहीं हो पाया है।

12. अल्पसंख्यकों के हितों की रक्षा विषयक परिकल्पना स्वतन्त्रता से पूर्व भारत में अनेक अल्पसंख्यक वर्ग थे और आज भी पाए जाते हैं। अल्पसंख्यकों को धर्म, जाति, भाषा, संस्कृति के आधार पर वर्गीकृत किया जा सकता है। मुस्लिम अपने-आपको अल्पसंख्यक मानते थे और उन्होंने ब्रिटिश सरकार से मुस्लिम संरक्षण की माँग की।

सरकार ने उनकी माँग को मान लिया और कानून द्वारा उनके हितों की रक्षा का प्रावधान किया। विभाजन के समय अल्पसंख्यकों पर हुए अत्याचारों के कारण संविधान सभा के सदस्य भयभीत थे और वे स्वतन्त्र भारत में इसकी पुनरावृत्ति नहीं चाहते थे। इसलिए संवैधानिक सभा की एक समिति का गठन किया गया . जो भारतीय अल्पसंख्यकों के हितों की रक्षा सम्बन्धी समस्या पर विचार करेगी।

भारतीय संविधान में अल्पसंख्यकों के हितों की रक्षा हेतु विभिन्न प्रावधान दिए गए हैं। ऐंग्लो इण्डियन के लिए विधानपालिका में स्थान सुरक्षित किए गए हैं। भाषा व संस्कृति के आधार पर अल्पसंख्यकों को अपनी शिक्षण संस्थाएँ खोलने का अधिकार दिया गया है।

13. संयुक्त चुनाव-प्रणाली विषयक परिकल्पना-संविधान सभा के सदस्य स्वतन्त्रता से पूर्व प्रचलित साम्प्रदायिक चुनाव-प्रणाली के कुप्रभावों से अवगत थे। मुस्लिम मतदाता मुस्लिम उम्मीदवार को तथा हिन्दू मतदाता हिन्दू उम्मीदवार को वोट डालते थे। इस चुनाव-प्रणाली के आधार पर हिन्दुओं और मुसलमानों में घृणा की भावना पैदा हुई, जिसके फलस्वरूप भारत का विभाजन हआ तथा पाकिस्तान राज्य की स्थापना हुई।

संविधान निर्माता इस प्रणाली को समाप्त करने के पक्ष में थे। भारतीय नवीन संविधान के द्वारा इस प्रणाली को समाप्त करके संयुक्त चुनाव-प्रणाली का प्रावधान किया गया है। अब भारत में सभी सम्प्रदाय मिल-जुलकर अपने प्रतिनिधियों को निर्वाचित करते हैं।

14. विश्व शान्ति तथा अन्तर्राष्ट्रीय सुरक्षा की परिकल्पना संविधान सभा के सदस्य दो विश्वयुद्धों का कटु अनुभव अपने साथ लिए हुए थे जिसमें लाखों लोगों की जानें गई थीं। उनकी इच्छा थी कि फिर से धरती पर ऐसा युद्ध न हो। उधर वे जापान की युद्ध-त्याग की नीति से भी प्रभावित थे। इसलिए उन्होंने विश्व-शान्ति व अन्तर्राष्ट्रीय सुरक्षा स्थापित करने का संकल्प लिया।

नवीन भारतीय संविधान विश्व-शान्ति का प्रतीक है। संविधान के अनुच्छेद 51 के अनुसार भारत राष्ट्रों के बीच न्याय तथा समानतापूर्ण सम्बन्ध स्थापित करने की चेष्टा करेगा, अन्तर्राष्ट्रीय शान्ति एवं सुरक्षा बनाए रखने के लिए विधि एवं सन्धि बन्धनों के प्रति आदर का भाव रखेगा और अन्तर्राष्ट्रीय विवादों को मध्यस्थता के द्वारा सुलझाने का प्रयास करेगा। इस सन्दर्भ में पं० जवाहरलाल नेहरू जी की पंचशील नीति की सराहनीय भूमिका रही है। वास्तव में भारत ने अन्तर्राष्ट्रीय शान्ति एवं सुरक्षा को बनाए रखने के लिए विशेष भूमिका निभाई है। आज विश्व की राजनीति में भारत का प्रमुख स्थान है।

निष्कर्ष-उपरोक्त परिकल्पनाओं व दृष्टिकोणों का विस्तृत अध्ययन करने से स्पष्ट हो जाता है कि संविधान निर्माताओं के मस्तिष्क में नवीन भारत की एक कल्पना थी जिसे वे कार्य रूप देना चाहते थे तथा उसी कल्पना के आधार पर उन्होंने एक संविधान का निर्माण किया। संविधान का निर्माण करते समय निर्माताओं ने इस तथ्य को भी ध्यान में रखा कि

ऐसा संविधान बने जिसे कार्यान्वित भी किया जा सके, क्योंकि कल्पनाओं व सपनों में बहकर कहीं ऐसा संविधान न बन जाए जो मात्र आदर्श बनकर रह जाए। अतः संविधान निर्माता वर्तमान संविधान में अपना उद्देश्य (जनता को सामाजिक, आर्थिक व राजनीतिक न्याय) दिलवाने में सफल रहे।

वस्तुनिष्ठ प्रश्न

निम्नलिखित प्रश्नों का उत्तर दिए गए विकल्पों में से उचित विकल्प छाँटकर लिखें

1. भारतीय संविधान की प्रस्तावना का आरंभ निम्नलिखित शब्दों से होता है
(A) भारत एक समाजवाद……
(B) भारत के लोग……
(C) भारत को प्रभुसत्ता संपन्न
(D) हम भारत के लोग
उत्तर:
(D) हम भारत के लोग

2. भारतीय संविधान की प्रस्तावना में ‘समाजवादी’ तथा ‘धर्म-निरपेक्ष’ शब्द जोड़े गए
(A) 42वें संशोधन द्वारा
(B) 44वें संशोधन द्वारा
(C) 46वें संशोधन द्वारा
(D) 52वें संशोधन द्वारा
उत्तर:
(A) 42वें संशोधन द्वारा

3. भारतीय संविधान लागू हुआ
(A) 15 अगस्त, 1947 को
(B) 26 जनवरी, 1946 को
(C) 26 जनवरी, 1950 को
(D) 26 जनवरी, 1952 को
उत्तर:
(C) 26 जनवरी, 1950 को

4. भारतीय संविधान में कुल अनुच्छेद हैं
(A) 395
(B) 238
(C) 138
(D) 295
उत्तर:
(A) 395

HBSE 11th Class Political Science Important Questions Chapter 10 संविधान का राजनीतिक दर्शन

5. प्रस्तावना में वर्णन नहीं किया गया है
(A) प्रेस की स्वतंत्रता का
(B) पद और अवसर की समानता का
(C) विचारों की स्वतंत्रता का
(D) विश्वास और पूजा की स्वतंत्रता का
उत्तर:
(A) प्रेस की स्वतंत्रता का

6. प्रस्तावना में किस न्याय का वर्णन नहीं है?
(A) सामाजिक न्याय
(B) आर्थिक न्याय
(C) राजनीतिक न्याय
(D) धार्मिक न्याय
उत्तर:
(D) धार्मिक न्याय

7. प्रस्तावना में संशोधन किया जा सकता है
(A) राष्ट्रपति द्वारा
(B) प्रधानमंत्री द्वारा
(C) संसद द्वारा
(D) राज्यपालों द्वारा
उत्तर:
(C) संसद द्वारा

8. प्रस्तावना में अब तक संवैधानिक विधि द्वारा संशोधन किया गया है
(A) दो बार
(B) चार बार
(C) एक बार
(D) तीन बार
उत्तर:
(C) एक बार

9. निम्नलिखित में कौन प्र में कौन प्रभुसत्ता संपन्न है?
(A) लोकसभा
(B) राष्ट्रपति
(C) जनता
(D) संविधान
उत्तर:
(C) जनता

10. संविधान में उद्देश्य का वर्णन करते हैं
(A) प्रस्तावना में
(B) मौलिक अधिकारों में
(C) संकटकालीन अनुच्छेदों में
(D) राज्य-नीति के निदेशक सिद्धांतों में
उत्तर:
(A) प्रस्तावना में

11. प्रस्तावना में किस प्रकार के न्याय का वर्णन किया गया है?
(A) सामाजिक न्याय का
(B) सामाजिक, आर्थिक, राजनीतिक न्याय का
(C) आर्थिक न्याय का
(D) धार्मिक न्याय का
उत्तर:
(B) सामाजिक, आर्थिक, राजनीतिक न्याय का

12. संविधान की प्रस्तावना में निम्नलिखित में से कौन-से उद्देश्य घोषित नहीं हैं?
(A) सामाजिक, आर्थिक एवं राजनीतिक न्याय
(B) विचार अभिव्यक्ति, विश्वास, धर्म और उपासना की स्वतंत्रता
(C) व्यक्ति की गरिमा व अवसर की समानता
(D) इसमें तीन सूचियाँ संघ सूची, राज्य सूची और समवर्ती सूची शामिल हैं
उत्तर:
(D) इसमें तीन सूचियाँ-संघ सूची, राज्य सूची और समवर्ती सूची शामिल हैं

13. भारतीय संविधान में धर्म-निरपेक्षता का अर्थ है
(A) राज्य धर्म विरुद्ध है
(B) राज्य धार्मिक है
(C) राज्य अधार्मिक है
(D) राज्य का कोई राजधर्म नहीं है
उत्तर:
(D) राज्य का कोई राजधर्म नहीं है

14. संविधान की प्रस्तावना में प्रयुक्त शब्द ‘सेक्यूलर’ का अर्थ है
(A) सभी नागरिकों को धर्म एवं उपासना की स्वतंत्रता
(B) एकेश्वरवाद
(C) बहुदेववाद
(D) सभी धर्मों को अस्वीकृत
उत्तर:
(A) सभी नागरिकों को धर्म एवं उपासना की स्वतंत्रता

15. भारतीय संविधान के किस काम को उसकी ‘आत्मा’ की संज्ञा दी जाती है?
(A) मौलिक अधिकारों को
(B) राज्य के नीति-निदेशक सिद्धांतों को
(C) संविधान की प्रस्तावना को
(D) अनुसूचियों को
उत्तर:
(C) संविधान की प्रस्तावना को

16. भारतीय संविधान की प्रस्तावना में लिखा है ‘We, the people of India’ इससे निम्नलिखित में से क्या अर्थ निकलता है?
(A) भारतीय गणराज्य की प्रभुसत्ता जनता में निहित है
(B) संविधान निर्माता भारत की जनता का प्रतिनिधित्व करते हैं।
(C) भारत का संविधान भारत की जनता की सहमति पर आधारित है
(D) उपर्युक्त सभी
उत्तर:
(D) उपर्युक्त सभी

17. संविधान की प्रस्तावना में वर्णन मिलता है
(A) संविधान के मूल्यों का
(B) राष्ट्रपति की शक्तियों का
(C) नागरिकों के अधिकारों का
(D) नागरिकों के कर्त्तव्यों का
उत्तर:
(A) संविधान के मूल्यों का

18. भारत गणराज्य है
(A) लोकतंत्रीय व्यवस्था के कारण
(B) संघात्मक प्रणाली के कारण
(C) राज्याध्यक्ष निर्वाचित होने के कारण
(D) इनमें से कोई नहीं
उत्तर:
(A) लोकतंत्रीय व्यवस्था के कारण

निम्नलिखित प्रश्नों का उत्तर एक शब्द में दें

1. किसी देश में संविधान के मूल्यांकन के क्या आधार होते हैं?
उत्तर:
संविधान में निहित उद्देश्य, आदर्श एवं मूल्य संविधान के मूल्यांकन का आधार होते हैं।

2. क्या प्रस्तावना संविधान का कानूनी भाग होती है?
उत्तर:
संविधान में निहित उद्देश्य, आदर्श एवं मूल्य संविधान के मूल्यांकन का आधार होते हैं।

3. क्या भारतीय संविधान की प्रस्तावना में संसद संशोधन कर सकती है?
उत्तर:
हाँ।

4. भारतीय संविधान में मान्य सवैधानिक भाषाएँ कितनी हैं?
उत्तर:
22

5. भारतीय संविधान की प्रस्तावना का प्रारम्भ किन शब्दों से होता है?
उत्तर:
हम भारत के लोग ……….. शब्दों से होता है।

6. 42वें संवैधानिक संशोधन द्वारा राष्ट्र की एकता के साथ कौन-सा शब्द और जोड़ा गया?
उत्तर:
अखण्डता।

7. भारतीय संविधान में संशोधन प्रक्रिया का उल्लेख संविधान की किस धारा में किया गया है?
उत्तर:
368 में।

रिक्त स्थान भरें

1. भारतीय संविधान में राष्ट्रपति ………….. का प्रतीक है।
उत्तर:
गणराज्य

2. ……………. संशोधन द्वारा प्रस्तावना में धर्मनिरपेक्ष एवं समाजवादी शब्द जोड़े गए।
उत्तर:
42वें

HBSE 11th Class Political Science Important Questions Chapter 10 संविधान का राजनीतिक दर्शन

3. भारतीय संविधान में ……………. मान्य सवैधानिक भाषाएँ हैं।
उत्तर:
22

4. भारत में …………… नागरिकता दी गई है।
उत्तर:
इकहरी

5. ……………. सवैधानिक संशोधन द्वारा संविधान में आरक्षण की अवधि को सन 2030 तक बढ़ा दिया गया।
उत्तर:
104वें

6. भारत में ……………. निर्वाचन प्रणाली अपनाई गई है।
उत्तर:
संयुक्त

7. प्रस्तावना को संविधान की ……………. कहा जाता है।
उत्तर:
आत्मा

HBSE 11th Class Political Science Important Questions Chapter 9 संविधान : एक जीवंत दस्तावेज़

Haryana State Board HBSE 11th Class Political Science Important Questions Chapter 9 संविधान : एक जीवंत दस्तावेज़ Important Questions and Answers.

Haryana Board 11th Class Political Science Important Questions Chapter 9 संविधान : एक जीवंत दस्तावेज़

अति लघूत्तंरात्मक प्रश्न

प्रश्न 1.
भारतीय संविधान एक जीवंत प्रलेख के रूप में कैसे कार्य कर रहा है? संक्षेप में समझाइए।
उत्तर:
भारतीय संविधान की जीवंत निरन्तरता के लिए संविधान निर्माताओं ने भविष्य की आवश्यकता के अनुरूप संशोधन प्रक्रिया का उल्लेख अनुच्छेद 368 में किया जिसके अनुसार संविधान में समाज की आवश्यकता के अनुरूप संशोधन सम्भव हो सके। इसके साथ-साथ संविधान की व्याख्या में बहुत अधिक लचीलेपन ने भी संविधान के वास्तविक कार्यकरण को निरन्तर बनाए रखा। यही कारण है कि भारतीय संविधान एक कठोर, स्थिर संविधान न होकर एक जीवंत प्रलेख के रूप में कार्य कर रहा है।

प्रश्न 2.
विगत लगभग आठ दशकों से हमारा संविधान उसी मूल रूप में कार्य कर रहा है, कोई दो कारण लिखिए।
उत्तर:

  • हमें विरासत के रूप में सशक्त संविधान मिला है और संविधान की मूल संरचना हमारे देश के बहुत अनुकूल है,
  • हमारे संविधान निर्माता बहुत दूरदर्शी थे जिन्होंने भविष्य की आने वाली परिस्थितियों के अनुरूप संविधान में अनेक समाधान किए।

प्रश्न 3.
क्या भारतीय संविधान एक ऐसा पवित्र दस्तावेज है कि कोई इसे परिवर्तित नहीं कर सकता? संक्षेप में समझाइए।
उत्तर:
भारतीय संविधान कोई स्थायी एवं कठोर प्रलेख नहीं है। यह प्रत्येक व्यवस्था के सम्बन्ध में अन्तिम शब्द नहीं है। यह अपरिवर्तनीय नहीं है। वास्तव में भारतीय संविधान एक ऐसा पवित्र दस्तावेज है जो समय एवं समाज की आवश्यकता के अनुरूप निश्चित संवैधानिक प्रक्रिया के अनुसार परिवर्तनीय है।

HBSE 11th Class Political Science Important Questions Chapter 9 संविधान : एक जीवंत दस्तावेज़

प्रश्न 4.
भारतीय संविधान में संशोधन की प्रक्रिया का वर्णन संविधान के किस अनुच्छेद में किया जाता है? भारतीय संविधान लचीला है अथवा कठोर?
उत्तर:
भारतीय संविधान में संशोधन करने की प्रक्रिया का वर्णन संविधान के अनुच्छेद 368 में किया गया है। भारतीय संविधान न तो पूर्ण रूप से लचीला है और न ही पूर्ण रूप से कठोर है। यह अंशतः लचीला व अंशतः कठोर है।

प्रश्न 5.
ऐसे दो संवैधानिक संशोधन बताओ जिन पर विचार-विमर्श चल रहा है?
उत्तर:

  • संसद तथा राज्य विधानमंडलों में एक-तिहाई स्थान महिलाओं के लिए आरक्षित करना,
  • अन्य पिछड़े वर्गों के लिए सरकारी नौकरियों तथा शिक्षा संस्थाओं में स्थान आरक्षित करना। .

प्रश्न 6.
ऐसे चार विषयों के नाम बताओ जिनमें संशोधन करने के लिए संसद की मन्जूरी के साथ-साथ आधे राज्यों की मन्जूरी लेना आवश्यक होता है।
उत्तर:
ऐसे चार विषय इस प्रकार हैं-

  • राष्ट्रपति के चुनाव का ढंग,
  • संघ की कार्यपालिका शक्ति का विस्तार,
  • राज्यों की कार्यपालिका शक्ति की सीमा,
  • केंद्र द्वारा प्रशासित क्षेत्रों (Union Territories) के लिए उच्च न्यायालयों की व्यवस्था।

प्रश्न 7.
संसद द्वारा साधारण बहुमत से संविधान में संशोधन किए जाने वाले कोई दो विषय लिखिए।
उत्तर:

  • नए राज्यों को भारतीय संघ में शामिल करना, नए राज्यों की स्थापना करना, उनके क्षेत्र, सीमाओं तथा नामों में परिवर्तन करना (अनुच्छेद 2, 3 एवं 4),
  • किसी राज्य की विधानसभा की सिफारिश पर उस राज्य में विधान परिषद् स्थापित करना अथवा उसे समाप्त करना (अनुच्छेद 169)।

प्रश्न 8.
भारतीय संविधान में दो-तिहाई बहुमत द्वारा संशोधन किस भाग में किया जा सकता है?
उत्तर:
भारतीय संविधान के भाग तीन (मौलिक अधिकार) एवं भाग चार (राज्य-नीति के निदेशक सिद्धांत) में संशोधन दो-तिहाई बहुमत वाली विधि से किया जा सकता है।

प्रश्न 9.
भारतीय संविधान में किए गए संशोधनों को किन श्रेणियों में विभाजित किया जाता है?
उत्तर:
संशोधनों को दी गई तीन श्रेणियों में विभाजित किया जाता है

  • तकनीकी एवं प्रशासनिक समस्याओं से संबंधित संशोधन,
  • न्यायपालिका एवं विधायिका (संसद) के बीच उत्पन्न मतभेद सम्बन्धी संशोधन,
  • राजनीतिक सर्वसम्मति से किए गए संशोधन।

प्रश्न 10.
संविधान के तकनीकी संशोधन से क्या अभिप्राय है?
उत्तर:
संविधान के तकनीकी या प्रशासनिक संशोधन प्रायः ऐसे संशोधन होते हैं जो संविधान के मूल उपबन्धों को स्पष्ट करने । तथा उनकी व्याख्या करने से संबंधित छोटे-मोटे संशोधन होते हैं। वास्तव में इन संशोधनों से कोई विशेष बदलाव नहीं होते, बल्कि ये संशोधन किन्हीं विशेष परिस्थितियों से निपटने के लिए ही किए गए हैं। इसीलिए इन्हें तकनीकी संशोधनों का नाम दिया जाता है।

प्रश्न 11.
भारतीय संविधान के किन्हीं दो तकनीकी संशोधनों के उदाहरण दीजिए।
उत्तर:
(1) भारतीय संविधान में प्रारम्भ में की गई अनुसूचित जातियों एवं अनुसूचित जनजातियों के लिए लोकसभा एवं राज्य की विधानसभाओं में सीटों के आरक्षण प्रावधान को प्रत्येक 10 वर्षों के बाद विभिन्न संवैधानिक संशोधनों द्वारा सन् 2030 तक बढ़ाना,

(2) 54वें संवैधानिक संशोधन द्वारा सर्वोच्च एवं उच्च न्यायालयों के न्यायाधीशों के वेतन, भत्ते, पैंशन एवं सेवानिवृत्ति सम्बन्धी प्रावधानों में उल्लेखनीय सुधार करना।

प्रश्न 12.
भारतीय संविधान में राजनीतिक सर्वसम्मति के आधार पर किए गए किन्हीं दो संशोधनों का उल्लेख कीजिए।
उत्तर:

  • दल-बदल विरोधी कानून, 1985 राजनीतिक सर्वसम्मति के आधार पर हुआ संवैधानिक संशोधन था,
  • मूल संविधान में मतदाता की आयु 21 वर्ष से घटाकर 61वें संवैधानिक संशोधन द्वारा 18 वर्ष करना भी राजनीतिक सर्वसम्मति का ही परिणाम कहा जाता है।

प्रश्न 13.
भारतीय संविधान के किन संशोधनों को अवसरवादी अधिनियम कहकर पुकारा गया?
उत्तर:
स्व० इन्दिरा गाँधी के नेतृत्व में आपात्काल के दौरान विशेषकर अपने तानाशाही एवं स्वेच्छाचारी व्यवहार के आधार पर विशेषकर 38वें, 39वें एवं 42वें संवैधानिक संशोधनों द्वारा संविधान के मूल स्वरूप से छेड़छाड़ के आधार पर किए गए संशोधनों को ‘अवसरवादी अधिनियम’ वाले संशोधनों के नाम आलोचकों के द्वारा दिए गए थे।

प्रश्न 14.
38वाँ संवैधानिक संशोधन विशेषतः न्यायपालिका के किस क्षेत्र पर प्रतिबन्ध लगाने वाला था?
उत्तर:
जुलाई, 1975 में पारित 38वें संवैधानिक संशोधन के द्वारा विशेषकर राष्ट्रपति, राज्यपाल एवं उप-राज्यपाल के द्वारा जारी किए गए अध्यादेशों एवं राष्ट्रपति की आपात्कालीन शक्तियों को न्यायपालिका के अधिकार क्षेत्र से बाहर कर उनकी शक्तियों पर प्रतिबन्ध लगाया गया था।

प्रश्न 15.
42वें संवैधानिक संशोधन की कोई दो विशेषताएँ लिखिए।
उत्तर:

  • राष्ट्रपति के लिए संघीय मंत्रिपरिषद् के दिए गए परामर्श को मानना आवश्यक होगा,
  • भारतीय संविधान की प्रस्तावना में ‘धर्मनिरपेक्ष’ एवं ‘समाजवाद’ शब्द को जोड़ा गया,
  • लोकसभा एवं राज्य विधानसभा का कार्यकाल 5 वर्ष से बढ़ाकर 6 वर्ष किया गया था।

प्रश्न 16.
भारतीय संविधान में किए गए 44वें संशोधन की किन्हीं दो विशेषताओं का उल्लेख कीजिए।
उत्तर:

  • संपत्ति के मूल अधिकार को एक साधारण कानूनी अधिकार में परिवर्तित कर दिया गया,
  • स्वतंत्रता के अधिकार के अंतर्गत अनुच्छेद 19(1) एफ को भी समाप्त कर दिया गया।

HBSE 11th Class Political Science Important Questions Chapter 9 संविधान : एक जीवंत दस्तावेज़

प्रश्न 17.
संविधान में न्यायिक निर्णयों के परिणामस्वरूप हुए किन्हीं दो संशोधनों के उदाहरण दीजिए।
उत्तर:
(1) सर्वोच्च न्यायालय के द्वारा जनवरी, 2004 को यह व्यवस्था की गई कि भारत में नागरिकों को सम्मान एवं प्रतिष्ठा के साथ राष्ट्रीय ध्वज फहराना संविधान के अनुच्छेद 19(1) में उनका एक मौलिक अधिकार है,

(2) सर्वोच्च न्यायालय ने जीवन के अधिकार की एक व्यापक व्याख्या करते हुए जीवन के अधिकार के अंतर्गत भोजन का अधिकार, शुद्ध पेयजल का अधिकार एवं आश्रय के अधिकार को भी शामिल किया है।

प्रश्न 18.
भारतीय संविधान के संदर्भ में ‘संविधान समीक्षा’ की बहस क्यों आरम्भ हुई?
उत्तर:
भारतीय संविधान के संदर्भ में नब्बे के दशक में ‘संविधान समीक्षा’ के पक्षधरों का कहना है कि विश्व अर्थव्यवस्था में उदारीकरण के दौर के परिणामस्वरूप देश की परिस्थितियाँ एवं वातावरण भी तेजी के साथ बदल रहा है। ऐसी स्थिति में भारत में सामाजिक-आर्थिक परिपेक्ष्य में संरचनात्मक सुधार के रास्ते में संविधान की अनेक धाराएँ मेल नहीं खाती हैं। इसलिए यह आवश्यक हो जाता है कि नवीन परिस्थितियों के अनुरूप पूरे संविधान की ही समीक्षा की जाए।

प्रश्न 19.
भारत सरकार ने कब और किसकी अध्यक्षता में ‘संविधान समीक्षा’ आयोग का गठन किया था?
उत्तर:
भारत सरकार ने 22 फरवरी, 2000 को सर्वोच्च न्यायालय के सेवानिवृत्त मुख्य न्यायाधीश श्री वेंकटचलैया की अध्यक्षता में 11 सदस्यीय आयोग का गठन किया था।

प्रश्न 20.
भारतीय लोकतंत्र को शक्तिशाली बनाने में किन्हीं दो गैर-संवैधानिक व्यवस्थाओं का उल्लेख कीजिए।
उत्तर:

  • विपक्षी दलों द्वारा सत्ताधारी दल की अनुचित एवं जनविरोधी नीतियों का डटकर विरोध करना,
  • भारतीय लोगों का संवैधानिक साधनों के प्रति आस्था का होना।

प्रश्न 21.
सामाजिक तनाव एवं हिंसा भारतीय लोकतंत्र के लिए खतरा है, क्यों?
उत्तर:
भारत में जाति, धर्म, धन तथा सामाजिक असमानता के कारण समय-समय पर उत्पन्न हिंसात्मक घटनाओं के कारण जहाँ शांति व्यवस्था भंग होती है, वहाँ लोगों के बीच तनाव एवं वैमनस्यता की भावना होती है जो किसी भी लोकतान्त्रिक पद्धति के लिए खतरे का संकेत होती है। यही बात भारतीय लोकतंत्र पर लागू होती है।

लघूत्तरात्मक प्रश्न

प्रश्न 1.
भारतीय संविधान लचीला है अथवा कठोर। कारणों सहित बताएँ।
उत्तर:
भारतीय संविधान की एक मुख्य विशेषता यह है कि यह न तो पूर्ण रूप से लचीला है और न ही पूर्ण रूप से कठोर है। यह अंशतः लचीला तथा अंशतः कठोर (Partly Flexible and Partly Rigid) है। इसका कारण यह है कि संशोधन के लिए भारतीय संविधान को तीन भागों में बाँटा गया है, जो इस प्रकार हैं

1. संसद द्वारा साधारण बहुमत द्वारा संशोधन-संविधान की कुछ धाराएँ ऐसी हैं, जिनमें संसद अपने साधारण बहुमत से परिवर्तन कर सकती है। इस दृष्टि से हमारा संविधान लचीला है। इस श्रेणी में शामिल विषय इस प्रकार हैं-

  • नए राज्यों को भारतीय संघ में शामिल करना, नए राज्यों की स्थापना करना, उनके क्षेत्र, सीमाओं तथा नामों में परिवर्तन करना,
  • भारत की नागरिकता सम्बन्धी विषय निश्चित करना,
  • किसी राज्य की विधानसभा की सिफारिश पर उस राज्य में विधानपरिषद् स्थापित करना अथवा उसे समाप्त करना।

2. संसद के विशेष बहुमत तथा आधे राज्यों के विधानमंडल द्वारा स्वीकृति-द्विवतीय ढंग में विधेयक को संसद के किसी भी सदन में प्रस्तुत किया जा सकता है। उसे संसद के प्रत्येक सदन को समस्त संख्या के साधारण बहुमत तथा उपस्थित व मतदान संविधान : एक जीवंत दस्तावेज़ के रूप में भाग लेने वाले सदस्यों के 2/3 (दो-तिहाई) बहुमत से पारित किया जाना आवश्यक है।

संसद के दोनों सदनों द्वारा जब वह प्रस्ताव पास हो जाता है, तो उसे आधे राज्यों के विधानमंडल द्वारा स्वीकृति मिलनी चाहिए। उसके पश्चात् इसे राष्ट्रपति की स्वीकृति के लिए भेजा जाता है, जिसकी स्वीकृति मिलने पर वह संशोधन लागू हो जाता है। संविधान संशोधन की यह प्रक्रिया निम्नलिखित विषयों के संबंध में अपनाई जाती है

  • राष्ट्रपति के चुनाव का ढंग,
  • संघ की कार्यपालिका शक्ति का विस्तार,
  • राज्यों की कार्यपालिका शक्ति की सीमा,
  • केन्द्र द्वारा प्रशासित क्षेत्रों (Union Territories) के लिए उच्च न्यायालयों की व्यवस्था,
  • राज्यों के उच्च न्यायालयों सम्बन्धी व्यवस्था।

3. संसद द्वारा दो-तिहाई बहुमत से संशोधन-संविधान के संशोधन की तीसरी प्रणाली दूसरी प्रणाली से कुछ कम कठोर है। इसके अन्तर्गत संविधान में संशोधन सम्बन्धी बिल संसद के किसी भी सदन में पेश किया जा सकता है और यदि वह सदन की कुल संख्या के साधारण बहुमत द्वारा या उपस्थित व मतदान में भाग लेने वाले सदस्यों के 2/3 बहुमत द्वारा पास हो जाता है तो उसे दूसरे सदन में स्वीकृति के लिए भेज दिया जाता है। संविधान के जिन विषयों का वर्णन ऊपर प्रथम तथा दूसरे वर्ग में किया गया है, उनको छोड़कर सभी विषयों में परिवर्तन इसी प्रक्रिया से किया जा सकता है।

संसद में बिल पास होने के बाद राष्ट्रपति की स्वीकृति के लिए भेजा जाता है। संविधान में संशोधन 24 के अनुसार राष्ट्रपति को अपनी स्वीकृति देनी पड़ती है। ऊपर दिए गए विवरण से यह स्पष्ट हो जाता है कि भारत में संविधान संशोधन के लिए संविधान के अलग-अलग भागों के लिए कठोर एवं लचीली प्रक्रिया दोनों अपनाई गई हैं; इसलिए भारतीय संविधान न पूर्ण रूप से लचीला है और न ही पूर्णतः कठोर है। यह अंशतः लचीला तथा अंशतः कठोर है।

HBSE 11th Class Political Science Important Questions Chapter 9 संविधान : एक जीवंत दस्तावेज़

प्रश्न 2.
कोई चार ऐसे संशोधन बताइए जो राजनीतिक सर्वसम्मति से किए गए हैं?
उत्तर:
संविधान के कुछ संशोधन ऐसे भी हैं जो राजनीतिक सर्वसम्मति के आधार पर संविधान के भाग बने हैं। ऐसे संशोधन वास्तव में तत्कालीन राजनीतिक दर्शन एवं समाज की आकांक्षाओं की अभिव्यक्ति माने जाते हैं। ऐसे संशोधनों के कुछ उदाहरण हम निम्नलिखित प्रकार से दे सकते हैं

1. 52वां संवैधानिक संशोधन भारतीय राजनीति को सबसे अधिक प्रभावित करने वाली दल-बदल की प्रवृत्ति पर अंकुश लगाने के लिए लाया गया दल-बदल निरोधक कानून, 1985 वास्तव में राजनीतिक आम सहमति का ही परिणाम था। इसके अतिरिक्त 52वें संशोधन की कुछ कमियों को दूर करने के लिए पुनः 91वां संशोधन (2003) भी वास्तव में राजनीतिक सर्वसम्मति की ही देन कहा जा सकता है।

इस संशोधन द्वारा विशेषकर दल-बदल को निरुत्साहित करने के लिए मन्त्रिमंडल के आकार को सीमित किया गया जिसके द्वारा निश्चित किया गया कि मन्त्री पदों की संख्या लोकसभा या राज्य विधानसभा के सदस्यों की कुल संख्या के 15 प्रतिशत से अधिक नहीं होनी चाहिए। जैसा कि सर्वविदित है दल-बदल का मुख्य कारण सत्ता प्रलोभन रहा है। ऐसे कानून ऐसी प्रवृत्ति के राजनेताओं पर एक महत्त्वपूर्ण अंकुश के रूप में काम करेगा।

2. 61वां संवैधानिक संशोधन इसके द्वारा मतदान की आयु 21 वर्ष से घटाकर 18 वर्ष करना भी आम राजनीतिक सहमति का ही परिणाम था।

3. 73वां एवं 74वां संवैधानिक संशोधन-इन संशोधनों के द्वारा संपूर्ण भारत में पंचायती राज को लागू करना भी आम राजनीतिक सहमति का अच्छा उदाहरण कहा जा सकता है।

4. 93 वां संवैधानिक संशोधन इसके द्वारा उच्च शिक्षण संस्थाओं में पिछड़े वर्गों को आरक्षण देना भी आम राजनीतिक सहमति का ही उदाहरण कहा जा सकता है।

अतः स्पष्ट है कि संविधान में कुछ महत्त्वपूर्ण संशोधन राजनीतिक आम सहमति के परिणामस्वरूप किए गए। ऐसे संशोधन तत्कालिक राजनीतिक दर्शन एवं समाज की महत्त्वाकांक्षाओं की पूर्ति करते हुए संविधान को जीवंत या गतिशील बनाए रखते हैं।

प्रश्न 3.
संविधान के बुनियादी ढाँचे (Basic Structure of the Constitution) पर नोट लिखें।
उत्तर:
बुनियादी या मूलभूत ढाँचा एक सैद्धांतिक बात है जो स्वयं में एक जीवंत संविधान का उदाहरण है। संविधान में इस अवधारणा का कोई उल्लेख नहीं मिलता। वास्तव में यह एक ऐसा विचार है जो न्यायिक व्याख्याओं से उत्पन्न हुआ है। मौलिक अधिकारों से संबंधित केशवानन्द भारती के विवाद में सर्वोच्च न्यायालय ने इस सिद्धांत (मूल ढाँचे का सिद्धांत) का प्रतिपादन किया था। इस निर्णय ने संविधान के विकास में निम्नलिखित सहयोग दिया

(1) इस निर्णय द्वारा संविधान में संशोधन करने की संसद की शक्तियों की सीमाएँ निर्धारित की गईं।

(2) यह निर्णय संविधान के किसी भी या संपूर्ण भागों के किसी भी प्रकार के संशोधन की स्वीकृति देता है। यहाँ तक कि मूल अधिकारों में भी संशोधन की पूर्ण स्वीकृति दी गई है। अतः संविधान के मूल ढाँचे में संशोधन किए बिना संविधान के किसी भी भाग में संसद द्वारा संशोधन या परिवर्तन किया जा सकता है।

(3) इस निर्णय से यह भी स्पष्ट है कि संविधान के मूल ढाँचे में उल्लंघन करने वाले संशोधन के संबंध में अन्तिम निर्णय करने का अधिकार न्यायपालिका का होगा। यद्यपि मूल ढाँचा क्या है? यह सर्वोच्च न्यायालय द्वारा अभी स्पष्ट नहीं किया गया है, लेकिन समय-समय पर विभिन्न विवादों में दिए गए निर्णयों से संविधान के मूलभूत ढाँचे के सम्बन्ध में कुछ बिन्दु उभरकर सामने आए हैं जो इस प्रकार हैं-

  • संविधान की सर्वोच्चता,
  • संविधान का धर्म-निरपेक्ष स्वरूप,
  • शासन का लोकतन्त्रात्मक एवं गणतन्त्रीय रूप,
  • कानून का शासन एवं न्यायिक समीक्षा,
  • संविधान का संघीय स्वरूप,
  • स्वतन्त्र एवं निष्पक्ष न्यायपालिका,
  • कार्यपालिका, व्यवस्थापिका एवं न्यायपालिका के मध्य स्थापित समीकरण,
  • भारत की प्रभुसत्ता,
  • देश की अखंडता आदि।

इस प्रकार मूल संरचना या ढाँचे का सिद्धांत संविधान की कठोरता एवं लचीलेपन की मिश्रित व्यवस्था को इंगित करता है। संविधान के मूल ढांचे को संविधान संशोधन प्रक्रिया के दायरे से बाहर रखने की व्यवस्था संविधान की कठोरता को व्यक्त करता है। दूसरे शब्दों में, संविधान का मल ढाँचे से संबंधित भाग अपरिवर्तनीय रहेगा जिसे कभी भी नहीं बदला जा सकता. जबकि दसरी तरफ संविधान के कुछ भागों को संशोधन प्रक्रिया के अधीन करने की व्यवस्था लचीलेपन को व्यक्त करती है। इस तरह हमारा संविधान अंशतः लचीले एवं अंशतः कठोर स्वरूप वाला कहा जाता है।

प्रश्न 4.
संविधान के मूल ढाँचे पर संविधान समीक्षा की स्थिति का वर्णन करें।
उत्तर:
भारत में नब्बे के दशक के अन्तिम वर्षों में एक अन्य विचार विशेषकर संविधान समीक्षा को लेकर भी उभरा। ऐसा विचार रखने वाले विचारकों का कहना है कि देश का वातावरण एवं परिस्थितियाँ तेजी से बदल रही हैं और इन विशेष परिस्थितियों से संविधान की अनेक धाराएँ मेल नहीं खाती हैं। इसके अतिरिक्त विश्व अर्थव्यवस्था में भी उदारीकरण का नया दौर प्रारम्भ हुआ है।

इसलिए भारत में भी आर्थिक उदारीकरण के इस दौर में आर्थिक-सामाजिक परिपेक्ष्य में संरचनात्मक सुधार का सिलसिला प्रारंभ करना आवश्यक है। इसलिए आवश्यकता इस बात की है कि नवीन परिस्थितियों के अनुरूप पूरे संविधान की ही समीक्षा की जाए। ने 22 फरवरी, 2000 को सर्वोच्च न्यायालय के सेवानिवृत्त मुख्य न्यायाधीश श्री वेंकटचलैया की अध्यक्षता में 11 सदस्यीय संविधान समीक्षा हेतु राष्ट्रीय आयोग का गठन किया गया। संविधान समीक्षा आयोग ने 31 मार्च, 2002 को अपना प्रतिवेदन भारत सरकार को सौंप दिया। कुल मिलाकर आयोग की रिर्पोट के 1976 पृष्ठों में 249 सिफारशें की गई हैं।

58 अनुशंसाएँ संविधान में संशोधन करने, 86 अनुशंसाएँ विधायी कार्रवाई करने और 106 अनुशंसाएँ कार्यकारी कार्रवाई के माध्यम से क्रियान्वित करने का सुझाव दिया है। परन्तु इसमें सबसे अधिक ध्यान देने योग्य बात यह है कि इस आयोग ने संविधान के बुनियादी ढाँचे पर पूर्ण विश्वास जताया है और इसमें किसी ऐसे परिवर्तन की सिफारिश नहीं की जो इसके मूल ढाँचे पर प्रतिकूल प्रभाव डाले। अतः स्पष्ट है संविधान का मूल ढाँचा पूर्व में हुए निर्णयों के अनुरूप अपरिवर्तित ही रहेगा।

HBSE 11th Class Political Science Important Questions Chapter 9 संविधान : एक जीवंत दस्तावेज़

प्रश्न 5.
भारतीय संविधान की संशोधन विधि की विशेषताएँ लिखिए।
उत्तर:
(1) संविधान में संशोधन विधेयक संसद के दोनों सदनों में से किसी भी सदन में पेश किया जा सकता है, परंतु संशोधन विधेयक दोनों सदनों में अलग-अलग रूप में पास होना आवश्यक है,

(2) संशोधन विधेयक पर दोनों सदनों में मतभेद होने पर विधेयक गिर जाएगा, क्योंकि असहमति की स्थिति में दोनों सदनों के संयुक्त अधिवेशन बुलाने की व्यवस्था का संविधान में उल्लेख नहीं किया गया है,

(3) राज्य विधानमंडलों को संशोधन प्रस्ताव पेश करने का अधिकार नहीं है। राज्यों को केवल तीसरी विधि के अंतर्गत किए गए संशोधनों पर स्वीकृति देने की शक्ति प्रदान की गई है। इसमें भी केवल आधे राज्यों के विधानमंडलों की स्वीकृति की आवश्यकता पड़ती है, परंतु इनके द्वारा दी जाने वाली स्वीकृति के लिए कोई सीमा निश्चित नहीं की गई है,

(4) संसद द्वारा पारित किए गए संशोधन विधेयकों को स्वीकृति के लिए राष्ट्रपति के पास भेजा जाएगा। जिस पर राष्ट्रपति को अपनी स्वीकृति देनी पड़ती है,

(5) संसद संविधान के किसी भी भाग में अनुच्छेद 368 के अधीन संशोधन कर सकती है, बशर्ते ऐसा संशोधन संविधान के मूल ढाँचे में परिवर्तन न करता हो,

(6) संसद द्वारा किए गए संवैधानिक संशोधन की वैधता की न्यायपालिका को जांच करने का अधिकार है,

(7) संविधान में किए जाने वाले संशोधनों पर जनमत संग्रह कराए जाने की व्यवस्था नहीं है।

निबंधात्मक प्रश्न

प्रश्न 1.
भारतीय संविधान में संशोधन करने की प्रक्रिया का वर्णन कीजिए। अथवा भारतीय संविधान में संशोधन करने की प्रक्रिया की आलोचनात्मक व्याख्या करें।
उत्तर:
भारतीय संविधान की एक मुख्य विशेषता यह है कि यह न तो पूर्णरूप से लचीला है और न ही पूर्ण रूप से कठोर। यह अंशतः लचीला तथा अंशतः कठोर (Partly Flexible and Partly Rigid) है। प्रो० वीयर (Prof. Viyar) के शब्दों में, “यह संविधान चरम कठोरता तथा अत्यन्त लचीलेपन में एक अच्छा सन्तुलन स्थापित करता है।”

संविधान के निर्माण के समय श्री जवाहरलाल नेहरू (Sh. Jawaharlal Nehru) ने इस सम्बन्ध में अपने विचार प्रकट करते हुए कहा था, “यद्यपि हम इस संविधान को इतना ठोस तथा स्थायी बनाना चाहते हैं जितना कि हम बना सकते हैं, परन्तु संविधान का स्थायी होना वांछनीय है। उसमें कुछ लचीलापन अवश्य होना चाहिए। यदि आप किसी वस्तु को पूर्ण रूप से स्थायी और कठोर बना देंगे तो इससे राष्ट्र का विकास रुक जाएगा क्योंकि राष्ट्र जीवित प्राणियों का समूह है। किसी भी स्थिति में हम अपने संविधान को इतना कठोर नहीं बना सकते कि वह बदलती हुई परिस्थितियों के अनुरूप बदल न सके।”

संघीय शासन-व्यवस्था के लिए संविधान का कठोर होना आवश्यक है, ताकि केन्द्रीय सरकार में सत्ताधारी दल उसमें अपनी इच्छानुसार परिवर्तन न कर सके, परन्तु संविधान इतना कठोर भी नहीं होना चाहिए कि बदलती हुई परिस्थितियों के अनुसार उसे बदला न जा सके। इसी कारण से हमारा संविधान न तो पूर्ण रूप से कठोर और न ही पूर्ण रूप से लचीला बनाया गया है।

भारतीय संविधान में संशोधन करने की प्रक्रिया का वर्णन अनुच्छेद 368 के अन्तर्गत किया गया है। इसके अनुसार संशोधन के कार्य के लिए संविधान को निम्नलिखित तीन भागों में बाँटा गया है

1. संसद द्वारा साधारण बहुमत से संशोधन (Amendment by the Parliament bya Simple Majority)-संविधान की कुछ धाराएँ ऐसी हैं, जिनमें संसद अपने साधारण बहुमत से परिवर्तन कर सकती है। इस दृष्टि से हमारा संविधान लचीला है। इस श्रेणी में ये विषय शामिल हैं

  • नए राज्यों को भारतीय संघ में शामिल करना, नए राज्यों की स्थापना करना, उनके क्षेत्र, सीमाओं तथा नामों में परिवर्तन करना,
  • भारत की नागरिकता सम्बन्धी विषय,
  • किसी राज्य की विधानसभा की सिफारिश पर उस राज्य में विधानपरिषद् स्थापित करना अथवा उसे समाप्त करना,
  • राष्ट्रभाषा सम्बन्धी विषय,
  • संसद तथा राज्य विधानमण्डलों के सदस्यों के लिए योग्यताएँ निश्चित करना,
  • संसद के सदस्यों को वेतन, भत्ते तथा अन्य सुविधाएँ प्रदान करना,
  • उच्चतम न्यायालय के न्यायाधीशों की संख्या,
  • अनुसूचित जातियों, अनुसूचित जनजातियों तथा पिछड़े हुए कबीलों के प्रशासन आदि से संबंधित विषयों में परिवर्तन।

2. संसद के बहुमत तथा आधे राज्यों के विधानमण्डलों द्वारा स्वीकृति (By a Special Majority of Parliament and Ratification by the Legislatures of at least fifty percent States) दूसरी श्रेणी में वे विषय शामिल किए गए हैं जो वास्तव में संघ एवं राज्यों से सम्बन्धित हैं। इनमें संशोधन करने के लिए अत्यन्त कठोर प्रणाली अपनाई गई है।

इन विषयों के सम्बन्ध में संशोधन के प्रस्ताव को दो चरण पार करने होते हैं। सर्वप्रथम, संशोधन विधेयक को संसद के किसी भी सदन में प्रस्तुत किया जा सकता है। उसे संसद के प्रत्येक सदन की समस्त जनसंख्या के साधारण बहुमत तथा उपस्थित व मतदान में भाग लेने वाले सदस्यों के 2/3 (दो-तिहाई बहुमत से पारित किया जाना आवश्यक है। संसद द्वारा ऊपर दी गई प्रक्रियानुसार जब वह प्रस्ताव पास हो जाता है तो उसे राज्यों की स्वीकृति के लिए भेज दिया जाता है।

उस विधेयक को कम-से-कम आधे राज्यों के विधानमण्डलों द्वारा स्वीकृति मिलनी चाहिए। उसके पश्चात् राष्ट्रपति की स्वीकृति से संविधान में आवश्यक संशोधन लागू होता है। संविधान संशोधन की यह प्रक्रिया दिए गए विषयों के सम्बन्ध में अपनाई जाती है-

  • राष्ट्रपति के चुनाव का ढंग,
  • संघ की कार्यपालिका शक्ति का विस्तार,
  • राज्यों की कार्यपालिका शक्ति की सीमा,
  • केन्द्र द्वारा प्रशासित क्षेत्रों (Union Territories) के लिए उच्च न्यायालयों की व्यवस्था,
  • राज्यों के उच्च न्यायालयों सम्बन्धी व्यवस्था,
  • संघ तथा राज्यों के विधायी सम्बन्ध,
  • संसद में राज्यों का प्रतिनिधित्व,
  • संघीय न्यायपालिका सम्बन्धी व्यवस्था,
  • संविधान में संशोधन की प्रक्रिया।

3. संसद द्वारा दो-तिहाई बहुमत से संशोधन (Amendment by Parliament by a two-third Majority)-संविधान में संशोधन की तीसरी प्रणाली दूसरी प्रणाली से कुछ कम कठोर है। इसके अन्तर्गत संविधान में संशोधन सम्बन्धी विधेयक संसद के किसी भी सदन में पेश किया जा सकता है और यदि वह सदन की कुल संख्या के साधारण बहुमत द्वारा या

उपस्थित व मतदान में भाग लेने वाले सदस्यों के 2/3 बहुमत द्वारा पास हो जाता है तो उसे दूसरे सदन में स्वीकृति के लिए भेज दिया जाता है। दूसरे सदन में भी यदि वह ऊपर दिए गए ढंग से पास हो जाता है तो उसे राष्ट्रपति की स्वीकृति के लिए भेज दिया जाता है। संविधान के जिन विषयों का वर्णन ऊपर प्रथम या दूसरे वर्ग में किया गया है, उनको छोड़कर शेष सभी विषयों में परिवर्तन इसी प्रक्रिया से किया जा सकता है।

इस प्रकार हम देखते हैं कि यद्यपि हमारा संविधान एक कठोर संविधान है, परन्तु यह इतना कठोर नहीं है कि इसमें देश की बदलती परिस्थितियों के अनुसार परिवर्तन न किया जा सके। संशोधन प्रक्रिया की आलोचना (Criticism of the Procedure of Amendment) भारतीय संविधान में संशोधन की प्रक्रिया की निम्नलिखित बातों के आधार पर आलोचना की गई है

(1) संविधान में संशोधन का प्रस्ताव केवल संघीय संसद में ही आरम्भ किया जा सकता है, राज्यों अथवा जनता के पास इस कार्य में पहल करने का अधिकार नहीं है।

(2) संविधान इस बारे में स्पष्ट नहीं है कि प्रस्तावित संशोधन को संसद के द्वारा पास होने के पश्चात् जब राज्यों के पास भेजा जाता है तो उनकी स्वीकृति कितने समय में मिल जानी चाहिए। अमेरिका में जब कांग्रेस किसी संशोधन विधेयक को पास करके राज्यों के पास स्वीकृति के लिए भेजती है तो वह उसके लिए समय निश्चित कर देती है

(3) संविधान में इस बात को भी स्पष्ट नहीं किया गया था कि संविधान में संशोधन का प्रस्ताव जब तक संसद के दोनों सदनों से पास होने के पश्चात् राष्ट्रपति के पास जाता है तो क्या राष्ट्रपति को उस पर निषेधाधिकार (Veto Power) प्राप्त है अथवा उसे स्वीकृति देनी ही पड़ेगी, परन्तु अब संविधान के 24वें संशोधन द्वारा इस अस्पष्टता को दूर कर दिया गया है। इस संवैधानिक संशोधन में यह कहा गया है कि जब कोई संशोधन विधेयक संसद के दोनों सदनों से पास होकर राष्ट्रपति के पास उसकी अनुमति के लिए भेजा जाए तो राष्ट्रपति को उस पर अपनी स्वीकृति देनी ही पड़ेगी।

(4) संविधान में इस बात को भी स्पष्ट नहीं किया गया है कि यदि किसी संविधान संशोधन के प्रस्ताव को पास करने के बारे में संसद के दोनों सदनों में मतभेद उत्पन्न हो जाए तो उसे कैसे सुलझाया जाएगा। अधिकांश लोगों का विचार है कि उस गतिरोध को संसद के दोनों सदनों की संयुक्त बैठक में सुलझाया जाएगा (जैसे किसी साधारण विधेयक के पास करने के सम्बन्ध में किया जाता है)।

(5) संविधान में कुछ अनुच्छेद ऐसे हैं, जिनको संशोधन प्रस्ताव पास किए बिना ही बदला जा सकता है। संघीय सरकार किसी विदेशी सन्धि अथवा समझौते की शर्तों को पूरा करने के लिए राज्य-सूची में दिए गए विषयों के सम्बन्ध में भी कानून बना सकती है। इसी प्रकार यदि राज्यसभा अपने 2/3 बहुमत से यह प्रस्ताव पास कर दे कि राज्य-सूची में दिया गया कोई विषय राष्ट्रीय महत्त्व का हो गया है तो संसद उस पर भी कानून बना सकती है। यह व्यवस्था संघीय प्रणाली के विरुद्ध है।।

निष्कर्ष (Conclusion)-ऊपर दी गई सभी बातों को ध्यान में रखते हुए हम यही कह सकते हैं कि भारतीय संविधान लचीला भी है और कठोर भी। संकटकाल में यह लचीला और एकात्मक बन जाता है। शान्तिकाल में इसका कुछ भाग काफी लचीला कुछ भाग कठोर हो जाता है। इस प्रकार भारतीय संविधान न तो इंग्लैण्ड के संविधान की तरह पूर्ण रूप से लचीला है और न ही अमेरिकन संविधान की भाँति बहुत कठोर है।

HBSE 11th Class Political Science Important Questions Chapter 9 संविधान : एक जीवंत दस्तावेज़

प्रश्न 2.
भारतीय संविधान के लागू होने से लेकर अब तक संविधान में कौन-से मुख्य परिर्वतन किए गए हैं? अथवा भारतीय संविधान में हुए परिवर्तनों ने किस प्रकार इसके रूप को बदला है? कुछ उदाहरणों सहित व्याख्या करें।
उत्तर:
जनवरी, 2019 तक हमारे संविधान को लागू हुए लगभग 70 वर्ष से अधिक हो गए हैं। इन थोड़े से वर्षों में ही हमारे संविधान में 104 संशोधन (दिसम्बर, 2019) तक हो चुके हैं। जबकि 1789 में लागू होने वाले अमेरिकन संविधान में मात्र 27 संशोधन ही हुए हैं। यहाँ पर यही प्रश्न हमारे मस्तिष्क में आता है कि इतने थोड़े से समय में भारतीय संविधान में संशोधन करने के क्या कारण रहे हैं और इसके निहितार्थ क्या हैं: यहाँ यह तथ्य उल्लेखनीय है कि संविधान लागू होने के प्रथम दस वर्षों में केवल 9 संशोधन हुए और आगामी 60 वर्षों में लगभग 93 संशोधन हो गए। यदि संविधान संशोधनों के इतिहास पर नजर डालें तो यह भी स्पष्ट हो जाता है कि संविधान में किए जाने वाले संशोधनों का भारत की राजनीतिक परिस्थितियों से घनिष्ठ सम्बन्ध रहा है।

संसद द्वारा संविधान में किया गया प्रत्येक संशोधन समय की आवश्यकता थी। संविधान में संसद द्वारा संशोधन तब ही किए गए जब न्यायिक निर्णयों के द्वारा संविधान के कुछ विशेष अनुच्छेदों में कमी बताई गई या संविधान निर्माताओं की महत्त्वाकांक्षाओं की गलत तरीके से व्याख्या की गई, जैसे संसद द्वारा मूल अधिकारों के सम्बन्ध में किए गए संशोधन का कारण समाजवादी समाज के रास्ते में बाधा का होना रहा तो कुछ संशोधनों का ध्येय राज्य-नीति के निदेशक सिद्धांतों को व्यावहारिक रूप देना रहा।

कई संशोधन भूमि-सुधार एवं गरीबों को न्यायपालिका की कार्य-पद्धति द्वारा शीघ्र न्याय दिलवाने में असमर्थता के कारण हुए। इन उदाहरणों से स्पष्ट है कि इन विभिन्न संवैधानिक संशोधनों के पीछे केवल सत्ताधारी दल की राजनीतिक सोच ही बहुत मायने नहीं रखती थी, बल्कि विभिन्न सवैधानिक संशोधन उत्पन्न राजनीतिक परिस्थितियों एवं समाज और संविधान के आदर्शों के अनुरूप ही किए गए। हम भारतीय संविधान में हुए संशोधनों की विषय-वस्तु को समझने के लिए इसे अग्रलिखित तीन श्रेणियों में विभाजित करके स्पष्ट कर सकते हैं

1. तकनीकी एवं प्रशासनिक समस्याओं से संबंधित संशोधन (Amendments Regarding Technical and Administrative Problems)-तकनीकी या प्रशासनिक प्रकृति के संशोधन प्रायः ऐसे संशोधन होते हैं जो संविधान के मूल उपबन्धों को स्पष्ट करने, उनकी व्याख्या करने से संबंधित छोटे-मोटे संशोधन होते हैं।

वास्तव में इन संशोधनों से कोई विशेष बदलाव नहीं होते, बल्कि ये किन्हीं विशेष परिस्थितियों से निपटने के लिए ही किए जाते हैं। इसीलिए इन्हें तकनीकी या प्रशासनिक संशोधनों का नाम दिया जाता है। संविधान में किए गए ऐसे तकनीकी संशोधनों के कुछ उदाहरण निम्नलिखित रूप में बताए जा सकते हैं

(1) आरक्षण की व्यवस्था से संबंधित भारतीय संविधान में प्रारम्भ में अनुसूचित जातियों एवं अनुसूचित जनजातियों के लिए लोकसभा एवं राज्य विधानसभाओं में सीटों का आरक्षण केवल 10 वर्षों के लिए किया गया था। परंतु इस वर्ग के लोगों के आर्थिक पिछडेपन के मध्यनजर प्रत्येक दस वर्ष के बाद आगामी दस वर्षों तक आरक्षण की अवधि को बढ़ाने के लिए संविधान में पाँच बार संशोधन किए जा चुके हैं।

जैसे 8वाँ संशोधन 1960 में, 23वाँ संशोधन 1969 में, 45वाँ संशोधन 1980 में, 62वाँ संशोधन 1989 में तथा 79वाँ संशोधन 1999 में 95वें संशोधन 2020 में किया गया। वर्तमान में इस वर्ग के आरक्षण प्रावट पान को 104वें संवैधानिक संशोधन (दिसम्बर, 2019) द्वारा 25 जनवरी, 2030 तक रखने की व्यवस्था कर दी गई है।

(2) 15वें संवैधानिक संशोधन द्वारा उच्च न्यायालय के न्यायाधीशों की सेवानिवृत्ति की आयु को 60 वर्ष से बढ़ाकर 62 वर्ष तक करना।

(3) 54वें संवैधानिक संशोधन द्वारा सर्वोच्च एवं उच्च न्यायालयों के न्यायाधीशों के वेतन, भत्तों, पेंशन एवं सेवा-निवृत्ति सम्बन्धी प्रावधानों में उल्लेखनीय सुधार करना।

(4) 21वाँ संशोधन, 1966 इस संशोधन के द्वारा संविधान की 8वीं अनुसूची में परिवर्तन करके सिन्धी भाषा को भी राष्ट्रीय भाषा के रूप में स्वीकार कर लिया गया जिससे 8वीं अनुसूची में 17 भाषाएँ हो गई थी।

(5) 71वाँ संशोधन, 1992-इस संशोधन के द्वारा मणिपुरी, कोंकणी व नेपाली भाषा को संविधान की आठवीं अनुसूची में शामिल कर लिया गया जिससे 8वीं अनुसूची में कुल 18 भाषाएँ हो गई थी।

(6) 92वाँ संशोधन, 2003-इस संवैधानिक संशोधन के द्वारा संविधान की आठवीं अनुसूची में संशोधन किया गया तथा बोडो, डोगरी, मैथिली एवं संथाली भाषाओं को संविधान की आठवीं अनुसूची में शामिल कर लिया गया है। इस प्रकार अब संविधान की आठवीं अनुसूची में 22 भाषाएं हो गई हैं।

(7) 96वाँ संशोधन इस संशोधन को राष्ट्रपति द्वारा 23 सितम्बर, 2011 को स्वीकृति दी गई। इस संशोधन द्वारा भारतीय संविधान की आठवीं अनुसूची में अंकित भाषा ‘ओरिया’ (Oriya) के स्थान पर ‘ओडिया’ (Odia) नाम परिवर्तित किया गया।

(8) 85वाँ संशोधन, 2002-इस संवैधानिक संशोधन (सन् 2002) के द्वारा अनुच्छेद 16 (4A) में परिवर्तन करते हुए सरकारी सेवाओं में अनुसूचित जातियों एवं जनजातियों के अभ्यर्थियों के लिए पदोन्नतियों में आरक्षण की व्यवस्था का प्रावधान किया गया।

(9) 89वाँ संशोधन, 2003–इस संशोधन अधिनियम द्वारा संविधान के अनुच्छेद 338 के तुरन्त बाद अनुच्छेद 338क जोड़कर अनुसूचित जनजातियों के लिए अलग से राष्ट्रीय अनुसूचित जनजाति आयोग के गठन करने का प्रावधान किया गया है। इस आयोग में एक अध्यक्ष, एक उपाध्यक्ष एवं तीन अन्य सदस्य होंगे तथा इनकी सेवा शर्तों एवं पदावधि के सम्बन्ध में निर्णय राष्ट्रपति द्वारा निर्धारित किए जाएंगे।

(10) 41वें संशोधन द्वारा राज्य लोक सेवा आयोग एवं संयुक्त लोक सेवा आयोग के सदस्यों की सेवा-निवृत्ति आयु को 60 वर्ष से बढ़ाकर 62 वर्ष करना आदि। अतः उपर्युक्त उदाहरणों से स्पष्ट है कि उपर्युक्त संशोधनों से संविधान के मूल उपबन्धों में कोई परिवर्तन नहीं आया। इसीलिए हम ऐसे संशोधनों को तकनीकी या प्रशासनिक संशोधनों का नाम देते हैं जो किन्हीं विशेष परिस्थितियों एवं समस्याओं के समाधान हेतु किए गए हैं।

2. राजनीतिक सर्वसम्मति से किए गए संशोधन (Amendments by Political Unanimity):
संविधान के कुछ संशोधन ऐसे भी हैं जो राजनीतिक सर्वसम्मति के आधार पर संविधान के भाग बने हैं। ऐसे संशोधन वास्तव में तत्कालीन राजनीतिक दर्शन एवं समाज की आकांक्षाओं की अभिव्यक्ति माने जाते हैं। ऐसे संशोधनों के कुछ उदाहरण हम निम्नलिखित प्रकार से दे सकते हैं

(1) 52वाँ संवैधानिक संशोधन भारतीय राजनीति को सबसे अधिक प्रभावित करने वाली दल-बदल की प्रवृत्ति पर अंकुश लगाने के लिए लाया गया दल-बदल निरोधक कानून, 1985 वास्तव में राजनीतिक आम सहमति का ही परिणाम था। इसके अतिरिक्त 52वें संशोधन की कुछ कमियों को दूर करने के लिए पुनः 91वाँ संशोधन (2003) भी वास्तव में राजनीतिक सर्वसम्मति की ही देन कहा जा सकता है।

इस संशोधन द्वारा विशेषकर दल-बदल को निरुत्साहित करने के लिए मंत्रिमंडल के आकार को सीमित किया गया जिसके द्वारा निश्चित किया गया कि मंत्री पदों की संख्या लोकसभा या राज्य विधानसभा के सदस्यों की कुल संख्या के 15 प्रतिशत से अधिक नहीं होनी चाहिए। जैसा कि सर्वविदित है, दल-बदल का मुख्य कारण सत्ता प्रलोभन रहा है। ऐसे कानून ऐसी प्रवृत्ति के राजनेताओं पर एक महत्त्वपूर्ण अंकुश के रूप में काम करेगा।

(2) 61वें संवैधानिक संशोधन के द्वारा मतदान की आयु 21 वर्ष से घटाकर 18 वर्ष करना भी आम राजनीतिक सहमति का ही परिणाम था।

(3) 73वाँ एवं 74वाँ सवैधानिक संशोधन-इन संशोधनों के द्वारा सम्पूर्ण भारत में पंचायती राज को लागू करना भी आम राजनीतिक सहमति का अच्छा उदाहरण कहा जा सकता है।

(4) 102वाँ सवैधानिक संशोधन-राष्ट्रीय पिछड़ा वर्ग आयोग विधेयक, 2017 को लोकसभा द्वारा 10 अप्रैल, 2017 तथा राज्यसभा द्वारा 11 अप्रैल, 2017 को राज्यसभा की सिलेक्ट कमेटी को सौंपा गया। जिसने 19 जुलाई, 2017 को अपनी रिपोर्ट सौंपी जिसके उपरान्त 6 अगस्त, 2018 को इसे पास किया गया। इस विधेयक के पारित होने पर आयोग को संवैधानिक दर्जा मिलने के कारण संविधान में अनुच्छेद 342 (क) जोड़कर नए पिछड़े वर्ग आयोग को सिविल न्यायालय के समकक्ष अधिकार प्राप्त होंगे जिसमें आयोग को पिछड़े वर्ग की शिकायतों का निवारण करने का अधिकार भी मिल जाएगा।

संविधान के अनुच्छेद 338
(क) के बाद नया अनुच्छेद 338
(ख) जोड़ा गया।

इसमें सामाजिक एवं शैक्षणिक दृष्टि से पिछड़े वर्गों के लिए राष्ट्रीय पिछड़ा वर्ग आयोग नामक एक नया आयोग होगा। संसद द्वारा पास किए गए विधेयक के अधीन इस आयोग में एक अध्यक्ष, उपाध्यक्ष एवं तीन अन्य सदस्य होंगे। इनकी पदावधि, सेवा एवं शर्ते आदि राष्ट्रपति द्वारा निर्धारित होगी। आयोग को अपनी स्वयं की प्रक्रिया विनियमित करने की शक्ति होगी। आयोग को संविधान के अधीन सामाजिक एवं शैक्षणिक दृष्टि से पिछड़े वर्गों के लिए उपबंधित सुरक्षा उपाय से सम्बन्धित मामलों की जांच और निगरानी करने का अधिकार होगा।

(5) 103वाँ संवैधानिक संशोधन सामान्य वर्ग के आर्थिक रूप से कमजोर लोगों को शिक्षा एवं रोजगार में 10% आरक्षण देने से सम्बन्धित 124वाँ संशोधन विधेयक (संख्या की दृष्टि से 103वाँ संशोधन विधेयक) लोकसभा द्वारा 8 जनवरी, 2019 को 323-3 मतान्तर से एवं राज्यसभा द्वारा 9 जनवरी, 2019 को 165-7 मतान्तर से पास होने के बाद राष्ट्रपति की स्वीकृति के लिए भेजा गया जिसे तत्कालीन राष्ट्रपति द्वारा 12 जनवरी, 2019 को स्वीकार करने पर सरकारी गजट में इसे 103वाँ संविधान संशोधन अधिनियम के रूप में 14 जनवरी, 2019 को सूचीबद्ध किया गया।

(6) 101वाँ संवैधानिक संशोधन भारतीय संविधान में वस्तु एवं सेवा कर (जी एस टी) से सम्बन्धित 101वाँ संविधान संशोधन विधेयक लोकसभा द्वारा 8 अगस्त, 2016 को एवं राज्यसभा द्वारा 3 अगस्त, 2016 को सर्वसम्मति के साथ पारित किया गया। संविधान संशोधन विधेयक को राष्ट्रपति की स्वीकृति से पूर्व आधे से अधिक राज्यों के विधानमण्डलों द्वारा इसका समर्थन किया जाना आवश्यक था। इस आवश्यक शर्त की पूर्ति होने के पश्चात् 8 सितम्बर, 2016 को राष्ट्रपति द्वारा इस विधेयक को स्वीकृति प्रदान कर दी गई। इसी के साथ संविधान का 101वाँ संशोधन अधिनियम, 2016 में लागू हुआ।

(7) 93वें संवैधानिक संशोधन के द्वारा उच्च शिक्षण संस्थाओं में पिछड़े वर्गों को आरक्षण देना भी आम राजनीतिक सहमति का ही उदाहरण कहा जा सकता है। अतः स्पष्ट है कि संविधान में कुछ महत्त्वपूर्ण संशोधन राजनीतिक आम सहमति के परिणामस्वरूप किए गए। ऐसे संशोधन तत्कालिक राजनीतिक दर्शन एवं समाज की महत्त्वाकांक्षाओं की पूर्ति करते हुए संविधान को जीवंत या गतिशील बनाए रखते हैं।

3. न्यायपालिका एवं विधायिका (संसद) के बीच उत्पन्न मतभेद सम्बन्धी संशोधन (Amendment Regarding Controversy Arise between Judiciary and Parliament)-न्यायपालिका एवं विधायिका (संसद) के बीच उत्पन्न मतभेद सम्बन्धी संशोधन इस प्रकार हैं-

1. मौलिक अधिकारों सम्बन्धी विवाद-बुनियादी या मूलभूत ढाँचा एक सैद्धान्तिक बात है जो स्वयं में एक जीवंत संविधान का उदाहरण है। संविधान में इस अवधारणा का कोई उल्लेख नहीं मिलता। वास्तव में यह एक ऐसा विचार है जो न्यायिक व्याख्याओं हुआ है। मौलिक अधिकारों से संबंधित केशवानंद भारती के विवाद में सर्वोच्च न्यायालय ने इस सिद्धांत (मूल ढाँचे का सिद्धांत) का प्रतिपादन किया था। इस निर्णय ने संविधान के विकास में निम्नलिखित सहयोग दिया

(1) इस निर्णय द्वारा संविधान में संशोधन करने की संसद की शक्तियों की सीमाएँ निर्धारित की गईं।
संविधान : एक जीवंत दस्तावेज़ के रूप में

(2) यह निर्णय संविधान के किसी भी या सम्पूर्ण भागों के किसी भी प्रकार के संशोधन की स्वीकृति देता है। यहाँ तक कि मूल अधिकारों में भी संशोधन की पूर्ण स्वीकृति दी गई है। अतः संविधान के मूल ढाँचे में संशोधन किए बिना संविधान के किसी भी भाग में संसद द्वारा संशोधन या परिवर्तन किया जा सकता है।

(3) इस निर्णय से यह भी स्पष्ट है कि संविधान के मूल ढाँचे में उल्लंघन करने वाले संशोधन के सम्बन्ध में अन्तिम निर्णय करने का अधिकार न्यायपालिका का होगा। यद्यपि मूल ढाँचा क्या है? यह सर्वोच्च न्यायालय द्वारा अभी स्पष्ट नहीं किया गया है, लेकिन समय-समय पर विभिन्न विवादों में दिए गए निर्णयों से संविधान के मूलभूत ढाँचे के सम्बन्ध में निम्नलिखित कुछ बिंदु उभरकर सामने आए हैं

  • संविधान की सर्वोच्चता,
  • संविधान का धर्म-निरपेक्ष स्वरूप,
  • शासन का लोकतंत्रात्मक एवं गणतन्त्रीय रूप,
  • कानून का शासन एवं न्यायिक समीक्षा,
  • संविधान का संघीय स्वरूप,
  • स्वतंत्र एवं निष्पक्ष न्यायपालिका,
  •  कार्यपालिका, व्यवस्थापिका एवं न्यायपालिका के मध्य स्थापित समीकरण,
  • भारत की प्रभुसत्ता,
  • देश की अखण्डता आदि।

इस प्रकार मूल संरचना या ढाँचे का सिद्धांत संविधान की कठोरता एवं लचीलेपन की मिश्रित व्यवस्था को इंगित करता है। संविधान के मूल ढाँचे को संविधान संशोधन प्रक्रिया के दायरे से बाहर रखने की व्यवस्था संविधान की कठोरता को व्यक्त करता है। दूसरे शब्दों में, संविधान का मूल ढाँचे से संबंधित भाग अपरिवर्तनीय रहेगा जिसे कभी भी नहीं बदला जा सकता। जबकि दूसरी तरफ संविधान के कुछ भागों को संशोधन प्रक्रिया के अधीन करने की व्यवस्था लचीलेपन को व्यक्त करती है। इस तरह हमारा संविधान अंशतः लचीले एवं अंशतः कठोर स्वरूप वाला कहा जाता है।

2. 86वाँ संशोधन, 2002-इस संशोधन के द्वारा शिक्षा को मौलिक अधिकार घोषित किया गया है। इस संशोधन के द्वारा संविधान के अनुच्छेद 21 के तुरन्त बाद खण्ड 21क जोड़ा गया। इसमें यह व्यवस्था की गई कि 6 से 14 वर्ष तक की आयु के सभी बच्चों के लिए विधि द्वारा निर्धारित प्रक्रिया के अनुसार राज्य निःशुल्क एवं अनिवार्य शिक्षा प्रदान करेगा। इसी संशोधन में अनुच्छेद 51क में खण्ड (1) जोड़कर यह भी व्यवस्था की गई कि अभिभावकों का यह कर्त्तव्य है कि वे 6 से 14 वर्ष के अपने बच्चों को शिक्षा का अवसर प्रदान करें।

3. 99वाँ संशोधन-99वें संवैधानिक संशोधन के द्वारा सर्वोच्च न्यायालय एवं उच्च न्यायालयों में न्यायाधीशों की नियुक्ति के लिए चली आ रही कॉलेजियम प्रणाली के स्थान पर राष्ट्रीय न्यायिक नियक्ति आयोग का गठन किया गया। जिसे लोकसभा एवं राज्यसभा द्वारा पारित करने एवं 30 दिसम्बर, 2014 को राष्ट्रपति द्वारा इसे स्वीकृति देने के बाद केन्द्र सरकार द्वारा 13 अप्रैल, 2015 को अधिसूचित किया गया।

परन्तु इस संशोधन विधेयक को दी गई चुनौतियों के आधार पर सर्वोच्च न्यायालय के द्वारा 16 अक्टूबर, 2015 को इस सवैधानिक संशोधन को असंवैधानिक घोषित कर दिया गया। इस प्रकार न्यायाधीशों की नियुक्ति सम्बन्धी गठित राष्ट्रीय न्यायिक आयोग के स्थान पर पूर्व प्रचलित कॉलेजियम प्रणाली को ही कायम रखा गया है।

प्रश्न 3.
भारतीय संविधान में किए गए कुछ विवादस्पद संशोधनों (Controversial Amendments) का वर्णन करें।
उत्तर:
यदि हम संवैधानिक संशोधन के दृष्टिकोण से सत्तर से अस्सी के दशक में हुए संशोधनों पर दृष्टिपात करें तो स्पष्ट हो जाएगा कि इस दौरान हुए संशोधनों को लेकर विधि और राजनीति के दायरों में भारी बहस छिड़ी थी। वर्ष 1971 से 1976 के इन्दिरा गांधी के नेतृत्व में हुए विशेषकर 38वें, 39वें एवं 49वें संशोधनों की विधि-विशेषज्ञों के साथ-साथ विपक्षी दलों ने भी भारी आलोचना की।

कई विद्वानों ने तो ऐसे संशोधनों को ‘अवसरवादी अधिनियम’ (Acts of Opportunism) भी कहकर पुकारा। विपक्षी दलों का तो यहाँ तक मानना था कि ऐसे संशोधनों के माध्यम से सत्तारूढ़ दल संविधान के मूल स्वरूप को ही बिगाड़ना चाहते हैं। इन्दिरा गाँधी के नेतृत्व में हुए ऐसे संशोधनों का संक्षिप्त वर्णन निम्नलिखित प्रकार से है

(1) 38वां संवैधानिक संशोधन, 1975 जुलाई, 1975 में पारित इस संशोधन ने राष्ट्रपति, राज्यपाल तथा उप-राज्यपाल द्वारा जारी किए गए अध्यादेशों और राष्ट्रपति की आपातकालीन शक्तियों को न्यायपालिका के अधिकार-क्षेत्र से बाहर कर दिया।

(2) 39वां संवैधानिक संशोधन, 1975 अगस्त, 1975 में आपातकाल के दौरान पास हुए इस अधिनियम ने राष्ट्रपति, उप-राष्ट्रपति, लोकसभा अध्यक्ष और प्रधानमन्त्री के चुनाव को न्यायालय के क्षेत्र से बाहर करके उसके लिए संसद की अलग से एक समिति गठित करने की व्यवस्था की। इसने 1951 के जन-प्रतिनिधित्व कानून में 1974 और 1975 में किए गए संशोधनों, आन्तरिक सुरक्षा कानून (MISA) आदि 37 अन्य कानूनों को भी नौवीं अनुसूची में शामिल करके उन्हें न्यायपालिका के क्षेत्राधिकार से बाहर कर दिया।

(3) 40वां संवैधानिक संशोधन, 1975-संविधान का यह संशोधन यह व्यवस्था करता है कि राष्ट्रपति, प्रधानमन्त्री और राज्यों के राज्यपालों के विरुद्ध, उनके कार्यकाल में या अवकाश ग्रहण करने पर, उनके शासन-सम्बन्धी विषयों के बारे में का कोई फौजदारी या दीवानी मुकद्दमा नहीं चलाया जा सकेगा। वास्तव में ये सभी संशोधन आपातकाल के दौरान विशेषकर जून, 1975 के पश्चात् किए गए। इनमें सबसे अधिक विवादित संशोधन 42वां रहा है, जिसका उल्लेख निम्नलिखित है

(4) 42वां संशोधन, 1976-59 अनुभागों (Sections) वाला यह ‘लघु संविधान’ (Mini Constitution) सन् 1976 में आपातकाल के दौरान पास किया गया। इसने

  • राष्ट्रपति के लिए मन्त्रि-परिषद् की सलाह मानना आवश्यक बनाने,
  • लोकसभा और विधानसभाओं का कार्यकाल 6 वर्ष करने,
  • संसद की सर्वोच्चता कायम करने,
  • उद्देशिका या प्रस्तावना में समाजवादी, धर्म-निरपेक्षता (Socialist, Secular) शब्द बढ़ाने,
  • केन्द्र-राज्य संबंध में परिवर्तन करने,
  • मौलिक अधिकारों के ऊपर राज्य-नीति के निदेशक सिद्धान्तों को प्रमुखता देने,
  • न्यायपालिका की न्यायिक पुनर्निरीक्षण की शक्ति को निष्प्रभावी बनाने और
  • राज्यों में सशस्त्र सेनाएँ भेजने तथा देश के किसी एक भाग में आपात स्थिति लागू करने आदि की व्यवस्था की गई।

भूतपूर्व महान्यायवादी सी० के० दफ्तरी और एन० ए० पालकीवाला जैसे कानून शास्त्रियों ने इस संवैधानिक संशोधन कानून को प्रधानमन्त्री पद में पूरी राज्य सत्ता को निहित करना बताया था। इसके अतिरिक्त आलोचकों ने इन संशोधनों को न्यायपालिका की शक्ति को कम करने तथा लोकतन्त्र एवं कानून के शासन को नष्ट करने का एक प्रयास भी बताया।

इस प्रकार इन्दिरा गांधी की तानाशाही एवं स्वेच्छाचारी व्यवहार के अनुरूप किए गए संशोधनों ने राजनीतिक एवं वैधानिक क्षेत्र में नए विवादों को जन्म दिया। परन्तु इन्दिरा गांधी के तानाशाही शासन की समाप्ति के उपरांत सन् 1975 में हुए लोकसभा चुनावों में नवीन सत्तारूढ़ दल ने ऐसे विवादित संशोधनों की व्यवस्थाओं में पुनः संशोधन किया जो विशेषतः 43वें एवं 44वें संशोधन के द्वारा हुआ। इनका संक्षिप्त वर्णन निम्नलिखित है

(5) 43वां संशोधन, 1978-जनता सरकार ने अपनी चुनावी घोषणा के अनुसार 43वां संशोधन बिल पेश किया, परन्तु राज्यसभा के विरोध के कारण उसे वापिस लेना पड़ा। इसलिए 44वां संशोधन बिल पेश किया गया जिसे कि 13 अप्रैल, 1978 को राष्ट्रपति की स्वीकृति मिल गई और जो संविधान का 43वां संशोधन बन गया। इस संशोधन में 10 अनुच्छेद हैं और इसमें कुछ व्यवस्थाएँ निम्नलिखित हैं

(1) 42वें संशोधन द्वारा निर्मित 31D अनुच्छेद को समाप्त कर दिया, जिसके अनुसार संसद राष्ट्र-विरोधी गतिविधियों के लिए कानून बना सकती है,
(2) सर्वोच्च न्यायालय के लिए संसद के कानूनों की वैधानिकता को देखने के लिए कम-से-कम 7 न्यायाधीशों की पीठ की व्यवस्था 42वें संशोधन द्वारा की गई थी, वह समाप्त कर दी गई है,
(3) नए संशोधन बिल के अनुसार राज्य के उच्च न्यायालयों को देखने का अधिकार है,
(4) उच्च न्यायालयों के लिए कानूनों की वैधानिकता देखने के लिए जो 5 न्यायाधीशों की पीठ की व्यवस्था थी, वह समाप्त कर दी गई है।

(vi) 44वां संशोधन, 1979-जनता पार्टी की सरकार ने, जिसने कि मार्च, 1977 के लोकसभा चुनाव में सत्ता प्राप्त की थी, अपने चुनाव घोषणा-पत्र में कांग्रेस सरकार द्वारा पारित 42वें संशोधन को समाप्त करने का जनता को आश्वासन दिया था। उसी के अनुसार 45वां संशोधन बिल मई, 1978 में लोकसभा में पेश किया गया था, जिसमें 45 अनुच्छेद थे।

चूंकि राज्यसभा में विरोधी दलों का बहुमत था, इसलिए उसने इस संशोधन की कुछ धाराओं को अस्वीकृत करके शेष बिल पास कर दिया गया, जो दोबारा लोकसभा द्वारा पास किया गया। तत्पश्चात् संविधान की व्यवस्था के अनुसार इस बिल को 12 राज्यों की विधानसभाओं ने स्वीकृति दे दी। 30 अप्रैल, 1979 को राष्ट्रपति की स्वीकृति प्राप्त करने के बाद यह 45वां संशोधन विधेयक संविधान का 44वां संशोधन बन गया। इस संशोधन के अन्तर्गत

(1) सम्पत्ति के अधिकार को कानूनी अधिकार बना दिया गया है। किसी व्यक्ति को उसकी सम्पत्ति से कानून के अनुसार ही वंचित किया जा सकता है।

(2) अनुच्छेद 352 के अन्तर्गत राष्ट्रपति संकटकाल की घोषणा तभी कर सकता है, यदि मन्त्रिमण्डल राष्ट्रपति को संकटकाल की घोषणा करने की लिखित सलाह दे। संविधान के अनुच्छेद 352 से आन्तरिक अशान्ति शब्द की जगह सशस्त्र विद्रोह उपबन्ध जोड़ा गया। इस प्रकार अब संकटकाल की घोषणा तभी की जा सकती है, यदि भारत को अथवा भारत के किसी भाग की सुरक्षा को युद्ध अथवा बाहरी आक्रमण अथवा सशस्त्र विद्रोह (Armed Rebellion) से खतरा हो।

(3) जीवन और स्वतन्त्रता के अधिकार को सुरक्षित बनाने के लिए यह व्यवस्था की गई कि संकटकाल में भी इन अधिकारों के संबंध में न्यायालय में अपील करने के अधिकार को स्थगित नहीं किया जा सकता।

(4) राष्ट्रपति और उप-राष्ट्रपति के चुनाव से संबंधित सभी सन्देह और विवादों की जांच तथा निर्णय सर्वोच्च न्यायालय द्वारा किए जाएंगे और सर्वोच्च न्यायालय का निर्णय अन्तिम होगा।

(5) लोकसभा और राज्य विधानसभाओं की अवधि 6 वर्ष से घटाकर 5 वर्ष किए जाने की व्यवस्था की गई।

HBSE 11th Class Political Science Important Questions Chapter 9 संविधान : एक जीवंत दस्तावेज़

प्रश्न 4.
भारत में लोकतान्त्रिक परम्पराओं को मजबूत बनाने वाले तत्त्वों का वर्णन कीजिए।
उत्तर:
भारत विश्व का सबसे बड़ा लोकतन्त्रीय राज्य है जहाँ लोकतन्त्रीय शासन पद्धति अनेक वर्षों से सफलतापूर्वक कार्य कर रही है। यद्यपि भारत की इस व्यवस्था में ऐसे भी तत्त्व हैं, जो लोकतन्त्र के विकास में बाधक हैं, परन्तु भारत में संवैधानिक और गैर-सवैधानिक ऐसी लोकतन्त्रीय परम्पराएँ विकसित हो रही हैं जो लोकतन्त्र को दृढ़ता प्रदान करती हैं।

दूसरे शब्दों में, भारत में इस प्रकार के तत्त्व पाए जाते हैं जो लोकतन्त्र को सफल बनाने में सहायक होते हैं। गत वर्षों के अध्ययन से यह स्पष्ट हो जाता है कि भारत में लोकतन्त्र इतना मजबूत है कि वह सभी प्रकार की कठिनाइयों-राजनीतिक, अन्तर्राष्ट्रीय, आर्थिक, सामाजिक, साम्प्रदायिक, क्षेत्रीय तथा आतंकवाद का मुकाबला सफलतापूर्वक कर सकता है। भारतीय लोकतन्त्र को जिन लोकतन्त्रीय परम्पराओं ने शक्तिशाली बनाने में योगदान दिया है, उनका वर्णन निम्नलिखित भागों में किया जा सकता है

(क) संवैधानिक व्यवस्थाएँ (Constitutional System):
संविधान निर्माताओं ने एक पूर्णरूपेण लोकतन्त्रीय संविधान का निर्माण किया। इसके द्वारा जो व्यवस्थाएँ निश्चित की गईं, उनका उल्लेख निम्नलिखित भागों में स्पष्ट किया जा सकता है

1. प्रस्तावना (Preamble):
संविधान की प्रस्तावना द्वारा संविधान के आदर्शों व उद्देश्यों का स्पष्टीकरण होता है। इसके अनुसार भारत एक लोक-कल्याणकारी राज्य की स्थापना के लिए दृढ़-संकल्प है। इसके साथ-साथ संविधान सभी नागरिकों को राजनीतिक, आर्थिक व सामाजिक न्याय प्रदान करने की घोषणा करता है।

2. मौलिक अधिकार तथा राज्य के नीति निदेशक तत्त्व (Fundamental Rights and Directive Principles of State Policy):
नागरिकों के विकास के लिए जहां मौलिक अधिकार प्रदान किए गए हैं, वहां उनकी सुरक्षा के लिए निष्पक्ष तथा स्वतन्त्र न्यायपालिका की स्थापना की गई है। इस व्यवस्था से जहां नागरिकों की व्यक्तिगत स्वतन्त्रता की रक्षा होती है वहाँ सरकार पर भी रोक लगती है। आर्थिक और सामाजिक विकास के लिए राज्य के नीति निदेशक तत्त्वों द्वारा व्यवस्था की गई है। इस क्षेत्र में कुछ कार्य तो हुआ है, परन्तु अभी और कार्य करना शेष है।’

3. उत्तरदायी शासन (Responsible Government):
लोकतन्त्र शासन में सरकार का जनता के प्रति उत्तरदायी होना बहुत आवश्यक होता है। भारत में संघ तथा राज्यों में संसदीय प्रणाली अपनाई गई है। मन्त्रिपरिषद् अपने शासन-सम्बन्धी कार्यों के लिए जनता के चुने हुए प्रतिनिधियों के प्रति उत्तरदायी होती है।

4. वयस्क मताधिकार के आधार पर चुनाव (Elections based on Adult Franchise):
मौलिक प्रभुसत्ता के सिद्धांत के अनुसार 18 वर्ष से अधिक आयु के प्रत्येक नागरिक को मत देने का अधिकार प्राप्त है। एक निश्चित आयु प्राप्त करने पर प्रत्येक नागरिक चुनाव में भी भाग ले सकता है। इस प्रकार सभी नागरिकों को राजनीतिक समानता प्रदान की गई है जो लोकतन्त्र का एक प्रमुख तत्त्व है।

5. निष्पक्ष चुनाव के लिए निष्पक्ष चुनाव आयोग की व्यवस्था (Provision of Impartial Election Commission):
भारत में सन् 1952 से लेकर अब तक लोकसभा के 16 चुनाव और राज्यों की विधानसभाओं के अनेक चुनाव सफलतापूर्वक हो चुके हैं।

6. कानून का शासन (Rule of Law):
संविधान द्वारा सभी नागरिकों को कानूनी समानता प्रदान की गई है। कानून के समक्ष सभी नागरिक समान हैं। कानून व्यक्ति-व्यक्ति में भेदभाव नहीं करता। इस प्रकार संविधान द्वारा ऐसी व्यवस्थाएँ की गई हैं जो लोकतन्त्रीय परम्पराओं को शक्तिशाली बनाती हैं। इसके अलावा गैर-संवैधानिक संस्थाओं ने भी लोकतन्त्रीय परम्पराओं के विकास में सहायता दी है।

(ख) गैर-सवैधानिक व्यवस्थाएँ (Extra-constitutional Provisions):
ये वे व्यवस्थाएँ एवं परम्पराएँ हैं जिनका प्रत्यक्ष रूप से संविधान में उल्लेख नहीं है। ये इस प्रकार हैं

1. लोकतांत्रिक विरासत (Democratic Legacy):
भारतीय लोकतन्त्र की अब तक सफलता के मूल में भारत की जनता का लोकतन्त्र में अटूट विश्वास निहित है। अत्यन्त प्राचीन काल से ही हमने लोकतांत्रिक शासन व्यवस्था के विभिन्न रूपों का सफल प्रयोग किया है। सर्वसत्ता-सम्पन्न, केन्द्रीयकृत तथा अधिनायकवादी शासनों का जनता ने सदा ही विरोध किया है। इसी लोकतांत्रिक विरासत (Legacy) के प्रभाव के कारण ही अनेक विपरीत परिस्थितियों तथा समस्याओं के होते हुए लोकतन्त्र का सफलतापूर्वक संचालन कर रहे हैं।

2. महान् नेताओं का नेतृत्व (Leadership of Great Leaders):
स्वतन्त्रता-प्राप्ति के समय भारत को डॉ० राजेन्द्र प्रसाद, जवाहरलाल नेहरू, सरदार पटेल आदि ऐसे महान् नेताओं का नेतृत्व प्राप्त हुआ जो लोकतांत्रिक मूल्यों में आस्था रखते थे और उन्होंने लोगों में इन मूल्यों के प्रति आस्था पैदा की।

3. भारतीय जनता की जागरूकता (Awakening of Indian People):
भारतीय जनता ने सन् 1952 से 2014 तक हुए सभी चुनावों में पूर्ण जागरूकता का प्रदर्शन किया है। सन् 1975 की संकटकालीन घोषणा के विरोध में जनता ने जो मतदान किया, उससे यह स्पष्ट हो गया था कि वे लोकतन्त्र विरोधी कार्यों को सहन नहीं कर सकेंगे।

4. आर्थिक योजनाएँ (Economic Plans):
स्वतन्त्रता-प्राप्ति के पश्चात् भारत सरकार द्वारा देश के सामाजिक विकास के लिए जो कदम उठाए गए हैं, वे भी लोकतन्त्र में लोगों के विश्वास के लिए उत्तरदायी हैं। देश के आर्थिक विकास के लिए दस पंचवर्षीय योजनाएँ पूरी हो चुकी हैं और 12वीं पंचवर्षीय योजना भी अप्रैल, 2012 में प्रारम्भ हुई है। संघ तथा राज्यों की सरकारें इस बारे में प्रयत्नशील हैं कि सभी लोगों को रोजगार मिले, कृषि-उत्पादन बढ़े, औद्योगिक विकास हो और सामाजिक तथा आर्थिक असमानता दूर हो।

5. सवैधानिक उपायों में विश्वास (Faith in Constitutional Means):
भारत की जनता की सांस्कतिक धरोहर ने संवैधानिक उपायों का उपयोग करने के लिए निरन्तर प्रेरित किया है। छोटी समस्याओं से लेकर शासन के प्रति आक्रोश तक को यहां सवैधानिक उपायों से ही सुलझाने को समाज की स्वीकृति प्राप्त है।

असवैधानिक व हिंसक उपायों का उपयोग विदेशी विचारधाराओं से प्रेरित लोग करते हैं। आवश्यकता इस बात की है कि भारत के मूल चिन्तन को बढ़ावा दिया जाए तथा लोकतन्त्र के इस मौलिक सिद्धांत में आस्था को स्थायी बनाया जाए। शान्तिपूर्ण उपायों से हुआ परिवर्तन व विकास स्थायी व कल्याणकारी होता है।

6. विरोधी दलों का योगदान (Contribution byOpposition Parties):
यद्यपि भारत में विरोधी दल बहुत शक्तिशाली नहीं रहे हैं, परन्तु उन्होंने लोकतांत्रिक परंपराओं को बनाए रखने में योगदान अवश्य दिया है। जहां तक भी सम्भव हो सका है उन्होंने सत्ताधारी दल की अनुचित नीतियों का डटकर मुकाबला किया है। दिए गए विवरण से यह स्पष्ट है कि भारत में लोकतांत्रिक परंपराओं को शक्तिशाली बनाने वाले अनेक तत्त्व मौजूद हैं और कसित हो रहे हैं, जिससे यह आशा की जा सकती है कि भारत में लोकतन्त्र पहले से और अधिक मजबूत होगा और लोकतन्त्र के उद्देश्यों एवं लोकतांत्रिक राजनीति के अन्तिम लक्ष्य समाज एवं जनता की समृद्धि एवं जन-कल्याण के व्यापक स्वरूप को व्यावहारिकता दे पाएँगे।

वस्तुनिष्ठ प्रश्न

निम्नलिखित प्रश्नों का उत्तर दिए गए विकल्पों में से उचित विकल्प छाँटकर लिखें

1. भारतीय संविधान में संशोधन की प्रक्रिया का वर्णन संविधान के निम्नलिखित अनुच्छेद में किया गया है
(A) अनुच्छेद 372
(B) अनुच्छेद 368
(C) अनुच्छेद 374
(D) अनुच्छेद 376
उत्तर:
(B) अनुच्छेद 368

2. “यह संविधान चरम कठोरता एवं अत्यंत लचीलेपन में एक अच्छा संतुलन स्थापित करता है।” ये शब्द किस विद्वान के हैं?
(A) एम०पी० शर्मा
(B) डी०डी० बसु
(C) प्रो० के०सी० ह्वीयर
(D) वी०एन० शुक्ला
उत्तर:
(C) प्रो. के०सी० बीयर

3. भारतीय संविधान में संशोधन का प्रस्ताव आरंभ किया जा सकता है
(A) केवल संसद में
(B) जनता द्वारा
(C) केवल आधे राज्यों द्वारा
(D) संसद तथा आधे राज्यों द्वारा
उत्तर:
(A) केवल संसद में

4. साधारण बहुमत से निम्नलिखित अनुच्छेद में संवैधानिक संशोधन किया जा सकता है
(A) अनुच्छेद 3 में
(B) अनुच्छेद 343 में
(C) अनुच्छेद 106 में
(D) उपर्युक्त सभी
उत्तर:
(D) उपर्युक्त सभी

5. लोकसभा के कुल सदस्यों की संख्या 545 निम्नलिखित संशोधन द्वारा निश्चित की गई
(A) 17वें संशोधन द्वारा
(B) 25वें संशोधन द्वारा
(C) 31वें संशोधन द्वारा
(D) 42वें संशोधन द्वारा
उत्तर:
(D) 42वें संशोधन द्वारा

6. निम्नलिखित सवैधानिक संशोधन ने संविधान की प्रस्तावना में समाजवादी तथा धर्म-निरपेक्ष शब्द जोड़े हैं
(A) 41वें संशोधन द्वारा
(B) 42वें संशोधन द्वारा
(C) 43वें संशोधन द्वारा
(D) 44वें संशोधन द्वारा
उत्तर:
(B) 42वें संशोधन द्वारा

7. भारतीय संविधान में अब तक कुल संशोधन हो चुके हैं
(A) 103
(B) 102
(C) 104
(D) 105
उत्तर:
(C) 104

8. भारतीय संविधान में दो-तिहाई बहुमत द्वारा संशोधन किस भाग में किया जा सकता है?
(A) भाग तीन
(B) भाग पाँच
(C) भाग दो
(D) भाग सात
उत्तर:
(A) भाग तीन

HBSE 11th Class Political Science Important Questions Chapter 9 संविधान : एक जीवंत दस्तावेज़

9. निम्नलिखित संशोधन ने मताधिकार की आयु 18 वर्ष की है
(A) 54वाँ संशोधन
(B) 55वाँ संशोधन
(C) 63वाँ संशोधन
(D) 64वाँ संशोधन
उत्तर:
(B) 55वाँ संशोधन

10. निम्नलिखित संशोधन द्वारा अनुसूचित जातियों तथा अनुसूचित जनजातियों के लिए आरक्षण की अवधि 2020 तक कर दी है
(A) 54वाँ संशोधन
(B) 55वाँ संशोधन
(C) 95वाँ संशोधन
(D) 78वाँ संशोधन
उत्तर:
(C) 95वाँ संशोधन

11. भारतीय संविधान में संशोधन करने का अधिकार
(A) राष्ट्रपति के पास है
(B) प्रधानमंत्री के पास है
(C) केवल संसद के पास है
(D) संसद तथा राज्यों के विधानमंडल, दोनों इसमें भाग लेते हैं
उत्तर:
(C) केवल संसद के पास है

12. भारतीय संघ में नए राज्यों को शामिल करने से संबंधित संशोधन
(A) संसद अपने दोनों सदनों के साधारण बहुमत से कर सकती है
(B) संसद दोनों सदनों के 2/3 बहुमत से कर सकती है
(C) राष्ट्रपति कर सकता है
(D) संसद के दोनों सदनों के 2/3 बहुमत व आधे राज्यों की स्वीकृति से किया जा सकता है
उत्तर:
(A) संसद अपने दोनों सदनों के साधारण बहुमत से कर सकती है

13. राष्ट्रपति के निर्वाचन के ढंग से संबंधित संशोधन
(A) संसद अपने साधारण बहुमत से कर सकती है
(B) संसद के दोनों सदनों के दो-तिहाई बहुमत से किया जा सकता है
(C) संसद के दोनों सदनों के 2/3 बहुमत व आधे राज्यों की स्वीकृति से किया जा सकता है
(D) इनमें से कोई नहीं
उत्तर:
(C) संसद के दोनों सदनों के 2/3 बहुमत व आधे राज्यों की स्वीकृति से किया जा सकता है

14. भारतीय लोकतंत्र को प्रभावित करने वाले सामाजिक एवं आर्थिक कारक निम्नलिखित हैं
(A) गरीबी
(B) निरक्षरता
(C) बेरोज़गारी
(D) उपर्युक्त सभी
उत्तर:

15. लोकतंत्रीय परंपराओं को शक्तिशाली बनाने वाले तत्त्व हैं
(A) प्रस्तावना
(B) लोकतांत्रिक विरासत
(C) आर्थिक योजनाएँ
(D) उपर्युक्त सभी
उत्तर:
(D) उपर्युक्त सभी

16. भारतीय संविधान की 8वीं अनुसूची में वर्तमान में कितनी भाषाओं को मान्यता दी गई है?
(A) 20 भाषाओं को
(B) 22 भाषाओं को
(C) 24 भाषाओं को
(D) इनमें से कोई नहीं
उत्तर:
(B) 22 भाषाओं को

17. संपत्ति के अधिकार का वर्तमान स्वरूप कैसा है?
(A) संवैधानिक अधिकार
(B) नैतिक अधिकार
(C) मूल अधिकार
(D) सामाजिक अधिकार
उत्तर:
(A) संवैधानिक अधिकार

निम्नलिखित प्रश्नों का उत्तर एक शब्द में दें

1. कौन-से सवैधानिक संशोधन के द्वारा अनुसूचित जातियों एवं अनुसूचित जनजातियों के लिए आरक्षण की अवधि सन् 2030 तक बढ़ाई गई?
उत्तर:
104वें सवैधानिक संशोधन के द्वारा।

2. क्या संसद मूल अधिकारों में संशोधन कर सकती है?
उत्तर:
हाँ, संसद मूल अधिकारों में संशोधन कर सकती है, परंतु संविधान के मूल ढाँचे में परिवर्तन नहीं कर सकती।

3. संपत्ति के अधिकार का वर्तमान स्वरूप संविधान में कैसा है?
उत्तर:
संपत्ति के अधिकार का वर्तमान स्वरूप संविधान में अब एक कानूनी तथा संवैधानिक अधिकार के रूप में है, मूल अधिकार के रूप में नहीं है।

4. संविधान के किस संशोधन को ‘लघु संविधान’ (Mini Constitution) कहकर पुकारा जाता है?
उत्तर:
संविधान में किए गए 42वें संशोधन को ‘लघु संविधान’ कहकर पुकारा जाता है।

5. भारतीय संविधान की 8वीं अनुसूची में वर्तमान में कितनी भाषाओं को मान्यता दी गई है?
उत्तर:
22 भाषाओं को वर्तमान में सवैधानिक मान्यता प्राप्त है।

रिक्त स्थान भरें

1. भारतीय संविधान के अनुच्छेद ………. में संशोधन विधि का उल्लेख किया गया है।
उत्तर:
368

2. भारतीय संविधान में अब तक कुल ……………. संशोधन हो चुके हैं।
उत्तर:
104

3. दिसम्बर, 2019 में भारतीय संविधान में ………….. संवैधानिक संशोधन हुआ।
उत्तर:
एस.सी. एवं एस.टी आरक्षण सम्बन्धी

HBSE 11th Class Political Science Important Questions Chapter 9 संविधान : एक जीवंत दस्तावेज़

4. भारतीय संविधान अंशतः …………… और अंशतः ……………. है।
उत्तर:
लचीली एवं कठोर

5. भारतीय संविधान में सम्पत्ति का अधिकार एक …………. है।
उत्तर:
साधारण कानूनी अधिकार

6. ……………. मूल अधिकारों में संशोधन कर सकती है।
उत्तर:
संसद

7. भारतीय संविधान में मतदान आयु 21 से घटाकर 18 वर्ष ………… संवैधानिक संशोधन द्वारा की गई है।
उत्तर:
61वें

HBSE 11th Class Political Science Important Questions Chapter 8 स्थानीय शासन

Haryana State Board HBSE 11th Class Political Science Important Questions Chapter 9 संविधान : एक जीवंत दस्तावेज़ Important Questions and Answers.

Haryana Board 11th Class Political Science Important Questions Chapter 9 संविधान : एक जीवंत दस्तावेज़

अति लघूत्तरात्मक प्रश्न

प्रश्न 1.
भारत में वर्तमान पंचायती राज व्यवस्था का ढाँचा कितने स्तरीय है? उनके नाम बताएँ।
उत्तर:
भारत में वर्तमान पंचायती राज व्यवस्था का ढाँचा तीन स्तरीय है

  • ग्राम स्तर पर ग्राम पंचायत,
  • खण्ड (ब्लॉक) अथवा तहसील स्तर पर पंचायत समिति,
  • जिला स्तर पर जिला-परिषद् ।

प्रश्न 2.
ग्राम सभा किसे कहते हैं?
उत्तर:
एक ग्राम पंचायत के क्षेत्र में रहने वाले सभी व्यक्ति, जो 18 वर्ष की आयु पूरी कर चुके हैं और जिनका नाम मतदाताओं की सूची में होता है, मिलकर ग्राम सभा का निर्माण करते हैं। दूसरे शब्दों में, ग्राम पंचायत के क्षेत्र में रहने वाले सभी मतदाता ग्राम सभा के सदस्य होते हैं।

HBSE 11th Class Political Science Important Questions Chapter 8 स्थानीय शासन

प्रश्न 3.
ग्राम पंचायत के सदस्यों की संख्या कितनी होती है?
उत्तर:
एक ग्राम पंचायत के सदस्यों की संख्या गाँव की जनसंख्या के आधार पर राज्य सरकार द्वारा निश्चित की जाती है। पंजाब में यह संख्या 5 से 13 के बीच तथा हरियाणा में 5 से 9 तक हो सकती है।

प्रश्न 4.
ग्राम पंचायत के अध्यक्ष को क्या कहा जाता है? उसका चुनाव कैसे किया जाता है?
उत्तर:
ग्राम पंचायत के अध्यक्ष को सरपंच कहा जाता है। उसका चुनाव ग्राम सभा के सदस्यों द्वारा प्रत्यक्ष चुनाव-प्रणाली द्वारा किया जाता है।

प्रश्न 5.
ग्राम-पंचायत के सदस्यों का चुनाव कैसे किया जाता है?
उत्तर:
ग्राम पंचायत के सदस्यों का चुनाव ग्राम सभा के सदस्यों द्वारा प्रत्यक्ष चुनाव-प्रणाली के आधार पर किया जाता है।

प्रश्न 6.
ग्राम पंचायत का कार्यकाल कितना होता है?
उत्तर:
73वें संवैधानिक संशोधन के अनुसार ग्राम पंचायत का कार्यकाल 5 वर्ष निश्चित किया गया है। सरकार द्वारा उसे पहले भी भंग किया जा सकता है।

प्रश्न 7.
ग्राम पंचायत के कोई तीन कार्य लिखिए।
उत्तर:

  • अपने क्षेत्र के विकास के लिए बजट तैयार करना तथा उसे ग्राम सभा के सामने पेश करना,
  • गाँव की गलियाँ बनवाना तथा उनकी मुरम्मत करवाना,
  • पीने के लिए शुद्ध पानी का प्रबन्ध करना।

प्रश्न 8.
ग्राम पंचायत की आय के कोई तीन साधन बताएँ।
उत्तर:

  • गृह-कर,
  • सरकार तथा अन्य संस्थाओं द्वारा प्राप्त अनुदान,
  • पंचायत द्वारा किए गए जुर्माने से आय।

प्रश्न 9.
पंचायत समिति के कोई दो कार्य लिखें।
उत्तर:

  • सामुदायिक विकास पंचायत समिति का मुख्य कार्य है। इसके अतिरिक्त अन्य कार्य हैं-क्षेत्र में विकास योजनाएँ लागू करना, उत्पादन में वृद्धि के लिए प्रयास करना तथा लोगों के लिए रोज़गार आदि पैदा करना,
  • कुटीर उद्योगों, ग्रामीण कला तथा कारीगरी के विकास के लिए प्रशिक्षण केन्द्रों की स्थापना करना।

प्रश्न 10.
पंचायत समिति की आय के कोई साधन बताएँ।
उत्तर:

  • मेलों तथा मण्डियों से प्राप्त आय,
  • राज्य सरकार द्वारा वित्तीय सहायता अथवा अनुदान।

प्रश्न 11.
ज़िला-परिषद् के कोई दो कार्य बताएँ।
उत्तर:

  • अपने क्षेत्र में स्थापित पंचायत समितियों के कार्यों में समन्वय स्थापित करना,
  • पंचायत समितियों के बजट का निरीक्षण करना तथा उसे अपनी स्वीकृति प्रदान करना,
  • जिले में ग्रामीण विकास के सम्बन्ध में सरकार को सुझाव देना।

प्रश्न 12.
जिला-परिषद् की आय के कोई दो साधन बताएँ।
उत्तर:

  • राज्य सरकार द्वारा सहायता के रूप में दिया गया धन,
  • स्थानीय करों (Local Taxes) का कुछ भाग ज़िला-परिषद् को मिलता है।

प्रश्न 13.
74वाँ संवैधानिक संशोधन कब तथा किस लिए पास किया गया?
उत्तर:
74वाँ संवैधानिक संशोधन देश में शहरी स्थानीय संस्थाओं को अधिक मजबूत बनाने और उनमें लोगों की भागीदारी को बढ़ाने के लिए भारतीय संसद द्वारा सन 1992 में पास किया गया, जो सन 1993 में लागू हआ।

प्रश्न 14.
74वें संवैधानिक संशोधन की कोई दो विशेषताएँ लिखें।
उत्तर:

  • इस संवैधानिक संशोधन द्वारा स्थानीय नगर संस्थाओं को पहली बार संवैधानिक मान्यता प्रदान की गई,
  • इस संवैधानिक संशोधन के अनुसार तीन स्तरीय शहरी संस्थाओं की स्थापना की व्यवस्था की गई
    (a) परिवर्तनीय क्षेत्र के लिए नगर पंचायत,
    (b) छोटे नगरों के लिए नगर-परिषद् तथा
    (c) बड़े नगरों के लिए नगर-निगम।

प्रश्न 15.
नगर-निगम किसे कहते हैं? हरियाणा में कुल कितने नगर निगम हैं?
उत्तर:
नगर-निगम शहरी स्थानीय प्रशासन की सर्वोच्च संस्था है, जिसकी स्थापना बड़े-बड़े नगरों में की जाती है। हरियाणा में कुल 10 नगर-निगम हैं।

प्रश्न 16.
नगर-निगम में कितने सदस्य होते हैं?
उत्तर:
प्रत्येक नगर-निगम के सदस्यों की संख्या उस नगर की जनसंख्या के आधार पर राज्य सरकार द्वारा विशेष अधिनियम के अन्तर्गत निश्चित की जाती है।

प्रश्न 17.
नगर-निगम के सदस्यों का चुनाव कैसे किया जाता है?
उत्तर:
नगर-निगम के सदस्यों अर्थात् सभासदों (Councillors) का चुनाव नगर के मतदाताओं द्वारा प्रत्यक्ष चुनाव प्रणाली के अनुसार किया जाता है। सभासद कुछ अन्य सदस्यों का चुनाव करते हैं, जिन्हें एल्डरमैन (Aldermen) कहा जाता है, ये व्यक्ति सामाजिक, राजनीतिक अथवा सांस्कृतिक क्षेत्र में प्रसिद्धि प्राप्त व्यक्ति होते हैं जो साधारण चुनाव के झंझट में नहीं पड़ना चाहते।

प्रश्न 18.
नगर-निगम का कार्यकाल कितना होता है?
उत्तर:
गम का साधारण कार्यकाल 5 वर्ष निश्चित किया गया है। राज्य सरकार द्वारा इसे निश्चित कार्यकाल समाप्त होने से पहले भी भंग किया जा सकता है।

प्रश्न 19.
नगर-निगम के अध्यक्ष को क्या कहते हैं? उसका चुनाव कैसे किया जाता है?
उत्तर:
नगर-निगम के अध्यक्ष को महापौर (Mayor) कहा जाता है। उसका चुनाव नगर-निगम के सदस्यों द्वारा किया जाता है।

प्रश्न 20.
महापौर (Mayor) के कोई दो कार्य लिखें।।
उत्तर:

  • वह नगर-निगम की बैठकों की अध्यक्षता करता है,
  • महापौर नगर का प्रथम नागरिक कहलाता है। वह नगर में आने वाले विशिष्ट अतिथियों का अभिनन्दन करते है।

प्रश्न 21.
नगर-निगम के कोई चार कार्य लिखें।
उत्तर:

  • अपने क्षेत्र में सफाई का प्रबन्ध करना,
  • पीने के लिए पानी तथा बिजली की व्यवस्था करना,
  • गली-सड़ी तथा मिलावट वाली वस्तुओं की बिक्री पर रोक लगाना,
  • श्मशान घाट तथा कब्रिस्तान की व्यवस्था करना।

प्रश्न 22.
नगर-निगम की आय के कोई दो साधन लिखें।
उत्तर:

  • गृह-कर (House Tax),
  • व्यवसाय कर (Profession Tax)

प्रश्न 23.
नगर-परिषद् (Municipal Council) की स्थापना किन नगरों में की जाती है ?
उत्तर:
नगर-परिषद् की स्थापना प्रायः उन नगरों में की जाती है जिनकी जनसंख्या 20,000 से अधिक होती है, परन्तु बहुत अधिक नहीं होती। इसकी स्थापना छोटे नगरों में की जाती है।

प्रश्न 24.
नगर-परिषद् के सदस्यों की संख्या कितनी होती है?
उत्तर:
नगर-परिषद् के सदस्यों की संख्या नगर की जनसंख्या के आधार पर राज्य सरकार द्वारा निश्चित की जाती है।

प्रश्न 25.
नगर-परिषद् के सदस्यों का चुनाव कैसे किया जाता है?
उत्तर:
नगर-परिषद् के सदस्यों का चुनाव नगर में रहने वाले मतदाताओं द्वारा प्रत्यक्ष चुनाव-प्रणाली के आधार पर किया जाता है।

प्रश्न 26.
नगर-परिषद् का कार्यकाल कितना होता है?
उत्तर:
नगर-परिषद् का साधारण कार्यकाल 5 वर्ष निश्चित किया गया है, परन्तु राज्य सरकार द्वारा इसे पहले भी भंग किया जा सकता है।

प्रश्न 27.
नगर परिषद् के कोई दो कार्य लिखें।
उत्तर:

  • नगर में पीने के पानी का प्रबन्ध करना,
  • नगर में बीमारी को फैलने से रोकना।

HBSE 11th Class Political Science Important Questions Chapter 8 स्थानीय शासन

प्रश्न 28.
नगर-परिषद् की आय के कोई तीन साधन लिखें।
उत्तर:

  • गृह कर,
  • व्यवसाय कर,
  • मनोरंजन कर।

प्रश्न 29.
छावनी बोर्ड (Cantonment Board) की स्थापना कहाँ और क्यों की जाती है?
उत्तर:
छावनी बोर्ड की स्थापना उन शहरों में की जाती है जिनमें सैनिक छावनी (Army cantonment) है। यह बोर्ड केन्द्रीय सरकार के अधीन होता है। छावनी बोर्ड-छावनी क्षेत्र में वही कार्य करता है, जो नगर में नगर-परिषद् करती है।

प्रश्न 30.
शहरी स्थानीय संस्थाओं की कार्यविधि में कोई तीन दोष (त्रुटियाँ) लिखें।
उत्तर:

  • धन की कमी,
  • दलबन्दी की भावना,
  • जातिवाद तथा सम्प्रदायवाद।

प्रश्न 31.
स्थानीय सरकार के दोषों को दूर करने के कोई तीन उपाय लिखिये।
सरकार के दोषों को दूर करने के तीन उपाय इस प्रकार है-

  • शिक्षा का प्रचार करना,
  • राजनीतिक जागरूकता लाना,
  • अधिकार क्षेत्र में वृद्धि करना।

लघूत्तरात्मक प्रश्न

प्रश्न 1.
स्थानीय शासन की कोई दो परिभाषाएँ दीजिए।
उत्तर:
स्थानीय शासन. की दो मुख्य परिभाषाएँ निम्नलिखित हैं

1. पी० स्टोन्स के अनुसार, “स्थानीय शासन किसी देश के शासन का वह भाग है जो किसी विशेष क्षेत्र में जनता से सम्बन्धित मामलों का प्रशासन करता है।”
2. एल० गोल्डिंग के अनुसार, “स्थानीय शासन किसी क्षेत्र की जनता द्वारा अपने मामलों का स्वप्रबन्ध है।”

प्रश्न 2.
स्थानीय शासन की चार विशेषताएँ लिखें।
उत्तर:
स्थानीय शासन की चार विशेषताएँ निम्नलिखित हैं
1. निश्चित क्षेत्र स्थानीय शासन की प्रत्येक इकाई का निश्चित क्षेत्र होता है, जिसके अन्दर रहकर स्थानीय इकाई अपने दायित्वों का निर्वाह करती है।
2. स्थानीय सत्ता स्थानीय निकायों के पास स्थानीय सत्ता होती है तथा इसके निर्वाचित प्रतिनिधि इस सत्ता का प्रयोग करते हैं।
3. स्वायत्तता स्थानीय शासन की संस्थाएँ को स्वायत्तता प्राप्त होती है। अन्यथा ये संस्थाएँ अपना उत्तरदायित्व निभा नहीं सकती, लेकिन इन पर राज्य सरकार के कुछ अंकुश भी होते हैं।
4. सरकार व जनता के बीच कड़ी-स्थानीय स्वशासन की इकाइयों को सरकार एवं जनता के बीच कड़ी माना जाता है। जिला प्रशासन इनके माध्यम से ही जनता तक पहुँच सकता है।

प्रश्न 3.
स्थानीय शासन के महत्त्व के चार शीर्षक लिखें।
उत्तर:
स्थानीय शासन के चार महत्त्वपूर्ण शीर्षक निम्नलिखित हैं-
1. लोकतन्त्र की पाठशाला-स्थानीय शासन को लोकतन्त्र की पाठशाला कहा जाता है, क्योंकि इनके द्वारा स्थानीय स्तर पर नागरिकों को प्रशासन चलाने का ज्ञान प्राप्त होता है।

2. जनता की सेवा स्थानीय शासन की संस्थाएँ ऐसी सेवाओं और वस्तुओं का प्रबन्ध करती हैं, जिनसे स्थानीय जनता का जीवन सुखी और खुशहाल बनता है। ये संस्थाएँ कठिन परिस्थितियों में लोगों की सेवा भी करती हैं।

3. धन की बचत-स्थानीय संस्थाएँ केन्द्रीय सरकार का अनावश्यक खर्च कम करती हैं। ये स्थानीय स्तर पर कम खर्च से अधिक लाभ कमा सकती हैं।

4. राजनीतिक विकेन्द्रीयकरण स्थानीय शासन सत्ता के राजनीतिक विकेन्द्रीयकरण का उदाहरण है। इसमें सत्ता का केन्द्रीयकरण नहीं होता, बल्कि सत्ता लोगों में विभाजित होती है।

प्रश्न 4.
पंचायती राज का क्या अर्थ है? अथवा भारत में पंचायती राज की अवधारणा से आप क्या समझते हैं?
उत्तर:
पंचायती राज की धारणा ग्रामीण क्षेत्रों के विकास तथा प्रबन्ध के सम्बन्ध में एक नया विचार है। ग्रामीण क्षेत्रों के स्थानीय स्वशासन को पंचायती राज कहा जाता है। पंचायती राज उस व्यवस्था को कहते हैं जिसके अन्तर्गत गाँवों में रहने वाले लोगों को अपने गाँवों का प्रशासन तथा विकास सम्बन्धी कार्य स्वयं अपनी इच्छानुसार करने का अधिकार दिया जाता है।

ऐसा अनुभव किया जाता है कि एक क्षेत्र में रहने वाले लोग ही उस क्षेत्र की समस्याओं से भली भाँति परिचित होते हैं और उन्हीं के द्वारा ही स्थानीय समस्याओं को ठीक तरह से हल किया जा सकता है। पंचायती राज की स्थापना से ही ग्रामीण स्तरों पर लोकतान्त्रिक संस्थाओं की स्थापना करके शक्तियों का विकेन्द्रीयकरण किया जाता है जिससे लोकतन्त्र अधिक मज़बूत होता है।

प्रश्न 5.
पंचायती राज के मुख्य उद्देश्य क्या है?
उत्तर:
पंचायती राज के मुख्य उद्देश्य इस प्रकार हैं-

  • पंचायती राज का मुख्य उद्देश्य ग्रामीण क्षेत्र में लोकतन्त्र की स्थापना करना है,
  • पंचायती राज का उद्देश्य ग्रामीण क्षेत्रों में रहने वाले लोगों को अपने स्थानीय मामलों को स्वयं नियोजित करने तथा शासन प्रबन्ध करने का अधिकार देना है,
  • पंचायती राज का उद्देश्य गाँवों के लोगों में सामुदायिक और आत्म-निर्भरता की भावना उत्पन्न करना है,
  • पंचायती राज का उद्देश्य ग्रामीण क्षेत्रों में कमज़ोर तथा पिछड़े वर्ग के लोगों का स्वशासन में सहभागी बनाना है,
  • पंचायती राज का उद्देश्य साम्प्रदायिक विकास योजनाओं को लागू करने में ग्रामीण लोगों को उत्साहित करना है।

प्रश्न 6.
पंचायती राज का तीन-स्तरीय ढाँचा क्या है? अथवा भारत में पंचायती राज के त्रि-स्तरीय ढाँचे की कार्य-प्रणाली का विवरण दीजिए।
उत्तर:
संविधान के 73वें संशोधन द्वारा ग्रामीण क्षेत्रों के लिए पंचायती राज का ढाँचा निश्चित किया गया। कुछ छोटे राज्यों (जिनकी कुल जनसंख्या 20 लाख से कम है) के लिए यह ढाँचा दो-स्तरीय है, जबकि अन्य राज्यों में तीन-स्तरीय है। छोटे राज्यों में पंचायती राज की बीच की इकाई पंचायत समिति की स्थापना नहीं की जाएगी। पंचायती राज का तीन-स्तरीय ढाँचा निम्नलिखित प्रकार से कार्य करता है।

1. ग्राम पंचायतें पंचायती राज के ढाँचे में सबसे निचले स्तर पर ग्राम पंचायतें हैं। लगभग सभी गाँवों में ग्राम पंचायतें हैं और छोटे-छोटे गाँवों को पंचायत के लिए किसी साथ वाले गाँव के साथ मिला दिया जाता है। पंच गाँव में रहने वाले सभी मतदाताओं द्वारा चुने जाते हैं और इनका कार्यकाल पाँच वर्ष होता है। ग्राम पंचायतें अपने गाँव के चहुंमुखी विकास के लिए सभी कार्य करती हैं और न्यायिक पंचायतों को तो न्यायिक अधिकार भी दिए गए हैं। पंचायत अपना काम चलाने के लिए कर (Tax) भी लगाती है।

2. पंचायत समिति-पंजाब पंचायती राज अधिनियम, 1994 के अनुसार प्रत्येक विकास खण्ड में पंचायतों के ऊपर पंचायत समिति की व्यवस्था की गई है। इस अधिनियम के अनुसार पंचायत समिति में निम्नलिखित सदस्य होंगे…

(1) प्रत्यक्ष निर्वाचित सदस्य पंचायत समिति के क्षेत्र के मतदाताओं द्वारा प्रत्यक्ष रूप से कुछ पंचायत समिति के सदस्य च जाएँगे। जिनकी संख्या 6 से 10 तक होगी।

(2) सरपंचों द्वारा निर्वाचित सदस्य पंचायत समिति के कुछ सदस्य इसके क्षेत्र में आने वाली पंचायतों के सरपंच अपने में से चुनकर भेजेंगे। सरपंचों द्वारा निर्वाचित सदस्यों और जनता द्वारा निर्वाचित सदस्यों का पंचायत समिति में अनुपात 40:60 का होगा।

(3) विधान सभा के सदस्य पंजाब विधान सभा के वे सदस्य, जिनके चुनाव क्षेत्र का कुछ भाग पंचायत समिति के क्षेत्र में आता है, उसके पदेन सदस्य होंगे।

3. जिला परिषद्-पंजाब पंचायती राज अधिनियम 1994 के अनुसार प्रत्येक जिले में विकास खण्डों के ऊपर एक ज़िला परिषद् होती है। इस अधिनियम के अनुसार ज़िला-परिषद् के निम्नलिखित सदस्य होंगे

  • प्रत्यक्ष निर्वाचित सदस्य-ज़िला-परिषद् के क्षेत्र मतदाताओं द्वारा कुछ सदस्य प्रत्यक्ष रूप से चुने जाते हैं और हर निर्वाचित क्षेत्र से एक सदस्य चुना जाता है। एक ज़िला-परिषद् में 10 से लेकर 25 तक सदस्य हो सकते हैं। जिले के प्रत्येक निर्वाचन क्षेत्र की जनसंख्या लगभग समान होती है,
  • पंचायत समितियों के अध्यक्ष इसके सदस्य होते हैं,
  • लोक सभा और राज्य विधान सभा के वे सदस्य जिनका चुनाव क्षेत्र उस जिला परिषद् में आता है वे उसके पदेन सदस्य होते हैं,
  • राज्य सभा के वे सदस्य, जिनका नाम उस जिले की मतदाता की सूची में अंकित है, भी जिला परिषद् के पदेन सदस्य होंगे।

प्रश्न 7.
ग्राम सभा किसे कहते हैं?
उत्तर:
ग्राम सभा को पंचायती राज की नींव कहा जाता है। एक ग्राम पंचायत के क्षेत्र में रहने वाले सभी मतदाता ग्राम सभा के सदस्य होते हैं। ग्राम सभा की एक वर्ष में दो सामान्य बैठकों का होना अनिवार्य है। ग्राम सभा अपने अध्यक्ष (जिसे सरपंच कहते हैं) तथा कार्यकारी समिति (ग्राम पंचायत) का चुनाव करती है।

ग्राम सभा ग्राम पंचायत द्वारा बनाए गए वार्षिक बजट पर सोच-विचार करती है और अपने क्षेत्र के लिए विकास योजनाएँ तैयार करती है। ग्राम सभा विकास योजनाओं को लागू करने में भी सहायता करती है। परन्तु व्यवहार में ग्राम सभा कोई विशेष कार्य नहीं करती क्योंकि इसकी बैठकें बहुत कम होती हैं।

प्रश्न 8.
ग्राम पंचायत का गठन कैसे किया जाता है।
उत्तर:
ग्राम पंचायत ग्राम सभा की कार्यकारिणी तथा पंचायती राज की त्रि-स्तरीय प्रणाली में प्रथम अर्थात् सबसे निचले स्तर की इकाई है। पंजाब में ग्राम पंचायत के सदस्यों की संख्या एक सरपंच तथा 5-13 पंचों के बीच (ग्राम सभा की सदस्य-संख्या के आधार पर) निश्चित की जाती है। यह संख्या राज्य सरकार द्वारा निश्चित की जाती है।

ग्राम पंचायत के सदस्यों का चुनाव ग्राम सभा द्वारा प्रत्यक्ष चुनाव-प्रणाली द्वारा किया जाता है। प्रत्येक पंचायत में कुछ स्थान अनुसूचित जातियों, जनजातियों तथा महिलाओं के लिए आरक्षित रखे जाते हैं। सरपंच का चुनाव भी ग्राम सभा के सदस्यों द्वारा प्रत्यक्ष रूप से किया जाता है। सरपंच ग्राम पंचायत की बैठकों की अध्यक्षता करता है। ग्राम पंचायत में निर्णय बहुमत से लिए जाते हैं। ग्राम पंचायत का कार्यकाल पाँच वर्ष निश्चित किया गया है।

प्रश्न 9.
ग्राम पंचायत का अध्यक्ष कौन होता है? उसके क्या कार्य हैं? अथवा ग्राम पंचायत अथवा ग्राम के स्थानीय मामलों में सरपंच की भूमिका का वर्णन कीजिए।
उत्तर:
ग्राम पंचायत का अध्यक्ष (सभापति) सरपंच होता है। उसका चुनाव ग्राम सभा के सदस्यों द्वारा प्रत्यक्ष रूप से किया जाता है। सरपंच के मुख्य कार्य इस प्रकार हैं-

  • सरपंच ग्राम पंचायत की बैठकें बुलाता है तथा उनकी अध्यक्षता करता है,
  • सरपंच ग्राम पंचायत की बैठकों की कार्रवाई का रिकार्ड रखता है,
  • सरपंच अपने गाँव की पंचायत के वित्तीय और कार्यकारी प्रशासन के लिए उत्तरदायी होता है,
  • सरपंच ग्राम पंचायत के अधिकारी तथा अन्य कर्मचारी, जिनकी सेवाएँ ग्राम पंचायत को दी गई हों, उनके कार्यों की देखभाल तथा उन पर नियन्त्रण रखता है,
  • सरपंच अपने सभी पंचों के साथ मिलकर गाँव में शान्ति तथा व्यवस्था को बनाए रखता है।

प्रश्न 10.
पंचायत समिति का गठन कैसे किया जाता है।
उत्तर:
पंचायत समिति में निम्नलिखित तीन प्रकार के सदस्य शामिल होते हैं

1. निर्वाचित सदस्य पंचायत समिति के कुछ सदस्य समिति के क्षेत्र में रहने वाले मतदाताओं द्वारा प्रत्यक्ष चुनाव-प्रणाली के आधार पर चुने जाते हैं। पंजाब में इन सदस्यों की संख्या 6 तथा 10 के बीच में राज्य सरकार द्वारा पंचायत समिति क्षेत्र की जनसंख्या के आधार पर निश्चित की जाती है। प्रायः 15,000 की जनसंख्या पर एक सदस्य का निर्वाचन किया जाता है। परन्तु यह संख्या कम-से-कम 6 तथा अधिक-से-अधिक 10 होगी। पंचायत समिति के कुछ सदस्य सरपंचों तथा पंचों द्वारा चुने जाते हैं। इस प्रकार के सदस्यों तथा प्रत्यक्ष रूप से चुने गए सदस्यों का अनुपात 6 : 4 होगा।

2. सहायक सदस्य-पंचायत समिति द्वारा चुने गए राज्य विधान सभा के सदस्य तथा संसद के सदस्य समिति के सहायक सदस्यों के रूप में शामिल होते हैं। उन्हें पंचायत समिति की बैठकों में भाग लेने तथा पंचायत समिति के अध्यक्ष तथा उपाध्यक्ष के चुनाव तथा पदच्युत होने को छोड़कर अन्य सभी निर्णयों के लेने में मतदान का भी अधिकार होता है।

3. पदेन सदस्य-सब डिवीज़ीनल मैजिस्ट्रेट तथा खण्ड विकास अधिकारी पंचायत समिति के पदेन सदस्य होते हैं। उन्हें पंचायत समिति की बैठकों में भाग लेने तथा बोलने का अधिकार होता है, परन्तु वे मतदान में भाग नहीं ले सकते।

4. आरक्षित स्थान पंचायत समिति के निर्वाचित सदस्यों में कुछ स्थान अनुसूचित जातियों, अनुसूचित जन-जातियों के सदस्यों तथा महिलाओं के लिए आरक्षित रखे जाते हैं। पंचायत समिति का कार्यकाल 5 वर्ष निश्चित किया गया है।

प्रश्न 11.
पंचायत समिति के कोई पाँच कार्य लिखें।
उत्तर:
पंचायत समिति क्षेत्र के सभी विकास कार्यों के लिए उत्तरदायी होती है। इसके मुख्य कार्य इस प्रकार हैं-

  • पंचायत समिति का मुख्य कार्य सामुदायिक विकास है। इसमें विकास योजना को लागू करना, उत्पादन में वृद्धि के लिए प्रयत्न करना तथा लोगों के लिए रोज़गार पैदा करना,
  • क्षेत्र में लघु-सिंचाई योजनाएँ बनाना तथा भूमि को उपजाऊ बनाने के लिए नीति बनाना,
  • स्वास्थ्य केन्द्रों की स्थापना करना तथा बीमारी को रोकने के लिए टीके लगवाना,
  • आपात्कालीन सहायता का वितरण,
  • पिछड़ी जातियों के विकास के लिए योजनाएँ बनाना।

प्रश्न 12.
पंचायत समिति की आय के मुख्य साधन बताइए।
उत्तर:
पंचायत समिति की आय के मुख्य साधन इस प्रकार हैं-

  • पंचायत समिति के क्षेत्र में लगाए गए स्थानीय करों का कुछ भाग पंचायत समिति को मिलता है,
  • मेलों तथा मण्डियों से प्राप्त आय,
  • राज्य सरकार से सहायता के रूप में प्राप्त धन,
  • पंचायत समिति की अपनी सम्पत्ति से होने वाली आय,
  • समिति के क्षेत्र से इकट्ठा होने वाले लगान का कुछ भाग भी पंचायत समिति को मिलता है,
  • पंचायत समिति जिला परिषद् की अनुमति से कुछ अन्य कर भी कर लगा सकती है तथा अपनी आमदनी को बढ़ा सकती है।

प्रश्न 13.
हरियाणा राज्य पंचायती अधिनियम, 1994 के अधीन सरपंचों के तीन मुख्य कार्यों का वर्णन कीजिए।
उत्तर:
ग्राम पंचायत के सभापति (अध्यक्ष) को सरपंच कहा जाता है। उसके तीन कार्य इस प्रकार हैं-

  • सरपंच ग्राम पंचायत की बैठकें बुलाता है तथा उनका सभापतित्व करता है,
  • सरपंच ग्राम पंचायत की बैठकों की कार्रवाई का रिकार्ड रखता है,
  • सरपंच अपने गाँव की पंचायत के वित्तीय तथा कार्यकारिणी प्रशासन के लिए जिम्मेवार होता है।

प्रश्न 14.
जिला परिषद् की रचना का वर्णन कीजिए।
उत्तर:
जिला-परिषद् पंचायती राज की सबसे ऊपर की इकाई है। प्रत्येक जिले में पंचायत समिति के ऊपर जिला परिषद् का . गठन किया गया है। वर्तमान में हरियाणा में 21 जिला-परिषद् है क्योंकि नवनिर्मित नए चरखी दादरी जिले में जिला परिषद् के गठन की अधिसूचना अभी जारी नहीं हुई। इसमें ये सदस्य शामिल होते हैं

  • प्रत्येक जिले में कुछ सदस्य, जिनकी संख्या 10 से 25 तक होगी, जिला परिषद् क्षेत्र में रहने वाले मतदाताओं द्वारा प्रत्यक्ष रूप से चुने जाएँगे,
  • जिले में गठित पंचायत समितियों के अध्यक्ष,
  • राज्य सभा, विधान सभा तथा लोक सभा के वे सदस्य जो जिला परिषद् के क्षेत्र से चुने गए हों,
  • राज्य सभा के वे सदस्य जिनका नाम उस जिले के मतदाताओं की सूची में हो।

प्रत्येक जिला परिषद् में कुछ स्थान अनुसूचित जातियों तथा महिलाओं के लिए सुरक्षित किए जाते हैं। जिला-परिषद् का कार्यकाल (73वें संवैधानिक संशोधन के अनुसार) 5 वर्ष निश्चित किया गया है।

HBSE 11th Class Political Science Important Questions Chapter 8 स्थानीय शासन

प्रश्न 15.
जिला परिषद् के अध्यक्ष (Chairman) पर एक नोट लिखें।
उत्तर:
जिला-परिषद् के अध्यक्ष का चुनाव जिला परिषद् के निर्वाचित सदस्यों द्वारा प्रत्यक्ष रूप से अपनी पहली बैठक में किया जाता है। यह चुनाव डिप्टी कमिशनर अथवा उस द्वारा मनोनीत अधिकारी की देख-रेख में होता है। उसका चुनाव 5 वर्ष के लिए होता है। उसे इस अवधि के समाप्त होने से पहले भी सदस्यों के 2/3 बहुमत से हटाया जा सकता है। जिला-परिषद् के अध्यक्ष के मुख्य कार्य इस प्रकार हैं-

  • वह जिला परिषद् की बैठक बुलाता है तथा उसकी अध्यक्षता करता है,
  • वह जिला-परिषद् के लिए नियुक्त अधिकारियों व कर्मचारियों पर निगरानी व नियन्त्रण रखता है,
  • वह अपने जिले में प्राकृतिक आपदा के शिकार लोगों की सहायता के लिए प्रतिवर्ष एक लाख रुपए खर्च करता है,
  • वह जिला परिषद् के वार्षिक बजट पर भी निगरानी रखता है।

प्रश्न 16.
सरकार पंचायती राज पर कैसे नियन्त्रण रखती है?
उत्तर:
पंचायती राज पूर्णरूप से सरकार से स्वतन्त्र नहीं है। राज्य सरकारें पंचायती राज संस्थाओं पर नियन्त्रण रखती हैं और उन्हें आदेश दे सकती हैं। राज्य सरकार निम्नलिखित ढंग से पंचायती राज की संस्थाओं पर नियन्त्रण रखती है

  • पंचायती राज की संस्थाओं की स्थापना राज्य सरकार द्वारा की जाती है और इन संस्थाओं के संगठन, कार्य तथा क्षेत्र का निर्धारण सरकार द्वारा ही किया जाता है,
  • पंचायती राज संस्थाएँ नीति-निर्माण में स्वतन्त्र नहीं हैं। इन संस्थाओं द्वारा पास किए गए प्रस्तावों और नीतियों पर सरकार की स्वीकृति लेनी पड़ती है,
  • पंचायती राज संस्थाओं के सभी प्रथम अधिकारियों की नियुक्ति सरकार द्वारा की जाती है,
  • सरकार या उसका अधिकारी पंचायती राज संस्थाओं को अपने कर्तव्यों का पालन करने के लिए आदेश दे सकता है,
  • पंचायती राज संस्थाएँ वित्त के मामले में आत्म-निर्भर नहीं हैं। इन संस्थाओं को सरकार से अनुदान प्राप्त होता है। अतः सरकार इन संस्थाओं के हिसाब-किताब की जाँच-पड़ताल करने के लिए लेख परीक्षक भेजती है,
  • सरकार डिप्टी कमिशनर के माध्यम से पंचायत और पंचायत समिति के बजट पर नियन्त्रण रखती है,
  • इन संस्थाओं द्वारा बनाए गए उप-नियमों को सरकार की स्वीकृति मिलने के बाद ही लागू किया जा सकता है,
  • विशेष परिस्थितियों में पंचायती राज संस्थाओं को सरकार निलम्बित या भंग कर सकती है।

प्रश्न 17.
वर्तमान ग्रामीण जीवन की तीन प्रमुख समस्याओं पर प्रकाश डालिए।
उत्तर:
स्वतन्त्रता-प्राप्ति के 70 वर्षों के बाद भी ग्रामीण जीवन का पूर्ण विकास नहीं हो पाया है। इसकी तीन प्रमुख समस्याएँ निम्नलिखित हैं

1. जल-आपूर्ति ग्रामीण जनता को पीने का शुद्ध जल आज भी उपलब्ध नहीं है। अधिकतर गाँवों में मनुष्यों और पशुओं के लिए गाँव का तालाब ही जल-आपूर्ति का काम करता है। ऐसे तालाबों का पानी पशुओं के पीने के लिए भी शुद्ध नहीं कहा जा सकता, मनुष्यों की तो बात ही दूसरी है। इस उद्देश्य की पूर्ति में कुछ प्रगति अवश्य हुई है, परन्तु अब भी इस लक्ष्य की प्राप्ति के लिए विशेष प्रयत्नों की आवश्यकता है।

2. ऊर्जा-ग्रामीण विकास के लिए ऊर्जा को उपलब्ध करवाना भी आवश्यक है। ग्रामीण जीवन का काफी बड़ा भाग आज भी ऊर्जा के साधन से वंचित है। कुछ ही राज्य ऐसे हैं जहाँ सभी गाँवों में बिजली की आपूर्ति की व्यवस्था की गई है। आज भी अधिकतर ग्रामवासी घरों में रोशनी करने तथा खाना पकाने के लिए तेल, लकड़ी एवं गोबर के प्रयोग पर निर्भर हैं। उन्हें बिजली की आपूर्ति तथा उन्नत किस्म के चूल्हों और गोबर गैस आदि की सुविधाएँ उपलब्ध कराने की जरूरत है।

3. भूमि का असमान वितरण-ग्रामीण जीवन की यह भी प्रमुख समस्या है कि यहाँ कृषि भूमि का बहुत असमान वितरण देखने को मिलता है। बड़े-बड़े ज़मींदारों के पास बहुत अधिक भूमि है, चाहे वह बेनामी तौर पर है या किसी गैर-कानूनी तरीके से रखी हुई है और वास्तविक रूप में खेती-बाड़ी करने वाले लोग भूमिहीन मज़दूर ही हैं।

प्रश्न 18.
74वें संवैधानिक संशोधन अधिनियम की कोई पाँच विशेषताएँ लिखिए।
उत्तर:
भारत में शहरी स्थानीय संस्थाओं के वर्तमान गठन तथा कार्यविधि का आधार 74वाँ संवैधानिक संशोधन अधिनियम है। इसकी पाँच विशेषताएँ निम्नलिखित हैं
1. संवैधानिक मान्यता इस संवैधानिक संशोधन द्वारा शहरी स्थानीय संस्थाओं को पहली बार सवैधानिक मान्यता प्रदान की गई।

2. तीन प्रकार की शहरी स्थानीय संस्थाओं की व्यवस्था इस अधिनियम द्वारा प्रत्येक राज्य के लिए तीन स्तरीय शहरी स्थानीय संस्थाओं की व्यवस्था की गई है-

  • परिवर्तनीय क्षेत्र के लिए नगर पंचायत,
  • छोटे नगरों के लिए नगर-परिषद्,
  • बड़े नगरों के लिए नगर-निगम।

3. आरक्षण की व्यवस्था इस संशोधन अधिनियम द्वारा सभी शहरी, स्थानीय संस्थाओं में अनुसूचित जातियाँ तथा अनुसूचित जनजातियों के लिए कुछ स्थान आरक्षित करने की व्यवस्था है।

4. अवधि इस संशोधन द्वारा सभी स्तर की शहरी स्थानीय संस्थाओं का कार्यकाल 5 वर्ष निश्चित किया गया है।

5. विशेष व्यक्तियों को प्रतिनिधित्व-इस संशोधन के द्वारा यह भी व्यवस्था की गई है कि विशिष्ट व्यक्तियों, क्षेत्र से सम्बन्धित राज्य विधानमण्डल तथा संसद सदस्यों को भी नगर-परिषद् में शामिल किया जाएगा।

प्रश्न 19.
नगर-निगम का सभापति कौन होता है? उसके कार्य लिखें। अथवा महापौर (Mayor) कौन होता है?
उत्तर:
महापौर (Mayor) नगर-निगम का अध्यक्ष होता है। उसका चुनाव नगर-निगम के सभी सदस्यों, पार्षदों तथा एल्डरमैन द्वारा किया जाता है। साधारणतः उसका चुनाव एक वर्ष के लिए किया जाता है, परन्तु एक वर्ष की समाप्ति पर उसे प्रायः दोबारा महापौर चन लिया जाता है। महापौर नगर-निगम की बैठकों की अध्यक्षता करता है और निगम (सदन) में शान्ति तथा व्यवस्था बनाए रखता है। वह निगम का सबसे अधिक प्रतिष्ठित व्यक्ति होता है।

प्रश्न 20.
नगर-निगम अथवा नगरपालिका (नगर-परिषद्) के कोई छः कार्य लिखें।
उत्तर:
नगर-निगम दिए गए कार्य करती है

  • नगर में पानी तथा बिजली की व्यवस्था करना,
  • नगर की सफाई की व्यवस्था करना,
  • गली-सड़ी वस्तुओं की बिक्री पर रोक लगाना,
  • स्त्रियों के लिए प्रसूति-गृह (Maternity centres) तथा बच्चों के लिए स्वास्थ्य कल्याण केन्द्र स्थापित करना,
  • यातायात के लिए लोकल बसें चलाना,
  • श्मशान-भूमि तथा कब्रिस्तान का प्रबन्ध करना,
  • जन्म व मृत्यु के आँकड़े रखना,
  • गलियों व सड़कों को बनवाना तथा उनकी मुरम्मत करवाना।

प्रश्न 21.
नगर-परिषद् (Municipal Council) की रचना पर नोट लिखें।
उत्तर:
नगर-परिषद् के सदस्यों की संख्या राज्य सरकार द्वारा उस नगर की जनसंख्या के आधार पर निश्चित की जाती है। पंजाब सरकार द्वारा जारी की गई एक अधिसूचना के अनुसार पाँच हजार से कम जनसंख्या वाले क्षेत्रों के लिए 9 सदस्य और पाँच लाख से अधिक जनसंख्या वाले क्षेत्र के नगर-परिषद् में अधिक-से-अधिक 49 सदस्य होंगे। नगर-परिषद् के सदस्यों का चुनाव वयस्क मताधिकार के आधार पर नगर के मतदाताओं द्वारा प्रत्यक्ष रूप से किया जाता है।

चुनाव के लिए नगर को लगभग समान जनसंख्या वाले चुनाव क्षेत्रों में बाँट दिया जाता है और प्रत्येक चुनाव क्षेत्र (वाड) से एक सदस्य चुना जाता है। नगर के सदस्यों को प्रायः नगर पार्षद कहा जाता है। इन सदस्यों के अतिरिक्त राज्य विधान सभा के सभी सदस्य, जिनके चुनाव क्षेत्र पूर्ण अथवा आंशिक रूप से उस नगर की सीमा में आते हैं, नगर-परिषद् के सदस्य होंगे। उन्हें परिषद् की बैठक में भाग लेने तथा बोलने का अधिकार होता है। परन्तु वे मतदान में भाग नहीं ले सकते। नगर-परिषद् का साधारण कार्यकाल पाँच वर्ष निश्चित किया गया है।

प्रश्न 22.
नगर पंचायत पर एक नोट लिखिए।
उत्तर:
नगर पंचायत की स्थापना सरकार द्वारा उन परिवर्तनीय क्षेत्रों में की जाती है, जिसकी जनसंख्या 20,000 से अधिक न हो। पंजाब में इस समय 32 नगर पंचायतों की स्थापना की गई है। पंजाब तथा हरियाणा में नगर पंचायतों के सदस्यों की संख्या कम-से-कम पाँच रखी गई है। इन सदस्यों का चनाव परिवर्तनीय क्षेत्र के मतदाताओं की सूची में शामिल मतदाताओं द्वारा किया जाता है।

नगर पंचायत के सदस्यों की संख्या के आधार पर क्षेत्र को भिन्न-भिन्न वार्डों में बाँट दिया जाता है। प्रत्येक क्षेत्र (वाड) से एक सदस्य चुना जाता है। नगर पंचायत अपने क्षेत्र के लिए वही कार्य करती है जो नगर-परिषद् द्वारा अपने क्षेत्र के लिए किए जाते हैं। ये हैं-नगर की सफाई का प्रबन्ध, लोगों के स्वस्थ की देखभाल, सड़कें अथवा पुल बनवाना तथा उनकी मुरम्मत, पीने के पानी तथा बिजली का प्रबन्ध करना आदि।

प्रश्न 23.
नगर-परिषद् पर सरकार के नियन्त्रण के कोई तीन तरीकों का वर्णन करें।
उत्तर:
सरकार इस प्रकार तरीकों द्वारा नगर-परिषद् पर नियन्त्रण रख सकती है

  • जिले का जिलाधीश (Deputy Commissioner) नगर-परिषद् के कार्यों की देख-रेख करता है। वह नगर-परिषद् को उसके कार्यों के सम्बन्ध में आदेश दे सकता है,
  • नगरपालिका के प्रमुख कर्मचारियों की नियुक्ति राज्य सरकार द्वारा की जाती है,
  • सरकार किसी भी नगर परिषद् को उसका कार्यकाल समाप्त होने से पहले भँग करके वहाँ प्रशासक (Administrator) नियुक्त कर सकती है,
  • नगर-परिषद् जो भी नियम तथा अधिनियम बनाती है उनकी स्वीकृति सरकार से लेनी पड़ती है,
  • सरकार किसी भी नगर-परिषद् का रिकॉर्ड मँगवाकर उसकी देखभाल कर सकती है।

प्रश्न 24.
छावनी बोर्ड पर नोट लिखें।
उत्तर:
जिन नगरों में सैनिक छावनियाँ हैं, उन छावनी क्षेत्रों में स्थानीय विषयों का प्रबन्ध करने के लिए केन्द्रीय सुरक्षा मन्त्रालय छावनी बोर्ड की स्थापना की जाती है। इसके सदस्यों की संख्या केन्द्रीय रक्षा मन्त्रालय द्वारा निश्चित की जाती है। इसके आधे सदस्य सरकार द्वारा मनोनीत किए जाते हैं तथा शेष आधे सदस्यों का चुनाव छावनी क्षेत्र में रहने वाले मतदाताओं द्वारा प्रत्यक्ष रूप से किया जाता है। छावनी बोर्ड का कमांडिंग अफसर छावनी बोर्ड का पदेन (Ex-officio) अध्यक्ष होता है। इसके अतिरिक्त एक कार्यकारी अधिकारी होता है जिसकी नियुक्ति केन्द्रीय सरकार द्वारा की जाती है।

निबंधात्मक प्रश्न

प्रश्न 1.
स्थानीय शासन का अर्थ क्या है? स्थानीय शासन की परिभाषाएँ देते हुए इसकी विशेषताओं का उल्लेख कीजिए।
उत्तर:
स्थानीय शासन का अर्थ एवं परिभाषा (Meaning and Definition of Local Government) स्थानीय शासन हमारे देश में शासन की त्रि-स्तरीय प्रणाली का एक अनिवार्य भाग है अन्य दो स्तर हैं, केन्द्रीय शासन एवं राज्य शासन। दूसरे शब्दों में, स्थानीय शासन शासकीय संस्थाओं के स्तम्भ के निम्नतम स्तर पर अव्यवस्थित है राष्ट्रीय शासन इस स्तम्भ का शीर्ष स्तर है जबकि राज्य शासन मध्यवर्ती स्तर।

स्थानीय शासन ग्रामीण एवं नगरीय दोनों क्षेत्रों में कार्यशील है, अतएव इसे क्रमशः ग्रामीण एवं नगरीय शासन कहते हैं। नगरीय स्थानीय शासन नगर-निगमों, नगरपालिकाओं, टाउन एरिया एवं अधिसूचित क्षेत्र समितियों द्वारा क्रियारत है, जबकि ग्रामीण स्थानीय शासन जिला-परिषदों, पंचायत समितियों एवं ग्राम पंचायतों के माध्यम से कार्य करता है। स्थानीय शासन की परिभाषाएँ विभिन्न विद्वानों द्वारा निम्नलिखित प्रकार से दी गई हैं

1. पी० स्टोनस (P. Stones):
के अनसार, “स्थानीय शासन किस देश के शासन का वह भाग है जो किसी विशेष क्षेत्र में जनता से सम्बन्धित मामलों का प्रशासन करता है।” उसका आगे कथन है कि यह समुदाय की पत्नी के रूप में कार्य करता है, क्योंकि यह उसके पर्यावरण को रहने योग्य बनाता है, सड़कों को स्वच्छ रखता है, बच्चों को शिक्षा प्रदान करता है, मकानों का निर्माण करता है, एवं अन्य सभी ऐसे कार्य, जो सभ्य जीवन व्यतीत करने हेतु योग्य बनाते हैं, करता है।

2. वी. वेंकट राव (V.Venkat Rao):
का कथन है, “स्थानीय शासन सरकार का वह भाग है जो मुख्यतया स्थानीय विषयों से सम्बन्धित है, जिनका प्रशासन राज्य सरकार के अधीन प्राधिकारियों द्वारा होता है, परन्तु जिन्हें योग्यता प्राप्त निवासियों द्वारा स्वतन्त्र रूप से निर्वाचित किया जाता है।”

3. जोन जे० क्लार्क (John J. Clarke):
के शब्दों में, “स्थानीय शासन किसी राष्ट्र अथवा राज्य सरकार का वह भाग है जो मुख्यतया किसी विशेष जिले अथवा स्थान के निवासियों से सम्बन्धित मामलों का निपटारा करता है।”

4. एल० गोल्डिंग (L. Golding):
के अनुसार, “स्थानीय शासन किसी क्षेत्र की जनता द्वारा अपने मामलों का स्वप्रबन्ध है।”

5. जी० मोंटग्यू हैरिस (G Montagu Haris):
का कथन है, “स्थानीय शासन स्वतन्त्र रूप से निर्वाचित प्रतिनिधियों के माध्यम से स्वयं लोगों द्वारा प्रशासन है।”

6. के० वेंकट रंगैया (K. Venkatarangaiya):
के अनुसार, “स्थानीय शासन किसी क्षेत्र, देहात, नगर अथवा राज्य से छोटे अन्य किसी क्षेत्र का स्थानीय निवासियों के प्रतिनिधि निकाय द्वारा प्रशासन है जिसे पर्याप्त भागों में स्वायत्तता प्राप्त होती है, जो स्थानीय करारोपण द्वारा अपने राजस्व का कुछ अंश एकत्रित कर सकती है एवं अपनी आय को स्थानीय सेवाओं पर व्यय कर सकती है। अतः इस प्रकार यह राज्य एवं केन्द्रीय सेवाओं से विभिन्न होती है।”

उपर्युक्त परिभाषाओं के अध्ययन का सरल अर्थ यह निकलता है कि स्थानीय शासन एक ऐसी व्यवस्था है जिसके अन्तर्गत किसी विशेष स्थान का शासन वहाँ के लोगों द्वारा चलाया जाता है। इस व्यवस्था का रूप, संगठन व प्रकृति केन्द्रीय या राज्य सरकार के विशेष अधिनियम के अनुसार निर्धारित की जाती है।

स्थानीय शासन सत्ता के विकेन्द्रीयकरण पर आधारित है और इसका मुख्य उद्देश्य स्थानीय क्षेत्र के निवासियों का कल्याण व विकास करना होता है। स्थानीय स्तर पर गठित संस्थाएँ स्वतन्त्र व स्वायत्तता होती हैं और ये प्रशासन का संचालन बिना रोक-टोक व हस्तक्षेप से करती हैं। परन्तु इसका अर्थ यह नहीं कि ये संस्थाएँ पूर्णतः स्वतन्त्र होती हैं। ये राज्य या केन्द्रीय सरकारों के अधीन रहकर कार्य करती हैं और ये राज्य या केन्द्रीय सरकारों के कानूनों के विरुद्ध कोई कार्य नहीं कर सकतीं। स्थानीय शासन का अर्थ समझने के बाद अब हम स्थानीय शासन की मुख्य विशेषताओं का वर्णन करेंगे, जो इस प्रकार हैं

स्थानीय शासन की मुख्य विशेषताएँ (Main Features of Local Government):
स्थानीय शासन के अर्थ व परिभाषा से यह स्पष्ट हो जाता है कि किसी विशेष क्षेत्र के स्थानीय प्रतिनिधियों द्वारा संचालित शासन-प्रबन्ध ही स्थानीय शासन कहलाता है। स्थानीय शासन की मुख्य विशेषताएँ निम्नलिखित हैं

1. निश्चित स्थानीय क्षेत्र स्थानीय शासन की प्रत्येक इकाई, चाहे वह नगरपालिका की कोई संस्था हो या पंचायत की कोई संस्था हो, का अपना एक निश्चित कार्य क्षेत्र होता है। स्थानीय शासन की प्रत्येक इकाई का अधिकार क्षेत्र व सीमान्तर का निर्धारण राज्य सरकार/संघीय सरकार करती है। इस निश्चित अधिकार क्षेत्र के अन्दर रहकर ही स्थानीय शासन की इकाई अपने दायित्वों का निर्वाह करती है।

राज्य सरकार स्थानीय शासन की किसी भी इकाई की सीमा का निर्धारण करते समय सम्बन्धित क्षेत्र का जनसंख्या घनत्व (एक k.m. वर्ग में रहने वाले लोग), स्थानीय क्षेत्र की आय के साधन, शहरीकरण या ग्रामीण पृष्ठभूमि का विस्तार आदि तत्त्वों को ध्यान में रखती है।

2. स्थानीय सत्ता-स्थानीय शासन की संस्थाओं का प्रशासन कुशलतापूर्वक चलाने के लिए वहाँ के नागरि को सत्ता व शक्ति सौंपना आवश्यक है। ये चुने हुए प्रतिनिधि स्थानीय शासन की संस्थाओं के प्रशासन का संचालन करते हैं और अपने क्षेत्र के लोगों के प्रति उत्तरदायी होते हैं। इन प्रतिनिधियों को स्थानीय समस्याओं के बारे में ज्ञान होता है और ये अपने विचार, केन्द्र या राज्य सरकार के पास प्रभावशाली तरीके से पहुंचा सकते हैं। इसलिए स्थानीय शासन की संस्थाओं के जन-प्रतिनिधियों को स्थानीय स्तर पर शक्ति सौंप देनी चाहिए।

3. स्थानीय लोगों की सेवा-स्थानीय शासन पर गठित सभी संस्थाएँ लोगों की सेवा और सुविधा के लिए कार्यरत होती हैं। इन संस्थाओं का मख्य उद्देश्य स्थानीय क्षेत्र में रहने वाले लोगों का कल्याण करना होता है। बिजली, पानी, स्वास्थ्य, यातायात. शिक्षा आदि सभी प्रकार की सेवाओं का लाभ स्थानीय क्षेत्र के लोगों को मिलता है। यदि वित्तीय स्थिति अच्छी हो और राज्य सरकार कुछ सहायता प्रदान कर दे तो स्थानीय लोगों की सेवा के स्तर व गुणों में वृद्धि हो सकती है।

4. स्थानीय आय के साधन-स्थानीय शासन की संस्थाएँ स्थानीय सेवाओं व प्रशासन का प्रबन्ध करने हेतु स्थानीय वित्तीय स्रोतों से प्राप्त धन का ही प्रयोग करती हैं। इन संस्थाओं की आय की अनेक साधन होते हैं जिनमें स्थानीय कर (Local Tax) राज्य से प्राप्त ऋण व सेवाओं के बदले ली गई फीस शामिल है।

स्थानीय संस्थाओं को अपना बजट बनाने की स्वतन्त्रता होती है। वे अपने शासन को सुचारु रूप से चलाने के लिए आय के सभी उपलब्ध साधनों का पूरा लाभ उठाती हैं। कई बार स्थानीय नागरिक . भी स्वेच्छा से स्थानीय क्षेत्र के विकास के लिए चन्दा, दान आदि भी देते हैं।

5. स्वायत्तता स्थानीय शासन की संस्थाएँ स्थानीय लोगों की जरूरतों को पूरा करती हैं। यदि इन संस्थाओं पर अत्यधिक नियन्त्रण व प्रतिबन्ध लगाए जाएँ तो ये कुशलतापूर्वक कार्य नहीं कर सकतीं। इन संस्थाओं को एक अधिकार क्षेत्र की सीमा में रहकर पूर्ण स्वतन्त्रता व स्वायत्तता होती है।

परन्तु इसका अर्थ यह नहीं लगाया जाना चाहिए कि ये संस्थाएँ अपनी इच्छा से कुछ भी कार्य कर सकती हैं। राज्य या केन्द्रीय सरकार के कानूनों व नियमों के विरुद्ध ये संस्थाएँ कुछ भी नहीं कर सकतीं। इन संस्थाओं को किस हद तक स्वायत्तता दी जाए व किस सीमा तक स्वतन्त्रता दी जाए, इसका निर्णय सम्बन्धित विषय पर निर्भर करता है।

6. अप्रभुसत्तामयी अस्तित्व-ये संस्थाएँ प्रभुसत्ताधारी नहीं होतीं। इन पर केन्द्र व राज्य सरकारों के आदेश व शक्ति का नियन्त्रण होता है। ये केवल स्वायत्तता होती हैं। पूर्ण प्रभुसत्ता केवल राज्य के पास होती है। पूर्ण प्रभुसत्ता का अर्थ यह होता है कि किसी मामले पर निर्णय लेने का अन्तिम अधिकार किसके पास हो। स्थानीय प्रशासन चलाने के लिए इन संस्थाओं को स्वायत्तता दी गई है, परन्तु कर्तव्यों का पालन ठीक प्रकार से नहीं करने पर सरकार इनके कार्यों में हस्तक्षेप भी कर सकती है।

ये संस्थाएँ अपने कार्यों व शक्तियों के सफलतापूर्वक संचालन के लिए, वित्तीय सहायता, तकनीकी सहायता, कानूनों की स्वीकृति एवं कर्मचारियों की भर्ती व प्रशिक्षण के लिए राज्य सरकार की स्वीकृति पर निर्भर रहती हैं। इस प्रकार हम कह सकते हैं कि राज्य सरकार स्थानीय शासन की संस्थाओं पर वैधानिक, प्रशासनिक व वित्तीय नियन्त्रण रखती है। इस प्रकार ये संस्थाएँ पूर्णतः प्रभुसत्ता सम्पन्न नहीं होती।

7. संवैधानिक दर्जा प्रत्येक देश की स्थानीय स्वशासन की संस्थाओं को संविधान या विधायिका द्वारा बनाए गए कानूनों या कार्यकारी आदेशों के द्वारा वैधानिक दर्जा दिया जाता है। भारत में 73वें व 74वें संवैधानिक संशोधन के पारित हो जाने के उपरान्त पंचायतों व नगरपालिकाओं से सम्बन्धित सभी महत्त्वपूर्ण बातों का वर्णन संविधान की धारा 243, 11वीं अनुसूची व 12वीं अनुसूची में किया गया है। इससे पूर्व पंचायतों का जिक्र धारा 40 में आता था।

8. सभी स्तरों पर गठित सरकारों का आधार अन्तर्राष्ट्रीय, राष्ट्रीय, राज्य तथा जिला स्तर पर गठित सरकारों का आधार व नींव स्थानीय शासन की इकाइयाँ होती हैं। इन्हें सरकार की स्टाफ एजेंसी माना जाता है। केन्द्रीय व राज्य सरकार के कल्याणकारी व विकास सम्बन्धी कार्यक्रम स्थानीय स्तर पर ही लागू किए जाते हैं।

सभी विषय चाहे वह कानून व व्यवस्था का विषय हो या फिर समाज कल्याण या राष्ट्रीय विकास का, स्थानीय लोगों के लिए होते हैं। सरकार व जनता के बीच कड़ी-स्थानीय स्वशासन की इकाइयों को सरकार व जनता को जोड़ने वाली कड़ी माना जाता है। जिला प्रशासन जिसे भारतीय लोक प्रशासन का आधार माना जाता है, वास्तव में स्थानीय शासनीय इकाइयों के माध्यम से ही जनता तक पहुँचता है। इन इकाइयों को राजनीतिक शक्ति भी प्राप्त होती है जिनकी उपेक्षा करना व्यावहारिक तौर पर सम्भव नहीं होता।

10. जनता का शासन-स्थानीय शासन को जनता का शासन कहा जाता है, क्योंकि जनता के प्रतिनिधि, जो कि प्रत्यक्ष चुनाव से चुने जाते हैं, ही लोगों के सहयोग व परामर्श के अनुसार स्थानीय स्वशासन का प्रबन्ध करते हैं। इस स्तर पर जन-प्रतिनिधियों और जनता के मध्य विशेष दूरी नहीं होती। प्रजातन्त्र, जिसका अर्थ लोगों का शासन होता है, स्थानीय शासन की इकाइयों में ही विराजमान होता है।

11. स्थानीय शासन का विभाजन प्रत्येक देश की स्थानीय शासन की इकाइयों का विभाजन दो प्रकार से होता है ग्रामीण एवं शहरी संस्थाएँ। भारत में ग्रामीण स्तर पर गठित शासन की इकाइयों में पंचायत, पंचायत समिति तथा जिला परिषद और शहरी स्तर पर गठित शहरी शासन की इकाइयों में नगर परिषद, नगर पालिका एवं नगर-निगम सम्मिलित हैं। ग्रामीण इकाइयों में जिला-परिषद और शहरी इकाइयों में नगर-निगम उच्चतम, स्वायत्तता, शक्तिशाली तथा स्वतन्त्र निकाय माने जाते हैं।

12. सरकार पर निर्भरता स्थानीय स्तर पर क्षेत्रीय लोगों द्वारा गठित प्रशासकीय इकाइयों, जो कि स्वयं निर्णय करती हैं, को स्थानीय सरकार कहा जाता है, परन्तु व्यवहार में ये निकाय पूर्णतः स्वतन्त्र नहीं होते। केन्द्रीय या राज्य सरकार भिन्न-भिन्न आधारों पर इनकी आन्तरिक कार्य-प्रणाली में हस्तक्षेप करती रहती है।

सरकार द्वारा नियुक्त अधिकारी इन संस्थाओं की स्वायत्तता पर प्रभाव डालते हैं और इन अधिकारियों को स्थानीय शासन की इकाइयों की स्वतन्त्रता के रास्ते में गले की हड्डी माना जाता है। ये निकाय धन की उपलब्धता, नियमों की व्याख्या, तकनीकी परामर्श, पदाधिकारियों की अवधि तथा कार्यकाल आदि महत्त्वपूर्ण विषयों पर सरकार के दृष्टिकोण व सोच पर ही निर्भर होते हैं।

13. निरन्तर विकास की अवस्था में स्थानीय प्रशासन की संस्थाओं की एक और विशेषता यह होती है कि ये संस्थाएँ निरन्तर विकास की अवस्था में रहती हैं क्योंकि इनमें हर समय विकास की गुंजाइश रहती है। इतिहास के भिन्न-भिन्न कालों; जैसे प्राचीन युग, वैदिक युग, मौर्य युग, मुगल युग, ब्रिटिश युग और स्वतन्त्रता के पश्चात् के युग में इन संस्थाओं में किसी-न-किसी प्रकार का सुधार हुआ है। आज संवैधानिक मान्यता मिलने के बाद भी इन संस्थाओं के विकास का क्रम रुका नहीं है।

14. प्रदत्त शक्तियों का प्रयोग-स्थानीय सरकार की भिन्न-भिन्न इकाइयों में राज्य या केन्द्रीय सरकारों द्वारा प्रदत्त की गई शक्तियों का प्रयोग किया जाता है। सम्बन्धित उचित सत्ता (Concerned Appropriate Authority) अधिनियमों व उप-नियमों के अनुसार इन इकाइयों की शक्तियाँ व कार्य हस्तान्तरित करती रहती है, ताकि प्रजातन्त्र का विकास हो सके और ये संस्थाएँ सरकार के अधीन रहकर अपने कर्तव्यों का निर्वाह कर सकें।

निष्कर्ष स्थानीय शासन की इकाइयों को संवैधानिक दर्जा मिलने के बाद और प्रजातान्त्रिक विकेन्द्रीयकरण की अवधारणा के विकास के फलस्वरूप स्थानीय शासन का महत्त्व बढ़ गया है। प्रजातन्त्र की गति और सफलता स्थानीय शासन की कार्यकुशलता व श्रेष्ठता पर निर्भर करती है।

HBSE 11th Class Political Science Important Questions Chapter 8 स्थानीय शासन

प्रश्न 2.
स्थानीय शासन के महत्त्व का वर्णन करें।
उत्तर:
स्थानीय शासन का महत्त्व (Significance of Local Government)-स्थानीय शासन प्रजातन्त्र की नींव (Foundation) है। यह केन्द्रीय स्तर पर गठित प्रशासनिक व्यवस्था की आधारशिला है। यदि स्थानीय स्तर पर लोगों को शासन करने का अधिकार न दिया जाए तो प्रजातन्त्र का विकास नहीं हो सकता। कल्याणकारी राज्य की अवधारणा और लोगों के अपने अधिकारों ता के कारण स्थानीय शासन की संस्थाओं का महत्त्व बढ़ रहा है।

सरकार की एक विशेष इकाई के रूप में स्थानीय शासन के उद्भव के कई कारण रहे हैं। इनमें ऐतिहासिक, वैधानिक और प्रशासनिक कारण मुख्य हैं। यदि भिन्न-भिन्न लेखकों के विचारों का विश्लेषण किया जाए तो स्थानीय शासन के महत्त्व को आसानी से समझा जा सकता है, जो इस प्रकार हैं

1. लोकतन्त्र की पाठशाला स्थानीय शासन को लोकतन्त्र की पाठशाला कहा जाता है क्योंकि इनके द्वारा स्थानीय स्तर पर नागरिकों को प्रशासन चलाने के दायित्वों का ज्ञान मिलता है और वे शासन-प्रणाली के संचालन की तथा प्रबन्ध की शिक्षा ग्रहण करते हैं। स्थानीय शासन लोगों को प्रशासनिक प्रक्रियाओं में प्रत्यक्ष भाग लेने का अवसर प्रदान करता है।

लोग स्थानीय समस्याओं व झगड़ों को दूर करने का उत्तदायित्व सम्भालते हैं और उनकी प्रजातन्त्र में रुचि बढ़ जाती है। ब्राइस (Bryce) के अनुसार, स्थानीय शासन को लोकतन्त्र की सर्वोत्तम पाठशाला और लोकतन्त्र का सूक्ष्म रूपं मानता है। वी. वेंकट राव (V.Venkat Rao) के अनुसार, “स्थानीय शासन लोकतन्त्र का आधार है। इसके बिना लोकतन्त्र की कल्पना भी नहीं की जा सकती अथवा इसके बिना लोकतान्त्रिक ढाँचा ही छिन्न-भिन्न हो जाएगा। दूसरी ओर स्थानीय शासन का स्थायित्व भी लोकतन्त्र पर निर्भर है। यह सम्भवतः लोकतन्त्रात्मक प्रणाली में सम्भव है।” इसलिए हम कह सकते हैं कि स्थानीय शासन प्रजातन्त्र का पालना है, जहाँ पर प्रजातन्त्र जन्म लेता है और विकसित होता है।

2. सामाजिक सहयोग का साधन स्थानीय शासन की संस्थाएँ लोगों में सामाजिक भावना का विकास करती हैं। स्थानीय स्तर पर किसी भी प्रकार की समस्या, झगड़े व वाद-विवादों का हल वे मिल-जुलकर निकाल लेते हैं। स्थानीय शासन सामाजिक सहयोग व भाईचारे का उचित साधन है। स्थानीय शासन की संस्थाएँ एक ऐसा माध्यम हैं जिनसे समाज को व्यक्तिगत व स्वार्थी तत्त्वों के कारण टूटने से बचाया जा सकता है। स्थानीय स्तर पर लोगों में परस्पर सहयोग, पहलकदमी और उत्तरदायित्व की भावना का विकास होता है, जिससे सामुदायिकी भावना और प्रजातन्त्र को शक्ति मिलती है।

3. जनता की सेवा स्थानीय शासन की संस्थाओं का मुख्य उद्देश्य स्थानीय लोगों की सेवा करना होता है। ये संस्थाएँ ऐसी सेवाओं व वस्तुओं का प्रबन्ध करती हैं जिनसे स्थानीय जनता का जीवन सुखी व खुशहाल बनता है। इन संस्थाओं द्वारा लोगों को प्रदान की गई महत्त्वपूर्ण सेवाओं में सफाई, स्वास्थ्य, शिक्षा, मनोरंजन, पानी का प्रबन्ध, लाइब्रेरी व यातायात के साधनों का प्रबन्ध आदि आते हैं। ये संस्थाएँ स्थानीय लोगों की कठिन परिस्थितियों में भी सेवा करती हैं।

4. प्रशासन में कुशलता यदि स्थानीय संस्थाओं का अस्तित्व न हो तो केन्द्रीय सत्ता द्वारा प्रशासन का संचालन करना अति कार्यों की अधिकता व स्थानीय समस्याओं के ज्ञान के अभाव के कारण राज्य सरकार के अधिकारी अपना पूरा ध्यान स्थानीय समस्याओं की ओर नहीं दे पाते। स्थानीय लोगों को शासन का अधिकार देकर हम उन्हें प्रशासन का भाग बन सकते हैं और आवश्यकता पड़ने पर उन्हें कोई भी उत्तरदायित्वपूर्ण प्रशासकीय कार्य सौंपा जा सकता है। ये संस्थाएँ लोगों के सबसे नजदीक होती हैं जिसके कारण लोगों के द्वारा स्थानीय समस्याओं का हल करना सम्भव हो जाता है।

5. केन्द्रीय शासन के कार्यभार को कम करना स्थानीय शासन की संस्थाएँ केन्द्र व राज्य सरकार तथा जिला प्रशासन के कार्यभार को हल्का करती हैं। इससे उनका बचा हुआ समय व ऊर्जा राष्ट्रीय महत्त्व के विषयों की ओर केंद्रित हो सकते हैं। इससे राज्य व केन्द्रीय स्तर की सरकारें अपना-अपना उत्तरदायित्व अच्छी तरह निभा सकती हैं।

6. अधिकारी वर्ग को तानाशाह बनने से रोकना-स्थानीय शासन अधिकारी तन्त्र की तानाशाही प्रवृत्ति के विरुद्ध एक कवच के रूप में कार्य करता है। कई बार अधिकारी गण लोगों की समस्याओं का समाधान करने की बजाय स्वेच्छाचारी व्यवहार करते हैं और जन-भावनाओं का बिल्कुल आदर नहीं करते। ऐसे समय में स्थानीय शासन की संस्थाएँ, जोकि एक महत्त्वपूर्ण राजनीतिक शक्ति होती है, अधिकारियों पर पूर्ण अनुशासन बनाए रखती हैं जिससे अधिकारियों की तानाशाही नीतियों को रोकने में सफलता मिलती है। डॉ० फाइनर (Dr. Finer) ने इस विषय पर बहुत सटीक विचार व्यक्त करते हुए कहा है, “स्थानीय शासन केन्द्रीयकरण के बढ़ते हुए खतरे की प्रतिक्रिया है।”

7. विकास योजनाओं की सफलता-देश के विकास के लिए चलाई जा रही विकास योजनाओं की सफलता स्थानीय शासन की संस्थाओं पर निर्भर करती है। विकास व कल्याणकारी योजनाएँ तब तक सफल नहीं हो सकती जब तक ये संस्थाएँ अपना योगदान न दें। विकास योजनाओं में लोगों की सहभागिता इन संस्थाओं के माध्यम से सुनिश्चित हो जाती है। संविधान के 73वें व 74वें संशोधन के लागू होने के बाद ग्राम सभाओं व जिला योजना समितियों को विकास का मुख्य उत्तरदायित्व सौंपा गया है। केन्द्र व राज्य सरकारें सभी विकास योजनाओं को इन संस्थाओं के माध्यम से लागू करती हैं।

8. राजनीतिक विकेन्द्रीयकरण स्थानीय शासन सत्ता के राजनीतिक विकेन्द्रीयकरण का उदाहरण है। इस प्रकार की व्यवस्था में सत्ता का केन्द्र एक नहीं होता, बल्कि लोगों के मध्य विभाजित हो जाता है। स्थानीय नागरिकों को राजनीतिक निर्णय लेने की है और वे बार-बार आवश्यक कार्यों हेतु केन्द्रीय सत्ता या अधिकारियों के पास नहीं जाते। प्राकृतिक आपदाओं में ये संस्थाएँ लोगों की कुशलता से सेवा करती हैं।

9. लोकतन्त्र को मजबूत बनाना-स्थानीय शासन से लोकतन्त्र मजबूत बनता है। लोग स्थानीय स्तर पर सभी समस्याओं को हल कर लेते हैं और उन्हें निर्णय लेने की स्वतन्त्रता प्राप्त होती है जिससे जनता में प्रजातान्त्रिक भावना का विकास होता है। इन संस्थाओं से लोगों में नागरिकता के गुण पनपते हैं। इन संस्थाओं को अधिक उपयोगी और कारगर बनाने के लिए अधिक-से-अधिक अधिकार दिए जाने चाहिएँ।

10. धन की बचत-स्थानीय शासन की संस्थाएँ केन्द्रीय सरकार का अनावश्यक खर्च कम करती हैं। ये संस्थाएँ स्थानीय स्तर पर उपलब्ध साधनों का कुशलतम प्रयोग करके कम-से-कम धन से अधिक लाभ कमा सकती हैं। यदि ये संस्थाएँ न हों तो केन्द्रीय सरकार को विशेष कार्य लागू करने के लिए अलग-अलग विभागों की स्थापना करनी पड़ती और इन विभागों में भर्ती किए गए कर्मचारियों को वेतन एवं अन्य सुविधाओं के लिए धन खर्च करना पड़ता।

11. मानव संसाधनों का विकास-स्थानीय स्तर पर कुछ ऐसे व्यक्ति होते हैं जो कोई कार्य नहीं करते, जबकि उनमें कार्य करने की क्षमता, योग्यता और प्रतिभा होती है। ये व्यक्ति स्थानीय संस्थाओं में अपना योगदान देते हैं जिस कारण इन्हें अपने गुणों का विकास करने का अवसर मिल जाता है। स्थानीय शासन की संस्थाओं के अभाव में स्थानीय स्तर पर उपलब्ध मानव व भौतिक संसाधन व्यर्थ में ही नष्ट हो जाते हैं।

12. नागरिक गुणों व राष्ट्रीय भावना का विकास-जब स्थानीय लोग पंचायतों व नगरपालिकाओं की कार्य-प्रणालियों में भाग लेते हैं तो उन्हें अपने अधिकारों व कर्त्तव्यों का ज्ञान हो जाता है। इससे उनमें एक अच्छे नागरिक के कर्तव्यों का विकास होता है। इसके अतिरिक्त उनमें जन-सेवा, उदारता, त्याग, राष्ट्र-भक्ति, कर्तव्य-पालन आदि की भावनाओं का विकास होता है। प्रो० लॉस्की का विचार है कि स्थानीय शासन की संस्थाएँ नागरिक गुणों और राष्ट्रीय भावना के विकास का मुख्य साधन हैं। इस स्तर पर वे देश-भक्ति का प्रथम पाठ भी सीखते हैं।

स्थानीय शासन के उपर्युक्त महत्त्वों को देखते हुए हम कह सकते हैं कि स्थानीय शासन का.मानव जीवन में अत्यधिक महत्त्व है। हर आदमी को किसी-न-किसी काम के लिए स्थानीय शासन की जरूरत पड़ती है। संक्षेप में यह स्वीकार किया जा सकता है कि लोगों की सेवा करने, प्रजातन्त्र का विकास करने, अधिकारी वर्ग की तानाशाही को कम करने, आर्थिक मितव्ययता तथा योजनाओं को सफल बनाने में स्थानीय शासन की संस्थाएँ अति महत्त्वपूर्ण भूमिका अदा करती हैं।

प्रश्न 3.
पंचायती राज की धारणा की व्याख्या करें। इसके मुख्य उद्देश्य क्या हैं? अथवा पंचायती राज का क्या अर्थ है? इसकी तीन-स्तरीय संरचना की व्याख्या करें।
उत्तर:
पंचायती राज स्वतन्त्र भारत की एक महान् उपलब्धि है। पंचायती राज उस व्यवस्था को कहते हैं जिसके अन्तर्गत गाँवों में रहने वाले लोगों को अपने गाँवों का प्रशासन तथा विकास सम्बन्धी कार्य स्वयं अपनी इच्छानुसार करने का अधिकार दिया गया है। भारत में प्राचीनकाल से ही ग्राम पंचायतों का महत्त्व रहा है। वैदिक काल में स्थानीय समस्याओं को स्थानीय लोग स्वयं ही सुलझा लेते थे।

वैदिक काल के पश्चात् भी सैंकड़ों वर्षों तक ग्रामीण जीवन स्वतन्त्रता-पूर्वक व्यतीत होता रहा। मुस्लिम युग में यद्यपि मुस्लिम शासकों ने पंचायतों को महत्त्व नहीं दिया, परन्तु पंचायतें फिर भी झगड़ों का निपटारा करती थीं और कई अन्य महत्त्वपूर्ण कार्य भी करती थीं। अंग्रेजी शासन के प्रारम्भिक काल में पंचायतों को नष्ट कर दिया गया।

उसके पश्चात् 20वीं शताब्दी के आरम्भ में यद्यपि पंचायतों की फिर से स्थापना करने के लिए एक कानून भी पास किया गया, परन्तु इस दिशा में कोई विशेष प्रगति नहीं हुई। स्वतन्त्रता-प्राप्ति के पश्चात् भारतीय संविधान के निर्माताओं ने देश में पंचायतों की स्थापना की ओर भी ध्यान दिया। ग्रामों में पंचायतों की स्थापना के महत्त्व को समझते हुए संविधान के अनुच्छेद 40 में यह व्यवस्था की गई कि राज्य ग्राम पंचायतों का गठन करेगा और उन्हें इतनी शक्तियाँ व अधिकार देगा, जिससे वे स्वशासन की इकाइयों के रूप में कार्य करने के योग्य बन सकें। इस कार्य के लिए सन् 1956 में श्री बलवन्त राय मेहता की अध्यक्षता में एक समिति का गठन किया गया।

इस समिति की रिपोर्ट के आधार पर ही सम्पूर्ण देश में तीन-स्तरीय (Three-Tier). पंचायती राज की स्थापना की गई। इसमें सबसे निचले अथवा गाँव के स्तर पर ग्राम पंचायत (Gram Panchayat), ब्लॉक अथवा खण्ड के स्तर पर पंचायत समिति (Panchayat Samiti) तथा जिले । स्तर पर जिला परिषद् (Zila Parishad) की व्यवस्था की गई। इस ढाँचे का कार्य-क्षेत्र जिले के ग्रामीण क्षेत्रों से सम्बन्धित था।

पंचायती राज की स्थापना भारत में सबसे पहले 2 अक्तूबर, 1959 को राजस्थान में हुई। आन्ध्र प्रदेश ने 1 नवम्बर, 1959 को पंचायती राज व्यवस्था को अपनाया तथा असम, कर्नाटक एवं उड़ीसा ने भी उसी वर्ष इसे लागू किया। गुजरात, बिहार, पंजाब, महाराष्ट्र तथा उत्तर प्रदेश ने सन् 1961 में, मध्य प्रदेश ने सन् 1962 में, पश्चिम बंगाल ने सन् 1963 में तथा हिमाचल प्रदेश ने इसे सन् 1968 में अपनाया।

संविधान के 73वें संशोधन, जो सन् 1992 में पास हुआ तथा 1993 में लागू हुआ, द्वारा पंचायती राज व्यवस्था को सं मान्यता प्रदान की गई है। इस संशोधन द्वारा संसद ने इस व्यवस्था का एक सामान्य ढाँचा (General Structure) निर्धारित कर दिया जिसके आधार पर राज्यों को पंचायती राज व्यवस्था के लिए नए कानून बनाने के लिए कहा गया। पंचायती राज के मुख्य उद्देश्य पंचायती राज के मुख्य उद्देश्य इस प्रकार हैं

  • ग्रामीण क्षेत्रों में रहने वाले लोगों को अपने स्थानीय मामलों में स्वयं नियोजन तथा प्रबन्ध का अवसर देना,
  • लोकतान्त्रिक शासन-प्रणाली को स्थानीय स्तर पर क्रियान्वित करना तथा शक्ति के विकेन्द्रीयकरण को विश्वसनीय बनाना है,
  • ग्रामीण क्षेत्रों के पिछड़े तथा कमज़ोर लोगों को ग्रामीण विकास के कार्यक्रम में भागीदार बनाने का अवसर प्रदान करना है,
  • ग्रामीण सामुदायिक विकास योजनाओं को क्रियान्वित करने के लिए ग्रामीण लोगों को सहभागी बनाने, उनकी पहल शक्ति (Initiative) को उत्साहित करना तथा उनके जीवन-स्तर को ऊँचा उठाने के लिए प्रयत्न करना है,
  • गाँवों में उपलब्ध मानव-शक्ति तथा अन्य साधनों का उचित प्रयोग करना,
  • ग्रामीण क्षेत्रों में गरीबी दूर करना तथा सामाजिक न्याय का विकास करना।

पंचायती राज की संरचना संविधान के 73वें संशोधन, जो सन् 1993 में लागू हुआ, द्वारा यह व्यवस्था की गई थी कि प्रत्येक राज्य सरकार के लिए इस संशोधन की व्यवस्थाओं के अनुसार पंचायती राज सम्बन्धी नए अधिनियम का निर्माण करना अनिवार्य है। ऐसा अधिनियम 73वें संवैधानिक संशोधन के लागू होने के एक वर्ष के अन्दर बन जाना चाहिए।

इस व्यवस्था के अनुसार हरियाणा विधान सभा ने ‘हरियाणा पंचायती राज अधिनियम, 1994’ (Haryana Panchayati RajAct, 1994) पास किया, जो 21 अप्रैल, 1994 को लागू हुआ। इस अधिनियम द्वारा हरियाणा में ग्रामीण क्षेत्रों के लिए पंचायती राज के नए ढाँचे की व्यवस्था की गई है। इसके अनुसार हरियाणा में पंचायती राज व्यवस्था की निम्नलिखित चार संस्थाएँ स्थापित की गई हैं ।

  • ग्राम सभा,
  • ग्राम पंचायत,
  • पंचायत समिति,
  • ज़िला-परिषद्।

1. ग्राम सभा हरियाणा पंचायती राज अधिनियम, 1994 के अधीन यह व्यवस्था की गई है कि सरकार 500 अथवा उससे अधिक की जनसंख्या वाले एक गाँव तथा कुछ पड़ोसी गाँवों को सामूहिक तौर पर ग्राम सभा क्षेत्र (Gram Sabha Area) घोषित कर सकती है। इस क्षेत्र में रहने वाला प्रत्येक व्यक्ति, जो 18 वर्ष की आयु पूरी कर चुका है तथा उसका नाम मतदाता सूची में है, ग्राम सभा का सदस्य होता है। दूसरे शब्दों में, ग्राम सभा क्षेत्र में रहने वाले सभी मतदाता ग्राम सभा के सदस्य होते हैं।

2. ग्राम पंचायत (Gram Panchayat)-ग्राम पंचायत ग्रामीण स्थानीय स्वशासन की मुख्य इकाई है। इस समय देश में 2,50,000 के लगभग ग्राम पंचायतें मौजूद हैं जबकि हरियाणा में 6311 ग्राम पंचायतें वर्तमान में हैं। हरियाणा में पंचायती राज का गठन हरियाणा पंचायती राज अधिनियम, 1994 (Haryana Panchayati RajAct, 1994) के आधार पर किया गया है।

हरियाणा के प्रत्येक गांव जिसकी जनसंख्या 500 या इससे अधिक है, में एक ग्राम सभा (Gram Sabha) की स्थापना की जाती है। इससे कम जनसंख्या वाले गाँव को इस उद्देश्य से किसी साथ वाले गांव से मिलाकर एक सांझी ग्राम सभा की स्थापना की जाती है। उस क्षेत्र में रहने वाले सभी नागरिक, जिनकी आयु 18 वर्ष अथवा उससे अधिक होती है, ग्राम सभा के सदस्य होते हैं।

हरियाणा में एक ग्राम पंचायत में एक सरपंच (Sarpanch) तथा 6 से 20 तक पंच (Members) होते हैं। यह संख्या गांव की जनसंख्या, के आधार पर निश्चित की जाती है। सरपंच तथा अन्य पंचों (सदस्यों) का चुनाव ग्राम सभा के सदस्यों द्वारा प्रत्यक्ष चुनाव-प्रणाली के आधार पर किया जाता है।

3. पंचायत समिति-पंचायत समिति का गठन खण्ड (Block) स्तर पर किया जाता है। पंचायत समिति के कुछ सदस्यों का चुनाव प्रत्यक्ष रूप से ग्राम सभा के सदस्यों द्वारा किया जाता है। इसके कुछ संदस्य सरपंचों तथा पंचों द्वारा निर्वाचित किए जाते हैं। लोगों द्वारा चुने गए सदस्यों तथा पंचायतों के सदस्यों द्वारा चुने गए पंचायत समिति के सदस्यों का अनुपात 60:40 होता है। इसके अतिरिक्त राज्य विधान सभा के सदस्य तथा सहकारी संस्थाओं के प्रतिनिधि भी इसके सदस्य होते हैं। पंचायत समिति का मुख्य कार्य अपने क्षेत्र में ग्रामीण विकास तथा सामुदायिक विकास योजनाओं को क्रियान्वित करना होता है।

4. जिला परिषद्-जिला-परिषद् की स्थापना जिला स्तर पर की जाती है। प्रत्येक जिले में एक ज़िला-परिषद् होती है। जिले । में गठित सभी पंचायत समितियों के अध्यक्ष जिला परिषद् के सदस्य होते हैं। जिले से चुने गए विधान सभा तथा संसद के सदस्य इसके सहायक सदस्य होते हैं। जिलाधीश (Deputy Commissioner) ज़िला-परिषद् का पदेन सदस्य होता है। जिला-परिषद् का मुख्य कार्य अपने अधीन पंचायत समितियों के कार्यों की देखभाल करना, उनमें तालमेल बनाए रखना तथा उन्हें निर्देश देना होता है।

प्रश्न 4.
पंचायती राज से सम्बन्धित 73वें संवैधानिक संशोधन पर संक्षिप्त नोट लिखें। अथवा 73वें संवैधानिक संशोधन के अन्तर्गत पंचायती राज प्रणाली में क्या परिवर्तन किए गए हैं?
उत्तर:
पंचायती राज से संबंधित 72वां सवैधानिक संशोधन बिल 16 सितंबर, 1992 को लोकसभा में पेश किया गया। यह संशोधन लोकसभा द्वारा 22 दिसंबर को और राज्यसभा द्वारा 23 दिसंबर, 1992 को पास कर दिया गया। इसे 24 अप्रैल, 1993 को राष्ट्रपति की मंजूरी 73वें संवैधानिक संशोधन के रूप में मिली। इस संशोधन द्वारा प्रत्येक राज्य के लिए एक वर्ष के अन्दर पंचायती राज एक्ट के आधार पर राज्य विधानमण्डल द्वारा कानून बनाना अनिवार्य किया गया। यह संशोधन 24 अप्रैल, 1993 को लागू हुआ। 73वें संवैधानिक संशोधन अधिनियम की मुख्य विशेषताएँ निम्नलिखित हैं

1. पंचायती राज संस्थाओं को संवैधानिक मान्यता (Constitutional Recognition to Panchayati Raj Institutions):
73वें संवैधानिक संशोधन के पास होने से पहले स्थानीय स्वशासन संस्थाओं को संविधान द्वारा मान्यता प्राप्त नहीं थी और संविधान में इनकी कोई व्यवस्था भी नहीं थी। इस संशोधन द्वारा संविधान में एक नया भाग (Part-IX) तथा एक नई अनुसूची शामिल किए गए हैं। भाग 9 में अंकित सभी अनुच्छेद तथा 11वीं अनुसूची में उच्च विषयों का वर्णन किया गया है, जिनके संबंध में शक्तियाँ पंचायतों को सौंपी जा सकती हैं।

2. ग्रामसभा की परिभाषा (Definition of Gram Sabha):
73वें संवैधानिक संशोधन में ग्राम सभा की परिभाषा दी गई है। इस परिभाषा के अनुसार एक पंचायत के क्षेत्र में आए गाँवों के जिन लोगों के नाम मतदाताओं की सूची में दर्ज हैं-वे सभी लोग सामूहिक रूप से ग्रामसभा का निर्माण करेंगे। ग्रामसभा गाँव के स्तर पर ऐसी शक्तियों का प्रयोग करेगी और ऐसे कार्यों को करेगी जो राज्य विधानमंडल कानून बनाकर निश्चित करेगी। .

3. पंचायत की परिभाषा तथा संरचना (Definition and Composition of Panchayat):
73वें सवैधानिक संशोधन के अनुसार पंचायत शासन की एक ऐसी संस्था है जिसकी स्थापना सरकार द्वारा ग्रामीण क्षेत्रों के लिए की जाती है।

4. तीन स्तरीय पंचायती राज प्रणाली (Three Tier System of Panchayati Raj):
अनुच्छेद 243 (ख) त्रिस्तरीय पंचायती राज की व्यवस्था करता है। प्रत्येक राज्य में निम्न स्तर पर ग्राम पंचायत, मध्यवर्ती स्तर पर पंचायत समिति तथा सबसे ऊपर जिला स्तर पर जिला परिषद् का गठन किया जाएगा परंतु उस राज्य में जिसकी जनसंख्या 20 लाख से अधिक नहीं है, वहाँ मध्यवर्ती स्तर पर पंचायत समितियों का गठन करना आवश्यक नहीं होगा। प्रत्येक राज्य में पंचायतों की संरचना संबंधी व्यवस्था संबंधित राज्य के विधानमंडल द्वारा की जाएगी।

5. सदस्यों का प्रत्यक्ष चुनाव (Direct Election of Members of Panchayats):
पंचायती राज की नई प्रणाली के अधीन यह व्यवस्था की गई है कि प्रत्येक पंचायत-क्षेत्र को विभिन्न चुनाव क्षेत्रों में बांटा जाएगा और इन चुनाव-क्षेत्रों से पंचायत के सदस्यों का चुनाव प्रत्यक्ष रूप से लोगों द्वारा किया जाएगा। ग्राम स्तर पर पंचायत का अध्यक्ष ऐसी रीति से चुना जाएगा, जो राज्य विधानमंडल द्वारा कानून बनाकर निश्चित की जाएगी। मध्यवर्ती तथा जिला स्तर पर पंचायत के अध्यक्ष का चुनाव उसके सदस्यों द्वारा अपने में से किया जाएगा।

6. पंचायतों के अध्यक्ष को पद से हटाना (Removal of Chairman from office):
73वें संवैधानिक संशोधन में यह व्यवस्था की गई है कि ग्राम स्तर की पंचायत के अध्यक्ष (Sarpanch) को उसके निश्चित कार्यकाल की समाप्ति से पूर्व ग्राम सभा द्वारा उसके पद से हटाया जा सकता है। ग्राम सभा ऐसी कार्यवाही केवल तभी कर सकती है यदि इस संबंध में पंचायत ग्राम सभा को ऐसा करने की सिफारिश करे।

पंचायत द्वारा ऐसी सिफारिश करने के लिए यह आवश्यक है कि पंचायत अपने कुल सदस्यों के बहुमत तथा उपस्थित व मतदान में भाग लेने वाले सदस्यों के बहुमत से अध्यक्ष को हटाने संबंधी प्रस्ताव प्राप्त करे। जब पंचायत का ऐसा प्रस्ताव ग्राम सभा को प्राप्त होगा, तो ग्राम सभा उस पर विचार करने के लिए अपनी एक विशेष बैठक बुलाएगी। ऐसी बैठक 15 दिन की पूर्व सूचना देने के बाद की जाएगी तथा ऐसी बैठक में ग्राम सभा के 50 प्रतिशत सदस्यों की उपस्थिति आवश्यक होगी। यदि ऐसी बैठक में ग्राम सभा अपने उपस्थिति तथा मतदान में भाग लेने वाले सदस्यों के बहुमत द्वारा अध्यक्ष के विरुद्ध प्रस्ताव पास कर देती है, तो ग्राम पंचायत के अध्यक्ष को पद से हटा दिया जाएगा।

मध्यवर्ती स्तर तथा जिला स्तर की पंचायतों के अध्यक्षों (Chairmen of Panchayat Samitis and Zila Parishads) के किसी अध्यक्ष को उसका निश्चित कार्यकाल समाप्त होने से पहले उसके पद से हटाने के लिए यह आवश्यक है कि संबंधित पंचायत इस उद्देश्य के लिए अपने चुने हुए कुल सदस्यों के बहुमत तथा उपस्थित व मतदान में भाग लेने वाले सदस्यों के 2/3 बहुमत के साथ अपने अध्यक्ष के विरुद्ध प्रस्ताव पास करे।

7. विभिन्न प्रकार के व्यक्तियों, अधिकारियों का पंचायतों में प्रतिनिधित्व (Representation of certain categories of Persons and Officials in Panchayats):
73वें संवैधानिक संशोधन द्वारा राज्य विधानमंडल को यह शक्ति दी गई है कि वह भिन्न-भिन्न स्तर की पंचायतों में निम्नलिखित व्यक्तियों को प्रतिनिधित्व दल के संबंध में व्यवस्था कर सकती है-

  • सदन स्तर की पंचायतों के अध्यक्षों (Chairmen of Panchayat Samitis) का जिला स्तर की पंचायतों (Zila Parishad) में प्रतिनिधित्व,
  • ग्राम स्तर की पंचायतों के अध्यक्षों (Sarpanches) का मध्यम स्तर की पंचायतों (Panchayat Samiti) में प्रतिनिधित्व,
  • मध्यम स्तर तथा जिला स्तर की पंचायतों में लोकसभा तथा राज्य विधानसभा के उन सदस्यों को प्रतिनिधित्व जो उनके क्षेत्र में निर्वाचित हुए हों,
  • राज्य सभा तथा राज्य विधान परिषद् के सदस्यों को उन पंचायतों में प्रतिनिधित्व जिस पंचायत के क्षेत्र में उनका नाम मतदाता के रूप में शामिल हो।

यदि उनका नाम मध्यम स्तर की पंचायत के क्षेत्र में मतदाता के रूप में शामिल होगा तथा उस मध्यम स्तर की पंचायत (Panchayat Samiti) में प्रतिनिधित्व दिया जा सकता है और यदि उसका नाम जिला-परिषद् के क्षेत्र में मतदाता सूची में शामिल होगा तो उसे जिला परिषद् में प्रतिनिधित्व दिया जा सकता है। यहाँ पर यह बात ध्यान देने योग्य है कि ग्राम पंचायत में ऐसे सदस्यों को प्रतिनिधित्व नहीं दिया जा सकता है।

8. स्थानों का आरक्षण (Reservation of seats)-नई पंचायती राज प्रणाली में विभिन्न पंचायतों में अनुसूचित जातियों, अनुसूचित जनजातियों और महिलाओं के लिए स्थान आरक्षित करने की व्यवस्था की गई है। इसके लिए संविधान में निम्नलिखित व्यवस्थाएँ की गई हैं-

(1) अनुसूचित जातियों तथा अनुसूचित जनजातियों के लिए प्रत्येक पंचायत में स्थान आरक्षित किए जाएंगे। उनकी संख्या पंचायत के क्षेत्र की जनसंख्या के अनुपात के अनुसार निश्चित की जाएगी,

(2) आरक्षित स्थानों के चुनाव-क्षेत्रों में समय-समय पर परिवर्तन किया जाएगा,

(3) प्रत्येक पंचायत में चुनाव द्वारा भरे जाने वाले स्थान में से 1/3 स्थान महिलाओं के लिए आरक्षित किया जाएगा,

(4) अनुसूचित जातियों तथा अनुसूचित जनजातियों के लिए आरक्षित किए गए स्थानों में से 1/3 स्थान उन जातियों की महिलाओं के लिए आरक्षित किए जाएंगे,

(5) ग्राम स्तर, मध्यम स्तर तथा जिला स्तर की पंचायतों के अध्यक्षों के पदों की अनुसूचित जातियों तथा अनुसूचित जनजातियाँ और महिलाओं के लिए आरक्षित स्थानों की व्यवस्था संबंधित राज्य के विधानमंडल द्वारा कानून पास करके की जाएगी,

(6) पंचायतों के अध्यक्षों के पदों की कुल संख्या का 50% भाग महिलाओं के लिए आरक्षित करने की घोषणा की गई,

(7) ऊपर दी गई व्यवस्थाओं के अतिरिक्त राज्य विधानमंडल कानून द्वारा नागरिकों की किसी अन्य पिछड़ी श्रेणी के लिए पंचायतों में स्थान आरक्षित करने की व्यवस्था कर सकता है।

9. पंचायतों का कार्यकाल (Tenure of Panchayats):
पंचायती राज संस्थाओं का कार्यकाल 5 वर्ष होगा। यह कार्यकाल पंचायत की पहली बैठक की तिथि से आरंभ होगा। किसी भी स्तर की पंचायत का कार्यकाल 5 वर्ष से अधिक नहीं हो सकता।

10. पंचायतों के कार्य (Functions of Panchayats):
संविधान की 11वीं अनुसूची में कुल 29 विषय रखे गए हैं, जिन पर पंचायत कानून बनाकर उन कार्यों को कर सकती है। इनमें से कुछ प्रमुख विषय इस प्रकार हैं

  • कृषि, जिसमें कृषि विस्तार शामिल है,
  • भूमि सुधार, चकबंदी तथा भूमि संरक्षण,
  • लघु सिंचाई तथा जल-प्रबंध,
  • पशु पालन, दुग्ध उद्योग तथा मुर्गी पालन व मत्स्य उद्योग,
  • खादी ग्राम और कुटीर उद्योग,
  • पीने का पानी,
  • ईंधन और चारा,
  • सड़कें, पुल, गोघाट, जलमार्ग तथा संचार के अन्य साधन,
  • गैर पारंपरिक ऊर्जा स्रोत,
  • शिक्षा, जिसमें प्राथमिक तथा माध्यमिक विद्यालय शामिल हैं;
  • तकनीकी तथा व्यावसायिक शिक्षा,
  • पुस्तकालय,
  • बाजार तथा मेले,
  • परिवार कल्याण, स्त्री तथा बाल विकास,
  • समाज के कमजोर वर्गों, अनुसूचित जातियों तथा अनुसूचित जन-जातियों का कल्याण,
  • स्वास्थ्य और स्वच्छता जिसके अंतर्गत अस्पताल तथा प्राथमिक स्वास्थ्य केंद्र और औषधालय आदि शामिल हैं।

11. वित्त आयोग की स्थापना (Establishment of Finance Commission):
73वें संवैधानिक संशोधन द्वारा एक महत्त्वपूर्ण व्यवस्था यह की गई है कि इस संशोधन के लागू होने के एक वर्ष के भीतर राज्य या राज्यपाल एक वित्त आयोग की नियुक्ति करेगा जो निम्नलिखित विषयों पर राज्यपाल को अपनी सिफारिश करेगा-

  • ऐसे करों, शुल्कों तथा फीसों को दर्शाना जो पंचायतों को प्रदान किए जा सकें,
  • राज्य की संचित निधि (Consolidated fund of the State) से पंचायतों के लिए सहायता अनुदान,
  • पंचायतों की वित्तीय स्थिति में सुधार के लिए सुझाव देना, वित्त-आयोग में कितने सदस्य होंगे, उनके लिए कौन-सी योग्यताएँ आवश्यक होंगी, उनका चुनाव किस प्रकार होगा तथा आयोग को क्या शक्तियाँ प्राप्त होंगी, इन सभी बातों के संबंध में व्यवस्था राज्य विधानमंडल के कानून द्वारा की जाएगी।

निष्कर्ष (Conclusion)-73वाँ संवैधानिक संशोधन अधिनियम इस कारण से बहुत महत्त्वपूर्ण है कि इसके अनुसार पंचायती राज की संस्थाओं को संवैधानिक मान्यता प्रदान की गई है। इस अधिनियम के अनुसार पंचायतों का कार्यकाल निश्चित करके उनके चुनावों की निश्चित अवधि पर यह व्यवस्था की गई है कि इन संस्थाओं को अब छः महीने से अधिक समय तक भंग अथवा स्थगित नहीं किया जा सकता। इस एक्ट के पास होने से पहले इन संस्थाओं के चुनाव कई-कई वर्षों तक नहीं होते थे। इसके अतिरिक्त पंचायतों की वित्तीय स्थिति में सुधार करने के लिए उन्हें कर लगाने की शक्ति देना तथा वित्तीय आयोग की स्थापना करना भी पंचायती राज प्रणाली को मजबूत बनाने के महत्त्वपूर्ण कदम हैं।

प्रश्न 5.
ग्राम पंचायत के संगठन एवं कार्यों का वर्णन करें। अथवा ग्राम पंचायत की संरचना, कार्यों तथा आय के साधनों का वर्णन करें।
उत्तर:
ग्राम पंचायत (Gram Panchayat) ग्राम पंचायत ग्रामीण स्थानीय स्वशासन की मुख्य इकाई है। इस समय देश में 2,50,000 के लगभग ग्राम पंचायतें मौजूद हैं। जबकि हरियाणा में 6311 ग्राम पंचायतें वर्तमान में है। रचना (Composition) हरियाणा में पंचायती राज का गठन हरियाणा पंचायती राज अधिनियम, 1994 (Haryana Panchayati RajAct, 1994) के आधार पर किया गया है।

हरियाणा के प्रत्येक गांव जिसकी जनसंख्या 500 या इससे अधिक है, में एक ग्राम सभा (Gram Sabha) की स्थापना की जाती है। इससे कम जनसंख्या वाले गाँव.को इस उद्देश्य से किसी साथ वाले गांव से मिलाकर एक सांझी ग्राम सभा की स्थापना की जाती है। उस क्षेत्र में रहने वाले सभी नागरिक, जिनकी आयु 18 वर्ष अथवा उससे अधिक होती है, ग्राम सभा के सदस्य होते हैं।

हरियाणा में एक ग्राम पंचायत में एक सरपंच (Sarpanch) तथा 6 से 20 तक पंच (Members) होते हैं। यह संख्या गांव की जनसंख्या, के आधार पर निश्चित की जाती है। सरपंच तथा अन्य पंचों (सदस्यों) का चुनाव ग्राम सभा के सदस्यों द्वारा प्रत्यक्ष चुनाव-प्रणाली के आधार पर किया जाता है। योग्यताएँ-ग्राम पंचायत का सदस्य बनने के लिए व्यक्ति में निम्नलिखित योग्यताएँ होनी चाहिए-

  • वह भारत का नागरिक हो,
  • वह 21 वर्ष की आयु पूरी कर चुका हो,
  • वह दिवालिया अथवा पागल न हो,
  • वह किसी अपराध के लिए दण्डित न किया गया हो तथा न्यायालय द्वारा चुनाव लड़ने के अयोग्य घोषित न किया गया हो,
  • वह संसद अथवा राज्य विधानमण्डल का सदस्य न हो,
  • वह कम-से-कम एक वर्ष से उस गाँव में रह रहा हो।

स्थानों का आरक्षण-पंजाब पंचायती राज अधिनियम, 1994 के अनुसार पंचायतों में अनुसूचित जातियों, अनुसूचित जनजातियों (पंजाब में अनुसूचित जनजातियाँ नहीं हैं) महिलाओं तथा अन्य पिछड़े वर्गों के लिए स्थान सुरक्षित रखने की व्यवस्था की गई है। यह आरक्षण इस प्रकार है

(1) अनुसूचित जातियों के लिए प्रत्येक क्षेत्र में इन जातियों की जनसंख्या के अनुपात. के आधार पर स्थान सुरक्षित रखे जाएँगे,

(2) आरक्षित स्थानों की कुल संख्या का कम-से-कम एक-तिहाई (1/3) स्थान अनुसूचित जातियों की महिलाओं के लिए आरक्षित होंगे और ऐसे स्थान विभिन्न आरक्षित वार्डों को चक्रानुक्रम द्वारा तथा लॉटरी द्वारा (By rotation and by lottery) आबंटित किए जाएँगे,

(3) प्रत्येक ग्राम पंचायत में प्रत्यक्ष चुनाव द्वारा भरे जाने वाले स्थानों की कुल संख्या के 1/3 आरक्षित स्थान (अनुसूचित जातियों से सम्बन्धित महिलाओं के लिए आरक्षित स्थानों की संख्या सहित) महिलाओं के

(4) प्रत्येक खण्ड में ग्राम पंचायतों के सरपंचों के कुछ पद अनुसूचित जातियों तथा महिलाओं के लिए आरक्षित रखे जाएँगे।

हरियाणा में जनवरी सन् 2016 में हुए पंचायी राज संस्थाओं के हुए चुनावों से पहले कुछ अन्य योग्यताएं भी जोड़ दी गई हैं। ये इस प्रकार हैं हरियाणा में जनवरी 2016 में हुए पंचायती राज संस्थाओं के चुनावों से पहले हरियाणा विधानसभा में एक संशोधन बिल पास करके इसमें चार शर्ते और जोड़ दी गई हैं जो इस प्रकार हैं-

  • महिलाओं व एस सी (S.C.) वर्ग के लिए शैक्षिक योग्यता (Educational Qualification) 8वीं, बाकी सभी के लिए 10वीं पास, 5वीं पास दलित महिलाएं पंच पद के लिए चुनाव लड़ सकेंगी,
  • पर्चा भरने से पहले प्रत्याशी के घर में टॉयलेट (Toilet) होना जरूरी है,
  • सहकारी लोन, बिजली बिल तथा गृह कर (House Tax) समेत सभी सरकारी देनदारियों का भुगतान हो,
  • 10 वर्ष की सज़ा वाले गंभीर अपराधिक मामलों में चार्जशीट न हो।

कार्यकाल (Tenure):
73वें संवैधानिक संशोधन के अनुसार समस्त भारत में पंचायतों का कार्यकाल 5 वर्ष निश्चित कर दिया गया । है। यदि सरकार की राय में कोई पंचायत अपनी शक्तियों का दुरुपयोग करती है अथवा अपने कर्तव्यों का ठीक प्रकार से पालन नहीं करती, तो सरकार द्वारा उसे भंग किया जा सकता है। किसी पंचायत के भंग होने के छह महीने के अन्दर उसके नए चुनाव करवाने अनिवार्य हैं।

अध्यक्ष तथा उपाध्यक्ष-ग्राम पंचायत का एक अध्यक्ष होता है, जिसे सरपंच (Sarpanch) कहते हैं। उसका चुनाव मतदाताओं द्वारा प्रत्यक्ष रूप से किया जाता है। उसे अपनी अवधि पूरी करने से पहले भी पद से हटाया जा सकता है। सरपंच ग्राम सभा तथा ग्राम पंचायतों की बैठकें बुलाता है तथा उनकी अध्यक्षता करता है। उसकी अनुपस्थिति में ये कार्य उप-सरपंच द्वारा पूरे किए जाते हैं। उप-सरपंच का चुनाव पंचायत के सदस्यों द्वारा किया जाता है।

बैठकें ग्राम पंचायत की एक महीने में कम-से-कम दो बैठकें होनी अनिवार्य हैं। यदि पंचायत के सदस्यों का बहमत लिखित रूप में विशेष बैठक की माँग करता है तो सरपंच द्वारा तीन दिन के अन्दर ऐसी बैठक बुलाना अनिवार्य है। पंचायत की बैठकों के लिए गणपूर्ति (Quorum) (सरपंच सहित कुल सदस्यों का बहुमत) निश्चित की गई है। पंचायत में निर्णय बहुमत से लिए जाएँगे। किसी विषय पर समान मत पड़ने की स्थिति में सरपंच को निर्णायक मत (Casting Vote) देने का अधिकार होगा। ग्राम पंचायत के कार्य-ग्राम पंचायत के कार्यों का वर्णन (73वें संवैधानिक संशोधन, 1992 के अनुसार) संविधान की 11वीं अनुसूची में किया गया है। ये कार्य इस प्रकार हैं

(i) प्रशासनिक कार्य-

  • (1) ग्राम पंचायत ग्राम सभा द्वारा निश्चित किए गए कार्यों को पूरा करेगी,
  • यह क्षेत्र के विकास के लिए बज़ट तैयार करेगी और ग्राम सभा में पेश करेगी तथा विकास की योजनाओं को भी तैयार करेगी। ये कार्य मुख्य रूप से प्रशासनिक हैं।

(ii) कल्याणकारी कार्य-पंचायती राज व्यवस्था का मुख्य उद्देश्य ग्रामीण क्षेत्रों का विकास करना है। इसीलिए ग्राम पंचायत के कल्याणकारी कार्य निश्चित इस प्रकार किए गए हैं-

(1) कृषि के क्षेत्र में कृषि तथा बागवानी का विकास, बंजर भूमि का विकास और चराई भूमि का विकास आदि,

(2) पशुपालन के क्षेत्र में पशु धन में सुधार, डेरी उद्योग, मुर्गी पालन आदि को प्रोत्साहन देना। इसके साथ ही मछली उत्पादन में विकास भी शामिल है,

(3) ग्राम पंचायत का यह भी कार्य है कि वह सड़कों के दोनों ओर वृक्ष लगवाए और वन के विकास में सहयोग दे,

(4) ग्रामीण लोगों को रोजगार देने के लिए कुटीर उद्योगों को प्रोत्साहन देना,

(5) ग्रामीणों के लिए आवास की व्यवस्था करना तथा पीने के पानी के लिए कुओं, तालाबों आदि का निर्माण कराना,

(6) गाँवों में ऊर्जा की योजनाओं का विकास करना और विकसित चूल्हों का प्रचार करना,

(7) शिक्षा के क्षेत्र में लोगों में शिक्षा के प्रसार के लिए वयस्क, औपचारिक और अनौपचारिक शिक्षा का प्रसार करना। पुस्तकालय खोलना और सांस्कृतिक कार्यक्रम आयोजित करना,

(8) ग्रामीण क्षेत्र में सफाई के लिए सड़कें और सार्वजनिक शौचालयों का निर्माण करना तथा इसी प्रकार की अन्य व्यवस्थाएँ करना,

(9) सार्वजनिक स्वास्थ्य के क्षेत्र में परिवार कल्याणकारी कार्यक्रम लागू करना और बीमारियों की रोकथाम के लिए उपाय करना। इसी के साथ महिला, शिशु और दिव्यांगों के कल्याण के लिए भी कार्य करना, (10) अनुसूचित जातियों के कल्याण के लिए भी कार्य करना,

(11) कानून द्वारा अन्य और भी ऐसे कार्य निश्चित किए गए हैं जिससे कि ग्रामीण क्षेत्रों के विकास के साथ चेतना भी जागृत क्षेत्र में यह भी व्यवस्था है कि पंचायत समिति भी ग्राम पंचायत को कोई कार्य सौंप सकती है,

(12) बाढ़, सूखा तथा अन्य संकटकाल की स्थिति में गाँव वालों की सहायता करना,

(13) प्रसूति केन्द्र (Maternity Centre) तथा बाल कल्याण केन्द्र (Child Welfare Centre) खोलना तथा उनका प्रबन्ध करना,

(14) पुलों तथा पुलियों का निर्माण तथा मुरम्मत करवाना,

(15) कुएँ, तालाब आदि बनवाना तथा उनकी रक्षा करना,

(16) श्मशान भूमि तथा कब्रिस्तान की देखभाल करना,

(17) मेलों तथा मण्डियों का आयोजन करना,

(18) सार्वजनिक उद्यान, खेल के मैदान आदि का प्रबन्ध करना तथा सार्वजनिक मनोरंजन के लिए आयोजन करना,

(19) पुस्तकालयों तथा वाचनालयों (Libraries and Reading Rooms) की स्थापना व देखभाल करना,

(20) महामारियों के विरुद्ध निवारणात्मक तथा औपचारिक उपाय करना,

(21) यात्रियों के ठहरने का प्रबन्ध करना,

(22) पंचायत किसी पटवारी, सन्तरी, चौकीदार, टीकाकरण कर्मचारी, कनिष्ठ अभियन्ता या गाँव से सम्बन्धित किसी अन्य सरकारी कर्मचारी के विरुद्ध शिकायत ज़िलाधीश (उपायुक्त) को पहुंचा सकती है।

(i) न्याय सम्बन्धी कार्य-हरियाणा पंचायती राज अधिनियम, 1994 के अनुसार पंचायतों को निम्नलिखित न्याय-सम्बन्धी कार्य सौंपे गए हैं .. यदि कोई व्यक्ति पंचायत के फौजदारी आदेशों का उल्लंघन करता है तो पंचायत को उस पर 100 रुपए तक जुर्माना करने का अधिकार है। यदि कोई व्यक्ति पंचायत के आदेशों का लगातार उल्लंघन करता है तो पंचायत उस पर 10 रुपए प्रतिदिन की दर से जुर्माना कर सकती है जो 1000 रुपए तक हो सकता है। पंचायत के इस निर्णय के विरुद्ध निर्देशक पंचायत’ (Director of Panchayats) के पास 30 दिन के अन्दर अपील की जा सकती है, जिसका निर्णय अन्तिम माना जाएगा।

ग्राम पंचायत की आय के साधन प्रत्येक ग्राम पंचायत के लिए एक ग्राम निधि (Fund) होगी जिसमें से ग्राम पंचायत खर्च कर सकेगी। ग्राम पंचायत की आय के स्रोत इस प्रकार हैं-

  • सरकार या अन्य संस्थाओं द्वारा प्राप्त अनुदान,
  • वसूल किए गए हार, पंचायत भमि आदि से प्राप्त आय,
  • क्षेत्र के कड़ा-कर्कट, पशुओं के शव, गोबर या कड़ा आदि बेचने से आय,
  • ग्राम मछली क्षेत्रों से हुई आय जो सम्बन्धित ग्राम पंचायत के अधीन है,
  • गृह-कर द्वारा आय,
  • ग्राम-कर द्वारा आय,
  • पंचायत द्वारा किए गए जुर्माने से प्राप्त आय,
  • पशु मेलों या अन्य मेलों में दुकानदारों से प्राप्त आय,
  • जनता से प्राप्त अनुदान अथवा ऋण,
  • 73वें संवैधानिक संशोधन के अनुसार राज्य विधानमण्डल पंचायतों की आय में वृद्धि करने हेतु राज्य सरकार द्वारा लगाए गए कुछ करों को भी पंचायतों को सौंप सकता है।

HBSE 11th Class Political Science Important Questions Chapter 8 स्थानीय शासन

प्रश्न 6.
जिला-परिषद् के गठन, कार्यों तथा आय के साधनों का वर्णन करें।
उत्तर:
पंचायती राज के तीनों स्तरों में सबसे ऊपर जिला परिषद् है। प्रत्येक जिले में पंचायत समिति के ऊपर एक ज़िला-परिषद् की स्थापना की जाती है ज़िला-परिषद् का गठन-जिला-परिषद् पंचायती राज की सबसे ऊपर की इकाई है। प्रत्येक जिले में पंचायत समिति के ऊपर जिला परिषद का गठन किया गया है। वर्तमान में हरियाणा में 21 जिला परिषद हैं क्योंकि नवनिर्मित नए चरखी दादरी जिले में जिला परिषद् के गठन की अधिसूचना अभी जारी नहीं हुई है। हरियाणा पंचायती राज अधिनियम, 1994 के अनुसार प्रत्येक जिला परिषद् में प्रत्यक्ष रूप से चुने गए सदस्य होंगे

1. प्रत्यक्ष रूप से चुने गए सदस्य

(1) ज़िला-परिषद् क्षेत्र में रहने वाले लोगों द्वारा प्रत्यक्ष रूप से कुछ सदस्यों का चुनाव किया जाता है। इन सदस्यों की संख्या 10 से लेकर 25 तक हो सकती है, जो जिले की जनसंख्या के आधार पर निश्चित की जाती है। जिस जिला-परिषद् के क्षेत्र की जनसंख्या 5 लाख से अधिक नहीं होगी, उसमें न्यूनतम 10 सदस्य चुने जाएँगे। वह ज़िला-परिषद् क्षेत्र, जिसकी जनसंख्या 12 लाख से अधिक होगी, उसके सदस्यों की संख्या 25 से अधिक नहीं हो सकती,

(2) जिले में गठित सभी पंचायत समितियों के अध्यक्ष,

(3) राज्य विधान सभा तथा लोकसभा के वे सदस्य जिसका चुनाव-क्षेत्र उस जिले में पड़ता है, जिला-परिषद् के सदस्य होंगे,

(4) राज्य सभा तथा राज्य विधान परिषद् क्षेत्र (यदि राज्य में विधान परिषद् हो) यदि उनका नाम ज़िला-परिषद् क्षेत्र के मतदाताओं की सूची में अंकित है, तो वह ज़िला-परिषद् का सदस्य होगा। स्थानों का आरक्षण प्रत्येक जिला परिषद में कुछ स्थान अनुसूचित जातियों, जनजातियों तथा पिछड़े वर्गों के लिए आरक्षित किए जाते हैं। अनुसूचित जातियों तथा पिछड़ी जातियों के लिए प्रत्यक्ष चुनाव द्वारा भरे जाने वाले स्थानों का अनुपात वही होगा जो उस ज़िला-परिषद् क्षेत्र की कुल जनसंख्या का होगा।

2. स्त्रियों के लिए आरक्षण-प्रत्येक ज़िला-परिषद् में प्रत्यक्ष चुनावों से भरे जाने वाले स्थानों की कुल संख्या का 1/3 भाग स्त्रियों के लिए आरक्षित किया जाएगा। स्त्रियों के लिए आरक्षित किए गए स्थानों में अनुसूचित जातियों की स्त्रियों के लिए आरक्षित किए गए स्थान भी शामिल होंगे।

3. पिछड़ी श्रेणियों के लिए आरक्षण-जिस ज़िला-परिषद् क्षेत्र की कुल जनसंख्या का 20 प्रतिशत भाग पिछड़ी श्रेणियों के । लोगों का होगा, उस ज़िला-परिषद् में एक स्थान उन श्रेणियों के लिए भी आरक्षित रखा जाएगा।

कार्यकाल (Tenure):
संविधान के 73वें संशोधन के अनुसार जिला परिषद् का कार्यकाल पाँच वर्ष निश्चित किया गया है। यदि जिला परिषद् का कोई सदस्य बिना सूचना दिए परिषद की लगातार चार बैठकों में अनुपस्थित रहता है तो उसकी ज़िला-परिषद् की सदस्यता समाप्त हो जाएगी। जिला परिषद् को उसकी अयोग्यता, भ्रष्टाचार अथवा अपनी शक्ति के दुरुपयोग के कारण राज्य सरकार द्वारा उसे उसका कार्यकाल समाप्त होने से पहले भी भँग किया जा सकता है।

बैठकें तथा गणपूर्ति ज़िला-परिषद् की तीन मास में कम-से-कम एक बैठक अवश्य होनी चाहिए। इन साधारण बैठकों के अतिरिक्त ज़िला-परिषद् के कुल सदस्यों के एक-तिहाई सदस्यों के लिखित अनुरोध पर जिला परिषद् के अध्यक्ष द्वारा दो सप्ताह के भीतर इसकी विशेष बैठक बुलाई जाएगी। जिला-परिषद् की साधारण बैठक के लिए कुल सदस्य संख्या का 1/3 भाग तथा विशेष बैठक के लिए कुल सदस्यों का 1/2 भाग उपस्थित होना आवश्यक होता है।

अध्यक्ष-जिला-परिषद् के सदस्य अपने में से एक अध्यक्ष और एक उपाध्यक्ष का चुनाव करते हैं। अध्यक्ष ज़िला-परिषद् की बैठकों में सभापतित्व करता है और उनमें अनुशासन बनाए रखता है तथा उसकी कार्रवाई को ठीक प्रकार से चलाने का उत्तरदायी है। अध्यक्ष की अनुपस्थिति में ये कार्य उपाध्यक्ष द्वारा किए जाते हैं।

सचिव जिला परिषद् के निर्णयों को लागू करने तथा इसके दैनिक कार्यों को चलाने के लिए एक सचिव की नियुक्ति की जाती है जो मासिक वेतन लेता है और वह सरकार द्वारा नियक्त किया जाता है तथा अपने कार्यों के लिए जिला परिषद के प्रति उत्तरदायी होता है।

समितियाँ-जिला-परिषद् अपना काम समितियों के माध्यम से करती है। इसकी विभिन्न समितियाँ हैं कार्य समिति, विकास समिति, योजना तथा वित्त समिति, समाज कल्याण समिति, स्वास्थ्य समिति, शिक्षा समिति आदि। इन समितियों का गठन ज़िला-परिषद् आरम्भ में ही कर लेती है।

जिला-परिषद् के कार्य-ज़िला-परिषद् के कार्य इस प्रकार हैं-

  • यह मूल रूप से अपने क्षेत्र में स्थित पंचायत समितियों के कार्यों में समन्वय पैदा करती है और जिले का सामूहिक विकास करने का प्रयत्न करती है,
  • ज़िला-परिषद् पंचायत समितियों के बजट का निरीक्षण करती है और उन पर अपनी स्वीकृति देती है,
  • यह पंचायत समितियों के कार्यों पर निगरानी रखती है, उन्हें परामर्श देती है और आवश्यक हो तो आदेश देती है, ताकि पंचायत समिति अपने कार्य ठीक प्रकार से कर सके,
  • यह दो या दो से अधिक पंचायत समितियों से सम्बन्धित विकास योजनाएँ बनाती है और उन्हें कार्यान्वित करती है,
  • यह पंचायत समितियों की विकास योजनाओं को समस्त जिले की विकास योजनाओं के साथ समन्वित करने का प्रयत्न करती है,
  • यह जिले के ग्रामीण विकास के सम्बन्ध में सरकार को सुझाव दे सकती है,
  • सरकार किसी भी योजना को लागू करने का उत्तरदायित्व जिला परिषद् को सौंप सकती है,
  • ज़िला-परिषद् ग्रामीण जीवन को विकसित करने और उनके जीवन-स्तर को ऊँचा उठाने के लिए अनिवार्य कदम उठा सकती है।

ज़िला-परिषद् की आय के साधन (Sources of Income of Zila Parishad):
जिला-परिषद् को जिन साधनों से आय प्राप्त होती है वे इस प्रकार हैं-

  • स्थानीय करों (Local Taxes) का कुछ भाग ज़िला-परिषद् को मिलता है,
  • ज़िला-परिषद् सरकार की आज्ञा से कुछ धन पंचायतों से ले सकती है,
  • राज्य सरकार जिला परिषद् को सहायता के रूप में प्रतिवर्ष कुछ धन देती है,
  • ज़िला-परिषद् को अपनी सम्पत्ति से बहुत-सी आय होती है,
  • ज़िला-परिषद् सरकार की अनुमति से ब्याज पर धन उधार भी ले सकती है,
  • केन्द्रीय सरकार द्वारा ज़िला-परिषदों के लिए निश्चित किया गया धन,
  • कुटीर व लघु उद्योगों की उन्नति के लिए अखिल भारतीय संस्थाओं द्वारा दिए गए अनुदान और अन्य विशेष योजनाओं के लिए समय-समय पर मिलने वाले अनुदान।

प्रश्न 7.
भारत में पंचायती राज की कार्य-प्रणाली में मौजूद त्रुटियों पर एक लेख लिखिए।
अथवा
पंचायती राज से सम्बन्धित मुख्य समस्याओं का वर्णन कीजिए। साथ ही यह भी बताइए कि पंचायती राज की समस्याओं को कैसे दूर किया जा सकता है?
अथवा
भारत की पंचायती राज प्रणाली को सफलतापूर्वक चलाने के मार्ग में आने वाली समस्याओं की चर्चा करें।
उत्तर:
इसमें सन्देह नहीं है कि पंचायती राज की स्थापना से कई लाभ हुए हैं, परन्तु इस व्यवस्था में अनेक त्रुटियाँ (कमियाँ) हैं, जिनके कारण भारत में पंचायती राज अधिक सफल नहीं हो पाया है। ये त्रुटियाँ निम्नलिखित हैं

1. अनपढ़ता-स्वतन्त्रता-प्राप्ति के 70 वर्षों के बाद भी निरक्षरता साक्षरता में नहीं बदली जा सकी है। गाँव के प्रौढ़ व्यक्ति तो विशेषकर अनपढ़ हैं। उन्हें पंचायती राज के उद्देश्यों और कार्य-प्रणाली का ज्ञान नहीं है। कई जगह तो ग्राम सभा की बैठकें होती ही नहीं, वैसे ही कागजों में दिखा दी जाती हैं। कुछ पढ़े-लिखे और चालाक व्यक्ति अधिकाँश अनपढ़ लोगों को बेवकूफ बनाकर पंचायतों को अपने लाभ के लिए चलाते रहते हैं और अधिकाँश जनता तक उसके लाभ नहीं पहुँचते।

2. राजनीतिक दलों का हस्तक्षेप-पंचायती राज एक ऐसी व्यवस्था है, जिसमें राजनीतिक दलों की कोई आवश्यकता नहीं, जिसमें गाँव के सभी लोग एक मंच पर इकट्ठे होकर समस्याओं पर विचार करें तथा निर्णय करें और उस निर्णय को लागू करें। परन्तु राजनीतिक दल अपनी हितपूर्ति के लिए गाँव के लोगों को एक मंच पर इकट्ठा नहीं होने देते, उन्हें जाति, धर्म आदि के आधार पर बाँटे रखते हैं, उन्हें लड़ाते रहते हैं और साम्प्रदायिकता तथा गुटबन्दी की भावना को बढ़ाकर शक्ति निर्माण कार्यों में नहीं लगने देते। इससे पंचायती राज का उद्देश्य पूरा नहीं हो पाता और प्रगति भी धीमी पड़ती है।

3. धन का अभाव पंचायती राज की संस्थाओं के अपनी आय के साधन सीमित हैं, उनके पास धन की कमी रहती है, वे स्वतन्त्रतापूर्वक अपनी इच्छानुसार धन इकट्ठा नहीं कर सकतीं। परिणाम यह होता है कि वे अपनी योजनाओं को लागू करने, ग्रामीण जीवन को विकसित करने, उनके जीवन को सुविधाजनक बनाने के काम नहीं कर पातीं। बिना धन के कौन-सी योजना लागू की जा सकती है? बहुत-से गाँवों में तो सड़कों और नालियों की व्यवस्था भी पंचायत नहीं कर पाती, क्योंकि इसके लिए उसके पास धन नहीं है।

4. सरकार का अत्यधिक नियन्त्रण-पंचायती राज जिस भावना से लागू होना चाहिए, वह भावना इसमें विद्यमान नहीं है। पंचायती राज एक प्रकार से लोगों पर थोपा गया है और सरकार पंचायती राज की संस्थाओं पर बहुत अधिक नियन्त्रण रखती है। जिले के उपायुक्त महोदय पंचायती राज संस्थाओं को अनावश्यक आदेश देता है। यहाँ तक कि इन संस्थाओं के अनेक निर्णयों को निरस्त कर देता है।

वास्तव में सरकार की भूमिका तो उन्हें सलाह तथा सहायता देने की होनी चाहिए, आदेश देने की नहीं, परन्तु व्यवहार में ये संस्थाएँ भी सरकार के नियन्त्रण में काम करती हैं और सरकार इन्हें आदेश तथा निर्देश अधिक देती है। इस कारण ग्रामीण यह महसूस नहीं करते कि यह उनकी अपनी स्वतन्त्र संस्था है। यद्यपि 1994 के हरियाणा पंचायती राज अधिनियम के पास होने के बाद सरकारी हस्तक्षेप में कमी तो आई है, फिर भी सरकारी नियन्त्रण से इन संस्थाओं को अभी और स्वतन्त्रता की आवश्यकता है।

5. जातिवाद तथा सम्प्रदायवाद-ग्रामीण लोगों में निर्धनता तथा निरक्षरता के कारण जातिवाद तथा साम्प्रदायिकता की दुर्भावना बहुत अधिक मात्रा में पाई जाती है। ये भावनाएँ गाँव के लोगों में भाई-चारा, आपसी सहयोग तथा प्रेम की भावनाओं को जागृत नहीं होने देतीं। पंचायती राज संस्थाओं के चुनाव जाति-भेद के आधार पर लड़े जाते हैं और चुने जाने के बाद भी सदस्यों के बीच ये भेदभाव बने रहते हैं।

पंचायती राज संस्थाओं के सदस्य जातिवाद के आधार पर आपस में लड़ते-झगड़ते रहते हैं, जिससे पंचायत का काम ठीक प्रकार से नहीं हो सकता। प्रायः पंचायती राज संस्थाओं में आर्थिक दृष्टि से सम्पन्न तथा उच्च जाति के लोगों का ही प्रभुत्व रहता है। जाति, धर्म तथा कई बार दलों के आधार पर भी इसके सदस्यों में गुटबन्दी (Groupism) बनी रहती है, जिससे संस्था का काम ठीक प्रकार से नहीं चल पाता।

6. निर्धनता-गाँवों में रहने वाले लोगों की गरीबी भी पंचायती राज की एक मुख्य समस्या है। गरीब व्यक्ति दिन भर अपना तथा अपने परिवार का पेट भरने की चिन्ता में रहता है और उसके लिए सार्वजनिक मामलों अथवा गाँव की समस्याओं के बारे में सोचने का समय ही नहीं होता। वे ग्राम पंचायत की कार्यविधि में उत्साहपूर्वक भाग नहीं ले सकते।

इसके अतिरिक्त गरीबी के कारण लोग कर नहीं दे सकते जिससे पंचायतें नए कर लगाकर अपनी आमदनी को बढ़ा नहीं सकतीं। भारत में निर्धनता का मुख्य कारण संपत्ति का असमान वितरण है, जिसके कारण अमीरों की आय में तेजी से वृद्धि हो रही है और निर्धन निर्धनता की जंजीरों में ही जकड़े हुए हैं। निर्धनों की संपत्ति कच्चे मकान, बर्तन और पशु ही हैं, जिसके कारण उनके लिए आय के नए स्रोत प्राप्त करना असंभव है।

7. सरकारी कर्मचारियों की भूमिका सरकारी कर्मचारियों की भूमिका ने भी, जोकि बड़ी निराशाजनक तथा आश्चर्यजनक है, पंचायती राज की प्रगति को धीमा कर रखा है। पंचायत समिति के निर्णयों को लागू करने के कार्य जिन कर्मचारियों के जिम्मे हैं, वे सरकार के अधीन हैं और वे पंचायत समिति को अपना स्वामी न मानकर उसे आदेश देने का प्रयत्न करते हैं। सरकारी कर्मचारी समितियों तथा पंचायतों को स्वतन्त्रतापूर्ण काम नहीं करने देते, जिससे कि पंचायती राज के कार्य में रुकावट आती है।

8. ग्राम सभा की प्रभावहीन भूमिका-ग्राम सभा, जिसे गाँव की संसद भी कहा जाता है, आजकल नाममात्र की बनकर रह गई है। कहने को तो इसकी वर्ष में दो बैठकें अवश्य होनी चाहिएँ और बजट आदि इसके द्वारा पास होना चाहिए, परन्तु वास्तव में ऐसा नहीं है या तो इसकी बैठकें होती ही नहीं और हो भी तो ग्रामवासी इनमें रुचि नहीं लेते। केवल ग्राम पंचायत के चुनाव के समय ही इसके अस्तित्व का पता चलता है। इसके बाद तो पंचायत ही जो चाहे करती चली जाती है। इसकी बैठकों की कार्रवाई कागजों में ही लिख ली जाती है।

निष्कर्ष-आज देश में पंचायती राज की स्थिति सराहनीय नहीं है। यह जनता का राज है और लोगों को अपनी स्थानीय आवश्यकताएँ पूरी करने तथा विकास कार्य करने की पूर्ण स्वतन्त्रता है। परन्तु इसने बड़ी धीमी गति से कार्य किया है। जब तक इसमें पाई जाने वाली कमियों को दूर नहीं किया जाएगा, तब तक यह अपना प्रभाव नहीं दिखा सकती।

पंचायतों तथा पंचायत समितियों के पास अपना स्वतन्त्र पर्याप्त धन होना चाहिए और सरकार का इन पर नियन्त्रण कम होना चाहिए तथा सरकार इन्हें सहायता और सलाह देने का कार्य करे, न कि आदेश देने और इसके कार्य में रुकावट लाने का। आशा है कि इन सुविधाओं के परिणामस्वरूप पंचायती राज अपना प्रभाव अवश्य दिखाएगा और भारत के ग्रामीण जीवन का विकास होगा। त्रुटियों को दूर करने के उपाय-पंचायती राज व्यवस्था की त्रुटियों को दूर करने के लिए निम्नलिखित सुझाव दिए जा सकते हैं

1. शिक्षा का प्रसार-पंचायती राज संस्थाओं को सफल बनाने के लिए प्रथम आवश्यकता शिक्षा का प्रसार है। जब तक लोग शिक्षित नहीं होंगे और अपने अधिकारों तथा कर्त्तव्यों के प्रति सजग नहीं होंगे, तब तक ये संस्थाएँ सफलतापूर्वक काय

2. सदस्यों को प्रशिक्षण आज पंचायती राज संस्थाओं का कार्य बहुत अधिक पेचीदा तथा जटिल है। प्रशिक्षण (Training) के बिना पंचायती राज संस्थाओं के सदस्य अपना कार्यों को ठीक प्रकार से नहीं कर सकते और न ही उचित निर्णय ले सकते हैं। इस कारण इन सदस्यों के प्रशिक्षण की पूरी व्यवस्था होनी चाहिए।

3. योग्य और ईमानदार कर्मचारी स्थानीय स्वशासन संस्थाओं के कर्मचारियों की नियुक्ति योग्यता के आधार पर होनी चाहिए। उनके वेतन आदि अन्य सरकारी कर्मचारियों के बराबर होने चाहिएँ, ताकि योग्य व्यक्ति इन संस्थाओं में नौकरी प्राप्त करने के लिए आकर्षित हों। इन कर्मचारियों के प्रशिक्षण की ओर भी ध्यान देना आवश्यक है।

4. राजनीतिक दलों पर रोक स्थानीय स्वशासन की संस्थाओं में राजनीतिक दलों के लिए कोई स्थान नहीं है। यह एक सर्वमान्य तथ्य है। वैसे तो कानून द्वारा उन दलों पर कुछ एक प्रतिबन्ध लगाए हुए हैं, परन्तु व्यावहारिक रूप में ये प्रतिबन्ध निष्प्रभावी सिद्ध हए हैं। इस कप्रथा को रोकने के लिए कानून पर अधिक निर्भर रहना ठीक नहीं है, बल्कि लोगों की सावधानी, दलों की सद्भावना तथा शुभ नीति ही अधिक उपयोगी हो सकते हैं।

5. ग्राम सभा को सक्रिय करना चाहिए- पंचायती राज को सफल बनाने के लिए यह आवश्यक है कि ग्राम सभा को एक प्रभावशाली संस्था बनाया जाए। इसकी बैठकें नियमित रूप से होनी चाहिएँ और गाँव के नागरिकों को इसकी कार्रवाइयों में रुचि लेनी चाहिए। जब तक गाँव के नागरिक स्वयं अपनी समस्याओं तथा अपने क्षेत्र के प्रति उत्साह नहीं दिखाएँगे, तब तक पंचायती राज का सफल होना असम्भव है।

6. सरकारी हस्तक्षेप में कमी स्थानीय संस्थाओं को अपने कार्यों में अधिक-से-अधिक स्वतन्त्रता मिलनी चाहिए, तभी ये संस्थाएँ सफल हो सकती हैं। सरकारी हस्तक्षेप या नियन्त्रण की बजाय इन संस्थाओं को उचित परामर्श (Proper Guidance) की अधिक आवश्यकता है। इसलिए सरकार को चाहिए कि इन संस्थाओं को अधिक स्वतन्त्रता दे और इनके पथ-प्रदर्शन की व्यवस्था करे।

7. धन की अधिक सहायता-धन का अभाव भी इन संस्थाओं की सफलता में एक बहुत बड़ी बाधा है। सरकार को चाहिए कि इन संस्थाओं की धन से पूरी-पूरी सहायता करे। लोगों के जीवन में सुधार करने के लिए ये संस्थाएँ जो नई योजनाएँ बनाएँ, सरकार को चाहिए कि उसके लिए पर्याप्त मात्रा में धन दे।

8. नियमित चुनाव पंचायती राज संस्थाओं के चुनाव निश्चित अवधि के पश्चात् नियमित रूप से होने चाहिएँ। फरवरी, 1989 में तत्कालीन राष्ट्रपति श्री वेंकटरमन ने यह सुझाव दिया था कि पंचायत के चुनावों को राज्यों की विधान सभाओं तथा लोकसभा के चुनावों की तरह अनिवार्य बना दिया जाए और इसके लिए संविधान में संशोधन कर दिया जाए। पंचायती राज संस्थाओं के चुनाव आयोग की देख-रेख में होने चाहिएँ।

9. पंचायती राज की संस्थाओं को अधिक शक्तियाँ देनी चाहिएँ पंचायती राज की संस्थाओं को अधिक दायित्व तथा शक्तियाँ मिलनी चाहिएँ। इसी उद्देश्य की पूर्ति के लिए 23 दिसम्बर, 1992 को संविधान का 73वाँ संशोधन पारित किया गया है, जिसके अन्तर्गत अब पंचायती राज की संस्थाओं को अधिक शक्तियाँ प्राप्त हो गई हैं।

HBSE 11th Class Political Science Important Questions Chapter 8 स्थानीय शासन

प्रश्न 8.
भारत में पंचायती राज के महत्त्व की विवेचना कीजिए। अथवा पंचायती राज की स्थापना से भारत में गाँवों में रहने वाले लोगों को क्या-क्या लाभ हुए हैं?
उत्तर:
2 अक्तूबर, 1959 को राजस्थान में पंचायती राज का उद्घाटन करते हुए भारत के प्रथम प्रधानमन्त्री पं० जवाहरलाल नेहरू ने कहा था, “हम भारत के लोकतन्त्र की आधारशिला रख रहे हैं।” वास्तव में पंचायती राज की स्थापना से प्रजातन्त्र का विकेन्द्रीयकरण होता है और देश का प्रत्येक साधारण-से-साधारण नागरिक भी देश के प्रशासन में स्वयं को भागीदार समझने लगता है। अतः पंचायती राज का बहुत महत्त्व है। निम्नलिखित तथ्यों से पंचायती राज के महत्त्व का पता चलता है

1. प्रत्यक्ष लोकतन्त्र-गाँवों में पंचायती राज की स्थापना का होना प्रत्यक्ष लोकतन्त्र से कम नहीं है, क्योंकि गाँवों के सब लोग ग्राम सभा के सदस्य के रूप में, जिनकी बैठक एक वर्ष में दो बार बुलाई जाती है, गाँव की समस्याओं पर विचार करते हैं और उनका निर्णय करते हैं। ग्राम सभा गाँव की संसद (Parliament) की तरह है और गाँव के सभी नागरिक इस संसद के सदस्य हैं। पंचायती राज के वे सब लाभ हैं जो प्रत्यक्ष लोकतन्त्र से प्राप्त होते हैं। पंचायती राज की स्थापना से प्रत्यक्ष और वास्तविक लोकतन्त्र की स्थापना होती है।

2. जनता का अपना शासन-पंचायती राज जनता का अपना राज्य होता है। इसकी स्थापना से भारत के गाँवों में लोगों का अपना शासन स्थापित हो चुका है। गाँव का प्रत्येक वयस्क नागरिक ग्राम सभा का सदस्य है और ग्राम पंचायत के चुनाव में भाग लेता है। ग्राम पंचायत ग्राम के प्रशासन सम्बन्धी, समाज कल्याण सम्बन्धी, विकास सम्बन्धी तथा न्याय सम्बन्धी सब कार्य करती है।

इस प्रकार गाँव का व्यक्ति अपने भविष्य का स्वयं निर्माता बना हुआ है। भारत का नागरिक अब किसी के अधीन नहीं है और वह वास्तविक रूप में अपने ऊपर स्वयं शासन करता है और अपने शासन से अच्छा कोई शासन नहीं होता।

3. अधिकारों और कर्तव्यों का ज्ञान पंचायती राज की स्थापना से गाँव का प्रत्येक व्यक्ति स्वयं को गाँव के प्रशासन के साथ सम्बन्धित समझता है। आज के ग्रामीण को अपने कर्तव्यों के बारे में जानकारी है तथा वे अपने अधिकारों के बारे में पहले की अपेक्षा अधिक सचेत हैं। गाँव के निवासी अब ग्राम सभा की बैठकों में भाग लेकर ग्राम पंचायत आदि के चुनाव में दिलचस्पी लेते हैं और ग्राम पंचायतों द्वारा निश्चित ‘करों’ इत्यादि को देने से भी हिचकते नहीं हैं।

4. आत्म-निर्भरता-पंचायती राज का एक महत्त्वपूर्ण उद्देश्य ग्रामों को आत्म-निर्भर बनाना है। ग्राम पंचायत व पंचायत समितियाँ जनता की अपनी संस्थाएँ हैं और वे स्वयं आवश्यकतानुसार कर आदि लगाकर अपनी जरूरतों को पूरा करती हैं तथा अपने विकास की योजनाएँ बनाती हैं और उन्हें लागू करती हैं।

पंचायती राज से पहले ग्रामीण विकास की योजनाएँ सरकारी कर्मचारियों द्वारा बनाई जाती थीं और सरकारी कर्मचारी स्वेच्छा से उन्हें लागू करते थे, परन्तु अब यह लोगों की अपनी इच्छा पर निर्भर है। पंचायती राज की स्थापना के बाद गाँव के लोग अपनी विकास योजनाओं को स्वयं लागू करते हैं और इस प्रकार से हमारे बहुत-से गाँव आत्मनिर्भरता प्राप्त कर चुके हैं।

5. स्थानीय समस्याओं का समाधान-किसी विशेष क्षेत्र की स्थानीय समस्याओं का समाधान केन्द्र अथवा राज्य सरकार द्वारा शीघ्रता एवं सुगमता से नहीं किया जा सकता। इसका मुख्य कारण यह है कि सरकार के अधिकारियों का गाँव के लोगों के साथ सीधा सम्पर्क नहीं होता। पंचायती राज संस्थाओं के सदस्यों का जनता के साथ सीधा सम्पर्क होता है। वे क्षेत्र की समस्याओं से भली-भाँति परिचित होते हैं। अतः इस सदस्यों द्वारा स्थानीय समस्याओं को आसानी से हल किया जा सकता है।

6. स्वतन्त्रता की भावना का विकास-पंचायती राज की स्थापना से लोगों में स्वतन्त्रता की भावना का बहुत विकास हुआ है, जिससे देश में लोकतन्त्र की जड़ों को मजबूत करने में सहायता मिली है। इस सम्बन्ध में पंचायती राज के महत्त्व का वर्णन करते हुए स्वर्गीय प्रधानमन्त्री श्रीमती इन्दिरा गाँधी ने एक बार कहा था, “हमारे देश के लोकतन्त्र की बुनियाद मज़बूत होने में पंचायती राज संस्थाओं का प्रमुख स्थान रहा है। इन पर जो बड़ी आशाएँ रखी गई हैं वे पूरी हो सकती हैं जबकि गाँवों में रहने वाले कमज़ोर वर्ग के लोग, ग्रामीण, स्वायत्तता शासन में सक्रिय रूप से भाग लें।”

7. न्यायिक कार्य का महत्त्व पंचायती राज की संस्थाएँ गाँवों के लोगों को सस्ता तथा शीघ्र न्याय दिलाने में बहुत महत्त्वपूर्ण भूमिका निभा रही हैं। साधारण झगड़ों को सुलझाने के लिए ग्रामीण लोगों को न्यायालयों में नहीं जाना पड़ता। इससे धन तथा समय दोनों की बहुत बचत होती है।

8. कृषि का विकास व आर्थिक उन्नति-पंचायती राज की स्थापना का एक और महत्त्वपूर्ण पहलू यह है कि इसकी स्थापना के बाद से ग्रामों में कृषि का बहुत अधिक विकास हुआ है। उसका कारण यह है कि ग्राम पंचायतें, पंचायत समितियाँ व जिला परिषदें सरकारी अधिकारियों व कर्मचारियों की सहायता से किसानों को अच्छे किस्म के बीज प्रदान करती रहती हैं व वैज्ञानिक कृषि तथा नवीनतम कृषि-यन्त्रों के बारे में जानकारी देती रहती है।

परिणामस्वरूप कृषि की उपज बढ़ी है। कृषि की उपज बढ़ने से जहाँ एक ओर किसानों व गाँव के अन्य लोगों की आर्थिक व सामाजिक रूप से उन्नति हुई है, वहाँ दूसरी ओर देश अनाज के मामले में आत्मनिर्भर बन गया है। यह पंचायती राज का ही महत्त्व है।

9. प्रशासन की शिक्षा भारत में लोकतन्त्रात्मक सरकार की स्थापना की गई है। इस प्रकार की सरकार में प्रत्येक व्यक्ति को प्रशासन की ज़िम्मेदारी सम्भालने के लिए तैयार रहना चाहिए। पंचायती राज गाँव के अशिक्षित व कम शिक्षा प्राप्त लोगों को प्रशासन की शिक्षा देने का सबसे अच्छा साधन है। पंचायती राज द्वारा स्थापित स्थानीय स्वशासन शहरों की स्थानीय स्वशासन संस्थाओं की अपेक्षा लोगों को प्रशासन की अधिक शिक्षा प्रदान करता है।

जैसे कि नगरों की स्थानीय संस्थाएँ नियम बनाने का प्रशिक्षण तो लोगों को देती हैं, लेकिन न्याय करने का प्रशिक्षण उन संस्थाओं के सदस्यों को नहीं मिलता। पंचायती राज गाँव के लोगों को नियम बनाने, उन्हें लागू करने, विकास योजनाएँ बनाने तथा उन्हें लागू करने और न्याय करने का भी अवसर देता है। इस प्रकार पंचायती राज द्वारा गाँव के प्रत्येक व्यक्ति को प्रशासन की शिक्षा प्राप्त होती है।

निष्कर्ष इस प्रकार हम कह सकते हैं कि पंचायती राज लोगों में आत्मविश्वास तथा आत्मनिर्भरता की भावना पैदा करता है और उनकी स्थानीय मामलों में रुचि उत्पन्न करता है। यह ग्रामीण क्षेत्रों में रहने वाले लोगों में स्वतन्त्रता तथा उत्पन्न करता है। इससे ग्रामीण जीवन के विकास में बहुत सहायता मिलती है। लोगों को अपने अधिकारों तथा कर्तव्यों के बारे में जागरूक करके पंचायती राज लोकतन्त्र को सफल बनाने में बहुत ही महत्त्वपूर्ण भूमिका निभाता है।

प्रश्न 9.
शहरी स्वशासन संस्थाओं से सम्बन्धित 74वें संवैधानिक संशोधन की मुख्य धाराओं का वर्णन करें। अथवा 74वें संविधान संशोधन द्वारा शहरी स्थानीय संस्थाओं में किए गए परिवर्तनों का वर्णन करें।
उत्तर:
देश में शहरी स्थानीय संस्थाओं को अधिक मजबूत बनाने और उनमें लोगों की भागीदारी को बढ़ाने के लिए भारतीय संसद द्वारा सन् 1992 में 74वाँ संवैधानिक संशोधन पास किया गया जो 1993 में लागू हुआ। इस संवैधानिक संशोधन के मुख्य अनुच्छेद निम्नलिखित हैं-

1. स्थानीय संस्थाओं की संवैधानिक मान्यता-74वें संशोधन द्वारा स्थानीय नगर संस्थाओं को पहली बार संवैधानिक मान्यता प्रदान की गई है। इस संशोधन द्वारा संविधान में भाग 9A शामिल किया गया तथा 12वीं सूची इसमें जोड़ी गई जिसमें 18 विषय शामिल हैं। संविधान के इस भाग में शहरी स्थानीय संस्थाओं की संरचना, कार्य, शक्तियों आदि का उल्लेख है।

2. तीन प्रकार की स्थानीय संस्थाओं का वर्णन-74वें संविधान संशोधन ने प्रत्येक राज्य के लिए तीन स्तरीय शहरी स्थानीय संस्थाओं की व्यवस्था की है; जैसे-

  • परिवर्तनीय क्षेत्र के लिए नगर पंचायत-नगर पंचायत 10,000 से 20,000 तक की जनसंख्या के क्षेत्र में होगी। ये ऐसे क्षेत्र हैं जो ग्रामीण क्षेत्र से शहरी क्षेत्र में परिवर्तित हो रहे हैं,
  • छोटे नगरीय क्षेत्रों के लिए नगरपालिका,
  • विशाल शहरी क्षेत्रों के लिए नगर-निगम। यहाँ पर स्पष्ट करना आवश्यक है कि इन संस्थाओं का निर्णय राज्य सरकार द्वारा किया जाएगा।

3. नगरपालिका का संगठन प्रत्येक नगरपालिका का क्षेत्र राज्य सरकार द्वारा निश्चित किया जाएगा। नगरपालिका को वार्डो में विभाजित किया जाएगा और प्रत्येक वार्ड से प्रत्यक्ष रूप से मतदाता एक प्रतिनिधि को निर्वाचित करेंगे। नगरपालिका के चुनाव सम्बन्धी विषय राज्य के विधानमण्डल द्वारा निश्चित किए जाएंगे।

4. विशेष प्रतिनिधित्व की व्याख्या इस संशोधन के द्वारा यह भी व्यवस्था है कि राज्य का विधानमण्डल विशिष्ट व्यक्तियों, क्षेत्र से सम्बन्धित लोकसभा, विधानसभा और राज्यसभा के सदस्यों को भी नगरपरिषद् में शामिल कर सकता है।

5. अध्यक्ष या सभापति का चुनाव इसकी व्यवस्था राज्य का विधानमण्डल कानून द्वारा करेगा।

6. वार्ड समितियों की व्यवस्था इस संशोधन के द्वारा यह भी व्यवस्था की गई है कि जिस नगरपरिषद् के क्षेत्र में तीन लाख या इससे अधिक जनसंख्या होगी, वहाँ वार्ड समितियाँ स्थापित की जाएँगी। इनका संगठन और अधिकार-क्षेत्र राज्य विधानमण्डल के कानून द्वारा किया जाएगा।

7. आरक्षण की व्यवस्था-74वें संविधान संशोधन ने शहरी स्थानीय संस्थाओं में स्थानों के लिए आरक्षण की निम्नलिखित व्यवस्था की है-

  • प्रत्येक नगरपालिका में जनसंख्या के अनुपात के अनुसार अनुसूचित जातियों और जनजातियों के स्थान सुरक्षित होंगे। इन स्थानों में से 1/3 स्थान अनुसूचित जाति की महिलाओं के लिए सुरक्षित किए जाएँगे,
  • नगरपालिका के कुल स्थानों में 50% स्थान महिलाओं के लिए सुरक्षित करने की घोषणा की गई,
  • पिछड़ी जाति के लिए भी एक स्थान सुरक्षित करने की व्यवस्था है,
  • इस संशोधन द्वारा यह भी व्यवस्था की गई है कि राज्य का विधानमण्डल प्रधान पदों के लिए अनुसूचित जातियों, जनजातियों और महिलाओं के लिए स्थान आरक्षित करेगा।

8. अवधि-शहरी स्थानीय संस्थाओं का कार्यकाल 5 वर्ष निश्चित किया गया है। यदि निश्चित अवधि से पूर्व नगरपालिका भंग कर दी जाती है तो उसके चुनाव छह महीने के अन्दर करवाने आवश्यक हैं।

9. सदस्यो की योग्यताएँ-शहरी स्थानीय संस्थाओं का सदस्य होने के लिए अन्य योग्यताओं के साथ 21 वर्ष की आय आवश्यक है।

10. नगरपालिका की शक्तियाँ-74वें संविधान संशोधन द्वारा 18 विषयों पर नगरपालिका को कार्य करने का अधिकार दिया गया है। इन 18 विषयों का उल्लेख संविधान की 12वीं सूची में किया गया है। इसके साथ यह भी व्यवस्था है कि राज्य का विधानमण्डल नगरपालिका को कुछ ‘कर’ लगाने और उन्हें इकट्ठे करने का भी अधिकार दे सकता है।

11. चुनाव आयोग की व्यवस्था-शहरी क्षेत्र की स्थानीय संस्थाओं के चुनाव राज्य के चुनाव आयोग द्वारा करवाए जाएँगे। राज्य चुनाव आयुक्त की नियुक्ति राज्य के राज्यपाल द्वारा होगी।

12. वित्त आयोग की व्यवस्था शहरी स्थानीय संस्थाओं के गठन के बाद राज्य सरकार एक वित्तीय आयोग नियुक्त करेगी जो इन संस्थाओं की वित्त सम्बन्धी विषयों पर विचार करेगा।

13. जिला आयोजन समिति-74वें संशोधन में यह भी व्यवस्था है कि राज्य विधानमण्डल कानून द्वारा जिला आयोजन समिति की स्थापना करेगा। इसका संगठन भी राज्य विधानमण्डल द्वारा ही निश्चित किया जाएगा। जिला आयोजन समिति का कार्य ज़िला पंचायतों और नगरपालिकाओं द्वारा तैयार की गई योजनाओं को समन्वित करके पूरे जिले के विकास के लिए कार्य करना है।

14. महानगर योजना समिति-74वें संविधान संशोधन द्वारा यह भी व्यवस्था की गई है कि ऐसे क्षेत्रों में, जिनकी जनसंख्या 20 लाख अथवा उससे अधिक है, महानगर योजना समिति की स्थापना की जाए। यह समिति महानगर के सम्पूर्ण क्षेत्र के विकास के लिए योजनाएँ तैयार करेगी।

15. नगरपालिकाओं को कर लगाने की शक्ति और निधियाँ-

  • राज्य विधानमण्डल कानून द्वारा नगरपरिषदों को कर (Tax) शुल्क (Duty), पथ-कर (Toll-tax) तथा फीसें (Fees) लगाने की शक्ति प्रदान कर सकता है।
  • राज्य विधानमण्डल राज्य सरकार द्वारा लगाए गए करों, शुल्कों, पथ-करों और फीसों को नगरपालिका को सौंप सकती है।
  • राज्य की संचित निधि में से (Consolidated Fund of State) नगरपालिकाओं को सहायता अनुदान (Grant-in-aid) दिया जा सकता है।
  • राज्य विधानमण्डल कानून द्वारा प्राप्त होने वाले धनों (Moneys) के लिए निधियों (Funds) और उन निधियों से निकाले जाने वाले धनों की व्यवस्था कर सकता है।

16. नगरपालिकाओं के चुनाव-सम्बन्धी मामले न्यायालय के क्षेत्राधिकार से बाहर-74वें संवैधानिक संशोधन अधिनियम के अनुसार यह व्यवस्था की गई है कि नगरपालिकाओं के चुनाव-क्षेत्र के परिसीमित (Delimitation of constituencies) या ऐसे चुनाव-क्षेत्रों के बँटवारे से सम्बन्धित किसी कानून को न्यायालय में चुनौती नहीं दी जा सकेगी। नगरपालिका के किसी सदस्य के चुनाव को उसी प्राधिकारी के पास चुनौती दी जा सकती है जिसकी व्यवस्था राज्य विधानमण्डल द्वारा कानून बना कर की गई हो।

17. वर्तमान कानूनों तथा नगरपालिकाओं का जारी रहना-74वें संवैधानिक संशोधन में यह व्यवस्था की गई है कि नगरपालिकाओं से सम्बन्धित कोई भी कानून और नगरपालिकाएँ, जो 74वें संवैधानिक संशोधन के लागू होने से पहले अस्तित्व में थीं, तब तक अपने पदों पर कायम रहेंगी जब तक कि राज्य विधानमण्डल द्वारा प्रस्ताव पारित करके उनको भंग नहीं किया जाता।

18. उपरोक्त व्यवस्थाएँ कुछ क्षेत्रों पर लागू नहीं होंगी-74वें संवैधानिक संशोधन अधिनियम की उपरोक्त व्यवस्थाएँ निम्नलिखित क्षेत्रों पर लागू नहीं होंगी।

  • संविधान के अनुच्छेद 244 पद (1) और पद (2) में वर्णित अनुसूचित क्षेत्रों और कबायली क्षेत्रों (Scheduled areas and Tribal areas)।
  • पश्चिमी बंगाल के दार्जिलिंग जिले के पहाड़ी क्षेत्र जहाँ ‘दार्जिलिंग गोरखा परिषद्’ (Darjeeling Gorkha Hill Council) स्थापित है।

19. संघीय क्षेत्रों पर लागू होना-74वें संवैधानिक संशोधन अधिनियम की सभी व्यवस्थाएँ संघीय क्षेत्रों पर लागू होंगी। संघीय क्षेत्र के सम्बन्ध में इस अधिनियम में राज्य के राज्यपाल’ के स्थान पर ‘संघीय क्षेत्र प्रशासक’ (Administrative of Union Territory) और ‘राज्य विधानमण्डल’ के स्थान पर ‘संघीय विधानसभा’, यदि उस क्षेत्र में विधानसभा’ ही प्रयोग, किए जाएँगे।

इस विवरण से यह स्पष्ट है कि 74वें संविधान संशोधन द्वारा शहरी स्थानीय संस्थाओं के लिए एक नई प्रणाली की व्यवस्था की गई है। इस नई प्रणाली के अन्तर्गत नगरपालिकाओं को पहले की अपेक्षा बहुत अधिक दृढ़ और की भागीदारी को बेहतर बनाया गया है। शहरी स्थानीय संस्थाओं को संवैधानिक मान्यता प्रदान की गई है और पहले की तुलना में उन्हें अधिक शक्तियाँ, जिम्मेवारियाँ तथा वित्तीय साधन सौंपे गए हैं।

इन संस्थाओं में स्त्रियों, अनुसूचित जातियों, अनुसूचित जनजातियों को पर्याप्त प्रतिनिधित्व दिया गया है और स्थानीय स्तर पर लोकतन्त्र को अधिक सशक्त बनाने का प्रयत्न किया गया है। इस प्रकार इस संविधान संशोधन द्वारा शहरी स्थानीय संस्थाओं की स्थिति में क्रान्तिकारी परिवर्तन किया गया है।

प्रश्न 10.
नगर-निगम की संरचना, कार्यों तथा आय के साधनों का वर्णन करें। अथवा . नगर निगम की संरचना और कार्यों का वर्णन करें।
उत्तर:
नगर-निगम शहरी क्षेत्र में गठित स्थानीय शासन की इकाइयों में सर्वोच्च, सम्माननीय तथा स्वायत्त निकाय होता है। नगर पालिकाओं की अपेक्षा नगर-निगम को अधिक शक्तियाँ प्राप्त होती हैं और इन पर सरकारी नियन्त्रण भी कम होता है। केन्द्रीय शासित क्षेत्र में नगर-निगम स्थापित करने के लिए तथा संसद और राज्यों में नगर-निगम गठित करने के लिए राज्य विधानमण्डल द्वारा विशेष अधिनियम पारित करने पड़ते हैं।

भारत में सबसे पहले चेन्नई में नगर-निगम गठित किया गया था। हमारे देश में इस समय हैदराबाद, पटना, सूरत, शिमला, ग्वालियर, मुम्बई, आगरा, कोलकाता, दिल्ली, अमृतसर आदि बड़े-बड़े शहरों में नगर-निगम गठित किए गए हैं। प्रत्येक नगर-निगम के लिए एक विशेष अधिनियम पारित किया गया है। उदाहरण के लिए दिल्ली में, दिल्ली नगर-निगम अधिनियम, 1957, मुम्बई में मुम्बई नगर-निगम अधिनियम, 1888, पंजाब में पंजाब नगर-निगम अधिनियम, 1976, हरियाणा में फरीदाबाद में नगर-निगम का गठन पहले से ही किया हुआ है। इसके बाद गुड़गाँव को भी नगर-निगम बना दिया गया।

इस तरह हरियाणा में अम्बाला, हिसार, करनाल, पानीपत, पंचकूला, यमुनानगर, सोनीपत और रोहतक को मिलाकर दस नगर निगम स्थापित किए गए थे। इसी तरह जुलाई, 2015 को सोनीपत को हरियाणा का दसवाँ नगर निगम बनाया गया। यहाँ यह स्पष्ट है कि फरवरी, 2018 में हरियाणा मंत्रिमण्डल के द्वारा लिए गए निर्णय के अनुसार अम्बाला नगर निगम को भंग कर दिया गया था,

लेकिन 23 जुलाई, 2019 को कोर्ट केस में कोर्ट ने बीच का रास्ता अपनाते हुए अम्बाला सिटी को नगर निगम और अम्बाला कैंट को नगर परिषद का रूप दे दिया। इसके बाद हरियाणा सरकार ने 27 जुलाई, 2019 को अधिसूचना जारी करते हुए अम्बाला शहर को नगर-निगम एवं अम्बाला छावनी को नगर परिषद बना दिया गया। इस प्रकार वर्तमान में हरियाणा में 10 नगर निगम हैं।

हरियाणा में इस प्रकार का कानून हरियाणा नगर-निगम अधिनियम, 1994 (Haryana Municipal Corporation Act, 1994) विधानमण्डल ने 31 मई, 1994 को पारित किया और इसके अनुसार फरीदाबाद में नगर-निगम गठित किया गया। राज्य सरकार अधिसूचना जारी करके इस अधिनियम में अन्य नगरीय क्षेत्रों को भी शामिल कर सकती है।

यह निगम एक सतत् उत्तराधिकार प्राप्त निकाय (Body of Perpetual Succession) है जिसको एक सामान्य मुहर (Common Seal), सम्पत्ति तथा स्वयं के नाम से दावा करने या अभियोग चलाने की शक्ति प्राप्त है। रचना (Composition) हरियाणा नगर निगम अधिनियम के अनुसार जिन शहरों की जनसंख्या 3 लाख से अधिक है वहाँ नगर निगम की स्थापना की जाती है। इसके अतिरिक्त 74वें संवैधानिक संशोधन द्वारा की गई व्यवस्था के अनुसार राज्य विधानमण्डल कानून द्वारा निम्नलिखित वर्गों के व्यक्तियों को नगर-निगम में प्रतिनिधित्व देने की व्यवस्था करता है

(क) तीन व्यक्ति, जिन्हें नगरीय प्रशासन के अनुभव का विशेष ज्ञान प्राप्त हो,
(ख) नगर-निगम के नगरीय क्षेत्रों का प्रतिनिधित्व करने वाले लोकसभा तथा विधानसभा के सदस्य,
(ग) राज्य सभा के सदस्य जिनका नगरीय क्षेत्र में मतदाता के रूप में पंजीकरण है। उपर्युक्त मनोनीत सदस्यों में से (क) वर्ग के सदस्यों को नगर-निगम की बैठकों में मताधिकार से तथा (ख) एवं (ग) वर्ग के सदस्यों को मेयर, वरिष्ठ उप-मेयर या उप-मेयर के चुनाव तथा हटाने की प्रक्रिया में मताधिकार से वंचित रखा गया है।

निगम का कार्यकाल (Duration of Corporation):
नगर-निगम का कार्यकाल, सिवाय भंग होने की अवस्था में, पाँच वर्ष होता है। यदि किसी कारणवश निगम को भंग कर दिया जाता है तो अगली चुनी गई नगर-निगम शेष बचे समय के लिए कार्य करेगी। निगम को भंग करने से पूर्व उसे अपने विचार व्यक्त करने का पूरा अवसर दिया जाता है। सदस्यों की

अयोग्यताएँ (Disqualifications of the Members):
74वें संवैधानिक संशोधन के आधार पर हरियाणा विधानसभा द्वारा पारित हरियाणा नगर-निगम अधिनियम, 1994 के अनुसार नगर-निगम का सदस्य बनने के लिए नगरीय क्षेत्र के वार्ड में पंजीकृत मतदाता होने के अतिरिक्त 25 वर्ष की उम्र होना आवश्यक है। अधिनियम के अनुच्छेद 8 के अनुसार सदस्य बनने के लिए निम्नलिखित कारणों को अयोग्य माना गया है

  • राज्य विधानमण्डल के निर्वाचन के उद्देश्य से मानी गई अयोग्यता,
  • विकृत चित,
  • दिवालिया,
  • विदेशी नागरिकता,
  • इस अधिनियम की धारा 22 के अन्तर्गत परिभाषित भ्रष्ट विधि,
  • भारतीय दंड संहिता, 1860 की धारा 171-E या 171-F या इस अधिनियम की धारा 30 या 31 के अनुसार कोई अपराधिक मुकद्दमा,
  • अनैतिक आचार के कारण दोष-सिद्धि या हिरासत,
  • निगम के अन्तर्गत किसी भी लाभ के पद पर होना,
  • सरकार के अन्तर्गत किसी भी लाभ के पद पर होना,
  • निगम से लाइसेन्सशुदा, वास्तुविद् (Architect) ड्राफ्ट्स मैन, इंजीनियर, पैमाइश करने वाला (Surveyer), पलम्बर या नगर योजनाकार,
  • निगम के किसी कार्य या संविदा से लाभ प्राप्तकर्ता,
  • निगम के द्वारा संचालित किसी फर्म या व्यवसाय का हिस्सेदार,
  • सरकार, निगम या स्थानीय सत्ता के अधीन किसी सेवा से पदच्युति,
  • निगम को देय किसी फीस या जुर्माने को अदा करने में असफल होने पर,
  • दो से अधिक जीवित बच्चों का माँ-बाप होने पर। यद्यपि यह शर्त उन माता-पिताओं पर नहीं लागू होती जिनके पास 31 मई, 1995 तक या इससे पहले दो जीवित बच्चे हो।

निगम में आरक्षित सीट (Reserved Seats of the Corporation):
74वें संशोधन के आधार पर हरियाणा नगर-निगम अधिनियम, 1994 की धारा 11 के अनुसार निम्नलिखित वर्गों के लिए सीटों का आरक्षण किया गया है-

(1) नगरीय क्षेत्र की कुल जनसंख्या और अनुसूचित जातियों की जनसंख्या के मध्य अनुपात के अनुसार सीटें आरक्षित की जाती हैं। इनका कुल जनसंख्या तथा कुल सीटों में एक समान अनुपात होना चाहिए,

(2) उपर्युक्त सीटों में से एक-तिहाई सीटें अनुसूचित जातियों की महिलाओं के लिए आरक्षित की जाएँगी,

(3) प्रत्यक्ष चुनाव के द्वारा भरी गई कुल सीटों में से एक-तिहाई सीटें (अनुसूचित जातियों की महिलाओं के लिए आरक्षित सीटों समेत) महिलाओं के लिए आरक्षित की जाएंगी। इन सीटों का बँटवारा भिन्न-भिन्न वार्डों में लॉटरी के द्वारा किया जाएगा,

(4) दो सीटें पिछड़ी जातियों के लिए, जिन वार्डों में इन जातियों की अधिकतम संख्या है, आरक्षित की जाएंगी।

(5) मेयर, वरिष्ठ उप-मेयर तथा उप-मेयर के पदों का आरक्षण सामान्य वर्ग, अनुसूचित जातियों, पिछड़ी जातियों तथा महिलाओं के लिए क्रमवार (By Rotation) लॉटरी पद्धति के अनुसार किया जाएगा। स्थानीय शासन नगर-निगम के अधिकारीगण तथा संस्थाएँ-नगर-निगम में प्रायः निम्नलिखित अधिकारीगण तथा संस्थाएँ होती हैं

1. महापौर (Mayor):
नगर-निगम का अध्यक्ष महापौर (Mayor) कहलाता है। इसका चुनाव नगर-निगम के सदस्यों द्वारा किया जाता है। नगर-निगम के सदस्य एक ‘उप-महापौर’ (Deputy Mayor) का भी चुनाव करते हैं। आमतौर पर महापौर तथा उप-महापौर का चुनाव एक वर्ष के लिए किया जाता है, परन्तु कार्यकाल की समाप्ति पर उन्हें प्रायः दुबारा चुन लिया जाता है। महापौर नगर-निगम की बैठकों की अध्यक्षता करता है। अव्यवस्था उत्पन्न होने की स्थिति में वह सदन की बैठक को स्थगित कर सकता है। महापौर नगर का प्रथम नागरिक कहलाता है। नगर में आने वाले विशिष्ट अतिथियों का वह अभिनन्दन करता है।

2. निगम की सामान्य परिषद् (General Council of the Corporation):
निगम की सामान्य परिषद् में सभी सभासद तथा एल्डरमैन शामिल होते हैं। सामान्य परिषद् के सदस्यों की संख्या सभी नगर-निगमों के लिए एक-जैसी नहीं है। निगम आयुक्त को छोड़कर नगर-निगम के अन्य अधिकारियों की नियुक्ति सामान्य परिषद् द्वारा की जाती है।

दिल्ली नगर-निगम के आयुक्त की नियुक्ति केन्द्रीय सरकार द्वारा की जाती है, परन्तु राज्यों में निगम आयुक्त राज्य सरकार द्वारा नियुक्त किए जाते हैं। सामान्य परिषद् निगम क्षेत्र की विभिन्न समस्याओं पर विचार करती है तथा सामान्य नीति बनाती है, जिसे विभिन्न समितियों द्वारा क्रियान्वित किया जाता है।

3. स्थायी समितियाँ (Standing Committees):
सामान्य परिषद् अपने कार्यों का संचालन स्थायी समितियों के द्वारा करती है। ये समितियाँ सामान्य परिषद् द्वारा निर्वाचित की जाती हैं। ये समितियाँ मुख्य रूप से करारोपण तथा वित्त (Taxation and Finance), शिक्षा, स्वास्थ्य, बजट तैयार करना, निर्माण कार्य, बिजली तथा यातायात आदि विषयों से सम्बन्धित होती हैं।

4. निगम आयुक्त (Municipal Commissioner):
निगम आयुक्त नगर-निगम का प्रमुख प्रशासनिक अधिकारी होता है। वह महापौर को प्रशासन सम्बन्धी विवरण देता रहता है। निगम के प्रशासन को चलाने का उत्तरदायित्व मुख्य प्रशासनिक अधिकारी पर होता है, जिसे कमीश्नर (आयुक्त) कहा जाता है। दिल्ली में वह केन्द्रीय सरकार द्वारा तथा राज्यों के निगमों में राज्य सरकार द्वारा पाँच वर्ष के लिए नियुक्त किया जाता है, परन्तु सरकार इस अवधि को घटा-बढ़ा सकती है।

निगम आयुक्त निगम के अन्य सभी अधिकारियों के ऊपर होता है। वह विभिन्न संस्थाओं के कर्त्तव्य निर्धारित करता है और उनके कार्यों की देखभाल करता है। वह कई नियुक्तियाँ भी करता है। उसका कर्त्तव्य है कि वह निगम को अपने कार्यों का विवरण देता रहे। नगर-निगम के सदस्य जो भी जानकारी प्राप्त करना चाहें, उसे निगम आयुक्त ही देता है।

निगम की बैठकें (Meetings of the Corporation)-निगम अपने कार्य का संचालन करने के लिए प्रति माह एक बैठक का आयोजन करता है। निगम की प्रथम बैठक मण्डल-आयुक्त द्वारा बुलाई जाती है जिसमें मेयर का चुनाव किया जाता है। ऐसी बैठक की अध्यक्षता मण्डल-आयुक्त द्वारा नियुक्त सदस्य के द्वारा की जाती है। यदि मेयर के उम्मीदवारों के मध्य बराबर-बराबर के मत पड़ते हैं तो अध्यक्ष अतिरिक्त मत के द्वारा निर्णय करता है।

इन बैठकों की अध्यक्षता (सिवाय मेयर, वरिष्ठ उप-मेयर या उप-मेयर के निर्वाचन सम्बन्धी बैठक) मेयर, उसकी अनुपस्थिति में उप-मेयर द्वारा की जाती है। प्रत्येक बैठक की सूचना बैठक आरम्भ होने से 5 दिन पूर्व भेजी जाती है। निगम का सचिव प्रत्येक बैठक की कार्रवाई का संक्षिप्त रिकार्ड एक रजिस्टर में दर्ज करता है और अगली बैठक में इसकी जानकारी देता है। वह बैठक के अध्यक्ष के हस्ताक्षर भी करवाता है। कार्रवाई रिपोर्ट की सूचना प्रत्येक निगम के सदस्य को दी जाती है।

निगम का सचिव बैठक आरम्भ होने के तीन बाद बैठक में लिए गए निर्णय की रिपोर्ट सरकार को प्रेषित करता है और सरकार निगम सचिव से बैठक के सम्बन्ध में किसी कागजात की प्रति मंगवा सकती है। प्रत्येक बुलाई गई बैठक के लिए आवश्यक गणपूर्ति 1/3 निश्चित की गई है जिसके अभाव में बैठक स्थगित कर दी जाती है।

निगम के कार्य (Functions of Corporation)-देश में भिन्न-भिन्न नगर-निगमों को कई प्रकार के कार्य करने पड़ते हैं। कई राज्यों में निगम को सामान्य कार्य करने पड़ते हैं और उनकी सूची अधिनियम में दी जाती है। परन्तु कई राज्यों में निगमों को दो प्रकार के कार्य करने पड़ते हैं अनिवार्य और ऐच्छिक।हरियाणा नगर-निगम अधिनियम, 1994 के अध्याय III की धाराओं 42, 43 और 44 के अनुसार निगम को निम्नलिखित कार्य सौंपे गए हैं

HBSE 11th Class Political Science Important Questions Chapter 8 स्थानीय शासन

1. आर्थिक विकास व सामाजिक न्याय सम्बन्धी कार्य (Functions related with Economic Development and Social Justice)-नगर-निगम आर्थिक विकास, सामाजिक न्याय तथा संविधान की 12वीं अनुसूची में सम्मिलित 18 विषयों को लागू करने से सम्बन्धित योजनाओं और कार्यों का क्रियान्वयन करता है। इसमें निम्नलिखित कार्य आते हैं …

  • नगरीय योजना,
  • भूमि उपयोग का विनियमन और भवनों का निर्माण,
  • आर्थिक और सामाजिक विकास योजना।
  • सड़कें और पुल,
  • घरेलू, औद्योगिक और वाणिज्यिक प्रयोजनों के लिए जल प्रदाय (Water Supply),
  • लोक स्वास्थ्य, स्वच्छता, सफाई और कूड़ा-कर्कट प्रबन्ध,
  • अग्निशमन सेवाएँ,
  • नगरीय वानिकी, पर्यावरण संरक्षण और पारिस्थितिक आयामों की अभिवृद्धि (Ecological Aspects),
  • समाज के दुर्बल वर्गों, दिव्यांगों और मानसिक रोगियों का हित,
  • गन्दी बस्ती सुधार,
  • नगरीय निर्धनता उन्मूलन,
  • नगरीय सुख-सुविधाओं; जैसे पार्क, उद्यान तथा खेलों के मैदानों की व्यवस्था,
  • सांस्कृतिक, शैक्षणिक और सौन्दर्य के आयामों की वृद्धि,
  • कब्रिस्तान, शवदाह और श्मशान,
  • पशु तालाब तथा पशुओं के प्रति क्रूरता का निवारण,
  • जन्म-मरण सांख्यिकी,
  • सार्वजनिक सुख-सुविधाओं में वृद्धि; जैसे सड़कों पर प्रकाश तथा जन-सुविधाएँ,
  • वध-शालाओं (Slaughters) और चर्मशोधन-शालाओं (Tanneries) का विनियमन।

2. आवश्यक कार्य (Obligatory Functions):
निगम के लिए यह आवश्यक होगा कि वह नीचे लिखे कार्यों को लागू करने के लिए, जोकि उसके कर्तव्यों का हिस्सा होंगे, हर सम्भव प्रयास करेगा

  • शौचालयों तथा मूत्रालयों का निर्माण, रख-रखाव व सफाई,
  • जल-आपूर्ति व्यवस्था का प्रबन्ध,
  • कूड़ा-कर्कट, गन्दगी व वातावरण को दूषित करने वाले पदार्थों को हटाना,
  • हानिकारक पौधों तथा कष्टदायक बाधाओं को हटाना,
  • मृतकों का दाह-संस्कार करने के लिए स्थानों का नियमन,
  • पशु तालाबों का रख-रखाव,
  • हानिकारक बीमारियों की रोकथाम,
  • नगरीय बाजारों का नियमन,
  • घृणोत्पादक व्यवसायों (Offensive Traders) पर रोक,
  • खतरनाक भवनों को हटाना,
  • सार्वजनिक गलियों, पुलियों तथा उच्च मार्गों (Causeways) का निर्माण, रख-रखाव व सुधार,
  • गलियों की सफाई व प्रकाश का प्रबन्ध,
  • गलियों से बाधाएँ हटाना,
  • घरों और गलियों में नम्बर लगाना,
  • नगरीय कार्यालयों का रख-रखाव
  • सार्वजनिक पार्को, उद्यानों तथा मनोरंजन के साधनों का प्रबन्ध,
  • अग्नि-शामक दल व यन्त्र का प्रबन्ध व आग लगने की अवस्था में जानमाल की रक्षा करना,
  • स्मारकों का रख-रखाव,
  • निगम की सभी सम्पत्तियों का विकास करना।
  • सड़क के किनारों पर लगे पेड़-पौधों की देखभाल करना,
  • भवनों व भूमि का सर्वेक्षण,
  • इस अधिनियम के अधीन सौंपा गया कोई भी अन्य कार्य।

3. ऐच्छिक कार्य (Discretionary Functions):
नगर-निगम उपलब्ध वित्तीय साधनों की पर्याप्तता के आधार पर निम्नलिखित कार्य करता है

  • सांस्कृतिक, शारीरिक व सामान्य शिक्षा का प्रबन्ध,
  • पुस्तकालयों, संग्रहालयों व चिड़ियाघरों की स्थापना करना,
  • खेल-कूद व व्यायाम के लिए व्यायामशालाओं व अखाड़ों का निर्माण करना,
  • विवाहों का पंजीकरण,
  • जनगणना,
  • अति विशिष्ट व्यक्तियों का स्वागत करना,
  • जनता के लिए संगीत, मनोरंजन व सिनेमाघरों की व्यवस्था करना,
  • प्रदर्शनियों व मेलों का प्रबन्ध करना,
  • विश्राम-गृहों, वृद्ध-आश्रमों, बाल-भवनों, अनाथालयों तथा बहरों और गूगों के लिए आवासों का प्रबन्ध करना,
  • निगम अधिकारियों व कर्मचारियों के लिए आवास स्थानों का प्रबन्ध करना,
  • निगम कर्मचारियों का कल्याण,
  • जन-स्वास्थ्य से सम्बन्धित पानी, स्वास्थ्य और औषधियों का परीक्षण करने के लिए प्रयोगशालाओं का प्रबन्ध करना,
  • निराश्रितों और दिव्यांगों की सहायता करना,
  • जन टीकाकरण,
  • जनता के लिए तैरने और नहाने के लिए स्थानों का रख-रखाव करना,
  • दुग्ध डेयरियों का प्रबन्ध करना,
  • कुटीर उद्योगों तथा हस्तशिल्प कला-केन्द्रों का प्रबन्ध करना।
  • गोदामों का प्रबन्ध करना,
  • वाहनों तथा पशुओं के ठहरने के स्थानों का प्रबन्ध करना,
  • नगर निवासियों के ठहरने के लिए आवास स्थलों का निर्माण करना,
  • अस्पतालों, औषधालयों तथा प्रसूति एवं शिशु-कल्याण केन्द्रों की स्थापना करना,
  • जन-सुरक्षा, स्वास्थ्य तथा जन-कल्याण के लिए किसी भी प्रकार का कदम उठाना,
  • निगम से स्वीकृत योजनाओं के अनुसार नगरीय क्षेत्रों का सुधार करना।

निगम निधि (Corporation Fund)-नगर-निगम भिन्न-भिन्न वित्तीय देनदारियों के लिए एक निगम निधि की स्थापना करेगा जिसमें निम्नवत सम्मिलित हैं

  • निगम स्थापित करने से पूर्व सभी फंड,
  • इस अधिनियम के अन्तर्गत सभी प्रकार के प्राप्त धन,
  • निगम की ओर से की गई सभी सम्पत्तियों की आय का निपटारा,
  • निगम की किसी भी प्रकार की सम्पत्तियों से प्राप्त किराया,
  • सभी प्रकार के शुल्क और जुर्माने,
  • निगम के द्वारा सरकार, किसी व्यक्ति या संस्था से प्राप्त धन,
  • निगम के द्वारा जमा किए गए धन पर ब्याज या अन्य लाभ,
  • निगम के द्वारा अन्य किसी भी स्रोत से प्राप्त धन।

उपर्युक्त सभी प्रकार की वित्तीय देनदारियों के लिए गठित निगम फंड को किसी राष्ट्रीय बैंक, सरकारी कोषागार या सरकार निगम आयुक्त द्वारा रखा जाएगा। किसी भी प्रकार के पैसे का लेन-देन तब तक नहीं किया जाएगा, जब तक कि लेखा अधिकारी (Account Officer) तथा निगम आयुक्त (Commissioner) हस्ताक्षर न कर दें। निगम की आय के साधन (Sources of Income of Corporation) : निगम की आय के साधन निम्नलिखित हैं

1. निगम द्वारा लगाए गए कर (Taxes imposed by the Corporation):
निगम भिन्न-भिन्न प्रकार के करों को लगाने का अधिकार रखता है, परन्तु कुछ कर सिर्फ सरकार की स्वीकृति से ही लगाए जा सकते हैं। निगम जिन करों को सरकार की पूर्व स्वीकृति के बिना लगा सकता है, उनमें निम्नलिखित सम्मिलित हैं

  • भवनों व भूमि पर कर,
  • सरकार द्वारा निर्धारित दर के अनुसार चुंगीकर,
  • सम्पत्तियों के बेचने (Sale), परिवर्तन करने (Exchange) रेहन या बन्धक (Mortgage) या पट्टे (Lease) पर देने के बदले में प्राप्त शुल्क। इसके अतिरिक्त निगम सरकार की स्वीकृति से निम्नलिखित कर लगा सकता है

(1) व्यवसाय, व्यापार, रोजगार और अन्य धन्धों पर कर,
(2) वाहन-कर (सिवाय मोटर-वाहन तथा पशु),
(3) शहरी भूमि की कीमत बढ़ने पर विकास-कर (Development Tax),
(4) तमाशा कर (Show Tax),
(5) ऊर्जा का उपभोग करने पर (5 पैसे प्रति यूनिट से कम) ऊर्जा-कर,
(6) हरियाणा नगर पालिका अधिनियम के अन्तर्गत लगाया गया अन्य कोई भी कर।

2. निगम द्वारा ली जाने वाली फीसें (Fees Charged by the Corporation) राज्य सरकार की पूर्व स्वीकृति से निगम स्थापित विधि के अनुसार निम्नलिखित फीसों को वसूल कर सकता है

  • समाचार-पत्रों के सिवाय अन्य विज्ञापनों पर शुल्क,
  • भवन के लिए आवेदन पत्रों पर शुल्क,
  • कुछ क्षेत्रों में स्वच्छता सुविधाओं को प्रदान करने और रख-रखाव पर विकास शुल्क,
  • प्रकाश शुल्क,
  • गन्दगी दूर हटाने पर शुल्क,
  • भवन योजनाओं के अन्तर्गत आन्तरिक सेवाओं पर शुल्क,
  • निगम द्वारा प्रदान की गई सेवाओं के लिए उचित समझा गया अन्य कोई शुल्क।

3. करों और फीसों की वसूली (Recovery of Taxes and Fees):
निगम सभी प्रकार के करों व फीसों को वसूली करने के लिए विधि या प्रक्रिया का निर्णय करता है। वह इन करों और फीसों को निम्नलिखित तरीके से भी वसूल कर सकता है

  • भू-राजस्व की बकाया राशि,
  • दोषी की चल-सम्पत्ति को बेचना या उसकी कुड़की (Distrairt) करना,
  • दोषी की अचल सम्पत्ति को बेचना या उसकी कुड़की करना,
  • चुंगी या पथ-कर के मामले में दोषी की किसी वस्तु या वाहन को कब्जे में लेना,
  • भूमि और भवन-कर के मामले में सम्पत्ति की कुड़की करना,
  • अभियोग करना।

4. निगम द्वारा उधार लेना (Borrowing by Corporation)-निगम एक प्रस्ताव पारित कर ऋण पत्र (Debenture) या किसी अचल सम्पत्ति अथवा सभी या किसी वसूल किए जाने वाले कर या फीस की जमानत पर किसी लोक संस्था से उधार ले सकता है। निगम जिन उद्देश्यों के लिए धन उधार ले सकता है, उनमें निम्नलिखित आते हैं

  • किसी प्रकार की जमीन का अधिग्रहण करने के लिए,
  • किसी भवन को ऊँचा (Eracting) उठाने के लिए,
  • किसी ऐसे स्थायी कार्य को आरम्भ करने के लिए जिसकी कई वर्षों तक चलने की सम्भावना हो,
  • सरकार के किसी ऋण को अदा करने के लिए,
  • पहले लिए गए किसी ऋण की अदायगी करने के लिए,
  • इस अधिनियम के अन्तर्गत किसी अन्य वैधानिक उद्देश्य के लिए।

निगम उपर्युक्त कार्यों को पूरा करने के लिए सरकार की पूर्व स्वीकृति के बिना कोई भी ऋण उधार नहीं ले सकता। इसके अतिरिक्त ऋण की सीमा, ब्याज की दर, ऋण का भुगतान करने की विधि, समय तथा अन्य शर्तों के निर्णयों में भी सरकार की स्वीकृति ली जाती है।

यद्यपि उधार लिए गए धन की अदायगी 60 वर्षों से कम समय में करनी पड़ती है। निगम ऋण पत्रों की जमानत पर उधार लिए गए धन की अदायगी करने के लिए ऋण परिषोध कोष (Sinking Fund) की स्थापना कर सकता है।

5. सरकारी अनुदान (Govt. Grants):
सरकार समय-समय पर निगमों की आर्थिक सहायता करने के लिए सरकारी अनुदान देती है। यह निगमों की आय का महत्त्वपूर्ण साधन है। निगम के द्वारा बजट तैयार करना (Preparation of the Budget by the Corporation)-निगम आय और व्यय का अनुमान लगाने के लिए प्रत्येक वर्ष फरवरी के प्रथम सप्ताह से पहले बजट तैयार करवाता है।

इस प्रकार तैयार किया गया बजट फरवरी के अन्तिम सप्ताह से पहले-पहले सरकार के पास स्वीकृति के लिए भेज दिया जाता है। सरकार यदि उचित समझती है तो कुछ संशोधनों सहित या बिना संशोधन के 31 मार्च से पहले यह बजट निगम के पास वापिस भेज देती है।

निगम के लेखों की तैयारी व उनका परीक्षण (Preparation and Audit of Corporation Accounts)-निगम के लेखों को रखने और उनकी परीक्षा करने का उत्तरदायित्व एक अधिकारी का होता है, जिसे स्थानीय कोष लेखा अधिकारी (Examiner Local Fund Accounts) कहते हैं। वह निगम के लेखों की रिपोर्ट निगम-परिषद् के समकक्ष प्रस्तुत करता है

और निगम की प्राप्तियों व खर्चों का संक्षिप्त विवरण (Abstract of Receipts and Expenditure) तैयार करवाता है। वह निगम के वित्तीय लेन-देनों में पालन किए गए नियमों की देख-रेख करता है। वह किसी प्रकार की वित्तीय अनियमितता व गबन की सूचना आयुक्त को और उसके माध्यम से सरकार को देता है।

HBSE 11th Class Political Science Important Questions Chapter 8 स्थानीय शासन

प्रश्न 11.
नगरपरिषद् की रचना, कार्यों तथा आय के साधनों का वर्णन करें।
उत्तर:
नगर-परिषद् शहरी स्थानीय स्वशासन की सबसे महत्त्वपूर्ण इकाई है। 74वें संवैधानिक संशोधन एक्ट के पास होने से पहले इसे नगरपालिका (Municipal Committee) के नाम से पुकारा जाता था। परन्तु इस एक्ट द्वारा इसे नगर-परिषद् (Municipal Council) का नाम दिया गया है। हरियाणा में प्रायः जिन नगरों की जनसंख्या 50 हजार तक होती है,

वहाँ नगरपालिका का गठन किया जाता है तथा जिन नगरों की जनसंख्या 50 हजार से 3 लाख के बीच में होती है, उनमें राज्य सरकार द्वारा नगर-परिषद् की स्थापना कर दी जाती है। वर्तमान में हरियाणा में 21 नगरपरिषद् एवं 57 नगरपालिकाएँ हैं। नगर की जनसंख्या, रोजगार की उपलब्धता, आय तथा आर्थिक स्थिति को ध्यान में रखते हुए राज्य सरकार द्वारा

उस क्षेत्र में नगर-परिषद् की स्थापना की जाती है। ऊपर दिए गए तत्त्वों को ध्यान में रखते हुए पंजाब में नगर-परिषदों को तीन श्रेणियों में बाँटा गया है। इस समय पंजाब में 27 ‘ए’ श्रेणी की, 39 ‘बी’ श्रेणी की तथा 31 ‘सी’ श्रेणी की नगर-परिषदें हैं।

1. नगर-परिषद् की रचना (Composition of Municipal Council):
प्रत्येक नगर-परिषद् में सदस्यों की संख्या नगर की जनसंख्या के आधार पर राज्य सरकार द्वारा निश्चित की जाती है। 11 नवंबर, 1994 को पंजाब सरकार द्वारा जारी की गई एक अधिसूचना के अनुसार यह संख्या 9 से कम तथा 49 से ऊपर नहीं होगी। नगर-परिषद् में निम्नलिखित दो प्रकार के सदस्य होते हैं-

1. सदस्यों का चुनाव-वे सदस्य, जिनकी संख्या समय-समय पर राज्य सरकार द्वारा निश्चित की जाती है, का चुनाव नगर के मतदाताओं द्वारा वयस्क मताधिकार प्रणाली के आधार पर प्रत्यक्ष रूप से किया जाता है। चुनाव के लिए नगर को लगभग समान जनसंख्या वाले वार्डों (Wards) में बाँट दिया जाता है और प्रत्येक वार्ड (चुनाव-क्षेत्र) में से एक सदस्य चुना जाता है। नगरपरिषद् के सदस्यों को प्रायः नगर पार्षद् (Municipal Councillor) कहा जाता है।

2. सहायक सदस्य राज्य विधानसभा के वे सभी सदस्य, जिनके निर्वाचन-क्षेत्र उस नगरपरिषद् क्षेत्र में पूर्णतः अथवा आंशिक रूप से आते हों, ऐसे सदस्य, नगरपरिषद् के सहायक सदस्य (Associate Members) होते हैं। उन्हें नगरपरिषद् की बैठकों में भाग लेने तथा बोलने का अधिकार तो होता है, परन्तु वे परिषद् में होने वाले मतदान में भाग नहीं ले सकते।

स्थानों का आरक्षण
(1) प्रत्येक नगरपरिषद् में अनुसूचित जातियों व अनुसूचित जनजातियों के लिए उनकी जनसंख्या के अनुसार स्थान सुरक्षित किए जाने का प्रावधान है। इन स्थानों में समय-समय पर परिवर्तन करने की व्यवस्था है,

(2) इन जातियों के लिए सुरक्षित किए गए स्थानों में से 1/3 स्थान इन जातियों से सम्बन्धित महिलाओं के लिए आरक्षित होंगे,

(3) प्रत्येक नगरपरिषद् में 1/3 स्थान महिलाओं (जिनमें अनुसूचित जातियों व अनुसूचित जनजातियों की महिलाओं के स्थान भी शामिल होंगे) के लिए आरक्षित किए जाएँगे,

(4) प्रत्येक नगरपरिषद् में एक स्थान पिछड़े वर्गों के लिए सुरक्षित करने की व्यवस्था है। योग्यताएँ संविधान के 74वें संशोधन के अनुसार नगरपरिषद् का सदस्य बनने के लिए निम्नलिखित योग्यताएँ निश्चित की गई भारत का नागरिक हो,

(2) वह 21 वर्ष की आयु पूरी कर चुका हो,
(3) वह पागल, दिवालिया या अपराधी न हो,
(4) वह केन्द्रीय सरकार अथवा किसी राज्य सरकार के अधीन किसी लाभ के पद पर कार्य न कर रहा हो,
(5) उसका नाम मतदाताओं की सूची में होना चाहिए,
(6) उसे नगरपरिषद् के चुनाव के लिए मताधिकार प्राप्त हो अथवा चुनाव के अयोग्य घोषित न किया गया हो।

कार्यकाल-74वें संविधान संशोधन द्वारा नगरपरिषद का कार्यकाल 5 वर्ष निश्चित किया गया है। यदि किसी नगरपरिषद को उसकी अवधि समाप्त होने से पूर्व भंग कर दिया जाता है, तो 6 महीने में नई नगरपरिषद् का चुनाव कराना अनिवार्य है। नव-निर्वाचित नगरपरिषद् शेष (बचे हुए) काल के लिए ही कार्य करेगी।

यदि भंग की गई नगरपरिषद् का शेष कार्यकाल 6 महीने से कम हो तो फिर नई नगरपरिषद् का चुनाव नहीं करवाया जाएगा। अध्यक्ष व उपाध्यक्ष नगरपरिषद् अपने निर्वाचित सदस्यों में से एक अध्यक्ष तथा उपाध्यक्ष का चुनाव करेगी। नगरपरिषदों में अध्यक्ष तथा उपाध्यक्ष पद अनुसूचित जातियों व अनुसूचित जनजातियों एवं महिलाओं के लिए आरक्षित किए जाने की भी व्यवस्था है।

यदि किसी अध्यक्ष या उपाध्यक्ष के विरुद्ध नगरपरिषद् के सदस्य 2/3 बहुमत से अविश्वास प्रस्ताव पास कर देते हैं, तो उसे अपने पद से त्यागपत्र देना पड़ेगा। बैठकें प्रत्येक नगरपरिषद् की एक मास में कम-से-कम एक बैठक होनी अनिवार्य है। इसके अतिरिक्त, यदि नगरपरिषद् की संख्या के आधे सदस्य लिखित रूप में विशेष अधिवेशन बुलाने की माँग करें, तो अध्यक्ष द्वारा 10 दिन के अन्दर ऐसी विशेष बैठक अवश्य बुलाई जाएगी।

नगरपरिषद की बैठक में सभी निर्णय बहुमत से लिए जाते हैं और किसी विषय पर समान मत पड़ने की स्थिति में अध्यक्ष को निर्णायक मत (Casting Vote) देने का अधिकार होगा।

स्थायी कर्मचारी नगरपरिषद् का दैनिक कार्य चलाने के लिए एक कार्यकारी अधिकारी होता है। उसे मासिक वेतन मिलता है। उसकी नियुक्ति नगरपरिषद् ही करती है। कहीं-कहीं कार्यकारी अधिकारी और सचिव दोनों ही होते हैं। सचिव कार्यकारी अधिकारी के अधीन कार्य करता है। इसके अतिरिक्त नगरपरिषद् के अन्य स्थायी अधिकारी भी होते हैं; जैसे इंजीनियर, स्वास्थ्य अधिकारी आदि। नगरपरिषद् के कार्य नगरपरिषद् के कार्यों को हम निम्नलिखित दो भागों में बाँट सकते हैं

1. अनिवार्य कार्य-ये वे कार्य हैं जो हर नगरपरिषद् को करने पड़ते हैं। नगरपरिषद् द्वारा अनिवार्य रूप से किए जाने वाले कार्य निम्नलिखित हैं-

  • नगर में सड़कों तथा गलियों का निर्माण करवाना और पुरानी सड़कों तथा गलियों की मुरम्मत करवाना,
  • शहरों की सड़कों तथा गलियों में सफाई का प्रबन्ध करना,
  • बिजली तथा पानी का प्रबन्ध करना,
  • आग बुझाने के लिए ‘फायर ब्रिगेड’ (Fire Brigade) की व्यवस्था करना,
  • नागरिकों के स्वास्थ्य की रक्षा का उचित प्रबन्ध करना,
  • नगर में बनने वाले नए मकानों के नक्शे पास करना। पुराने तथा खतरनाक मकानों को गिरवाना या मालिकों को निर्देश देकर उनकी मुरम्मत करवाना,
  • नगर में जन्म और मृत्यु का रिकॉर्ड रखना।

2. ऐच्छिक कार्य-ये वे कार्य हैं, जो यदि साधन और सुविधा हो तो नगरपरिषदों को करने चाहिएँ। बहुत-सी नगरपरिषदें, जिनके आय के साधन अच्छे हैं, ये कार्य भी करती हैं। नगरपरिषद् द्वारा किए जाने वाले निम्नलिखित कार्य उसके ऐच्छिक कार्यों के क्षेत्र में आते हैं-

  • पुस्तकालयों तथा वाचनालयों की स्थापना करना और उनका प्रबन्ध करना,
  • कुएँ, तालाब, खेल के मैदान आदि बनवाना,
  • स्थानीय बसों, रेलगाड़ियों आदि की व्यवस्था करना,
  • मेलों तथा मण्डियों का प्रबन्ध करना,
  • पार्क, उद्यान आदि बनवाना,
  • श्मशान तथा कब्रिस्तान के लिए स्थान निश्चित करना तथा उनका प्रबन्ध करना,
  • शुद्ध दूध की सप्लाई का प्रबन्ध करना,
  • खतरनाक उद्योग-धन्धों पर प्रतिबन्ध लगाना,
  • गन्दी तथा मिलावट वाली वस्तुओं की बिक्री पर रोक लगाना,
  • प्रसूति केन्द्र, बाल-कल्याण केन्द्र आदि खोलना और उनका प्रबन्ध करना,
  • प्रारम्भिक शिक्षा का प्रबन्ध करना,
  • प्रदर्शनियों का आयोजन करना,
  • यात्रियों के ठहरने के लिए धर्मशाला या सराय आदि का निर्माण करना।

आय के साधन-नगरपरिषद की आय के साधन निम्नलिखित हैं-

  • नगर में बाहर से आने वाली वस्तुओं पर नगरपरिषद् चुंगी कर (Octroi Duty) लगाती है। यह नगरपरिषद् की आय का मुख्य साधन है,
  • नगरपरिषद् अपने क्षेत्र के मकानों पर कर लगाती है,
  • नगरपरिषद् कई वस्तुओं के रखने पर लाइसेंस फीस लेती है। ताँगे, रेहड़ी, ठेला, साइकिल, रिक्शा आदि रखने के लिए नगरपरिषद् से लाइसेंस लेना होता है। इस लाइसेंस फीस से भी नगरपरिषद् को काफी आय होती है,
  • पानी तथा बिजली कर से भी नगरपरिषद्को काफी आय होती है,
  • नगरपरिषद् द्वारा स्थापित स्कूलों तथा अस्पतालों से प्राप्त फीस से भी इसे आय होती है,
  • नगरों में किसी विशेष स्थान पर जाने के लिए या किसी नदी या पुल का प्रयोग करने के लिए नगरपरिषद् टोल-टैक्स अथवा मार्ग-कर लगाती है,
  • नगरपरिषद् सिनेमा, नाटक, दंगल आदि पर भी मनोरंजन कर लगा सकती है,
  • पशुओं पर कर लगाने से भी इसकी आय होती है,
  • नगरपरिषद् की अपनी जिससे होने वाली आय नगरपरिषद के पास ही रहती है. (1) नगरपरिषद अपने क्षेत्र में कई प्रकार के ठेके देती है। जैसे मुख्य स्थानों पर साइकिल स्टैण्ड का ठेका, कूड़ा-कर्कट, खाद आदि का ठेका। इससे हुई आय भी नगरपरिषद् की आय है,
  • मण्डियों, मेलों आदि पर कर लगाने से भी इसकी आय होती है,
  • नौकाओं, पुलों पर कर लगाने से भी आय प्राप्त होती है।

नगरपरिषद पर सरकार का नियन्त्रण-नगरपरिषद् को स्थानीय शासन चलाने में पूर्ण स्वतन्त्रता नहीं होती। उसे राज्य सरकार के निर्देशन तथा नियन्त्रण में कार्य करना पड़ता है। कभी-कभी तो सरकार का हस्तक्षेप इतना अधिक हो जाता है कि नगरपरिषद् के सदस्य न कोई स्वतन्त्र निर्णय ले पाते हैं और न ही कोई ठोस काम कर पाते हैं। राज्य सरकार अग्रलिखित तरीकों से नगरपरिषदों पर नियन्त्रण करती है

(1) ज़िले का ज़िलाधीश सरकार का प्रतिनिधि होता है। वह नगरपरिषद् के काम की देख-रेख करता है। नगरपरिषद् द्वारा किए गए हर प्रस्ताव की सूचना ज़िलाधीश को दी जाती है। यदि जिलाधीश यह समझे कि वह प्रस्ताव जनहित के विरुद्ध है, या उसे नगरपरिषद् ने अपनी शक्ति का दुरुपयोग करके पास किया है, तो वह (जिलाधीश) उस प्रस्ताव को निरस्त कर सकता है,

(2) अपना काम ठीक से न करने पर नगरपरिषद् को जिलाधीश से चेतावनी मिलती है। यदि फिर भी नगरपरिषद् ठीक प्रकार से कार्य नहीं करती तो ज़िलाधीश सरकार से उस नगरपरिषद् को भंग करने की सिफारिश कर सकता है,

(3) सरकार कभी भी नगरपरिषद् के हिसाब-किताब की जाँच-पड़ताल कर सकती है,

(4) नगरपरिषद् द्वारा अपनाए गए उप-नियम सरकार की स्वीकृति के बाद ही लागू हो सकते हैं,

(5) सरकार कभी भी नगरपरिषद् को भंग करके वहाँ प्रशासक (Administrator) नियुक्त कर सकती है,

(6) नगरपरिषद् के सदस्यों की संख्या सरकार ही निश्चित करती है,

(7) सरकार नगरपरिषद् के सदस्यों या सभापति को दोषी ठहराकर उनकी अवधि समाप्त होने से पहले भी उन्हें पद से हटा सकती है,

(8) नगरपरिषद् को अपनी वार्षिक रिपोर्ट सरकार को भेजनी होती है,

(9) नगरपरिषद् के प्रमुख अधिकारियों को सरकार की स्वीकृति के बाद ही नियुक्त किया जाता है। सरकार की सिफारिश पर उन्हें पद से हटाया जा सकता है,

(10) नगरपरिषद् अपना बजट पास करके सरकार के पास भेजती है, सरकार उसमें परिवर्तन कर सकती है,

(11) सरकार नगरपरिषद् को धन से सहायता देती है। इस सहायता के द्वारा वह नगरपरिषद् पर नियन्त्रण रखती है। वह कुछ शर्ते लगा सकती है कि सहायता तभी मिलेगी, जब नगरपरिषद् ऐसा करेगी अथवा ऐसा नहीं करेगी।

निष्कर्ष-नगरपरिषदों का लोकतन्त्र में बहुत ही महत्त्वपूर्ण स्थान है। नगरपरिषदें नगर के विकास और कल्याण के लिए वे सब काम करती हैं जो दूर बैठी केन्द्रीय या राज्य सरकारें नहीं कर सकतीं। परन्तु हम देखते हैं कि हमारे यहाँ नगरपरिषदों पर राज्य सरकारों का बहुत अधिक नियन्त्रण है। अधिकतर नगरपरिषदें धन की कमी और अनावश्यक राजनीतिक हस्तक्षेप के कारण अपना कार्य ठीक तरह से नहीं कर पातीं।

स्वतन्त्र भारत में नए-नए नगर बस रहे हैं। पुराने नगरों की जनसंख्या और क्षेत्र बढ़ रहे हैं। ऐसे समय में नगरपरिषदों को समाज के विकास में महत्त्वपूर्ण योगदान करना है। राज्य सरकार को चाहिए कि वे नगरपरिषदों को राज्य के अनावश्यक हस्तक्षेप से स्वतन्त्र करे, राजनीतिक दलों के हस्तक्षेप से उन्हें मुक्त रखे तथा उन्हें पर्याप्त धन की सहायता दे।

प्रश्न 12.
शहरी स्थानीय संस्थाओं की कार्य-विधि में कौन-से दोष (त्रुटियाँ) हैं? उन्हें कैसे दूर किया जा सकता है? अथवा शहरी स्थानीय संस्थाओं की कार्य-विधि के दोष बताते हुए इन्हें दूर करने के उपाय बताएँ।
उत्तर:
भारत में शहरी स्थानीय संस्थाओं को पूरी तरह से लागू कर दिया गया है। परन्तु ये संस्थाएँ उतनी सफलतापूर्वक कार्य नहीं कर रही हैं जितनी कि इनसे अपेक्षा की गई है। स्थानीय संस्थाओं की कार्यविधि में अनेक दोष हैं, जो निम्नलिखित हैं

1. धन की कमी स्थानीय संस्थाओं के कार्य तो राष्ट्र-निर्माण को ही सौंपे जाते हैं, परन्तु इनकी आर्थिक स्थिति इतनी सुदृढ़ नहीं होती, जिससे ये अपना कार्य सन्तोषजनक ढंग से कर सकें। स्थानीय संस्थाओं के पास उचित मात्रा में धन नहीं होता। इनकी आय के साधन बहुत कम हैं, जिनसे इतनी आय नहीं हो पाती कि वे अपने सभी कार्यों को ठीक प्रकार से कर सकें। धन की कमी के कारण ये संस्थाएँ अपने कर्तव्यों का पालन ठीक तरह से नहीं कर पातीं।

2. दलबन्दी की भावना-यह बात सर्वमान्य है कि स्थानीय संस्थाओं में राजनीतिक दलों का कोई उचित स्थान नहीं होता है। स्थानीय संस्था का सम्बन्ध एक नगर या गाँव से होता है और इसके कार्य सफाई, पानी, शिक्षा, सड़कों आदि तक सीमित होते हैं। इसके कुछ ऐसे कार्य भी होते हैं जिनमें राजनीतिक मतभेद की भावना नहीं की जा सकती।

इसके अतिरिक्त राजनीतिक दल स्थानीय स्वशासन के सारे ढाँचे को दलबन्दी की भावना से दूषित कर देते हैं। उत्तर प्रदेश और अन्य राज्यों में स्थानीय संस्थाओं के चुनाव भी दल-पद्धति के आधार पर होते हैं।

3. लोगों की उदासीनता-आम व्यक्ति स्थानीय संस्थाओं के प्रति उदासीन रहते हैं। स्थानीय संस्थाओं के चुनावों में लोगों की रुचि बहत कम है। आम स्थानीय प्रशासन के बारे में भी उनका रवैया ऐसा ही होता है जैसे कि उनका उससे कोई वास्ता ही न हो। लोगों की उदासीनता कई अन्य दोषों का कारण बन जाती है।

4. अयोग्य व्यक्तियों का चुना जाना प्रायः यह देखा गया है कि स्थानीय संस्थाओं में योग्य व्यक्तियों की बजाय अयोग्य व्यक्ति ही अधिक चुने जाते हैं। गाँवों में अधिकतर नागरिक अशिक्षित होते हैं, जिससे वे अपने मत का उचित प्रयोग नहीं कर पाते। मतदान योग्यता के आधार पर नहीं होता। कुछ चालाक व्यक्ति नागरिकों को बेवकूफ बनाकर वोट ले लेते हैं और अपनी स्वार्थ-सिद्धि के लिए शासन चलाते हैं। .

5. अयोग्य कर्मचारी स्थानीय संस्थाओं के कर्मचारी प्रायः अयोग्य और भ्रष्ट होते हैं। उन्हें अपने कार्य के बारे में कोई अधिक जानकारी नहीं होती। वे रिश्वत (Bribery) जैसे भ्रष्ट तरीकों का भी प्रयोग करते हैं।

6. निष्पक्षता का अभाव-स्थानीय संस्थाएँ निष्पक्षता से कार्य नहीं करतीं। इसका यह परिणाम निकलता है कि लोगों का इनमें विश्वास नहीं रहता। पंचायतों में तो इनका और भी बुरा परिणाम निकलता है क्योंकि पंचायतों को न्यायिक कार्य भी करने पड़ते हैं। इससे पंचायत के द्वारा किए गए निर्णय निष्पक्ष तथा ठीक नहीं होते हैं।

7. केन्द्रीय तथा प्रान्तीय सरकार का हस्तक्षेप स्थानीय संस्थाएँ इसलिए भी अपना कार्य ठीक प्रकार से नहीं कर पाती क्योंकि इनके कार्यों में केन्द्रीय तथा प्रान्तीय सरकारें अधिक हस्तक्षेप करती रहती हैं और वे स्वतन्त्रता-पूर्वक अपनी इच्छा के अनुसार कोई कार्य कर ही नहीं सकतीं। ऐसी दशा में स्थानीय संस्थाएँ सरकार के अधीन हो जाती हैं और वह उद्देश्य, जिसकी प्राप्ति के लिए इनकी स्थापना होती है, पूरा नहीं हो पाता।

8. जातीयता और साम्प्रदायिकता-उत्तर प्रदेश में स्थानीय संस्थाएँ जातीय और साम्प्रदायिकता के आधार पर कार्य करती हैं। स्थानीय सरकार के दोषों को दूर करने के उपाय-निम्नलिखित उपायों के द्वारा स्थानीय स्वशासन के दोषों को दूर किया जा सकता है तथा उसे सफल बनाया जा सकता है

1. शिक्षा का प्रसार स्थानीय संस्थाओं की सफलता के लिए जनता का शिक्षित होना पहली आवश्यकता है। शिक्षित व्यक्ति ही अपने कर्तव्यों का ठीक प्रकार से पालन कर सकते हैं और अपने उत्तरदायित्व को समझ सकते हैं। शिक्षित होने के कारण वे अपने मत का ठीक प्रयोग करेंगे और योग्य व्यक्तियों को चुनेंगे। योग्य व्यक्ति चुने जाने के पश्चात् निष्पक्षता से काम करेंगे और दलबन्दी के झगड़ों में नहीं पड़ेंगे।

2. आर्थिक दशा में सुधार-सरकार को चाहिए कि स्थानीय संस्थाओं की आर्थिक दशा में सुधार करे। उन्हें ‘कर’ (Tax) लगाने के लिए और भी अधिकार होने चाहिएँ तथा सरकार की तरफ से धन की अधिक सहायता मिलनी चाहिए। सरकार उन्हें जो धन दे, उसके साथ अनुचित शर्ते न रखे और न ही उन पर अधिक नियन्त्रण रखने का प्रयत्न करे।

उच्च नैतिक स्तर-स्थानीय स्वशासन की सफलता के लिए लोगों का उच्च नैतिक स्तर भी बड़ा आवश्यक है। यदि लोगों का चरित्र ऊँचा होगा तो वे जो भी कार्य करेंगे, जन-हित के लिए करेंगे और निजी स्वार्थ या अपने दल, जाति आदि के स्वार्थ के लिए नहीं करेंगे। ऐसे व्यक्ति ही स्थानीय संस्थाओं को निष्पक्ष बना सकते हैं।

4. आदर्श जनमत-स्थानीय स्वशासन की सफलता अथवा असफलता जनमत पर भी निर्भर करती है। कई बार लोगों के गलत प्रचार अथवा अफवाहों से नगर या ग्राम का वातावरण खराब हो जाता है, जिसका बुरा प्रभाव स्थानीय संस्थाओं पर भी पड़ता है। अतः स्थानीय स्वशासन की सफलता के लिए आदर्श जनमत का निर्माण करना आवश्यक है।

5. सक्रिय रुचि स्थानीय संस्थाओं की सफलता के लिए यह आवश्यक है कि नगर अथवा गाँव के नागरिक अपनी स्थानीय समस्याओं के हल के लिए सक्रिय रूप से भाग लें। यह आमतौर पर देखा जाता है कि नागरिक अपने व्यक्तिगत कार्यों में इतने व्यस्त रहते हैं कि वे स्थानीय समस्याओं को हल करने में कोई रुचि नहीं रखते। सक्रिय रुचि के बिना स्थानीय संस्थाओं में सुधार होना कठिन है।

6. स्वायत्तता-स्थानीय संस्थाओं को और भी अधिक अधिकार मिलने चाहिएँ तथा उनके कार्यों में सरकारी हस्तक्षेप कम-से-कम होना चाहिए। सरकारी हस्तक्षेप के कारण इन संस्थाओं में कार्य करने का उत्साह नहीं रहता। सरकार इन संस्थ निगरानी रखे, इनको आवश्यक सहयोग दे और उनके कार्यों में समन्वय लाने का कार्य करे। भाव यह है कि सरकार । पथ-प्रदर्शक की भूमिका अभिनीत करे। सरकार को इस बात की चिन्ता नहीं करनी चाहिए कि कोई संस्था कुछ गलतियाँ करती है। कुछ गलतियों के बाद ही उसमें सुधार स्वयंमेव हो जाएगा।

7. कर्मचारियों में सुधार स्थानीय संस्थाओं में किए गए सुधार तब तक लाभदायक सिद्ध नहीं हो सकते, जब तक कर्मचारियों को योग्यता के आधार पर नियुक्त नहीं किया जाएगा। कर्मचारियों को प्रशिक्षण देने के लिए उचित व पर्याप्त व्यवस्था करनी चाहिए और उनके वेतन भी सरकारी कर्मचारियों के समान होने चाहिएँ। इस सम्बन्ध में प्रान्तीयकरण (Provincialization) की नीति को अपनाना चाहिए।

निष्कर्ष-73वें तथा 74वें संवैधानिक संशोधनों द्वारा पंचायती राज तथा शहरी स्थानीय संस्थाओं को संवैधानिक मान्यता प्रदान की गई है और इन संस्थाओं के दोषों को दूर करने का प्रयत्न किया गया है। आशा है कि अब इन संस्थाओं के कुशल संचालन से भारत में लोकतन्त्र की जड़ें और अधिक मजबूत होंगी।

वस्तु निष्ठ प्रश्न

निम्नलिखित प्रश्नों का उत्तर दिए गए विकल्पों में से उचित विकल्प छाँटकर लिखें

1. भारत में वर्तमान ग्रामीण स्थानीय शासन (पंचायती राज) व्यवस्था निम्नलिखित संवैधानिक संशोधन पर आधारित है
(A) 73वें
(B) 74वें
(C) 75वें
(D) 72वें
उत्तर:
(A) 73वें

2. निम्न स्थानीय शासन का महत्त्व है
(A) लोकतंत्र की पाठशाला
(B) प्रशासन में कुशलता
(C) धन का बजट
(D) उपर्युक्त सभी
उत्तर:
(D) उपर्युक्त सभी

3. 73वें संवैधानिक संशोधन की विशेषता है
(A) दो-स्तरीय पंचायती राज
(B) तीन-स्तरीय पंचायती राज
(C) चार-स्तरीय पंचायती राज
(D) इनमें से कोई नहीं
उत्तर:
(B) तीन-स्तरीय पंचायती राज

HBSE 11th Class Political Science Important Questions Chapter 8 स्थानीय शासन

4. ग्राम पंचायत की बैठकों की अध्यक्षता करता है
(A) सरपंच
(B) महापौर
(C) वरिष्ठ पंच
(D) डिप्टी कमिश्नर
उत्तर:
(A) सरपंच

5. पंचायती राज संस्थाओं (ग्राम पंचायत, पंचायत समिति तथा जिला परिषद्) का निश्चित कार्यकाल निम्नलिखित है
(A) 3 वर्ष
(B) 4 वर्ष
(C) 5 वर्ष
(D) 6 वर्ष
उत्तर:
(C) 5 वर्ष

6. सरपंच का चुनाव किया जाता है
(A) पंचायत के सदस्यों द्वारा
(B) राज्य सरकार द्वारा
(C) प्रत्यक्ष रूप से मतदाताओं द्वारा
(D) इनमें से कोई नहीं
उत्तर:
(C) प्रत्यक्ष रूप से मतदाताओं द्वारा

7. महिलाओं के लिए स्थानीय संस्थाओं में सदस्यों की निम्न मात्रा में स्थान सुरक्षित रखे जाते हैं
(A) 1/2
(B) 1/5
(C) 1/3
(D) 1/4
उत्तर:
(A) 1/2

8. ग्राम पंचायत का सदस्य बनने के लिए व्यक्ति की न्यूनतम आयु होनी चाहिए
(A) 18 वर्ष
(B) 21 वर्ष
(C) 25 वर्ष
(D) 20 वर्ष
उत्तर:
(B) 21 वर्ष

9. वर्तमान स्थिति में शहरी स्थानीय संस्थाओं की स्थापना निम्नलिखित संवैधानिक संशोधन के आधार पर की जाती है
(A) 73वें
(B) 74वें
(C) 75वें
(D) 76वें
उत्तर:
(B) 74वें

10. बड़े शहरों में स्थापित स्थानीय शासन की इकाई है
(A) ज़िला-परिषद्
(B) नगर-परिषद्
(C) नगर-निगम
(D) छावनी बोर्ड
उत्तर:
(C) नगर-निगम

11. प्रत्येक नगर-निगम में महिलाओं के लिए स्थान आरक्षित होंगे
(A) 1/4
(B) 1/3
(C) 1/5
(D) 1/10
उत्तर:
(B) 1/3

12. नगर निगम का अध्यक्ष होता है
(A) अध्यक्ष
(B) गवर्नर
(C) मुख्यमंत्री
(D) महापौर
उत्तर:
(D) महापौर

13. नगर निगम का सदस्य बनने के लिए व्यक्ति की न्यूनतम आयु होनी चाहिए
(A) 18 वर्ष
(B) 20 वर्ष
(C) 21 वर्ष
(D) 25 वर्ष
उत्तर:
(C) 21 वर्ष

14. महापौर का चुनाव किया जाता है
(A) मतदाताओं द्वारा प्रत्यक्ष रूप से
(B) नगर निगम के सदस्यों द्वारा किसी भी व्यक्ति को
(C) नगर निगम के सदस्यों द्वारा अपने में से
(D) राज्य सरकार द्वारा
उत्तर:
(C) नगर निगम के सदस्यों द्वारा अपने में से

15. नगर-परिषद् की स्थापना की जाती है
(A) छोटे शहरों में
(B) बहुत बड़े शहरों में
(C) जिला स्तर पर
(D) इनमें से कोई नहीं
उत्तर:
(A) छोटे शहरों में

16. निम्न नगर-परिषद्/नगर निगम की आय का साधन है
(A) विभिन्न करों से होने वाली आय
(B) सरकारी अनुदान
(C) गृह-कर
(D) उपर्युक्त सभी
उत्तर:
(D) उपर्युक्त सभी

निम्नलिखित प्रश्नों का उत्तर एक शब्द में दें

1. पंचायत समिति का गठन किस स्तर पर किया जाता है?
उत्तर:
पंचायत समिति का गठन ब्लॉक (Block) अथवा खण्ड स्तर पर किया जाता है।

2. पंचायत समिति की योजनाओं को लागू करने में सहायता करने वाले अधिकारी का नाम लिखें।
उत्तर:
खण्ड विकास अधिकारी (Block Development Officer)।

3. भारत में पहली पंचायत कब और कहाँ पर गठित की गई थी?
उत्तर:
2 अक्तूबर, 1959 को नागौर (राजस्थान) में।

4. ग्राम सभा की बैठकें कितनी बार होनी आवश्यक हैं?
उत्तर:
वर्ष में दो बार, प्रतिवर्ष 13 अप्रैल और 2 अक्तूबर को ग्राम सभा की बैठकें होनी आवश्यक हैं।

5. संविधान के कौन-से संशोधन के अनुसार नगरपालिकाओं की स्थिति को संवैधानिक दर्जा दिया गया है?
उत्तर:
74वें संशोधन में।

6. संविधान की 12वीं अनुसूची में नगरपालिकाओं के अधिकार क्षेत्र में कितने विषय रखे गए हैं?
उत्तर:
18 विषय।

7. ग्राम पंचायत के अध्यक्ष को क्या कहते हैं?
उत्तर:
सरपंच।

8. नया पंचायती राज अधिनियम देश में कब लागू किया गया?
उत्तर:
सन् 1993 में।

9. ग्राम सभा का आषाढ़ी अधिवेशन कब बुलाया जाता है?
उत्तर:
13 अप्रैल को।

10. पंचायत समिति के अध्यक्ष एवं उपाध्यक्ष को निश्चित अवधि से पूर्व कैसे हटाया जा सकता है?
उत्तर:
2/3 बहुमत द्वारा।

11. पंचायती राज संस्थाओं की सफलता के मार्ग में कोई एक बाधा बताएँ।
उत्तर:
निरक्षरता।

12. बलवंत राय मेहता समिति ने देहातों के विकास के लिए किस प्रणाली को लागू करने का सुझाव दिया?
उत्तर:
पंचायती राज को लागू करने का सुझाव।

13. पंचायती राज संस्थाओं पर सरकार के नियंत्रण का कोई एक साधन लिखिए।
उत्तर:
प्रशासकीय नियंत्रण।

14. हरियाणा पंचायती राज अधिनियम राज्य विधानसभा द्वारा कब पारित किया गया?
उत्तर:
17 मार्च, 1994 को।

15. ग्राम सभा में विशेष बैठक या अधिवेशन हेतु कितने सदस्यों को अनुरोध करना पड़ता है?
उत्तर:
1/5 सदस्यों को।

HBSE 11th Class Political Science Important Questions Chapter 8 स्थानीय शासन

16. भारत में पंचायती राज व्यवस्था को संवैधानिक दर्जा कब दिया गया?
उत्तर:
73वें संवैधानिक संशोधन द्वारा सन् 1992 में।

17. हरियाणा में कितने नगर निगम हैं?
उत्तर:
10

18. नगर निगम के अध्यक्ष को क्या कहते हैं?
उत्तर:
मेयर।

19. हरियाणा पंचायती राज अधिनियम कब पारित किया गया?
उत्तर:
सन् 1994 में।

20. जिला-परिषद् का गठन किस स्तर पर किया जाता है?
उत्तर:
ज़िला-स्तर पर।

रिक्त स्थान भरें

1. 73वां संवैधानिक संशोधन सन् ……………. में लागू हुआ।
उत्तर:
24 अप्रैल, 1993

2. ग्राम पंचायत एवं जिला परिषद के मध्य ……………. का गठन किया गया है।
उत्तर:
पंचायत समिति

3. हरियाणा में पंचायती राज अधिनियम …………… को लागू किया गया।
उत्तर:
22 अप्रैल, 1994

4. ग्राम पंचायत के सदस्य हेतु न्यूनतम आयु …………. वर्ष होनी चाहिए।
उत्तर:
21

5. ग्राम पंचायत के अध्यक्ष को …………… कहते हैं।
उत्तर:
सरपंच

6. नगर निगम के सदस्य को ……………. कहते हैं।
उत्तर:
महापौर

7. नगर निगम के अध्यक्ष को ……………. कहते हैं।
उत्तर:
मेयर

8. हरियाणा में ……………. नगर निगम हैं।
उत्तर:
10

9. स्थानीय संस्थाओं पर ……………. सरकार का नियन्त्रण है।
उत्तर:
राज्य

10. नगरपालिका के सदस्य को ……………. कहा जाता है।
उत्तर:
नगर-पार्षद

HBSE 11th Class Political Science Important Questions Chapter 7 संघवाद

Haryana State Board HBSE 11th Class Political Science Important Questions Chapter 7 संघवाद Important Questions and Answers.

Haryana Board 11th Class Political Science Important Questions Chapter 7 संघवाद

अति लघूत्तरात्मक प्रश्न

प्रश्न 1.
संघात्मक सरकार किसे कहा जाता है?
उत्तर:
संघात्मक सरकार उसे कहा जाता है, जिसमें शासन की शक्तियाँ संविधान द्वारा केंद्र तथा राज्यों में बंटी हुई हों तथा प्रत्येक इकाई अपने अधिकार क्षेत्र में स्वतन्त्र हो।

प्रश्न 2.
भारत के संविधान में संघ के स्थान पर किन शब्दों का प्रयोग किया गया है?
उत्तर:
भारत के संविधान में भारत के लिए ‘संघ’ के स्थान पर ‘राज्यों का संघ’ शब्द का प्रयोग किया गया है।

प्रश्न 3.
भारतीय संविधान के कोई दो संघात्मक लक्षण बताएँ।
उत्तर:

  • भारतीय संविधान लिखित तथा कठोर है,
  • संविधान द्वारा केंद्र तथा राज्यों के बीच शक्तियों का बँटवारा किया गया है।

प्रश्न 4.
भारतीय संविधान के कोई दो एकात्मक लक्षण बताइए।
उत्तर:

  • संकटकालीन स्थिति की घोषणा होने पर हमारा संघीय ढाँचा एकात्मक में बदल जाता है।
  • भारतीय संविधान द्वारा देश के प्रत्येक नागरिक के लिए इकहरी नागरिकता के सिद्धान्त को अपनाया गया है।

प्रश्न 5.
भारत में केंद्रीय सरकार को अधिक शक्तिशाली बनाने के कोई दो कारण बताएँ।
उत्तर:

  • समस्त देश के आर्थिक विकास के लिए,
  • स्वतन्त्रता-प्राप्ति के समय देशी रियासतों की समस्या तथा भारत की एकता व अखण्डता को बनाए रखना।

प्रश्न 6.
भारतीय संविधान द्वारा शासन की शक्तियों को कितनी तथा कौन-सी सूचियों में बाँटा गया है? प्रत्येक सूची में दिए गए तीन-तीन विषय लिखिए।
उत्तर:
भारतीय संविधान द्वारा शासन की शक्तियों को तीन सूचियों में बाँटा गया है। ये हैं-

  • संघीय सूची,
  • राज्य सूची तथा
  • समवर्ती सूची।

संघीय सूची में विदेशी मामले, प्रतिरक्षा व रेलवे, राज्य सूची में कृषि, पुलिस व जेलें तथा समवर्ती सूची में शिक्षा, विवाह व तलाक आदि विषय शामिल हैं।

प्रश्न 7.
समवर्ती सूची में दिए गए विषयों के सम्बन्ध में कानून बनाने की शक्ति किसके पास है ?
उत्तर:
समवर्ती सूची में दिए गए विषयों पर संसद तथा राज्य विधानमण्डल दोनों को कानून बनाने का अधिकार है। दोनों द्वारा परस्पर विरोधी कानून बनाने की स्थिति में संसद द्वारा बनाया गया कानून लागू होगा।

HBSE 11th Class Political Science Important Questions Chapter 7 संघवाद

प्रश्न 8.
अवशिष्ट शक्तियाँ (Residuary Powers) किसे कहते हैं? भारतीय संविधान द्वारा ये शक्तियाँ किसे दी गई हैं?
उत्तर:
अवशिष्ट शक्तियाँ वे विषय हैं जिनका वर्णन तीनों सूचियों संघ सूची, राज्य सूची तथा समवर्ती सूची में से किसी में भी नहीं किया गया है। इन पर कानून बनाने की शक्ति संसद को दी गई है।

प्रश्न 9.
अन्तर्राज्यीय परिषद् (Inter-State Council) की स्थापना क्यों तथा किसके द्वारा की जाती है?
उत्तर:
अन्तर्राज्यीय परिषद् की स्थापना राज्यों के आपसी झगड़ों का निपटारा करने के लिए राष्ट्रपति द्वारा की जाती है।

प्रश्न 10.
पॉल एपलबी (Paul Appleby) ने भारतीय संघात्मक व्यवस्था को कैसी संघात्मक व्यवस्था का नाम दिया है?
उत्तर:
पॉल एपलबी (Paul Appleby) ने भारतीय संघवाद व्यवस्था को अत्यन्त संघात्मक (Extremely Federal) व्यवस्था का नाम दिया है।

प्रश्न 11.
सातवीं अनुसूची में कितनी सूचियाँ दी गई हैं और प्रत्येक में कितने विषय शामिल हैं?
उत्तर:
सातवीं अनुसूची में तीन सूचियाँ-संघ सूची, समवर्ती सूची व राज्य सूची हैं। संघ सूची में मूलतः 97 विषय (वर्तमान में 100 विषय), समवर्ती सूची में मूलतः 47 (वर्तमान में 52 विषय) विषय तथा राज्य सूची में मूलतः 66 विषय (वर्तमान में 61 विषय) शामिल हैं।

प्रश्न 12.
समवर्ती सूची में मूल रूप से कितने विषय थे और वर्तमान में कितने विषय हैं?
उत्तर:
समवर्ती सूची में मूल रूप से 47 विषय थे और वर्तमान में 52 विषय हैं।

प्रश्न 13.
संघ सूची में मूल रूप से कितने विषय थे और वर्तमान में कितने विषय हैं ?
उत्तर:
संघ सूची में मूल रूप से 97 विषय थे और वर्तमान में भी 100 विषय ही हैं।

प्रश्न 14.
राज्य सूची में मूल रूप से कितने विषय थे और अब कितने विषय हैं?
उत्तर:
राज्य सूची में मूल रूप से 66 विषय थे और वर्तमान में 61 विषय हैं।

प्रश्न 15.
राज्य सूची में दिए गए किन्हीं दो विषयों के नाम बताइए।
उत्तर:
राज्य सूची में निम्नलिखित दो विषय हैं-

  • कानून व शान्ति-व्यवस्था,
  • कृषि।

प्रश्न 16.
समवर्ती सूची में दिए गए दो विषयों के नाम बताइए।
उत्तर:
समवर्ती सूची में निम्नलिखित दो विषय हैं-

  • दण्ड विधान,
  • विवाह।

प्रश्न 17.
केंद्र की आय के दो साधन बताइए।
उत्तर:
केंद्र की आय के दो साधन हैं-

  • सीमा-शुल्क,
  • आय-कर।

प्रश्न 18.
राज्यों की आय के दो साधन बताइए।
उत्तर:
राज्यों की आय के दो साधन हैं

  • बिक्री-कर
  • कषि भूमि के उत्तराधिकार के विषय में शल्क।

प्रश्न 19.
केंद्र व राज्यों के बीच तनाव के दो कारण बताइए।
उत्तर:
केंद्र व राज्यों के बीच तनाव के दो प्रमुख कारण हैं-

  • राज्यपालों की भेदभावपूर्ण तथा विवादास्पद भूमिका,
  • केंद्र द्वारा उन राज्यों के साथ भेदभाव किया जाना, जिनमें विपक्षी दलों की सरकारें होती हैं।

HBSE 11th Class Political Science Important Questions Chapter 7 संघवाद

प्रश्न 20.
वित्त आयोग की नियुक्ति कौन करता है और उसके क्या कार्य हैं?
अथवा
वित्त आयोग की स्थापना क्यों की जाती है? इसके सदस्यों की नियुक्ति कौन करता है?
उत्तर:
वित्त आयोग की नियुक्ति राष्ट्रपति करता है। वित्त आयोग देश की वित्तीय व्यवस्था का परीक्षण करता है और केंद्र व राज्यों के बीच वित्तीय सम्बन्धों के बारे में भी सिफारिश करता है।

प्रश्न 21.
केंद्र-राज्य सम्बन्धों पर पुनः विचार करने के लिए कब और किस आयोग की स्थापना की गई थी? अथवा सरकारिया आयोग की स्थापना कब और किस उद्देश्य के लिए की गई थी?
उत्तर:
9 जून, 1983 को केंद्र-राज्य सम्बन्धों पर पुनः विचार करने के लिए केंद्रीय सरकार ने सरकारिया आयोग की स्थापना की थी, जिसके अध्यक्ष उच्चतम न्यायालय के भूतपूर्व न्यायाधीश श्री आर०एस० सरकारिया थे।

प्रश्न 22.
सरकारिया आयोग की कोई दो सिफारिशें बताइए।
उत्तर:
सरकारिया आयोग की दो प्रमुख सिफारिशें थीं-

  • केंद्र को राज्यों में केंद्रीय पुलिस बल नियुक्त करने का अधिकार बना रहे,
  • अन्तर्राज्यीय परिषदों की स्थापना की जाए।

प्रश्न 23.
उन दो परिस्थितियों को बताएँ जिनमें कि संसद राज्य सूची पर कानून बना सकती है।
उत्तर:
संसद राज्य सूची पर दी गई दो परिस्थितियों में कानून बना सकती है-

  • अन्तर्राष्ट्रीय सन्धि व समझौते लागू करने पर,
  • जब राज्यसभा राज्य सूची के किसी विषय को 2/3 बहुमत से राष्ट्रीय महत्त्व को घोषित कर दे।

प्रश्न 24.
केंद्र व राज्यों के बीच दो विधायी सम्बन्ध बताइए।
उत्तर:
केंद्र व राज्यों के बीच दो विधायी सम्बन्ध इस प्रकार हैं-(1) समवर्ती सूची के विषय पर केंद्र व राज्य दोनों ही कानून बना सकते हैं, (2) राज्यपाल विधान सभा द्वारा पारित विधेयक को राष्ट्रपति के पास भेज सकता है।

प्रश्न 25.
केंद्र व राज्यों के बीच दो वित्तीय सम्बन्ध बताइए।
उत्तर:
केंद्र व राज्यों के बीच दो वित्तीय सम्बन्ध इस प्रकार हैं-

  • केंद्र राज्यों को अनुदान देता है,
  • वित्तीय संकट की स्थिति में केंद्र राज्यों की आय के साधनों में परिवर्तन कर सकता है।

प्रश्न 26.
केंद्र व राज्यों के बीच दो प्रशासनिक सम्बन्ध बताइए।
उत्तर:
केंद्र व राज्यों के बीच दो प्रशासनिक सम्बन्ध इस प्रकार हैं-

  • केंद्र राज्यों में केंद्रीय पुलिस बल भेज सकता है,
  • राज्य के उच्च प्रशासनिक अधिकारी अखिल भारतीय सेवाओं के सदस्य होते हैं।

प्रश्न 27.
केंद्र कब और किस आधार पर राज्य में आपात स्थिति लागू कर सकता है?
उत्तर:
केंद्र राज्य में संवैधानिक तन्त्र विफल हो जाने पर आपात स्थिति लागू कर सकता है। ऐसा केंद्र राज्यपाल की रिपोर्ट पर अथवा बिना रिपोर्ट के भी कर सकता है।

प्रश्न 28.
केंद्र-राज्य सम्बन्धों पर विचार-विमर्श हेतु गठित सरकारिया आयोग ने अपनी रिपोर्ट कब प्रस्तुत की?
उत्तर:
केंद्र-राज्य सम्बन्धों पर विचार-विमर्श हेतु गठित सरकारिया आयोग ने अपनी रिपोर्ट 27 अक्तूबर, 1987 को प्रस्तुत की।

प्रश्न 29.
राज्य की स्वायत्तता की मांग के कोई दो कारण लिखें।
उत्तर:
राज्यों की स्वायत्तता की माँग के दो कारण इस प्रकार हैं-

  • संसद की व्यापक विधि निर्माण शक्तियाँ,
  • वित्तीय दृष्टि से राज्यों की केंद्र पर निर्भरता।

HBSE 11th Class Political Science Important Questions Chapter 7 संघवाद

प्रश्न 30.
राज्य स्वायत्तता के कोई दो साधन लिखें।
उत्तर:
राज्यों की स्वायत्तता के लिए विभिन्न सुझाए गए साधनों में दो निम्नलिखित हैं-

  • संविधान में संघात्मक शासन का स्थापित किया जाना,
  • राष्ट्रपति के परामर्श हेतु एक समिति का गठन हो जो उसे निष्पक्ष परामर्श प्राप्त करा सके।

लघूत्तरात्मक प्रश्न

प्रश्न 1.
संघात्मक सरकार से क्या अभिप्राय है? संक्षेप में व्याख्या करें।
उत्तर:
संघ जिसे अंग्रेजी में (Federation) अथवा (Federal) कहा जाता है, वास्तव में लैटिन भाषा के एक शब्द (Foedus) से बना है जिसका अर्थ है सन्धि अथवा समझौता। इस प्रकार संघ सरकार कुछ राज्यों का एक ऐसा स्थायी संगठन है, जिसकी स्थापना एक समझौते के आधार पर की जाती है। जब दो या अधिक स्वतन्त्र राज्य कुछ सामान्य उद्देश्यों की पूर्ति के लिए एक केंद्रीय सरकार संगठित करते हैं तथा शेष उद्देश्यों की पूर्ति वे स्वयं करते हैं तो एक संघात्मक शासन की स्थापना हो जाती है।

प्रश्न 2.
संघात्मक सरकार के चार लक्षण बताएँ।।
उत्तर:
संघात्मक सरकार के चार लक्षण निम्नलिखित हैं
1. शक्तियों का बँटवारा संघात्मक सरकार में शक्तियों का बँटवारा होता है। महत्त्वपूर्ण शक्तियाँ केंद्र के पास तथा स्थानीय महत्त्व की शक्तियाँ राज्य सरकारों को दी जाती हैं। दोनों इकाइयाँ अपने-अपने अधिकार क्षेत्र में स्वतन्त्र होती हैं।

2. लिखित तथा कठोर संविधान संघ राज्य कई राज्यों के बीच एक समझौते का परिणाम होता है, इसलिए समझौते की सभी शर्ते लिखित रूप में होनी चाहिएँ। साथ ही ये शर्ते स्थायी हों, अतः संविधान लिखित तथा कठोर होता है।

3. स्वतन्त्र तथा सर्वोच्च न्यायालय केंद्र तथा राज्यों के आपसी विवादों को निपटाने के लिए तथा कोई भी इकाई संविधान के विरुद्ध कानून न बना सके, इसके लिए एक स्वतन्त्र तथा सर्वोच्च न्यायालय की स्थापना की जाती है।

4. दो-सदनीय विधानपालिका-संघात्मक सरकार में दो-सदनीय विधानमण्डल की आवश्यकता होती है। विधानमण्डल का निम्न सदन राष्ट्र की जनता का प्रतिनिधित्व करता है तथा ऊपरी सदन संघ की इकाइयों का प्रतिनिधित्व करता है।

प्रश्न 3.
संघात्मक सरकार के चार लाभ बताएँ।
उत्तर:
संघात्मक सरकार के चार मुख्य लाभ निम्नलिखित हैं
1. शक्तिशाली राज्य की स्थापना-संघ सरकार स्थापित होने से छोटे-छोटे राज्यों को मिलाकर एक शक्तिशाली संघ राज्य कायम हो जाता है।

2. सरकार में अधिक कार्यकुशलता-संघ सरकार में केंद्र तथा राज्यों में शक्तियों तथा कार्य-क्षेत्रों का बँटवारा हो जाने से सरकारों की प्रशासनिक दक्षता बढ़ जाती है।

3. बड़े राज्यों के लिए उपयोगी-संघ सरकार अधिक जनसंख्या तथा भिन्नता वाले राज्यों के लिए उपयुक्त है।

4. अधिक लोकतन्त्रीय संघ सरकार में लोकतन्त्र की संस्थाएँ अधिक तथा प्रत्येक स्तर पर संगठित की जाती हैं। इसमें स्थानीय स्वशासन संस्थाएँ लोगों को लोकतन्त्र का प्रशिक्षण देती हैं।

प्रश्न 4.
भारतीय संविधान संघात्मक है, स्पष्ट करो।
अथवा
भारतीय संविधान के उन प्रावधानों का उल्लेख करें जो इसे संघीय स्वरूप प्रदान करते हैं।
अथवा
भारतीय संविधान की कोई पाँच संघीय विशेषताओं का वर्णन करें।
उत्तर:
भारतीय संविधान की संघीय विशेषताएँ (लक्षण) इस प्रकार हैं-
(1) भारतीय संविधान द्वारा शासन की शक्तियों का तीन सूचियों में विभाजन किया गया है-

1. संघीय सूची (Union List), इस सूची में राष्ट्रीय महत्त्व के मूलतः 97 विषय (वर्तमान में 100 विषय) हैं जिन पर कानून बनाने की शक्ति संघीय संसद के पास है।

2. राज्य सूची (State List) में मूलतः 66 विषय (वर्तमान में 61 विषय) हैं। ये विषय स्थानीय महत्त्व के हैं और उन पर कानून बनाने का अधिकार राज्यों के विधानमण्डलों को दिया गया है।

3. समवर्ती सूची (Concurrent List), इस सूची में दिए गए मूल 47 विषयों (वर्तमान में 52 विषय) पर संसद तथा राज्य विधानमण्डल दोनों ही कानून बना सकते हैं,
(2) संविधान देश का सर्वोच्च कानून (Supremacy of the Constitution) है,
(3) भारतीय संविधान लिखित तथा कठोर है,
(4) संविधान की रक्षा करने के लिए सर्वोच्च न्यायालय की स्थापना की गई है,
(5) संघीय विधानमण्डल (संसद) का गठन द्वि-सदनीय प्रणाली के आधार पर किया गया है। लोकसभा देश की जनता का प्रतिनिधित्व करती है, जबकि राज्यसभा में राज्यों के प्रतिनिधि बैठते हैं।

HBSE 11th Class Political Science Important Questions Chapter 7 संघवाद

प्रश्न 5.
भारत में शक्तिशाली केंद्र की स्थापना के पाँच कारण बताएँ।
उत्तर:
भारत में शक्तिशाली केंद्र की स्थापना के कारण इस प्रकार हैं-

  • ऐतिहासिक अनुभव के कारण संविधान के निर्माताओं ने शक्तिशाली केंद्रीय सरकार की स्थापना की,
  • तत्कालीन परिस्थितियों भारत का विभाजन, साम्प्रदायिक दंगों, तेलंगाना में सशस्त्र किसान आन्दोलन, भारतीय देशी रियासतों की समस्या इत्यादि ने भी संविधान निर्माताओं को प्रभावित किया,
  • भारतीय आर्थिक समस्याओं के समाधान हेतु भी केंद्रीय शासन का शक्तिशाली होना अनिवार्य था,
  • समस्त संसार में केंद्रीयकरण की प्रवृत्ति विद्यमान है, भारत इसका कोई अपवाद नहीं है,
  • स्वतन्त्रता प्राप्ति के समय भारत में 562 देशी रियासतें मौजूद थीं। संविधान निर्माताओं ने देशी रियासतों की समस्या से निपटने के लिए शक्तिशाली केंद्रीय सरकार की स्थापना करने का निर्णय दिया।

प्रश्न 6.
केंद्र तथा राज्यों में कोई छह प्रशासनिक सम्बन्ध बताएँ।
उत्तर:
केंद्र तथा राज्यों में प्रशासनिक सम्बन्ध इस प्रकार हैं-

  • केंद्रीय सरकार को राज्य सरकारों को निर्देश तथा आदेश देने का अधिकार है,
  • राज्यपाल केंद्रीय सरकार के एजेण्ट के रूप में कार्य करता है,
  • बाहरी आक्रमण अथवा सशस्त्र विद्रोह की स्थिति में केंद्रीय सरकार द्वारा प्रान्तीय सरकारों को निर्देश दिया जाता है,
  • राष्ट्रपति को प्रान्तीय संकट की घोषणा (अनुच्छेद 356) करने का अधिकार है,
  • राज्यों के आपसी झगड़ों का निपटारा करने के लिए राष्ट्रपति अन्तर्राज्यीय परिषद् (Inter-State Council) की स्थापना कर सकता है,
  • राष्ट्रपति अनुसूचित जातियों, जन-जातियों तथा अन्य पिछड़ी हुई जातियों के कल्याण को ध्यान में रखते हुए राज्यों को निर्देश दे सकता है।

प्रश्न 7.
अर्द्ध-संघात्मक (Quasi-Federal) से क्या अभिप्राय है? संक्षेप में व्याख्या करें।
उत्तर:
भारत में संघात्मक सरकार की स्थापना की गई है, परन्तु पूर्ण संघात्मक नहीं, अर्द्ध-संघात्मक। अर्द्ध-संघात्मक से अभिप्राय है कि केंद्र तथा राज्यों में शक्तियों का बँटवारा तो किया गया है, परन्तु शक्ति सन्तुलन केंद्र के पक्ष में है। संघ के अधिक शक्तिशाली होने के कारण भारतीय संघीय व्यवस्था को अर्द्ध-संघात्मक कहा जाता है। प्रो० के०सी० व्हीयर के शब्दों में, “भारत का नया संविधान ऐसी शासन-व्यवस्था को जन्म देता है जो अधिक-से-अधिक अर्द्ध-संघीय है।”

प्रश्न 8.
राज्यों की स्वायत्तता पर संक्षेप में एक लेख लिखिए। .
उत्तर:
संविधान के लागू होने से लेकर अब तक केंद्र-राज्यों के सम्बन्धों में तनाव है। राज्य अधिक-से-अधिक स्वायत्तता की माँग करते रहे हैं तथा केंद्र के नियन्त्रण को कम करने के लिए संघर्ष करते रहे हैं। राज्यों की स्वायत्तता का अर्थ है कि राज्यों को अपने आन्तरिक क्षेत्र में अपनी शक्तियों का प्रयोग करने की स्वतन्त्रता हो। संविधान द्वारा जो शक्तियाँ राज्यों को दी गई हैं, उनमें केंद्र हस्तक्षेप न करे।

प्रश्न 9.
संसद किन परिस्थितियों में राज्य सूची में दिए गए विषयों के सम्बन्ध में कानून बना सकती है?
उत्तर:
ये परिस्थितियाँ इस प्रकार हैं-

  • देश में संकटकालीन स्थिति की घोषणा होने पर,
  • यदि राज्यसभा दो-तिहाई बहुमत से यह प्रस्ताव पास कर दे कि राज्य सूची में दिया गया कोई विषय राष्ट्रीय महत्त्व का बन गया है और उस पर संसद को कानून बनाना चाहिए,
  • किसी अन्तर्राष्ट्रीय सन्धि अथवा समझौते को लागू करने के लिए,
  • यदि दो अथवा दो से अधिक राज्यों के विधानमण्डल प्रस्ताव पास करके संसद को ऐसा करने की प्रार्थना करें,
  • किसी राज्य में राष्ट्रपति शासन लागू होने की स्थिति में उस राज्य के लिए।

प्रश्न 10.
वित्त आयोग पर संक्षिप्त टिप्पणी लिखिए।
उत्तर:
संविधान की धारा 280 के अन्तर्गत यह व्यवस्था की गई है कि देश की आर्थिक परिस्थिति का अध्ययन करने के लिए राष्ट्रपति द्वारा समय-समय पर एक वित्त आयोग की नियुक्ति की जाएगी। वित्त आयोग की स्थापना प्रायः 5 वर्ष के लिए की जाती है। इस आयोग के सदस्यों की योग्यताएँ तथा नियुक्ति का तरीका निश्चित करने का अधिकार संसद को प्राप्त है। वित्त आयोग सरकार को दी गई कुछ बातों के बारे में परामर्श देता है

  • संघ तथा राज्यों में राजस्व का विभाजन,
  • केंद्र सरकार द्वारा राज्यों को दिए जाने वाले अनुदान की मात्रा के बारे में,
  • अन्य कोई भी मामला जो राष्ट्रपति द्वारा आयोग को सौंपा गया है। अब तक पन्द्रह (11 अप्रैल 2020 से) वित्त आयोग नियुक्त किए जा चुके हैं।

प्रश्न 11.
‘अवशिष्ट शक्तियाँ’ (Residuary Powers) पर संक्षिप्त नोट लिखें।
उत्तर:
संविधान के अनुच्छेद 248 के अनुसार उन सब विषयों को, जिनका वर्णन किसी भी सूची अर्थात् संघीय सूची, राज्य सूची, अथवा समवर्ती सूची में नहीं किया गया है, उन्हें अवशिष्ट शक्तियों का नाम दिया गया है। इन विषयों पर कानून बनाने का अधिकार संसद के पास है। अमेरिका में अवशिष्ट शक्तियाँ केंद्र को नहीं, बल्कि राज्य सरकारों को सौंपी गई हैं।

प्रश्न 12.
सरकारिया आयोग पर संक्षेप में लेख लिखिए।
उत्तर:
सन् 1983 में तत्कालीन प्रधानमन्त्री श्रीमती इन्दिरा गाँधी ने केंद्र व राज्यों के सम्बन्धों पर विचार करके रिपोर्ट देने के । लिए न्यायमूर्ति श्री आर०एस० सरकारिया के नेतृत्व में तीन-सदस्यीय आयोग का गठन किया। इस आयोग ने भारतीय संविधान के अन्तर्गत केंद्र तथा राज्यों के सम्बन्धों को ठोस बताया और अपनी रिपोर्ट में कहा कि इसमें किसी परिवर्तन की आवश्यकता नहीं है। केवल कार्य-प्रणाली में परिवर्तन की आवश्यकता है और उन्हें ईमानदारी से लागू करना है। केंद्र सरकार ने इन सुझावों के आधार पर कोई ठोस कार्य नहीं किया।

प्रश्न 13.
योजना आयोग पर संक्षिप्त टिप्पणी लिखिए।
उत्तर:
योजना आयोग एक संविधानोत्तर संस्था है, क्योंकि संविधान में इसकी स्थापना की कोई व्यवस्था नहीं है। भारत में योजना आयोग की स्थापना भारत सरकार द्वारा 15 मार्च, 1950 को एक प्रस्ताव पारित करके की गई थी और 28 मार्च, 1950 से योजना आयोग ने अपना कार्य प्रारंभ कर दिया था परन्तु 16वीं लोकसभा चुनाव के बाद नवगठित भाजपा नेतृत्व वाली सरकार ने 65 वर्ष पुराने योजना आयोग के स्थान पर नेशनल इंस्टीट्यूशन फॉर ट्रांसफॉर्मिंग इण्डिया (NITI) आयोग का गठन करने का निर्णय लिया जिसके परिणामस्वरूप एक नई संस्था नीति (NITI-National Institute for Transforming India) का मार्ग प्रशस्त किया तथा 5 जनवरी, 2015 को इसकी नियुक्ति कर दी गई।

यद्यपि यहाँ हम सर्वप्रथम पाठ्यक्रम के अनुसार योजना आयोग की ऐतिहासिक पृष्ठभूमि के अनुसार योजना आयोग के स्वरूप, इसके कार्यों एवं भारत के विकास में योजना आयोग द्वारा निभाई गई भूमिका का भी संक्षेप में उल्लेख करेंगे एवं तत्पश्चात् नवगठित नीति आयोग की संरचना, उद्देश्यों एवं कार्यों पर भी संक्षेप में प्रकाश डालेंगे।

भारत में 1950 में कार्यरत योजना आयोग में प्रधानमंत्री इसका अध्यक्ष (Ex-officio Chairman) होता था और वही इसकी बैठकों की अध्यक्षता करता था। इसके अतिरिक्त आयोग का एक उपाध्यक्ष होता था जिसकी नियुक्ति मंत्रिमंडल के द्वारा की जाती थी। उपाध्यक्ष पद पर देश के विख्यात अर्थशास्त्री या प्रसिद्धि प्राप्त वित्त विशेषज्ञ विराजमान रहे हैं।

यद्यपि उपाध्यक्ष मंत्रिमंडल का सदस्य नहीं होता था परंतु उसका स्तर कैबिनेट मंत्री के समान होता था। उपाध्यक्ष वास्तव में योजना आयोग का सबसे प्रभावकारी अधिकारी होता था। इसके अतिरिक्त उपाध्यक्ष को कैबिनेट मंत्री के समान वेतन एवं भत्ते मिलते थे। अध्यक्ष और उपाध्यक्ष के अलावा योजना आयोग में 14-15 व्यक्ति और होते थे।

इनमें सात-आठ तो मंत्री शामिल थे; जैसे मानव संसाधन और विकास मंत्री, वित्त मंत्री, गृह मंत्री, रक्षा मंत्री, कृषि मंत्री और योजना-राज्य मंत्री तथा पांच-छः अन्य सदस्य होते थे। सदस्यों में से कोई एक सदस्य आयोग के सचिव (Member Secretary) के रूप में कार्य करता था। आयोग के विशेषज्ञ सदस्यों को केंद्रीय राज्य मंत्री का दर्जा दिया जाता था।

प्रश्न 14.
समवर्ती सूची पर एक नोट लिखिए।
उत्तर:
इस सूची में साधारणतः वे विषय रखे गए हैं, जिनका महत्त्व क्षेत्रीय व संघीय दोनों ही दृष्टियों से है। इस सूची के विषयों पर संघ तथा राज्य, दोनों को ही कानून बनाने का अधिकार प्राप्त है। यदि इस सूची के किसी विषय पर संघीय तथा राज्य व्यवस्थापिका द्वारा निर्मित कानून परस्पर विरोधी हों, तो सामान्यतः संघ का कानून मान्य होगा।

इस सूची में वर्तमान समय में कुल 52 विषय हैं, जिनमें से कुछ प्रमुख हैं फौजदारी विषय, विवाह और विवाह-विच्छेद, दत्तक और उत्तराधिकार, कारखाने, श्रमिक-संघ, औद्योगिक विवाद, आर्थिक और सामाजिक योजना, सामाजिक सुरक्षा और सामाजिक बीमा, पुनर्वास और पुरातत्व आदि। 42वें संवैधानिक संशोधन में 4 विषय शिक्षा, वन, जंगली जानवरों और पक्षियों की रक्षा और नाप-तोल राज्य सूची में से समवर्ती सूची में परिवर्तित कर दिए गए हैं। इसके अतिरिक्त समवर्ती सूची में एक नवीन विषय-‘जनसंख्या नियन्त्रण’ और ‘परिवार नियोजन’ रखा गया है।

प्रश्न 15.
केंद्र तथा राज्यों के बीच तनाव के पाँच कारण लिखें।
उत्तर:
केंद्र तथा राज्यों के बीच तनाव के पाँच कारण निम्नलिखित हैं-
(1) केंद्र तथा राज्यों के बीच तनाव का मुख्य कारण वित्त रहा है। राज्यों को हमेशा केंद्र से यह शिकायत रहती है कि वह सहायता देते समय भेदभावपूर्ण नीति अपनाता है,

(2) केंद्र व राज्यों के बीच तनाव का कारण राज्यपाल की भूमिका भी है। राज्यपाल राष्ट्रपति के एजेन्ट के रूप में कार्य करता है। विशेष रूप से ऐसे राज्यों में जहाँ विरोधी दलों की सरकारें होती हैं, वहाँ राज्यपाल की भूमिका विवा का विषय बनी रहती है,

(3) नौकरशाही की भूमिका भी तनाव का अन्य कारण है, क्योंकि प्रशासनिक अधिकारियों की नियुक्ति केंद्र द्वारा किए जाने के कारण राज्य सरकारों का उन पर नियन्त्रण नहीं होता,

(4) कानून तथा व्यवस्था की समस्याएँ भी तनाव का कारण हैं, क्योंकि केंद्र शान्ति व व्यवस्था का बहाना लेकर राज्य के आन्तरिक मामलों में हस्तक्षेप करता है,

(5) दलीय भावना के कारण भी केंद्र राज्यों तथा राज्य केंद्र पर दोषारोपण करते रहते हैं, विशेष रूप से उस समय जब केंद्र में एक दल की सरकार हो तथा राज्य में किसी दूसरे दल की।

HBSE 11th Class Political Science Important Questions Chapter 7 संघवाद

प्रश्न 16.
राज्य की स्वायत्तता से आप क्या समझते हैं? .
उत्तर:
राज्यों की स्वायत्तता का अर्थ स्वतन्त्रता नहीं है, बल्कि इसका अर्थ है कि राज्यों को उनके मामलों में केंद्रीय सरकार द्वारा किसी भी प्रकार का हस्तक्षेप न किया जाना। राज्यों को जो शक्तियाँ संविधान द्वारा उपलब्ध करवाई गई हैं उन्हें उनका प्रयोग बिना किसी रोक-टोक के करने की आज्ञा होनी चाहिए। इस प्रकार राज्यों की स्वायत्तता का अर्थ न तो राज्यों की स्वतन्त्रता से है और न ही प्रभुसत्ता से। यह एक ऐसा वैधानिक दर्जा है जिसमें राज्यों को कुछ क्षेत्रों में पूर्ण स्वतन्त्रता तथा कम-से-कम केंद्रीय हस्तक्षेप का आश्वासन प्राप्त होता है। राज्यों को अपने एक निश्चित क्षेत्र में स्वतन्त्रतापूर्वक कार्य के अधिकार का नाम ही राज्यों की स्वायत्तता है।

प्रश्न 17.
राज्य की स्वायत्तता की माँग के मुख्य कारणों का उल्लेख करो।
उत्तर:
राज्य की स्वायत्तता की माँग के मुख्य कारण निम्नलिखित हैं-

  • संसद की व्यापक विधि निर्वाण शक्तियाँ,
  • वित्तीय दृष्टि से राज्यों की केंद्र पर निर्भरता,
  • अखिल भारतीय सेवाएँ तथा राज्यपाल,
  • राज्यों के बीच भाषायी एवं सांस्कृतिक विभिन्नता,
  • राज्यपाल की भूमिका एवं राष्ट्रपति शासन,
  • अन्तर्राष्ट्रीय झगड़े,
  • राज्यसभा में राज्यों का असमान प्रतिनिधित्व ।

प्रश्न 18.
भारतीय संविधान की कोई तीन एकात्मक विशेषताएँ लिखिए।
उत्तर:
भारतीय संविधान की तीन एकात्मक विशेषताएँ निम्नलिखित हैं

1. शक्तिशाली केंद्रीय सरकार-वैसे तो संविधान ने केंद्र तथा राज्य सरकारों के बीच शक्तियों का स्पष्ट विभाजन किया है, परंतु इस विभाजन में केंद्र को अधिक शक्तियाँ दी गई हैं। संघीय सूची में 97 विषय हैं, जबकि राज्य सूची में केवल 66 विषय हैं। राज्य विषयों की संख्या ही कम नहीं, इनका महत्त्व भी कम है। जेलें, पुलिस तथा अन्य स्थानीय विषयों पर ही राज्य सरकारों को कानून बनाने का अधिकार है।

समवर्ती सूची में दिए गए विषयों पर राज्य सरकारें तथा केंद्रीय सरकारें दोनों ही कानून बना सकती हैं, परंतु मतभेद या विरोध की स्थिति में केंद्रीय मंत्रिमंडल द्वारा बनाया गया कानून ही माना जाएगा और राज्य विधानमंडल के कानून को रद्द कर दिया जाएगा। अवशिष्ट शक्तियों (Residuary Powers) के विषय भी केंद्रीय सरकार को ही सौंपे गए हैं।

2. राज्यों के राज्यपालों की नियुक्ति राज्यों के राज्यपालों की नियुक्ति भी राष्ट्रपति प्रधानमंत्री की सलाह से करता है। वह जिस राज्यपाल को जब चाहे, उसके पद से हटा सकता है। इस प्रकार गवर्नरों द्वारा भी केंद्र सरकार राज्य सरकारों के शासन-प्रबंध में हस्तक्षेप कर सकती है।

3. राज्यों की सीमाओं में परिवर्तन-संसद कानून पास कर किसी भी राज्यों की सीमाओं में परिवर्तन करने का अधिकार रखती है। वह दो या दो से अधिक राज्यों को मिलाकर एक राज्य बना सकती है या एक राज्य को दो भागों में बाँट सकती है; जैसे पंजाब का विभाजन करके (पंजाब व हरियाणा) दो राज्य बनाए गए थे। यह अधिकार केंद्र को बहुत अधिक शक्ति देता है। इससे वह चाहे तो राज्यों पर निरंकुश होकर नियंत्रण कर सकता है।

निबंधात्मक प्रश्न

प्रश्न 1.
संघीय सरकार की परिभाषा देते हुए इसकी विशेषताओं का वर्णन करें।
अथवा
संघ किसे कहते हैं? इसके आवश्यक लक्षणों का वर्णन कीजिए।
अथवा
संघात्मक सरकार की परिभाषा दीजिए तथा उसके गुण-दोषों की विवेचना कीजिए।
अथवा
संघ क्या है? संघ की सफलता में सहायक अनिवार्य तत्त्वों का वर्णन कीजिए।
अथवा
संघात्मक सरकार की परिभाषा दें। एकात्मक तथा संघात्मक सरकार में भेद बतलाइए। अथवा संघ क्या होता है? एक अच्छे संघ के निर्माण के लिए कौन-से तथ्य सहायक होते हैं? अथवा संघ क्या होता है? भारत में कौन-सी सरकार उपयुक्त है?
उत्तर:
संघात्मक सरकार ऐसी शासन-व्यवस्था है, जिसमें शासन-सत्ता का विकेंद्रीयकरण किया जाता है तथा जिसमें दोहरी सरकारें स्थापित की जाती हैं और उनकी शक्तियों का बँटवारा कर दिया जाता है।

संघ जिसे अंग्रेज़ी में ‘Federation’ अथवा ‘Federal’ कहा जाता है, वास्तव में लैटिन भाषा के एक शब्द ‘फोडस’ (Foedus) से बना है जिसका अर्थ है ‘सन्धि अथवा समझौता’ । इस प्रकार संघ सरकार कुछ राज्यों का एक ऐसा स्थायी संगठन है, जिसकी स्थापना एक समझौते के आधार पर की जाती है। जब दो अथवा दो से अधिक स्वतन्त्र राज्य कुछ सामान्य उद्देश्यों की पूर्ति के लिए एक केंद्रीय सरकार संगठित करते हैं तथा शेष उद्देश्यों की पूर्ति वे स्वयं करते हैं तो एक संघात्मक शासन की स्थापना हो जाती है। विभिन्न विद्वानों ने संघ सरकार की निम्नलिखित परिभाषाएँ दी हैं

1. मॉण्टेस्क्यू (Montesquieu):
के शब्दों में, “संघात्मक सरकार एक ऐसा समझौता है जहाँ बहुत-से एक जैसे राज्य बड़े राज्य के सदस्य बनने के लिए सहमत हों।”

2. हेमिल्टन (Hemilton):
का कथन है, “संघ राज्य, राज्यों का एक ऐसा समुदाय है जो एक नवीन राज्य की स्थापना करता है।”

3. गार्नर (Garner):
का कथन है, “संघ सरकार एक ऐसी प्रणाली है जिसमें केंद्रीय तथा स्थानीय सरकारें एक ही प्रभुसत्ता के अधीन होती हैं। ये सरकारें संविधान द्वारा अथवा संसदीय कानून द्वारा निर्धारित अपने-अपने क्षेत्रों में सर्वोच्च होती हैं।”

4. जेलीनेक (Jelineck):
के अनुसार, “संघात्मक राज्य कई राज्यों के मेल से बना हुआ एक प्रभुसत्ता सम्पन्न राज्य है।”

5. फाइनर (Finer):
के शब्दों में, “संघात्मक राज्य वह राज्य है जिसमें अधिकार और शक्ति का कुछ भाग स्थानीय राज्यों को दिया जाए, दूसरा भाग संघात्मक सरकार को दिया जाए जो कि अपने स्थानीय राज्यों की इच्छा से बनी होती है।”

इन परिभाषाओं के आधार पर संघात्मक सरकार वह शासन-प्रणाली है, जिसमें कई स्वतन्त्र राज्य मिलकर समान उद्देश्य की प्राप्ति के लिए एक संघ स्थापित कर लेते हैं। इस संघ में प्रत्येक सदस्य-राज्य कुछ विशेष क्षेत्रों में अपनी स्वतन्त्रता बनाए रखता है तथा सामान्य हित के विषयों को एक केंद्रीय सत्ता के सुपुर्द कर देता है। संघात्मक सरकार के लक्षण (Features of Federal Government)-संघात्मक सरकार के आवश्यक लक्षण अथवा विशेषताएँ निम्नलिखित हैं-

1. लिखित, कठोर तथा सर्वोच्च संविधान संघ राज्य कई राज्यों के बीच एक समझौते का परिणाम होता है, इसलिए समझौते की सभी शर्ते लिखित रूप में होनी चाहिएँ। साथ ही ये शर्ते स्थायी भी हों। इसलिए संघ सरकार का संविधान केवल लिखित ही नहीं, कठोर भी होता है, जिससे कोई भी इकाई मनमाने ढंग से इसमें परिवर्तन न कर सके। संविधान सर्वोच्च भी होता है ताकि कोई भी सरकार उस संविधान के विरुद्ध कानून बनाकर दूसरी इकाई के अधिकार क्षेत्र में हस्तक्षेप न कर सके।

2. शक्तियों का बँटवारा-संघात्मक सरकार में केंद्रीय महत्त्व के विषय केंद्रीय सरकार को तथा प्रान्तीय और स्थानीय महत्त्व के विषय राज्य सरकारों को सौंप दिए जाते हैं। दोनों सरकारें अपने-अपने क्षेत्र में कानून बनाती तथा प्रशासन चलाती हैं। वे एक-दूसरे के मामले में हस्तक्षेप नहीं करतीं। भारत में शक्तियों के विभाजन के अधीन तीन सूचियाँ केंद्रीय सूची, राज्य सूची तथा समवर्ती सूची बनाई गई हैं।

3. स्वतन्त्र तथा सर्वोच्च न्यायपालिका-संघात्मक शासन में एक निष्पक्ष तथा स्वतन्त्र संघीय न्यायालय का होना भी जरूरी है। संघ सरकार में यद्यपि केंद्र व राज्यों में अधिकारों का स्पष्ट विभाजन किया जाता है, फिर भी उनमें कई बातों में विवाद होना स्वाभाविक है। संघ न्यायालय उनके विवादों को हल करता है। यह न्यायालय संविधान के संरक्षण का भी कार्य करता है। केंद्रीय तथा प्रान्तीय सरकारें संविधान के विरुद्ध कानून न बना सकें, इसके लिए एक स्वतन्त्र तथा सर्वोच्च न्यायपालिका का होना बहुत आवश्यक है।

4. द्वि-सदनीय विधानपालिका-संघात्मक सरकार में द्वि-सदनीय विधानमण्डल की आवश्यकता पड़ती है। विधानमण्डल का निम्न सदन सारे राष्ट्र की जनता का तथा उच्च सदन संघ की इकाइयों का प्रतिनिधित्व करता है। ऊपरी सदन राज्यों के हितों की रक्षा करने के लिए गठित किया जाता है। इसमें संघ की इकाइयों को बराबर सीट देने की व्यवस्था की जाती है, जिससे उनकी संवैधानिक समानता स्थापित हो सके।

5. दोहरा शासन-संघात्मक सरकार में दोहरा शासन-प्रबन्ध होता है। एक केंद्रीय शासन तथा दूसरा स्थानीय अथवा प्रान्तीय शासन। संघ तथा प्रान्तों के अधिकार संविधान द्वारा निश्चित होते हैं। दोनों सरकारें अपने-अपने क्षेत्र में स्वतन्त्र होती हैं।

6. दोहरी नागरिकता-संघ सरकार में नागरिकों को दोहरी नागरिकता प्राप्त होती है। एक उस राज्य की नागरिकता जहाँ वह निवास करता है, तथा दूसरी संघ की नागरिकता। ऊपर वर्णित तत्त्व संघीय सरकार के निर्माण में आवश्यक हैं। इन तत्त्वों के आधार पर ही संघात्मक सरकार की स्थापना होती है।

HBSE 11th Class Political Science Important Questions Chapter 7 संघवाद

प्रश्न 2.
“भारतीय संविधान का स्वरूप या ढाँचा संघात्मक है, लेकिन उसकी आत्मा एकात्मक है।” इस कथन की व्याख्या कीजिए।
अथवा
भारतीय संविधान की संघीय विशेषताएँ बताएँ। भारतीय संविधान में दिए गए एकात्मक तत्त्वों का विवरण दो।
उत्तर:
26 जनवरी, 1950 को भारतीय संविधान के लागू होने पर अधिकांश विद्वानों तथा राजनेताओं ने इसे एक आवाज में संघात्मक संविधान माना। फिर भी संविधान का संघात्मक ढाँचा विद्वानों में हमेशा विवाद का विषय रहा है। एक ओर प्रो० अलैक्जेण्ड्रोविक्स, के० संथानम, मोरिस जोन्स, एम०वी० पायली, डॉ० अम्बेडकर, पाल एपेल्बी आदि विद्वान् और राजनेता .. भारतीय संविधान को संघात्मक मानते थे तो दूसरी ओर कुछ विचारक इस बात से सहमत नहीं थे।

प्रो० अलेक्जेण्ड्रोविक्स के शब्दों में, “भारत निस्सन्देह एक संघ है जिसमें प्रभुसत्ता के तत्त्वों को केंद्र और राज्यों के बीच विभाजित किया गया है।” डॉ० के०सी० व्हीयर, जेनिंग्ज, डी०डी० बसु, डी०एन० बैनर्जी, के०पी० मुखर्जी, के०वी० राव आदि भारत के संविधान को संघात्मक मानने के लिए तैयार नहीं हैं। के०पी० मुखर्जी के अनुसार,

“भारतीय संविधान निश्चय ही गैर-संघात्मक या एकात्मक संविधान है।” इसी तरह के०सी० व्हीयर का कहना है, “भारतीय संविधान अर्द्ध-संघात्मक है। वह नाममात्र की एकात्मक विशेषताओं के साथ संघात्मक राज्य होने के बजाय गौण संघीय विशेषताओं के साथ एकात्मक राज्य है।” कहने का तात्पर्य यह है कि भारतीय संविधान में संघात्मक एवं एकात्मक दोनों तरह के तत्त्व पाए जाते हैं।

इस तरह भारतीय संविधान संघात्मक होते हुए भी उसका झुकाव एकात्मकता की ओर है। भारतीय संविधान के संघात्मक तत्त्व (Federal Elements of Indian Constitution)-भारतीय संविधान में निम्नलिखित संघात्मक तत्त्व मौजूद हैं

1. लिखित संविधान भारत का संविधान एक लिखित संविधान है। 9 दिसम्बर, 1946 में संविधान का कार्य आरम्भ हुआ तथा 26 नवम्बर, 1949 में संविधान पूरा हुआ। 26 जनवरी, 1950 को संविधान लागू हुआ। वर्तमान में संविधान में 395 अनुच्छेद हैं जिन्हें 12 अनुसूचियों और 22 अध्यायों में बाँटा गया है। भारतीय संविधान में अब तक 104 संशोधन हो चुके हैं।

2. संविधान की कठोरता भारत का संविधान एक कठोर संविधान है। यद्यपि भारत का संविधान अमेरिका के संविधान की तरह कठोर तो नहीं है, परन्तु फिर भी इसमें संशोधन करने की प्रक्रिया इतनी सरल नहीं रखी गई है। भारतीय संविधान के अनुच्छेद 368 ने भी इसीलिए संविधान में संशोधन करने के लिए संसद का 2/3 बहुमत तथा केंद्र-राज्यों के सम्बन्धों को प्रत्यक्ष या अप्रत्यक्ष रूप से प्रभावित करने वाली व्यवस्थाओं पर आधे राज्यों के अनुमोदन के साथ संसद के दो-तिहाई बहुमत का जटिल तरीका अपनाया गया है।

3. संविधान की सर्वोच्चता भारत में संविधान को सर्वोच्च बनाया गया है। यदि किसी समय केंद्र तथा राज्यों के बीच अधिकार क्षेत्र के किसी मामले पर विवाद हो तो उसका हल संविधान में दी गई व्यवस्थाओं के अन्तर्गत ही निकाला जाएगा। अमेरिका आदि संघीय संविधानों की तरह भारत में भी यही तरीका अपनाया गया है।

4. शक्तियों का विभाजन-भारत में संघीय शासन के अन्तर्गत शक्तियों को केंद्र तथा राज्यों में बाँटा गया है। इस उद्देश्य के लिए तीन सूचियाँ (संघ सूची, राज्य सूची, समवर्ती सूची) बनाई गई हैं। संघ सूची में 97 विषय (वर्तमान में 100 विषय), राज्य सूची में 66 विषय (वर्तमान में 61 विषय) तथा समवर्ती सूची में 47 विषय (वर्तमान में 52 विषय) रखे गए हैं। अवशिष्ट शक्तियाँ केंद्र सरकार को दी गई हैं।

राष्ट्रीय महत्त्व के विषय, यथा-देश की सुरक्षा, संचार साधन, विदेश-नीति, मुद्रा, बैंकिंग आदि महत्त्वपूर्ण विषय संघ सूची में रखे गए हैं। पुलिस, जेल, स्वास्थ्य, स्थानीय प्रशासन आदि विषय राज्य सूची में रखे गए हैं। दोनों के लिए महत्त्वपूर्ण समझे जाने वाले विषय समवर्ती सूची में रखे गए हैं, परन्तु इस सूची पर सर्वोच्चता केंद्र सरकार को दी गई है।

5. दोहरी शासन-प्रणाली-भारत में दोहरी शासन-प्रणाली अपनाई गई है। केंद्र तथा राज्यों की अलग-अलग सरकारें हैं। दोनों को शासन की शक्तियाँ संविधान ने दी हैं। यद्यपि सहकारी संघवाद के कारण दोनों सरकारें आपस में सहयोग करके अपने-अपने दायरे में शासन चलाती हैं, फिर भी दोनों में से कोई किसी के अधीन नहीं हैं।

6. न्यायपालिका की विशेष स्थिति-संघ-शासन में केंद्र तथा राज्यों के बीच अधिकार क्षेत्र के विवाद को संविधान में की गई. व्यवस्थाओं के आधार पर निपटाने की शक्ति न्यायपालिका को दी जाती है। इस आधार पर भारत में भी न्यायपालिका को सर्वोच्च शक्ति प्रदान की गई है। उच्चतम न्यायालय केंद्र तथा राज्यों के आपसी विवादों का निपटारा करता है। इसके निर्णय अन्तिम होते हैं।

7. दो-सदनीय विधानमण्डल-संघ शासन में दो-सदनीय विधानमण्डल होता है। एक सदन सारे देश का तथा दूसरा सदन उन इकाइयों का प्रतिनिधित्व करता है जो मिलकर संघीय सरकार का निर्माण करती हैं। भारत में लोकसभा सारे देश का तथा राज्यसभा राज्यों का प्रतिनिधित्व करती है। यह व्यवस्था इसलिए की जाती है, जिससे केंद्र सरकार राज्यों के हितों को नुकसान न पहुँचा सके।

संविधान में एकात्मक तत्त्व (Unitary Elements of Indian Constitution)-भारतीय संविधान में संघ शासन की इन आधारभूत विशेषताओं के होते हुए भी एकात्मक तत्त्वों की कमी नहीं है। भारत के संविधान में निम्नति

1. इकहरी नागरिकता भारत में अमेरिका के विरुद्ध इकहरी नागरिकता प्रदान की गई है। कोई भी व्यक्ति किसी भी राज्य में रहे (जम्मू-कश्मीर राज्य को छोड़कर) वह राज्य का नहीं, भारत का नागरिक है। इकहरी नागरिकता की यह व्यवस्था संघ शासन के विरुद्ध है।

2. केंद्र तथा राज्यों के लिए एक संविधान-संघ शासन वाले राज्यों में संघ की इकाइयों को अलग-अलग संविधान बनाने का अधिकार होता है। अमेरिका, स्विट्जरलैण्ड तथा अन्य संघीय देशों में यही तरीका अपनाया गया है। लेकिन भारत में जम्मू-कश्मीर को छोड़कर अन्य राज्यों का अपना अलग संविधान नहीं है। पूरे देश पर एक ही संविधान लागू होता है।

3. शक्तियों का विभाजन केंद्र के पक्ष में शक्तियों का विभाजन भी केंद्र के पक्ष में है। संघ सूची में 97 विषय हैं जिन पर कानून बनाने का अधिकार संघ सरकार को है। राज्य सूची में दिए गए विषयों पर भी केंद्र कानून बना सकता है, यदि राज्यसभा ऐसा प्रस्ताव पास कर दे, राज्य स्वयं कानून बनाने की प्रार्थना करे अथवा राज्य में राष्ट्रपति शासन लागू हो। समवर्ती सूची पर केंद्र तथा राज्यों को कानून बनाने का अधिकार है, परन्तु विवाद की दशा में केंद्र का कानून लागू होगा। ऐसी दशा में केंद्र सरकार राज्यों की तुलना में अधिक शक्तिशाली है।

4. अवशिष्ट शक्तियाँ केंद्र के पास आमतौर पर संघ शासन में अवशिष्ट शक्तियाँ राज्यों के पास होती हैं, लेकिन भारतीय संविधान में साफ तौर पर यह व्यवस्था की गई है कि जो विषय तीनों सूचियों में नहीं हैं ऐसे सभी अवशिष्ट विषयों पर संसद कानून बनाएगी, राज्यों के विधानमण्डल नहीं।

5. राज्यपालों की नियुक्ति-भारतीय संघ के राज्यों के राज्यपालों की नियुक्ति राष्ट्रपति करता है। राष्ट्रपति ही राज्यपाल को एक राज्य से दूसरे राज्य में भेजता है तथा किसी भी राज्यपाल का कार्यकाल बढ़ाता है अथवा पाँच वर्ष की अवधि से पहले हटाता भी है। राज्यपाल की नियुक्ति करते समय उस राज्य के मुख्यमन्त्री से परामर्श लिया जाता है, परन्तु उस परामर्श को मानना आवश्यक नहीं है। राज्यपाल अपने कार्यों के लिए राज्य के प्रति नहीं, वरन् राष्ट्रपति के प्रति उत्तरदायी होता है, यह संघात्मक गुणों के विपरीत है।

6. राज्यों में राष्ट्रपति शासन-राष्ट्रपति राज्यपाल के सुझाव अथवा स्वयं निर्णय करके किसी भी राज्य में वहाँ की विधिवत निर्वाचित सरकार को भंग करके राष्ट्रपति शासन लाग कर सकता है। ऐसी दशा में शासन की सारी शक्तियाँ केंद्र के पास आ जाती हैं।

7. राज्यसभा में असमान प्रतिनिधित्व-अमेरिकी सीनेट की तरह संघ शासन वाले देशों में केंद्रीय विधानमण्डल के ऊपरी सदन में सभी राज्यों के बराबर संख्या में प्रतिनिधि होते हैं, परन्तु भारत में इस सिद्धान्त को नहीं अपनाया गया है। राज्यसभा में राज्यों को उनकी आबादी के अनुसार स्थान दिए गए हैं। जिस कारण राज्यसभा अधिक प्रभावशाली बनकर राज्यों के हितों की पूरी तरह रक्षा नहीं कर सकती।

8. संसद को राज्यों के पुनर्गठन का अधिकार संघ शासन में केंद्रीय विधानमण्डल सम्बन्धित राज्य की इच्छा के बिना उसकी सीमाओं तथा नामों आदि में परिवर्तन नहीं कर सकता। लेकिन भारत में संसद को यह अधिकार दिया गया है।

सन् 1956 में केंद्र ने राज्य पुनर्गठन अधिनियम पास किया, जिसके अधीन कितने ही नए राज्य बनाए गए हैं, उनकी सीमाओं में परिवर्तन किए गए हैं तथा उनके नामों में भी परिवर्तन किए गए हैं। इसी आधार पर के०पी० मुखर्जी ने लिखा है, “अगर एकात्मक सरकार की परिभाषा यह नहीं है तो मैं नहीं जानता कि वह क्या है।”

9. इकहरी न्याय पद्धति संघ शासन में न्याय व्यवस्था दोहरी होती हैं, परन्तु भारत में नीचे से ऊपर तक न्यायपालिका का ढाँचा एकीकृत है। उच्चतम न्यायालय न्यायपालिका के शिखर पर है। इसके साथ ही पूरे देश के लिए एक जैसी दीवानी तथा फौजदारी व्यवस्था है।

में राज्य केंद्र पर आश्रित केंद्र की अपेक्षा राज्य सरकारों की आय के साधन बहुत कम हैं। इसलिए राज्यों को अपनी वित्तीय आवश्यकताओं को पूरा करने के लिए केंद्र सरकार पर निर्भर रहना पड़ता है। अनेक केंद्रीय करों से प्राप्त राजस्व का राज्यों में वितरण किया जाता है। इन सभी व्यवस्थाओं और नीति आयोग के निर्देशन में पूरे राष्ट्र के सुनियोजित विकास की व्यवस्था ने राज्यों को केंद्र पर निर्भर बना दिया है।

11. नीति आयोग पर केंद्र का प्रभुत्व (Dominance of Centre on NITI Commission)-1 जनवरी, 2015 को 65 वर्ष पुराने योजना आयोग के स्थान पर अस्तित्व में आए नीति आयोग पर भी केंद्र का ही प्रभुत्व है। नीति आयोग का अध्यक्ष भी योजना आयोग की तरह प्रधानमन्त्री होगा तथा उपाध्यक्ष की नियुक्ति भी प्रधानमन्त्री करता है। इसके अतिरिक्त दो पूर्वकालिक सदस्यों की नियुक्ति भी प्रधानमन्त्री द्वारा की जाती है। इसके साथ-साथ समय-समय पर विशिष्ट सदस्यों को भी प्रधानमन्त्री द्वारा ही आमन्त्रित किया जाता है।

पदेन सदस्यों में भी केंद्रीय मन्त्री ही सम्मिलित होते हैं। नीति आयोग में उल्लेखित प्रशासनिक परिषद् में यद्यपि राज्यों के मुख्यमन्त्री सदस्य होंगे परन्तु इसकी अध्यक्षता प्रधानमन्त्री द्वारा की जाएगी। इसके अतिरिक्त विशिष्ट क्षेत्रीय परिषदों का गठन भी प्रधानमन्त्री द्वारा ही किया जाएगा। इस प्रकार स्पष्ट है कि नवगठित नीति आयोग में केंद्र का ही प्रभुत्व बना हुआ है जो भारतीय शासन को एकात्मकता की ओर झुकाता है।

निष्कर्ष-उपरोक्त संघात्मक एवं एकात्मक तत्त्वों का अध्ययन करने के पश्चात् इस निष्कर्ष पर पहुँचा जा सकता है कि भारतीय संविधान न तो पूर्णतः एकात्मक है और न ही संघात्मक। डॉ० जैनिंग्ज के अनुसार, यह कहना उचित होगा कि, “भारत सशक्त केंद्रीयकरण वाली विशेषताओं से युक्त संघ है।”

प्रश्न 3.
केंद्र तथा राज्यों के बीच विधायी सम्बन्धों का वर्णन करें। अथवा केंद्र राज्यों की वित्तीय जरूरतों को कैसे पूरा करता है? स्पष्ट करें। अथवा केंद्र तथा राज्यों के प्रशासनिक सम्बन्धों का विवरण दो।
अथवा
भारत में संघ तथा राज्यों के आपसी सम्बन्धों का संक्षेप में वर्णन करें।
उत्तर:
भारत में संघात्मक सरकार की स्थापना की गई है। इस व्यवस्था के अन्तर्गत शासन की शक्तियों को तीन सूचियों में बाँटा गया है और अवशेष शक्तियाँ केंद्र सरकार को सौंपी गई हैं। भारत में पूर्णतः संघात्मक सरकार नहीं है और जिन देशों में पूर्ण संघात्मक सरकार की स्थापना की गई है, उन देशों में भी केंद्र का राज्यों पर प्रभुत्व बढ़ता ही जा रहा है तथा राज्यों की भी केंद्र पर निर्भरता बढ़ती जा रही है। भारत में केंद्र अथवा संघ सरकार और विभिन्न राज्य सरकारों के आपसी सम्बन्धों को तीन भागों में बाँटा जा सकता है

  • विधायी सम्बन्ध,
  • प्रशासनिक सम्बन्ध,
  • वित्तीय सम्बन्ध ।

1. केंद्र तथा राज्यों के विधायी सम्बन्ध (Legislative Relation between Centre and States) भारतीय संविधान ने केंद्र तथा राज्य सरकारों के बीच विधायी सम्बन्धों का विवरण संविधान के ग्यारहवें भाग के अध्याय एक में अनुच्छेद .245 से 254 तक में दिया है।

(1) संघ सूची इस सूची में संविधान लागू होते समय मूलतः 97 विषय (वर्तमान में 100 विषय) रखे गए है। इनमें प्रतिरक्षा, अणुशक्ति, विदेशी मामले, युद्ध और सन्धि, रेलवे, मुद्रा, बैंकिंग, डाक- तार आदि विषय शामिल हैं। संघ सूची में इन सभी मामलों पर कानून बनाने की शक्ति संसद को प्राप्त है और उन पर केंद्र सरकार का ही प्रशासनिक नियन्त्रण कायम है।

(2) राज्य सूची इस सूची में मूल रूप से 66 विषय (वर्तमान में 61 विषय) रखे गए हैं। राज्य सूची में पुलिस, जेल, न्याय . प्रबन्ध, शिक्षा, स्वास्थ्य तथा स्थानीय स्वशासन आदि शामिल हैं। समय-समय पर संविधान संशोधनों द्वारा इस सूची में विषयों को निकाला तथा जोड़ा जाता रहा है, परन्तु मूल रूप में इनकी संख्या 66 ही रही है। इस सूची में शामिल विषयों पर राज्य सरकार कानून बनाती है तथा उन पर उनका ही प्रशासनिक नियन्त्रण कायम रहता है।

(3) समवर्ती सूची-इस सूची में मूल 47 विषय (वर्तमान में 52 विषय) रखे गए हैं। इनमें फौजदारी कानून, निवारक नजरबंदी कानून, विवाह, तलाक, ट्रेड यूनियन, श्रम, कल्याण, खाद्य पदार्थों में मिलावट आदि विषय शामिल हैं। इन विषयों पर कानून बनाने का अधिकार केंद्र तथा राज्यों को प्रदान किया गया है। इसके अतिरिक्त जिन विषयों का उल्लेख तीनों सूचियों में से किसी में भी नहीं है, ऐसे सभी मामलों पर अवशिष्ट अधिकार के रूप में कानून बनाने की शक्ति संसद को दी गई है।

संसद द्वारा राज्य-सूची पर कानून निर्माण-यद्यपि राज्य सूची में दिए गए विषयों पर कानून बनाने का अधिकार राज्यों का – है, परन्तु निम्नलिखित परिस्थितियों में राज्य सूची के विषयों पर भी संसद कानून बना सकती है-

(1) यदि राज्यसभा अपने उपस्थित स्यों के बहमत और कल सदस्य संख्या के 2/3 बहमत से प्रस्ताव पास करके राज्य सूची के किसी विषय को राष्ट्रीय महत्त्व का घोषित कर दे,

(2) राष्ट्रपति देश में संकटकाल की घोषणा कर दे,

(3) अगर दो या दो से अधिक राज्यों के विधानमण्डल एक प्रस्ताव पारित करके राज्य सूची के किसी विषय पर संसद को कानून बनाने की अपील करें,

(4) अन्तर्राष्ट्रीय सन्धियों तथा समझौतों को लागू कराने के लिए संसद राज्य सूची के किसी विषय पर कानून बना सकती है,

(5) राज्य में संवैधानिक व्यवस्था के विफल हो जाने पर राष्ट्रपति संकटकाल की घोषणा कर सकता है। तब उस राज्य के लिए राज्य सूची में दिए गए सभी विषयों पर कानून बनाने का अधिकार संसद को मिल जाता है,

(6) राज्यपाल राज्य विधानमण्डल द्वारा पारित किसी भी विधेयक को राष्ट्रपति की स्वीकृति के लिए उसके पास भेज सकता है। राष्ट्रपति ऐसे विधेयकों पर अपनी स्वीकृति दे सकता है अथवा उसे रोक सकता है,

(7) राज्य सरकारों को एक राज्य से दूसरे राज्यों अथवा संघ प्रशासित क्षेत्रों से आने वाली वस्तुओं या उनके व्यापार और वाणिज्य से सम्बन्धित मामलों पर युक्तिसंगत प्रतिबन्ध लगाने का अधिकार दिया गया है। लेकिन ऐसा विधेयक विधानमण्डल में पेश करने से पहले राष्ट्रपति की स्वीकृति अनिवार्य है।

इस विवरण से स्पष्ट है कि विधायी क्षेत्र में राज्य सूची में दिए गए मामलों पर भी केंद्र सरकार को बहुत अधिक महत्त्वपूर्ण शक्तियाँ प्रदान की गई हैं। इसके अतिरिक्त समवर्ती सूची पर केंद्र तथा राज्य दोनों को ही कानून बनाने का अधिकार है, परन्तु विवाद की दशा में राज्य विधानमण्डल द्वारा बनाया गया कानून लागू नहीं होगा, वरन् केंद्रीय संसद द्वारा बनाया गया कानून लागू होगा।

2. केंद्र तथा राज्यों के प्रशासनिक सम्बन्ध (Administrative Relation of Centre and State) भारतीय संविधान ने केंद्र तथा राज्य सरकारों को जिन मामलों पर कानून बनाने का अधिकार दिया है, उन्हीं मामलों पर उसने उन्हें अपना प्रशासनिक नियन्त्रण कायम करने की शक्ति भी दी है। संविधान में केंद्र और राज्यों के प्रशासनिक सम्बन्धों का विवरण दिया गया है, जो इस प्रकार है

(1) संविधान के अनुच्छेद 256 में यह व्यवस्था की गई है कि राज्य सरकारें अपनी प्रशासनिक शक्ति का प्रयोग इस तरह करेंगी जिससे संसद के कानूनों का पालन होता रहे,

(2) केंद्र सरकार राज्य सरकारों को अपनी प्रशासनिक शक्ति का प्रयोग करने के बारे में जरूरी निर्देश दे सकती है। केंद्र सरकार नदियों, जलाशयों और महत्त्वपूर्ण सार्वजनिक मार्गों को राष्ट्रीय महत्त्व का घोषित कर सकती है तथा रेलवे सम्पत्ति के संरक्षण के बारे में निर्देश दे सकती है,

(3) राष्ट्रपति ऐसे किसी भी कार्य को केंद्र सरकार की ओर से राज्य सरकार या उसके किसी अधिकारी को सौंप सकता है जो केंद्र सरकार के अधिकार क्षेत्र में आता है,

(4) केंद्र तथा राज्य सरकारें न्यायालयों द्वारा दिए गए निर्णयों को लागू करेगी और इस बारे में आवश्यक कानून संसद द्वारा बनाए जाएँगे,

(5) नदियों के पानी के प्रयोग, वितरण और नियन्त्रण के मामलों पर संसद कानून बना सकती है तथा उस बारे में राज्यों के बीच विवाद होने पर मामले को मध्यस्थ या पंच के द्वारा हल करने के निर्देश केंद्र सरकार सम्बन्धित राज्य सरकारों को दे सकती है,

(6) भारतीय संघ के राज्यों के बीच होने वाले आपसी विवादों का निपटारा करने के लिए संविधान ने केंद्र सरकार की ओर से राष्ट्रपति को अन्तराज्यीय-परिषद् (Inter-state Council) कायम करने की शक्ति दे रखी है,

(7) राज्यों के राज्यपालों की नियुक्ति राष्ट्रपति द्वारा की जाती है। राज्यपाल राज्य सरकार का संवैधानिक मुखिया होने के साथ सम्बन्धित राज्य में राष्ट्रपति के प्रतिनिधि के रूप में भी कार्य करता है। अतः उसके द्वारा केंद्र सरकार राज्य के प्रशासन को प्रभावित करती है,

(8) केंद्र सरकार को अनुसूचित जातियों, अनुसूचित जनजातियों और सामाजिक तथा शैक्षणिक दृष्टि से पिछड़े वर्गों के कल्याण के लिए आयोग नियुक्त करने की शक्ति दी गई है। ऐसे आयोगों से प्राप्त रिपोर्टों के आधार पर या अपनी ओर से भी इन वर्गों के कल्याण के लिए निर्देश देने का अधिकार राष्ट्रपति के माध्यम से केंद्र सरकार को प्राप्त है। ऊपर वर्णित विवरण से स्पष्ट है कि प्रशासनिक सम्बन्धों के मामलों में राज्यों की तुलना में केंद्र सरकार की स्थिति बहुत अधिक प्रभावशाली है।

3. केंद्र तथा राज्यों के वित्तीय सम्बन्ध (Financial Relations between Centre and State Relation)-संविधान में केंद्र तथा राज्यों के मध्य वित्तीय सम्बन्धों की भी व्यवस्था की गई है जिसके अन्तर्गत वित्तीय मामलों में केंद्रीय सरकार राज्य सरकारों की तुलना में अधिक शक्तिशाली है। केंद्र तथा राज्यों के वित्तीय सम्बन्धों को निम्नलिखित रूप में समझा जा सकता है

(1) संघ तथा राज्यों की आय के साधन-संविधान के अनुसार संघ तथा केंद्र सरकार की आय के मुख्य साधनों में आय-कर, आयात व निर्यात-कर, सीमा शुल्क, समाचार-पत्रों की बिक्री तथा विज्ञापन पर कर, शराब तथा अन्य वस्तुओं पर उत्पादन कर, सम्पत्ति शुल्क, डाक, टेलीफोन से प्राप्त आय आदि शामिल हैं। दूसरी ओर राज्य सरकार की आय के प्रमुख साधनों में कृषि से होने वाली आय पर कर, शराब तथा ऐसी अन्य वस्तुओं पर उत्पादन कर, कृषि पर सम्पदा शुल्क, बिक्री-कर, बिजली के उपभोग पर कर, आमोद-प्रमोद के साधनों पर कर, वाहनों पर कर आदि शामिल किए गए हैं।

(2) करों के वितरण की व्यवस्था केंद्र की तुलना में राज्यों की आय के स्रोत बहुत कम हैं। राज्यों को शिक्षा, स्वास्थ्य और चिकित्सा आदि सुविधाओं के लिए धन की बहुत जरूरत पड़ती है। इसीलिए संविधान में व्यवस्था की गई है कि कुछ कर ऐसे होंगे जिन्हें लगाने का अधिकार तो केंद्र सरकार को होगा, लेकिन इन करों से होने वाली आय को केंद्र प्रशासित क्षेत्रों को छोड़कर राज्य : सरकारें अपने-अपने राज्यों में इकट्ठा करेंगी और वे ही उसे खर्च करेंगी।

इन करों में स्टाम्प शुल्क, दवाइयों और शृंगार की वस्तुओं पर उत्पादन शुल्क आदि शामिल हैं। संविधान में यह व्यवस्था की गई है कि कुछ वस्तुओं पर केंद्र सरकार कर लगाएगी और वसूली करेगी, परन्तु आय का कुछ भाग राज्यों को दिया जाएगा। उदाहरणस्वरूप, कृषि-भूमि को छोड़कर अन्य सम्पत्तियों पर उत्तराधिकार कर, रेल-भाड़े और माल-भाड़े पर लगने वाला कर, समाचार-पत्रों पर बिक्री तथा विज्ञापन कर, आय-कर, पटसन तथा जूट के निर्यात पर कर आदि शामिल हैं।

(3) राज्यों को अनुदान-ऊपर वर्णित वित्तीय व्यवस्था के अतिरिक्त संविधान में व्यवस्था की गई है कि केंद्र सरकार राज्यों को वित्तीय अनुदान या आर्थिक सहायता देगी। इनके अन्तर्गत राज्य सरकारें केंद्र से विकास की नई परियोजनाओं और दूसरे विकास कार्यों के लिए अनुदान प्राप्त कर सकती हैं। इसी तरह बाढ़, भूकम्प, अकाल तथा अन्य प्राकृतिक विपदाओं की हालत में पीड़ित जनता की सहायता के लिए केंद्र सरकार राज्यों को वित्तीय सहायता देती है। आदिम जातियों और कबीलों की उन्नति के लिए भी केंद्र सरकार राज्यों को अनुदान देती है। पूर्वोत्तर भारत में अनुसूचित क्षेत्रों की उन्नति तथा विकास के लिए केंद्र सरकार उन्हें वित्तीय सहायता देती है।

(4) ऋण-संविधान ने केंद्र तथा राज्य सरकारों को ऋण लेने का अधिकार भी दे रखा है। केंद्र सरकार अपनी संचित निधि की जमानत पर संसद की अनुमति से देशवासियों तथा विदेशी सरकारों अथवा अन्तर्राष्ट्रीय वित्तीय इकाइयों से ऋण ले सकती है। राज्य सरकारें देशवासियों तथा केंद्र सरकार से ही ऋण प्राप्त कर सकती हैं।

(5) वित्तीय संकटकाल में केंद्र तथा राज्यों के सम्बन्ध-संविधान के अनुच्छेद 360 के अन्तर्गत राष्ट्र की वित्तीय साख अथवा वित्तीय स्थिरता को खतरा पैदा हो जाने की दशा में राष्ट्रपति आर्थिक संकट की घोषणा कर सकता है। इस घोषणा के परिणामस्वरूप राज्यों के आर्थिक मामलों में केंद्र सरकार को निर्देश देने का अधिकार मिल जाता है। राज्यों के विधानमण्डलों द्वारा पास किए गए धन सम्बन्धी विधेयकों और प्रस्तावों पर राष्ट्रपति की स्वीकृति लेनी जरूरी हो जाती है।

(6) वित्त आयोग-संविधान के अनुसार संविधान के लागू होने के दो साल के अन्दर राष्ट्रपति एक वित्त आयोग (Finance Commission) गठित करेगा और फिर हर पाँच वर्ष बाद एक नया वित्त आयोग कायम किया जाएगा। आयोग में अध्यक्ष समेत पाँच सदस्य होंगे। यह आयोग केंद्र तथा राज्यों के बीच राजस्व के बंटवारे के बारे में निर्णय करके अपनी सिफारिशें राष्ट्रपति को पेश करता है।

(7) भारत का नियन्त्रक तथा महालेखा परीक्षक-संविधान ने पूरे देश के लिए एक नियन्त्रक तथा महालेखा परीक्षक नियुक्त करने की व्यवस्था की है। उसकी नियुक्ति राष्ट्रपति करता है। यह महालेखा परीक्षक केंद्र तथा राज्य सरकारों के लेखा-जोखा की निष्पक्ष तरीके से जाँच करके अपनी रिपोर्ट राष्ट्रपति को देता है, जिसे राष्ट्रपति संसद में प्रति वर्ष पेश करता है।

(8) योजना आयोग-योजना आयोग का उल्लेख संविधान में नहीं किया गया है, फिर भी संसद के 15 मार्च, 1950 को पास किए गए एक कानून के अनुसार उसे कायम किया गया था। देश के सम्पूर्ण सुनियोजित विकास की जिम्मेदारी योजना आयोग की थी और वही उस सम्बन्ध में योजना का प्रारूप तैयार करता था। इस आयोग की अध्यक्षता प्रधानमन्त्री करता है। वर्तमान में योजना आयोग की जगह नीति आयोग का गठन किया गया है।

HBSE 11th Class Political Science Important Questions Chapter 7 संघवाद

प्रश्न 4.
संघ (केंद्र) तथा राज्यों के बीच सम्बन्धों की आलोचनात्मक व्याख्या करें।
उत्तर:
संविधान द्वारा केंद्र तथा राज्यों के बीच शक्तियों का जो विभाजन किया गया है, वह संघ के पक्ष में है अर्थात् संघीय सरकार को राज्य सरकारों के मुकाबले में बहुत ही अधिक शक्तिशाली बनाया गया है। ऐसा होने पर भी सन् 1967 तक केंद्र तथा राज्यों के सम्बन्ध अच्छे रहे और उनमें किसी प्रकार का संघर्ष अथवा तनाव (Tension) उत्पन्न नहीं हुआ।

इसका मुख्य कारण था केंद्र तथा राज्यों में एक ही राजनीतिक दल अर्थात् काँग्रेस दल की सरकारों का होना। सन् 1967 के चुनावों के पश्चात् केंद्र में तो पुनः काँग्रेस दल की सरकार की स्थापना हुई, परन्तु कई राज्यों-बंगाल, पंजाब, तमिलनाडु आदि में गैर-काँग्रेसी सरकारों की स्थापना हुई। इन सरकारों ने राज्यों को और अधिक शक्तियाँ देने की माँग की, जिससे केंद्र तथा राज्यों में संघर्ष आरम्भ हो गया। संघ तथा राज्यों के बीच संघर्ष अथवा तनाव के मुख्य कारण निम्नलिखित रहे हैं और आज भी हैं

1. वित्तीय समस्या केंद्र तथा राज्यों के बीच तनाव का एक मुख्य कारण वित्त रहा है। राज्य सरकारों को सदा यह शिकायत रहती है कि उन्हें जन-कल्याण के अनेक कार्य करने पड़ते हैं और उनकी आय के साधन बहुत कम हैं, इस कारण से उन्हें केंद्र द्वारा दी जाने वाली वित्तीय सहायता पर निर्भर रहना पड़ता है। केंद्र राज्यों को सहायता देते समय राज्यों में विरोधी दलों की सरकारों के साथ पक्षपात करता है। अतः अधिकतर राज्यों की यह माँग रहती है कि उनकी आय के साधनों में वृद्धि की जाए, ताकि वित्त के मामले में उनकी केंद्र पर निर्भरता कुछ कम हो।

2. राज्यपाल की भूमिका केंद्र तथा राज्यों के बीच तनाव का एक बहुत बड़ा कारण यह है कि राज्यपाल की नियुक्ति राष्ट्रपति के द्वारा की जाती है और वह अपने कार्यों के लिए राष्ट्रपति के प्रति उत्तरदायी होता है। राष्ट्रपति राज्यपाल को पद से हटाने की भी शक्ति रखता है, राज्यपाल राष्ट्रपति के एजेन्ट के रूप में कार्य करता है। पिछले कुछ समय से राज्यपाल की भूमिका के बारे में देश में काफी विवाद रहा है। विशेष रूप से ऐसे राज्यों में जहाँ विरोधी दलों की सरकारें बनी हैं। इस विवाद का एक मुख्य प्रश्न यह रहा है कि राज्यपाल को किन परिस्थितियों में राष्ट्रपति शासन लागू करने की सिफारिश करनी चाहिए।

3. नौकरशाही की भूमिका भारत में राज्यों के उच्च अधिकारी अखिल भारतीय प्रशासनिक सेवाओं (I.A.S.) के सदस्यों में से नियुक्त किए जाते हैं, जिन पर राज्य सरकारों का पूरा नियन्त्रण नहीं होता। उनकी नियुक्ति केंद्रीय सरकार द्वारा की जाती है। इसलिए राज्य सरकारों की यह माँग रही है कि उन सभी कर्मचारियों पर, जो राज्य में काम कर रहे हैं, राज्य सरकार का ही नियन्त्रण होना चाहिए। परन्तु अभी स्थिति पहले जैसी ही बनी हुई है।

4. कानून तथा व्यवस्था की समस्याएँ-संविधान के अनुसार कानून तथा व्यवस्था को बनाए रखना राज्य सरकार का दायित्व है, परन्तु केंद्रीय सरकार किसी भी राज्य में शान्ति तथा व्यवस्था को बनाए रखने के लिए केंद्रीय सुरक्षा पुलिस बल भेज सकती है। राज्य सरकारों का यह विचार रहा है कि केंद्र द्वारा राज्यों में सशस्त्र सेनाएँ तभी भेजनी चाहिएँ जब राज्य द्वारा ऐसी माँग की जाए, परन्तु व्यवहार में केंद्र ने कई बार अपनी इस शक्ति का प्रयोग राज्य की इच्छा के विरुद्ध किया है।

5. संकटकालीन प्रावधान-संविधान के अनुच्छेद 356 के अनुसार केंद्रीय सरकार राज्य की विधायी एवं प्रशासनिक सत्ता अपने हाथों में ले सकता है। इस प्रावधान का प्रयोग राज्य में “सवैधानिक तन्त्र की विफलता” (Failure of Constitutional Machinery in the State) की स्थिति में होता है। यह कदम राज्य के राज्यपाल की रिपोर्ट के आधार पर सर्वप्रथम सन् 1959 में केरल में ई०एम०एस० नम्बूदरिपाद की सरकार को निलम्बित करके राष्ट्रपति शासन लागू किया गया था।

उसके बाद लगभग सभी राज्यों को किसी-न-किसी कारण से राष्ट्रपति शासन के अन्तर्गत रहना पड़ा है। जिन राज्यों में विरोधी दल की सरकार को बर्खास्त किया जाता है, उनके द्वारा केंद्र पर सदा ही पक्षपात का आरोप लगाया जाता है। इसी प्रकार सन् 1989 में कर्नाटक में जनता दल की सरकार को हटाकर राष्ट्रपति शासन का लागू किया जाना इस बात का स्पष्ट उदाहरण है।

सन् 1997 में उत्तर प्रदेश में राज्यपाल रोमेश भण्डारी द्वारा भारतीय जनता पार्टी के नेतृत्व वाली सरकार को बर्खास्त करना भी एक ऐसा ही उदाहरण है। इस कारण से राज्य सरकारों ने प्रायः यह कहा है कि संवैधानिक तन्त्र की विफलता के प्रावधान की व्याख्या केंद्र ने समय-समय पर भिन्न-भिन्न प्रकार से की है।

6. दलीय कारण-दलीय भावना के कारण भी केंद्र राज्यों पर तथा राज्य केंद्र पर दोषारोपण करते रहते हैं, विशेष रूप से उस समय जब केंद्र में एक दल की सरकार हो तथा राज्य में किसी दूसरे दल की।

7. केंद्रीय कानूनों का कार्यान्वन-संविधान के अनुच्छेद 256 तथा 257 केंद्र को यह अधिकार देते हैं कि वह राज्य सरकारों को संसद के कानून के अनुसार कार्य करने तथा राज्य के कार्यकारी अधिकारों के प्रयोग के बारे में आदेश दे। अनुसूचित जातियों के कल्याण के बारे में भी उन्हें आदेश दिए जा सकते हैं। ऐसे कई आदेश राज्य सरकारों द्वारा अस्वीकार भी कर दिए जाते हैं, जिससे तनाव की स्थिति उत्पन्न होती है।

8. अन्य फुटकर कारण-इस श्रेणी में राज्यों की सीमाओं अथवा नाम में परिवर्तन, नए राज्यों का निर्माण, भाषा का विवाद तथा नदियों के जल के बँटवारे से सम्बन्धित प्रश्न शामिल हैं। कई बार केंद्र द्वारा बनाई गई औद्योगिक नीति भी केंद्र तथा राज्यों के बीच तनाव का कारण बन जाती है।

प्रश्न 5.
सरकारिया आयोग पर एक नोट लिखें।
उत्तर:
सन् 1983 में तत्कालीन प्रधानमन्त्री श्रीमती इन्दिरा गाँधी ने केंद्र-राज्यों के सम्बन्धों पर विचार करने के लिए एक तीन-सदस्यीय आयोग का गठन किया। न्यायमूर्ति आर०एस० सरकारिया को इस आयोग का अध्यक्ष तथा पी० शिवरामन और एस०आर० सेन को इस आयोग का सदस्य नियुक्त किया गया था। इस आयोग ने सरकार को अपनी रिपोर्ट 27 अक्तूबर, 1987 को पेश की। इस आयोग की मुख्य सिफारिशें इस प्रकार हैं

(1) आयोग ने राज्य सरकारों के वित्तीय साधन बढ़ाने के उपायों पर जोर दिया है,

(2) आयोग ने राज्यों के अधिकार बढ़ाने के लिए संविधान में संशोधन के सुझाव दिए हैं। एक सुझाव यह है कि संविधान में संशोधन करके राज्यों को अनुच्छेद 242 के अन्तर्गत राज्य सूची के कानूनों में संशोधन करने का अधिकार दिया जाए,

(3) आयोग ने केंद्र व राज्य के विवादों को सुलझाने के लिए स्थायी अन्तर्राज्यीय परिषद् गठित किए जाने की सिफारिश की है। प्रधानमन्त्री, केंद्रीय मन्त्रियों और सभी मुख्यमन्त्रियों को इस परिषद् में रखा जाए,

(4) आयोग के अनुसार राज्यपालों की नियुक्ति से पहले सम्बन्धित मुख्यमन्त्री से परामर्श किया जाना चाहिए,

(5) राज्यों के विधेयकों पर राष्ट्रपति की स्वीकृति में विलम्ब रोका जाना चाहिए,

(6) उच्च न्यायालयों में न्यायाधीशों की नियुक्ति में विलम्ब रोका जाना चाहिए,

(7) उच्च न्यायालयों के न्यायाधीशों का स्थानान्तरण उनकी स्वीकृति से किया जाना चाहिए,

(8) आयोग ने संविधान की समवर्ती सूची के अन्तर्गत विषयों पर कानून बनाने के सम्बन्ध में संयम बरतने की सलाह दी है,

(9) आयोग ने राज्य सूची के विषयों पर केंद्रीय योजनाएँ बनाने पर रोक लगाने का सुझाव दिया है,

(10) आयोग ने सुझाव दिया है कि स्थानीय निकायों के नियमित चुनाव कराने और इनके समुचित कामकाज का संसद द्वारा कानून बनाना चाहिए,

(11) समवर्ती सूची के विषयों पर कानून बनाने से पहले केंद्र द्वारा राज्यों से परामर्श अनिवार्य होना चाहिए। इसके लिए दृढ़ परम्परा का पालन किया जाए,

(12) आयोग ने कहा है कि सामान्य तौर पर केंद्र को केवल उन क्षेत्रों में कार्रवाई करनी चाहिए जिनमें राष्ट्र के व्यापक हित में एक-सी नीति और कार्रवाई जरूरी है,

(13) आयोग ने राष्ट्रीय विकास परिषद्, नीति आयोग जैसे संगठनों तथा अखिल भारतीय सेवाओं को मजबूत बनाने के भी सुझाव दिए हैं,

(14) आयोग की राय में राज्यों की क्षेत्रीय परिषदों की व्यवस्था असफल रही है और इसे फिर सक्रिय किया जाना चाहिए,

(15) आयोग ने देश की एकता और अखण्डता के हित में सभी राज्यों में समान रूप से त्रि-भाषा फार्मूले को इसकी सही भावना से लागू करने के लिए प्रभावशाली कदम उठाने की सिफारिश की है,

(16) राजभाषा के रूप में हिन्दी को सरल बनाने पर जोर देते हुए आयोग ने कहा है कि आसानी से समझे जाने वाले शब्दों के स्थान पर कठिन सांस्कृतिक शब्दों का उपयोग उचित नहीं है,

(17) आयोग ने कहा है कि राजभाषा के विकास के लिए यदि अंग्रेज़ी सहित विभिन्न क्षेत्रीय भाषाओं के उन प्रचलित शब्दों और रूप को, जो अब हिन्दुस्तान के हिस्से बन चुके हैं, निकाला गया तो वह संविधान के अनुच्छेद की भावना के विरुद्ध होगा।

आयोग के अनुसार संविधान के अन्तर्गत केंद्र और राज्यों के बीच अधिकारों के वितरण में राज्यों की स्वतन्त्रता की आवश्यकताओं के साथ एक मजबूत केंद्र की जरूरत को महत्त्व दिया गया है। सरकारिया आयोग की सभी सिफारिशें केंद्रीय सरकार के विचाराधीन हैं और अभी तक इन पर कोई कार्रवाई नहीं की गई है।

प्रश्न 6.
राज्य की स्वायत्तता का क्या अर्थ है? इसकी माँग के मुख्य कारणों का वर्णन करें।
उत्तर:
भारत में केंद्र तथा राज्यों के बीच शक्तियों का विभाजन प्रारम्भ से ही विवादपूर्ण रहा है। संविधान सभा में भी अनेक सदस्यों ने यह आपत्ति उठाई थी कि शक्ति विभाजन की यह योजना भारत संघ के घटक राज्यों को ‘नगरपालिकाओं’ का रूप देती है। वास्तविकता भी यही है कि भारत के संविधान में केंद्र सरकार को विस्तृत शक्तियाँ दी गई हैं और राज्य नामक इकाइयों को निश्चित रूप से कमजोर रखा गया है।

सन् 1967 तक राज्यों में तथा केंद्र में एक ही दल काँग्रेस की सरकार सत्ता में रहने से केंद्र राज्यों के बीच विवाद नहीं उठे, किन्तु सन् 1967 के बाद जब देश के 8 राज्यों में दूसरे दलों की गैर-काँग्रेसी सरकारें बनी तो केंद्र-राज्यों के बीच शक्तियों के वितरण और सामंजस्य की समस्या पैदा हो गई। राज्यों की ओर से स्वायत्तता की माँग उठी।

राज्यों की स्वायत्तता की माँग के समर्थक यह मानते हैं कि संविधान में कई ऐसे प्रावधान हैं जो राज्यों की स्वायत्तता को सीमित करते हैं। यहाँ हम राज्यों की स्वायत्तता से सम्बन्धित विभिन्न प्रश्नों; जैसे राज्य की स्वायत्तता की माँग के कारण एवं स्वायत्तता के पक्ष एवं विपक्ष में विभिन्न तर्कों का उल्लेख करेंगे।

राज्य की स्वायत्तता का अर्थ साधारण शब्दों में स्वायत्तता का अर्थ है कि किसी को भी अपने क्षेत्र में निर्बाध कार्य करने की स्वतन्त्रता अर्थात् आन्तरिक व बाह्य कार्य-क्षेत्र में किसी भी प्रकार का हस्तक्षेप न होना। राज्यों के मामले में स्वायत्तता का अर्थ थोड़ा-सा भिन्न है। राज्यों की स्वायत्तता का अर्थ स्वतन्त्रता नहीं है, बल्कि इसका अर्थ है कि राज्यों को उनके मामलों में केंद्रीय सरकार द्वारा किसी भी प्रकार का हस्तक्षेप नहीं किया जाना चाहिए।

राज्यों को शक्तियाँ संविधान द्वारा उपलब्ध कराई गई हैं और उन्हें उनका प्रयोग करने की पूर्ण स्वतन्त्रता होनी चाहिए। राज्यों को जन-कल्याण की योजनाएँ बनाने एवं उन्हें लागू करने की शक्तियाँ बिना किसी रोक-टोक के प्राप्त होनी चाहिएँ। यही नहीं वित्तीय क्षेत्र में भी राज्य स्वतन्त्र होने चाहिएँ। तभी राज्य की स्वायत्तता को लागू किया जा सकता है।

दूसरे शब्दों में, केंद्र का राजनीतिक व प्रशासनिक मामलों में न्यूनतम हस्ताक्षेप होना चाहिए। केंद्र का कार्य-क्षेत्र सीमित होना चाहिए। उसे केवल विदेश सम्बन्ध, रक्षा, मुद्रा और जन-संचार के विषयों के मामलों में शक्तियाँ प्रदान की जानी चाहिएँ। कराधान के क्षेत्र में भी उनकी शक्तियाँ सीमित होनी चाहिएँ। उन्हें केवल उतने ही कर लगाने का अधिकार दिया ।

जाना चाहिए, जितने उन्हें उपरोक्त कार्य सम्पन्न करने के लिए आवश्यक हों। राज्यों को कराधान के इतने अधिकार प्रदान किए जाने चाहिएँ, जिससे कि वे साधनों का अभाव महसूस न करें। अतः राज्यों की स्वायत्तता का अर्थ न तो राज्यों की स्वतन्त्रता से है और न ही प्रभुसत्ता से। यह एक ऐसा वैधानिक दर्जा है जिसमें राज्यों को कुछ क्षेत्रों में पूर्ण स्वतन्त्रता तथा कम-से-कम केंद्रीय हस्तक्षेप का आश्वासन प्राप्त हो एक निश्चित क्षेत्र में स्वतन्त्रतापूर्वक कार्य के अधिकार का नाम ही राज्यों की स्वायत्तता है।

राज्य की स्वायत्तता की मांग के कारण (Causes of demand for State Autonomy) भारत में राज्यों पर केंद्र के नियन्त्रण के अनेक साधन हैं। इन साधनों के कारण ही राज्यों की स्वायत्तता की माँग ने जन्म लिया। उन कारणों का विवरण निम्नलिखित है

1. संसद की व्यापक विधि निर्माण शक्तियाँ संविधान द्वारा केंद्र और राज्यों के बीच विधायी शक्तियों का बंटवारा अवश्य किया गया है, परन्तु निम्नलिखित परिस्थितियों में संसद उन विषयों पर भी कानून बना सकती है जो राज्य सूची में दिए गए हैं (1) यदि राज्यसभा दो-तिहाई बहुमत से यह प्रस्ताव पास कर दे कि राष्ट्रीय हित के लिए यह आवश्यक है।

संसद राज्य सूची के किसी विषय पर भी कानून बनाए तो संसद उस पर कानून बना सकती है, (2) राष्ट्रपति द्वारा आपात्काल की घोषणा हो जाने पर संसद राज्य सूची में सम्मिलित विषयों पर भी कानून बना सकती है। केंद्रीय संसद की शक्ति की व्यापकता का तीन बातों से पता चलता है-प्रथम, यदि समवर्ती सूची में तथा उन दोनों में कोई विरोध हो तो संसद द्वारा निर्मित कानून मान्य होगा। द्वितीय, अवशिष्ट

शक्तियाँ केंद्र को प्राप्त हैं। तृतीय, यदि राज्य विधानमण्डल द्वारा स्वीकृत किसी विधेयक का सम्बन्ध किसी निजी सम्पत्ति पर कब्जा करने अथवा उच्च न्यायालयों की शक्तियों को कम करने से हो तो राज्यपाल के लिए यह जरूरी है कि उस विधेयक को वह राष्ट्रपति के पास उनकी स्वीकृति के लिए भेजे। अतः राज्य प्रशासन में केंद्र के अत्यधिक हस्तक्षेप के कारण ही राज्यों के द्वारा स्वायत्तता की माँग जोर पकड़ रही है।

2. वित्तीय दृष्टि से राज्यों की केंद्र पर निर्भरता वित्तीय दृष्टि से भी राज्यों को केंद्र का मुँह ताकना पड़ता है। केंद्र पर राज्यों की आर्थिक निर्भरता दिन-प्रतिदिन बढ़ती जा रही है।

3. अखिल भारतीय सेवाएँ तथा राज्यपाल अखिल भारतीय सेवाएँ जैसे भारतीय प्रशासनिक सेवा (I.A.S.) तथा भारतीय . पुलिस सेवा (I.P.S.) पर भारत की संघीय सरकार का नियन्त्रण है। इन सेवाओं से सम्बन्धित उच्च अधिकारी राज्यों में अनेक महत्त्वपूर्ण पदों पर नियुक्त होते हैं। अतएव इन अधिकारियों के माध्यम से ही केंद्रीय सरकार राज्यों की सरकारों पर नियन्त्रण रखती है। जहाँ तक राज्यपाल का प्रश्न है, उनकी नियुक्ति राष्ट्रपति करता है तथा वह राज्य में केंद्र के एजेण्ट के रूप में कार्य करता है। .

4. राज्यों के बीच भाषायी एवं सांस्कृतिक विभिन्नता भारत में राज्यों की भाषायी एवं सांस्कृतिक विभिन्नता भी राज्यों की स्वायत्तता की माँग को बढ़ाने में एक महत्त्वपूर्ण कारक रहा है। इसी आधार पर कुछ राज्य यह महसूस करते हैं कि हिन्दी भाषी राज्य गैर-हिन्दी भाषी राज्यों पर अपना आधिपत्य एवं प्रभाव जमाने का प्रयास कर रहे हैं। इसी भावना से ग्रस्त होकर सन् 1960 के दशक में हिन्दी भाषा के विरोध में गैर-हिन्दी भाषायी राज्यों के द्वारा आन्दोलन चलाए गए। अतः ऐसी माँग एवं भावनाएँ ही भारतीय संघवाद के स्वरूप को चुनौती देने के साथ-साथ राज्यों की स्वायत्तता की माँग के रूप में आगे बढ़ती जा रही हैं।

5. राज्यपाल की भूमिका एवं राष्ट्रपति शासन-अमेरिका, ऑस्ट्रेलिया, स्विट्जरलैंड जैसे संघों में केंद्र को यह शक्ति प्रदान नहीं है कि राज्यों की स्वायत्तता (Autonomy) समाप्त कर सके, किंतु भारत में आपात्काल की घोषणा किए जाने पर संविधान एकात्मक रूप धारण कर लेता है। आपात्काल में केंद्रीय संसद उन विषयों पर कानून बना सकती है जो राज्य सूची में सम्मिलित हैं।

जब राष्ट्रपति यह घोषणा कर देता है कि किसी राज्य की सरकार संविधान की धाराओं के अनुसार नहीं चलाई जा रही, तो राज्य की विधानसभा भंग कर दी जाती है और राष्ट्रपति के माध्यम से राज्यपाल को राज्य की सभी प्रशासनिक शक्तियों का प्रयोग करने का अधिकार प्राप्त हो जाता है। राज्य का राज्यपाल जनता के द्वारा निर्वाचित नहीं होता। राज्यपाल केंद्र सरकार के हस्तक्षेप से राष्ट्रपति द्वारा नियुक्त होता है।

इसलिए राज्यपाल केवल मात्र राज्यों में केंद्र सरकार का एजेण्ट बनकर कार्य करता है। केंद्र सरकार अपनी राजनीतिक महत्त्वाकांक्षाओं की पूर्ति करने के लिए राज्यों में अनुच्छेद 356 का प्रयोग कर विशेषतः विपक्षी दलों की सरकार को भंग करने का प्रयास करते हैं। इस प्रकार केंद्र सरकार राज्यपाल के पद के माध्यम से राज्यों में अनावश्यक हस्तक्षेप करती है।

इसी कारण आज राज्यपाल का पद सबसे अधिक विवादित पद बना हुआ है। राज्यों में राज्यपाल के निर्णयों एवं केंद्र द्वारा राष्ट्रपति शासन लागू करने की शक्ति के विरुद्ध सर्वोच्च न्यायालय में विभिन्न राज्यों द्वारा दायर याचिकाओं पर, न्यायपालिका ने निर्णय दिया कि केंद्र सरकार द्वारा अनुच्छेद 356 का प्रयोग गलत ढंग से किया जाता रहा है।

इसीलिए केंद्र राज्य सम्बन्धों सम्बन्धी गठित सरकारिया आयोग ने भी अपनी रिपोर्ट में यह संस्तुति दी थी कि राज्यपाल के पद पर गैर-राजनीतिक व्यक्ति की नियुक्ति की जाए ताकि यह संविधान के अनुसार बिना केंद्र सरकार के हस्तक्षेप के निष्पक्ष होकर अपना कार्य कर सके, परन्तु अभी तक इस समस्या का कोई हल नहीं हो पाया है। इसीलिए राज्यों के द्वारा निरन्तर स्वायत्तता की माँग जोर पकड़ती जा रही है।

6. अन्तर्राज्यीय झगड़े-राज्यों के बीच भाषायी एवं सांस्कृतिक विभिन्नता के साथ-साथ राज्यों के बीच सीमा सम्बन्धी एवं नदी जल सम्बन्धी विवादों का केंद्र सरकार द्वारा समुचित समाधान न करवाने के कारण भी राज्यों की स्वायत्तता की माँग को बल दिया है। महाराष्ट्र एवं कर्नाटक, मणिपुर और नागालैंड एवं पंजाब व हरियाणा के बीच राजधानी चण्डीगढ़ को लेकर आज भी विवाद कायम है।

इसी तरह से तमिलनाडु एवं कर्नाटक के बीच कावेरी नदी जल विवाद एवं गुजरात, मध्यप्रदेश एवं महाराष्ट्र के बीच नर्मदा नदी जल-विवाद आदि ने राज्यों के बीच संघात्मक भावना को जहाँ ठेस पहुंचाई है, वहीं केंद्र के प्रभावी नियन्त्रण के अभाव ने भी राज्यों की स्वायत्तता की माँग को बल देने का कार्य किया है।

7. संसद किसी नवीन राज्य का निर्माण कर सकती है और किसी भी राज्य की सीमा घटा या बढ़ा सकती है –अमेरिका . या ऑस्ट्रेलियाई संघ व्यवस्था में केंद्र राज्यों की इच्छा के विरुद्ध उनकी सीमाओं में हेर-फेर नहीं कर सकता, परन्तु भारत में केंद्रीय संसद नवीन राज्यों का निर्माण कर सकती है और राज्यों के आकार को घटा या बढ़ा सकती है। ऐसा करने के लिए संसद को राज्यों की अनुमति नहीं लेनी पड़ती।

8. राज्यसभा में सभी राज्यों को समान प्रतिनिधित्व नहीं विश्व की अधिकांश संघ व्यवस्थाओं में संसद के उच्च सदन का संगठन राज्यों की समानता के सिद्धान्त के आधार पर किया गया है। समानता का सिद्धान्त इसलिए अपनाया गया है जिससे केंद्रीय संसद पर बड़े राज्य का आधिपत्य कायम न हो सके, परन्तु भारत के उच्च सदन अर्थात् राज्यसभा में सभी राज्यों का बराबर संख्या में प्रतिनिधित्व नहीं होता।

9. राज्यों के अपने संविधान नहीं हैं-अमेरिका और स्विट्जरलैंड के राज्यों के अपने पृथक् संविधान हैं और उनमें संशोधन करने की शक्तियाँ भी राज्यों के विधानमण्डलों को ही प्राप्त हैं, परन्तु भारत में एक ही संविधान है जो केंद्र और राज्यों दोनों की संरचना और शक्तियों का उल्लेख करता है। राज्यों को यह अधिकार प्राप्त नहीं है कि वे भारतीय संविधान की उन धाराओं में संशोधन कर सकें जिनका उनकी संरचना और प्रकार्यों से सम्बन्ध है। भारतीय संविधान में संशोधन प्रक्रिया की शुरुआत केवल संसद ही कर सकती है।

निष्कर्ष दिए गए कारणों की वजह से विभिन्न राज्यों में राज्य की स्वायत्तता की माँग ने जोर पकड़ा। डी०एम०के० तथा : अन्ना डी०एम०के० ने अपने राज्य के लिए स्वायत्तता की माँग की। मार्क्सवादी दल ने संविधान की प्रस्तावना से ‘यूनियन’ शब्द को हटाकर ‘फेडरल’ शब्द का उल्लेख करने की माँग की। इसी प्रकार अकाली दल ने पंजाब में राज्य की स्वायत्तता के लिए उग्र आन्दोलन चलाया। जम्मू और कश्मीर में भी इसी प्रकार की माँग को लेकर कई सम्मेलनों का आयोजन किया गया।

प्रश्न 7.
राज्यों की स्वायत्तता के विभिन्न प्रश्नों पर प्रकाश डालिए।
उत्तर:
राज्यों की स्वायत्तता के विभिन्न प्रश्न (Various Issues of State Autonomy)-राज्यों की स्वायत्तता का मामला एक अहम् एवं कठिन विषय है जिसके चलते देश की एकता पर संकट आ सकता है। समय-समय पर राज्यों की स्वायत्तता से सम्बन्धित जिन मामलों पर आवाज उठाई गई वे विषय इस प्रकार हैं

1. राज्यपाल की नियुक्ति राज्यपाल की नियुक्ति का अधिकार भारत के राष्ट्रपति का है। वह राज्यपाल की नियुक्ति प्रधानमन्त्री के परामर्श से करता है। साधारणतया राज्यपाल की नियुक्ति में अमुक राज्य के मुख्यमन्त्री की सलाह नहीं ली जाती। विपक्षी दलों की यह माँग रही है कि राज्यपाल की नियुक्ति के समय राज्य के मुख्यमन्त्री की सलाह ली जानी चाहिए।

केंद्र द्वारा अनेकों बार राज्य की सलाह लिए बिना राज्यपाल की नियुक्ति की है जिसका राज्य सरकार द्वारा विरोध किया जाता है। इस प्रकार राज्य सरकार और राज्यपाल के सम्बन्धों में खटास पैदा हो जाती है। यही नहीं जब केंद्र में सत्ता परिवर्तन होता है तो जिन राज्यों में विपक्ष सत्ता में होता है तो राज्यपालों के इस्तीफे माँग लिए जाते हैं।

जैसा 2004 में काँग्रेस के सत्ता में आने के बाद, एन०डी०ए० (N.D.A.) द्वारा नियुक्त राज्यपालों के इस्तीफे माँग लिए गए। इस विवाद को लेकर अन्तर्राज्यीय परिषद् में कई बार चर्चा हुई और यह सुझाव दिया गया कि राज्यपाल की नियुक्ति में सम्बन्धित राज्य से सलाह लेने की प्रक्रिया को संवैधानिक दर्जा दिया जाए, लेकिन अभी तक कुछ नहीं हुआ।

2. राज्यपाल की भूमिका तथा राष्ट्रपति शासन-राज्यपाल की नियुक्ति केंद्र द्वारा की जाती है और वह केंद्र के प्रतिनिधि के रूप में कार्य करता है। राज्यपाल कई बार केंद्र में सत्तारूढ़ दल के दबाव में आकर ऐसे कार्य कर देता है जो उसे नहीं वह राज्य में संवैधानिक मशीनरी फेल होने की रिपोर्ट राष्ट्रपति को भेज देता है जिसके स्वीकार होते ही राज्य में राष्ट्रपति शासन लागू हो जाता है।

इस मामले में राज्यपाल की भूमिका विवादास्पद रही है। राज्यपाल निष्पक्ष तरीके से काम नहीं करता। अभी तक अनेकों उदाहरण हैं जबकि राज्यपाल से रिपोर्ट प्राप्त होने के बाद राष्ट्रपति ने अपनी शक्ति का दुरुपयोग किया। सन् 1952 से अब तक लगभग 130 बार से भी अधिक राज्यों में राष्ट्रपति शासन लागू किया जा चुका है। इन मामलों में राज्यपाल की भूमिका निन्दा का पात्र बनी। सन् 2005 में बिहार में राज्यपाल की सिफारिश पर राष्ट्रपति शासन लागू कर दिया गया जिसकी विपक्ष ने जमकर आलोचना की। ऐसे में राज्यों की स्वायत्तता माँगना स्वाभाविक है।

3. राज्यों के विधेयकों पर राष्ट्रपति की मंजूरी मिलने में देरी-राज्यों को यह शिकायत रहती है कि जिन विधेयकों को विधानमण्डल पारित कर देती है, उन पर स्वीकृति देने में केंद्र अनावश्यक देरी करता है। कुछ पर स्वीकृति नहीं दी जाती और इसकी सूचना तक भी राज्य सरकार को नहीं दी जाती। कुछ पर भेदभाव की नीति अपनाई जाती है। माँग होने पर भी राष्ट्रपति की स्वीकृति की अवधि निश्चित करने के लिए संविधान में संशोधन नहीं किया जाता। इस प्रकार राज्यों के विधेयकों पर राष्ट्रपति की स्वीकृति मिलने में देरी होना भी राज्यों की स्वायत्तता की माँग बढ़ाता है।

4. अखिल भारतीय सेवाओं का पक्षपातपूर्ण प्रयोग-अखिल भारतीय सेवाएँ केंद्र द्वारा नियन्त्रित और अनुशासित हैं। इन सेवाओं के किसी सदस्य का व्यवहार सेवा की शर्तों, आचार संहिता आदि के कितना ही विरुद्ध, आपत्तिजनक एवं पक्षपातपूर्ण क्यों न हो राज्य सरकारें केंद्र की अनुमति के बिना इन सेवाओं के सदस्यों के विरुद्ध कोई अनुशासनात्मक कार्रवाई नहीं कर सकती।

इससे राज्य सरकारें शिथिल होती हैं। कई बार तो केंद्र से प्रोत्साहन पाकर इन सेवाओं के सदस्य राज्य सरकारों की योग्यता, कुशलता पर ही प्रश्न-चिह्न लगा देते हैं। केंद्र में जब सत्ताधारी पार्टी इन सेवाओं का प्रयोग विपक्षी राज्य सरकारों को बदनाम करने के लिए करती है और सेवा के उन सदस्यों को पदोन्नत कर पुरस्कृत करती है जो राज्य सरकारों की उपेक्षा कर केंद्र के आदेशों का सीधे पालन करते हैं तो राज्यों में बेचैनी पैदा होती है।

इस प्रकार अखिल भारतीय सेवाओं का पक्षपातपूर्ण प्रयोग, केंद्र एवं राज्यों के सम्बन्धों में दरार पैदा करता है। अतः स्पष्ट है कि अखिल भारतीय सेवाओं के पक्षपातपूर्ण ढंग के प्रयोग ने राज्यों की स्वायत्तता की माँग को बढ़ावा दिया है।

5. केंद्र का राज्यों की विपक्षी सरकारों एवं उनकी समस्याओं के प्रति असवेदनशील एवं उदासीन व्यवहार केंद्र का दृष्टिकोण राज्यों में विपक्षी दलों की सरकारों के प्रति असंवेदनशील और उदासीन रहा है। केंद्र एक लम्बे समय तक समस्याओं पर हाथ पर हाथ धरे बैठा रहता है, जो अन्ततः केंद्र और राज्यों में टकराव पैदा करती हैं।

विपक्षी दल की राज्य सरकारों के प्रति असंवेदनशीलता का एक अन्य पहलू दो राज्य सरकारों के परस्पर विवाद में उस राज्य सरकार की समस्या के प्रति उदासीन हो जाना भी देखने में आया, जो दूसरे दल की थी। जनवरी, 1994 में पानी को लेकर हरियाणा और दिल्ली की सरकार के विवाद को इस श्रेणी में रखा जा सकता है। इसके अन्तर्गत हरियाणा के मुख्यमन्त्री ने दिल्ली को होने वाली पानी की सप्लाई को कम कर दिया, चूंकि दिल्ली में भाजपा की सरकार थी।

इस पर केंद्र सरकार चुप बनी रही और उस पर भी तभी जूं रेंगी जब दिल्ली सरकार ने केंद्रीय मन्त्रियों की पानी की सप्लाई को रोकने की चेतावनी दी। उपर्युक्त विवेचन से स्पष्ट होता है कि केंद्र-राज्य, सम्बन्ध में केंद्र सरकार की नीति प्रारम्भ से ही राज्यों को अपने ऊपर आश्रित बनाए रखने की रही है। सन् 1967 से राज्यों में केंद्र के हस्तक्षेप की यह नीति खुलकर सामने आई जो कि सन् 1995 तक जारी रही।

सन् 1996 से केंद्र में मिली-जुली सरकारों में क्षेत्रीय दलों के प्रभावी होने के कारण यह प्रवृत्ति कमजोर हो रही है और आम-सहमति की नीति का विकास हो रहा है। अतः कहा जा सकता है कि केंद्र के राज्यों के प्रति असंवेदनशील व्यवहार ने राज्यों की स्वायत्तता की माँग को न केवल जन्म दिया, बल्कि उसे बढ़ावा भी दिया है।

6. राज्यों की वित्तीय दुर्दशा-राज्यों की वित्तीय स्थिति में निरन्तर गिरावट ने नीति निर्धारकों के सामने जटिल समस्याएँ खड़ी कर दी हैं जिनके कारण राज्यों में न तो विकास गति पकड़ रहा है और न ही पर्याप्त बुनियादी सुविधाएं मुहैया कराई जा रही हैं। राजस्व में निरन्तर कमी, वित्तीय और प्राथमिक घाटे, कों के बढ़ते बोझ और अन्य देनदारियाँ, पूँजीगत खाते और रख-रखाव के खर्चों में कमी इत्यादि राज्यों की वित्तीय दुर्दशा के संकेत हैं।

साथ ही राज्यों को मिलने वाली केंद्रीय सहायता की वृद्धि का संकुचन और केंद्रीय वेतन पुनः निरीक्षण के प्रभाव ने राज्यों की वित्तीय स्थिति पर घातक प्रभाव डाला है। ऐसी स्थिति के लिए केंद्र की अपेक्षा राज्य सरकारें स्वयं ही अधिक जिम्मेदार हैं। अतः राज्यों की स्वायत्तता के लिए राज्यों की वित्तीय दुर्दशा भी एक महत्त्वपूर्ण मुद्दा है।

7. दूरदर्शन और आकाशवाणी का पक्षपातपूर्ण इस्तेमाल केंद्र का दूरदर्शन और आकाशवाणी जैसे महत्त्वपूर्ण जन-संचार साधनों पर पूर्ण नियन्त्रण और एकाधिकार है। सन् 1966 में चन्दा समिति ने इन्हें स्वायत्तशासी बनाने का सुझाव दिया था, लेकिन इसे नामंजूर कर दिया। दूरदर्शन और आकाशवाणी एक ऐसा संवेदनशील मुद्दा बन गया है जिसने केंद्र और राज्यों के बीच उत्तेजना पैदा की है।

राज्यों की जन-संचार के इन साधनों के विरुद्ध ये शिकायतें रहीं-

(1) खबरें निष्पक्ष भाव से एवं पक्षपातरहित होकर प्रसारित नहीं की जातीं। यह स्थिति अत्यधिक उत्तेजना उस समय पैदा करती है जब किसी महत्त्वपूर्ण मुद्दे पर केंद्र और विपक्षी राज्य सरकारों में नीति सम्बन्धी कोई महत्त्वपूर्ण मतभेद होता है और दूरदर्शन एवं आकाशवाणी राज्यों के दृष्टिकोण की उपेक्षा करके केवल केंद्र में सत्ताधारी पार्टी के दृष्टिकोण को प्रस्तुत करते हैं।

(2) राज्यों को विशेषकर चुनाव के समय प्रचार सुविधाएँ नहीं दी जातीं।

(3) देश के विविध जातीय एवं सांस्कृतिक समूहों को समुचित प्रतिनिधित्व न देना। इस प्रकार केंद्र द्वारा दूरदर्शन और आकाशवाणी को पक्षपातपूर्ण इस्तेमाल करना, राज्यों के लिए द्वितीय या अतिरिक्त चैनल खोलने की माँग को जन्म देती है। अतः इस प्रकार दूरदर्शन और आकाशवाणी का पक्षपातपूर्ण तरीके से प्रयोग राज्यों की स्वायत्तता के लिए एक मुद्दा है।

8. समवर्ती सूची-राज्य और केंद्र के बीच शक्तियों के विभाजन के लिए संविधान में तीन सूचियों की व्यवस्था है। केंद्रीय सूची, राज्य सूची और समवर्ती सूची। समवर्ती सूची में ऐसे विषय रखे गए हैं जिन पर केंद्र और राज्यों दोनों को कानून बनाने का अधिकार प्राप्त है। संसद के द्वारा कानून बनाने की स्थिति में राज्यों द्वारा बनाया गया कानून मान्य नहीं होता। इसलिए विपक्षी दल समवर्ती सूची को पूर्णरूपेण राज्यों के अधीन करने के पक्ष में हैं।

9. अवशिष्ट शक्तियाँ अवशिष्ट शक्तियों को लेकर भी केंद्र व राज्यों में तनाव है। यह भी राज्यों की स्वायत्तता की माँग को बढ़ावा देने का कारण है। राज्य सभी अवशिष्ट शक्तियों को राज्यों के देने के पक्ष में है।

वस्तुनिष्ठ प्रश्न

निम्नलिखित प्रश्नों का उत्तर दिए गए विकल्पों में से उचित विकल्प छाँटकर लिखें

1. संविधान द्वारा भारत को कहा गया है
(A) संघीय राज्य
(B) एकात्मक राज्य
(C) राज्यों का संघ
(D) बनावट में संघात्मक, परंतु भाव में एकात्मक
उत्तर:
(C) राज्यों का संघ

2. संविधान में संघीय सूची में मूलतः विषयों की संख्या कितनी है?
(A) 97
(B) 99
(C) 100
(D) 102
उत्तर:
(A)97

3. संघीय सूची में दिए गए विषयों के सम्बन्ध में कानून बनाने का अधिकार है
(A) संसद के पास
(B) राज्यसभा के पास
(C) लोकसभा के पास
(D) राज्य विधानमंडल के पास
उत्तर:
(A) संसद के पास

4. संविधान में राज्य-सूची में मूलतः कुल विषय कितने दिए गए हैं?
(A) 97
(B) 61
(C) 66
(D) 47
उत्तर:
(C) 66

HBSE 11th Class Political Science Important Questions Chapter 7 संघवाद

5. संविधान में समवर्ती सूची में मूलतः कुल विषय कितने दिए गए हैं?
(A) 61
(B) 47
(C) 52
(D) 66
उत्तर:
(B) 47

6. समवर्ती सूची में दिए गए विषयों के संबंध में कानून पास करने का अधिकार है
(A) संसद के पास
(B) राज्य विधानमंडलों के पास
(C) संसद तथा राज्य विधानमंडल दोनों के पास
(D) इनमें से कोई नहीं
उत्तर:
(C) संसद तथा राज्य विधानमंडल दोनों के पास

7. समवर्ती सूची में दिए गए किसी विषय के संबंध में संसद तथा राज्य विधानमंडल द्वारा परस्पर विरोधी कानून बनाने की स्थिति में
(A) दोनों कानून रद्द हो जाएँगे
(B) संसद द्वारा पास किया गया कानन लाग होगा
(C) राज्य विधानमंडल द्वारा पास किया गया कानून लागू होगा
(D) दोनों में गतिरोध उत्पन्न हो जाएगा
उत्तर:
(B) संसद द्वारा पास किया गया कानून लागू होगा

8. भारतीय संघ में कुल कितने राज्य हैं?
(A) 26
(B) 21
(C) 28
(D) 29
उत्तर:
(C) 28

9. निम्नलिखित परिस्थितियों में संसद राज्य-सूची में दिए गए विषयों के संबंध में भी कानून बना सकती है
(A) संकटकालीन स्थिति की घोषणा होने पर
(B) दो अथवा दो से अधिक राज्यों द्वारा ऐसी प्रार्थना करने पर
(C) किसी अंतर्राष्ट्रीय संधि अथवा समझौते की शर्तों को लागू करने के लिए
(D) उपर्युक्त सभी परिस्थितियों में
उत्तर:
(D) उपर्युक्त सभी परिस्थितियों में

10. वित्त आयोग के सदस्यों की नियुक्ति करने का अधिकार निम्नलिखित को है
(A) राष्ट्रपति
(B) उप-राष्ट्रपति
(C) संसद
(D) प्रधानमंत्री
उत्तर:
(A) राष्ट्रपति

11. अंतर्राज्यीय परिषद् (Inter-State Council) की स्थापना करने का अधिकार निम्नलिखित को प्राप्त है
(A) राज्यपाल
(B) राष्ट्रपति
(C) प्रधानमंत्री
(D) मुख्य न्यायाधीश
उत्तर:
(B) राष्ट्रपति

12. भारत की संघीय व्यवस्था का संरक्षक है
(A) राष्ट्रपति
(B) संसद
(C) प्रधानमंत्री
(D) सर्वोच्च न्यायालय
उत्तर:
(D) सर्वोच्च न्यायालय

13. भारतीय संघ में नए राज्यों को शामिल तथा राज्यों की सीमाओं में परिवर्तन करने का अधिकार किसके पास है?
(A) राष्ट्रपति
(B) संसद
(C) प्रधानमंत्री
(D) सर्वोच्च न्यायालय
उत्तर:
(B) संसद

14. शिक्षा निम्नलिखित सूची का विषय है
(A) संघ सूची
(B) राज्य-सूची
(C) समवर्ती सूची
(D) अवशिष्ट शक्तियाँ
उत्तर:
(C) समवर्ती सूची

15. भारतीय संघ में कुल कितने संघीय क्षेत्र हैं?
(A) 7
(B) 10
(C) 9
(D) 8
उत्तर:
(D) 8

16. शेष शक्तियों (Residuary Powers) के संबंध में कानून बनाने का अधिकार निम्नलिखित को है
(A) राष्ट्रपति
(B) संसद
(C) राज्य विधानमंडल
(D) इनमें से कोई नहीं
उत्तर:
(B) संसद

17. निम्नलिखित में से कौन-सा विषय संघीय सूची में है?
(A) विदेशी मामले
(B) विवाह और तलाक
(C) शिक्षा
(D) स्थानीय सरकार
उत्तर:
(A) विदेशी मामले

18. निम्नलिखित विषय राज्य-सूची में शामिल है
(A) प्रतिरक्षा
(B) डाक
(C) कृषि
(D) इनमें से कोई नहीं
उत्तर:
(C) कृषि

19. निम्नलिखित विषय समवर्ती-सूची में शामिल हैं
(A) विदेशी मामले
(B) शिक्षा
(C) कृषि
(D) पुलिस एवं जेलें
उत्तर:
(B) शिक्षा

20. वित्त आयोग (Finance Commission) नियुक्त करने का अधिकार प्राप्त है
(A) प्रधानमंत्री को
(B) राष्ट्रपति को
(C) संसद को
(D) लोकसभा को
उत्तर:
(B) राष्ट्रपति को

21. “भारतीय संविधान रूप में तो संघात्मक है पर भावना में एकात्मक है।” यह किसने कहा?
(A) डी०एन० बनर्जी ने
(B) दुर्गादास बसु ने
(C) डॉ० अम्बेडकर ने
(D) के०सी० बीयर ने
उत्तर:
(A) डी०एन० बनर्जी ने

22. “भारतीय संविधान न तो पूर्ण रूप से संघात्मक है और न ही एकात्मक, बल्कि दोनों का मिश्रण है।” यह कथन किसका है?
(A) डी०डी० बसु का
(B) के०सी० बीयर का
(C) जी०एन० जोशी का
(D) जी०एन० सिंह का
उत्तर:
(A) डी०डी० बसु का

23. 42वें संवैधानिक संशोधन द्वारा समवर्ती सूची में जोड़े गए विषयों से समवर्ती सूची के विषयों की कुल संख्या कितनी हो गई?
(A) 47
(B) 49
(C) 52
(D) 66
उत्तर:
(C) 52

24. वर्तमान में राज्यसूची में कितने विषय हैं?
(A) 47
(B) 52
(C) 66
(D) 61
उत्तर:
(D) 61

निम्नलिखित प्रश्नों का उत्तर एक शब्द में दें

1. भारतीय संविधान में संघ के स्थान पर किस शब्द का प्रयोग किया गया है?
उत्तर:
राज्यों का संघ (Union of States)।

2. शक्तियों के विभाजन की तीन सूचियों में मूलतः कितने-कितने विषय शामिल हैं?
उत्तर:
संघीय सूची में 97, राज्य-सूची में 66 तथा समवर्ती सूची में 47 विषय शामिल हैं।

3. भारतीय संघ में कुल कितने राज्य एवं संघीय क्षेत्र हैं?
उत्तर:
भारतीय संघ में कुल 28 राज्य एवं 8 संघीय क्षेत्र हैं।

4. भारतीय संविधान का स्वरूप कैसा है?
उत्तर:
भारतीय संविधान का स्वरूप संघात्मक है।

5. भारत में अंतर्राज्यीय परिषद् की स्थापना कब हुई?
उत्तर:
भारत में अंतर्राज्यीय परिषद् की स्थापना 28 मई, 1990 को हुई।

6. संविधान में शक्तियों के विभाजन के आधार पर कौन-सी शासन-प्रणाली अपनाई गई है?
उत्तर:
संविधान में शक्तियों के विभाजन के आधार पर संघात्मक शासन-प्रणाली अपनाई गई है।

7. भारतीय संविधान द्वारा अवशिष्ट शक्तियाँ किसे प्रदान की गई हैं?
उत्तर:
भारतीय संविधान द्वारा अवशिष्ट शक्तियाँ संघीय संसद को प्रदान की गई हैं।

8. जम्मू-कश्मीर पुनर्गठन विधेयक राष्ट्रपति द्वारा कब पारित किया गया?
उत्तर:
जम्मू-कश्मीर पुनर्गठन विधेयक राष्ट्रपति द्वारा 9 अगस्त, 2019 को पारित हुआ।

9. नीति आयोग कब अस्तित्व में आया?
उत्तर:
1 जनवरी, 2015 को।

10. जम्मू-कश्मीर राज्य पुनर्गठन विधेयक लोकसभा एवं राज्यसभा द्वारा कब पारित किया गया?
उत्तर:
जम्मू-कश्मीर राज्य पुनर्गठन विधेयक लोकसभा एवं राज्यसभा में क्रमशः 5 एवं 6 अगस्त, 2019 को पारित हुआ।

रिक्त स्थान भरें

1. “भारतीय संविधान रूप में तो संघात्मक है, परन्तु भाव में एकात्मक है।” यह कथन ……………. ने कहा।
उत्तर:
डी०एन० बनर्जी

2. अवशिष्ट विषयों पर कानून बनाने का अधिकार ………….. को प्राप्त है।
उत्तर:
संसद

3. भारतीय संघ में …………… केंद्र शासित प्रदेश हैं।
उत्तर:
8

4. संघीय सूची में ……………. विषय हैं।
उत्तर:
97

5. राज्य सूची में …………… विषय हैं।
उत्तर:
61

6. शिक्षा ……………. सूची का विषय है।
उत्तर:
समवर्ती

7. भारत में वित्त आयोग नियुक्त करने का अधिकार को है।
उत्तर:
राष्ट्रपति

8. नीति आयोग के उपाध्यक्ष ………………….हैं।
उत्तर:
राजीव कुमार

HBSE 11th Class Political Science Important Questions Chapter 6 न्यायपालिका

Haryana State Board HBSE 11th Class Political Science Important Questions Chapter 5 विधायिका Important Questions and Answers.

Haryana Board 11th Class Political Science Important Questions Chapter 5 विधायिका

अति लघूत्तरात्मक प्रश्न

प्रश्न 1.
न्यायपालिका किसे कहते हैं?
उत्तर:
न्यायपालिका सरकार का तीसरा महत्त्वपूर्ण अंग है। इसका मुख्य कार्य कानूनों की व्याख्या करना तथा अपराधियों को दण्ड देना है।

प्रश्न 2.
न्यायपालिका का महत्त्व बताइए।
उत्तर:
लॉर्ड ब्राइस के अनुसार, “किसी शासन की श्रेष्ठता जानने के लिए उसके न्याय प्रबन्ध से बढ़िया कोई अन्य कसौटी नहीं।”

प्रश्न 3.
न्यायपालिका को स्वतन्त्र रखने के उपाय बताएँ।
उत्तर:

  • न्यायाधीशों का अच्छा वेतन।
  • न्यायाधीशों का लम्बा तथा निश्चित कार्यकाल।

प्रश्न 4.
न्यायाधीशों को अधिक वेतन देने के पक्ष में दो तर्क दीजिए।
उत्तर:

  • न्यायाधीशों को अधिक वेतन देने से वे रिश्वत और भ्रष्टाचार से बचे रह सकते हैं।
  • वे निष्पक्ष और स्वतंत्र निर्णय देने में सक्षम बन सकते हैं।

HBSE 11th Class Political Science Important Questions Chapter 6 न्यायपालिका

प्रश्न 5.
न्यायपालिका की स्वतन्त्रता का क्या अर्थ है?
उत्तर:
न्यायपालिका की स्वतन्त्रता का अर्थ है कि न्यायपालिका विधानमण्डल, कार्यपालिका तथा किसी अन्य व्यक्ति अथवा समूह के दबाव अथवा प्रभाव से पूरी तरह स्वतन्त्र हो।

प्रश्न 6.
संसार के विभिन्न देशों में न्यायाधीशों की नियुक्ति के लिए कौन-कौन से तरीके अपनाए जाते हैं तथा उनमें से सबसे अच्छा तरीका कौन-सा है?
उत्तर:
संसार के विभिन्न देशों में न्यायाधीशों की नियुक्ति के लिए प्रायः तीन तरीके अपनाए जाते हैं

  • जनता द्वारा चुनाव,
  • विधानमण्डल द्वारा चुनाव,
  • कार्यपालिका द्वारा नियुक्ति।

इन तीनों तरीकों में से कार्यपालिका द्वारा नियुक्ति ही अन्य दोनों तरीकों से अच्छी मानी गई है, क्योंकि इसमें न्यायाधीशों पर दबाव की सम्भावना कम-से-कम होती है।

प्रश्न 7.
सर्वोच्च न्यायालय के न्यायाधीश को कुल कितना वेतन मिलता है?
उत्तर:
सर्वोच्च न्यायालय के मुख्य न्यायाधीश को ₹ 2,80,000 तथा अन्य न्यायाधीशों को ₹ 2,50,000 मासिक वेतन मिलता है।

प्रश्न 8.
सर्वोच्च न्यायालय के न्यायाधीश के पद पर नियुक्त होने के लिए कोई दो योग्यताएँ लिखिए।
उत्तर:

  • वह भारत का नागरिक हो।
  • वह कम-से-कम 5 वर्ष तक उच्च न्यायालय में न्यायाधीश के रूप में अथवा कम-से-कम 10 वर्ष तक उच्च न्यायालय में वकील के रूप में कार्य कर चुका हो।

प्रश्न 9.
सर्वोच्च न्यायालय के मुख्य न्यायाधीश की नियुक्ति कौन करता है और किसकी सलाह से करता है?
उत्तर:
सर्वोच्च न्यायालय के मुख्य न्यायाधीश की नियुक्ति राष्ट्रपति सर्वोच्च न्यायालय के अन्य न्यायाधीशों तथा उच्च न्यायालयों के उन न्यायाधीशों, जिन्हें वह उचित समझता है, की सलाह से करता है। .

प्रश्न 10.
सर्वोच्च न्यायालय को किन दो मामलों में प्रारम्भिक क्षेत्राधिकार प्राप्त है?
उत्तर:

  • जब केन्द्र और राज्यों के बीच विवाद उत्पन्न हो जाए।
  • मौलिक अधिकारों के सम्बन्ध में।

प्रश्न 11.
सर्वोच्च न्यायालय को किन दो मामलों में अपीलीय क्षेत्राधिकार प्राप्त है?
उत्तर:

  • संविधान की व्याख्या के सम्बन्ध में।
  • उच्च न्यायालय ने अभियुक्त को मृत्यु-दण्ड की सजा दी हो।

प्रश्न 12.
सर्वोच्च न्यायालय के दो कार्य बताएँ।
उत्तर:

  • नागरिकों के मौलिक अधिकारों की रक्षा करना।
  • संविधान की व्याख्या एवं रक्षा करना।

प्रश्न 13.
मौलिक अधिकारों की रक्षा के लिए सर्वोच्च न्यायालय कौन-कौन-से लेख जारी करता है?
उत्तर:

  • बन्दी प्रत्यक्षीकरण लेख,
  • परमादेश लेख,
  • प्रतिषेध लेख,
  • उत्प्रेषण लेख,
  • अधिकार-पृच्छा लेख।

प्रश्न 14.
किन्हीं ऐसे दो राज्यों के नाम बताइए जिसका एक ही (साझा) उच्च न्यायालय है? वह कहाँ पर स्थित है?
उत्तर:
हरियाणा तथा पंजाब का एक ही साझा उच्च न्यायालय है जो चण्डीगढ़ में स्थित है।

प्रश्न 2.
न्यायपालिका का महत्त्व बताइए।
उत्तर:
लॉर्ड ब्राइस के अनुसार, “किसी शासन की श्रेष्ठता जानने के लिए उसके न्याय प्रबन्ध से बढ़िया कोई अन्य कसौटी नहीं।

प्रश्न 3.
न्यायपालिका को स्वतन्त्र रखने के उपाय बताएँ।
उत्तर:

  • न्यायाधीशों का अच्छा वेतन।
  • न्यायाधीशों का लम्बा तथा निश्चित कार्यकाल।

प्रश्न 4.
न्यायाधीशों को अधिक वेतन देने के पक्ष में दो तर्क दीजिए।
उत्तर:

  • न्यायाधीशों को अधिक वेतन देने से वे रिश्वत और भ्रष्टाचार से बचे रह सकते हैं।
  • वे निष्पक्ष और स्वतंत्र निर्णय देने में सक्षम बन सकते हैं।

HBSE 11th Class Political Science Important Questions Chapter 6 न्यायपालिका

प्रश्न 5.
न्यायपालिका की स्वतन्त्रता का क्या अर्थ है?
उत्तर:
न्यायपालिका की स्वतन्त्रता का अर्थ है कि न्यायपालिका विधानमण्डल, कार्यपालिका तथा किसी अन्य व्यक्ति अथवा समूह के दबाव अथवा प्रभाव से पूरी तरह स्वतन्त्र हो।

प्रश्न 6.
संसार के विभिन्न देशों में न्यायाधीशों की नियुक्ति के लिए कौन-कौन-से तरीके अपनाए जाते हैं तथा उनमें से सबसे अच्छा तरीका कौन-सा है?
उत्तर:
संसार के विभिन्न देशों में न्यायाधीशों की नियुक्ति के लिए प्रायः तीन तरीके अपनाए जाते हैं-

  • जनता द्वारा चुनाव,
  • विधानमण्डल द्वारा चुनाव,
  • कार्यपालिका द्वारा नियुक्ति।

इन तीनों तरीकों में से कार्यपालिका द्वारा नियुक्ति ही अन्य दोनों तरीकों से अच्छी मानी गई है, क्योंकि इसमें न्यायाधीशों पर दबाव की सम्भावना कम-से-कम होती है।

प्रश्न 7.
सर्वोच्च न्यायालय के न्यायाधीश को कुल कितना वेतन मिलता है?
उत्तर:
सर्वोच्च न्यायालय के मुख्य न्यायाधीश को ₹ 2,80,000 तथा अन्य न्यायाधीशों को ₹2,50,000 मासिक वेतन मिलता है।

प्रश्न 8.
सर्वोच्च न्यायालय के न्यायाधीश के पद पर नियुक्त होने के लिए कोई दो योग्यताएँ लिखिए।
उत्तर:

  • वह भारत का नागरिक हो।
  • वह कम-से-कम 5 वर्ष तक उच्च न्यायालय में न्यायाधीश के रूप में अथवा कम-से-कम 10 वर्ष तक उच्च न्यायालय में वकील के रूप में कार्य कर चुका हो।

प्रश्न 9.
सर्वोच्च न्यायालय के मुख्य न्यायाधीश की नियुक्ति कौन करता है और किसकी सलाह से करता है?
उत्तर:
सर्वोच्च न्यायालय के मुख्य न्यायाधीश की नियुक्ति राष्ट्रपति सर्वोच्च न्यायालय के अन्य न्यायाधीशों तथा उच्च न्यायालयों के उन न्यायाधीशों, जिन्हें वह उचित समझता है, की सलाह से करता है। .

प्रश्न 10.
सर्वोच्च न्यायालय को किन दो मामलों में प्रारम्भिक क्षेत्राधिकार प्राप्त है?
उत्तर:

  • जब केन्द्र और राज्यों के बीच विवाद उत्पन्न हो जाए।
  • मौलिक अधिकारों के सम्बन्ध में।

प्रश्न 11.
सर्वोच्च न्यायालय को किन दो मामलों में अपीलीय क्षेत्राधिकार प्राप्त है?
उत्तर:

  • संविधान की व्याख्या के सम्बन्ध में।
  • उच्च न्यायालय ने अभियुक्त को मृत्यु-दण्ड की सजा दी हो।

प्रश्न 12.
सर्वोच्च न्यायालय के दो कार्य बताएँ।
उत्तर:

  • नागरिकों के मौलिक अधिकारों की रक्षा करना।
  • संविधान की व्याख्या एवं रक्षा करना।

प्रश्न 13.
मौलिक अधिकारों की रक्षा के लिए सर्वोच्च न्यायालय कौन-कौन-से लेख जारी करता है?
उत्तर:

  • बन्दी प्रत्यक्षीकरण लेख,
  • परमादेश लेख,
  • प्रतिषेध लेख,
  • उत्प्रेषण लेख,
  • अधिकार-पृच्छा लेख।

प्रश्न 14.
किन्हीं ऐसे दो राज्यों के नाम बताइए जिसका एक ही (साझा) उच्च न्यायालय है? वह कहाँ पर स्थित है?
उत्तर:
हरियाणा तथा पंजाब का एक ही साझा उच्च न्यायालय है जो चण्डीगढ़ में स्थित है।

प्रश्न 15.
उच्च न्यायालय के न्यायाधीशों को कितना वेतन मिलता है?
उत्तर:
उच्च न्यायालय के मुख्य न्यायाधीश को ₹ 2,50,000 तथा अन्य न्यायाधीशों को ₹ 2,25,000 मासिक वेतन मिलता है।

प्रश्न 16.
किन्हीं दो मामलों में उच्च न्यायालय का प्रारम्भिक क्षेत्राधिकार बताइए।
उत्तर:

  • मौलिक अधिकारों के सम्बन्ध में।
  • संविधान की व्याख्या सम्बन्धी।

प्रश्न 17.
चुनाव याचिकाएँ किस न्यायालय में पेश की जाती हैं?
उत्तर:
एम०एल०ए० (M.L.A.) तथा संसद सदस्य (M.P.) के चुनावों से सम्बन्धित चुनाव याचिकाएँ उच्च न्यायालय में पेश की जाती हैं। राष्ट्रपति व उप-राष्ट्रपति के चुनाव से सम्बन्धित चुनाव याचिकाएँ केवल सर्वोच्च न्यायालय में ही पेश की जा सकती हैं।

प्रश्न 18.
न्यायपालिका की स्वतन्त्रता के दो तरीके बताइए।
उत्तर:

  • न्यायाधीशों के वेतन व भत्तों में उनके कार्यकाल के दौरान कटौती नहीं की जा सकती।
  • न्यायाधीशों को संसद, महाभियोग के अतिरिक्त अन्य किसी तरीके से हटा नहीं सकती।

प्रश्न 19.
सर्वोच्च न्यायालय के न्यायाधीशों की स्वतन्त्रता को किन दो तरीकों से सुनिश्चित किया गया है?
उत्तर:

  • न्यायाधीशों को वेतन व भत्ते संचित निधि से दिए जाते हैं और उनके कार्यकाल के दौरान उनमें कटौती नहीं की जा सकती।
  • दुर्व्यवहार अथवा अक्षमता के आरोपों की निष्पक्ष जाँच-पड़ताल होने पर संसद महाभियोग के अतिरिक्त न्यायाधीशों को अन्य किसी तरीके से नहीं हटा सकती।

प्रश्न 20.
क्या सर्वोच्च न्यायालय के न्यायाधीश अवकाश-प्राप्ति के बाद वकालत कर सकते हैं? क्या उन्हें जाँच आयोग का सदस्य बनाया जा सकता है?
उत्तर:
सर्वोच्च न्यायालय के न्यायाधीश अवकाश-प्राप्ति के बाद वकालत नहीं कर सकते, लेकिन उन्हें जाँच आयोग के सदस्य अथवा अध्यक्ष नियुक्त किया जा सकता है।

प्रश्न 21.
उच्च न्यायालयों के न्यायाधीशों की स्वतन्त्रता कैसे स्थापित की गई है?
उत्तर:

  • उच्च न्यायालयों के न्यायाधीशों की नियक्ति सर्वोच्च न्यायालय के मुख्य न्यायाधीश की सलाह से की जाती है।
  • उनको संसद महाभियोग के अतिरिक्त अन्य किसी तरीके से नहीं हटा सकती।

प्रश्न 22.
उच्च न्यायालय के न्यायाधीश अवकाश-प्राप्ति के बाद कहाँ वकालत कर सकते हैं ?
उत्तर:

  • सर्वोच्च न्यायालय में,
  • दूसरे राज्य के उच्च न्यायालय में।

प्रश्न 23.
ऐसे दो मामले बताइए जिनमें सर्वोच्च न्यायालय ने न्यायिक पुनरावलोकन के अधिकार का प्रयोग किया हो?
उत्तर:

  • ए०के० गोपालन केस, 1951,
  • मेनका गाँधी केस, 1980।

HBSE 11th Class Political Science Important Questions Chapter 6 न्यायपालिका

प्रश्न 24.
न्यायिक सक्रियता के तीन साधन लिखें।
उत्तर:
न्यायिक सक्रियता के तीन साधन हैं

  • न्यायिक पुनर्निरीक्षण,
  • मौलिक अधिकारों की व्याख्या,
  • संविधान की व्याख्या।

प्रश्न 25.
न्यायिक सक्रियता की आलोचना के तीन शीर्षक लिखें। अथवा न्यायिक सक्रियता के विपक्ष में तीन तर्क लिखिए।
उत्तर:

  • संसदीय व्यवस्था के विरुद्ध,
  • न्यायपालिका के प्राथमिक कार्य में बाधा,
  • प्रजातन्त्र के विरुद्ध।

प्रश्न 26.
न्यायिक क्रियाशीलता के पक्ष में तीन तर्क लिखें।
उत्तर:

  • सरकार को क्रियाशील बनाने के लिए आवश्यक,
  • मानवीय अधिकारों की रक्षा में सहायक,
  • लोगों का समर्थन।

प्रश्न 27.
सार्वजनिक हित के लिए मुकद्दमेबाजी का क्या अर्थ है?
उत्तर:
जब कोई व्यक्ति अथवा व्यक्ति समूह सार्वजनिक अथवा सामान्य हित के विषयों को लेकर न्यायालय का दरवाजा खटखटाता है तो उसे सार्वजनिक हित की मुकद्दमेबाजी (P.I.L) कहा जाता है।

प्रश्न 28.
न्यायपालिका कितने प्रकार के लेख (Writ) जारी कर सकती है?
उत्तर:
न्यायपालिका निम्नलिखित पाँच प्रकार के लेख (Writ) जारी कर सकती है

  • बन्दी प्रत्यक्षीकरण (Habeas Corpus),
  • परमादेश (Mandamus),
  • प्रतिषेध (Prohibition),
  • अधिकार पृच्छा (Quo-warranto),
  • उत्प्रेषण (Certiorari)।

लघूत्तरात्मक प्रश्न

प्रश्न 1.
न्यायपालिका का महत्त्व बताएँ।
उत्तर:
न्यायपालिका सरकार का तीसरा महत्त्वपूर्ण अंग है। इसका मुख्य कार्य विधानमण्डल द्वारा बनाए गए कानूनों की व्याख्या करना तथा उन्हें तोड़ने वालों को दण्ड देना है। एक राज्य में विधानमण्डल तथा कार्यपालिका की व्यवस्था चाहे कितनी भी अच्छी तथा श्रेष्ठ क्यों न हो, परन्तु यदि न्याय-व्यवस्था दोषपूर्ण है अर्थात निष्पक्ष तथा स्वतंत्र नहीं है, तो नागरिकों का जीवन सुखी नहीं रह सकता। न्यायपालिका ही नागरिकों के अधिकारों की रक्षा करती है और उन्हें सरकार के अत्याचार से बचाती है।

संघीय राज्यों में न्यायपालिका संविधान की रक्षा करती है और केन्द्रीय सरकार तथा राज्य सरकारों के बीच उत्पन्न होने वाले झगड़ों का निपटारा करती है। लॉर्ड ब्राइस (Lord Bryce) ने ठीक ही लिखा है, “किसी शासन की श्रेष्ठता जानने के लिए उसकी न्याय व्यवस्था की कुशलता से बढ़कर और कोई अच्छी कसौटी नहीं है, क्योंकि किसी और वस्तु का नागरिक की सुरक्षा और हितों पर इतना प्रभाव नहीं पड़ता जितना उसके इस ज्ञान से कि वह एक निश्चित, शीघ्र तथा निष्पक्ष न्याय व्यवस्था पर निर्भर करता है।”

प्रश्न 2.
न्यायपालिका के गठन पर प्रकाश डालो।
उत्तर:
न्यायपालिका का गठन भिन्न-भिन्न राज्यों में भिन्न-भिन्न पाया जाता है। वास्तविकता में न्यायपालिका का गठन देश में पाई जाने वाली शासन-व्यवस्था पर निर्भर करता है। कई देशों में न्यायपालिका का एक निश्चित अवधि के लिए विधानपालिका द्वारा चुनाव किया जाता है। इसके विपरीत कुछ देशों में न्यायपालिका की कार्यपालिका द्वारा नियुक्ति की जाती है और नियुक्ति का आधार न्यायाधीशों की योग्यताएँ होती हैं। न्यायाधीशों की अवधि भी भिन्न-भिन्न देशों में भिन्न-भिन्न होती है; जैसे भारत में न्यायाधीश 65 वर्ष तक अपने पद पर रहते हैं।

प्रश्न 3.
स्वतन्त्र तथा निष्पक्ष न्यायालय से क्या अभिप्राय है? संक्षेप में व्याख्या करें।
उत्तर:
ब्राइस का यह कथन सत्य है कि किसी भी सरकार की श्रेष्ठता की सबसे अच्छी कसौटी न्यायपालिका की दक्षता है। न्यायपालिका की दक्षता उसकी निष्पक्षता तथा स्वतन्त्रता से है। न्यायपालिका की स्वतन्त्रता का अर्थ है कि न्यायपालिका विधानमण्डल तथा कार्यपालिका से स्वतन्त्र हो। उस पर मतदाताओं तथा राजनीतिक दलों का नियन्त्रण और प्रभाव न हो।

न्यायाधीश पक्षपात तथा बाहरी दबाव के आधार पर निर्णय न करें। न्यायाधीशों के निर्णयों पर किसी प्रकार का प्रशासनिक, आर्थिक तथा राजनीतिक दबाव नहीं होना चाहिए। ऐसा तभी हो सकता है जब न्यायाधीश पूर्णरूप से स्वतन्त्र हों। वे बिना किसी भय, लालच . तथा पक्षपात के पूर्ण न्याय करने वाले हों। न्यायाधीश योग्य, ईमानदार, निष्पक्ष तथा स्वतन्त्र हों। न्याय में देरी न हो। न्याय में देरी का अभिप्राय है-न्याय से इन्कार करना।।

प्रश्न 4.
न्यायपालिका की स्वतन्त्रता में सहायक चार तत्त्वों का वर्णन करें।
उत्तर:
न्यायपालिका की स्वतन्त्रता में निम्नलिखित चार तत्त्व सहायक हैं
1. पद की अवधि-न्यायाधीशों को प्रलोभन तथा पक्षपात से दूर रखने के लिए न्यायाधीशों का कार्यकाल लम्बा रखा जाए। भारत में सर्वोच्च न्यायालय के न्यायाधीश तथा उच्च न्यायालयों के न्यायाधीश 65 वर्ष तक अपने पद पर रहते हैं।

2. पद से हटना-न्यायाधीशों को मनमाने ढंग से न हटाया जा सके। पूरी छानबीन के पश्चात् ही एक विशेष विधि द्वारा उन्हें उनके पद से हटाया जाना चाहिए।

3. अच्छा वेतन-न्यायाधीशों को अच्छा वेतन मिलना चाहिए, जिससे वे अपने समकक्ष दूसरे अधिकारियों की तरह आर्थिक स्तर स्थापित कर सकें।

4. योग्यता के आधार पर नियुक्ति-न्यायाधीशों की नियुक्ति करते समय उनके ज्ञान, बुद्धि तथा योग्यता को ध्यान में रखना चाहिए।

प्रश्न 5.
शक्तियों के पृथक्करण के सिद्धान्त की संक्षेप में व्याख्या करें।
उत्तर:
किसी देश में कैसी भी शासन व्यवस्था की स्थापना क्यों न की गई हो, प्रत्येक सरकार के तीन. कार्य होते हैं; जैसे कानून बनाना, कानूनों को लागू करना तथा न्याय करना। विधानपालिका कानून बनाती है, कार्यपालिका उन्हें लागू करती है तथा न्यायपालिका न्याय करती है। यदि सरकार की तीनों शक्तियों को अलग-अलग रखा जाए तो इस सिद्धान्त को शक्तियों के पृथक्करण का सिद्धान्त कहा जाता है। इस सिद्धान्त के मुख्य समर्थक मोण्टेस्क्यू का हना है कि सरकार की तीनों नहीं, दो शक्तियाँ भी एक व्यक्ति के हाथ में नहीं होनी चाहिएँ। व्यक्तियों के अधिकार और स्वतन्त्रता की रक्षा तथा सरकार को निरंकुश होने से इस सिद्धान्त द्वारा ही रोका जा सकता है।

HBSE 11th Class Political Science Important Questions Chapter 6 न्यायपालिका

प्रश्न 6.
शक्तियों के पृथक्करण के सिद्धान्त की आलोचना के चार आधार बताएँ।
उत्तर:
शक्तियों के पृथक्करण के सिद्धान्त की आलोचना के आधार इस प्रकार हैं

  • सरकार एक इकाई है तथा शरीर के अंगों के अनुसार कार्य करती है, अतः शक्तियों का पूर्ण पृथक्करण सम्भव नहीं है,
  • यह सिद्धान्त व्यावहारिक नहीं है, क्योंकि शासन के सभी अंग एक-दूसरे पर निर्भर करते हैं,
  • शक्तियों का पूर्ण पृथक्करण इसलिए भी सम्भव नहीं है, क्योंकि सरकार के तीनों अंग समान नहीं हैं,
  • मोण्टेस्क्यू का यह कथन उचित नहीं है कि व्यक्तिगत स्वतन्त्रता तथा अधिकार के लिए शक्तियों का पृथक्करण अनिवार्य है।

प्रश्न 7.
न्यायिक पुनर्निरीक्षण का क्या अर्थ है?
उत्तर:
न्यायालय का यह वह अधिकार है जिसके प्रयोग से वह संसद अथवा राज्य विधानमण्डलों द्वारा बनाए गए कानूनों तथा कार्यपालिका द्वारा जारी किए गए आदेशों की संवैधानिकता के बारे में निर्णय करता है और यदि वे संविधान के उल्लंघन में होते हैं तो उसे असंवैधानिक घोषित करके रद्द करने का अधिकार रखता है। भारत में सर्वोच्च न्यायालय तथा उच्च न्यायालयों को यह अधिकार प्राप्त है। सर्वोच्च न्यायालय ने ‘प्रिवी पर्स’ (Privy Purses) तथा बैंकों के राष्ट्रीयकरण (Nationalization of Banks) के आदेशों को इसी शक्ति के आधार पर अवैध घोषित कर दिया गया था।

प्रश्न 8.
न्यायिक पुनर्निरीक्षण के किन्हीं दो गुणों की व्याख्या कीजिए।
उत्तर:
न्यायिक पुनर्निरीक्षण के गुण इस प्रकार हैं-
(1) भारत का संविधान लिखित है। लिखित संविधान की भाषा अथवा शब्दावली कहीं-कहीं अस्पष्ट तथा उलझी हुई हो सकती है। उसकी व्याख्या तथा स्पष्टीकरण करने के लिए सर्वोच्च न्यायालय का यह अधिकार बहुत ही महत्त्वपूर्ण है। संघीय शासन-प्रणाली में केन्द्रीय सरकार तथा राज्य सरकारों के बीच शक्तियों का विभाजन होता है। उनमें उत्पन्न होने वाले विवादों को निपटाने के लिए भी सर्वोच्च न्यायालय का यह अधिकार महत्त्वपूर्ण है,

(2) नागरिकों के अधिकारों तथा स्वतन्त्रताओं की रक्षा के लिए भी न्यायालयों का यह अधिकार आवश्यक है, परन्तु यह बात भी ठीक है कि ये अधिकार असीमित नहीं हैं और राज्य की सुरक्षा अथवा सार्वजनिक हित को ध्यान में रखते हुए उन पर प्रतिबन्ध लगाए जा सकते हैं, परन्तु प्रतिबन्ध उचित है अथवा अनुचित, इसकी जाँच भी न्यायालय ही कर सकते हैं।

प्रश्न 9.
न्यायपालिका तथा विधायिका के सम्बन्धों का संक्षिप्त विवेचन कीजिए।
उत्तर:
न्यायपालिका तथा विधायिका के सम्बन्ध इस प्रकार हैं-
(1) विश्व के कई देशों में न्यायपालिका को विधायिका द्वारा बनाए गए कानूनों को रद्द करने का अधिकार है, यदि वे संविधान के उल्लंघन में बनाए गए हैं। भारत तथा अमेरिका में सर्वोच्च न्यायालयों को यह अधिकार दिया गया है,

(2) कई राज्यों में विधायिका न्यायपालिका के कई कार्यों का निष्पादन करती है। इंग्लैण्ड में संसद का उच्च सदन (हाउस ऑफ लॉर्ड्स) सर्वोच्च अपीलीय न्यायालय के रूप में कार्य करता है। अमेरिका में सीनेट निचले सदन (प्रतिनिधि सदन) द्वारा दोषी बनाए गए कार्यपालिका के कर्मचारियों की जाँच की सुनवाई करती है,

(3) कई राज्यों (भारत) में विधायिका को न्यायाधीशों के विरुद्ध महाभियोग (Impeachment) का प्रस्ताव पास करने का भी अधिकार है।

प्रश्न 10.
न्यायपालिका द्वारा नागरिकों के अधिकारों की रक्षा कैसे की जाती है?
उत्तर:
वर्तमान प्रजातन्त्रीय राज्यों में नागरिकों के विकास के लिए भिन्न-भिन्न प्रकार के अधिकार तथा स्वतन्त्रताएँ प्रदान की जाती हैं। न्यायपालिका नागरिकों के इन अधिकारों और स्वतन्त्रताओं की रक्षा करती है। भारतवर्ष में नागरिकों को जो मौलिक अधिकार दिए गए हैं उनकी रक्षा की जिम्मेदारी राज्यों के उच्च न्यायालय और सर्वोच्च न्यायालय को सौंपी गई है।

ये न्यायालय लेख (Writs) जारी करके नागरिकों के मौलिक अधिकारों की रक्षा करते हैं। इसके साथ-ही-साथ यदि विधानपालिका कोई ऐसा कानून बनाती है जोकि मौलिक अधिकारों के विरुद्ध है तो ये न्यायालय उसे अवैध घोषित कर सकते हैं। इस प्रकार न्यायपालिका नागरिकों के अधिकारों तथा स्वतन्त्रताओं की रक्षा करके विकास के अवसर प्रदान करती है।

प्रश्न 11.
न्यायपालिका संघीय ढाँचे के संरक्षक के रूप में कैसे कार्य करती है?
उत्तर:
जिस देश में संघीय शासन-प्रणाली की व्यवस्था की गई है वहाँ न्यायपालिका संविधान के संरक्षक के रूप में भी कार्य करती है। ऐसा शासन-प्रणाली में संविधान देश का सर्वोच्च कानून होता है और उसके द्वारा ही संघीय सरकार तथा राज्य सरकारों के अधिकारों व कार्य-क्षेत्र को निश्चित किया जाता है। जब कभी केन्द्र तथा राज्यों के बीच कोई झगड़ा उत्पन्न हो जाता है, तो न्यायालय ही संविधान की सही व्याख्या कर उसका निर्णय करता है।

यदि संघीय अथवा राज्य विधानमण्डल द्वारा कोई ऐसा कानून पास किया जाता है, जो संविधान का उल्लंघन करता है, तो न्यायपालिका को उसे असंवैधानिक घोषित करके रद्द करने का अधिकार दिया जाता है। भारत तथा अमेरिका के सर्वोच्च न्यायालयों को संविधान के संरक्षक के रूप में कार्य करने का अधिकार दिया गया है।

प्रश्न 12.
सर्वोच्च न्यायालय के न्यायाधीश के पद पर नियुक्त होने के लिए अनिवार्य योग्यताएँ क्या हैं?
उत्तर:
सर्वोच्च न्यायालय के न्यायाधीश के पद की नियुक्ति के लिए अनिवार्य योग्यताएँ दी गई हैं-

  • वह भारत का नागरिक हो,
  • वह कम-से-कम पाँच वर्षों तक एक या एक से अधिक उच्च न्यायालयों में न्यायाधीश के पद पर कार्य कर चुका हो। अथवा वह कम-से-कम दस वर्ष तक किसी उच्च न्यायालय में वकील रह चुका हो,
  • वह राष्ट्रपति की दृष्टि में प्रसिद्ध विधि-विशेषज्ञ हो।

प्रश्न 13.
उच्चतम न्यायालय का संगठन क्या है? इसके न्यायाधीशों की नियुक्ति कौन करता है?
उत्तर:
उच्चतम न्यायालय में एक मुख्य न्यायाधीश और 33 अन्य न्यायाधीश सहित कुल 34 न्यायाधीश हैं। उच्चतम न्यायालय के मुख्य न्यायाधीश और अन्य न्यायाधीशों की नियुक्ति राष्ट्रपति करता है। मुख्य न्यायाधीश की नियुक्ति के लिए राष्ट्रपति उच्चतम न्यायालय के अन्य न्यायाधीशों व उच्च न्यायालय के उन न्यायाधीशों से सलाह करता है जिन्हें वह उचित समझता है। उच्चतम न्यायालय के अन्य न्यायाधीशों की नियुक्ति में राष्ट्रपति मुख्य न्यायाधीश की सलाह लेता है।

HBSE 11th Class Political Science Important Questions Chapter 6 न्यायपालिका

प्रश्न 14.
उच्चतम न्यायालय के न्यायाधीशों के वेतन तथा भत्तों पर संक्षिप्त लेख लिखें। .
उत्तर:
उच्चतम न्यायालय के मुख्य न्यायाधीश को ₹ 2,80,000 मासिक वेतन मिलता है जबकि अन्य न्यायाधीशों को ₹ 2,50,000 मासिक वेतन मिलता है। इसके अतिरिक्त बिना किराए के मकान व अन्य सुविधाएँ मिलती हैं। न्यायाधीशों को वेतन संचित निधि से दिया जाता है जिस पर संसद की स्वीकृति मात्र औपचारिकता ही होती है। संसद न्यायाधीशों के कार्यकाल के दौरान उनके वेतन व भत्तों में कटौती नही कर सकती, हाँ, बढ़ा अवश्य सकती है। केवल वित्तीय संकट में उनके वेतन व भत्तों में कटौती की जा सकती है।

प्रश्न 15.
सर्वोच्च न्यायालय मौलिक अधिकारों की रक्षा कैसे करता है?
उत्तर:
यदि सरकार के किसी कार्य से नागरिकों के मौलिक अधिकारों का उल्लंघन होता है तो कोई भी उच्चतम न्यायालय में अपील कर सकता है। उच्चतम न्यायालय उस कार्य को अवैध घोषित कर सकता है। इसी प्रकार से यदि संसद ऐसा कोई कानून बनाती है कि जिससे मौलिक अधिकार कम होते हो अथवा उनका हनन होता हो तो उच्चतम न्यायालय ऐसे कानूनों को भी अवैध घोषित कर सकता है। उच्चतम न्यायालय ने बैंक राष्ट्रीयकरण मामला, मेनका गाँधी मामला इत्यादि में मौलिक अधिकारों की रक्षा की है।

प्रश्न 16.
उच्चतम न्यायालय तथा उच्च न्यायालयों के न्यायाधीशों को पद से कैसे हटाया जा सकता है?
उत्तर:
उच्चतम न्यायालय तथा उच्च न्यायालय के न्यायाधीशों को उनका निश्चित कार्यकाल समाप्त होने से पूर्व महाभियोग द्वारा हटाया जा सकता है। इसके लिए पहले न्यायाधीश का दुर्व्यवहार अथवा असमर्थता का आरोप एक निष्पक्ष जाँच-पड़ताल द्वारा सिद्ध हो । उसके बाद संसद के दोनों सदन अलग-अलग रूप से सदन की कुल संख्या के बहुमत अथवा उपस्थित व मतदान में भाग लेने वाले सदस्यों की संख्या के दो-तिहाई (2/3) बहुमत से प्रस्ताव पारित कर देते हैं, तो वह प्रस्ताव राष्ट्रपति के पास भेज दिया जाता है। राष्ट्रपति प्रस्ताव को स्वीकृति देकर न्यायाधीश को पद से हटा देता है।

प्रश्न 17.
उच्चतम न्यायालय की संविधान के संरक्षक की भूमिका पर संक्षिप्त नोट लिखें।
उत्तर:
भारतीय संविधान उच्चतम न्यायालय को संविधान की व्याख्या तथा न्यायिक पुनरावलोकन का अधिकार प्रदान करता है, जिसके अन्तर्गत उच्चतम न्यायालय कार्यपालिका तथा विधायिका के उन सभी कार्यों व कानूनों को अवैध घोषित करता है जो संविधान का उल्लंघन करते हों। भारतीय उच्चतम न्यायालय ने संविधान की रक्षा करने के उद्देश्य से संवैधानिक संशोधनों को भी अवैध घोषित किया है और अनेक मामलों में कहा है, “संसद ऐसा कोई संशोधन नहीं कर सकती जिससे संविधान का मूलभूत ढाँचा विकृत होता हो अथवा नष्ट होता हो।”

प्रश्न 18.
न्यायिक पुनरावलोकन से आप क्या समझते हैं?
उत्तर:
न्यायिक पुनरावलोकन का अर्थ न्यायपालिका के उस अधिकार से है, जिसके अन्तर्गत न्यायपालिका संविधान के आधार पर कार्यपालिका व व्यवस्थापिका के उन कार्यों व कानूनों को अवैध घोषित कर सकती है जो कि न्यायालय के विचार में संविधान अथवा संविधान में निहित व्यवस्था का उल्लंघन करते हों। भारतीय संविधान के अनेक प्रावधान जैसे अनुच्छेद 13 न्यायपालिका को न्यायिक पनरावलोकन का अधिकार प्रदान करते हैं। इसी प्रकार से न्यायपालिका न्यायिक पनरावलोकन के अधिकार के अन्तर्गत संघीय व राज्य सरकारों के ऐसे कानूनों को भी अवैध घोषित करती है जिनमें इन सरकारों ने अपने क्षेत्राधिकार का उल्लंघन किया हो। इस प्रकार न्यायिक पुनरावलोकन का उद्देश्य संघात्मक व्यवस्था तथा संवैधानिक व्यवस्था व मूल्यों की रक्षा करना है।

प्रश्न 19.
उच्च न्यायालयों के न्यायाधीशों के वेतन व भत्तों पर संक्षिप्त टिप्पणी करें।
उत्तर:
उच्च न्यायालय के मुख्य न्यायाधीश को ₹ 2,50,000 मासिक वेतन मिलता है। उच्च न्यायालयों के अन्य न्यायाधीशों को ₹ 2,25,000 मासिक वेतन मिलता है। वेतन के अतिरिक्त न्यायाधीशों को अनेक भत्ते व सुविधाएँ भी दी जाती हैं। न्यायाधीशों के वेतन व भत्ते संचित निधि से दिए जाते हैं। संसद न्यायाधीशों के कार्यकाल के दौरान उनके वेतन व भत्तों में कमी नहीं कर सकती, हाँ, बढ़ा अवश्य सकती है। केवल वित्तीय संकट में ही उनके (न्यायाधीशों के) वेतन व भत्तों में कटौती कर सकती है।

प्रश्न 20.
उच्च न्यायालय के न्यायाधीशों के लिए कौन-सी योग्यताएँ होना आवश्यक है?
उत्तर:
भारतीय संविधान उच्च न्यायालय के न्यायाधीशों के लिए दी गई योग्यताएँ निर्धारित करता है-

  • वह भारत का नागरिक हो,
  • वह भारत में कम-से-कम 10 वर्षों तक किसी न्यायिक पद पर रह चुका हो अथवा किसी भी राज्य के उच्च न्यायालय या एक से अधिक राज्यों के उच्च न्यायालयों में कम-से-कम 10 वर्षों तक वकील रह चुका हो।

प्रश्न 21.
न्यायपालिका मौलिक अधिकारों की रक्षा कैसे करती है?
उत्तर:
मौलिक अधिकारों की रक्षा करना न्यायपालिका का अति महत्त्वपूर्ण कार्य है। यदि कोई व्यक्ति या संस्था यह समझे कि सरकार ने उसके या किसी और व्यक्ति के मौलिक अधिकारों का उल्लंघन किया है, तो वह अनुच्छेद 32 के अधीन उच्चतम न्यायालय में अथवा अनुच्छेद 226 के अधीन उच्च न्यायालय में अपील कर सकता है। न्यायपालिका मौलिक अधिकारों की रक्षा के लिए कई प्रकार के लेख; जैसे बन्दी प्रत्यक्षीकरण, परमादेश इत्यादि जारी कर सकती है और किसी भी कार्य तथा कानून को अवैध घोषित कर सकती है।

प्रश्न 22.
संविधान किस प्रकार से न्यायपालिका की स्वतन्त्रता की स्थापना करता है ?
उत्तर:
संविधान दिए गए उपायों व प्रावधानों से न्यायपालिका की स्वतन्त्रता सुनिश्चित करता है-

  • न्यायाधीशों की नियुक्ति में उच्चतम न्यायालय के मुख्य न्यायाधीश से सलाह ली जाती है,
  • न्यायाधीशों की नियुक्ति लम्बी अवधि के लिए की जाती है,
  • कार्यकाल के दौरान न्यायाधीशों के वेतन व भत्तों में कटौती नहीं की जा सकती,
  • न्यायाधीशों को समय से पूर्व केवल महाभियोग द्वारा ही हटाया जा सकता है,
  • न्यायाधीशों को संविधान में अनेक उन्मुक्तियाँ प्रदान की गई हैं।

प्रश्न 23.
उच्च न्यायालय की प्रशासन संबंधी शक्तियों का वर्णन कीजिये।
उत्तर:
उच्च न्यायालय को न्याय संबंधी अधिकारों के अतिरिक्त कुछ प्रशासन संबंधी अधिकार भी प्राप्त हैं, जो इस प्रकार हैं

  • उच्च न्यायालय अपने अधीन किसी भी न्यायालय के कागजों को मँगवाकर उनकी जाँच-पड़ताल कर सकता है,
  • उच्च न्यायालय किसी मुकद्दमे को एक न्यायालय से हटाकर निर्णय के लिए दूसरे न्यायालय में भेज सकता है,
  • यदि किसी न्यायालय में ऐसा मुकद्दमा चल रहा हो जिसमें भारतीय संविधान की व्याख्या का प्रश्न पैदा होता है, तो उच्च न्यायालय उस मुकद्दमे को अपने पास मँगवाकर निर्णय दे सकता है,
  • उच्च न्यायालय अपने अधीन न्यायालयों के शैरिफ, क्लर्क, वकील तथा अन्य कर्मचारियों की फीस निश्चित करता है। इसके अलावा उच्च न्यायालय अपने अधीन न्यायालयों के अधिकारियों की नियुक्ति, अवनति, उन्नति और छुट्टी के बारे में नियम बनाता है,
  • उच्च न्यायालय अपने अधीन न्यायालयों की कार्य-पद्धति, रिकॉर्ड, रजिस्टर बना सकता है तथा हिसाब-किताब देख सकताहै,
  • उच्च न्यायालय को राज्य के अन्य सभी न्यायालयों तथा न्यायाधिकरणों की देख-रेख करने तथा उन पर निरीक्षण रखने का अधिकार प्राप्त है।

प्रश्न 24.
न्यायिक सक्रियता के कोई चार कारण लिखें।
उत्तर:
न्यायिक सक्रियता के मुख्य चार कारण निम्नलिखित हैं
1. सरकार व देश में फैला व्यापक भ्रष्टाचार-वर्तमान समय में कुछ ही ऐसे नेता हैं जिन पर भ्रष्टाचार का आरोप न लगा हो। भ्रष्टाचार अपनी चरम सीमा पर पहुंच चुका है। न्यायालय भ्रष्ट लोगों का पर्दाफाश करने में लगा हुआ है।

2. संविधान के नियमों का उल्लंघन सरकार द्वारा राजनीतिक दलों द्वारा जब भी संविधान के नियमों की अवहेलना होती है तब न्यायपालिका ही हस्तक्षेप करके नियमों की या संविधान की रक्षा करती है।

3. लालसा-राजनेता जब सत्ता का स्वाद चख लेते हैं तो उनमें सत्ता में बने रहने की लालसा भ्रष्टाचार को जन्म देती है। ऐसी स्थिति में न्यायपालिका का सक्रिय होना स्वाभाविक है।

4. विधानमण्डलों में अनुशासनहीनता-विधानमण्डलों में अनुशासनहीनता राजनेताओं का दैनिक कार्य बन चुका है। ऐसी स्थिति से निपटने के लिए न्यायपालिका की सक्रियता के इलावा और कोई चारा नहीं है।

HBSE 11th Class Political Science Important Questions Chapter 6 न्यायपालिका

प्रश्न 25.
न्याय सक्रियता के चार साधनों का वर्णन करो।
उत्तर:
1. न्यायिक पुनर्निरीक्षण न्यायिक पुनर्निरीक्षण न्यायिक सक्रियता का पहला एवं महत्त्वपूर्ण साधन है। इस प्रकार की शक्ति भारत में सर्वोच्च न्यायालय के पास है।।

2. मौलिक अधिकारों की व्याख्या-न्यायिक सक्रियता का दूसरा साधन मौलिक अधिकारों की व्याख्या से सम्बन्धित है। सर्वोच्च न्यायालय भारतीय नागरिकों के मौलिक अधिकारों की रक्षा करता है।

3. संविधान की व्याख्या संविधान एक पवित्र पुस्तक मात्र है जो अपनी सुरक्षा नहीं कर सकता। न्यायपालिका का उत्तरदायित्व है कि संविधान के विरुद्ध बने कानूनों को गैर-कानूनी घोषित कर उसकी सुरक्षा करे।

4. कानून निर्माण-न्यायपालिका का कार्य कानून निर्माण करना नहीं है फिर भी कानून की नई व्याख्या कानून निर्माण का कार्य करती है। यह न्यायिक सक्रियता का स्पष्ट उदाहरण है।

प्रश्न 26.
सर्वोच्च न्यायालय के प्रांरभिक क्षेत्राधिकार का वर्णन कीजिए।
उत्तर:
सर्वोच्च न्यायालय के इस अधिकार क्षेत्र में वे मुकद्दमें आते हैं जो सीधे सर्वोच्च न्यायलय में ले जाए जा सकते हैं और जिनका सम्बन्ध संघीय व्यवस्था से है। ये इस प्रकार के हैं।

  • संघ सरकार तथा एक अथवा अधिक राज्यों के बीच झगड़े।
  • ऐसे झगड़े जिनमें संघीय सरकार तथा कुछ राज्य एक तरफ हों और कुछ अन्य राज्य तरफ हों।
  • राज्यों के आपसी झगड़े जिनका सम्बन्ध कानून अथवा संविधान की व्याख्या से हो।

प्रश्न 27.
न्यायिक क्रियाशीलता से क्या तात्पर्य है।
उत्तर:
न्यायिक सक्रियवाद का अर्थ यह है कि न्यायपालिका द्वारा उन कार्यों को करना जो उसके अधिकार क्षेत्र में नहीं आते। इसे इस तरह भी कहा जा सकता है कि जो कार्य विधानपालिका या कार्यपालिका के क्षेत्र में आते हैं और वे न्यायपालिका द्वारा किए जाते हैं। न्यायपालिका अन्य सरकारी सदस्यों को कुछ कार्य करने के निर्देश देती है और उनका पालन करना अनिवार्य होता है। इस प्रकार न्यायिक सक्रियावाद का अभिप्राय न्यायपालिका द्वारा अपनी शक्तियों के क्षेत्र में क्रियाशील भूमिका निभाना नहीं है, बल्कि अपने क्षेत्र से बाहर होकर ऐसे काम करना है जो सरकार के अन्य अंगों के निर्धारित अधिकार क्षेत्र में होते हैं।

निबंधात्मक प्रश्न

प्रश्न 1.
आधुनिक लोकतन्त्रीय राज्यों में न्यायपालिका के कार्यों तथा शक्तियों की व्याख्या करें।
उत्तर:
न्यायपालिका सरकार का तीसरा तथा महत्त्वपूर्ण अंग है। यद्यपि यह विभाग उतना चर्चित नहीं जितने अन्य दोनों विभाग, फिर भी इसका महत्त्व किसी भी प्रकार अन्य विभागों से कम नहीं है। न्यायपालिका ही लोगों के अधिकारों तथा उनकी स्वतन्त्रताओं की रक्षा करती है तथा उन्हें अधिकारियों की निरंकुशता से बचाती है।

1. गार्नर (Garmer) के अनुसार, “बिना न्याय विभाग के किसी सभ्य राज्य की कल्पना नहीं की जा सकती है।” 2. लॉर्ड ब्राईस (Lord Bryce) का कथन है, “सरकार की उत्तमता की कसौटी उसकी न्यायपालिका की दक्षता है।” जिस राज्य के लोगों को निष्पक्ष तथा तुरन्त न्याय नहीं मिलता, वहाँ लोग सुखी नहीं रह सकते। इसलिए एक स्वतन्त्र तथा निष्पक्ष न्यायपालिका का होना अति अनिवार्य है।

न्यायपालिका के कार्य (Functions of Judiciary)- न्यायालय का मुख्य कार्य लोगों में न्याय वितरित करना है। यद्यपि न्यायपालिका का मुख्य कार्य न्याय करना है तथा सार्वजनिक कानूनों को व्यक्तिगत मामलों में लागू करना है, परन्तु न्याय करने के अतिरिक्त भी न्यायपालिका को बहुत-से कार्य करने पड़ते हैं। न्यायपालिका के मुख्य कार्य निम्नलिखित हैं

1. अपराधियों को दण्ड देना-न्यायपालिका का कार्य अपराधियों को दण्ड देना है जिससे दूसरे व्यक्ति अपराध न करें। जब कोई व्यक्ति राज्य के कानूनों का उल्लंघन करता है तो न्यायालय उसे उपयुक्त दण्ड देता है।

2. विवादों का निर्णय करना-न्यायालय लोगों के आपसी दीवानी, फौजदारी तथा राजस्व सम्बन्धी विवादों का फैसला करता है। यदि निम्न न्यायालयों के निर्णय से कोई पक्ष असन्तुष्ट रहे तो उसे उच्च न्यायालय में अपील करने का अधिकार है। सर्वोच्च न्यायालय का फैसला दोनों पक्षों को मानना पड़ता है।

3. नागरिकों के अधिकारों की रक्षा करना-राज्य व्यक्ति को अपने विकास के लिए बहुत-से अधिकार प्रदान करता है। जिन देशों के संविधान लिखित होते हैं, वहाँ नागरिकों को संविधान द्वारा ही बहुत-से अधिकार प्रदान किए जाते हैं। नागरिकों के सभी अधिकारों की रक्षा करना न्यायपालिका का कार्य है। यदि न्यायपालिका द्वारा नागरिकों के अधिकारों की रक्षा न की जाए तो अधिकार अर्थहीन बन जाते हैं।

4. कानून को व्यक्तिगत मामलों में लागू करना राज्य अलग-अलग लोगों तथा वर्गों के लिए अलग-अलग कानून नहीं बनाता। कानून सभी के लिए समान रूप से बनाए जाते हैं। न्यायपालिका सार्वजनिक कानूनों को व्यक्तिगत मामलों में लागू करते हैं।

5. कानूनों की व्याख्या विधानमण्डल द्वारा बनाए गए कानून सामान्य होते हैं। इनकी विस्तार सहित व्याख्या करना न्यायपालिका का कार्य है। विभिन्न पक्ष अपने-अपने ढंग से कानून का अर्थ लगाते हैं। ऐसे मामलों में न्यायालय ही उसकी सही व्याख्या करता है। न्यायालय की व्याख्या अन्तिम होती है, जिसे सबको मानना पड़ता है।

6. नए कानूनों का निर्माण-जब न्यायालय के सामने कोई ऐसा विवाद आता है, जिसके सम्बन्ध में कोई कानून नहीं है तो न्यायाधीश अपनी योग्यता व निष्पक्षता के आधार पर न्याय करते हैं। उनका यह निर्णय कानून बन जाता है। भविष्य में उसी के आधार पर न्याय किया जाता है। ऐसे कानूनों को न्यायाधीश द्वारा निर्मित कानून (Judge made Laws) कहा जाता है।

7. संविधान की रक्षा-न्यायालय देश के संविधान का संरक्षक होता है। यदि कार्यपालिका अथवा विधानपालिका द्वारा संविधान की धाराओं का उल्लंघन किया जाता है तो न्यायालय उनके कार्यों को अवैध घोषित करके निरस्त कर सकता है। संविधान की रक्षा करना न्यायालय का ही कार्य है। वह केन्द्र तथा राज्य सरकारों के संविधान विरोधी कार्यों को निरस्त कर देता है। इस प्रकार संविधान की सुरक्षा बनी रहती है।

8. परामर्श सम्बन्धी कार्य आवश्यकता पड़ने पर न्यायपालिका राज्याध्यक्ष को कानूनी सलाह भी देता है। भारत का राष्ट्रपति सर्वोच्च न्यायालय के न्यायाधीशों से किसी भी मामले में कानूनी सलाह ले सकता है। न्यायालय के परामर्श को मानना अथवा न मानना राष्ट्रपति का कार्य है।

9. विविध कार्य-न्यायपालिका अन्य कई विविध कार्य भी करती है। आन्तरिक प्रबन्ध करना, अपने रिकॉर्ड सुरक्षित रखना, विविध प्रकार की आज्ञाएँ जारी करके अधिकारियों के किसी अनुचित कार्य को रोकना न्यायालय का कार्य है। नाबालिगों की सम्पत्ति के लिए ट्रस्टी नियुक्त करना, स्त्रियों तथा पागलों के लिए संरक्षक नियुक्त करना, व्यक्तियों की सम्पत्ति का प्रबन्ध करना, दिवालियों की सम्पत्ति के लिए रिसीवर नियुक्त करना भी न्यायालय के कार्य हैं। इनके अतिरिक्त विवाह तथा तलाक को मान्यता देना, विदेशियों को नागरिकता प्रदान करना आदि भी न्यायालय के अधिकार क्षेत्र में आते हैं। इस तरह न्यायपालिका का काम न्याय करना ही नहीं, अन्य और भी बहुत से कार्य करना है।

प्रश्न 2.
न्यायपालिका की स्वतन्त्रता का महत्त्व बताइए। न्यायपालिका को निष्पक्ष या स्वतन्त्र कैसे रखा जा सकता है ?
उत्तर:
न्यायपालिका की स्वतन्त्रता का अभिप्राय है कि वह विधानमण्डल तथा कार्यपालिका के नियन्त्रण से मुक्त हो। उस पर मतदाताओं तथा राजनीतिक दलों का प्रभाव न हो। न्यायाधीश पक्षपातपूर्ण तथा किसी तरह के बाहरी दबाव में आकर न्याय न करें। न्यायाधीशों के निर्णयों पर किसी प्रकार का प्रशासनिक, आर्थिक, राजनीतिक प्रभाव नहीं होना चाहिए। ऐसा तभी हो सकता है जब न्यायाधीश पूर्णरूप से स्वतन्त्र हों। वे बिना किसी भय, लालच अथवा पक्षपात के पूर्ण न्याय करने वाले हों।

स्वतन्त्र न्यायपालिका का महत्त्व स्पष्ट है। देश के कानून चाहे कितने ही अच्छे क्यों न हों, वास्तविक न्याय तब तक नहीं हो सकता, जब तक न्यायाधीश योग्य, ईमानदार, निष्पक्ष तथा स्वतन्त्र न हों। यदि न्याय करने में देरी लगती है या भेदभाव रखा जाता है तो लोगों के जान व माल की रक्षा सम्भव नहीं। स्वतन्त्र न्यायपालिका ही जनता के अधिकारों तथा स्वतन्त्रताओं की संरक्षक है।

यदि न्याय रूपी दीपक बुझ जाता है तो कितना गहन अन्धकार होगा, इसकी कल्पना भी नहीं की जा सकती। – प्रजातन्त्र की सफलता भी इस बात पर निर्भर है, इसलिए लॉर्ड ब्राइस का यह कथन पूर्णतः सत्य है कि किसी भी सरकार की श्रेष्ठता की सबसे अच्छी कसौटी न्यायपालिका की दक्षता है। एक स्वतन्त्र तथा निष्पक्ष न्याय व्यवस्था संगठित करने के लिए निम्नलिखित बातों का ध्यान रखना चाहिए

1. न्यायाधीशों की नियुक्ति-न्यायपालिका की स्वतन्त्रता के लिए न्यायाधीशों का उचित ढंग से नियुक्त किया जाना बहुत अनिवार्य है।
न्यायाधीशों की नियुक्ति के निम्नलिखित तीन तरीके प्रचलित हैं

(1) जनता द्वारा चुनाव-जनता द्वारा चुनाव करने की प्रथा रूस तथा अमेरिका में निम्नतर न्यायालयों के न्यायाधीशों की नियुक्ति करने में अपनाई जाती है, परन्तु यह पद्धति अत्यन्त दोषपूर्ण है। जनता यह नहीं जानती कि कौन व्यक्ति न्यायाधीश बनने के योग्य है। दूसरे, न्यायाधीश अपने मतदाताओं को खुश रखने का प्रयत्न करेंगे जिससे निष्पक्ष न्याय नहीं होगा। तीसरे, इस विधि से केवल राजनीतिज्ञ ही न्यायाधीश बन सकेंगे जो किसी भी रूप में उचित नहीं है।

(2) विधानमण्डल द्वारा चुनाव-विधानमण्डल द्वारा चुनाव की पद्धति रूस तथा स्विट्ज़रलैण्ड के उच्च न्यायालयों के लिए अपनाई जाती है। यह विधि भी ठीक नहीं है, क्योंकि विधायक योग्य नहीं होते। विधानसभा में बहुमत दल का प्रभाव होता है, इससे न्यायाधीश भी दलबन्दी में फंसकर कार्य करेंगे। वे न्याय करने की जगह अपने निर्वाचकों को खुश करने के प्रयत्न में ही लगे रहेंगे।

(3) कार्यपालिका द्वारा नियुक्ति कार्यपालिका द्वारा नियुक्ति सबसे अच्छी है। इसमें कार्यपालिका व्यक्ति की योग्यता के आधार पर नियुक्ति करेगी। भारत में सर्वोच्च न्यायालय द्वारा राज्यों के उच्च न्यायालयों के न्यायाधीशों की नियुक्ति राष्ट्रपति करता है। यह कार्य राष्ट्रपति मुख्य न्यायाधीश की सलाह पर करता है।

2. पद की अवधि-न्यायाधीशों को प्रलोभन तथा पक्षपात से दूर रखने के लिए उनका सेवाकाल भी लम्बा रखा जाना अनिवार्य है। यदि न्यायाधीशों का कार्यकाल थोड़ा होगा तो वे भविष्य के लिए रिश्वत लेने से भी नहीं हिचकेंगे, इसलिए न्यायाधीशों का कार्यकाल लम्बा होना चाहिए जिससे वे अपने पद के दौरान अपने कर्तव्यों को पूरा कर सकें। आज न्यायाधीश ‘सदाचार पर्यन्त पद’ (Good Behaviour Tenure) के सिद्धान्त पर एक लम्बे समय तक अपने पद पर बने रहते हैं। भारत में उच्चतम न्यायालय के न्यायाधीश तथा उच्च न्यायालयों के न्यायाधीश 65 वर्ष की आयु तक अपने पद पर बने रहते हैं।

3. न्यायाधीशों को पद से हटाने की विधि अयोग्य तथा दुराचारी व्यक्ति को न्याय के पद पर नहीं रहने देना चाहिए। यदि न्यायाधीश गलत काम करें तो उन्हें अपने पद से हटा देना चाहिए, परन्तु न्यायाधीशों का हटाया जाना इतना सरल तथा सुलभ न हो कि झूठा दोष लगाकर किसी भी न्यायाधीश को हटा दिया जाए। न्यायाधीशों को कार्यपालिका द्वारा विधानपालिका के दोषारोपण करने तथा उसकी जाँच-पड़ताल करने पर ही हटाया जाना चाहिए। भारत में संसद द्वारा महाभियोग सिद्ध होने पर ही राष्ट्रपति किसी न्यायाधीश को अपने पद से हटा सकता है, अन्यथा नहीं।

4. न्यायाधीशों का अच्छा वेतन-उचित न्याय के लिए न्यायाधीशों को प्रलोभनों से मुक्त रखा जाए। इसलिए उन्हें इतना वेतन अवश्य दिया जाए, जिससे वे अपने समकक्ष दूसरे अधिकारियों की तरह आर्थिक स्तर स्थापित कर सकें। उनको वेतन समय पर मिले तथा वेतन, भत्ते तथा अन्य सुविधाएँ उनके कार्यकाल में कम न किए जाएँ। ऐसा होने पर ही न्यायाधीशों से निष्पक्ष तथा ईमानदार होकर न्याय करने की अपेक्षा की जा सकती है।

5. योग्यता के आधार पर नियुक्ति न्यायाधीशों को नियुक्त करते समय उनके ज्ञान, बुद्धि तथा योग्यताएँ भी देखी जानी चाहिएँ। न्याय कार्य अन्य शासन कार्यों से भिन्न है। इसके लिए उन्हीं व्यक्तियों को नियुक्त किया जाना चाहिए जो कानून तथा न्याय सम्बन्धी पर्याप्त ज्ञान रखते हों। उन्हें देश के कानूनों तथा परम्पराओं का पर्याप्त ज्ञान हो।

6. सेवा निवृत्त होने पर वकालत की मनाही न्यायाधीशों के सम्मान तथा निष्पक्षता के लिए सेवानिवृत्त होने के बाद उन्हें वकालत करने का अधिकार नहीं होना चाहिए। यदि ऐसा होगा तो वर्तमान न्यायाधीश अपने पुराने साथियों के प्रति पक्षपातपूर्ण व्यवहार कर सकते हैं।

7. विधानपालिका तथा कार्यपालिका से पृथक्कता-न्यायपालिका को विधानपालिका तथा कार्यपालिका से पृथक् रखा जाना चाहिए अन्यथा न्यायाधीश स्वतन्त्र रूप से न्याय नहीं कर सकेंगे। एक स्वतन्त्र न्यायालय जनता के अधिकारों व स्वतन्त्रता की रक्षा का दुर्ग है। भारत में न्यायपालिका को विधानपालिका तथा कार्यपालिका से स्वतन्त्र रखा गया है। न्यायाधीश राष्ट्रपति द्वारा योग्यता के आधार पर नियुक्त किए जाते हैं तथा विधानपालिका ही महाभियोग द्वारा न्यायाधीशों को हटाने का प्रस्ताव पास कर सकती है।

तभी न्यायाधीश अपने पद से हटाए जा सकते हैं। निष्कर्ष-उपर्युक्त वर्णित तथ्यों के आधार पर हम यह कह सकते हैं कि इन विधियों को अपनाने से न्यायपालिका को स्वतन्त्र तथा निष्पक्ष रखा जा सकता है। इन विधियों को अपनाए बिना न्यायपालिका का स्वतन्त्र रूप से कार्य करना सम्भव नहीं है।

HBSE 11th Class Political Science Important Questions Chapter 6 न्यायपालिका

प्रश्न 3.
उच्चतम न्यायालय के संगठन एवं रचना का वर्णन करें।
उत्तर:
भारतीय संविधान एक एकीकृत न्यायपालिका की स्थापना करता है जिसमें शीर्ष पर उच्चतम न्यायालय (Supreme Court) है। इसके संगठन, शक्तियाँ तथा कार्य निम्नलिखित प्रकार से हैं रचना भारतीय संविधान के अनुच्छेद 124 के अन्तर्गत सर्वोच्च न्यायालय की व्यवस्था की गई है। सर्वोच्च न्यायालय में प्रारम्भ में एक मुख्य न्यायाधीश और 7 अन्य न्यायाधीश होते थे।

1957 में संसद में एक कानून पास हुआ, जिसके अनुसार मुख्य न्यायाधीश को छोड़कर बाकी न्यायाधीशों की संख्या 7 से 10 कर दी गई। सन् 1960 में संसद ने अन्य न्यायाधीशों की संख्या 10 से बढ़ाकर 13 कर दी, परन्तु दिसम्बर, 1977 में संसद ने एक कानून पास किया, जिसके अनुसार सर्वोच्च न्यायालय के अन्य न्यायाधीशों की अधिकतम संख्या 13 से बढ़ाकर 17 कर दी गई। अप्रैल, 1986 में सर्वोच्च न्यायालय के न्यायाधीशों की संख्या 17 से 25 कर दी गई।

21 फरवरी, 2008 को केन्द्रीय मन्त्रिमण्डल के निर्णयानुसार उच्चतम न्यायालय (न्यायाधीशों की संख्या) अधिनियम, 1956 में संशोधन कर न्यायाधीशों की संख्या को 25 से 30 कर दी गई। जबकि संसद द्वारा (लोकसभा द्वारा 5 अगस्त एवं राज्य सभा द्वारा 7 अगस्त) पारित सर्वोच्च न्यायालय (न्यायाधीशों की संख्या) संशोधन विधेयक, 2019 के उपरान्त 10 अगस्त, 2019 को राष्ट्रपति की स्वीकृति मिलने पर सर्वोच्च न्यायालय के न्यायाधीशों की अधिकतम संख्या मुख्य न्यायाधीश सहित 34 हो गई।

इस प्रकार अब एक मुख्य न्यायाधीश एवं 33 अन्य न्यायाधीशों के साथ कुल न्यायाधीशों की संख्या 34 हो गई है। वर्तमान में श्री शरद अरविंद बोबड़े सर्वोच्च न्यायालय के मुख्य न्यायाधीश हैं जिन्होंने श्री रंजन गोगोई के 17 नवम्बर, 2019 को सेवानिवृत होने के बाद 18 नवम्बर, 2019 को 47वें मुख्य न्यायाधीश के रूप में अपना कार्यभार सम्भाला जो 23 अप्रैल, 2021 तक अपने पद पर रहेंगे।

न्यायाधीशों की नियुक्ति-उच्चतम न्यायालय के न्यायाधीशों की नियुक्ति संविधान के अनुच्छेद 124 के अधीन राष्ट्रपति करता है। मुख्य न्यायाधीश की नियुक्ति में राष्ट्रपति उच्चतम न्यायालय के अन्य न्यायाधीशों व उच्च न्यायाधीशों, जिन्हें वह उचित समझता है, से सलाह करता है। मुख्य न्यायाधीश की नियुक्ति में वरिष्ठता के सिद्धान्त के पालन’ की परम्परा विकसित करने का प्रयास किया गया है।

हालाँकि दो बार इस परम्परा का राजनीतिक कारणों से उल्लंघन किया गया है। 1973 में श्री हेगड़े, श्री शैल व श्री ग्रोवर की वरिष्ठता की उपेक्षा करके श्री ए०एन० रे को मुख्य न्यायाधीश नियुक्त किया गया था। उच्चतम न्यायालय के अन्य न्यायाधीशों की नियुक्ति में राष्ट्रपति उच्चतम न्यायालय के मुख्य न्यायाधीश से सलाह करता है। हालाँकि न्यायाधीशों की सलाह मानने के लिए राष्ट्रपति बाध्य नहीं है।

न्यायाधीशों की योग्यताएँ संविधान द्वारा उच्चतम न्यायालय का न्यायाधीश बनने के लिए निश्चित की गई योग्यताएँ इस प्रकार हैं-

  • वह भारत का नागरिक हो,
  • वह किसी एक या दो अथवा दो से अधिक उच्च न्यायालयों का निरन्तर 5 वर्ष तक न्यायाधीश रह चुका हो,
  • वह कम-से-कम 10 वर्ष तक एक अथवा दो से अधिक उच्च न्यायालयों में वकालत कर चुका हो,
  • वह राष्ट्रपति के विचार में एक प्रतिष्ठित विधिवेत्ता (Jurist) हो।

शपथ उच्चतम न्यायालयों के न्यायाधीश अपना पद सम्भालने से पूर्व राष्ट्रपति अथवा उप-राष्ट्रपति द्वारा नियुक्त किए गए व्यक्ति के सम्मुख यह शपथ लेते हैं, “भारतीय संविधान के प्रति निष्ठावान् रहेंगे, उसमें सच्चा विश्वास रखेंगे, अपने कार्य को अपनी पूर्ण योग्यता, ज्ञान एवं न्याय की भावना सहित और निर्भीक व निष्पक्ष, बिना प्रेम या बुरी इच्छा के साथ निष्ठा से निभाएँगे तथा भारतीय संविधान व कानून की पुष्टि करेंगे।”

वेतन-उच्चतम न्यायालय के मुख्य न्यायाधीश को ₹ 2,80,000 तथा अन्य न्यायाधीशों को ₹ 2,50,000 मासिक वेतन दिए जाने की व्यवस्था की गई है। इसके अतिरिक्त उन्हें अन्य भत्ते, अवकाश तथा निःशुल्क निवास स्थान भी मिलता है। किसी न्यायाधीश की नियुक्ति के पश्चात् संसद द्वारा उनके वेतन तथा अन्य सुविधाओं में कमी नहीं की जा सकती। सेवा से मुक्त होने के पश्चात् उन्हें पेन्शन (Pension) भी दी जाती है। उच्चतम न्यायालय में न्यायाधीशों का वेतन केवल आर्थिक संकट की दशा में . ही कम किया जा सकता है। न्यायाधीशों का वेतन भारत की संचित निधि से दिया जाता है।

कार्यकाल-उच्चतम न्यायालय के न्यायाधीश 65 वर्ष की आयु तक अपने पद पर रहते हैं। न्यायापालिका की स्वतन्त्रता को ध्यान में रखते हुए संविधान निर्माताओं ने राष्ट्रपति को यह अधिकार नहीं दिया कि वह अपनी इच्छानुसार किसी न्यायाधीश को उसके पद से हटा सके। 65 वर्ष की आयु से पूर्व वह स्वयं त्यागपत्र दे सकता है या उसकी मृत्यु से उसका पद रिक्त हो सकता है।

इसके अतिरिक्त उन्हें बुरे व्यवहार व अयोग्यता के आधार पर ही 65 वर्ष की आयु से पूर्व उनके पद से हटाया जा सकता है। न्यायाधीशों को पद से हटाने की यह विधि महाभियोग के समान है। बुरे व्यवहार व अयोग्यता का यह प्रस्ताव संसद के दोनों सदनों में अलग-अलग उनके कुल सदस्यों के बहुमत तथा उपस्थित और मतदान में भाग लेने वाले सदस्यों के 2/3 बहुमत से पास किया जाना चाहिए। इस प्रक्रिया के अन्तर्गत एक ही अधिवेशन में संसद द्वारा न्यायाधीश के विरुद्ध प्रस्ताव पास हुए बिना राष्ट्रपति किसी न्यायाधीश को उसके पद से नहीं हटा सकता।

उच्चतम न्यायालय का कार्य स्थान साधारणतया उच्चतम न्यायालय का कार्य स्थान नई दिल्ली निश्चित किया गया है, परन्तु मुख्य न्यायाधीश राष्ट्रपति की स्वीकृति व पूर्वाज्ञा द्वारा इसकी बैठक किसी अन्य स्थान पर भी बुला सकता है। सेवा मुक्त होने के पश्चात् वकालत की मनाही-उच्चतम न्यायालय के न्यायाधीश सेवानिवृत्ति के पश्चात् भारत के किसी भी न्यायालय में वकालत नहीं कर सकते, परन्तु उन्हें किसी आयोग के सदस्य, राज्यपाल, राजदूत आदि उच्च पदों पर नियुक्त किया जा सकता है।

प्रश्न 4.
भारत के उच्चतम न्यायालय का संगठन, शक्तियों तथा कार्यों का वर्णन करो। अथवा उच्चतम न्यायालय के प्रारम्भिक तथा अपीलीय क्षेत्राधिकार का वर्णन कीजिए। अथवा सर्वोच्च न्यायालय की शक्तियों तथा क्षेत्राधिकार का वर्णन कीजिए।
उत्तर:
भारतीय संविधान संघात्मक शासन-व्यवस्था की स्थापना के साथ-साथ एकीकृत न्यायिक व्यवस्था की स्थापना भी करता है जिसमें उच्चतम न्यायालय (Supreme Court) शीर्ष पर है। भारतीय न्यायपालिका को संविधान-निर्माताओं ने स्वतन्त्र व निष्पक्ष बनाए रखने के लिए प्रावधान संविधान में ही कर दिए हैं।

संविधान में न्यायपालिका की संविधान की संरक्षक, नागरिकों के मौलिक अधिकारों की रक्षक, लोकतन्त्र व जन-कल्याण में सहायक तन्त्र के रूप में भूमिका निर्धारित की गई है। संविधान न्यायपालिका को न्यायिक पुनराविलोकन (Judicial Review) का अधिकार भी सौंपता है जिसके अन्तर्गत न्यायपालिका किसी भी कानून को अवैध घोषित कर सकती है, लेकिन न्यायपालिका उच्च विधायिका (Super Legislature) नहीं है। भारत में उच्चतम न्यायालय का क्षेत्राधिकार एवं शक्तियों का वर्णन निम्नलिखित है

उच्चतम न्यायालय का अधिकार क्षेत्र अथवा शक्ति (Power of Jurisdiction of Supreme Court)-श्री अल्लादी कृष्णा स्वामी अय्यर (Sh. Aladi Krishna SwamiAiyar) के अनुसार, “भारत के उच्चतम न्यायालय का अधिकार क्षेत्र विश्व के किसी भी संघात्मक राज्य के उच्चतम न्यायालय से अधिक है। यहाँ तक कि यह अमेरिका के उच्चतम न्यायालय से भी अधिक शक्तिशाली है।” उच्चतम न्यायालय की शक्तियाँ इस प्रकार हैं-

  • प्रारम्भिक क्षेत्राधिकार,
  • अपीलीय क्षेत्राधिकार,
  • परामर्शदात्री क्षेत्राधिकार,
  • संविधान का व्याख्याकार,
  • निर्णयों का पुनर्निरीक्षण,
  • अभिलेख का न्यायालय,
  • अन्य अधिकार,
  • संविधान का संरक्षक।

1. प्रारम्भिक क्षेत्राधिकार-उच्चतम न्यायालय के प्रारम्भिक क्षेत्रधिकार निम्नलिखित हैं
(क) संघीय मामले इस अधिकार क्षेत्र में वे मुकद्दमे आते हैं जो सीधे उच्चतम न्यायालय के पास ले जाए जा सकते हैं और जिनका सम्बन्ध संघीय व्यवस्था से है। इन्हें किसी निम्न न्यायालय में ले जाने की आवश्यकता नहीं होती। संविधान के अनुच्छेद 136 के अन्तर्गत उच्चतम न्यायालय का निम्नलिखित विषयों में एकमात्र प्रारम्भिक क्षेत्र है-

  • संघ सरकार तथा एक अथवा अधिक राज्यों के मध्य झगड़े,
  • ऐसे. झगड़े जिसमें संघ सरकार और कुछ राज्य एक ओर हों और कुछ राज्य दूसरी ओर हों,
  • राज्यों के पारस्परिक झगड़े जिनका सम्बन्ध कानून एवं संविधान की व्याख्या से हो।

अपवाद उपरोक्त विषयों से सम्बन्धित विषय किसी अन्य न्यायालय में पेश नहीं किए जा सकते, परन्तु निम्नलिखित प्रकार के विवाद उच्चतम न्यायालय के प्रारम्भिक क्षेत्राधिकार में नहीं आते

(1) उच्चतम न्यायालय के प्रारम्भिक क्षेत्राधिकार में केवल वही मुकद्दमे आते हैं जिनका सम्बन्ध कानूनी प्रश्न या तथ्य (Legal Question on Fact of Law) से हो अर्थात् राजनीतिक प्रकृति के विवाद उच्चतम न्यायालय के क्षेत्राधिकार में नहीं आते,

(2) उच्चतम न्यायालय के प्रारम्भिक क्षेत्राधिकार में वे मामले नहीं आते जिनका सम्बन्ध किसी ऐसी सन्धि, समझौते आदि से हो जो संविधान के लागू होने से पहले किए गए हों और जो अब तक निरन्तर जारी हों अथवा जिनमें यह स्पष्ट रूप से कहा गया हो कि उनके सम्बन्ध में उच्चतम न्यायालय को ऐसा अधिकार प्राप्त नहीं है। जैसे संविधान के.सातवें संशोधन के अनुसार भारत सरकार की देशी रियासतों के राजाओं से की हुई सन्धियाँ इस क्षेत्र में नहीं आतीं,

(3) उच्चतम न्यायालय के प्रारम्भिक क्षेत्राधिकार में वे विवाद भी नहीं आते जिनका सम्बन्ध अन्तर्राज्यीय नदियों (Inter-State Rivers), नदी घाटियों (River Valleys) तथा जल के वितरण आदि से हो,

(4) किसी नागरिक तथा किसी राज्य या भारत सरकार के बीच उत्पन्न होने वाले किसी भी झगड़े को सीधा उच्चतम न्यायालय में नहीं लाया जा सकता।

(ख) मौलिक अधिकारों का संरक्षक नागरिकों के मौलिक अधिकारों की सुरक्षा करना भी उच्चतम न्यायालय के प्रारम्भिक क्षेत्राधिकार में आता है। उच्चतम न्यायालय नागरिकों के अधिकारों तथा संविधान का संरक्षक है। यदि किसी नागरिक के अधिकारों का सरकार तथा अन्य संस्था द्वारा उल्लंघन किया जाता है तो वह सीधे उच्चतम न्यायालय की शरण ले सकता है।

संविधान के अनुच्छेद 32 के अन्तर्गत उच्चतम न्यायालय को ऐसे निर्देश, आदेश या प्रतिलेख जारी करने की शक्ति प्रदान की गई है, जिनके द्वारा यह नागरिकों के मौलिक अधिकारों की रक्षा करता है। ये लेख हैं बन्दी प्रत्यक्षीकरण लेख (Writ of Habeas Corpus), प्रतिषेध लेख (Writ of Prohibition), परमादेश लेख (Writ of Mandamus), उत्प्रेषण लेख (Writ of Certiorari) तथा अधिकार पृच्छा लेख (Writ of Quo-Warranto)।

(ग) राष्ट्रपति एवं उप-राष्ट्रपति के चुनाव सम्बन्धी झगड़े-राष्ट्रपति तथा उप-राष्ट्रपति के चुनाव से सम्बन्धित सभी विवादों के निर्णय केवल उच्चतम न्यायालय के द्वारा किए जाते हैं और इस प्रकार यह इसके प्रारम्भिक अधिकार के अन्तर्गत आता है।

(घ) संघीय लोक सेवा आयोग के सम्बन्ध में संघीय लोक सेवा आयोग के सदस्यों को महाभियोग के द्वारा तभी हटाया जा सकता है, जबकि इसके पूर्व उच्चतम न्यायालय सदस्यों पर लगे आरोपों की पुष्टि कर दे अर्थात् आयोग के सदस्यों के विरुद्ध लगे आरोपों की जाँच करने का एकमात्र अधिकार उच्चतम न्यायालय को प्राप्त है।

2. अपीलीय क्षेत्राधिकार-अपीलीय क्षेत्राधिकार से अभिप्राय उन विवादों से है जो आरम्भ तो उच्च न्यायालयों या निम्न न्यायालयों में होते हैं, लेकिन निर्णय के विरुद्ध अपील उच्चतम न्यायालय में की जा सकती है। उच्चतम न्यायालय के अपीलीय क्षेत्राधिकार को चार श्रेणियों में बाँटा जा सकता है।

(क) संवैधानिक मामले-किसी मुकद्दमे में अपना निर्णय देते हुए यदि उच्च न्यायालय यह प्रमाण पत्र दे दे कि यह विवाद संविधान की व्याख्या से सम्बन्धित है तो उसके विरुद्ध अपील उच्चतम न्यायालय में की जा सकती है। यदि उच्च न्यायालय ऐसा प्रमाण पत्र देने से इन्कार कर दे तो उच्चतम न्यायालय स्वयं भी संविधान के अनुच्छेद 132 के अन्तर्गत अपने सम्मुख अपील करने की आज्ञा दे सकता है।

(ख) फौजदारी मुकद्दमे-संविधान के अनुच्छेद 134 के अधीन फौजदारी मुकद्दमों में उच्च न्यायालयों के निर्णयों के विरुद्ध उच्चतम न्यायालयों के सामने उस समय अपील की जा सकती है यदि-

(1) किसी अभियुक्त को निम्न अदालत ने छोड़ दिया था, परन्तु उच्च न्यायालय ने अपील में उसे मृत्यु-दण्ड दिया हो तो अभियुक्त उच्चतम न्यायालय में अपील कर सकता है,

(2) यदि उच्च न्यायालय किसी फौजदारी मामले को निम्न न्यायालय से अपने पास मँगवाकर अभियोग की सुनवाई स्वयं करे और अभियुक्त को मृत्यु-दण्ड दे दे,

(3) ऐसे फौजदारी मामलों की अपील भी उच्चतम न्यायालय में की जा सकती है जिनके बारे में उच्च न्यायालय प्रमाण पत्र दे दे कि अमुक मामला उच्चतम न्यायालय के अधीन सुनवाई हेतु जा सकता है।

(ग) दीवानी अपीलें संविधान के अनुच्छेद 133 के अन्तर्गत उच्चतम न्यायालय को दीवानी अभियोगों में उच्च न्यायालयों के निर्णयों के विरुद्ध अपीलें सुनने का अधिकार प्राप्त है, लेकिन ऐसी अपील उसके सामने उसी समय की जा सकती है, जब उच्च न्यायायल यह प्रमाण पत्र दे कि इस मुकद्दमे में सार्वजनिक महत्त्व या हित का कोई कानूनी प्रश्न उलझा हुआ है और उसकी व्याख्या उच्चतम न्यायालय द्वारा की जानी आवश्यक है।

संविधान के 30वें संविधान संशोधन द्वारा बीस हजार रुपए वाली शर्त समाप्त कर दी गई है। दूसरे शब्दों में बीस हजार से कम या अधिक का कोई मामला तभी अपील के लिए उच्चतम न्यायालय में आएगा जब उसमें सामान्य हित का कोई कानूनी प्रश्न उलझा हुआ है।

1) विशेष अपीलें-संविधान के अनुच्छेद 136 के अधीन उच्चतम न्यायालय किसी भी व्यक्ति को किसी भी प्रकार के अभियोग के विषय में निम्न न्यायालय के विरुद्ध अपील करने की अनुमति दे सकता है, लेकिन उसे यह अधिकार सैनिक न्यायालयों के सम्बन्ध में प्राप्त नहीं है।

3. परामर्शदात्री क्षेत्राधिकार-संविधान के अनुच्छेद 143 द्वारा राष्ट्रपति को यह अधिकार दिया गया है कि वह किसी भी मामले पर उच्चतम न्यायालय से कानूनी सलाह माँग सकता है। ऐसे मामले पर कम-से-कम पाँच न्यायाधीश बैठकर विचार करते हैं और बहुमत से निर्णय करके निर्णय को राष्ट्रपति के पास भेज दिया जाता है। राष्ट्रपति इस निर्णय को मानने के लिए बाध्य नहीं है, लेकिन अभी तक उच्चतम न्यायालय के इस परामर्शदात्री कार्य का राष्ट्रपति द्वारा बराबर सम्मान किया गया है। राष्ट्रपति ने सर्वप्रथम उच्चतम न्यायालय से सलाह 1958 में केरल शिक्षा विधेयक के बारे में माँगी थी। अब तक अनेक बार उच्चतम न्यायालय राष्ट्रपति को सलाह दे चुका है।।

4. संविधान का व्याख्याका-संविधान की अन्तिम व्याख्या करने का अधिकार उच्चतम न्यायालय को है। इस अधिकार के अन्तर्गत न्यायालय को पुनरावलोकन का अधिकार दिया गया है। उच्चतम न्यायालय संसद तथा राज्य विधान-मण्डलों द्वारा बनाए गए कानून तथा कार्यपालिका द्वारा जारी किए गए किसी अध्यादेश को असंवैधानिक घोषित कर सकता है।

5. निर्णयों का पुनर्निरीक्षण-संविधान द्वारा उच्चतम न्यायालय को अपने द्वारा किए गए निर्णयों को दृष्टान्त या मिसाल के रूप में न मानकर कानून की नई व्याख्या करने की शक्ति दी गई है।

6. अभिलेख का न्यायालय उच्चतम न्यायालय अभिलेख का न्यायालय भी है। उसके निर्णय तथा कार्रवाई को भविष्य में उदाहरणार्थ पेश करने के लिए लिपिबद्ध करके रखा जाता है।

7. अन्य अधिकार-उच्चतम न्यायालय को अन्य दूसरे कार्य करने का अधिकार भी है। इन कार्यों में

  • उच्चतम न्यायालय उन लोगों को दण्ड दे सकता है जो इसका अपमान करें,
  • उच्चतम न्यायालय के आदेश और निर्णय भारत के सभी असैनिक न्यायालय तथा पदाधिकारियों को मानने पड़ते हैं,
  • भारत के सभी असैनिक न्यायालय इसके नियन्त्रण में हैं। यह किसी भी न्यायालय को कोई भी आदेश दे सकता है। अभियोगों को एक न्यायालय से दूसरे न्यायालय में भेजा जा सकता है। यह किसी भी तरह की सूचना मँगवा सकता है,
  • वह अपने अधीन काम करने वाले कर्मचारियों को नियुक्त करता है। उनके वेतन, भत्ते तथा सेवा की दूसरी शर्तों को निश्चित करता है,
  • उच्चतम न्यायालय किसी भी उच्च न्यायालय तथा अधीनस्थ न्यायालयों का निरीक्षण कर सकता है,
  • उच्च न्यायालयों के न्यायाधीशों की नियुक्ति करते समय राष्ट्रपति उच्चतम न्यायालय के न्यायाधीशों की सलाह लेता है,
  • उच्चतम न्यायालय निम्न न्यायालयों के फैसले, डिग्री, दण्ड, आज्ञा के विरुद्ध अपने आगे अपील करने की आज्ञा दे सकता है,
  • कोई भी नागरिक या सरकारी कर्मचारी न्यायालय के काम में हस्तक्षेप नहीं कर सकता। ऐसा करने पर उसे दण्ड दिया जा सकता है।

8. संविधान का संरक्षक-उच्चतम न्यायालय को सर्वाधिक महत्त्वपूर्ण शक्ति संविधान के संरक्षक के रूप में प्राप्त है। इसके अधीन उच्चतम न्यायालय कानूनों के साथ-साथ उन संविधान संशोधनों को भी अवैध घोषित कर सकता है जो उसके विचार में संविधान का उल्लंघन करते हों। अपने इस अधिकार के अन्तर्गत केशवानन्द भारती मामले सन् 1973 में उच्चतम न्यायालय ने संसद के संशोधन सम्बन्धी अधिकार को सीमित करते हुए कहा था, “संसद ऐसा कोई कानून या संविधान में संशोधन नहीं कर सकती कि जिससे संविधान का मूलभूत ढाँचा विकृत अथवा नष्ट होता हो।”

निष्कर्ष-उच्चतम न्यायालय की शक्तियाँ तथा कार्य उसे भारतीय संवैधानिक व्यवस्था में अति महत्त्वपूर्ण बना देते हैं। उच्चतम न्यायालय संघात्मक व्यवस्था के रक्षक, संविधान के रक्षक, संविधान के व्याख्याकार, मौलिक अधिकारों के रक्षक की भूमिका निभाने के साथ-साथ लोकतन्त्र व जनकल्याण के विकास में प्रभावशाली भूमिका निभाता है।

च्चतम न्यायालय दलित व गरीब लोगों के लिए न्याय का अन्तिम शरणगाह है। उच्चतम न्यायालय ने लोकहितवाद (Public Interest Litigation) के द्वारा अपने अधिकारों व शक्तियों में असीमित वृद्धि कर ली है। उच्चतम न्यायालय ने अपने अनेक निर्णयों में कहा है, “न्यायपालिका कानूनों व संविधान की व्याख्याकार मात्र नहीं है, बल्कि न्याय की अन्तिम शरणस्थली है।” उच्चतम न्यायालय ने न्यायिक पुनरावलोकन की शक्ति द्वारा संसद व कार्यपालिका की शक्ति को सीमित करके लोकतन्त्र की रक्षा करने में महत्त्वपूर्ण भूमिका निभाई है।

प्रश्न 5.
उच्च न्यायालय के न्यायाधीशों की नियुक्ति, योग्यताओं, कार्यकाल तथा वेतन का वर्णन करो। अथवा राज्य के उच्च न्यायालय की रचना तथा क्षेत्राधिकार का वर्णन करो।
उत्तर:
परिचय (Introduction) भारत में एकल न्याय-व्यवस्था है, सबसे ऊँचा न्यायालय सर्वोच्च न्यायालय है, उसके अधीन राज्यों में उच्च न्यायालय पाए जाते हैं। संविधान के अनुच्छेद 214 के अन्तर्गत राज्य स्तर पर प्रत्येक राज्य के लिए एक उच्च न्यायालय (High Court) की व्यवस्था की गई है। इसके अतिरिक्त अनुच्छेद 231 में यह भी कहा गया है कि संसद दो या दो से अधिक राज्यों और केन्द्र-प्रशासित क्षेत्र के लिए एक ही उच्च न्यायालय की व्यवस्था कर सकती है। सन् 1966 के पंजाब पुनर्गठन कानून के द्वारा संसद ने पंजाब और हरियाणा राज्यों और चण्डीगढ़ के केन्द्र-प्रशासित क्षेत्र के लिए एक उच्च न्यायालय स्थापित किया, जो चण्डीगढ़ में है। राज्यों के ये उच्च न्यायालय अखिल भारतीय न्याय-व्यवस्था का अंग होते हुए भी अपने आप में स्वतन्त्र इकाइयाँ हैं।

उनके ऊपर अपने राज्यों के विधानमण्डल या कार्यपालिका का कोई नियन्त्रण नहीं है। मार्च, 2013 में मेघालय, मणिपुर एवं त्रिपुरा राज्य के अलग से उच्च न्यायालय अस्तित्व में आने पर कुल उच्च न्यायालयों की संख्या 21 से बढ़कर 24 हुई थी जो 1 जनवरी, 2019 को आन्ध्रप्रदेश की राजधानी अमरावती में आन्ध्रप्रदेश राज्य का अलग उच्च न्यायालय अस्तित्व में आने के बाद देश में कुल उच्च न्यायालयों की संख्या वर्तमान में 25 हो गई हैं। यहाँ यह उल्लेखनीय है कि आन्ध्रप्रदेश पुनर्गठन अधिनियम के तहत आन्ध्रप्रदेश एवं तेलंगाना दो अलग-अलग राज्य बने थे .. जिनका हैदराबाद में संयुक्त उच्च-न्यायालय था। अब हैदराबाद उच्च-न्यायालय को तेलंगाना उच्च न्यायालय के नाम से जाना जाएगा। उच्च न्यायालय एक राज्य का सबसे बड़ा न्यायालय है और एक राज्य के अन्य सभी न्यायालय उसके अधीन होते हैं।

रचना (Composition)-उच्च न्यायालयों के गठन सम्बन्धी प्रावधान संविधान के अनुच्छेद 216 में हैं। प्रत्येक उच्च न्यायालय में एक मुख्य न्यायाधीश (Chief Justice) और कुछ अन्य न्यायाधीश होते हैं। इनकी संख्या सम्बन्धित राज्यों की जरूरतों को ध्यान में रखकर राष्ट्रपति द्वारा तय की जाती है। सभी उच्च न्यायालयों के न्यायाधीशों की संख्या समान नहीं होती।

उनकी संख्या राष्ट्रपति द्वारा निश्चित की जाती है; जैसे हरियाणा एवं पंजाब उच्च न्यायालय के न्यायाधीशों की कुल संख्या 85 निश्चित की गई जबकि हिमाचल प्रदेश के उच्च न्यायालय में कुल संख्या 13 निश्चित की गई है। काम की अधिकता होने पर एक उच्च न्यायालय में अधिक-से-अधिक दो वर्षों के लिए अतिरिक्त न्यायाधीश (Additional Judges) भी नियुक्त किए जा सकते हैं। इसके अतिरिक्त उच्च न्यायालय का मुख्य न्यायाधीश राष्ट्रपति की स्वीकृति से उच्च न्यायालय के किसी सेवा-निवृत्त न्यायाधीश को भी उच्च न्यायालय के न्यायाधीश के रूप में कार्य करने के लिए प्रार्थना कर सकता है।

कार्यकारी मुख्य न्यायाधीश की नियुक्ति-अनुच्छेद 223 के अन्तर्गत यदि किसी उच्च न्यायालय के मुख्य न्यायाधीश का पद रिक्त हो जाए या जब मुख्य न्यायाधीश अनुपस्थित हो या अन्य किसी कारण से अपने पद के कर्त्तव्यों का पालन करने में असमर्थ हो तो राष्ट्रपति उस न्यायालय के अन्य न्यायाधीशों में से किसी एक को उस पद के लिए कार्यकारी मुख्य न्यायाधीश नियुक्त करता है। अतिरिक्त तथा कार्यकारी न्यायाधीशों की नियुक्ति अनुच्छेद 224

(1) के अन्तर्गत यदि किसी उच्च न्यायालय में अस्थायी रूप में कार्य बढ़ गया है और राष्ट्रपति यह अनुभव करता है कि इस कार्य के लिए न्यायाधीशों की संख्या को अस्थायी रूप से बढ़ाना आवश्यक है तो वह उपयुक्त योग्यता रखने वाले व्यक्तियों को अधिक-से-अधिक दो वर्ष के लिए अतिरिक्त (Additional) न्यायाधीश के रूप में नियुक्त कर सकता है।

इसी प्रकार अनुच्छेद 224 (2) के अन्तर्गत उच्च न्यायालय के किसी न्यायाधीश की अनुपस्थिति में राष्ट्रपति उस काल के लिए कार्यकारी न्यायाधीश की नियुक्ति कर सकता है। – न्यायाधीशों का दूसरे राज्यों के उच्च न्यायालयों में स्थानान्तरण-अनुच्छेद 222 के अन्तर्गत राष्ट्रपति को उच्चतम न्यायालय के मुख्य न्यायाधीश की सलाह से किसी न्यायाधीश को एक उच्च न्यायालय से दूसरे उच्च न्यायालय में स्थानान्तरित करने का अधिकार है।

उच्चतम न्यायालय ने एस०पी० गुप्ता बनाम रतभा सरकार 1981 (न्यायाधीश मामला) में कहा, “सरकार का उच्च न्यायालयों के न्यायाधीशों का दूसरे राज्यों में तबादला करने का अधिकार पूर्णतः वैधानिक है।” इसके लिए सम्बन्धित न्यायाधीश की सहमति लेना आवश्यक नहीं है। निर्णय में स्पष्ट रूप से कहा गया है कि न्यायाधीशों के स्थानान्तरण केवल जनहित (Public Interest) में ही किए जा सकते हैं, उन्हें दण्ड देने के लिए नहीं। इसके लिए देश के मुख्य न्यायाधीश से प्रभावी सलाह (Effective Consultation) लिया जाना आवश्यक है।

योग्यताएँ-अनुच्छेद 217 (2) में उच्च न्यायालय के न्यायाधीशों की निम्नलिखित योग्यताएँ निश्चित की गई हैं-

(1) वह भारत का नागरिक हो,
(2) वह भारत में किसी न्याय के पद पर कम-से-कम पाँच वर्ष तक रह चुका हो। अथवा वह किसी उच्च न्यायालय या उच्च न्यायालयों में निरन्तर दस वर्ष तक वकील के रूप में कार्य कर चुका हो।

कार्यकाल-उच्च न्यायालय के न्यायाधीश 65 वर्ष पूर्ण होने तक अपने पद पर रह सकेंगे। इसके अतिरिक्त दी गई अवस्थाओं में कोई न्यायाधीश पद-मुक्त हो सकता है-
(1) यदि उसकी पदोन्नति करके उच्चतम न्यायालय में स्थानान्तरित कर दिया जाए,

(2) यदि भ्रष्टाचार या अयोग्यता के आधार पर संसद अपनी कुल सदस्य संख्या के बहुमत और उपस्थित सदस्यों के दो-तिहाई बहुमत से दोनों सदनों में अलग-अलग किन्तु संसद के एक ही सत्र (Session) में उच्च न्यायालय के किसी न्यायाधीश के विरुद्ध महाभियोग का प्रस्ताव पारित कर दे,

(3) स्वयं त्यागपत्र देने पर,

(4) अवकाश-प्राप्ति के पश्चात् वह अपने न्यायालय के अतिरिक्त किसी भी उच्च न्यायालय या उच्चतम न्यायालय में वकालत कर सकता है।

वेतन-उच्च न्यायालय के न्यायाधीशों को समस्त भारत में समान वेतन देने की व्यवस्था की गई है। उच्च न्यायालयों के मुख्य न्यायाधीशों को ₹ 2,50,000 तथा अन्य न्यायाधीशों को ₹ 2,25,000 मासिक वेतन व भत्ते तथा रहने के लिए बिना किराए का निवास स्थान मिलता है। नियुक्ति के बाद उनके वेतन व भत्तों में कमी नहीं की जा सकती। न्यायाधीशों को वेतन संचित निधि में से दिया जाता है। वित्तीय आपातस्थिति में ही उनके वेतन में कमी की जा सकती है। उच्च न्यायालय का अधिकार क्षेत्र भारत के उच्च न्यायालयों के अधिकार क्षेत्र को निम्नलिखित भागों में बाँटा जा सकता है

1. साधारण अधिकार क्षेत्र-अनुच्छेद 225 के द्वारा भारत के उच्च न्यायालयों को वे सभी अधिकार क्षेत्र प्राप्त हैं जो संविधान के लागू होने से पूर्व उच्च न्यायालयों को प्राप्त थे। इस अनुच्छेद में कहा गया है कि इस संविधान के उपबन्धों के अधीन रहते हुए तथा इस संविधान द्वारा विधान-मण्डल को प्रदत्त शक्तियों के आधार पर विधानमण्डल द्वारा बनाई गई किसी समुचित विधि के उपबन्धों के अधीन रहते हुए किसी वर्तमान उच्च न्यायालय का क्षेत्राधिकार और उसमें प्रशासित विधि तथा उस न्यायालय में न्याय के प्रशासन के सम्बन्ध में उसके न्यायाधीशों की अपनी-अपनी शक्तियाँ, जिनके अन्तर्गत न्यायालय के नियम बनाने का तथा उस न्यायालय की बैठकों की शक्तियाँ वैसी ही रहेंगी, जैसी इस संविधान के प्रारम्भ से ठीक पहले थीं।।

2. प्रारम्भिक अधिकार क्षेत्र-कई मुकद्दमे सीधे उच्च न्यायालय में पेश किए जा सकते हैं। निम्नलिखित मामलों में उच्च न्यायालयों को आरम्भिक क्षेत्राधिकार प्राप्त हैं

(1) संविधान के अनुच्छेद 226 के अधीन मूल अधिकारों से सम्बन्धित कोई भी मामला उच्चतम न्यायालय की तरह उच्च न्यायालय में भी लाया जा सकता है। मौलिक अधिकारों को लागू करवाने के लिए उच्च न्यायालय को कई तरह के लेख (Writs)
जारी करने का अधिकार है; जैसे बन्दी प्रत्यक्षीकरण लेख (Writ of Habeas Corpus),
परमादेश लेख (Writ of Mandamus)
प्रतिषेध लेख (Writ of Prohibition),
अधिकार पृच्छा लेख (Writ of Quo-warranto),
उत्प्रेषण लेख (Writ of Certiorari),

(2) यदि राज्य विधान-मण्डल ने कोई कानून संविधान के विरुद्ध बनाया है तो उसको उच्च न्यायालय अवैध घोषित कर सकता है। यद्यपि उसके फैसले के विरुद्ध उच्चतम न्यायालय में अपील की जा सकती है,

(3) तलाक, विवाह, वसीयत, न्यायालय का अपमान, . कम्पनी कानून आदि के मुकद्दमे भी उच्च न्यायालय द्वारा सीधे सुने जाते हैं,

(4) यदि उच्च न्यायालय ठीक समझे तो अधीनस्थ न्यायालयों से कोई मुकद्दमा सीधे अपने पास मंगवाकर उसकी सुनवाई कर सकता है,

(5) चुनाव सम्बन्धी मामले भी उच्च न्यायालय में सीधे लाए जाते हैं,

(6) मुम्बई, चेन्नई तथा कोलकाता के उच्च न्यायालयों को कुछ अधिक प्रारम्भिक क्षेत्राधिकार प्राप्त हैं।

3. अपीलीय अधिकार क्षेत्र-उच्च न्यायालयों को निम्न न्यायालयों के निर्णयों के विरुद्ध दीवानी तथा फौजदारी अभियोगों में अपीलें सुनने का अधिकार भी प्राप्त है-

(1) जिस मुकद्दमे में पाँच हजार या उससे अधिक राशि अथवा इतने ही मूल्य की सम्पत्ति का प्रश्न हो तो ऐसे दीवानी मुकद्दमों की अपील उच्च न्यायालय द्वारा सुनी जा सकती है,

(2) फौजदारी मामलों में सत्र न्यायालय के निर्णय के विरुद्ध अपील उच्च न्यायालय में की जा सकती है, यदि न्यायालय ने अपराधी को 4 वर्ष अथवा इससे अधिक कैद की सजा दी है,

(3) यदि अभियुक्त को मृत्यु-दण्ड दिया गया है,

(4) दीवानी क्षेत्र में पेटेन्ट (Patent) तथा डिज़ाइन (Design) उत्तराधिकार, भूमि-प्राप्ति, दिवालियापन और संरक्षकता आदि मामलों में उच्च न्यायालय अपील सुन सकता है,

(5) राजस्व सम्बन्धी मामलों में निम्न न्यायालयों और आय कर से सम्बन्धित मामलों में आय-कर अधिकारियों के निर्णयों के विरुद्ध अपीलें सुनी जा सकती हैं।

4. प्रशासनिक अधिकार क्षेत्र-देश के उच्च न्यायालयों को अपने-अपने राज्य में अधीनस्थ सभी न्यायालयों एवं न्यायाधिकरणों . पर निरीक्षण (Superintendence) का अधिकार है। 42वें संशोधन ने उच्च न्यायालयों के अधिकार क्षेत्र से अधिकरणों को निकाल , दिया था, किन्तु 44वें संविधान संशोधन अधिनियम ने अनुच्छेद 227 में संशोधन करके न्यायाधिकरणों को पुनः उच्च न्यायालय के अधिकार क्षेत्र में दे दिया। उच्च न्यायालय अपने क्षेत्र के अधीनस्थ न्यायालयों पर अपना पूर्ण नियन्त्रण रखता है। यह नियन्त्रण इस प्रकार से स्थापित किया जा सकता है-

  • कार्रवाई का विवरण माँग सकता है,
  • अधीनस्थ न्यायालयों में कर्मचारियों की नियुक्ति, वेतन, भत्ते, अवकाश की शर्ते आदि निश्चित करता है,
  • निम्न न्यायालय उच्च न्यायालय की इच्छानुसार तथा आदेश-निर्देश के अनुसार कार्य करते हैं,
  • उच्च न्यायालय अधीनस्थ न्यायालयों के कागजात माँगकर जाँच-पड़ताल कर सकता है,
  • उनके रिकॉर्ड रखने की व्यवस्था कर सकता है,
  • किसी एक न्यायालय में किसी एक मुकद्दमे को हटाकर दूसरे अधीनस्थ न्यायालयों में विचार के लिए भेज सकता है।

5. न्यायिक पुनरावलोकन-उच्चतम न्यायालय की तरह राज्य के उच्च न्यायालय को न्यायिक पुनरावलोकन का अधिकार प्राप्त है। विधानसभा द्वारा पारित कानून यदि संविधान के निश्चित नियमों के विरुद्ध हो तो उच्च न्यायालय के सामने मुकद्दमा आने पर अपनी इस शक्ति का प्रयोग करते हुए वह उस कानून को गैर-कानूनी घोषित कर सकता है। उच्च न्यायालय के ऐसे निर्णय के विरुद्ध उच्चतम न्यायालय में अपील की जा सकती है।

6. प्रमाण-पत्र देने का अधिकार उच्च न्यायालय के निर्णयों के विरुद्ध उच्चतम न्यायालय में तभी अपील की जा सकती है. यदि उच्च न्यायालय ऐसा प्रमाण-पत्र जारी करे। विशेष परिस्थिति में उच्चतम न्यायालय उच्च न्यायालय के प्रमाण-पत्र के बिना भी अपील की अनुमति दे सकता है।

7. अभियोगों को स्थानान्तरित करना यदि उच्च न्यायालय को यह विश्वास हो कि किसी निम्न न्यायालय में चल रहे अभियोग में कानून की व्याख्या की आवश्यकता है अथवा अभियुक्त को किसी कारण पूरा न्याय मिलने की आशा नहीं है तो उच्च न्यायालय मुकद्दमा अपने पास मँगवा सकता है अथवा किसी दूसरे न्यायालय में भेज सकता है अथवा कानून की व्याख्या कर सकता है। ऐसी स्थिति में निम्न न्यायालय को उच्च न्यायालय की व्याख्या को ध्यान में रखकर निर्णय करना पड़ता है।

8. अभिलेख न्यायालय राज्य का उच्च न्यायालय अभिलेख का न्यायालय भी होता है। उसके सभी निर्णयों को रिकॉर्ड के रूप में रखा जाता है तथा उन्हें प्रकाशित किया जाता है। इसमें निर्णय दृष्टान्त के रूप में विभिन्न आदेश में पेश किए जाते हैं। उच्च न्यायालय अपनी अवमानना करने (Contempt of Court) के लिए भी किसी व्यक्ति को दण्ड दे सकता है। उच्च न्यायालय की स्थिति-उच्च न्यायालय (High Court) न्यायिक मामलों में राज्य का उच्चतम न्यायालय है।

उसे प्रारम्भिक तथा अपीलीय क्षेत्राधिकार के साथ-साथ मौलिक अधिकारों के सम्बन्ध में कई प्रकार के लेख (Writ) जारी करने का अधिकार प्राप्त है, जिससे वह कार्यपालिका को नियन्त्रित करता है। उच्च न्यायालय न्यायिक पुनरावलोकन (Judicial Review) के अधीन संसद व राज्य विधानसभा द्वारा बनाए गए कानूनों को अवैध घोषित कर सकता है। राज्य के अन्य अधीनस्थ न्यायालय उसके नियन्त्रण में होते हैं, लेकिन उच्च न्यायालय न्यायिक शक्ति का शिखर नहीं है। उसके निर्णयों के विरुद्ध उच्चतम न्यायालय में अपील की जा सकती है।

HBSE 11th Class Political Science Important Questions Chapter 6 न्यायपालिका

प्रश्न 6.
भारतीय न्यायपालिका की स्वतन्त्रता पर एक निबन्ध लिखें।
अथवा
उच्चतम न्यायालय की स्वतन्त्रता पर एक लेख लिखें।
उत्तर:
लोकतान्त्रिक शासन-व्यवस्था के लिए स्वतन्त्र व निष्पक्ष न्यायपालिका का होना अनिवार्य है, ताकि कानून के शासन को बनाए रख सके तथा नागरिकों के अधिकारों की रक्षा कर सके। संघात्मक व्यवस्था वाले राज्यों में स्वतन्त्र भूमिका और भी बढ़ जाती है। न्यायपालिका ही इस बात की गारन्टी करती है कि संघीय सरकार व राज्य सरकारें अपने-अपने क्षेत्राधिकारों में ही रहें। संविधान की सर्वोच्चता को स्वतन्त्र न्यायपालिका ही बनाए रख सकती है।

अर्थ-न्यायपालिका की स्वतन्त्रता से अभिप्राय ऐसी व्यवस्था से है जिसमें कार्यपालिका अथवा विधायिका न्यायाधीशों के निर्णयों को प्रभावित अथवा दुष्प्रभावित न कर सके। न्यायपालिका की स्वतन्त्रता से अभिप्राय है कि न्यायाधीश अपने निर्णय निर्भीक होकर निष्पक्ष रूप से देने की स्थिति में हों, उन्हें कोई प्रभावित कर सकने की स्थिति में न हो। भारतीय संविधान-निर्माताओं ने न्यायपालिका की स्वतन्त्रता को सुनिश्चित करने के लिए निम्नलिखित प्रावधान संविधान में किए हैं

1. न्यायाधीशों की नियुक्ति-संविधान के अनुच्छेद 124 के अधीन राष्ट्रपति उच्चतम न्यायालय के मुख्य न्यायाधीश की नियुक्ति उच्चतम न्यायालय के न्यायाधीशों तथा उच्च न्यायालय के उन न्यायाधीशों की सलाह करके (जिन्हें वह सलाह लेने के उपयुक्त समझता है) करता है, और उच्चतम न्यायालय में अन्य न्यायाधीशों की नियुक्ति मुख्य न्यायाधीश की सलाह से करता है। इस प्रकार न्यायाधीशों की नियुक्ति में जनता अथवा विधायिका की कोई भूमिका नहीं है। यद्यपि संसदात्मक व्यवस्था होने के कारण सलाह अवश्य करता है, लेकिन वह न्यायाधीशों की सलाह की आमतौर पर उपेक्षा नहीं कर पाता है। इस प्रकार न्यायाधीशों की नियुक्ति में न्यायपालिका अधिक प्रभावी है, न कि कार्यपालिका अथवा विधायिका।

2. न्यायाधीशों की योग्यता संविधान में ही न्यायाधीशों की योग्यता निर्धारित की गई है, ताकि केवल अनुभवी विधि-विशेषज्ञ ही उच्चतम न्यायालय में न्यायाधीश के रूप में नियुक्त किए जा सकें और सरकार मनमाने राजनीतिक आधार पर किसी को भी न्यायाधीश नियुक्ति न कर सके। एक उच्च कोटि के विद्वान् से सदैव यह आशा की जाती है कि वह अपना कार्य स्वतन्त्रता-पूर्वक व निर्भीक होकर करता है, उसे कोई सरकार प्रभावित नहीं कर सकती।

3. लम्बी अवधि-न्यायपालिका की स्वतन्त्रता को सुनिश्चित करने के लिए उच्चतम न्यायालय के न्यायाधीश की कार्य-अवधि लम्बी रखी गई है, जबकि सामान्यतया सरकारी कर्मचारी 60 वर्ष की आयु में सेवानिवृत्त होते हैं। उच्चतम न्यायालय के न्यायाधीश 65 वर्ष की आयु तक पद पर बने रहते हैं।

4. पद की सुरक्षा न्यायपालिका की स्वतन्त्रता को सुनिश्चित करने के लिए उच्चतम न्यायालय के न्यायाधीशों को पद की सुरक्षा प्रदान की गई है। उच्चतम न्यायालय के मुख्य न्यायाधीश व अन्य न्यायाधीशों को महाभियोग जैसी जटिल पद्धति से ही हटाया जा सकता है। इसके लिए न्यायाधीश के विरुद्ध कदाचार (बरा व्यवहार) अथवा अयोग्यता का आरोप स्वतन्त्र व निष्पक्ष जाँच-पड़ताल द्वारा सिद्ध करना होता है

और उसके बाद संसद के दोनों सदन अपने-अपने सदन के कुल सदस्यों के बहुमत तथा उपस्थित व मतदान में भाग लेने वाले सदस्यों के 2/3 बहुमत से महाभियोग प्रस्ताव का समर्थन करते हैं तो न्यायाधीश को अपने पद से हटना होगा। महाभियोग की इस प्रक्रिया की जटिलता का ज्ञान इसी तथ्य से हो जाता है कि उच्चतम न्यायालय के न्यायाधीश श्री रामास्वामी के विरुद्ध महाभियोग सम्बन्धी आरोप-पत्र 1990 में 101 सांसदों ने दिया था, जिस पर अभी तक विवाद चल रहा । है और महाभियोग सम्बन्धी प्रक्रिया शुरु भी नहीं हो पाई है।

5. वेतन व भत्तों की सुरक्षा संविधान में ही न्यायाधीशों के वेतन व भत्तों की सुरक्षा की गई है। संसद न्यायाधीशों के . कार्यकाल के दौरान उनके वेतन व भत्तों में कटौती नहीं कर सकती, हाँ, बढ़ा अवश्य सकती है। भत्ते भारत की संचित निधि से दिए जाते हैं जिस पर संसद की स्वीकृति मात्र औपचारिकता होती है। न्यायाधीशों के वेतन व भत्तों में केवल वित्तीय संकट के दौरान ही कटौती की जा सकती है।

6. कर्मचारी वर्ग पर नियन्त्रण उच्चतम न्यायालय में राष्ट्रपति केवल न्यायाधीशों की नियुक्ति करता है। उच्चतम न्यायालय के अन्य सभी कर्मचारियों की नियुक्ति मुख्य न्यायाधीश करता है और उन पर उच्चतम न्यायालय का नियन्त्रण होता है, न कि सरकार का।

7. नियम बनाना-उच्चतम न्यायालय अपनी कार्य प्रणाली के लिए संसदीय कानूनों के अनुसार नियम स्वयं बनाता है। नियमों के बनाने में संसद की कोई भूमिका नहीं होती, लेकिन ये नियम उच्चतम न्यायालय के साथ-साथ भारत के अन्य सभी न्यायालयों में मान्य होते हैं।

8. अवकाश-प्राप्ति के बाद वकालत की मनाही-न्यायपालिका की स्वतन्त्रता को बनाए रखने के लिए संविधान में ही व्यवस्था की गई है कि उच्चतम न्यायालय का न्यायाधीश अवकाश ग्रहण करने के बाद देश के किसी भी न्यायालय में वकालत नहीं कर सकता, लेकिन वह किसी आयोग का सदस्य अथवा अध्यक्ष बन सकता है।

9. उन्मुक्तियाँ-न्यायपालिका की स्वतन्त्रता को सुनिश्चित करने के लिए न्यायाधीशों को कुछ उन्मुक्तियाँ व विशेषाधिकार प्राप्त हैं, जैसे न्यायाधीशों की आलोचना उनके द्वारा किए गए निर्णयों के कारण नहीं की जा सकती। संसद न्यायाधीशों के ऐसे कार्यों . पर, जिसे उन्होंने अपने कर्त्तव्य का पालन करते हुए किया है, विचार-विमर्श नहीं कर सकती। धान-निर्माताओं ने व्यापक व विस्तृत रूप से न्यायपालिका की स्वतन्त्रता को सुनिश्चित करने की कोशिश की है, लेकिन व्यवहार में ऐसे अनेक तथ्य सामने आए हैं जिनसे न्यायपालिका की स्वतन्त्रता को खतरा उत्पन्न हुआ है।

प्रश्न 7.
न्यायिक क्रियाशीलता से आपका क्या तात्पर्य है? इसके मुख्य साधनों की व्याख्या करो।
उत्तर:
सरकार के तीन महत्त्वपूर्ण अंग-विधानपालिका, कार्यपालिका व न्यायपालिका हैं। विधानपालिका कानून बनाने, कार्यपालिका कानूनों को लागू करने तथा न्यायपालिका निर्णय देने का कार्य करती है। मान्टेस्क्यू ने शक्ति विभाजन के सिद्धान्त में यह बताया है कि इन तीनों अंगों को अपने उप-क्षेत्र में काम करना चाहिए तथा एक-दूसरे के क्षेत्र में हस्तक्षेप नहीं करना चाहिए।

भारत में इसी प्रकार की व्यवस्था है कि सरकार के तीनों अंग अपने-अपने क्षेत्रों में काम करते हैं और एक-दूसरे के क्षेत्र में हस्तक्षेप नहीं करते हैं। परन्तु पिछले कुछ वर्षों में न्यायपालिका ने कार्यपालिका व विधानपालिका के क्षेत्र में हस्तक्षेप करना प्रारम्भ कर दिया है और इसी हस्तक्षेप को विभिन्न कानूनवेताओं, राजनीतिकों और संवैधानिक विशेषज्ञों ने न्यायिक क्रियाशीलता का नाम दिया है।

न्यायिक क्रियाशीलता का अर्थ (Meaning of Judicial Activism) न्यायिक क्रियाशीलता भारत में एक नवीन अवधारणा है। इसलिए इसकी ठीक ढंग से व्याख्या करना कठिन है। न्यायपालिका का मुख्य कार्य निर्णय देना है। जब निर्णय संविधान अथवा कानून के अनुसार दिए जाते हैं तो इसे न्यायिक क्रियाशीलता का नाम नहीं दिया जाता। परन्तु जब न्यायपालिका अपने क्षेत्राधिकार से बाहर जाकर विधानपालिका और कार्यपालिका के क्षेत्राधिकार में हस्तक्षेप करती है तब इसे न्यायिक क्रियाशीलता कहा जाता है। इस प्रकार न्यायपालिका द्वारा नीतियों और कार्यक्रमों को निश्चित कर और उन्हें लागू करने के आदेश देना। योजनाओं के संचालन को अपने हाथ में लेना, स्वाधीन संस्थाओं की कार्य-प्रणाली में हस्तक्षेप करना।

कानून का निर्माण करने के लिए विधानपालिका को निर्देश देना आदि, न्यायिक क्रियाशीलता के प्रतीक माने जाते हैं। साधारण शब्दों में, न्यायपालिका की किसी भी कार्रवाई को न्यायिक क्रियाशीलता की संज्ञा दी जा सकती है। जिसके द्वारा न्यायपालिका की प्राथमिकता स्थापित होती है। अतः न्यायिक क्रियाशीलता का अर्थ न्यायपालिका द्वारा अपने क्षेत्र में क्रियाशील भूमिका निभाना न होकर क्षेत्राधिकार से बाहर क्रियाशील भूमिका निभाने व कहते हैं। सर्वोच्च न्यायालय के भूतपूर्व न्यायाधीश पी०बी० सावंत (P.B. Swant) के शब्दों में, “कानून की व्याख्या करने और इसे लागू करते समय सवैधानिक सीमा की अवहेलना करना, शक्तियों के पृथक्करण के सिद्धान्त की अवहेलना करते हुए विधानपालिका के कार्यों को अपने हाथ में लेने को न्यायिक क्रियाशीलता कहा जाता है।”

न्यायिक क्रियाशीलता की आवश्यकता (Need of Judicial Activism)-प्रशासन एक इकाई है सरकार के तीनों अंग अलग-अलग होते हुए मिलकर कार्य करते हैं तभी एक योग्य प्रशासन सम्भव होता है, परन्तु जब सरकार अपने दायित्वों को निभाने में ढीलापन अथवा अनियमितता दिखाता है, तब कानून का शासन स्थापित करना कठिन हो जाता है। हर तरफ भ्रष्टाचार फैल जाता है। नौकरशाही तथा राजनीतिज्ञों द्वारा अपनी विशिष्ट स्थिति का गलत प्रयोग किया जाता है और उनके विरुद्ध कोई कार्रवाई नहीं अर्थात् एक तरह का जंगल राज होता है।

ऐसी स्थिति में न्यायपालिका को क्रियाशील होना ही पड़ता है। न्यायमूर्ति जे०एस० वर्मा के शब्दों में, “न्यायिक क्रियाशीलता की आवश्यकता केवल तब ही होती है जब अन्य अक्रियाशील होते हैं। एक स्थान पर जे०एस० वर्मा ने फिर कहा है, “न्यायिक क्रियाशीलता अक्रियाशीलता को क्रियाशील बनाती है।” अतः दूसरों की अक्रियाशीलता ही न्यायिक क्रियाशीलता को जन्म देती है। न्यायिक क्रियाशीलता के साधन (Means (Devices) of Judicial Activism)-न्यायिक क्रियाशीलता के साधन निम्नलिखित हैं

1. न्यायिक पुनर्निरीक्षण-न्यायिक पुनर्निरीक्षण शक्ति न्यायिक क्रियाशीलता का प्रथम व महत्त्वपूर्ण साधन है। भारतीय संविधान द्वारा न्यायिक पुनर्निरीक्षण की शक्ति भारतीय सर्वोच्च न्यायालय और उच्च न्यायालयों के पास है। न्यायिक पुनर्निरीक्षण की शक्ति के अधीन सर्वोच्च न्यायालयों व उच्च न्यायालयों को उन कानूनों व कार्यपालिका के आदेशों को रद्द करने का अधिकार है जो संविधान की इच्छा के विरुद्ध है। न्यायपालिका न्यायिक पुनर्निरीक्षण से ऐसे सिद्धान्त निश्चित करती है जो सरकार के अन्य अंगों पर बाध्यकारी होते हैं। इस तरह न्यायिक पुनर्निरीक्षण न्यायिक क्रियाशीलता का एक साधन है; जैसे 1973 का केशवानन्द भारती का मुकद्दमा।

2. मौलिक अधिकारों की व्याख्या-न्यायिक क्रियाशीलता का द्वितीय साधन मौलिक अधिकार की व्याख्या है। भारतीय संविधान में नागरिकों को मौलिक अधिकार प्रदान किए गए हैं तथा धारा 31 में संवैधानिक उपचारों का अधिकार प्राप्त है। इस अधिकार के अन्तर्गत प्रत्येक व्यक्ति मौलिक अधिकारों की अवहेलना होने पर न्यायालय का दरवाजा खटखटा सकता है और अपने-अपने अधिकारों की सुरक्षा कर सकता है।

न्यायपालिका अधिकारों की रक्षा करते समय कुछ ऐसे निर्देश जारी करती है जो न्यायिक क्रियाशीलता का उदाहरण होते हैं; जैसे 4 फरवरी, 1993 को सर्वोच्च न्यायालय ने निर्णय दिया कि 14 वर्ष की आयु तक प्राथमिक शिक्षा प्राप्त करना प्रत्येक भारतीय बालक का मौलिक अधिकार है। इसी तरह सर्वोच्च न्यायालय ने यह भी कहा कि उच्च स्तरीय शिक्षा प्राप्त करना नागरिक का मौलिक अधिकार है। इसमें सर्वोच्च न्यायालय संविधान धारा 21 की नई व्याख्या न्यायिक क्रियाशीलता का स्पष्ट प्रमाण है।

3. कानून निर्माण साधारणतः कानून निर्माण न्यायपालिका का कार्य नहीं है, परन्तु अनेक बार न्यायपालिका कानून निर्माण भी करती है जो न्यायिक क्रियाशीलता का स्पष्ट उदाहरण है; जैसे कई बार न्यायपालिका के समक्ष ऐसा विचित्र झगड़ा पेश होता है जिस पर विधानपालिका का कोई कानून नहीं तो न्यायाधीश अपनी सूझ-बूझ, अनुभव तथा योग्यता के आधार पर जो निर्णय कर देते हैं वह कानून का एक अंग बन जाता है। न्यायाधीशों द्वारा इस प्रकार दिए गए निर्णयों को न्यायाधीशों द्वारा निर्मित कानून (Judge Maker) या केस लॉ (Case Law) कहा जाता है ये निर्णय न्यायिक प्रमाण (Judicial Precedent) बन जाते हैं और अन्य न्यायालयों पर भी बाध्य होते हैं। ऐसे निर्णय न्यायपालिका की क्रियाशीलता के उदाहरण है।

4. संविधान की व्याख्या न्यायपालिका को संविधान का संरक्षक कहा जाता है। लिखित संविधान में संविधान की अनेक धाराओं से सम्बन्धित अंतिम व्याख्या न्यायपालिका का दायित्व होता है। संविधान की व्याख्या करते समय सर्वोच्च न्यायालय नवीन कानून सिद्धान्त, नियम व विधियाँ आदि निर्धारित करती हैजैसे 6 अक्तूबर, 1998 को सर्वोच्च न्यायालय ने एक निर्णय किया कि न्यायाधीशों की नियुक्ति में न्यायपालिका को कार्यपालिका की तुलना में प्राथमिकता दी जाएगी। अतः संविधान की व्याख्या न्यायिक क्रियाशीलता का एक अन्य साधन है।

5. सार्वजनिक हित के लिए मुकद्दमेबाजी-सार्वजनिक हित के लिए मुकद्दमेबाजी न्यायिक क्रियाशीलता का महत्त्वपूर्ण उदाहरण है। साधारणतः कानून के अन्तर्गत के केवल प्रभावित व्यक्ति ही न्यायालय में मुकद्दमा कर सकते हैं। दूसरे किसी व्यक्ति को मुकद्दमा दायर करने का अधिकार नहीं है, परन्तु सार्वजनिक हित के मुकद्दमेबाजी में कोई व्यक्ति सार्वजनिक विषय के सम्बन्ध में न्यायालय में मुकद्दमा दायर कर सकता है तथा उस मुकद्दमे से उस व्यक्ति का सम्बन्धित होना आवश्यक नहीं है।

भारतीय संविधान का अनुच्छेद 39 (क) वह स्रोत है जो सार्वजनिक हित के विवादों की अवधारणा को पर्याप्त सीमा तक मान्यता प्रदान करता है। सर्वोच्च न्यायालय ने सार्वजनिक हित के न्याय के लिए बहुत ही महत्त्वपूर्ण कदम उठाए हैं। यदि कोई व्यक्ति पोस्ट कार्ड पर भी आवेदन-पत्र लिखकर या अन्याय की शिकायत सर्वोच्च न्यायालय में करता है तो शिकायत दर्ज हो जाती है; जैसे कमजोर वर्ग, मज़दूरों, स्त्रियों व बच्चों की शिकायतों को विशेष महत्त्व दिया जाता है।

निष्कर्ष-भारत में न्यायिक क्रियाशीलता दिन-प्रतिदिन बढ़ती जा रही है। कुछ इसके पक्ष में और अन्य इसके विपक्ष में हैं। भारत में न्यायिक क्रियाशीलता का महत्त्वपूर्ण साधन सार्वजनिक हित के सम्बन्ध में मुकद्दमेबाजी है। इसमें न्यायालय कार्यपालिका सरकार को या संस्थाओं को कुछ ऐसे आदेश या निर्देश देती है जो उन्हें मानने पड़ते हैं न मानने पर न्यायपालिका की अवहेलना होने का डर है। न्यायिक क्रियाशीलता में असंख्य पीड़ित बालकों, स्त्रियों व श्रमिकों को लाभ हुआ है। न्यायिक क्रियाशीलता से समाज की कई बुराइयों का अन्त भी हुआ है; जैसे बन्धुआ मजदूरी का अन्त आदि।

प्रश्न 8.
न्यायपालिका की सक्रियता के कारणों या तत्त्वों का वर्णन करो।
उत्तर:
न्यायिक क्रियाशीलता के तत्त्व या कारण (Causes or Elements of Judicial Activism)-संवैधानिक रूप में न्यायिक क्रियाशीलता का केवल एक ही तत्त्व-न्यायिक पुनर्निरीक्षण है, परन्तु इसके अतिरिक्त कुछ अन्य तत्त्व भी हैं जिन्होंने न्यायिक क्रियाशीलता को बढ़ावा दिया है। ऐसे तत्त्वों का विवरण निम्नलिखित है

1. यदि प्रत्येक व्यक्ति या विभाग अपने दायित्वों को निभाता रहे तो दूसरे के हस्तक्षेप की आवश्यकता नहीं। किसी दूसरों के मामलों में हस्तक्षेप की आवश्यकता तब अवश्य हो जाती है जब कोई अपने दायित्वों को नहीं निभाता। ऐसा न्यायपालिका की स्थिति में हुआ है। न्यायपालिका ने कार्यपालिका के कार्यों में हस्तक्षेप उसकी कर्त्तव्य विमुखता और पथ भ्रमिता के कारण किया। कार्यपालिका न केवल केन्द्रीय स्तर पर, बल्कि राज्य स्तर पर भी अपने कर्तव्यों को निभाने में असफल रही है। कार्यपालिका के इस कर्त्तव्य न निभाने की प्रवृत्ति ने लोगों को हताश कर दिया है।

अतः न्यायक्रियाशीलता का प्रारम्भ हुआ। कार्यपालिका की अक्रियाशीलता के कारण न्यायपालिका की क्रियाशीलता के लिए यह उदाहरण दिए जा सकते हैं। जब सन् 1994 में उत्तर प्रदेश (उत्तराखण्ड) में प्रदर्शनकारी अपनी मांगों को मनवाने के लिए आन्दोलन कर रहे थे तो आन्दोलन के दौरान प्रशासन द्वारा उन पर अत्याचार किए गए। यहाँ तक कि महिलाओं के साथ बलात्कार की घटनाएँ भी घटीं। परन्तु राज्य व केन्द्रीय सरकार ने इस पर कोई कार्रवाई नहीं की, तब इलाहाबाद हाईकोर्ट ने इस सम्बन्ध में जाँच के आदेश दिए। इसी प्रकार बिहार में चारा घोटाला के मामले में भी राज्य सरकार को वहाँ के उच्च न्यायालय ने ही जाँच के आदेश दिए। अतः स्पष्ट है कि कार्यपालिका की अनदेखी अक्रियाशीलता ही न्यायिक क्रियाशीलता का महत्त्वपूर्ण तत्त्व है।

2. जनहित मुकद्दमे-जनहित के मुकद्दमे भी न्यायिक क्रियाशीलता का महत्त्वपूर्ण तत्त्व है। साधारणतः व्यक्ति अपने हित की रक्षा हेतु ही मुकद्दमा कर सकता है, किसी अन्य व्यक्ति की रक्षा के लिए नहीं। इस प्रकार कोई व्यक्ति सार्वजनिक हित की प्राप्ति के लिए कोई मुकद्दमा नहीं लड़ सकता। परन्तु हाल ही में जनहित के मुकद्दमों की धारणा सामने आई है।

जिसके अनुसार यदि कोई व्यक्ति या वर्ग किसी कारण से हितों या अधिकारों की रक्षा नहीं कर सकता तो किसी भी व्यक्ति या संस्था को उनके अधिकारों या हितों की रक्षा के लिए मुकद्दमा डालने का अधिकार है। जनहित की पद्धति का आरम्भ न्यायमूर्ति श्री० पी०एन० भगवती ने किया इस पद्धति ने गरीब, अनपढ़ और अनजान लोगों के लिए न्याय का रास्ता खोला है। अतः स्पष्ट है कि जनहित के मुकद्दमों ने न्यायिक क्रियाशीलता को बढ़ावा दिया है। ताजमहल को प्रदूषण से बचाना, शहरी आवास आबंटन घोटाला, बिहार (भागलपुर) क्षेत्र के मामले, श्रमिकों के शोषण के मामले आदि जनहित मुकद्दमों के कतिपय उदाहरण हैं।

3. स्वतन्त्र न्यायपालिका-न्यायपालिका की स्वतन्त्रता भी न्यायिक क्रियाशीलता का महत्त्वपूर्ण तत्त्व या कारण है। स्वतन्त्र न्यायपालिका का अर्थ है कि न्यायपालिका पर कोई आन्तरिक व बाह्य दबाव नहीं होता। न्यायपालिका निर्भीकता से कार्य करती है। इससे न्यायपालिका की कार्यशीलता बढ़ती है। इस प्रकार न्यायपालिका स्वतन्त्र रूप से किसी भी बड़े-से-बड़े पदाधिकारी के विरुद्ध कार्रवाई कर सकती है। अतः स्पष्ट है कि कर्त्तव्यपरायण, निर्भीक और प्रखर न्यायाधीश ही न्यायपालिका को सक्रियता प्रदान कर सकते हैं।

4. जनता का कार्यपालिका के प्रति घटता विश्वास जनता के कार्यपालिका के प्रति घटते विश्वास ने भी न्यायिक क्रियाशीलता को जन्म ही नहीं दिया, बल्कि उसको बढ़ावा भी दिया है। पिछले दशक में न्यायिक अस्थिरता ने कार्यपालिका के प्रति जनता के विश्वास को घटाया है और वह निरन्तर घटता ही जा रहा है। इसके विपरीत न्यायपालिका ने कार्यपालिका को उसकी कर्तव्य विमुखता उदासीनता के लिए चेतावनी दी है जिससे आम जनता में न्यायपालिका के प्रति विश्वास बढ़ा है और आम जनता विभिन्न प्रकार के प्रशासनिक सुधारों के लिए न्यायपालिका से आशा लगाए बैठी है। यही नहीं न्यायपालिका ने जनता के दुःख-दर्द को पहचाना है और उसे दूर करने के लिए अपने दायित्व को निभाया है। अतः इससे न्यायिक क्रियाशीलता का बढ़ना स्वाभावि न्यायिक पनर्निरीक्षण की शक्ति-न्यायिक क्रियाशीलता का एक और महत्त्वपूर्ण कारण है-न्यायिक पुनर्निरीक्षण की शक्ति।

5. भारतीय संविधान के द्वारा सर्वोच्च न्यायालय व उच्च न्यायालय को न्यायिक पुनर्निरीक्षण की शक्ति प्रदान की गई है। इस शक्ति के अन्तर्गत सर्वोच्च न्यायालय और उच्च न्यायालय संविधान के विरुद्ध बने कानूनों को रद्द कर सकती है। इससे न्यायालय को अपनी शक्तियों का प्रयोग करने का अवसर मिलता है। इसके अतिरिक्त भारत में संघीय व्यवस्था को अपनाया गया है। इस व्यवस्था में भी न्यायपालिका को अपनी शक्तियों का प्रयोग करने का सही अवसर प्राप्त होता है। इस प्रकार अनेक मामलों में न्यायपालिका ने संविधान की व संघात्मक व्यवस्था की रक्षा के लिए अपने अधिकारों का प्रयोग करके न्यायिक क्रियाशीलता का प्रदर्शन किया है।

6. राजनीतिक अस्थिरता राजनीतिक अस्थिरता भी न्यायिक क्रियाशीलता या न्यायिक सक्रियता का महत्त्वपूर्ण कारण है। कभी एक समान या संसद में एक दल को बहुमत प्राप्त हो जाता था और सरकार स्थायी व सुचारू रूप से कार्य करती थी, परन्तु पिछले कुछ वर्षों से संसद में किसी भी दल को स्पष्ट बहुमत प्राप्त नहीं हुआ है। परिणामस्वरूप साँझा सरकारें बनी हैं। साँझा सरकार में निश्चित रूप से सरकार की गुणात्मक शक्ति को धक्का लगा है। संसद की क्रियात्मक शक्तियों का ह्रास हुआ है। अतः संसद की शक्ति कमजोर होने से न्यायपालिका की शक्तियों को बढ़ावा मिला है। जिसे न्यायिक क्रियाशीलता ही कहा जा सकता है।

7. राजनीतिक नेताओं की छलिया राजनीति राजनीतिक नेताओं की छलिया राजनीति ने भी न्यायिक सक्रियता को जन्म दिया है। राजनीतिक नेताओं की कपटी गतिविधियों के कारण न्यायपालिका को हस्तक्षेप करना पड़ता है। कई बार तो न्यायपालिका को न चाहते हुए भी क्रियाशील होना पड़ता है। आजकल राजनीतिक नेता जिन मामलों को स्वयं सुलझा नहीं सकते हैं और जिन मामलों को सुलझाने में बुराई या भलाई दोनों मिल सकती है तो

इसलिए वे बुराई मिलने के भय से उस मामले को सुलझाने का दायित्व न्यायपालिका पर छोड़ देते हैं। जिससे राजनीतिक नेता साफ बच जाते हैं। अतः स्पष्ट है कि अपने स्वार्थों की पूर्ति के लिए न्यायपालिका को सक्रिय होने पर विवश कर देते हैं। उपर्यक्त विवेचन से यह सिद्ध होता है कि न्यायिक क्रियाशीलता के कई कारण हैं और इन कारणों या तत्त्वों ने न्यायपालिका को सक्रिय होने में अहम भूमिका निभाई है। यहाँ यह उल्लेखनीय है कि वर्तमान राजनीतिक परिदृश्य में इस प्रकार के परिवर्तन हो रहे हैं, जिनसे न्यायिक क्रियाशीलता के बढ़ने की अधिक सम्भावना है।

प्रश्न 9.
न्यायिक सक्रियता से आप क्या समझते हैं? इसके पक्ष व विपक्ष में तर्क दीजिए। अथवा न्यायिक क्रियाशीलता के पक्ष व विपक्ष में तर्क दीजिए। अथवा न्यायिक क्रियाशीलता की आलोचनात्मक व्याख्या कीजिए।
उत्तर:
सामान्यतः संसदीय जनतन्त्र में संसद को सर्वोच्च स्थान प्राप्त होता है किन्तु भारत में यही बात संविधान के ऊपर लागू होती है। यहाँ संविधान को सर्वोच्च स्थान प्राप्त है और न्यायपालिका अपने न्यायिक पुनरावलोकन के अधिकार के बल पर संसद से उच्च सिद्ध हो जाती है। हालाँकि भारतीय संविधान में कहीं भी न्यायिक पुनरावलोकन का अलग से कोई उल्लेख नहीं किया गया है।

किन्तु वहीं पर इस बात का उल्लेख अवश्य मिलता है कि यदि कोई भी क्रिया-कलाप मौलिक अधिकारों का उल्लंघन करता है तो उसे संवैधानिक घोषित किया जा सकता है और इस घोषणा को करने का अधिकार न्यायिक पुनरावलोकन के तहत न्यायपालिका को प्राप्त है।

आरम्भ में न्यायपालिका का यह अधिकार सिर्फ इस बात की समीक्षा तक ही सीमित था कि कार्यपालिका अथवा विधायिका द्वारा लिए गए किसी निर्णय अथवा उठाए गए किसी कदम से कहीं मौलिक अधिकारों का हनन तो नहीं हो रहा है किन्तु धीरे-धीरे न्यायपालिका ने अपने इस अधिकार का विस्तार करना शुरु कर दिया और सामाजिक मुद्दों से जुड़े सवालों, पर्यावरण सम्बन्धी समस्याओं तथा जनताँत्रिक संगठनों के क्रिया-कलापों की भी समीक्षा करनी प्रारम्भ कर दी। न्यायपालिका की इसी सक्रियता को न्यायिक सक्रियतावाद कहा जाता है।

न्यायिक क्रियाशीलता के विपक्ष में तर्क (Arguments in Against of Judicial Activism)-न्यायिक क्रियाशीलता की आलोचना निम्नलिखित आधारों पर की जा सकती है

1. यह शक्ति पृथक्करण के सिद्धान्त के विरुद्ध है-शक्ति पृथक्करण के सिद्धान्त के अनुसार सरकार के तीनों अंग अपने क्षेत्र में स्वतन्त्र रूप से कार्य करते हैं। विधानपालिका कानून बनाने, कार्यपालिका कानूनों को लागू करने तथा न्यायपालिका निर्णय देने का कार्य करती है। किसी भी अंग को दूसरे अंग के क्षेत्र में हस्तक्षेप करने का अधिकार नहीं होता तथा हस्तक्षेप असवैध माना जाता है।

संविधान में न्यायपालिका को एक सीमा तक विधानपालिका के कार्यों में हस्तक्षेप करने का अधिकार प्राप्त है, यदि विधानपालिका संविधान की इच्छा के विरुद्ध कानून बनाती है, तब न्यायपालिका को ऐसे कानूनों को अवैध घोषित करने का अधिकार प्राप्त है। परन्तु न्यायिक क्रियाशीलता में न्यायपालिका अपने अधिकार-क्षेत्र से बाहर जाकर विधानपालिका और कार्यपालिका के क्षेत्र में हस्तक्षेप करती है। परिणामस्वरूप सरकार के सुचारू रूप से चलने में बाधा उत्पन्न होती है, अतः न्यायिक क्रियाशीलता शक्ति पृथक्करण के सिद्धान्त के विरुद्ध है।

2. उत्तरदायित्व के सिद्धान्त के विरुद्ध न्यायिक क्रियाशीलता की आलोचना उत्तरदायित्व की अवधारणा के विरुद्ध है। उत्तरदायित्व की अवधारणा का अर्थ है कि किसी का किसी के प्रति उत्तरदायी होना। कार्यपालिका और विधानपालिका दोनों लोगों के प्रति उत्तरदायी है। लोग विधानपालिका और कार्यपालिका दोनों का समय-समय पर मूल्याँकन करते रहते हैं; जैसे मन्त्रिमण्डल लोगों द्वारा निर्वाचित संसद के प्रति अपने क्रियाकलापों के लिए उत्तरदायी है।

मन्त्रिमण्डल तभी तक सत्ता में रह सकता है जब तक उसे संसद का विश्वास प्राप्त होता है। इसी प्रकार विधानपालिका अर्थात् संसद के सदस्य भी जनता के प्रति उत्तरदायी हैं। जनता ऐसे सांसदों का दुबारा निर्वाचित नहीं करती जो उनकी कसौटी पर खरे नहीं उतरते अर्थात् मन्त्रियों और सांसदों के लिए दोबारा चुनाव जीतना कठिन हो जाता है, जो लोगों की आशाओं पर पूर्ण नहीं उतरते हैं, परन्तु न्यायपालिका विधानपालिका या कार्यपालिका के प्रति उत्तरदायी नहीं है।

यहाँ तक न्यायाधीश भी जनता के प्रति उत्तरदायी नहीं है। न्यायपालिका न्यायिक क्रियाशीलता में किसी भी तरह से उत्तरदायी नहीं। इस प्रकार अनुत्तरदायित्व की भावना से न्यायपालिका अपने अधिकार-क्षेत्र से बाहर जाकर कार्य कर लेती है वैसे भी एक अनुत्तरदायी संस्था उत्तरदायी संस्थाओं को आदेश दे, यह अनुचित है।

3. संसदीय व्यवस्था के विरुद्ध न्यायिक क्रियाशीलता को संसदीय व्यवस्था के विरुद्ध मानकर उसकी आलोचना की जाती है। संसदीय व्यवस्था में कानून बनाने का कार्य विधानपालिका करती है। न्यायपालिका को संविधान द्वारा केवल यह अधिकार दिया गया है कि विधानपालिका द्वारा संविधान के विरुद्ध बनाए गए कानूनों को अवैध घोषित करें।

न्यायपालिका को कानून बनाने का अधिकार प्राप्त नहीं है लेकिन न्यायिक क्रियाशीलता के अन्तर्गत न्यायपालिका ने तो संसद को कानून बनाने के आदेश देने आरम्भ कर दिए हैं। न्यायपालिका विधानपालिका का तीसरा सदन बनने का प्रयत्न कर रही है जो संसदीय व्यवस्था की आत्मा के विरुद्ध है अर्थात न्यायिक क्रियाशीलता ससंदीय लोकतन्त्र की कार्य-प्रणाली पर बुरा प्रभाव डालेगी।

4. न्यायपालिका के कार्यों पर कुप्रभाव न्यायिक क्रियाशीलता की इस आधार पर भी आलोचना की गई है कि यह न्यायपालिका की दैनिक कार्य-प्रणाली पर बुरा प्रभाव डालती है। न्यायिक क्रियाशीलता सार्वजनिक हित के मुकद्दमेबाजी को प्रोत्साहन देती है। न्यायिक क्रियाशीलता से न्यायपालिका ने सार्वजनिक हित के मुकद्दमों का अम्बार लग जाएगा, न्यायालय या उच्च न्यायालय में सार्वजनिक हित के मुकद्दमे को ले जाना आसान हो गया है।

इस प्रकार यह उच्च न्यायालय या सर्वोच्च न्यायालय के कार्यों में वृद्धि कर देगी, जबकि उच्च और सर्वोच्च न्यायालय पहले से ही अधिक काम के बोझ के नीचे दबे हुए हैं। आज की तिथि में भी लाखों मुकद्दमें न्यायालयों में पड़े हुए हैं जिनको समय के अभाव के कारण से निपटाया नहीं जा सका है। न्यायिक क्रियाशीलता न्यायपालिका के दैनिक कार्यों को भी और बढ़ा देगी जिससे न्यायपालिका की कार्य-प्रणाली पर बुरा प्रभाव पड़ना स्वाभाविक है।

5. प्रजातन्त्र के विरुद्ध-न्यायिक क्रियाशीलता को प्रजातन्त्र के विरुद्ध माना जाता है। प्रजातन्त्र में जनता अपने प्रतिनिधियों को चुनती है और उन प्रतिनिधियों के द्वारा प्रशासन चलाया जाता है तथा ये निर्वाचित प्रतिनिधि जनता के प्रति उत्तरदायी होते हैं। साथ ही प्रशासन एक कला है जिसे केवल प्रशासनिक कला में माहिर लोगों के द्वारा ही चलाया जा सकता है। माना न्यायाधीश द्धजीवी होते हैं परन्तु वे कानून के क्षेत्र में ही माहिर होते हैं। प्रशासन से उनका कुछ लेना-देना नहीं होता।

इसलिए यदि न्यायाधीश न्यायिक क्रियाशीलता के अन्तर्गत प्रशासन में हस्तक्षेप करते हैं तो यह उचित नहीं होगा और साथ में यह प्रजातन्त्र विरोधी भी होगा क्योंकि प्रशासन चलाना प्रशासकों का काम है न कि न्यायाधीशों का। हाँ यदि प्रशासन दोषपूर्ण है तो कानून द्वारा उसमें सुधार करवाने के लिए न्यायपालिका अवश्य ही सकारात्मक भूमिका निभा सकती है परन्तु न्यायिक क्रियाशीलता द्वारा शासन को अपने हाथों में लेना निश्चित रूप से प्रजातन्त्र के विरुद्ध है।

6. न्यायिक निरंकुशता को प्रोत्साहन-न्यायिक क्रियाशीलता की इस आधार पर भी आलोचना की जा सकती है कि यह न्यायिक निरंकुशता को प्रोत्साहित करती है। न्यायिक क्रियाशीलता से न्यायपालिका की निरंकुशता स्थापित होने का भय बना रहता है। न्यायिक क्रियाशीलता यहाँ तक तो सही है कि यह विधानपालिका और कार्यपालिका को अपने-अपने कर्तव्यों का पालन करने के लिए आगाह करती है।

परन्तु एक महत्त्वपूर्ण प्रश्न यह उठता है कि यदि न्यायपालिका अपने कर्तव्यों का पालन नहीं करती तो उसे कौन अपने कर्तव्यों का पालन करने के लिए कहेगा। न्यायपालिका के कार्यों को कौन असंवैधानिक घोषित करेगा अर्थात् न्यायपालिका पर कोई अंकुश नहीं है और उसके निरंकुश बनने की पूरी सम्भावना है। अतः न्यायपालिका की निरंकुशता भी अन्य निरंकुशताओं की तरह होगी, जिसमें लोगों के अधिकार छीन लिए जाएँगें परन्तु लोग न्यायपालिका की निरंकुशता को सहन नहीं करेंगे और उसके विरुद्ध विद्रोह कर बैठेंगे।

न्यायिक क्रियाशीलता के पक्ष में तर्क (Arguments in favour of Judicial Activism)-न्यायिक क्रियाशीलता के विपक्ष . में तर्क देखने के पश्चात् ऐसा लगता है कि न्यायिक क्रियाशीलता अनुचित है परन्तु न्यायिक क्रियाशीलता ने भारतीय राजनीति को कई प्रकार से प्रभावित किया है इसलिए न्यायिक क्रियाशीलता के पक्ष में कुछ तर्क दिए जा सकते हैं। इन तर्कों का विवरण निम्नलिखित है

1. सरकार को क्रियाशील बनाने के लिए आवश्यक न्यायिक क्रियाशीलता के पक्ष में प्रथम और महत्त्वपूर्ण तर्क यह दिया जाता है कि यह सरकार के विभिन्न अंगों को सक्रिय बनाने में सहायता करती है। साधारणतः यह देखा गया है कि सरकार के अंग विधानपालिका और कार्यपालिका अपने कर्तव्यों के प्रति विमुख हो जाते हैं।

भारत का अब तक का इतिहास इस बात का गवाह है कि कार्यपालिका, विधानपालिका व अन्य संस्थाओं ने अपने कर्तव्यों की ओर से मुँह मोड़ लिया था। वह अपने उत्तरदायित्व को ईमानदारी से नहीं निभा रहे थे। काफी समय तक न्यायपालिका ने इस क्षेत्र में कोई हस्तक्षेप नहीं किया परन्तु कुछ वर्षों से न्यायपालिका ने इस अक्रियाशीलता के क्षेत्र में हस्तक्षेप किया है और उसके हस्तक्षेप से बहुत से घोटाले सामने आए हैं। यही नहीं कुछ भ्रष्ट राजनीतिज्ञों के विरुद्ध कार्रवाई भी की गई है। अतः स्पष्ट है कि न्यायिक क्रियाशीलता से सरकार की क्रियाशीलता को सक्रिय बनाया जा सकता है।

2. राजनीतिक प्रणाली की सही स्थिति जानने में सहायक-न्यायिक क्रियाशीलता के पक्ष में एक और तर्क यह दिया जाता है कि यह देश की राजनीतिक प्रणाली की क्रियाशीलता को जानने में या उसका सही रूप पहचानने में सहायता करती है। न्यायपालिका स्वतन्त्र होती है। न्यायपालिका पर कार्यपालिका या विधानपालिका का किसी भी प्रकार से नियन्त्रण नहीं होता।

इसलिए न्यायपालिका कार्यपालिका और विधानपालिका को निर्देश दे सकती है और उन्हें इसके निर्देशों का पालन करना पड़ता है। न्यायपालिका की क्रियाशीलता से अनेक घोटालों का पर्दाफाश हुआ है। भ्रष्ट राजनीतिज्ञों के भ्रष्टाचार सामने आए हैं। न्यायपालिका की क्रियाशीलता के कारण ही लोगों को हैरान कर देने वाली बातों का पता लगा है। अतः स्पष्ट है कि न्यायिक क्रियाशीलता की वजह से ही सरकार की अक्रियाशीलता और राजनीतिक प्रणाली का सही रूप देखने को मिलता है।

3. संविधान की सुरक्षा-न्यायिक कार्यशीलता के पक्ष में एक अन्य महत्त्वपूर्ण तर्क यह है कि यह संविधान की सुरक्षा करती है। यह कानून के शासन को स्थापित करने में सहायता प्रदान करती है। संविधान में विधानपालिका को अधिकार-क्षेत्र का वर्णन होता है और उसके लिए यह अपेक्षा की जाती है कि वह अपने-अपने क्षेत्राधिकार में रहकर कार्य करें।

यदि कार्यपालिका या विधानपालिका अपने अधिकार-क्षेत्र की अवहेलना करती है तो न्यायपालिका उनके ऐसे कार्यों को अवैध घोषित करके न्यायिक क्रियाशीलता का परिचय देती है। अतः स्पष्ट है कि कार्यपालिका और विधानपालिका द्वारा अपने कर्तव्यों का पालन करने की स्थिति में न्यायपालिका निर्देश दे सकती है। इसलिए यह कहा जा सकता है कि कानून के शासन व संविधान की रक्षा के लिए न्यायिक क्रियाशीलता की आवश्यकता है।

4. कानूनों की व्याख्या में सहायक-न्यायिक क्रियाशीलता के पक्ष में एक और तर्क यह भी दिया जाता है कि इससे कानूनों की व्याख्या करने में सहायता मिलती है। यही नहीं कई बार क्रियाशील न्यायपालिका नए कानून बनाने और पुराने कानूनों की त्रुटियों को दूर करने में भी सहायता करती है। कानून साधारणतः विधानपालिका के द्वारा बनाए जाते हैं। विधानपालिका के सदस्य कानून-वेता नहीं होते, बल्कि साधारण व्यक्ति होते हैं जिनके द्वारा अस्पष्ट कानूनों का निर्माण किया जा सकता है।

कई बार कानूनों की भाषा इस प्रकार की होती है कि उसके एक में से अधिक अर्थ निकलते हैं। ऐसी स्थिति में क्रियाशील न्यायपालिका ही कानूनों की सही व्याख्या करती है। यह कानूनों की त्रुटियों को दूर करने में सहायता करती है। क्रियाशील न्यायपालिका नवीन धारणाओं और कानूनों का जन्म देती है। अतः न्यायिक क्रियाशील के अभाव में कानूनों की त्रुटियों को दूर नहीं किया जा सकता।

5. भावी कानूनों के निर्माण में सहायक निःसन्देह विधानपालिका मनुष्यों की आवश्यकताओं को ध्यान में रखकर कानूनों का निर्माण करती है, परन्तु समय बदलता रहता है। कई बार जीवन में विचित्र प्रकार की परिस्थितियाँ उत्पन्न हो जाती हैं और उन परिस्थितियों से निपटने के लिए कोई कानून नहीं होता। इसका अर्थ यह लगाना चाहिए जब तक कोई कानून नहीं बनेगा, तब तक कोई निर्णय होगा।

ऐसी स्थिति में न्यायाधीश अपनी बुद्धिमता के अनुसार निर्णय देता है। वे अपनी न्याय भावना का प्रयोग करके अमुक मुकद्दमे का फैसला कर देते हैं। अतः स्पष्ट है कि जिस क्षेत्र में किसी प्रकार कानून बना नहीं होता। वहाँ न्यायिक क्रियाशीलता की सहायता से मामले को निपटा दिया जाता है। इस प्रकार न्यायिक क्रियाशीलता निर्णय करने में सहायता प्रदान करती है।

6. मानवीय अधिकारों की रक्षा में सहायक-न्यायिक क्रियाशीलता ने मानवीय अधिकारों की रक्षा में महत्त्वपूर्ण योगदान दिया है। न्यायिक क्रियाशीलता के अभाव में गैर-कानूनी नज़रबन्दी बच्चों के शोषण के मामले, स्त्रियों के शोषण के मामले, स्त्रियों के अनैतिक व्यापार के मामले आदि जनता के सामने नहीं आते। यद्यपि भारतीय संविधान में लोगों को अपने अधिकारों की रक्षा का अधिकार प्राप्त है।

वे अधिकारों की अवहेलना की स्थिति में न्यायपालिका का दरवाजा खटखटा सकते हैं, परन्तु कितने लोग ऐसा करते हैं? बहुत कम! क्योंकि उनके पास इतने साधन नहीं होते। धन के अभाव तथा जानकारी के अभाव में लोगों के अधिकारों का लगातार हनन होता रहता है। लेकिन न्यायिक क्रियाशीलता ने सार्वजनिक हित की मुकद्दमेबाजी (Public Interest Litigation) को जन्म दिया है जिसके द्वारा आम जनता के अधिकार की या मानवाधिकारों की रक्षा होती है।

7. लोगों का समर्थन-न्यायिक क्रियाशीलता के पक्ष में अन्तिम तर्क यदि दिया जा सकता है कि इसे जनता का समर्थन प्राप्त हुआ है। लोगों ने हार्दिक दिल से न्यायिक क्रियाशीलता का समर्थन किया है। क्योंकि इसके कारण बहुत से घोटाले, भ्रष्ट राजनीतिज्ञों के कारनामें, उच्च अधिकारियों की वास्तविक तस्वीर सामने आई है। आम जनता कार्यपालिका व नौकरशाही की लाल फीताशाही से तंग आ चुकी थी। केवल न्यायिक क्रियाशीलता ने लोगों को एक निष्पक्षता की किरण दिखाई। जनता ने यह समझ लिया है कि न्यायपालिका की क्रियाशीलता ही उन्हें भ्रष्ट राजनीतिज्ञों और अधिकारियों से छुटकारा दिला सकती है।

निष्कर्ष-न्यायिक क्रियाशीलता के पक्ष व विपक्ष के तकों का अध्ययन करने से यह स्पष्ट हो जाता है कि भारत जैसे विकासशील देश में न्यायिक क्रियाशीलता की आवश्यकता है। भारत के विगत वर्षों के प्रशासन ने यह साबित कर दिया है कि भारत में नौकरशाही व भ्रष्टाचार को बढ़ावा मिला। राजनीतिक नेताओं ने जनता की सहायता करने की बजाय अपने घरों को भरने का प्रयत्न किया है।

अतः वर्तमान स्थिति में न्यायिक क्रियाशीलता की अत्यन्त आवश्यकता है ताकि बढ़ते भ्रष्टाचार व नौकरशाही पर रोक लगाई जा सके, परन्तु जहाँ जरूरत हो वहाँ न्यायिक क्रियाशीलता पर रोक की भी आवश्यकता है। न्यायपालिका को केवल सरकार के दूसरे अंगों को क्रियाशील बनाने में आवश्यक रूप से हस्तक्षेप करना चाहिए परन्तु स्वयं अधिक क्रियाशील नहीं होना चाहिए, क्योंकि इस प्रकार की न्यायिक क्रियाशीलता न्यायपालिका की निरंकुशता को जन्म दे सकती है तथा निरंकुशता तो किसी की भी क्यों न हो, अच्छी नहीं होती।

वस्तुनिष्ठ प्रश्न

निम्नलिखित प्रश्नों का उत्तर दिए गए विकल्पों में से उचित विकल्प छाँटकर लिखें

1. निम्नलिखित में से न्यायपालिका का कार्य है
(A) कानूनों को लागू करना
(B) चोरों को पकड़ना
(C) खेती को बढ़ावा देना
(D) झगड़ों का निपटारा करना
उत्तर:
(D) झगड़ों का निपटारा करना

2. निम्नलिखित में से न्यायपालिका का कार्य नहीं है
(A) मुकद्दमों का फैसला करना
(B) अन्य देशों के साथ सम्बन्ध स्थापित करना
(C) कानूनों की व्याख्या करना
(D) नागरिकों के अधिकारों की रक्षा करना
उत्तर:
(B) अन्य देशों के साथ सम्बन्ध स्थापित करना

3. न्यायपालिका की स्वतंत्रता आवश्यक है
(A) लोकतंत्र की रक्षा के लिए
(B) संविधान के संरक्षण के लिए
(C) नागरिकों के अधिकारों की रक्षा के लिए
(D) उपर्युक्त सभी
उत्तर:
(D) उपर्युक्त सभी

HBSE 11th Class Political Science Important Questions Chapter 6 न्यायपालिका

4. निम्नलिखित न्यायपालिका की स्वतंत्रता का साधन है
(A) न्यायाधीशों की नियुक्ति का उचित तरीका
(B) अच्छे वेतन
(C) न्यायाधीशों को पद से हटाने का कठिन तरीका
(D) उपर्युक्त सभी
उत्तर:
(D) उपर्युक्त सभी

5. सर्वोच्च न्यायालय के न्यायाधीशों की कुल संख्या कितनी है?
(A) 30
(B) 26
(C) 31
(D) 34
उत्तर:
(D) 34

6. सर्वोच्च न्यायालय के न्यायाधीशों की नियुक्ति करता है
(A) प्रधानमन्त्री
(B) उप-राष्ट्रपति
(C) राष्ट्रपति
(D) इनमें से कोई नहीं
उत्तर:
(C) राष्ट्रपति

7. उच्चतम न्यायालय के मुख्य न्यायाधीश अपने पद पर रह सकते हैं
(A) 62 वर्ष
(B) 65 वर्ष
(C) 70 वर्ष
(D) 60 वर्ष
उत्तर:
(B) 65 वर्ष

8. सर्वोच्च न्यायालय के मुख्य न्यायाधीश को मासिक वेतन मिलता है
(A) ₹ 1,00,000
(B) ₹ 2,50,000
(C) ₹ 2,80,000
(D) ₹ 1,25,000
उत्तर:
(C) ₹ 2,80,000

9. सर्वोच्च न्यायालय के अन्य न्यायाधीशों को मासिक वेतन मिलता है
(A) ₹ 1,00,000
(B) ₹ 90,000
(C) ₹ 1,50,000
(D) ₹2,50,000
उत्तर:
(D) ₹ 2,50,000

10. राष्ट्रपति सर्वोच्च न्यायालय का न्यायाधीश नियुक्त करता है
(A) लोकसभा अध्यक्ष को
(B) राज्यसभा के अध्यक्ष को
(C) किसी प्रसिद्ध राजनीतिज्ञ को
(D) किसी प्रसिद्ध विधिवेत्ता को
उत्तर:
(D) किसी प्रसिद्ध विधिवेत्ता को

11. सर्वोच्च न्यायालय से किसी मामले सम्बन्धी कानूनी परामर्श ले सकता है
(A) प्रधानमन्त्री
(B) राष्ट्रपति
(C) उप-राष्ट्रपति
(D) लोकसभा अध्यक्ष
उत्तर:
(B) राष्ट्रपति

12. भारतीय नागरिकों के मौलिक अधिकारों का अन्तिम रक्षक कौन है?
(A) राष्ट्रपति
(B) उप-राष्ट्रपति
(C) संसद
(D) सर्वोच्च न्यायालय
उत्तर:
(D) सर्वोच्च न्यायालय

13. भारत के सर्वोच्च न्यायालय की व्यवस्था की गई है
(A) संसद द्वारा
(B) संविधान द्वारा
(C) राष्ट्रपति के आदेश द्वारा
(D) इनमें से कोई नहीं
उत्तर:
(B) संविधान द्वारा

14. भारत का सर्वोच्च न्यायालय किस स्थान पर स्थित है?
(A) मुम्बई में
(B) चंडीगढ़ में
(C) नई दिल्ली में
(D) चेन्नई में
उत्तर:
(C) नई दिल्ली में

15. सर्वोच्च न्यायालय के न्यायाधीश को पद से किस प्रकार हटाया जा सकता है?
(A) राष्ट्रपति के आदेश द्वारा
(B) प्रधानमंत्री की सलाह पर राष्ट्रपति द्वारा
(C) सर्वोच्च न्यायालय के मुख्य न्यायाधीश द्वारा
(D) संसद के दोनों सदनों के महाभियोग द्वारा
उत्तर:
(D) संसद के दोनों सदनों के महाभियोग द्वारा

16. राष्ट्रपति सर्वोच्च न्यायालय द्वारा दी गई सलाह को मानने के लिए
(A) बाध्य है
(B) बाध्य नहीं है
(C) सीमित रूप से बाध्य है
(D) इनमें से कोई नहीं
उत्तर:
(B) बाध्य नहीं है

17. निम्नलिखित में से कौन भारत में सर्वोच्च अपीलीय न्यायालय है?
(A) संसद
(B) राष्ट्रपति
(C) सर्वोच्च न्यायालय
(D) सर्वोच्च न्यायालय तथा उच्च न्यायालय
उत्तर:
(C) सर्वोच्च न्यायालय

18. उच्च न्यायालय के न्यायाधीश की नियुक्ति करता है
(A) राष्ट्रपति
(B) प्रधानमंत्री
(C) संसद
(D) मुख्य न्यायाधीश
उत्तर:
(A) राष्ट्रपति

19. उच्च न्यायालय के न्यायाधीश अपने पद पर रह सकते हैं
(A) 58 वर्ष तक
(B) 60 वर्ष तक
(C) 62 वर्ष तक
(D) 65 वर्ष तक
उत्तर:
(D) 65 वर्ष तक

20. उच्च न्यायालय के मुख्य न्यायाधीश को मासिक वेतन मिलता है
(A) ₹ 1,00,000
(B) ₹ 2,50,000
(C) ₹ 2,80,000
(D) ₹ 1,50,000
उत्तर:
(B) ₹ 2,50,000

21. उच्च न्यायालय के अन्य न्यायाधीशों को मासिक वेतन मिलता है
(A) ₹ 90,000
(B) ₹ 1,00,000
(C) ₹2,50,000
(D) ₹ 2,25,000
उत्तर:
(D) ₹ 2,25,000

22. सर्वोच्च न्यायालय के न्यायाधीश
(A) सेवा-निवृत्त होने के बाद सर्वोच्च न्यायालय में वकालत कर सकते हैं
(B) उच्च न्यायालय में वकालत कर सकते हैं
(C) सेवा-निवृत्त होने के बाद वकालत नहीं कर सकते
(D) 65 वर्ष से पूर्व हटाए नहीं जा सकते
उत्तर:
(C) सेवा-निवृत्त होने के बाद वकालत नहीं कर सकते

23. उच्च न्यायालय के न्यायाधीश का तबादला (Transfer) कर सकता है
(A) राष्ट्रपति
(B) राज्यपाल
(C) संसद
(D) मुख्यमंत्री
उत्तर:
(A) राष्ट्रपति

24. पंजाब और हरियाणा का उच्च न्यायालय स्थित है
(A) हिसार में
(B) पटियाला में
(C) अमृतसर में
(D) चंडीगढ़ में
उत्तर:
(D) चंडीगढ़ में

25. 23 अप्रैल, 2021 तक सर्वोच्च न्यायालय के मुख्य न्यायाधीश कौन रहे? ।
(A) श्री दीपक मिश्रा
(B) श्री जगदीश सिंह खेहर
(C) श्री रंजन गोगोई
(D) श्री शरद अरविंद बोबड़े
उत्तर:
(D) श्री शरद अरविंद बोबड़े

26. उच्च न्यायालय का निम्नलिखित प्रारंभिक क्षेत्राधिकार है
(A) मौलिक अधिकार के मामले
(B) फौजदारी मामले
(C) अपीलें सुनना
(D) कानून निर्माण
उत्तर:
(A) मौलिक अधिकार के मामले

27. उच्च न्यायालय के क्षेत्राधिकार में कौन वृद्धि कर सकता है?
(A) राष्ट्रपति
(B) राज्यपाल
(C) प्रधानमंत्री
(D) संसद
उत्तर:
(D) संसद

28. भारत में इस समय कुल उच्च न्यायालय हैं
(A) 23
(B) 24
(C) 25
(D) 26
उत्तर:
(C)25

निम्नलिखित प्रश्नों का उत्तर एक शब्द में दें

1. किन्हीं ऐसे दो देशों के नाम बताएँ, जिनमें न्यायपालिका की स्वतंत्रता को कोई महत्त्व नहीं दिया जाता।
उत्तर:

  • चीन,
  • रूस।

2. भारत के सर्वोच्च न्यायालय में कुल कितने न्यायाधीश हैं?
उत्तर:
भारत के सर्वोच्च न्यायालय में इस समय एक मुख्य न्यायाधीश तथा 33 अन्य न्यायाधीश (कुल 34) हैं।

3. भारत में इस समय कुल कितने उच्च न्यायालय हैं?
उत्तर:
भारत में इस समय कुल 25 उच्च न्यायालय हैं।

4. किन्हीं ऐसे दो राज्यों के नाम बताइए जिनका एक ही (साझा) उच्च न्यायालय है। वह कहाँ पर स्थित है?
उत्तर:
हरियाणा तथा पंजाब का एक ही साझा उच्च न्यायालय है जो चण्डीगढ़ में स्थित है।

HBSE 11th Class Political Science Important Questions Chapter 6 न्यायपालिका

5. उच्च न्यायालय के क्षेत्राधिकार में वृद्धि करने का अधिकार किसके पास है ?
उत्तर:
उच्च न्यायालय के क्षेत्राधिकार में वृद्धि करने का अधिकार संसद के पास है।

6. भारतीय संविधान के किस अनुच्छेद में सर्वोच्च न्यायालय की व्यवस्था की गई है?
उत्तर:
अनुच्छेद 124 में।

7. सर्वोच्च न्यायालय में मुख्य न्यायाधीश द्वारा तदर्थ न्यायाधीशों की नियुक्ति किसकी पूर्व अनुमति से की जा सकती है?
उत्तर:
राष्ट्रपति की।

8. संविधान द्वारा सर्वोच्च न्यायालय के न्यायाधीश हेतु कितनी न्यूनतम आयु निश्चित की गई है?
उत्तर:
न्यूनतम आयु का कोई उल्लेख नहीं है।

9. सर्वोच्च न्यायालय के न्यायाधीशों को वेतन कहाँ से दिया जाता है?
उत्तर:
भारत की संचित निधि से।

10. सर्वोच्च न्यायालय के मुख्य न्यायाधीश को शपथ कौन दिलाता है?
उत्तर:
राष्ट्रपति।

11. सर्वोच्च न्यायालय के न्यायाधीश कितने वर्ष की आयु में सेवानिवृत्त होते हैं?
उत्तर:
65 वर्ष।

12. सर्वोच्च न्यायालय के न्यायाधीशों को पद से हटाने की क्या विधि है?
उत्तर:
महाभियोग विधि।

13. उच्च न्यायालय के न्यायाधीश कितने वर्ष की आयु में सेवानिवृत्त होते हैं?
उत्तर:
65 वर्ष।

14. भारत में किसे अभिलेख न्यायालय माना गया है?
उत्तर:
सर्वोच्च न्यायालय को।

15. उच्चतम न्यायालय में न्यायाधीशों की संख्या में वृद्धि की शक्ति किसके पास है?
उत्तर:
संसद के पास।

16. न्यायायिक सक्रियता की आवश्यकता के दो कारण लिखें।
उत्तर:
सरकार की कमजोरी एवं देश में फैला व्यापक भ्रष्टाचार ।

रिक्त स्थान भरें

1. “सर्वोच्च न्यायालय ने समस्त संघात्मक ढाँचे को संबद्ध रखने में सीमेंट का काम किया है।” यह कथन का है।
उत्तर:
हरमन फ़ाइनर

2. सर्वोच्च न्यायालय की व्यवस्था अनुच्छेद …………….. में की गई है।
उत्तर:
124

3. भारत में सर्वोच्च न्यायालय का उद्घाटन ………….. को हुआ।
उत्तर:
28 जनवरी, 1950

4. सर्वोच्च न्यायालय के द्वारा ……………. को सलाह दी जाती है।
उत्तर:
राष्ट्रपति

5. पंजाब और हरियाणा का उच्च न्यायालय ……………. में स्थित है।
उत्तर:
चण्डीगढ़

6. संघीय व्यवस्था का संरक्षक ………….. होता है।
उत्तर:
सर्वोच्च न्यायालय

7. उच्चतम न्यायालय के मुख्य न्यायाधीश अपने पद पर …………… वर्ष तक रह सकते हैं।
उत्तर:
65

8. मौलिक अधिकारों की रक्षा ………………. के द्वारा की जाती है।
उत्तर:
सर्वोच्च न्यायालय

9. भारत में सर्वोच्च एवं उच्च न्यायालय के न्यायाधीशों की नियुक्ति ………………. प्रणाली द्वारा की जाती है।
उत्तर:
कॉलेजियम

10. सर्वोच्च न्यायालय में तदर्थ न्यायाधीशों की नियुक्ति ………………. द्वारा की जाती है।
उत्तर:
मुख्य न्यायाधीश

11. भारत में इस समय कुल ………………. उच्च न्यायालय हैं।
उत्तर:
25

12. ………………. भारत में सर्वोच्च अपीलीय न्यायालय है।
उत्तर:
सर्वोच्च न्यायालय

HBSE 11th Class Political Science Important Questions Chapter 5 विधायिका

Haryana State Board HBSE 11th Class Political Science Important Questions Chapter 5 विधायिका Important Questions and Answers.

Haryana Board 11th Class Political Science Important Questions Chapter 5 विधायिका

अति लघूत्तरात्मक प्रश्न

प्रश्न 1.
सरकार के कितने अंग होते हैं? उनके नाम बताएँ।
उत्तर:
सरकार के तीन अंग होते हैं विधानपालिका, कार्यपालिका तथा न्यायपालिका।

प्रश्न 2.
विधानपालिका के दो कार्य बताइए।
उत्तर:

  • कानूनों का निर्माण करना,
  • संविधान में संशोधन करना।

प्रश्न 3.
भारत में संघीय विधानमंडल को क्या कहा जाता है? उसके कितने सदन हैं?
उत्तर:
भारत में संघीय विधानमंडल को ‘संसद’ का नाम दिया गया है। इसके दो सदन हैं

  • राज्यसभा,
  • लोकसभा।

प्रश्न 4.
लोकसभा और राज्यसभा के सदस्यों का चुनाव कौन करता है?
उत्तर:
लोकसभा के सदस्यों का चुनाव जनता वयस्क मताधिकार के आधार पर प्रत्यक्ष रूप में करती है। राज्यसभा के सदस्यों का चुनाव राज्य विधानसभाओं के निर्वाचित सदस्य करते हैं।

प्रश्न 5.
लोकसभा व राज्यसभा का सदस्य बनने के लिए कम-से-कम कितनी आयु होनी चाहिए?
उत्तर:
लोकसभा का सदस्य बनने के लिए कम-से-कम 25 वर्ष तथा राज्यसभा का सदस्य बनने के लिए कम-से-कम 30 वर्ष की आयु होनी चाहिए।

प्रश्न 6.
लोकसभा अध्यक्ष एवं उपाध्यक्ष को शपथ कौन दिलाता है?
उत्तर:
लोकसभा अध्यक्ष एवं उपाध्यक्ष अपने पद को ग्रहण करते समय कोई शपथ नहीं लेते। वे केवल प्रारम्भ में संसद सदस्य होने की शपथ ही लेते हैं।

प्रश्न 7.
यदि संसद के संयुक्त अधिवेशन के समय लोकसभा अध्यक्ष एवं उपाध्यक्ष दोनों ही अनुपस्थित हों तो संयुक्त अधिवेशन की अध्यक्षता कौन करता है?
उत्तर:
संयुक्त अधिवेशन के समय लोकसभा अध्यक्ष एवं उपाध्यक्ष के अनुपस्थित होने पर राज्यसभा का उप-सभापति दोनों सदनों के संयुक्त अधिवेशन की अध्यक्षता करता है।

HBSE 11th Class Political Science Important Questions Chapter 5 विधायिका

प्रश्न 8.
लोकसभा का अध्यक्ष तथा राज्यसभा का सभापति बनने के लिए कम-से-कम कितनी आयु होनी चाहिए?
उत्तर:
लोकसभा का अध्यक्ष बनने के लिए कम-से-कम 25 वर्ष व राज्यसभा का सभापति बनने के लिए कम-से-कम 35 वर्ष की आयु होनी चाहिए।

प्रश्न 9.
लोकसभा व राज्यसभा का चुनाव कितने वर्ष के लिए होता है?
उत्तर:
लोकसभा का चुनाव साधारणतः 5 वर्ष के लिए तथा राज्यसभा के प्रत्येक सदस्य का चुनाव 6 वर्ष के लिए किया जाता है।

प्रश्न 10.
संसद की दो स्थायी समितियों के नाम लिखें।
उत्तर:
संसद की दो स्थायी समितियाँ हैं-लोक लेखा समिति, प्राक्कलन समिति।

प्रश्न 11.
लोकसभा के निर्वाचित सदस्यों की अधिकतम संख्या कितनी हो सकती है और 17वीं लोकसभा में कितनी है?
उत्तर:
लोकसभा के निर्वाचित सदस्यों की अधिकतम संख्या 550 हो सकती है। 17वीं लोकसभा, जिसके चुनाव सन् 2019 में हुए थे, के सदस्यों की संख्या 543 है।।

प्रश्न 12.
वर्तमान लोकसभा में भारतीय जनता पार्टी के कितने सांसद हैं?
उत्तर:
वर्तमान लोकसभा में भारतीय जनता पार्टी के 303 सांसद हैं।

प्रश्न 13.
राज्यसभा की अधिकतम सदस्य संख्या कितनी हो सकती है और वर्तमान में कितनी है?
उत्तर:
राज्यसभा की अधिकतम सदस्य संख्या 250 हो सकती है और वर्तमान सदस्य संख्या 245 है।

प्रश्न 14.
राज्यसभा में कितने निर्वाचित सदस्य हो सकते हैं और कितने मनोनीत ?
उत्तर:
राज्यसभा में 238 निर्वाचित और 12 मनोनीत सदस्य हो सकते हैं।

प्रश्न 15.
लोकसभा में सबसे ज्यादा और सबसे कम सदस्य किन राज्यों से चुने जाते हैं?
उत्तर:
लोकसभा में सबसे ज्यादा 80 सदस्य उत्तर प्रदेश से और सबसे कम सिक्किम, मेघालय व मिजोरम से चुने जाते हैं, जिनका एक-एक प्रतिनिधि होता है।

प्रश्न 16.
लोकसभा में राज्यों से कितने और संघीय क्षेत्रों से कितने प्रतिनिधि चुने जा सकते है?
उत्तर:
लोकसभा के लिए राज्यों से 530 तथा संघीय क्षेत्रों से 20 सदस्य चुने जा सकते हैं।

प्रश्न 17.
उस संघीय क्षेत्र का नाम बताइए जिससे लोकसभा के लिए एक से अधिक सदस्य चुने जाते हों और कितने सदस्य चुने जाते हैं?
उत्तर:
लोकसभा के लिए संघीय क्षेत्र दिल्ली से सात (7) सदस्य चुने जाते हैं।

प्रश्न 18.
हरियाणा व पंजाब से लोकसभा के लिए कितने सदस्य चुने जाते हैं?
उत्तर:
लोकसभा के लिए हरियाणा से दस (10) तथा पंजाब से तेरह (13) सदस्य चुने जाते हैं।

प्रश्न 19.
हरियाणा से लोकसभा व राज्यसभा के लिए कितने सदस्य चुने जाते हैं?
उत्तर:
हरियाणा से लोकसभा के लिए दस (10) तथा राज्यसभा के लिए पाँच (5) सदस्य चुने जाते हैं।

प्रश्न 20.
पंजाब से लोकसभा व राज्यसभा के लिए कितने सदस्य चुने जाते हैं?
उत्तर:
पंजाब से लोकसभा के लिए तेरह (13) व राज्यसभा के लिए सात (7) सदस्य चुने जाते हैं।

प्रश्न 21.
राज्यसभा में सबसे ज्यादा सदस्य किस राज्य से चुने जाते हैं?
उत्तर:
राज्यसभा में सबसे ज्यादा सदस्य उत्तर प्रदेश से 31 सदस्य चुने जाते हैं।

प्रश्न 22.
लोकसभा के लिए पहला साधारण निर्वाचन कब हुआ था और इसकी पहली बैठक कब हुई थी?
उत्तर:
लोकसभा के लिए पहला निर्वाचन वर्ष 1951-52 में हुआ था और लोकसभा की पहली बैठक 13 मई, 1952 को हुई थी।

प्रश्न 23.
लोकसभा और राज्यसभा का संयुक्त अधिवेशन कब बुलाया जाता है? इसकी अध्यक्षता कौन करता है?
उत्तर:
जब किसी साधारण विधेयक पर लोकसभा व राज्यसभा में गतिरोध उत्पन्न हो जाए तो दोनों का संयुक्त अधिवेशन बुलाया जा सकता है। संयुक्त अधिवेशन की अध्यक्षता लोकसभा का अध्यक्ष करता है।

प्रश्न 24.
संसद के दो कार्य बताइए।
उत्तर:

  • देश के लिए कानून बनाना,
  • संविधान में संशोधन करना।

प्रश्न 25.
संसद की शक्तियों पर लगी दो सीमाएँ बताइए।
उत्तर:

  • संसद ऐसा कोई संविधान संशोधन नहीं कर सकती, जिससे कि संविधान का मूलभूत ढाँचा विकृत अथवा नष्ट होता हो।
  • संसद ऐसा कोई कानून नहीं बना सकती, जिससे कि नागरिकों के मौलिक अधिकारों का उल्लंघन होता हो।

प्रश्न 26.
संसद कार्यपालिका पर नियंत्रण कैसे रखती है?
उत्तर:
संसद बजट को स्वीकार अथवा अस्वीकार करने के अधिकार तथा मंत्रिमंडल के विरुद्ध अविश्वास प्रस्ताव पारित करने के अधिकार द्वारा कार्यपालिका को नियन्त्रित करती है।

प्रश्न 27.
राज्यसभा की दो विशिष्ट शक्तियाँ बताइए।
उत्तर:

  • राज्यसभा 2/3 बहुमत से राज्य सूची के किसी विषय को राष्ट्रीय महत्त्व का घोषित कर सकती है।
  • राज्यसभा 2/3 बहमत से किसी नई अखिल भारतीय सेवा की स्थापना कर सकती है।

HBSE 11th Class Political Science Important Questions Chapter 5 विधायिका

प्रश्न 28.
अध्यक्ष की दो शक्तियाँ बताइए।
उत्तर:

  • अध्यक्ष धन विधेयक का निर्धारण करता है।
  • अध्यक्ष सदन की बैठकों की अध्यक्षता करता है।

प्रश्न 29.
राज्यसभा के सभापति की कोई दो शक्तियाँ बताइए।
उत्तर:

  • सभापति सदन में सदस्यों को विचार प्रकट करने की आज्ञा देता है।
  • सभापति आज्ञा न मानने वाले सदस्यों को सदन से बाहर जाने का आदेश दे सकता है।

प्रश्न 30.
अध्यक्ष को कौन हटा सकता है? क्या आज तक किसी अध्यक्ष को पद से हटाया गया है?
उत्तर:
अध्यक्ष को लोकसभा के सदस्य साधारण बहुमत से अविश्वास प्रस्ताव पारित करके हटा सकते हैं। आज तक किसी भी अध्यक्ष को पद से हटाया नहीं गया है।

प्रश्न 31.
राज्यसभा के सभापति को कौन हटा सकता है? क्या कभी हटाया गया है?
उत्तर:
राज्यसभा के सभापति (उप-राष्ट्रपति) को दोनों सदन अलग-अलग 2/3 बहुमत से महाभियोग प्रस्ताव पारित करके हटा सकते हैं। आज तक किसी सभापति को हटाया नहीं गया है।

प्रश्न 32.
लोकसभा की सदस्यता के लिए आवश्यक दो योग्यताएँ लिखें।
उत्तर:

  • वह भारत का नागरिक हो।
  • वह कम-से-कम 25 वर्ष की आयु पूरी कर चुका हो।

प्रश्न 33.
राज्यसभा की सदस्यता के लिए आवश्यक दो योग्यताएँ लिखें।
उत्तर:

  • वह भारत का नागरिक हो।
  • वह कम-से-कम 30 वर्ष की आयु पूरी कर चुका हो।

प्रश्न 34.
लोकसभा और राज्यसभा के समान अधिकारों के दो उदाहरण लिखें।
उत्तर:

  • संविधान के संशोधन के संबंध में।
  • राष्ट्रपति के विरुद्ध महाभियोग प्रस्ताव के संबंध में।

प्रश्न 35.
धन-विधेयक किस सदन में पेश किया जा सकता है? राज्यसभा कितने दिन तक धन-विधयेक रोक सकती है?
उत्तर:
धन-विधेयक केवल लोकसभा में ही प्रस्तावित किया जा सकता है। राज्यसभा धन-विधेयक को 14 दिन तक रोक सकती है।

प्रश्न 36.
धन-विधेयक क्या है? इसका निर्धारण कौन करता है?
उत्तर:
कर लगाने, घटाने, समाप्त करने, ऋण लेने, अनुदान देने इत्यादि से संबंधित विधेयक धन-विधेयक होता है। इसका निर्धारण लोकसभा का अध्यक्ष करता है।

प्रश्न 37.
साधारण विधेयक किस सदन में पेश किया जाता है? राज्यसभा इसे कितने समय तक रोक सकती है?
उत्तर:
साधारण विधेयक लोकसभा अथवा राज्यसभा किसी में भी पेश किया जा सकता है। राज्यसभा छः महीने तक साधारण विधेयक को रोक सकती है।

प्रश्न 38.
क्या राष्ट्रपति साधारण विधेयक पर निषेधाधिकार (Veto) शक्ति रखता है?
उत्तर:
भारतीय राष्ट्रपति के पास निषेधाधिकार शक्ति नहीं है, लेकिन वह विधेयक पर दोबारा विचार करने के लिए वापस संसद के पास भेज सकता है। यदि संसद दोबारा इसे पारित कर देती है तो उसे हस्ताक्षर करने ही होते हैं।

प्रश्न 39.
क्या राष्ट्रपति संविधान संशोधन पर निषेधाधिकार रखता है?
उत्तर:
नहीं, राष्ट्रपति को संविधान संशोधन पर हस्ताक्षर करने ही होते हैं। संविधान संशोधन को दोबारा विचार करने के लिए वापस भी नहीं भेज सकता।

प्रश्न 40.
यदि किसी साधारण विधेयक के विषय में दोनों सदनों में मतभेद उत्पन्न हो जाए तो निर्णय कैसे लिया जाता है ?
उत्तर:
राष्ट्रपति द्वारा दोनों सदनों का संयुक्त अधिवेशन बुलाया जाता है और निर्णय बहुमत से लिया जाता है।

प्रश्न 41.
दल-बदल विरोधी संबंधी कानून को तोड़ने से संबंधित मामले में निर्णय कौन लेता है? क्या उसके निर्णय को न्यायालय में चुनौती दी जा सकती है?
उत्तर:
दल-बदल विरोधी कानून से संबंधित निर्णय सदन का अध्यक्ष लेता है। उसके निर्णय के विरुद्ध न्यायालय में अपील की जा सकती है।

प्रश्न 42.
किन्हीं चार राज्यों के नाम बताएँ, जहाँ पर विधानमंडल द्वि-सदनीय है।
उत्तर:

  • उत्तर प्रदेश,
  • महाराष्ट्र,
  • बिहार,
  • कर्नाटक।

प्रश्न 43.
द्वि-सदनीय विधानमंडल वाले राज्यों में दोनों सदनों को क्या नाम दिया गया है?
उत्तर:

  • विधानसभा,
  • विधान परिषद्

प्रश्न 44.
विधानसभा का कार्यकाल कितना होता है? क्या उसे पहले भी भंग किया जा सकता है?
उत्तर:
विधानसभा का साधारण कार्यकाल पाँच वर्ष निश्चित किया गया है परन्तु उससे पहले भी मुख्यमंत्री की सिफारिश पर राज्यपाल द्वारा भंग किया जा सकता है।

प्रश्न 45.
विधानसभा की बैठकों की अध्यक्षता कौन करता है? उसका चुनाव कैसे किया जाता है?
उत्तर:
विधानसभा की बैठक की अध्यक्षता अध्यक्ष करता है। उसका चुनाव विधानसभा के सदस्यों द्वारा किया जाता है।

प्रश्न 46.
विधान परिषद् के सदस्यों की संख्या कितनी हो सकती है?
उत्तर:
विधान परिषद् के सदस्यों की संख्या उस राज्य की विधानसभा के सदस्यों की संख्या के 1/3 से अधिक तथा 40 से कम नहीं होनी चाहिए।

प्रश्न 47.
विधान परिषद् में राज्यपाल द्वारा कितने सदस्य मनोनीत किए जाते हैं?
उत्तर:
विधान-परिषद् की कुल सदस्य-संख्या का 1/6 भाग सदस्य राज्यपाल के द्वारा मनोनीत किए जाते हैं।

प्रश्न 48.
हरियाणा में विधानमंडल एक-सदनीय है अथवा द्वि-सदनीय? हरियाणा विधानसभा के सदस्यों की संख्या कितनी निश्चित की गई है?
उत्तर:
हरियाणा में एक-सदनीय विधानमंडल है। हरियाणा विधानसभा के सदस्यों की संख्या 90 निश्चित की गई है।

HBSE 11th Class Political Science Important Questions Chapter 5 विधायिका

प्रश्न 49.
विधानसभा के एक वर्ष में कितने अधिवेशन होते हैं?
उत्तर:
विधानसभा के एक वर्ष में कम-से-कम दो अधिवेशन अवश्य होने चाहिएँ। इसके अतिरिक्त राज्यपाल को किसी भी समय विधानसभा का विशेष अधिवेशन बुलाने का अधिकार है।।

प्रश्न 50.
ऐसे किन्हीं चार राज्यों के नाम बताएँ जिनमें विधानमंडल एक-सदनीय है।
उत्तर:

  • हरियाणा,
  • पंजाब,
  • हिमाचल प्रदेश तथा
  • राजस्थान।

प्रश्न 51.
राज्य विधानसभा का सदस्य बनने के लिए कोई दो योग्यताएँ बताइए।
उत्तर:

  • वह भारत का नागरिक हो,
  • वह 25 वर्ष की आयु पूरी कर चुका हो।

प्रश्न 52.
विधान परिषद की अवधि कितनी होती है?
उत्तर:
विधान-परिषद् एक स्थायी सदन है। इसका प्रत्येक सदस्य 6 वर्ष के लिए चुना जाता है। 1/3 सदस्य प्रत्येक दो वर्ष के पश्चात् सेवानिवृत हो जाते हैं और उनके स्थान पर नए सदस्य चुन लिए जाते हैं।

प्रश्न 53.
एक साधारण विधेयक तथा धन संबंधी विधेयक को विधान परिषद् कितने समय तक रोक सकती है?
उत्तर:
विधान परिषद् एक साधारण विधेयक को 4 महीने तक तथा धन-संबंधी विधेयक (Money Bill) को 14 दिन तक रोक सकती है।

प्रश्न 54.
किस राज्य की विधानसभा में सबसे अधिक तथा कितने सदस्य हैं?
उत्तर:
सबसे अधिक सदस्य उत्तर प्रदेश की विधानसभा में हैं। यह संख्या 403 है।

प्रश्न 55.
राज्यपाल राज्य विधानसभा में एंग्लो-इंडियन जाति के कितने सदस्य मनोनीत कर सकता है? हरियाणा विधानसभा में इस जाति के कितने सदस्य मनोनीत किए गए हैं?
उत्तर:
राज्यपाल विधानसभा में एंग्लो-इंडियन जाति के अधिक-से-अधिक 2 सदस्य मनोनीत कर सकता है। हरियाणा विधानसभा में इस जाति का कोई भी सदस्य मनोनीत नहीं किया गया है।

प्रश्न 56.
विधान परिषद् की बैठकों की अध्यक्षता कौन करता है? उसका चुनाव कैसे किया जाता है?
उत्तर:
विधान-परिषद् की बैठकों की अध्यक्षता उसका अध्यक्ष (Chairman) करता है। उसका चुनाव विधान परिषद् के सदस्यों द्वारा किया जाता है।

लघूत्तरात्मक प्रश्न

प्रश्न 1.
विधानपालिका के कोई तीन कार्य लिखें।
उत्तर:
विधानपालिका के तीन मुख्य कार्य निम्नलिखित हैं

1. कानून बनाना-विधानमंडल का मुख्य कार्य कानून बनाना है। इस क्षेत्र में वह नए कानून बनाती है, पुराने कानूनों में संशोधन करती है तथा अनावश्यक कानूनों को निरस्त करती है।

2. वित्त पर नियन्त्रण-विधानपालिका का धन पर पूर्ण नियन्त्रण होता है। सरकार विधानमंडल की स्वीकृति के बिना न तो एक पैसा खर्च कर सकती है और न ही कोई कर लगा सकती है, इसलिए प्रतिवर्ष कार्यपालिका अपना वार्षिक बजट विधानमंडल के सामने पेश करती है। विधानमंडल चाहे तो बजट को अस्वीकृत भी कर सकती है। सरकार के खर्च पर लेखा परीक्षक की रिपोर्ट भी विधानमंडल के सामने प्रस्तुत की जाती है।

3. कार्यपालिका पर नियन्त्रण-प्रजातन्त्र में कार्यपालिका पर विधानमंडल का प्रत्यक्ष अथवा अप्रत्यक्ष नियन्त्रण रहता है। संसदीय सरकार में कार्यपालिका विधानपालिका के प्रति उत्तरदायी रहती है। विधानमंडल के प्रत्येक सदस्य को मंत्रियों से प्रश्न पूछने तथा उनकी आलोचना करने का अधिकार होता है। मंत्रियों को सन्तोषप्रद उत्तर देने पड़ते हैं। विधानमंडल मंत्रिमंडल के विरुद्ध अविश्वास का प्रस्ताव पास कर सकता है जिस पर मंत्रिमंडल को त्याग-पत्र देना पड़ता है। अध्यक्षात्मक सरकार में भी विधानमंडल बजट पर नियन्त्रण करके अथवा जाँच समिति नियुक्त करके कार्यपालिका पर नियन्त्रण रखता है।

प्रश्न 2.
लोकसभा की रचना की संक्षेप में व्याख्या करें।
उत्तर:
लोकसभा संसद का निम्न सदन है, जिसके सदस्यों की संख्या 500 निश्चित की गई थी, परन्तु 1956 में इसके सदस्यों की संख्या 520 और 1963 में 525 कर दी गई, 1973 में इसकी संख्या 545 कर दी गई तथा 1987 में इसके सदस्यों की संख्या 550 की गई थी। एंग्लो-इण्डियन जाति के दो मनोनीत सदस्यों को मिलाकर अधिकतम संख्या 552 होगी। वर्तमान में लोकसभा में 545 सदस्य हैं।

इसमें से 543 निर्वाचित एवं 2 एंग्लो इण्डियन हैं। यहाँ यह उल्लेखनीय है कि दिसम्बर 2019 में पारित 104वें संवैधानिक संशोधन द्वारा एंग्लो इण्डियन जाति की मनोनयन प्रणाली को समाप्त करने का निर्णय किया गया है। ऐसी स्थिति में लोकसभा की अधिकतम संख्या 552 की जगह 550 ही रह जाएगी। सदस्यों का चुनाव प्रत्यक्ष रूप से जनता द्वारा किया जाता है तथा 18 वर्ष के प्रत्येक नागरिक को वोट डालने तथा 25 वर्ष के प्रत्येक नागरिक को चुनाव लड़ने का अधिकार है। सदस्य पाँच वर्ष के लिए चुने जाते हैं।

प्रश्न 3.
लोकसभा की पाँच शक्तियों का वर्णन करें।
उत्तर:
लोकसभा की पाँच विशेष शक्तियाँ निम्नलिखित हैं

1. विधायी शक्तियाँ-लोकसभा का मुख्य कार्य कानून बनाना है। केन्द्रीय सूची तथा समवर्ती सूची में दिए गए विषयों पर लोकसभा कानून बनाती है। विशेष परिस्थिति में राज्य-सूची में दिए गए विषय पर भी कानून बनाया जा सकता है।

2. वित्त पर नियन्त्रण-लोकसभा का देश के वित्त पर पूरा नियन्त्रण होता है। बजट लोकसभा द्वारा पास किया जाता है।

3. कार्यपालिका पर नियन्त्रण-लोकसभा कार्यपालिका पर नियन्त्रण रखती है। मन्त्रियों से पूरक प्रश्न पूछना, आलोचना करना तथा अविश्वास का प्रस्ताव पास करना लोकसभा के कार्य हैं।

4. चुनाव कार्य-लोकसभा के सदस्य राष्ट्रपति, उप-राष्ट्रपति तथा अपने अध्यक्ष व उपाध्यक्ष का चुनाव करते हैं।

5. न्यायिक कार्य-न्याय करने का काम न्यायपालिका का है, परन्तु संसद को न्यायिक शक्तियाँ भी प्राप्त हैं। राष्ट्रपति, उप-राष्ट्रपति, सर्वोच्च तथा उच्च न्यायालय के न्यायाधीश तथा अन्य पदाधिकारियों को महाभियोग द्वारा हटाने का कार्य संसद का है।

प्रश्न 4.
लोकसभा के अध्यक्ष के मुख्य कार्य बताइए।
उत्तर:
लोकसभा अध्यक्ष के मुख्य कार्य निम्नलिखित हैं

  • लोकसभा का अध्यक्ष सदन की बैठकों की अध्यक्षता करता है।
  • वह यह निर्णय करता है कि किस विधेयक पर किस दिन बहस होगी।
  • वह सदन का कार्यक्रम बनाता है, कार्रवाई के नियमों की व्याख्या करता है तथा उनसे सम्बन्धित झगड़ों का निपटारा करता है।
  • यदि किसी विधेयक पर यह विवाद हो जाए कि वह साधारण विधेयक है अथवा धन विधेयक तो लोकसभा के अध्यक्ष का निर्णय अन्तिम माना जाता है।
  • अध्यक्ष विधेयकों पर वाद-विवाद के पश्चात् मतदान करवाता है तथा निर्णय की घोषणा करता है।
  • वह सदन में विभिन्न प्रस्तावों को पेश करने की अनुमति देता है अथवा किसी प्रस्ताव को पेश करने से इनकार कर सकता है।
  • किसी विधेयक पर मतभेद की दशा में संयुक्त अधिवेशन की अध्यक्षता स्पीकर करता है।
  • वह राष्ट्रपति के सन्देशों को पढ़कर सुनाता है।

प्रश्न 5.
धन विधेयक किसे कहा जाता है? संक्षेप में व्याख्या करें।
उत्तर:
धन विधेयक उस विधेयक को कहा जाता है, जिसका सम्बन्ध कर लगाने, बढ़ाने तथा कम करने, खर्च करने, ऋण लेने तथा ब्याज देने आदि से सम्बन्धित हो। यदि कभी इस बात पर विवाद हो जाए कि अमुक विधेयक धन विधेयक है अथवा साधारण विधेयक है तो इस बात का निर्णय लोकसभा के अध्यक्ष द्वारा किया जाता है। उसका निर्णय अन्तिम होता है। यदि विधेयक धन विधेयक है तो वह केवल लोकसभा में ही पेश किया जाएगा, अन्यथा वह किसी भी सदन में पेश किया जा सकता है।

HBSE 11th Class Political Science Important Questions Chapter 5 विधायिका

प्रश्न 6.
राज्यसभा की पाँच विशेष शक्तियों का वर्णन करें।
उत्तर:
राज्यसभा की पाँच विशेष शक्तियाँ निम्नलिखित हैं

(1) राष्ट्रपति द्वारा आपात घोषणा की स्वीकृति राज्यसभा से भी 30 दिन के भीतर लेनी पड़ती है। लोकसभा के अधिवेशन न होने की दशा में आपात्काल की स्वीकृति राज्यसभा से ली जाती है। लोकसभा के अधिवेशन आरम्भ होने पर उसकी घोषणा पर अनुमोदन होना ज़रूरी है। यदि लोकसभा उस पर स्वीकृति नहीं देती तो 30 दिन के बाद यह घोषणा रद्द हो जाएगी।

(2) राज्यसभा 2/3 बहुमत से प्रस्ताव पास करके राज्य सूची के किसी विषय को राष्ट्रीय महत्त्व का घोषित कर उस पर संसद को कानून बनाने का अधिकार सौंप सकती है।

(3) राज्यसभा 2/3 बहुमत से प्रस्ताव पास करके किसी नई अखिल भारतीय सेवा को स्थापित कर सकती है।

(4) राज्यसभा की स्वीकृति के बिना किसी मौलिक अधिकार में परिवर्तन नहीं किया जा सकता।

(5) राष्ट्रपति द्वारा स्थापित आयोगों, यथा-संघीय लोक सेवा आयोग, अनुसूचित जातियों तथा कबीलों के आयोग, वित्त आयोग, अन्य आयोगों तथा महालेखा परीक्षक की रिपोर्ट राज्यसभा में भी प्रस्तुत की जाती है तथा उस पर विचार किया जाता है।

प्रश्न 7.
लोकसभा की सदस्यता ग्रहण करने के लिए संवैधानिक योग्यताएँ बताएँ।
उत्तर:
लोकसभा का सदस्य बनने के लिए प्रत्याशी में निम्नलिखित योग्यताएँ होनी चाहिएँ

  • वह भारत का नागरिक हो,
  • उसकी न्यूनतम आयु 25 वर्ष होनी चाहिए,
  • उसमें वे सभी योग्यताएँ होनी चाहिएँ । जो समय-समय पर संसद द्वारा निश्चित की गई हैं,
  • वह पागल, दिवालिया, घोर अपराधी तथा लाभकारी पद पर आसीन न हो,
  • उसका नाम मतदाता सूची में हो,
  • वह छुआछूत के विरुद्ध कानून द्वारा दण्डित न हो,
  • उस पर कोई फौजदारी मुकद्दमा न चल रहा हो।

प्रश्न 8.
राज्यसभा का सदस्य बनने के लिए योग्यताओं का उल्लेख करें।
उत्तर:
राज्यसभा का सदस्य बनने के लिए संविधान द्वारा कुछ योग्यताएँ निर्धारित की गई हैं जो निम्नलिखित हैं

  • वह भारत का नागरिक हो तथा उसकी आयु 30 वर्ष से कम न हो,
  • वह केन्द्र तथा राज्य सरकार के अधीन किसी लाभदायक पद पर न हो,
  • वह अपने राज्य का निवासी हो,
  • उसमें अन्य योग्यताएँ भी हों जो संसद कानून बनाकर निश्चित कर दे,
  • वह पागल, दिवालिया तथा अपराधी न हो,
  • संसद के किसी कानून द्वारा उसे चुनाव लड़ने के अयोग्य न ठहराया गया हो।

प्रश्न 9.
संसद के सदस्यों के विशेषाधिकारों का वर्णन कीजिए।
उत्तर:
संसद के दोनों सदनों के सदस्यों को निम्नलिखित विशेषाधिकार प्राप्त हैं
(1) संसद सदस्यों को सदन में, जिस सदन का वह सदस्य है, बोलने की पूर्ण स्वतन्त्रता प्राप्त है।

(2) सदन में कही गई किसी भी बात के लिए किसी सदस्य के विरुद्ध किसी भी न्यायालय में मुकद्दमा नहीं चलाया जा सकता।

(3) सदन का अधिवेशन आरम्भ होने के 40 दिन पहले, अधिवेशन के दौरान तथा अधिवेशन समाप्त होने के 40 दिन बाद तक उनको किसी भी दीवानी मुकद्दमे में गिरफ्तार नहीं किया जा सकता। उन्हें फौजदारी मुकद्दमे में गिरफ्तार किया जा सकता है, परन्तु सदस्य के बन्दी बनाए जाने की सूचना शीघ्र ही सदन के अध्यक्ष को देनी होती है।

(4) संसद अपने सदस्यों के विशेषाधिकारों का उल्लंघन करने वाले व्यक्ति अथवा संस्था के विरुद्ध मान-हानि की कार्रवाई . कर सकती है और उसके लिए दण्ड निश्चित कर सकती है।

(5) संसद सदस्यों को समस्त देश में कहीं भी जाने के लिए प्रथम श्रेणी का मुक्त रेलवे पास मिलता है। उन्हें मकान तथा टेलीफोन आदि की सुविधाएँ भी प्राप्त होती हैं।

प्रश्न 10.
राज्यसभा के अध्यक्ष के कार्य बताएँ।
उत्तर:
भारत का उप-राष्ट्रपति राज्यसभा का पदेन अध्यक्ष (Ex-officio Chairman) होता है। राज्यसभा का पदेन अध्यक्ष होने के नाते उसे 4 लाख रुपए प्रति मास वेतन तथा रहने के लिए निवास स्थान मिलता है। राज्यसभा का अध्यक्ष निम्नलिखित कार्य करता है
(1) वह राज्यसभा की बैठकों की अध्यक्षता करता है तथा सदन में शान्ति व्यवस्था बनाए रखता है।

(2) वह सदस्यों को सदन में बोलने की आज्ञा देता है।

(3) चूंकि वह राज्यसभा का सदस्य नहीं है, उसे मतदान का अधिकार नहीं है, परन्तु जब किसी विषय पर सदन में समान मत पड़ें तो उसे निर्णायक मत (Casting Vote) देने का अधिकार होता है।

(4) वह सदन में गणपूर्ति (Quorum) का निर्णय करता है।

(5) वह राज्यसभा के सदस्यों के विशेषाधिकारों की रक्षा करता है।

(6) वह सदन द्वारा पास किए गए विधेयकों पर हस्ताक्षर करता है और उन्हें लोकसभा अथवा राष्ट्रपति के पास भेजता है।

प्रश्न 11.
भारतीय संसद की कोई पाँच शक्तियाँ अथवा कार्य बताएँ।
उत्तर:
भारतीय संसद की पाँच शक्तियाँ निम्नलिखित हैं
(1) संसद का प्रमुख कार्य कानूनों का निर्माण करना है।

(2) संसद का राष्ट्रीय वित्त पर नियन्त्रण होता है। वह वार्षिक बजट पास करती है।

(3) संसद मन्त्रिमण्डल पर नियन्त्रण बनाए रखती है। मन्त्रिमण्डल संसद के प्रति उत्तरदायी है। संसद मन्त्रिमण्डल के विरुद्ध अविश्वास का प्रस्ताव पास करके उसे पद से हटा भी सकती है।

(4) संसद संविधान के संशोधन में भाग लेती है।

(5) भारत में संसद द्वारा राष्ट्रीय नीतियाँ निर्धारित की जाती हैं। मन्त्रि-परिषद् को संसद द्वारा निर्धारित नीतियों का पालन करना पड़ता है।

(6) संसद राष्ट्रपति एवं उप-राष्ट्रपति का चुनाव करती है। लोकसभा अपने अध्यक्ष एवं उपाध्यक्ष का चुनाव करती है। राज्यसभा अपने उपाध्यक्ष का चुनाव करती है।

प्रश्न 12.
भारतीय संसद की शक्तियों पर लगी कोई पाँच सीमाएँ बताएँ।
उत्तर:
भारतीय संसद की शक्तियों पर लगी सीमाएँ इस प्रकार हैं-

  • भारतीय संसद संविधान के उल्लंघन में कोई कानून नहीं बना सकती है,
  • संसद मौलिक अधिकारों का उल्लंघन करने वाला कोई कानून नहीं बना सकती,
  • संसद को संविधान की सभी धाराओं में अपनी इच्छानुसार संशोधन करने की शक्ति नहीं है। कई धाराओं में संशोधन के लिए उसे आधे राज्यों की स्वीकृति भी लेनी पड़ती है,
  • राष्ट्रपति को संसद द्वारा पास किए गए किसी भी विधेयक को अस्वीकार करने का अधिकार है,
  • भारत में शक्तियाँ केन्द्र तथा राज्य में बँटी हुई हैं और साधारणतः संसद राज्य सूची के विषयों पर कानून नहीं बना सकती।

प्रश्न 13.
सार्वजनिक लेखा समिति पर संक्षिप्त नोट लिखें।
अथवा
सार्वजनिक लेखा समिति के मुख्य कार्य बताएँ।
उत्तर:
यह संसद की एक महत्त्वपूर्ण समिति है। इसके कुल 22 सदस्य हैं जिनमें 15 लोकसभा तथा 7 राज्यसभा से लिए जाते हैं। इस समिति का अध्यक्ष प्रायः विरोधी दल का कोई सदस्य होता है जिसकी नियुक्ति समिति के सदस्यों में से लोकसभा के स्पीकर द्वारा की जाती है। इस समिति के मुख्य कार्य इस प्रकार हैं-
(1) नियन्त्रक एवं महालेखा परीक्षक की रिपोर्ट पर विचार करना तथा इस बात का परीक्षण करना कि सरकार द्वारा खर्च बजट के अनुसार हुआ है अथवा नहीं,

(2) यह देखना है कि जिस अधिकारी द्वारा खर्च किया जाता है, वह अधिकारी खर्च करने का अधिकार भी रखता है अथवा नहीं,

(3) समिति सरकार के अधीन किसी भी संस्था के खर्च की जाँच-पड़ताल कर सकती है तथा उसकी रिपोर्ट संसद के सामने पेश करती है।

प्रश्न 14.
अनुमान (आंकलन) समिति पर संक्षिप्त नोट लिखें।
उत्तर:
अनुमान समिति में 30 सदस्य होते हैं, जिनकी नियुक्ति लोकसभा के सदस्यों में से आनुपातिक प्रतिनिधित्व के आधार पर एक समय में एक वर्ष के लिए की जाती है। यह लोकसभा की एक बहुत महत्त्वपूर्ण समिति है। इसका गठन बजट में दिए गए विभिन्न अनुमानों का परीक्षण करने के लिए किया जाता है। इस समिति के मुख्य कार्य निम्नलिखित हैं

  • लोकसभा को प्रशासन में मितव्ययता (Economy) तथा कार्य-कुशलता लाने का सुझाव देना।
  • लोकसभा की वैकल्पिक नीतियाँ (Alternative Policies) पेश करना, जिससे प्रशासन में सुधार हो सके।
  • लोकसभा को प्रशासकीय कार्यों में लगे हुए धन की उपयोगिता के परीक्षण पर रिपोर्ट देना।

प्रश्न 15.
लेखानुदान तथा पूरक माँगों पर नोट लिखें।
उत्तर:
यदि किसी वित्तीय वर्ष के आरम्भ होने से पहले विनियोग विधेयक पास न किया जा सके तो सरकार के पास आवश्यक व्यय के लिए कोई धनराशि नहीं होगी, इसलिए यह व्यवस्था की गई है कि कुछ आवश्यक खर्चों के लिए सरकार को सीमित रकम दे दी जाए, उसे लेखानुदान कहते हैं। सन् 1991-92 के वित्तीय वर्ष के शुरू होने से पहले चन्द्रशेखर सरकार ने इसी का सहारा लिया था। यदि किसी कार्य के लिए बजट में निश्चित की हुई रकम या धनराशि कम प्रतीत हो तो लोकसभा के सामने पूरक अनुदान की माँग पेश की जाती है। इनको पास करने का तरीका वही है जो बजट या अन्य धन विधेयकों पर लागू होता है।

प्रश्न 16.
संसद के एक सदन में पास होने के लिए एक साधारण विधेयक को किन-किन अवस्थाओं में से गुजरना पड़ता है?
उत्तर:
साधारण विधेयक संसद के किसी भी सदन में आरम्भ किया जा सकता है। एक सदन में पास होने के लिए विधेयक को निम्नलिखित स्तरों में से होकर गुजरना पड़ता है

  • विधेयक का पेश करना तथा उस पर प्रथम वाचन,
  • द्वितीय वाचन,
  • समिति अवस्था,
  • रिपोर्ट अवस्था,
  • तृतीय वाचन

HBSE 11th Class Political Science Important Questions Chapter 5 विधायिका

प्रश्न 17.
‘ध्यानाकर्षण प्रस्ताव’ क्या होता है?
उत्तर:
यदि सदन का कोई सदस्य सदन का ध्यान किसी महत्त्वपूर्ण विषय अथवा घटना की ओर आकर्षित करना चाहता है, तो वह सदन में ध्यानाकर्षण प्रस्ताव पेश करता है। ऐसे प्रस्ताव प्रायः सरकार अथवा किसी मन्त्री का ध्यान आकर्षित करने के लिए पेश किए जाते हैं।

प्रश्न 18.
विधान सभा का गठन किस प्रकार होता है?
उत्तर:
विधान सभा राज्य विधानमण्डल का निम्न सदन है जिसके सदस्यों का चुनाव प्रत्यक्ष रूप से जनता द्वारा किया जाता है। संविधान ने देश के राज्यों की विधानसभाओं के लिए अलग-अलग संख्या तय की है। फिर भी एक राज्य की विधान सभा में अधिक-से-अधिक 500 तथा कम-से-कम 60 सदस्य हो सकते हैं। बाद में बहुत छोटे-छोटे राज्य, जैसे मिजोरम तथा गोवा स्थापित हो गए, जिनमें विधान सभा के सदस्य 40 हैं। अब किसी विधान सभा में कम-से-कम 32 (सिक्किम) सदस्य हैं और अधिक-से-अधिक (उत्तर प्रदेश में) 403 सदस्य हैं। सदस्यों का चुनाव 5 वर्ष के लिए किया जाता है।

प्रश्न 19.
विधान सभा की पाँच मुख्य शक्तियों का वर्णन करें।
उत्तर:
विधान सभा की पाँच मुख्य शक्तियाँ इस प्रकार हैं-

  • राज्य सूची के विषयों पर कानून बनाने का कार्य विधान सभा का है। यदि किसी राज्य में दो सदन हैं तो विधान परिषद् के साथ मिलकर अन्यथा अकेली विधान सभा कानून बनाती है,
  • राज्य के वित्त पर विधान सभा का नियन्त्रण होता है। राज्य का वित्त मन्त्री प्रत्येक वर्ष बजट विधान सभा के समक्ष पेश करता है,
  • विधान सभा का राज्य मन्त्रिमण्डल पर पूरा नियन्त्रण होता है। मन्त्रिमण्डल को अविश्वास के प्रस्ताव द्वारा अपने पद से हटाया जा सकता है,
  • विधान सभा के निर्वाचित सदस्य राष्ट्रपति के चुनाव में भाग लेते हैं। वे अपने सदस्यों में से एक अध्यक्ष तथा एक उपाध्यक्ष का चुनाव भी करते हैं,
  • विधान सभा विधान परिषद् के एक-तिहाई सदस्यों का चुनाव करती है।

प्रश्न 20.
विधान परिषद् की पाँच उपयोगिताएँ बताएँ।
उत्तर:
विधान परिषद् की पाँच उपयोगिताएँ इस प्रकार हैं-

  • विधान परिषद् विधेयक को शीघ्रता से पास नहीं होने देती, जिससे जनता विधेयक के गुण-दोषों पर विचार कर लेती है,
  • विधान परिषद् विधेयक की त्रुटियों को दूर करती है,
  • विधान परिषद में योग्य तथा अनुभवी व्यक्तियों को मनोनीत किया जा सकता है, जिससे उनके ज्ञान का सारे राज्य को लाभ पहुँच सके,
  • विधान परिषद् में योग्य तथा अनुभवी व्यक्तियों को मनोनीत किया जा सकता है,
  • विधान परिषद् विधान सभा को निरंकुश तथा स्वेच्छाचारी बनने से रोकती है तथा शक्ति सन्तुलन बनाए रखती है।

प्रश्न 21.
विधान सभा के अध्यक्ष (स्पीकर) के पाँच कार्यों का वर्णन करें।
उत्तर:
विधान सभा के अध्यक्ष (स्पीकर) के पाँच कार्य इस प्रकार हैं-

  • अध्यक्ष विधान सभा के अधिवेशनों की अध्यक्षता करता है तथा सदन में शान्ति व्यवस्था बनाए रखता है,
  • सभी सदस्य अध्यक्ष की अनुमति से सदन की कार्रवाई के नियमों की व्याख्या करता है,
  • विधेयकों पर वाद-विवाद करवाना, मतदान करवाना तथा परिणाम घोषित करना स्पीकर के कार्य हैं,
  • सदन में किसी विषय पर समान मत पड़ने की स्थिति में वह अपने निर्णायक मत (Casting Vote) का प्रयोग कर सकता है।

प्रश्न 22.
विधान परिषद् का सदस्य बनने की योग्यताओं का वर्णन करें।
उत्तर:
विधान-परिषद् का सदस्य बनने के लिए दी गई योग्यताओं का होना आवश्यक है-

  • वह भारत का नागरिक हो,
  • उसकी आयु 30 वर्ष से कम न हो,
  • वह केन्द्र तथा राज्य सरकार के अधीन किसी लाभदायक पद पर न हो,
  • वह पागल, दिवालिया तथा चुनाव लड़ने के अयोग्य घोषित न किया गया हो,
  • वह विधान सभा तथा संसद के किसी सदन का सदस्य न हो।

प्रश्न 23.
राज्य विधान सभा तथा विधान परिषद् की शक्तियों में तुलना करें।
उत्तर:
(1) साधारण विधेयक दोनों में से किसी में भी प्रस्तावित किया जा सकता है। विधान परिषद् किसी भी साधारण विधेयक में अधिक-से-अधिक चार महीने की देरी कर सकती है,

(2) धन विधेयक केवल विधान सभा में ही प्रस्तावित किए जा सकते हैं। विधान परिषद् धन विधेयक को 14 दिन से अधिक नहीं रोक सकती,

(3) मन्त्रिमण्डल के विरुद्ध अविश्वास प्रस्ताव पास करने का अधिकार केवल विधान सभा को ही प्राप्त है। विधान परिषद् मन्त्रिमण्डल के विरुद्ध अविश्वास प्रस्ताव पारित नहीं कर सकती,

(4) राष्ट्रपति के निर्वाचन में केवल विधान सभा के सदस्य भाग लेते हैं। विधान परिषद् के सदस्यों को राष्ट्रपति के चुनाव में भाग लेने का अधिकार नहीं है,

(5) विधान सभा 2/3 बहुमत से विधान परिषद् समाप्त करने का प्रस्ताव पारित कर सकती है।

प्रश्न 24.
राज्य विधान सभा के अध्यक्ष पर संक्षिप्त नोट लिखें।
उत्तर:
अध्यक्ष का चुनाव विधान सभा द्वारा अपने सदस्यों में से किया जाता है। अध्यक्ष अधिकांशतया बहुमत दल का ही व्यक्ति बनता है। अध्यक्ष चुने जाने के बाद वह अपने दल से नाता नहीं तोड़ता, लेकिन दल की सक्रिय राजनीति से अलग रहता है। अध्यक्ष अपने पद पर विधान सभा में विश्वास प्राप्ति तक बना रहता है। विधान सभा उसको हटाने सम्बन्धी प्रस्ताव साधारण बहुमत से पारित कर सकती है, लेकिन ऐसा नोटिस कम-से-कम 14 दिन पूर्व देना आवश्यक है। अध्यक्ष सदन की बैठकों की अध्यक्षता करता है तथा सदन की कार्रवाई का संचालन करता है।

प्रश्न 25.
विधान परिषद् के सदस्यों की योग्यताएँ बताएँ।
उत्तर:
विधान-परिषद् के सदस्यों में दी गई योग्यताएँ होनी चाहिएँ-

  • वह भारत का नागरिक हो,
  • उसकी आयु 30 वर्ष से कम न हो,
  • वह संघीय सरकार अथवा राज्य सरकार के अधीन किसी लाभ के पद पर न हो,
  • उसके पास वे सभी योग्यताएँ हों जो संसद कानून द्वारा समय-समय पर निर्धारित करे,
  • वह न्यायालय द्वारा अपराधी, दिवालिया अथवा पागल घोषित न किया गया हो।

प्रश्न 26.
विधान परिषद् के विपक्ष में तर्क दीजिए।
उत्तर:
विधान परिषद् के विपक्ष में दीए गए तर्क इस प्रकार हैं-

  • दोनों सदनों में गतिरोध उत्पन्न होने की सम्भावना रहती है,
  • विधान परिषद् विधेयकों पर वाद-विवाद को मात्र दोहराती है,
  • यह आवश्यक नहीं है कि विधान-परिषद् में वाद-विवाद का स्तर ऊँचा हो,
  • योग्य व अनुभवी व्यक्ति विधान सभा के लिए भी चुने जाते हैं।

प्रश्न 27.
राज्यपाल की स्वविवेकी शक्तियों का वर्णन कीजिये।
उत्तर:
राज्यपाल कुछ शक्तियों का प्रयोग स्वेच्छा एवं स्व-विवेक से करता है, जो कि निम्नलिखित हैं
(1) जब विधानसभा में किसी भी दल को स्पष्ट बहुमत प्राप्त न हो या बहुमत दल अपना नेता चुनने में असमर्थ हो तो राज्यपाल अपनी इच्छानुसार मुख्यमन्त्री की नियुक्ति कर सकता है।

(2) राज्यपाल का यह देखना परम कर्त्तव्य है कि संघ सरकार के कानूनों, आदेशों तथा नीतियों का राज्य में ठीक प्रकार से पालन हो रहा है या नहीं।

(3) राज्य का राज्यपाल इस बात की रिपोर्ट राष्ट्रपति को भेजता है कि राज्य का शासन संविधान के अनुसार नहीं चलाया जा रहा है। राज्यपाल साधारण बिलों पर पहली बार निषेधाधिकार (Veto Power) का प्रयोग कर सकता है। राज्यपाल किसी बिल को राष्ट्रपति की अन्तिम स्वीकृति के लिए सुरक्षित रख सकता है। असम और नागालैण्ड के राज्यपाल के पास कबाइली इलाकों सम्बन्धी विशेष शक्तियाँ प्राप्त हैं। .

(4) जब राष्ट्रपति धारा 356 के अन्तर्गत संकटकाल की घोषणा करता है तो उस समय राज्यपाल राष्ट्रपति के एजेण्ट के रूप में कार्य करता है तथा मन्त्रि-परिषद् को समाप्त कर दिया जाता है।

(5) राज्यपाल को यह अधिकार प्राप्त है कि वह किसी भी समय मुख्यमन्त्री से शासन सम्बन्धी सूचना माँग सकता है। राज्यपाल किसी एक अकेले मन्त्री के निर्णय को वापस कर सकता है ताकि उस पर सारी मन्त्रि-परिषद् की राय जानी जाए।

इसमें तनिक भी सन्देह नहीं है कि वास्तव में मन्त्रि-परिषद् और राज्यपाल के आपसी सम्बन्ध कई आधारों पर निर्भर करते हैं। अगर मन्त्रि-परिषद् एक ही दल का है तथा उसी दल की सरकार केन्द्र में है तो ऐसी स्थिति में राज्यपाल पूर्णतः संवैधानिक मुखिया होता है। अगर मन्त्रि-परिषद् को विधानसभा में स्पष्ट बहुमत प्राप्त नहीं है तो ऐसी स्थिति में राज्यपाल की भूमिका महत्त्वपूर्ण होती है तथा वह वास्तविक शासक के रूप में कार्य करता है; जैसा कि चौथे आम चुनाव (1967) के बाद कई प्रान्तों में राज्यपालों को ऐसी स्थिति प्राप्त थी।

प्रश्न 28.
विधान सभा सदस्यों के पाँच विशेषाधिकार बताएँ।
उत्तर:
विधान सभा के सदस्यों के पाँच विशेषाधिकार इस प्रकार हैं-
(1) सदस्यों को सदन में भाषण देने की स्वतन्त्रता है,

(2) सदन में दिए गए भाषण के लिए सदस्य के विरुद्ध कोई मुकद्दमा नहीं किया जा सकता,

(3) सदस्यों को सदन का अधिवेशन शुरू होने से 40 दिन पहले, अधिवेशन के दौरान तथा अधिवेशन समाप्त होने के 40 दिन बाद तक किसी दीवानी मुकद्दमे में गिरफ्तार नहीं किया जा सकता,

(4) सदस्यों को मासिक वेतन के अतिरिक्त निर्वाचन क्षेत्र भत्ता तथा यात्रा आदि की सुविधाएँ भी प्राप्त हैं,

(5) सदस्यों को रहने के लिए निःशुल्क निवास स्थान मिलता है।

निबंधात्मक प्रश्न

प्रश्न 1.
भारतीय संसद की रचना, उसकी शक्तियों तथा कार्यों का वर्णन करें।
उत्तर:
रचना (Composition)-भारत की संघीय विधानपालिका को संसद का नाम दिया गया है। संविधान के अनुच्छेद 79 के अन्तर्गत, “संघ की एक संसद होगी, जिसमें राष्ट्रपति तथा दो सदन होंगे जिनका नाम लोकसभा तथा राज्यसभा होगा।” लोकसभा भारतीय संसद का निम्न सदन है। इसके सदस्यों की अधिक-से-अधिक संख्या 552 हो सकती है।

इनमें से 520 सदस्य राज्यों द्वारा तथा 30 सदस्य संघीय क्षेत्रों (Union Territories) द्वारा निर्वाचित किए जाते हैं। इसके अतिरिक्त यदि एंग्लो-इण्डियन जाति का कोई व्यक्ति निर्वाचित न हुआ हो तो ऐसे दो सदस्यों को राष्ट्रपति मनोनीत कर सकता है। इसके सदस्य पाँच वर्ष के लिए प्रत्यक्ष रूप से मतदाताओं द्वारा निर्वाचित किए जाते हैं। पाँच वर्ष की अवधि से पूर्व भी लोकसभा को भंग किया जा सकता है। लोकसभा के सदस्य अपने में से एक अध्यक्ष तथा उपाध्यक्ष का चुनाव करते हैं।

राज्यसभा संसद का उच्च सदन है, जिसमें 250 सदस्य होते हैं। इनमें से 12 सदस्य राष्ट्रपति द्वारा मनोनीत किए जाते हैं। इनको साहित्य, कला, विज्ञान, समाज-सेवा, शिक्षा आदि क्षेत्रों में उल्लेखनीय काम करने वाले व्यक्तियों में से चुना जाता है। शेष धानसभाओं के सदस्यों द्वारा निर्वाचित किए जाते हैं। इसके सदस्य 6 वर्ष के लिए चुने जाते हैं, परन्तु हर दूसरे वर्ष 1/3 सदस्य अपने पद से निवृत्त हो जाते हैं तथा उनकी जगह नए सदस्यों का चुनाव किया जाता है। भारत का उप-राष्ट्रपति राज्यसभा का पदेन अध्यक्ष होता है। राज्यसभा के सदस्य अपने में से एक उपाध्यक्ष का चुनाव करते हैं।

संसद सदस्यों के वेतन, भत्ते तथा पेंशन (Salary, Allowances and Pension of Members of Parliament)-संसद के . दोनों सदनों के सदस्यों को समान वेतन, भत्ते तथा पेंशन आदि मिलते हैं। वित्त मन्त्री अरुण जेटली के द्वारा 1 फरवरी, 2018 को बजट प्रस्तुत करते समय सांसदों के वेतन भत्तों में प्रस्तावित कानून के अनुरूप प्रत्येक 5 वर्ष में मुद्रास्फीति के अनुरूप सांसदों के वेतन में स्वतः संशोधन हो जाएगा। वित्तमन्त्री के संशोधित प्रस्ताव के

अनुसार मूल वेतन 50 हजार से बढ़ाकर 1 लाख, निर्वाचन क्षेत्र भत्ता 45 हजार से बढ़ाकर 70 हजार एवं सचिवालय भत्ता 30 हजार से बढ़ाकर 60 हजार, दैनिक भत्ता 2000 से बढ़ाकर 4000 करने का निर्णय लिया जो 1 अप्रैल, 2018 से लागू हुए। सांसदों को निःशुल्क आवास, परिवार सहित निःशुल्क चिकित्सा सुविधा, तीन लैंड लाइन टेलीफोन एवं एक बी०एस०एन०एल० मोबाइल तथा एक मोबाइल निर्वाचन क्षेत्र हेतु जिसमें एक लाख साठ हजार मुफ्त कॉलों के साथ 20 हजार अतिरिक्त मुफ्त कॉलें भी दी गई हैं।

प्रत्येक सांसद को कम्प्यूटर हेतु मुफ्त ब्राड बैंड सुविधा भी प्रदान की गई है। इसके अतिरिक्त प्रत्येक सांसद को देश में लगभग 34 हवाई यात्राएँ भी निःशुल्क प्रदान की गई हैं। सांसदों को मिलने वाली पेंशन 20,000 रुपए मासिक के स्थान पर योगी आदित्यनाथ कमेटी ने इसे बढ़ाकर 35 हजार रुपए करने की सिफारिश की है।

इसके अतिरिक्त 1988 में पास किए गए अधिनियम द्वारा सांसद की उसके कार्यकाल के दौरान इसकी मृत्यु होने पर उसकी पत्नी या पति या उसके आश्रित को पेंशन की सुविधा प्रदान की गई थी। संसद की शक्तियाँ तथा कार्य (Powers and Functions of the Parliament) भारत की संसद संघीय विधानपालिका है। संघ की सभी वैधानिक शक्तियों का प्रयोग संसद द्वारा किया जाता है। कानून निर्माण के अतिरिक्त भी संसद को अन्य कई प्रकार के काम करने पड़ते हैं जो निम्नलिखित हैं:

1. वैधानिक शक्तियाँ (Legislative Powers):
संसद को कानून बनाने की व्यापक शक्ति प्राप्त है। संघीय सूची के सभी विषयों पर संसद को कानून बनाने का अधिकार है। विशेष परिस्थितियों में राज्य सूची में दिए गए विषयों पर भी संसद कानून बना सकती है। समवर्ती सूची प कानून बनाने का अधिकार संसद तथा राज्य विधानसभाओं को प्राप्त है, परन्तु विवाद की दशा में संसद द्वारा बनाया गया कानून ही मान्य होगा।

2. वित्तीय शक्तियाँ (Financial Powers):
संसद को वित्तीय क्षेत्र में भी शक्तियाँ प्राप्त हैं। प्रतिवर्ष वित्तमन्त्री संसद में बजट पेश करता है। संसद इस पर विचार करके इसको पास करती है। संसद की स्वीकृति के बिना सरकार जनता पर कोई कर नहीं लगा सकती तथा न ही कोई पैसा एकत्रित करके खर्च कर सकती है।

3. कार्यपालिका पर नियन्त्रण (Control over the Executive):
मन्त्रिमण्डल अपने सभी कार्यों के लिए व्यक्तिगत तथा सामूहिक रूप से संसद के प्रति उत्तरदायी होता है। संसद मन्त्रियों से प्रश्न पूछकर, उनकी आलोचना करके तथा अविश्वास के प्रस्ताव द्वारा मन्त्रिमण्डल पर नियन्त्रण रखती है। मन्त्रिमण्डल संसद में बहुमत रहने तक ही कार्यरत रह सकता है।

4. यिक कार्य (Judicial Functions):
न्याय करने का काम न्यायपालिका का है, परन्तु संसद को न्यायिक शक्तियाँ भी प्राप्त हैं। राष्ट्रपति, उप-राष्ट्रपति, सर्वोच्च तथा उच्च न्यायालय के न्यायाधीश तथा अन्य पदाधिकारियों को महाभियोग द्वारा हटाने का कार्य संसद का है।

5. संविधान में संशोधन (Amendments in the Constitution):
संसद को संविधान में संशोधन करने का अधिकार है। संविधान की कुछ धाराओं को साधारण बहुमत से, कुछ धाराओं को 2/3 बहुमत से तथा कुछ धाराओं को संशोधित करने के लिए 2/3 बहुमत के साथ-साथ आधे राज्यों के विधानमण्डलों की स्वीकृति भी अनिवार्य है।

6. राष्ट्रीय नीतियों को निर्धारित करना (Determination of the National Policies):
भारत की संसद केवल कानून ही नहीं बनाती, बल्कि वह राष्ट्रीय नीतियाँ भी निर्धारित करती है। मन्त्रि-परिषद् को संसद द्वारा निर्धारित नीतियों का पालन करना पड़ता है, नहीं तो उन्हें संसद की तीव्र आलोचना सहनी पड़ती है। संसद सदस्य मन्त्रि-परिषद् से त्याग-पत्र देने की माँग भी कर सकते हैं।

7. विविध कार्य (Miscellaneous Functions)

  • संसद नए राज्यों का निर्माण करती है। सीमाओं में परिवर्तन करती है तथा उनके नामों में परिवर्तन करती है।
  • किसी राज्य में विधान परिषद् की स्थापना अथवा समाप्त करना संसद का काम है।
  • संसद, राष्ट्रपति की संकटकालीन घोषणा को स्वीकृति प्रदान करती है।
  • संसद को विस्तृत शक्तियाँ प्राप्त हैं, परन्तु फिर भी वह इंग्लैण्ड की संसद की तरह प्रभुसत्ता-सम्पन्न नहीं है।

निष्कर्ष (Conclusion):
उपर्युक्त अध्ययन से स्पष्ट होता है कि भारतीय संसद ब्रिटिश संसद का मुकाबला नहीं कर सकती। लेकिन फिर भी भारतीय संसद केवल कानून निर्माण ही नहीं करती, अपितु अन्य अनेक प्रकार के महत्त्वपूर्ण कार्य भी उसके द्वारा किए जाते हैं। भारतीय संसद के पास अनेक महत्त्वपूर्ण शक्तियाँ तो हैं, परन्तु : ब्रिटिश संसद के समान प्रभुसत्ता सम्पन्न संसद (Sovereign Body) नहीं है क्योंकि इसकी शक्तियाँ सीमित हैं। इसमें तनिक भी सन्देह नहीं है कि हमारी संसद का अधिकार क्षेत्र कितना भी सीमित क्यों न हो, फिर भी यह एक धुरी के समान है जिसके इर्द-गिर्द भारतीय सरकार की समस्त मशीनरी घूमती है।

HBSE 11th Class Political Science Important Questions Chapter 5 विधायिका

प्रश्न 2.
राज्यसभा की रचना, कार्य तथा शक्तियों का वर्णन कीजिए।
उत्तर:
राज्यसभा की रचना (Composition of Council of States) राज्यसभा भारतीय संसद का ऊपरी सदन (Upper House) है। संविधान द्वारा इसके सदस्यों की अधिक-से-अधिक संख्या 250 निश्चित की गई है, जिनमें से 12 सदस्य राष्ट्रपति द्वारा ऐसे व्यक्तियों में से मनोनीत किए जाते हैं जो साहित्य, कला, विज्ञान तथा समाज सेवा आदि के कारण प्रसिद्धि प्राप्त किए रहते हैं। जैसे जुलाई, 2018 में राष्ट्रपति द्वारा अन्तर्राष्ट्रीय ख्याति प्राप्त शास्त्रीय नृत्यांगना डॉ० सोनल मानसिंह, प्रसिद्ध मूर्तिकार डॉ० रघुनाथ महापात्र, दलित नेता राम सकल एवं संघ विचारक राकेश सिन्हा को मनोनीत किया गया।

शेष सदस्य राज्यों की विधानसभाओं द्वारा एकल संक्रमणीय मत (Single Transferable Vote) तथा आनुपातिक प्रतिनिधित्व (Proportional Representation) के आधार पर निर्वाचित किए जाते हैं। अनुच्छेद 80(5) के अनुसार, “संघ द्वारा प्रशासित क्षेत्र (Union Territories) के प्रतिनिधियों का चुनाव संसद के कानून द्वारा निश्चित की गई व्यवस्था के अनुसार किया जाता है।”

राज्यसभा में भारतीय संघ के विभिन्न राज्यों तथा संघीय क्षेत्रों को उनकी जनसंख्या के आधार पर प्रतिनिधित्व दिया गया है। इसका अर्थ यह है कि अधिक जनसंख्या वाले राज्यों को अधिक तथा कम जनसंख्या वाले राज्यों को कम प्रतिनिधि चुनने का अधिकार है। इस सम्बन्ध में संविधान यह व्यवस्था करता है कि एक राज्य की जनसंख्या के प्रथम 50 लाख व्यक्तियों तक हर 10 लाख व्यक्तियों के लिए एक और उसके बाद 20 लाख पर एक के हिसाब से प्रतिनिधित्व प्राप्त होगा। विभिन्न राज्यों तथा संघीय प्रदेशों की राज्यसभा में सदस्य संख्या संविधान की चौथी अनुसूची (IV Schedule) में लिखी गई है।

सदस्यों की योग्यताएँ (Qualifications of the Members):
राज्यसभा का सदस्य बनने के लिए संविधान द्वारा कुछ योग्यताएँ निर्धारित की गई है जो इस प्रकार हैं-

  • वह भारत का नागरिक हो तथा उसकी आयु 30 वर्ष से कम न हो,
  • वह केन्द्र तथा राज्य सरकार के अधीन किसी लाभदायक पद पर न हो,
  • वह अपने राज्य का निवासी हो,
  • उसमें अन्य योग्यताएँ भी हों जो संसद कानून बनाकर निश्चित कर दे,
  • वह पागल, दिवालिया तथा अपराधी न हो,
  • संसद के किसी कानून द्वारा उसे चुनाव लड़ने के अयोग्यं न ठहराया गया हो।

अवधि (Term)-राज्यसभा एक स्थायी सदन है, जिसके सभी सदस्यों का चुनाव एक साथ नहीं होता तथा न ही सम्पूर्ण सदन को कभी भंग किया जाता है। सदस्यों का कार्यकाल 6 वर्ष निश्चित किया गया है, परन्तु हर दूसरे वर्ष 1/3 सदस्य अपने पद से सेवानिवृत्त हो जाते हैं तथा उनकी जगह नए सदस्यों का चुनाव किया जाता है। सदस्यों को पुनर्निर्वाचित होने का अधिकार है। गणपूर्ति (Quorum) राज्यसभा की बैठकों की कार्रवाई चलाने के लिए इसके कुल सदस्यों के 1/10 भाग की उपस्थिति अनिवार्य है।

यदि इतने सदस्य उपस्थित नहीं हैं तो सदन की कार्रवाई नहीं चलेगी। 42वें संशोधन द्वारा राज्यसभा अपनी गणपूर्ति संख्या स्वयं निश्चित कर सकती है। राज्यसभा का सभापति (Chairman of Rajya Sabha) भारत का उप-राष्ट्रपति राज्यसभा का पदेन (Ex-officio) सभापति होता है। इसी तरह अमेरिका का उप-राष्ट्रपति सीनेट का पदेन (Ex-officio) सभापति होता है। राज्यसभा का एक उप-सभापति भी अपने सदस्यों में से निर्वाचित किया जाता है। सभापति का प्रमुख उद्देश्य सदन की अध्यक्षता करना तथा अनुशासन को बनाए रखना है।

प्रायः उप-राष्ट्रपति अपने वोट का प्रयोग नहीं करता, लेकिन दोनों पक्षों के बराबर मत होने पर, वह अपने निर्णायक मत (Casting Vote) का प्रयोग करता है। सभापति अथवा उप-राष्ट्रपति के वेतन तथा भत्ते संसद द्वारा निश्चित किए जाते हैं। सभापति को 4 लाख रुपए प्रति मास वेतन मिलता है। राज्यसभा के सदस्यों द्वारा अपने सदन से ही किसी भी सदस्य को उप-सभापति निर्वाचित किया जाता है जो सभापति की अनुपस्थिति में सदन की कार्रवाई का संचालन करता है। जब सभापति तथा उप-सभापति दोनों ही अनुपस्थित हों, तब ऐसा व्यक्ति सदन की अध्यक्षता करता है, जिसे सदन नियुक्त करे। आजकल राज्यसभा के अध्यक्ष श्री एम० वेंकैया नायडू (11 अगस्त, 2017 से) हैं तथा उप-सभापति श्री हरिवंश नारायण सिंह (10 अगस्त, 2018 से) हैं।

राज्यसभा के सदस्यों के विशेषाधिकार (Privileges of Members of Rajya Sabha) राज्यसभा के सदस्यों को अपने विचार प्रकट करने की पूर्ण स्वतन्त्रता है। सदन में दिए गए भाषणों के कारण उनके खिलाफ किसी भी न्यायालय के द्वारा कार्रवाई नहीं की जा सकती। इसके अलावा अधिवेशन के दिनों में तथा उससे 40 दिन पूर्व अथवा बाद में भी उनको किसी दीवानी अभियोग के कारण गिरफ्तार नहीं किया जा सकता।

सदन अपने विशेषाधिकारों का उल्लंघन करने वाले व्यक्ति के खिलाफ मान-हानि की कार्रवाई कर सकता है। . सदस्यों के वेतन तथा भत्ते (Salaries and Allowances of the Members)-राज्यसभा के सदस्यों को वेतन व भत्तों की सुविधा लोकसभा के सदस्यों के समान प्राप्त है। लोकसभा एवं राज्यसभा के सदस्यों के वेतन व भत्ते समय-समय पर संसद द्वारा निर्धारित विधि के अनुसार दिए जाएँगे।

राज्यसभा के कार्य व शक्तियाँ (Functions and Powers of Council of States)-राज्यसभा भारत की संसद का दूसरा अथवा उच्च सदन है। राज्यसभा को निम्नलिखित अधिकार व शक्तियाँ प्राप्त हैं

1. कानून-निर्माण का कार्य (Legislative Functions):
भारत में संघात्मक सरकार की स्थापना की गई है। संघात्मक सरकारयों का विभाजन होता है। भारत में सारी शक्तियाँ संघ-सूची, राज्य-सूची तथा समवर्ती सूची में बाँटी गई हैं। राज्यसभा को संघ-सूची तथा समवर्ती सूची में दिए गए सभी साधारण विषयों पर कानून बनाने का अधिकार है। राज्य सूची में दिए गए विषयों पर भी विशेष स्थिति में राज्यसभा कानून बनाने का कार्य कर सकती है। साधारण विधेयक किसी भी सदन में पेश किया जा सकता है।

राज्यसभा में पास होने पर उसे लोकसभा के पास भेजा जाता है। लोकसभा की स्वीकृति मिलने पर उसे अन्तिम स्वीकृति के लिए राष्ट्रपति के पास भेजा जाता है। यदि किसी विधेयक पर दोनों सदनों में मतभेद हो जाए तो राष्ट्रपति दोनों सदनों का संयुक्त अधिवेशन बुला सकता है। संयुक्त अधिवेशन में लोकसभा के सदस्यों की संख्या अधिक होने के कारण साधारणतः लोकसभा की बात मानी जाती है, परन्तु ऐसा हर समय आवश्यक नहीं है। संयुक्त बैठक की अध्यक्षता लोकसभा का अध्यक्ष करता है। सन् 1986 तक दोनों सदनों की संयुक्त बैठक दो बार 1961 और 1978 में बुलाई गई थी और उसमें लोकसभा की बात मानी गई थी।

2. वित्तीय कार्य (Financial Functions):
वित्तीय क्षेत्र में राज्यसभा की शक्तियाँ बहुत ही सीमित हैं। कोई भी धन विधेयक राज्यसभा में पेश नहीं किया जा सकता। धन विधेयक राष्ट्रपति की स्वीकृति से पहले लोकसभा में पेश किया जाता है। लोकसभा में धन विधेयक पास होने पर उसे राज्यसभा में भेजा जाता है, परन्तु राज्यसभा किसी भी धन विधेयक को अपने पास अधिक-से-अधिक 14 दिन तक रोक सकती है। इस अवधि के पश्चात् लोकसभा द्वारा पास विधेयक को दोनों सदनों द्वारा पास समझा जाता है।

3. कार्यपालिका पर नियन्त्रण (Control Over Executive):
यद्यपि मन्त्रि-परिषद् लोकसभा के प्रति ही सामूहिक रूप से उत्तरदायी होती है, परन्तु राज्यसभा को भी मन्त्रि-परिषद् पर नियन्त्रण रखने का अधिकार प्राप्त है, फिर भी उसका यह अधिकार सीमित है। राज्यसभा मन्त्रियों से प्रश्न व पूरक प्रश्न पूछने, आलोचना करने, ध्यानाकर्षण प्रस्ताव लाने व पास करने का अधिकार रखती है। इसमें राज्यसभा को मन्त्रि-परिषद् पर नियन्त्रण रखने का अधिकार प्राप्त हो जाता है, परन्तु राज्यसभा के पास मन्त्रि-परिषद के विरुद्ध अविश्वास का प्रस्ताव पास करने का अधिकार नहीं है। केन्द्रीय मन्त्रि-परिषद् में राज्यसभा के कुछ सदस्य होने से वह उसे प्रभावित करती रहती है।

4. चुनाव सम्बन्धी कार्य (Electoral Functions):
राज्यसभा को चुनाव सम्बन्धी अधिकार भी दिए गए हैं। राज्यसभा अपने सदस्यों में से उप-सभापति का चुनाव करती है। राष्ट्रपति के चुनाव में राज्यसभा के निर्वाचित सदस्य तथा उप-राष्ट्रपति के चुनाव में सभी सदस्य भाग लेते हैं।

5. संविधान में संशोधन (Amendment in the Constitution):
संविधान में संशोधन करने का अधिकार दोनों सदनों को समान रूप से प्राप्त है। संशोधन प्रस्ताव किसी भी सदन में पेश किया जा सकता है। जब तक दोनों सदन सहमत न हों, संविधान में संशोधन नहीं हो सकता। दोनों सदनों में मतभेद होने की दशा में वह निरस्त कर दिया जाता है। व्यावहारिक रूप में संशोधन के विषय में दोनों सदनों में दो बार मतभेद हुआ है।

1970 में राजाओं के प्रिवीपर्स को बन्द करने के संशोधन को राज्यसभा ने अस्वीकार कर दिया था तथा दूसरी ओर 1974 में 45वें संशोधन को राज्यसभा ने कुछ संशोधनों के साथ पास किया था और लोकसभा को उसे अस्वीकार करना पड़ा था।

6. न्यायिक कार्य (Judicial Functions):
राज्यसभा को न्यायिक कार्य करने का अधिकार भी प्राप्त है। राष्ट्रपति, उप-राष्ट्रपति, सर्वोच्च तथा उच्च न्यायालय के मुख्य न्यायाधीश तथा अन्य न्यायाधीशों तथा अन्य वे अधिकारी जो न्यायालय द्वारा दण्डित नहीं किए जाते, उनके विरुद्ध महाभियोग चलाकर उन्हें अपने पद से हटाने का लोकसभा के साथ-साथ राज्यसभा को भी समान अधिकार प्राप्त है।

7. अन्य कार्य व अधिकार (Miscellaneous Functions and Rights or Powers):
उपर्युक्त शक्तियों के अतिरिक्त राज्यसभा को अन्य बहुत से कार्य, अधिकार तथा शक्तियाँ प्राप्त हैं जो निम्नलिखित हैं

(1) राष्ट्रपति द्वारा आपातकाल की घोषणा की स्वीकृति राज्यसभा से भी 30 दिन के भीतर लेनी पड़ती है। लोकसभा के अधिवेशन न होने की दशा में आपात्काल की स्वीकृति राज्यसभा से ली जाती है। लोकसभा के अधिवेशन आरम्भ होने पर उसकी घोषणा पर अनुमोदन होना ज़रूरी है। यदि लोकसभा उस पर स्वीकृति नहीं देती तो 30 दिन के बाद यह घोषणा निरस्त हो जाएगी।

(2) राज्यसभा 2/3 बहुमत से प्रस्ताव पास करके राज्य सूची के किसी विषय को राष्ट्रीय महत्त्व का घोषित कर उस पर संसद को कानून बनाने का अधिकार सौंप सकती है।

(3) राज्यसभा 2/3 बहुमत से प्रस्ताव पास करके किसी नई अखिल भारतीय सेवा को स्थापित करना राष्ट्रहित में उचित घोषित कर सकती है। ऐसी अवस्था में संसद में कानून बनाकर ऐसी सेवा स्थापित कर सकती है।

(4) राज्यसभा की स्वीकृति के बिना किसी मौलिक अधिकार में परिवर्तन नहीं किया जा सकता।

(5) राष्ट्रपति द्वारा स्थापित आयोगों, यथा-संघीय लोक सेवा आयोग, अनुसूचित जातियों तथा कबीलों के आयोग, वित्त आयोग, अन्य आयोगों तथा महालेखा परीक्षक की रिपोर्ट राज्यसभा में भी प्रस्तुत की जाती है तथा उस पर विचार किया जाता है। ऊपर वर्णित तथ्यों के आधार पर कहा जा सकता है कि राज्यसभा को कई प्रकार के कार्य करने की शक्तियाँ प्राप्त हैं। वह कार्यपालिका तथा वित्तीय क्षेत्र को छोड़कर शेष कार्यों में लोकसभा के समान ही शक्तियाँ रखती है।

प्रश्न 3.
राज्यसभा एक गौण सदन नहीं, महत्त्वपूर्ण सदन है। व्याख्या कीजिए।
उत्तर:
भारत की संघीय संसद में राज्यसभा दूसरा अथवा उच्च सदन है। राज्यसभा को भले ही उच्च सदन कहा जाता हो, परन्तु इसका स्थान लोकसभा से निम्न है। कुछ राजनीतिक विचारक इसे दूसरा नहीं, वरन् दूसरे दर्जे का सदन मानते हैं, कुछ इसे सजावट का सदन मानते हैं। उनके ऐसा मानने के पीछे तर्क यह है कि धन विधेयकों, कार्यपालिका पर नियन्त्रण रखने, सरकार को बनाने-बिगाड़ने आदि मामलों में उसकी शक्ति बहुत कम है।

कानून-निर्माण क्षेत्र में भी प्रायः लोकसभा की ही महत्ता रहती है। केवल अपवादस्वरूप स्थिति में ही इसके विपरीत हो सकता है। इसमें सन्देह नहीं कि राज्यसभा निचले सदन की भांति एक शक्तिशाली संस्था नहीं है, मन्त्रि-परिषद् पर भी इसका कोई प्रभावी नियन्त्रण नहीं है तथा वित्तीय मामलों में इसके बहुत ही सीमित अधिकार हैं। फिर भी यह निरर्थक सदन नहीं है। यह एक उपयोगी सदन है। इसकी उपयोगिता निम्नलिखित है

1. विधेयकों का पुनर्निरीक्षण (Revision of the Bills):
राज्यसभा का प्रथम लाभ यह है कि यह लोकसभा द्वारा शीघ्रतापूर्वक बिना विचार किए गए विधेयकों पर रोक लगाती है। आधुनिक युग में विधेयकों की संख्या में बहुत अधिक वृद्धि हो रही है। लोकसभा की व्यस्तता तथा कम समय होने के कारण विधेयकों को जल्दी में पास कर दिया जाता है। इसका नतीजा यह निकलता है कि विधेयकों में कई त्रुटियाँ रह जाती हैं। लेकिन राज्यसभा विधेयकों पर अधिक विचार-विमर्श करके कमियों को दूर करने का हर सम्भव प्रयास करती है। 1979 में राज्यसभा ने लोकसभा द्वारा पारित विशेष न्यायालय विधेयक में महत्त्वपूर्ण संशोधन किए जिसे लोकसभा ने उसी समय मान लिया।

2. विवादहीन विधेयकों का पेश होना (Introduction of Non-Controversial Bills):”
अधिकतर महत्त्वपूर्ण विधेयक लोकसभा में ही पेश किए जाते हैं लेकिन विवादहीन विधेयक प्रायः राज्यसभा में ही पेश किए जाते हैं। राज्यसभा ऐसे विधेयकों पर अच्छी तरह विचार-विमर्श करती है तथा इसके बाद विधेयकों को लोकसभा के पास भेजती है। वस्तुतः ऐसे विधेयकों पर लोकसभा का अधिक समय बर्बाद नहीं होता है। इस तरह राज्यसभा लोकसभा के बहुमूल्य समय को बचाती है जिससे लोकसभा इस समय का सदुपयोग अन्य महत्त्वपूर्ण विधेयकों पर करती है।

3. वाद-विवाद का स्तर ऊँचा (High Standard to Discussion):
राज्यसभा में वाद-विवाद का स्तर लोकसभा की अपेक्षा अधिक ऊँचा है। राज्यसभा में प्रत्येक विधेयक पर शान्तिपूर्वक विचार होता है। राज्यसभा में सदस्यों की संख्या लोकसभा के सदस्यों की तुलना में कम है एवं इसके सदस्य अधिक अनुभवी व कुशल होते हैं। राज्यसभा कनाडा के सीनेट की अपेक्षा अधिक शक्तिशाली तथा उपयोगी है। वह अपने विवादों तथा सरकार की आलोचना द्वारा जनता पर अधिक प्रभाव डालती है। राज्यसभा को हम आदर्श द्वितीय सदन कह सकते हैं। यह लोकसभा पर नियन्त्रण का कार्य करती है तथा लोकप्रिय सदन के मार्ग में अड़चनें पैदा नहीं करती है।।

4. स्थायी सदन तथा विशेष अधिकार (Permanent House and Special Powers):
राज्यसभा एक स्थायी सदन है। स्थायी सदन होने के कारण राज्यसभा उस समय भी जनमत का प्रतिनिधित्व करती है जब लोकसभा भंग होती है। 21 जून, 1977 को राज्यसभा की 25वीं वर्षगाँठ के अवसर पर प्रधानमन्त्री मोरारजी देसाई ने कहा कि हमारी इस सवैधानिक व्यवस्था में राज्यसभा एक ऐसा निकाय है जिसे भंग नहीं किया जा सकता और जो लगातार बना रहता है।

कई विषयों में राज्यसभा को लोकसभा से कहीं अधिक शक्तियाँ प्राप्त हैं; जैसे अनुच्छेद 249 के अन्तर्गत राज्यसभा अपने प्रस्ताव के द्वारा राज्य-सूची के किसी विषय को संसद के अधिकार-क्षेत्र में ला सकती है। इसके अलावा राज्यसभा के परामर्श पर केन्द्रीय सरकार नई अखिल भारतीय सेवाओं, अखिल भारतीय न्यायिक सेवाओं की व्यवस्था कर सकती है।

5. योग्य व्यक्तियों का प्रतिनिधित्व (Representation of Able Persons):
इसमें देश के अनुभवी प्रतिनिधि होते हैं। राष्ट्रपति 12 ऐसे सदस्यों को मनोनीत करते हैं जिन्होंने विज्ञान, कला, साहित्य, देश-सेवा आदि क्षेत्रों में विशेष योग्यता प्राप्त की होती है।

6. संशोधन की समान शक्ति (Equal Amendment Powers):
संविधान में संशोधन करने की शक्ति राज्यसभा को लोकसभा के समान है। संविधान में संशोधन राज्यसभा की स्वीकृति के बिना सम्भव नहीं हो सकता। अगस्त, 1978 में राज्यसभा ने 44वें संशोधन प्रस्ताव की छह महत्त्वपूर्ण धाराओं को निरस्त कर दिया था और दिसम्बर, 1978 में लोकसभा ने 44वें संशोधन विधेयक को उसी तरह पास किया जिस तरह राज्यसभा ने पास किया।

7. आपात्कालीन घोषणा की स्वीकृति (Approval of Proclamation of Emergency):
भारत का राष्ट्रपति अनुच्छेद 352, 356, 360 के अन्तर्गत संकटकाल की घोषणा कर सकता है जिसकी स्वीकृति संसद से लेनी आवश्यक होती है। ऐसी स्थिति में यदि लोकसभा भंग हो तो उसकी स्वीकृति राज्यसभा के द्वारा ली जाती है। इसके अलावा जब राष्ट्रपति लोकसभा को भंग कर  देता है तब भी राज्यसभा बनी रहती है।

8. उच्चतम न्यायालय तथा उच्च न्यायालयों के न्यायाधीशों को पदच्युत करने में राज्यसभा को समान शक्ति (Equal Powers of Rajya Sabha for the Removal of the Judges of Supreme Court and High Courts):
उच्चतम न्यायालय तथा उच्च न्यायालय के न्यायाधीशों को सेवानिवृत्त होने से पहले केवल महाभियोग द्वारा ही हटाया जा सकता है।

न्यायाधीशों को महाभियोग द्वारा हटाने के लिए आवश्यक है कि संसद के दोनों सदन लोकसभा एवं राज्यसभा अलग-अलग अपनी कुल सदस्य-संख्या के बहुमत तथा उपस्थित एवं मतदान में भाग लेने वाले सदस्यों के 2/3 बहमत से न्यायाधीश के विरुद्ध प्रस्ताव पास करके राष्ट्रपति को भेजें।

9. भारतीय संवैधानिक प्रणाली के अनुकूल (InAccordance with Indian Constitutional System):
ब्रिटिश शासनकाल में भारत में संसदीय संस्थाओं का विकास हुआ था। ब्रिटिश शासनकाल में भी केन्द्रीय विधानमण्डल के दो सदनों की स्थापना की गई थी। अतः संविधान निर्माताओं ने चली आ रही परम्परा का पालन करना ही अच्छा समझा।

10. संघीय सिद्धान्त के अनुकूल (According to Federalism):
भारत में संघीय प्रणाली को अपनाया गया है। संघीय प्रणाली में इकाइयों को प्रतिनिधित्व देने के लिए दूसरे सदन का होना जरूरी होता है। इसलिए राज्यसभा इस शर्त को पूरा करने में सहायता करता है।

इस प्रकार राज्यसभा एक दूसरा उपयोगी सदन है। यह लोकसभा पर ब्रेक का कार्य करती है। भारत की राज्यसभा ब्रिटेन की लॉर्ड सभा तथा कनाडा की सीनेट से अधिक शक्तिशाली तथा उपयोगी सदन है। भारत की राज्यसभा अमेरिका की सीनेट की तरह शक्तिशाली नहीं है। अन्त में हम एम०वी० पायली (M.V. Paylee) के कथन से सहमत हैं कि राज्यसभा एक निरर्थक अथवा कानून पर रोक लगाने वाला सदन ही नहीं है। वास्तव में राज्यसभा शासन-तन्त्र का एक आवश्यक अंग है, केवल दिखावे मात्र का दूसरा सदन नहीं है।

HBSE 11th Class Political Science Important Questions Chapter 5 विधायिका

प्रश्न 4.
लोकसभा की रचना, कार्य तथा शक्तियों का वर्णन कीजिए।
उत्तर:
रचना (Composition)-आरम्भ में लोकसभा के सदस्यों की अधिक-से-अधिक संख्या 500 निश्चित की गई थी। सन् 1956 में इसे बढ़ाकर 520 तथा सन् 1963 में इसे 525 कर दिया गया। सन् 1973 में संविधान के 31वें संशोधन द्वारा, इसके निर्वाचित सदस्यों की अधिकतम संख्या 545 कर दी गई। सन् 1987 में पास किए गए ‘गोवा, दमन तथा दीव पुनर्गठन अधिनियम’ (Goa, Daman and Div Re-organization Act, 1987) द्वारा इसे 550 कर दिया गया।

इनमें से 530 सदस्य राज्यों में से तथा 20 सदस्य केन्द्र द्वारा प्रशासित क्षेत्रों (Union Territories) में से चुने जाएंगे। इसके अतिरिक्त यदि राष्ट्रपति यह अनुभव करे कि चुनाव के द्वारा एंग्लो-इण्डियन जाति को पर्याप्त प्रतिनिधित्व नहीं मिल सका, तो वह अनुच्छेद 331 के अधीन इस जाति के दो सदस्यों को लोकसभा में मनोनीत कर सकता है। इस प्रकार अब लोकसभा के सदस्यों की अधिकतम संख्या 552 हो सकती है। वर्तमान लोकसभा में 545 सदस्य हैं।

इनमें से 543 सदस्य निर्वाचित सदस्य हैं और 2 एंग्लो-इण्डियन सदस्य होते हैं जिन्हें राष्ट्रपति मनोनीत करता है। यहाँ यह उल्लेखनीय है कि दिसम्बर, 2019 में पारित 104वें संवैधानिक संशोधन द्वारा एंग्लो-इण्डियन जाति की मनोनयन प्रणाली को समाप्त करने का निर्णय किया गया है। ऐसी स्थिति में लोकसभा की अधिकतम संख्या 552 की जगह 550 ही रह जाएगी।

यद्यपि इस संशोधन को भारत संघ के आधे राज्य एवं केन्द्र शासित प्रदेशों की विधानसभाओं द्वारा अनुमोदित किए जाने के बाद क्रियान्वित करने के लिए राष्ट्रपति द्वारा स्वीकृति मिलेगी। … सदस्यों की योग्यताएँ (Qualifications of Members)-संविधान द्वारा लोकसभा के प्रत्याशी के लिए दी गई योग्यताएँ होनी आवश्यक हैं-

  • वह भारत का नागरिक हो,
  • उसकी आयु 25 वर्ष से कम न हो,
  • वह संसद द्वारा निर्धारित अन्य योग्यताएँ रखता हो,
  • वह संघ तथा राज्य सरकार के अधीन किसी लाभदायक पद पर न हो,
  • वह पागल, दिवालिया तथा अपराधी न हो।

कार्यकाल (Tenure) लोकसभा का कार्यकाल 5 वर्ष है, परन्तु इसका यह कार्यकाल निश्चित नहीं है। प्रधानमन्त्री के परामर्श पर राष्ट्रपति द्वारा लोकसभा को समय से पूर्व भी भंग किया जा सकता है। 1979 में चौधरी चरण सिंह ने समय से पूर्व लोकसभा भंग करवा दी थी। इस तरह नौवीं लोकसभा भी समय से पूर्व श्री चन्द्रशेखर ने भंग करवा दी थी। ग्यारहवीं व बारहवीं लोकसभा को भी समय से पहले भंग कर दिया गया था।

15वीं लोकसभा को राष्ट्रपति ने संविधान के अनुच्छेद 85 के अन्तर्गत भंग किया और 16वीं लोकसभा के चुनाव सन् 2014 में हुए। लोकसभा का कार्यकाल बढ़ाया भी जा सकता है। ऐसा केवल संकटकाल में ही किया जा सकता है। एक बार में लोकसभा की अवधि एक वर्ष के लिए बढ़ाई भी जा सकती है। 1975 में संकटकाल के कारण चुनाव 1976 की बजाए 1977 में हुए थे। आपात्काल के समाप्त होते ही 6 महीने के अन्दर नया चुनाव करवाना अनिवार्य है।

गणपूर्ति (Quorum) लोकसभा की कार्रवाई चलाने के लिए कुल सदस्य संख्या का कम-से-कम 1/10 सदस्यों का सदन में उपस्थित होना आवश्यक है। इसके बिना इसकी कार्रवाई स्थगित कर दी जाएगी। अधिवेशन (Sessions of the House)-संविधान के अनुच्छेद 85 के अनुसार राष्ट्रपति जब और जहाँ उचित समझे, संसद के दोनों सदनों अथवा एक सदन का अधिवेशन बुला सकता है, परन्तु पहले और दूसरे अधिवेशन के बीच 6 महीने से अधिक का समय नहीं होना चाहिए। इसका अभिप्राय यह है कि लोकसभा के वर्ष में दो अधिवेशन अवश्य होते हैं, परन्तु वास्तव में कई
अधिवेशन होते हैं।

सदन के पदाधिकारी (House Officer)-संविधान के अनुसार, लोकसभा की बैठक जारी रखने के लिए लोकसभा के कुल सदस्यों में से स्पीकर व डिप्टी-स्पीकर का चुनाव करती है। ये दोनों इन पदों पर उसी समय तक बने रह सकते हैं, जब तक वे सदन के सदस्य रहते हैं। सदन की सदस्यता समाप्त होते ही उन्हें अपने पदों से अलग होना पड़ता है। जैसे 17वीं लोकसभा चुनाव 2019 में भारतीय जनता पार्टी अकेले 303 लोकसभा स्थानों पर विजय प्राप्त कर पुनः स्पष्ट बहुमत के साथ सत्ता में आने पर अध्यक्ष एवं उपाध्यक्ष के चुनाव में निर्णायक स्थिति में आ गई।

इसी कारण 19 जून, 2019 को लोकसभा अध्यक्ष के चुनाव में भाजपा के लोकसभा सदस्य श्री ओम बिड़ला के नाम का प्रस्ताव प्रधानमंत्री श्री नरेन्द्र मोदी द्वारा रखा गया तथा गृहमंत्री श्री अमित शाह, रक्षा मन्त्री श्री राजनाथ सिंह एवं कांग्रेस सहित अन्य विपक्षी दलों ने उनके नाम का समर्थन किया। इस प्रकार श्री ओम बिड़ला को स्पीकर पद पर सर्वसम्मति से निर्वाचित किया गया।

यहाँ यह उल्लेखनीय है कि लोकसभा के अध्यक्ष इस पद को ग्रहण करते समय इस पद के लिए कोई शपथ नहीं लेते। अध्यक्ष प्रारम्भ में केवल संसद सदस्य होने की शपथ लेते हैं। संसद सदस्यों के वेतन, भत्ते तथा पेंशन (Salary,Allowances and Pension of Members of Parliament)-संसद के दोनों सदनों के सदस्यों को समान वेतन, भत्ते तथा पेंशन आदि मिलते हैं।

वित्त मन्त्री अरुण जेटली के द्वारा 1 फरवरी, 2018 को बजट प्रस्तुत करते समय सांसदों के वेतन भत्तों में प्रस्तावित कानून के अनुरूप प्रत्येक 5 वर्ष में मुद्रास्फीति के अनुरूप सांसदों के वेतन में स्वतः संशोधन हो जाएगा। वित्तमन्त्री के संशोधित प्रस्ताव के अनुसार मूल वेतन 50 हजार से बढ़ाकर 1 लाख, निर्वाचन क्षेत्र भत्ता 45 हजार से बढ़ाकर 70 हजार एवं सचिवालय भत्ता 30 हजार से बढ़ाकर 60 हजार, दैनिक भत्ता 2000 से बढ़ाकर 4000 करने का निर्णय लिया जो 1 अप्रैल, 2018 से लागू हुए।

सांसदों को निःशुल्क आवास, परिवार सहित निःशुल्क चिकित्सा सुविधा, तीन लैंड लाइन टेलीफोन एवं एक बी०एस०एन०एल० मोबाइल तथा एक मोबाइल निर्वाचन क्षेत्र हेतु जिसमें एक लाख साठ हजार मुफ्त कॉलों के साथ 20 हजार अतिरिक्त मुफ्त कॉलें भी दी गई हैं। प्रत्येक सांसद को कम्प्यूटर हेतु मुफ्त ब्राड बैंड सुविधा भी प्रदान की गई है। इसके अतिरिक्त प्रत्येक सांसद को देश में लगभग 34 हवाई यात्राएँ भी निःशुल्क प्रदान की गई हैं।

सांसदों को मिलने वाली पेंशन 20,000 रुपए मासिक के स्थान पर योगी आदित्यनाथ कमेटी ने इसे बढ़ाकर 35 हजार रुपए करने की सिफारिश की है। इसके अतिरिक्त 1988 में पास किए गए अधिनियम द्वारा सांसद की उसके कार्यकाल के दौरान इसकी मृत्यु होने पर उसकी पत्नी या पति या उसके आश्रित को पेंशन की सुविधा प्रदान की गई थी।

संसद सदस्यों के विशेषाधिकार (Privileges of Members of Parliament)-संसद सदस्य अपने कर्त्तव्य का उचित रूप से पालन कर सकें, इसके लिए उन्हें कुछ विशेषाधिकार प्राप्त हैं-
(i) संसद अथवा उसकी किसी समिति में कही गई बात पर सदस्य के विरुद्ध किसी न्यायालय में किसी प्रकार की कार्रवाई नहीं की जा सकती।

(ii) संसद का अधिवेशन आरम्भ होने के 40 दिन पहले तथा अधिवेशन समाप्त होने के 40 दिन बाद तक उन्हें किसी भी दीवानी मुकद्दमे में बन्दी नहीं बनाया जा सकता, परन्तु फौजदारी मुकद्दमे में उन्हें पकड़ा जा सकता है।

(iii) संसद के प्रत्येक सदस्य को स्थानीय क्षेत्र विकास योजना के अन्तर्गत अपने चुनाव क्षेत्र में विकास कार्यों पर प्रति वर्ष 5 करोड़ रुपए खर्च करने का अधिकार है। 6 मार्च, 2011 से यह लागू की गई है। लोकसभा की शक्तियाँ तथा कार्य (Powers and Functions of Lok Sabha) संविधान के विभिन्न अनुच्छेदों में लोकसभा की महत्त्वपूर्ण शक्तियों का वर्णन किया गया है। लोकसभा के कार्य व शक्तियाँ निम्नलिखित हैं

1. विधायी शक्तियाँ (Legislative Functions):
लोकसभा का मुख्य कार्य कानून बनाना है। कानून-निर्माण में लोकसभा के साथ-साथ राज्यसभा को भी अधिकार दिया गया है। संघ सूची तथा समवर्ती सूची के सभी विषयों पर लोकसभा को कानून बनाने का अधिकार है। यद्यपि समवर्ती सूची पर कानून बनाने का अधिकार राज्य विधानमण्डलों को भी है, परन्तु विवाद की दशा में अन्तिम निर्णय संसद का होता है। विशेष परिस्थिति में राज्य सूची में दिए गए विषयों पर भी लोकसभा कानून बना सकती है, यदि

  • राज्यसभा 2/3 बहुमत से प्रस्ताव पास कर दे,
  • दो अथवा दो से अधिक राज्यों की प्रार्थना पर,
  • राष्ट्रपति द्वारा संकटकाल की घोषणा की गई हो,
  • किसी राज्य में राष्ट्रपति शासन लागू हो गया हो।

2. राष्ट्रीय वित्त पर नियन्त्रण (Controlover National Finance):
राष्ट्रीय वित्तीय व्यवस्था पर लोकसभा को पूर्ण नियन्त्रण प्राप्त है। धन विधेयक पहले लोकसभा में ही प्रस्तुत किया जा सकता है। राज्यसभा धन विधेयक को केवल 14 दिनों तक पास होने से रोक सकती है। इस तरह सरकार को धन देने तथा खर्च करने की स्वीकृति देने वाला सदन लोकसभा ही है।

लोकसभा मन्त्रिमण्डल द्वारा प्रस्तुत बजट को निरस्त कर सकती है, कम कर सकती है, परन्तु उसे बढ़ा नहीं सकती। यदि सरकार का बजट निरस्त कर दिया जाए तो यह सरकार के विरुद्ध अविश्वास प्रस्ताव समझा जाएगा। ऐसी अवस्था में मन्त्रिमण्डल को त्याग-पत्र देना पड़ेगा।

3. कार्यपालिका पर नियन्त्रण (Control Over Executive):
लोकसभा का मन्त्रिमण्डल पर पूरा नियन्त्रण होता है। मन्त्रिमण्डल अपने कार्यकाल तथा कार्यों के लिए लोकसभा के प्रति उत्तरदायी ठहराया गया है। लोकसभा के सदस्य मन्त्रियों से प्रश्न तथा पूरक प्रश्न पूछकर, काम रोको प्रस्ताव पास करके, वेतन में कटौती करके तथा अविश्वास का प्रस्ताव पास करके उन पर नियन्त्रण रखती है।

मन्त्री लोकसभा में बहुमत-प्राप्ति तक ही अपने पदों पर रह सकते हैं। बहुमत समाप्त होते ही उन्हें अपने पद से त्याग-पत्र देना पड़ता है। लोकसभा मन्त्रियों के विभागों की जाँच-पड़ताल के लिए समिति नियुक्त कर सकती है। इस प्रकार लोकसभा की मन्त्रियों पर कड़ी निगरानी रहती है।

4. चुनाव कार्य (Electoral Functions):
लोकसभा के सदस्य अपने सदस्यों में से एक अध्यक्ष तथा एक उपाध्यक्ष का चुनाव करते हैं। इसके अतिरिक्त वे राष्ट्रपति तथा उप-राष्ट्रपति के चुनाव में भाग लेते हैं।

5. न्यायिक कार्य (Judicial Functions):
लोकसभा को राज्यसभा के समान ही राष्ट्रपति तथा उप-राष्ट्रपति, सर्वोच्च तथा उच्च न्यायालय के न्यायाधीशों, महालेखा परीक्षक आदि पर महाभियोग तथा अन्य कार्रवाई द्वारा हटाने का अधिकार है। राज्यसभा के दोषारोपण की जाँच-पड़ताल लोकसभा करती है। उन्हें दोषी पाए जाने पर अपने कुल सदस्यों की संख्या की उपस्थिति तथा मतदान करने वाले सदस्यों के 2/3 बहुमत से प्रस्ताव पास करके उन्हें उनके पद से हटा सकती है।

में संशोधन (Amendment in the Constitution) लोकसभा राज्यसभा के साथ मिलकर संविधान में संशोधन करती है। संविधान में संशोधन के लिए कुल सदस्य संख्या का बहुमत और उपस्थिति तथा मतदान करने वाले सदस्यों का 2/3 बहुमत अनिवार्य हैं। संवैधानिक संशोधनों के प्रति दोनों सदनों में मतभेद को दूर करने के लिए संयुक्त अधिवेशन बुलाने की व्यवस्था नहीं है। राष्ट्रपति को संवैधानिक संशोधनों के विषय में निषेधाधिकार (Veto Power) प्राप्त नहीं हैं।

7. सदन के विशेषाधिकारों के उल्लंघन करने के विषय में शक्ति (Power for Safe guarding the Privileges) लोकसभा को किसी भी ऐसे व्यक्ति को दण्ड देने का अधिकार है, चाहे वह सदन का सदस्य ही क्यों न हो, जो लोकसभा के किसी विशेषाधिकार का उल्लंघन करता है। इसी शक्ति के अधीन 15 नवम्बर, 1978 को लोकसभा की विशेष अधिकारों की समिति ने श्रीमती इन्दिरा गाँधी को सदन के विशेष अधिकारों को भंग करने तथा उसका अपमान करने का दोषी ठहराया था तथा दण्ड के रूप में उनकी सदस्यता समाप्त करके उन्हें अधिवेशन के विसर्जन तक कैद में रखने का आदेश दिया था।

8. विविध कार्य (Miscellaneous Functions)-लोकसभा राज्यसभा के साथ मिलकर दूसरे विविध कार्य भी करती है जो इस प्रकार हैं-

  • राष्ट्रपति की संकटकालीन घोषणा की स्वीकृति लोकसभा से ली जाती है,
  • लोकसभा विभिन्न आयोगों की रिपोर्टों पर विचार करती है,
  • राज्यसभा के प्रस्ताव पर अखिल भारतीय सेवा स्थापित करने की
  • जनता की शिकायतें सरकार के सामने रखती है तथा उन्हें दूर करवाती है,
  • सदन की कार्रवाई के नियम बनाती है,
  • संघ में नए राज्यों को सम्मिलित करना, उनके नामों तथा सीमाओं में परिवर्तन करना,
  • उच्चतम न्यायालय तथा उच्च न्यायालयों के अधिकार क्षेत्र में परिवर्तन करना,
  • अन्तर्राष्ट्रीय समझौतों को लागू करने के लिए कानून बनाती है,
  • दो अथवा दो से अधिक राज्यों के लिए संयुक्त लोक सेवा आयोग तथा उच्च न्यायालय स्थापित करती है।

लोकसभा की ऊपर वर्णित शक्तियों के आधार पर यह कहा जा सकता है कि कानून निर्माण में यह सर्वोपरि है। वित्तीय क्षेत्र में उसकी स्थिति महान है। उसकी अनुमति के बिना सरकार न तो कोई कर लगा सकती है और न ही धन एकत्रित करके खर्च कर सकती है। मन्त्रिमण्डल पर लोकसभा का नियन्त्रण अन्तिम होता है।

वह अपने सभी कार्यों के लिए लोकसभा के प्रति उत्तरदायी होता है। हमारे देश में यह प्रथा विकसित हो चुकी है कि प्रधानमन्त्री लोकसभा से ही लिया जाना चाहिए। प्रधानमन्त्री प्रायः लोकसभा के बहुमत दल का नेता ही होता है। श्री एम०पी० शर्मा के शब्दों में, “यदि संसद राज्य का सर्वोच्च अंग है तो लोकसभा संसद का सर्वोपरि भाग है। व्यावहारिक रूप से लोकसभा ही सभी निर्माणात्मक कार्यों के लिए सर्वोच्च है।”

प्रश्न 5.
लोकसभा के अध्यक्ष के कार्य तथा स्थिति का वर्णन करें।
उत्तर:
संविधान के अनुसार लोकसभा की बैठकों की अध्यक्षता करने के लिए एक अध्यक्ष (Speaker) तथा एक उपाध्यक्ष (Deputy Speaker) की व्यवस्था की गई है। सामान्यतः लोकसभा की बैठकों की अध्यक्षता अध्यक्ष करता है। उसकी अनुपस्थिति में उपाध्यक्ष या डिप्टी अध्यक्ष सदन की अध्यक्षता करता है।

अध्यक्ष का निर्वाचन (Election of the Speaker):
लोकसभा अपने सदस्यों में से अध्यक्ष का चुनाव करती है। साधारणतया शासक दल का नेता विरोधी दल के नेताओं से विचार करके अध्यक्ष का चुनाव सर्वसम्मति से करवाने का प्रयास करता है। वर्तमान लोकसभा के द्वारा श्री ओम बिड़ला को स्पीकर पद पर सर्वसम्मति से निर्वाचित किया गया।

2. कार्यकाल (Tenure):
अध्यक्ष का कार्यकाल लोकसभा के कार्यकाल के समान 5 वर्ष है। लोकसभा भंग होने पर अध्यक्ष तब तक अपने पद पर बना रहता है जब तक नई लोकसभा अपने अध्यक्ष का चुनाव न कर ले।

3. अध्यक्ष के पद का रिक्त होना (Vacancy in Speaker’s Office):
लोकसभा के अध्यक्ष का पद उसके अपने पद से त्याग-पत्र देने से रिक्त हो जाता है। इसके अतिरिक्त यदि उसकी सदस्यता समाप्त हो जाए, उसकी मृत्यु हो जाए, बीमारी अथवा असमर्थता की अवस्था में उसे उसके पद से हटा दिया जाए की स्थिति में उसका पद रिक्त माना जाता है। उसे पद से हटाने का प्रस्ताव सदन में प्रस्तुत करने से पहले उसे 14 दिन का नोटिस दिया जाता है। यदि सदन के कुल सदस्यों के स्पष्ट बहुमत से हटाए जाने का प्रस्ताव पास हो जाए तो अध्यक्ष को अपने पद से त्याग-पत्र देना पड़ता है। जब लोकसभा उसके हटाए जाने के प्रस्ताव पर विचार कर रही हो तो वह बैठक की अध्यक्षता नहीं कर सकता।

वेतन (Salary)-स्पीकर को संसद द्वारा निर्धारित वेतन तथा कई भत्ते मिलते हैं। वर्तमान में लोकसभा अध्यक्ष को 4,00,000 रुपए मासिक वेतन मिलता है। उसे निःशुल्क सरकारी निवास स्थान भी दिया जाता है। ये वेतन तथा भत्ते भारत की संचित निधि से दिए जाते हैं। अभिप्राय यह है कि स्पीकर के वेतन तथा भत्ते उसके कार्यकाल में कम नहीं किए जा सकते। स्पीकर के वेतन तथा भत्ते निश्चित करने का अधिकार केवल संसद को प्राप्त है। इस तरह लोकसभा के स्पीकर का पद बहुत महत्त्वपूर्ण है। उसका देश के सर्वोच्च पदाधिकारियों की सूची में सर्वोच्च न्यायालय के मुख्य न्यायाधीश के समान सातवाँ स्थान है।

अध्यक्ष के कार्य (Functions of the Speaker) लोकसभा के अध्यक्ष के कार्य निम्नलिखित हैं

  • वह सदन की बैठकों की अध्यक्षता करता है,
  • वह यह निर्णय करता है कि किस विधेयक पर किस दिन बहस हो,
  • वह सदस्यों के बोलने का क्रम निश्चित करता है। यदि एक ही समय दो या दो से अधिक सदस्य बोलने के लिए खड़े हो जाएँ तो पहले कौन बोलेगा, उसका निर्णय अध्यक्ष करता है। यदि कोई सदस्य उसकी आज्ञा का पालन न करे तो वह उसे सदन से बाहर जाने को कह सकता है। अध्यक्ष की आज्ञा न मानने पर सदस्य को मार्शल की मदद से बाहर निकलवा सकता है तथा सदस्य को अधिवेशन में भाग लेने से भी रोक सकता है,
  • वह सदन का कार्यक्रम बनाता है, कार्रवाई के नियमों की व्याख्या करता है तथा उनसे सम्बन्धित झगड़ों का निपटारा करता है, (
  • यदि किसी विधेयक पर विवाद हो जाए कि यह साधारण विधेयक है अथवा धन विधेयक तो अध्यक्ष का निर्णय अन्तिम होता है,
  • अध्यक्ष विधेयकों पर वाद-विवाद के पश्चात् मतदान करवाता है तथा निर्णय की घोषणा करता है,
  • वह सदन में विभिन्न प्रस्तावों को पेश करने की अनुमति देता है अथवा किसी प्रस्ताव को पेश करने से इन्कार कर सकता है,
  • किसी विधेयक पर मतभेद की दशा में दोनों सदनों के संयुक्त अधिवेशन की अध्यक्षता अध्यक्ष करता है,
  • वह राष्ट्रपति के सन्देशों को पढ़कर सुनाता है,
  • वह सदन की ओर से राज्यपाल तथा राष्ट्रपति के साथ पत्र-व्यवहार करता है,
  • वह सदन में विभिन्न समितियों का गठन करता है तथा कुछ की अध्यक्षता भी करता है,
  • वह विधेयकों पर मतदान में भाग नहीं लेता, परन्तु मत बराबर होने की दशा में उसे निर्णायक मत देने का अधिकार है,
  • सदन में सभी सदस्य उसकी अनुमति से तथा उसको सम्बोधित करके बोलते हैं,
  • वह सदन में अव्यवस्था होने पर सदन की कार्रवाई स्थगित कर सकता है,
  • सदस्य संसदीय भाषा का प्रयोग करें। अनुचित शब्दों को कार्रवाई से निकलवाने का अध्यक्ष को पूर्ण अधिकार है,
  • यदि कोई व्यक्ति सदन की मानहानि करता है तो उसके विरुद्ध अनुशासनात्मक कार्य करना अध्यक्ष का अधिकार है,
  • वह सदन के विशेषाधिकारों का संरक्षक है।

अध्यक्ष की स्थिति (Position of the Speaker) लोकसभा के अध्यक्ष को बहुत अधिक शक्तियाँ प्राप्त हैं। उसका पद बहुत महत्त्वपूर्ण, गरिमाशाली तथा सम्मान वाला है। एम०वी० पायली का कथन है, “अध्यक्ष सदस्यों के व्यक्तिगत दलीय आधार पर प्राप्त तथा विशेषाधिकारों का संरक्षक है। संक्षेप में अध्यक्ष स्वय सदन की शक्तियों, कारवाई व सम्मान का प्रतीक है।”

भारत के प्रथम प्रधानमन्त्री पं० जवाहरलाल नेहरू ने कहा था “अध्यक्ष सदन का प्रतिनिधित्व करता है। वह सदन के गौरव तथा स्वतन्त्रता का प्रतिनिधित्व करता है और क्योंकि सदन राष्ट्र का प्रतिनिधित्व करता है, अतः अध्यक्ष राष्ट्र की स्वतन्त्रता तथा स्वाधीनता का प्रतीक बन जाता है।” भूतपूर्व अध्यक्ष जी०वी० मावलंकर ने तो यहाँ तक कहा था “सदन में उसकी शक्तियाँ सर्वोच्च हैं।”

भारत के अध्यक्ष को बहुत अधिक शक्तियाँ प्राप्त हैं, परन्तु फिर भी भारतीय अध्यक्ष को ब्रिटिश अध्यक्ष की तरह मान्यता प्राप्त नहीं है। ब्रिटेन में एक बार का अध्यक्ष सदा का अध्यक्ष होता है। (Once a Speaker, always a speaker.) वह कॉमन सदन के लिए निर्विरोध चुना जाता है। भारत में ऐसी प्रथाएँ स्थापित नहीं हुई हैं। फिर भी अभी तक जितने भी अध्यक्ष चुने गए हैं, उन्होंने अपनी निष्पक्षता व कार्यकुशलता से इस पद का सम्मान बढ़ाया है।

निष्कर्ष (Conclusion) लोकसभा के अध्यक्ष की शक्तियों व स्थिति का अध्ययन करने के पश्चात् यह स्पष्ट हो जाता है कि लोकसभा के अध्यक्ष की स्थिति बड़ी गौरवशाली है। लोकसभा के अध्यक्ष की तुलना ब्रिटिश हाऊस ऑफ कॉमन्स के अध्यक्ष से की जा सकती है तथा इस रूप में यह कहा जा सकता है कि भारतीय अध्यक्ष को हाऊस ऑफ अध्यक्ष से अधिक शक्तियाँ प्राप्त हैं। वह भिन्न-भिन्न समितियों की नियुक्ति करता है और स्वयं तीन महत्त्वपूर्ण समितियों का अध्यक्ष भी होता है।

HBSE 11th Class Political Science Important Questions Chapter 5 विधायिका

प्रश्न 6.
लोकसभा तथा राज्यसभा के आपसी सम्बन्धों का वर्णन करें।
अथवा
राज्यसभा दूसरा नहीं, दूसरे दर्जे का सदन है। व्याख्या कीजिए।
उत्तर:
यद्यपि सैद्धान्तिक रूप में संसद के दोनों सदनों को समान अधिकार प्रदान किए गए हैं, परन्तु वास्तव में ऐसा नहीं है। राज्यसभा को लोकसभा की तुलना में कुछ अधिक शक्तियाँ प्राप्त हैं तो कुछ कार्यों में राज्यसभा की शक्तियाँ कम हैं। कुछ शक्तियाँ दोनों सदनों को समान रूप से प्राप्त हैं। इसलिए राज्यसभा को दूसरा सदन कहना तो उचित होगा, परन्तु दूसरे दर्जे का सदन कहना उचित नहीं है। निम्नलिखित तथ्यों से यह स्पष्ट हो जाता है

1. साधारण विधेयक (Ordinary Bills):
साधारण विधेयक संसद के किसी भी सदन में पेश किया जा सकता है। दोनों सदनों की स्वीकृति से ही विधेयक पास होगा, परन्तु अन्तिम शक्ति लोकसभा को प्राप्त है। राज्यसभा साधारण विधेयक को 6 महीने तक रोक सकती है। संयुक्त अधिवेशन की दशा में भी लोकसभा का ही प्रभुत्व रहता है।

2. धन विधेयक (Money Bills):
धन विधेयक पर राज्यसभा का नियन्त्रण बहुत कम है। धन विधेयक लोकसभा में ही पेश हो सकता है। राज्यसभा धन विधेयक को केवल 14 दिनों तक रोक सकती है। राज्यसभा के सुझावों को स्वीकार करना अथवा न करना लोकसभा का अधिकार है।

3. कार्यपालिका पर नियन्त्रण (Control Over Executive):
मन्त्रिमण्डल अपने कार्यों के लिए व्यक्तिगत तथा सामूहिक रूप से लोकसभा के प्रति उत्तरदायी होता है। यद्यपि दोनों सदनों के सदस्यों को मन्त्रियों से प्रश्न तथा पूरक प्रश्न पूछने का अधिकार है। दोनों सदन मन्त्रियों के कार्यों की आलोचना करते हैं, परन्तु अन्तिम शक्ति लोकसभा की है। लोकसभा मन्त्रिमण्डल के विरुद्ध अविश्वास का प्रस्ताव पास करके उसे हटा सकती है, राज्यसभा नहीं, इसलिए कार्यपालिका पर अन्तिम नियन्त्रण लोकसभा का है, राज्यसभा का नहीं।

4. न्यायिक कार्य (Judicial Functions):
राष्ट्रपति, उप-राष्ट्रपति, सर्वोच्च तथा उच्च न्यायालयों के मुख्य न्यायाधीश तथा अन्य प्रशासनिक अधिकारियों को अपने पद से हटाने में दोनों सदनों को समान अधिकार प्राप्त हैं। महाभियोग कोई भी सदन लगा सकता है तथा दूसरा सदन जाँच-पड़ताल करता है। महाभियोग तभी प्रभावकारी होगा यदि दोनों सदनों द्वारा अलग-अलग अपनी कुल सदस्य संख्या के बहुमत तथा उपस्थित और मतदान करने वाले सदस्यों के 2/3 बहुमत से प्रस्ताव प

5. चुनाव कार्य (Electoral Functions):
राष्ट्रपति तथा उप-राष्ट्रपति के चुनावों में दोनों सदनों के सदस्य भाग लेते हैं। राष्ट्रपति के चुनाव में केवल निर्वाचित सदस्यों को भाग लेने का अधिकार होता है, जबकि उप-राष्ट्रपति के चुनाव में मनोनीत सदस्यों को भी मतदान में भाग लेने का अधिकार होता है।

6. संविधान में संशोधन (Constitutional Amendments):
संविधान में संशोधन का अधिकार लोकसभा तथा राज्यसभा को समान रूप से प्रदान किया गया है। संशोधन प्रस्ताव किसी भी सदन में पेश किया जा सकता है। यदि दोनों सदनों में मतभेद हो जाए तो संशोधन प्रस्ताव निरस्त किया जाता है।

7. अखिल भारतीय सेवा की स्थापना (Creation of an All India Service):
देश में यदि कोई नई अखिल भारतीय सेवा की स्थापना की जानी है तो इस आशय का प्रस्ताव राज्यसभा पास करती है, तभी संसद नई अखिल भारतीय सेवा की स्थापना करती है।

8. राज्य-सूची पर कानून बनाना (Enactment on the Subject of State List):
शक्तियों के विभाजन के आधार पर राज्य सूची में दिए गए सभी विषयों पर कानून बनाने का अधिकार राज्य विधानसभाओं का है, परन्तु राज्यसभा के प्रस्ताव पास करने पर राज्य-सूची में दिए गए विषयों पर संसद को कानून बनाने का अधिकार प्राप्त हो जाता है।

9. आपातकाल की स्वीकृति (Consent on the Proclamation of Emergency):
राष्ट्रपति को संकटकाल की घोषणा की स्वीकृति संसद से लेनी आवश्यक है। यदि लोकसभा भंग हो चुकी हो तो राज्यसभा की स्वीकृति लेना आवश्यक है।

10. दोनों सदनों की अपनी विशेषताएँ (Peculiar Features of the Both Houses):
उक्त साझेदारी के अतिरिक्त प्रत्येक सदन की कुछ अलग-अलग विशेषताएँ भी हैं, जैसे

  • लोकसभा के सदस्य प्रत्यक्ष रूप से तथा राज्यसभा के सदस्य अप्रत्यक्ष रूप से चुने जाते हैं,
  • राज्यसभा में 12 व्यक्ति राष्ट्रपति मनोनीत करता है,
  • लोकसभा अपने सदस्यों में से अध्यक्ष का चुनाव करती है। उप-राष्ट्रपति राज्यसभा का पदेन अध्यक्ष होता है,
  • लोकसभा अपने अध्यक्ष को हटा सकती है, राज्यसभा को ऐसा कोई अधिकार प्राप्त नहीं है। उसे लोकसभा की स्वीकृति भी लेनी पड़ती है,
  • धन विधेयक तथा साधारण विधेयक पर विवाद की दशा में अन्तिम निर्णय लोकसभा के अध्यक्ष द्वारा किया जाता है,
  • दोनों सदनों के संयुक्त अधिवेशन की अध्यक्षता लोकसभा का अध्यक्ष करता है,
  • राज्यसभा एक स्थायी सदन है जिसके 1/3 सदस्य प्रति दूसरे वर्ष अपने पद से निवृत्त हो जाते हैं तथा नए सदस्यों का चुनाव किया जाता है।

निष्कर्ष (Conclusion) ऊपर वर्णित तथ्यों के आधार पर हम इस निष्कर्ष पर पहुँचते हैं कि लोकसभा और राज्यसभा का आपस में गहरा सम्बन्ध है। एक सदन सारे कार्य नहीं कर सकता। भारत में दोनों सदनों में न कोई सदन अधिक शक्तिशाली है और न ही कोई सदन कम शक्तिशाली है। यदि कुछ कार्यों में लोकसभा अधिक शक्तिशाली है तो कुछ कार्यों में राज्यसभा।

कुछ कार्य ऐसे भी हैं जिनमें दोनों सदनों को समान अधिकार प्राप्त हैं। अतः भारतीय लोकसभा और राज्यसभा संसद के अभिन्न अंग हैं। इनकी तुलना एक मनुष्य की दो भुजाओं से की जाती है। लोकसभा की तुलना दाएँ हाथ से तथा राज्यसभा की तुलना बाएँ हाथ से की जा सकती है।

प्रश्न 7.
भारत की संसद में अपनाई जाने वाली विधायनी प्रक्रिया का वर्णन करें। अथवा कोई साधारण विधेयक कानून बनने के लिए जिन अवस्थाओं में से गुजरता है, उनका वर्णन करें। अथवा एक विधेयक कैसे अधिनियम बनता है?
उत्तर:
कानून बनाने के लिए जो प्रस्ताव संसद के सामने प्रस्तुत किया जाता है, उसे विधेयक कहा जाता है। जब यह कर दिया जाता है तथा उस पर राष्ट्रपति की स्वीकृति मिल जाती है तब वह अधिनियम अर्थात् कानून बन जाता है। विधेयक दो प्रकार के होते हैं सरकारी विधेयक तथा गैर-सरकारी विधेयक। जो विधेयक सरकार की ओर से अर्थात् मन्त्री द्वारा पेश किया जाता है, उसे सरकारी विधेयक कहा जाता है और जो विधेयक मन्त्री के अतिरिक्त किसी और द्वारा पेश किया जाता है, उसे गैर-सरकारी विधेयक कहा जाता है।

विधेयक चाहे सरकारी हो अथवा गैर-सरकारी, दो तरह के होते हैं-साधारण विधेयक तथा धन विधेयक। साधारण विधेयक सामान्य लोकहित के बारे में होते हैं। प्रशासन को चलाने के लिए कानून बनाने से उनका सम्बन्ध होता है। धन विधेयक उसे कहा जाता है जो कर लगाने तथा सरकारी कोष से धन निकलवाकर खर्च करने के बारे में हो अथवा उनका सम्बन्ध राजस्व से हो। विधेयकों के विभाजन को इस आकृति से आसानी से समझा जा सकता है
HBSE 11th Class Political Science Important Questions Chapter 5 Img 1
एक विधेयक अधिनियम कैसे बनता है? (How does a Bill become an Act ?)

1. विधेयक पेश करना तथा प्रथम वाचन (Introduction and First Reading):
साधारण विधेयक किसी भी सदन में पेश किया जा सकता है। जो भी व्यक्ति विधेयक पेश करना चाहता है, वह इस उद्देश्य की सूचना एक.मास पूर्व अध्यक्ष को देता है। मन्त्रियों पर एक महीने की अवधि लागू नहीं होती। अध्यक्ष उस विधेयक पर कोई तिथि निश्चित कर देता है। निश्चित तिथि को विधेयक पेश करने वाला सदस्य अध्यक्ष की अनुमति से विधेयक पेश करता है। वह विधेयक का शीर्षक पढ़ता है। वह विधेयक के सम्बन्ध में भाषण भी देता है।

यदि शीर्षक के सम्बन्ध में मतभेद हो तो विरोधी दल को अपने विचार रखने का समय दिया जाता है। विधेयक को सरकारी गजट में छाप दिया जाता है। इस विधि को ही विधेयक का पेश करना तथा प्रथम वाचन कहा जाता है। कभी-कभी अध्यक्ष विधेयक को सीधे सरकारी गजट में छापने का आदेश दे सकता है।

इस अवस्था में विधेयक को प्रथम वाचन के लिए सदन में प्रस्तुत करने की आवश्यकता नहीं होती तथा उसका सरकारी गजट में छाप दिया जाना ही प्रथम वाचन मान लिया जाता है। विधेयक के प्रथम वाचन के पश्चात् विधेयक पेश करने वाले सदस्य के सुझाव पर तथा सदन की आज्ञानुसार विधेयक के दूसरे वाचन को आरम्भ करने के लिए तिथि निश्चित की जाती है।

2. दूसरा वाचन (Second Reading):
निश्चित तिथि पर विधेयक पर दूसरा वाचन आरम्भ होता है। विधेयक पेश करने वाला विधेयक पर धारावाहिक बहस करने अथवा विधेयक को किसी समिति के पास भेजने की सिफारिश करता है। विधेयक पर वाद-विवाद केवल विधेयक के साधारण सिद्धान्तों तथा उद्देश्यों तक सीमित रहता है। विधेयक की प्रत्येक धारा पर विस्तारपूर्वक बहस नहीं होती और न ही कोई संशोधन पेश किया जा सकता है। विधेयक पेश करने वाले सदस्य की बात को मानते हुए विधेयक को सम्बन्धित समिति के पास भेज दिया जाता है। विधेयक की कापियाँ सदस्यों में बाँटी जाती हैं।

3. समिति अवस्था (Committee Stage):
विधेयक जिस प्रवर समिति अथवा संयुक्त समिति के पास भेजा जाता है, उसमें सदन के विभिन्न राजनीतिक दलों को सदस्यों के अनुपात से प्रतिनिधित्व दिया जाता है। एक समिति में 20 से 30 तक सदस्य लिए जाते हैं। विधेयक पेश करने वाले को समिति में अवश्य लिया जाता है। समिति में विधेयक पर विस्तार से विचार किया जाता है।

समिति किसी भी व्यक्ति, संस्था तथा समूह को यदि उसके पास विधेयक सम्बन्धी कोई जानकारी है तो उसे अपने पास बुला सकती है। आवश्यकता पड़ने पर विशेषज्ञों की राय भी ली जा सकती है। विधेयक पर गम्भीरतापूर्वक विचार करने के बाद समिति विधेयक को संशोधन सहित अथवा बिना संशोधन के निश्चित समय के अन्दर सदन को वापिस भेज देती है।

4. रिपोर्ट अवस्था (Report Stage) समिति की रिपोर्ट के पश्चात् विधेयक पेश करने वाला प्रस्ताव रख सकता है कि समिति की रिपोर्ट के साथ विधेयक पर विचार किया जाए। उस अवस्था में विधेयक पर व्यापक वाद-विवाद होता है। विधेयक की प्रत्येक धारा पर चर्चा होती है। संशोधन पेश किए जाते हैं। प्रत्येक संशोधन और प्रत्येक धारा को सदन के सामने मतदान के लिए रखा जाता है।

यदि मतदान में धारा स्वीकृत हो जाती है तो वह विधेयक का भाग बन जाती है। जब विधेयक की सभी धाराओं पर विचार पूरा हो जाता है तो मतदान करवाया जाता है। यदि मतदान में विधेयक को बहुमत का समर्थन प्राप्त हो जाए तो उसे पास समझा जाता है अन्यथा वह निरस्त हो जाता है। यह विधेयक की सबसे महत्त्वपूर्ण अवस्था होती है। यही विधेयक का दूसरा वाचन होता है। यदि मन्त्री द्वारा रखा गया विधेयक निरस्त हो जाए तो यह समझा जाएगा कि मन्त्रिमण्डल का बहुमत नहीं है तथा उसे त्याग-पत्र देना पड़ता है। साधारणतया विधेयक इस अवस्था में पास हो जाता है।

5. तीसरा वाचन (Third Reading):
जब विधेयक का दूसरा वाचन समाप्त हो जाता है तो निश्चित तिथि पर विधेयक का तीसरा वाचन आरम्भ होता है। यह केवल औपचारिकता है। इस अवस्था में कोई नया संशोधन प्रस्ताव पेश नहीं किया जाता और न ही प्रत्येक धारा पर बहस होती है। विधेयक के सामान्य सिद्धान्तों पर केवल बहस होती है। इसके बाद विधेयक पर एक बार मतदान करवाया जाता है। इस अवस्था में विधेयक प्रायः स्वीकार ही कर लिया जाता है। इसके साथ ही विधेयक एक सदन में पास समझा जाता है।

6. विधेयक दूसरे सदन में (Bill in the other House):
एक सदन में पास होने के पश्चात् विधेयक को दूसरे सदन में भेजा जाता है। दूसरे सदन में भी विधेयक उन्हीं अवस्थाओं से गुजरता है, जिन अवस्थाओं से पहले सदन में गुजरा था। यदि दूसरा सदन विधेयक को उसी तरह पास कर दे तो विधेयक दोनों सदनों में पास समझा जाता है। मतभेद होने की दशा में राज्यसभा किसी भी साधारण विधेयक को 6 महीने तक रोक सकती है।

गतिरोध की दशा में राष्ट्रपति दोनों सदनों का संयुक्त अधिवेशन भी बुला सकता है। संयुक्त अधिवेशन में भी लोकसभा की बात को ही स्वीकार किया जाता है। लोकसभा के सदस्यों की संख्या भी अधिक होती है। संयुक्त अधिवेशन की अध्यक्षता लोकसभा का अध्यक्ष करता है।

7. राष्ट्रपति की स्वीकृति (Assent of the President):
संसद के दोनों सदनों में विधेयक पास होने पर उसे अन्तिम स्वीकृति के लिए राष्ट्रपति के पास भेजा जाता है। राष्ट्रपति की स्वीकृति मिल जाने पर वह विधेयक कानून बन जाता है तथा उसे भारतीय गजट में छाप दिया जाता है। राष्ट्रपति को निषेधाधिकार प्राप्त है, जिसके अन्तर्गत वह किसी विधेयक को संसद को दोबारा विचार करने के लिए भेज सकता है। यदि संसद राष्ट्रपति के सुझाव को स्वीकार करते हुए अथवा अस्वीकार करते हुए विधेयक को दोबारा बहुमत से पास कर दे तो राष्ट्रपति को स्वीकृति देनी पड़ती है। राष्ट्रपति बिना स्वीकृति दिए एक विधेयक को अपने पास कितने समय तक रोक सकता है, इस विषय में देश का संविधान मौन है।

HBSE 11th Class Political Science Important Questions Chapter 5 विधायिका

प्रश्न 8.
भारत की संसद में धन विधेयक कैसे पास होता है? अथवा वित्त विधेयक को पारित करने की प्रक्रिया का वर्णन करें।
उत्तर:
वित्त विधेयक को पारित करने की विधि साधारण विधेयक की विधि से भिन्न है। वित्त विधेयक केवल लोकसभा में ही पेश किया जा सकता है, राज्यसभा में नहीं। दूसरे, धन विधेयक सरकारी विधेयक होता है जो वित्तमन्त्री अथवा उसकी अनुपस्थिति में किसी और मन्त्री द्वारा पेश किया जाता है। धन विधेयक उस विधेयक को कहा जाता है जिसका निम्नलिखित विषयों से सम्बन्ध होता है-

  • किसी प्रकार का कर लगाना, घटाना, बढ़ाना तथा समाप्त करना,
  • भारत सरकार द्वारा ऋण लेने, गारण्टी देने तथा वित्तीय उत्तरदायित्व के लिए कानून बनाना,
  • भारत की संचित निधि से किसी व्यय के लिए धनराशि देना,
  • भारत की संचित निधि तथा आकस्मिक निधि का संरक्षण या निधि में धन जमा करना या निकालना,
  • किसी सरकारी खर्च को भारत की संचित निधि में से खर्च योग्य घोषित करना अथवा उसमें से किसी व्यय के लिए धन-राशि देना।

संक्षेप में, धन विधेयक का सम्बन्ध सरकार से धन प्राप्त करने अथवा खर्च करने अथवा सरकारी निधियों के संरक्षण या उसमें से व्यय से होता है। धन विधेयक लोकसभा में ही पेश किया जाता है तथा किसी विधेयक पर विवाद की दशा में लोकसभा के अध्यक्ष का निर्णय अन्तिम होता है। राज्यसभा धन विधेयक को 14 दिन के अन्दर अपनी सिफारिशों व सुझावों सहित लोकसभा को वापिस लौटा देती है। लोकसभा राज्यसभा के सुझावों को मानने के लिए बाध्य नहीं है। राष्ट्रपति धन विधेयक पर हस्ताक्षर करने से इन्कार नहीं कर सकता, क्योंकि धन विधेयक राष्ट्रपति की पूर्व स्वीकृति से ही लोकसभा में पेश किया जाता है।

साधारणतया धन विधेयक बजट के रूप में पेश किया जाता है। आवश्यकता पड़ने पर अतिरिक्त धन की माँग के लिए बजट पास होने से पहले तथा बाद में भी धन विधेयक के रूप में विधेयक पेश किया जा सकता है। बजट को वार्षिक वित्तीय विवरण (Annual Financial Statement) भी कहा जाता है। प्रत्येक वित्तीय वर्ष के लिए सरकार बजट तैयार करती है जिसे राष्ट्रपति की ओर से वित्तमन्त्री लोकसभा में पेश करता है। बजट के मुख्य रूप से दो भाग होते हैं-आमदनी का भाग तथा खर्च का भाग। खर्चे के अनुमान दो भागों में दिखाए जाते हैं। एक भाग में संचित निधि से किया जाने वाला खर्च होता है।

इन खर्चों पर संसद में मतदान नहीं होता। दूसरे भाग में अनुदान सम्बन्धी माँग (Demand for Grants) होती है। इन्हें साधारण खर्च भी कहा जाता है। अनुदान सम्बन्धी माँगों पर संसद में बहस होती है। संसद इन माँगों को स्वीकार तथा अस्वीकार कर सकती है। इन माँगों में कटौती भी की जा सकती है।

राष्ट्रपति, उप-राष्ट्रपति, उच्चतम न्यायालय के न्यायाधीश, संघीय लोक सेवा आयोग के सदस्य, नियन्त्रक तथा महालेखा परीक्षक, भारत सरकार द्वारा दिए गए कर्जे, लोकसभा के अध्यक्ष तथा उपाध्यक्ष, राज्यसभा के उपाध्यक्ष तथा अन्य अधिकारियों के वेतन, भत्ते तथा पेन्शन आदि भारत की संचित निधि से दिए जाते हैं। बजट राष्ट्रपति के अभिभाषण से आरम्भ होता है। राष्ट्रपति दोनों सदनों के संयुक्त अधिवेशन में भाषण देता है जिसमें सरकार की नीतियों की चर्चा की जाती है।

बजट को पारित होने के लिए निम्नलिखित अवस्थाओं में से गुजरना पड़ता है

1. बजट को पेश करना (Introduction of the Budget):
वित्तमन्त्री लोकसभा में बजट पेश करता है। वित्तमन्त्री बजट के सम्बन्ध में भाषण देता है जिसमें बजट की प्रकृति और आर्थिक नीति पर प्रकाश डाला जाता है। इसके बाद बजट भाषण प्रतियाँ राज्यसभा में भेज दी जाती हैं।

2. बजट पर सामान्य वाद-विवाद (General Discussion on Budget):
वित्तमन्त्री के बजट भाषण के दो-तीन दिन पश्चात् बजट पर सामान्य वाद-विवाद होता है। सरकार की आर्थिक नीतियों की आलोचना की जाती है। इस समय उन खर्चों पर भी वाद-विवाद होता है जो संचित निधि से किए जाने हैं।

3. माँगों पर मतदान (Demand for Grants):
सामान्य वाद-विवाद समाप्त होने के पश्चात लोकसभा अनुदान सम्बन्धी माँगों पर विचार करती है। प्रत्येक मन्त्रालय की माँगों पर अलग-अलग विचार किया जाता है। मन्त्रालय के पिछले वर्ष के कार्य की समीक्षा की जाती है। मोरिस,जोन्स का कहना है कि यह ऐसा अवसर होता है कि जिस पर सदन अपने मन की बात खुलकर कह सकता है तथा इस अवसर पर सरकार को यह पता चल जाता है कि वित्तीय सुझावों की माँगों को किस रूप में स्वीकार किया जाएगा। लोकसभा को इन अनुदानों को स्वीकार करने, अस्वीकार करने तथा कम करने का अधिकार होता है। लोकसभा माँगों को कम कर सकती है, बढ़ा नहीं सकती।

4. विनियोग विधेयक (Appropriation Bill):
जब अनुदान सम्बन्धी माँगें लोकसभा द्वारा स्वीकृत हो जाती हैं तो संचित निधि से होने वाले खर्चों के साथ ये माँगें एक विनियोग विधेयक के रूप में लोकसभा में पेश की जाती हैं। इसका अर्थ यह होता है कि लोकसभा द्वारा स्वीकृत सभी माँगों तथा संचित निधि पर पारित खर्चे को खर्च करने का अधिकार सरकार को मिल गया है। इस विधेयक के स्वीकृत हुए बिना सरकार कोई पैसा खर्च नहीं कर सकती।

5. वित्त विधेयक (Finance Bill):
विनियोग विधेयक के पास हो जाने पर सरकार को खर्च करने का अधिकार मिल जाता है, परन्तु इन खर्चों के लिए पैसों की आवश्यकता पड़ती है इसलिए सरकार वित्त विधेयक पेश करती है जिसमें करों तथा अन्य साधनों की व्यवस्था की जाती है। प्रस्तावित करों तथा साधनों के विषय में लोकसभा में विचार होता है तथा संशोधन भी रखे – जाते हैं। नए करों को स्वीकार तथा अस्वीकार किया जा सकता है। लोकसभा करों की माँग को बढ़ा नहीं सकती। विधेयक के पास होने पर सरकार को जनता पर नए कर लगाने का अधिकार प्राप्त हो जाता है।

6. बजट राज्यसभा में (Budget in the Rajya Sabha):
बजट लोकसभा में पास होने के पश्चात् राज्यसभा को भेजा जाता है। राज्यसभा धन विधेयक को 14 दिन तक विचार के लिए रख सकती है। राज्यसभा को सुझाव देने का अधिकार है, परन्तु सुझावों को मानना अथवा न मानना लोकसभा का कार्य है।

7. राष्ट्रपति की स्वीकृति (Assent of the President):
दोनों सदनों में पास होने के पश्चात विधेयक को अन्तिम स्वीकति के लिए राष्ट्रपति के पास भेजा जाता है। धन विधेयकों पर राष्ट्रपति को निषेधाधिकार प्राप्त नहीं हैं, क्योंकि धन विधेयक उसकी पूर्व स्वीकृति से ही लोकसभा में पेश किए जाते हैं। ऊपर वर्णित प्रक्रिया द्वारा भारत की संसद में धन विधेयक अथवा बजट पास होता है।

लेखानुदान तथा पूरक अनुदान (Vote on Account and Supplementary Grants):
यदि किसी वित्तीय वर्ष के आरम्भ होने से पहले विनियोग विधेयक पास न किया जा सके तो सरकार के पास आवश्यक व्यय के लिए कोई धनराशि नहीं होगी, इसलिए यह व्यवस्था की गई है कि कुछ आवश्यक खर्चों के लिए सरकार को सीमित रकम दे दी जाए, इसे लेखानुदान कहते हैं। वर्ष 1991-92 के वित्तीय वर्ष के शुरू होने से पहले चन्द्रशेखर सरकार ने इसी का सहारा लिया था।

यदि किसी कार्य के लिए निश्चित की हुई रकम या धनराशि कम प्रतीत हो तो लोकसभा के सामने पूरक अनुदान (Supplementary Grants) की माँग पेश की जाती है। इनको पास करने का तरीका वही है जो बजट या अन्य धन विधेयकों पर लागू होता है।

प्रश्न 9.
राज्य विधान सभा की रचना, शक्तियों तथा कार्यों का वर्णन करें।
उत्तर:
सभी राज्यों में कानून निर्माण के लिए विधानमण्डल की व्यवस्था की गई है। कुछ राज्यों में विधानमण्डल के दो सदन हैं और कुछ में केवल एक ही सदन है। विधान सभा विधानमण्डल का निम्न सदन है जो जनता का प्रतिनिधित्व करता है।

विधानसभा का संगठन (Composition of Legislative Assembly):
संविधान के अनुसार राज्य विधानसभा के सदस्यों की संख्या निश्चित नहीं की गई है। अनुच्छेद 170 में केवल इतना कहा गया है कि किसी राज्य की विधानसभा के सदस्यों की संख्या 500 से अधिक तथा 60 से कम नहीं होगी। प्रत्येक जनगणना के पश्चात् राज्य की विधानसभा के सदस्यों की संख्या राज्य की जनसंख्या के आधार पर निश्चित की जाती है।

इस कारण से यह संख्या भिन्न-भिन्न राज्यों में भिन्न-भिन्न है। संविधान के 42वें संशोधन द्वारा यह व्यवस्था की गई है कि सन 2000 के पश्चात होने वाली प्रथम जनगणना तक प्रत्येक राज्य के चुनाव-क्षेत्रों के विभाजन के लिए वही आँकड़े लिए जाएँगे जो 1971 की जनगणना के अनुसार निश्चित तथा प्रकाशित हो चुके हैं।

31 अगस्त, 2000 को केन्द्रीय मन्त्रिमण्डल ने राष्ट्रीय जनसंख्या नीति के तहत महत्त्वपूर्ण निर्णय एवं संसद द्वारा अगस्त 2001 में पास किए संवैधानिक संशोधन द्वारा यह व्यवस्था की गई कि लोकसभा व विधानसभाओं के निर्वाचन क्षेत्रों की संख्या सन् 2026 तक बढ़ाए बिना उनका परिसीमन किया जाएगा जिससे विभिन्न क्षेत्रों में मतदाताओं और प्रतिनिधियों के अनुपात में समानता लाई जा सके।

इसके अतिरिक्त अनुसूचित जाति एवं जनजाति के लिए आरक्षित सीटों की संख्या का आबादी के आधार पर पुनर्निर्धारण करने का निर्णय भी हुआ जो 1991 की जनसंख्या के आधार पर लागू होगा। यहाँ यह भी स्पष्ट है कि दिसम्बर, 2019 में पास किए गए 104वें संवैधानिक संशोधन के अनुसार लोकसभा के साथ-साथ राज्यों की विधानसभाओं में भी आरक्षण की व्यवस्था को 25 जनवरी, 2030 तक 10 वर्षों के लिए पूर्व की भाँति बढ़ा दिया गया है।

राज्यों में विधानसभा के सदस्यों की संख्या

राज्य का नाम सदस्यों की संख्या
1. आन्ध्र प्रदेश 175
2. असम 126
3. बिहार 243
4. गुजरात 182
5. हरियाणा 90
6. हिमाचल प्रदेश 68
7. कर्नाटक 224
8. केरल 140
9. मध्य प्रदेश 230
10. महाराष्ट्र 288
11. मणिपुर 60
12. मेघालय 60
13. नागालैण्ड 60
14. ओडिशा 147
15. पंजाब 117
16. राजस्थान 200
17. सिक्किम 32
18. तमिलनाडु 234
19. तेलंगाना 119
20. त्रिपुरा 60
21. उत्तर प्रदेश 403
22. पश्चिम बंगाल 294
23. मिज़ोरम 40
24. अरुणाचल प्रदेश 60
25. गोवा 40
26. छत्तीसगढ़ 90
27. उत्तराखण्ड 70
28. झारखण्ड 81
संघ क्षेत्र
1. अण्डमान और निकोबार द्वीप समूह
2. चण्डीगढ़
3. दादरा नागर हवेली और दमन दीव
4. लक्षद्वीप
5. पुद्दुचेरी 30
6. दिल्ली 50
7. जम्मू-कश्मीर 107
8. लद्दाख

विधान सभा के सदस्यों का चुनाव (Election of the members of Legislative Assembly)-राज्य की विधान सभा के सदस्यों का चुनाव प्रत्यक्ष रूप से होता है। यह चुनाव वयस्क मताधिकार के आधार पर राज्य की जनता द्वारा किया जाता है। प्रत्येक 18 वर्ष का या इससे अधिक आयु का नागरिक विधान सभा के सदस्यों के चुनाव में मत देने का अधिकार रखता है, परन्तु विदेशी, पागल, दिवालिए तथा भयंकर अपराधियों को इस चुनाव में भाग लेने का अधिकार नहीं है।

यह चुनाव संयुक्त चुनाव प्रणाली के आधार पर होता है, परन्तु कुछ पिछड़ी जातियों और कबीलों के लिए विधान सभा में कुछ सीटें सुरक्षित कर दी गई हैं। इसके अतिरिक्त यदि राज्यपाल यह अनुभव करे कि एंग्लो-इण्डियन जाति को विधान सभा में उचित प्रतिनिधित्व नहीं मिला तो वह इस जाति के एक व्यक्ति को जिसे हे योग्य समझे, विधान सभा के सदस्य के रूप में मनोनीत कर सकता है।

एंग्लो-इण्डियन अनुसूचित जातियों, जनजातियों के लिए स्थान सुरक्षित रखने की व्यवस्था सन् 2000 तक की गई थी, लेकिन अब दिसंबर, 2019 को लोकसभा ने सर्वसम्मति से 104वें संवैधानिक संशोधन पास किया जिसके द्वारा अनुसूचित जातियों व जनजातियों के स्थानों को सुरक्षित करने की व्यवस्था को वर्ष 2030 तक बढ़ा दिया गया है।

योग्यताएँ (Qualifications) संविधान में राज्य विधान सभा का सदस्य चुने जाने वाले व्यक्ति में दी गई योग्यताएँ होनी चाहिए-

  • वह भारत का नागरिक हो,
  • वह 25 वर्ष की आयु पूरी कर चुका हो,
  • वह संघ सरकार या राज्य सरकार के अधीन किसी लाभ के पद पर कार्य न कर रहा हो,
  • उसका नाम उस राज्य के मतदाताओं की सूची में हो,
  • वह पागल या दिवालिया । न हो,
  • वह किसी न्यायालय द्वारा इस पद के लिए अयोग्य घोषित न किया गया हो,
  • इसके अतिरिक्त समय-समय पर विधान सभा द्वारा निश्चित की गई सभी योग्यताएँ उसमें हों,
  • जुलाई, 1996 में जन-प्रतिनिधि कानून में संशोधन करके यह व्यवस्था की गई कि स्वतन्त्र उम्मीदवार तभी चुनाव लड़ सकता है, यदि उसका नाम दस मतदाताओं द्वारा प्रस्तावित किया गया हो।

कार्यकाल (Tenure) लोकसभा की तरह ही राज्यों की विधान सभाओं का चुनाव पाँच वर्ष के लिए होता है। 42वें संशोधन के अनुसार विधान सभा की अवधि 6 वर्ष कर दी गई थी लेकिन 44वें संवैधानिक संशोधन के अनुसार फिर यह अवधि 5 वर्ष कर . दी गई है। विधान सभा को अवधि समाप्त होने से पहले भी भंग किया जा सकता है। राज्यपाल मुख्यमन्त्री की सलाह से अथवा बिना सलाह के भी विधान सभा को भंग कर सकता है और नए चुनाव करा सकता है।

जैसे हरियाणा में श्री बंसी लाल की सरकार से भारतीय जनता पार्टी के द्वारा अपना समर्थन वापस लिए जाने पर सरकार अल्पमत में आ गई तथा राज्यपाल ने मुख्यमन्त्री को सदन के पटल पर बहुमत सिद्ध करने के लिए कहा था। श्री बंसी लाल बहुमत सिद्ध नहीं कर पाए और सरकार गिर गई। तत्पश्चात् श्री ओम प्रकाश चौटाला मुख्यमन्त्री बने और उन्होंने दिसम्बर, 1999 में राज्यपाल को विधान सभा भंग करने की सिफारिश की। इस प्रकार विधान सभा को उसकी अवधि समाप्त होने से पहले भंग कर दिया गया।

गणपूर्ति (Quorum):
विधान सभा की बैठकों में कार्रवाई प्रारम्भ होने के लिए यह आवश्यक है कि विधान सभा की कुल सदस्य-संख्या का 1/10वाँ भाग उपस्थित हो।

अधिवेशन (Session):
राज्यपाल विधान सभा और विधान परिषद् के अधिवेशन किसी समय भी बुला सकता है, परन्तु दो अधिवेशनों के बीच 6 मास से अधिक समय का अन्तर नहीं होना चाहिए। राज्यपाल दोनों सदनों का सत्रावसान भी कर सकता है।

वेतन एवं भत्ते (Salary and Allowances):
राज्य विधानसभा के सदस्यों के वेतन एवं भत्ते राज्य विधानमण्डल के द्वारा समय-समय पर पारित विधेयकों के द्वारा निश्चित किए जाते हैं। इसलिए प्रत्येक राज्य में वेतन एवं भत्ते अलग-अलग होंगे; जैसे दिल्ली विधानसभा के सदस्यों को 50,000 मूल वेतन सहित कुल 2,35,000 रुपए मासिक वेतन मिलता है।

इसी तरह 5 मई, 2017 को हरियाणा विधानसभा (सदस्य वेतन, भत्ते तथा पेंशन) संशोधन विधेयक, 2017 के द्वारा किए गए संशोधन अनुसार विधानसभा सदस्यों को 60 हजार रुपए मासिक वेतन, कार्यालय भत्ता 20,000 रुपए, दैनिक भत्ता 2000 रुपए प्रतिदिन जो अधिकतम 30,000 रुपए आदि प्रदान किए जाते हैं। यहाँ यह भी स्पष्ट है कि प्रत्येक राज्य में विधानमण्डल के द्वारा अपने सदस्यों के वेतन भत्तों के संबंध में अलग-अलग प्रस्ताव पारित किए जाते हैं। इसलिए सभी राज्यों में राज्य विधानमण्डल के सदस्यों (MLA) के वेतन भत्ते भी अलग-अलग होते हैं।

विधान सभा का अध्यक्ष (Speaker) लोकसभा के सदस्यों की भांति विधान सभा के सदस्य भी अपने में से एक अध्यक्ष (Speaker) तथा एक उपाध्यक्ष चुनते हैं। साधारणतः अध्यक्ष तथा उपाध्यक्ष अगली विधान सभा के बनने तक अपने पद पर बने रहते हैं, परन्तु विधान सभा के सदस्य अध्यक्ष तथा उपाध्यक्ष को बहुमत के प्रस्ताव पास करके उनको पद से हटा सकते हैं।

ऐसा प्रस्ताव पास करने से 14 दिन पहले इसकी सूचना देनी आवश्यक होती है। विधान सभा के अध्यक्ष की शक्तियाँ और कार्य लोकसभा के अध्यक्ष के समान ही हैं। वह विधान सभा की बैठकों की अध्यक्षता करता है। वह सदस्यों को बोलने, प्रश्न पूछने तथा प्रस्ताव पेश करने की आज्ञा देता है। विधेयकों पर मतदान कराकर परिणाम की घोषणा करता है। कोई विधेयक धन-विधेयक है या नहीं, इसका फैसला भी अध्यक्ष ही करता है। साधारणतया वह मतदान में भाग नहीं लेता, परन्तु उसे एक निर्णायक मत डालने का अधिकार होता है।

विधान सभा की शक्तियाँ तथा कार्य (Powers and Functions of the Legislative Assembly):
जिन राज्यों में केवल विधान सभा है, वहाँ विधानमण्डल की सभी शक्तियाँ विधान सभा प्रयोग करती है। जहाँ विधान परिषद् है, वहाँ भी वास्तविक शक्तियाँ विधान सभा के पास ही हैं। ये शक्तियाँ निम्नलिखित हैं

1. विधायी शक्तियाँ (Legislative Powers):
विधान सभा को राज्य सूची के 66 विषयों तथा समवर्ती सूची के 52 विषयों पर भी कानून बनाने का अधिकार है। समवर्ती सूची पर बनाया गया कानून यदि संघीय संसद द्वारा बनाए गए कानून का विरोध करता हो तो वह निरस्त समझा जाएगा। यदि विधानमण्डल द्वि-सदनीय है तो विधेयक वहाँ से विधान परिषद् के पास जाता है। विधान परिषद् यदि उसे निरस्त कर दे या तीन महीने तक उस पर कोई कार्रवाई न करे या उसमें ऐसे संशोधन कर दे जो विधान सभा को स्वीकृत न हों तो विधान सभा उस विधेयक को दोबारा पास कर सकती है और उसे दोबारा विधान परिषद् के पास भेजा जाता है।

यदि विधान परिषद् उस विधेयक पर दोबारा एक महीने तक कोई कार्रवाई न करे या उसे निरस्त कर दे या उसमें ऐसे संशोधन कर दे जो विधान सभा को स्वीकृत न हों तो तीनों अवस्थाओं में यह विधेयक संसद के दोनों सदनों में पास हुआ समझा जाता है। यदि राज्य में विधान परिषद् नहीं है जैसे हरियाणा में, तो विधान सभा के पास होने के बाद ही अन्यथा दोनों सदनों से पास होने पर विधेयक राज्यपाल को भेजा जाता है। राज्यपाल उस पर अपनी स्वीकृति दे सकता है, उसे दोबारा विचार करने के लिए वापस भेज सकता है और राष्ट्रपति की स्वीकृति के लिए भी भेज सकता है। यदि विधानमण्डल उसे दोबारा पास करके भेजे तो राज्यपाल को अपनी स्वीकृति देनी ही पड़ती है।

2. वित्तीय शक्तियाँ (Financial Powers):
वित्तीय विधेयक केवल विधान सभा में ही पेश हो सकता है। राज्य की आमदनी व खर्च पर विधान सभा का पूरा नियन्त्रण होता है। विधान परिषद् वित्त या धन विधेयक को अधिक-से-अधिक 14 दिन तक रोक सकती है। यदि इसके बाद विधान परिषद् उसे निरस्त कर दे या उसमें कुछ संशोधन कर दे तो भी वह विधेयक उसी रूप में पास समझा जाता है जिस रूप में विधान सभा में पास हुआ था। इसके बाद धन विधेयक राज्यपाल की स्वीकृति के लिए भेज दिया जाता है। राज्यपाल धन विधेयक पर अपनी स्वीकृति देने से इन्कार नहीं कर सकता, क्योंकि धन विधेयक उसकी पूर्व स्वीकृति से ही पेश होते हैं।

3. कार्यपालिका पर नियन्त्रण (Control over the Executive):
राज्य की मन्त्रिपरिषद् विधान सभा के प्रति उत्तरदायी है। विधान सभा में बहुमत दल अथवा दलीय गठबन्धन का नेता ही मुख्यमन्त्री पद सम्भालता है। अधिकांश मन्त्री विधान सभा से लिए जाते हैं। विधान सभा अविश्वास का प्रस्ताव पास कर मन्त्रि-परिषद् को पद से हटा सकती है। विधान सभा मन्त्रियों से उनके कार्यों तथा नीतियों के बारे में प्रश्न पूछती है।

यदि मन्त्रि-परिषद् द्वारा पेश किए गए किसी विधेयक को विधान सभा निरस्त कर दे तो मन्त्रिपरिषद् को त्याग-पत्र देना पड़ता है। विधान सभा मन्त्रियों के वेतन में कटौती करके अथवा किसी सरकारी विधेयक को अस्वीकार करके भी मन्त्रि-परिषद् को त्याग-पत्र देने के लिए मजबूर कर सकती है।

4. चुनाव सम्बन्धी शक्तियाँ (Electoral Powers):
विधान सभा के सदस्य राष्ट्रपति के चुनाव में भाग लेते हैं। विधान सभा के सदस्यों द्वारा ही राज्यसभा के सदस्य चुने जाते हैं। यदि राज्य में विधान परिषद् हो तो उसके 1/3 सदस्य विधान सभा द्वारा चुने , जाते हैं। विधान सभा के सदस्य अपने में से ही एक अध्यक्ष तथा एक उपाध्यक्ष का निर्वाचन करते हैं।

5. संविधान संशोधन में भाग (Participation in Constitutional Amendment):
संविधान के कुछ महत्त्वपूर्ण अनुच्छेदों में संशोधन करने के लिए संसद के दोनों सदनों से 2/3 बहुमत से प्रस्ताव पास होने के बाद उस पर कम-से-कम आधे राज्यों का समर्थन आवश्यक है। इस प्रकार संविधान के एक भाग में संशोधन प्रक्रिया में विधान सभा भाग लेती है। राज्य विधान सभाओं को संशोधन प्रस्ताव पेश करने का अधिकार नहीं है।

6. अन्य शक्तियाँ (Other Powers):
राज्य की विधान सभाओं को कुछ और भी कार्य करने पड़ते हैं जो कि इस प्रकार हैं

(1) विधान सभा 2/3 बहुमत से प्रस्ताव पास करके विधान परिषद् की स्थापना या समाप्ति की प्रार्थना कर सकती है और संसद इस प्रार्थना के अनुसार कानून बनाती है। फरवरी, 1989 में तमिलनाडु की विधान सभा ने विधान परिषद् की स्थापना के लिए प्रस्ताव पारित किया था,

(2) विधान सभा विधान परिषद् के साथ मिलकर (यदि विधान परिषद् हो) लोक सेवा आयोग की शक्तियों को बढ़ा सकती है,

(3) विधान सभा विधान परिषद् के साथ मिलकर आकस्मिक कोष स्थापित कर सकती है,

(4) राज्य की विधान सभा किसी व्यक्ति को सदन के विशेषाधिकारों के उल्लंघन करने पर दण्ड दे सकती है,

(5) यदि कोई सदस्य विधान सभा का अनुशासन भंग करता है और सदन की कार्रवाई शान्तिपूर्वक नहीं चलाने देता, तब सदन उस सदस्य को सदन से निलम्बित कर सकता है,

(6) विधान सभा विरोधी दल के नेता को सरकारी मान्यता और अन्य सुविधाएँ देने के लिए विधेयक पास कर सकती है।

विधान सभा की स्थिति (Position of the Legislative Assembly) राज्यों में विधान सभा की स्थिति बड़ी महत्त्वपूर्ण है। यह राज्य की सभी विधायी शक्तियों का प्रयोग करती है। जहाँ विधान परिषद् है तो भी वह इतनी शक्तिशाली नहीं है कि विधान सभा के रास्ते में रुकावट बन सके। विधान परिषद् का जीवन विधान सभा की इच्छा पर निर्भर करता है।

विधान सभा की शक्तियों पर सीमाएँ (Limitations on the Powers of Legislative Assembly)-विधान सभा उतनी शक्तिशाली नहीं जितनी कि केन्द्र में संसद है। इसकी शक्तियों तथा कार्यों पर दी गई कुछ सीमाएँ इस प्रकार हैं-

(1) राज्य विधान सभा या विधानमण्डल द्वारा पास किए गए कुछ विधेयकों पर राज्यपाल अपनी स्वीकृति नहीं दे सकता, बल्कि उन्हें राष्ट्रपति की स्वीकृति के लिए भेजता है, जैसे भूमि या सम्पत्ति को हस्तगत करने वाला विधेयक,

(2) कुछ विधेयक राष्ट्रपति की पूर्व स्वीकृति के. बिना राज्य विधानमण्डल में पेश नहीं किए जा सकते, जैसे कि अन्तर्राज्यीय व्यापार को नियन्त्रण करने वाला विधेयक,

(3) राज्यपाल किसी भी विधेयक को राष्ट्रपति की स्वीकृति के लिए भेज सकता है, यदि वह समझे कि ऐसा करना उचित है,

(4) राज्यसभा प्रस्ताव पास करके राज्य सूची के किसी विषय पर एक वर्ष के लिए कानून बनाने का अधिकार संसद को दे सकती है,

(5) आपात् काल के समय उसकी सभी शक्तियाँ संसद के पास चली जाती हैं।

प्रश्न 10.
विधान परिषद् की रचना, शक्तियों तथा कार्यों का वर्णन करें।
उत्तर:
विधान परिषद् की रचना (Composition of Legislative Council) भारत के 28 राज्यों में से केवल 6 राज्यों में विधान परिषद् की स्थापना की गई है। संविधान के अनुसार किसी भी राज्य में विधान परिषद् के सदस्यों की संख्या विधानसभा के सदस्यों के 1/3 से अधिक तथा 40 से कम नहीं हो सकती। यहाँ यह उल्लेखनीय है कि जम्मू-कश्मीर राज्य पुनर्गठन अधिनियम, 2019 के द्वारा जम्मू-कश्मीर को 8वें एवं लद्दाख को 9वें केन्द्र-शासित प्रदेश बनाए जाने के बाद जम्मू-कश्मीर केन्द्र शासित प्रदेश को विधानसभा वाला केन्द्र-शासित प्रदेश बनाने की घोषणा भी की गई।

यद्यपि यहाँ यह उल्लेखनीय है कि दादरा नगर हवेली और दमन दीव का विलय कर इन्हें एक ही केन्द्रशासित क्षेत्र के रूप में रूपान्तरित करने के लिए दादरा नगर हवेली और दमन-दीप (संघ राज्य क्षेत्रों का विलय) विधेयक, 2019 को संसद द्वारा पास करने के उपरान्त 9 दिसम्बर, 2019 को राष्ट्रपति की स्वीकृति मिलने के पश्चात् 26 जनवरी, 2020 को यह विलय प्रभावी हो गया, जिसके परिणामस्वरूप केन्द्र शासित प्रदेशों की संख्या 9 से घटकर 8 हो गई है। इस तरह जम्मू-कश्मीर में पूर्व में गठित विधानपरिषद् स्वतः ही समाप्त हो गई है।

राज्य विधान परिषद के कुल सदस्यों में से:
(1) लगभग 1/3 सदस्य स्थानीय संस्थाओं के सदस्यों द्वारा निर्वाचित किए जाते हैं,

(2) लगभग 1/12 सदस्य राज्य में रहने वाले ऐसे स्नातक मतदाताओं द्वारा निर्वाचित किए जाते हैं जिनको भारत के किसी विश्वविद्यालय से डिग्री प्राप्त किए तीन वर्ष हो चुके हों,

(3) लगभग 1/12 सदस्य राज्य के हायर सैकेण्डरी स्कूलों अथवा इससे उच्च शिक्षा संस्थाओं में से कम-से-कम तीन वर्ष से कार्य कर रहे उन अध्यापकों द्वारा निर्वाचित किए जाते हैं,

(4) लगभग 1/3 सदस्य सम्बन्धित राज्य की विधान सभा के सदस्यों द्वारा निर्वाचित किए जाते हैं,

(5) 1/6 सदस्य राज्यपाल द्वारा मनोनीत किए जाते हैं। राज्यपाल केवल उन्हीं व्यक्तियों को मनोनीत कर सकता है जिन्हें विज्ञान, कला, साहित्य, सहकारिता आन्दोलन तथा समाज-सेवा के क्षेत्र में विशेष ज्ञान अथवा ख्याति प्राप्त हो।

विभिन्न राज्यों की विधान परिषद् (Legislative Council) के सदस्यों की संख्या इस प्रकार है

क्रम संख्या राज्य का नाम कुल सदस्य संख्या
1. बिहार 96
2. कर्नाटक Har 75
3. उत्तर प्रदेश 100
4. महाराष्ट्र 78
5. आन्ध्रप्रदेश 50
6. तेलंगाना 40

योग्यताएँ (Qualifications):
संविधान ने विधान परिषद् के सदस्यों की योग्यताएँ इस प्रकार हैं-

  • वह भारत का नागरिक हो,
  • उसकी आयु 30 वर्ष से कम न हो,
  • उसके पास वे सब योग्यताएँ हों जो संसद कानून द्वारा समय-समय पर निर्धारित करे,
  • वह संघीय सरकार या राज्य सरकार के अधीन किसी लाभ के पद पर न हो,
  • वह पागल, दिवालिया या अन्य कारणवश अयोग्य घोषित न किया गया हो।

कार्यकाल (Term of Office):
विधान परिषद् एक स्थायी सदन है जो कभी भंग नहीं होता। प्रत्येक 2 वर्ष के बाद इसके एक-तिहाई सदस्य सेवानिवृत्त होते हैं। इस प्रकार एक व्यक्ति 6 वर्ष तक विधान परिषद् का सदस्य रह सकता है।

अध्यक्ष (Chairman):
विधान-परिषद् के सदस्य अपने में से एक अध्यक्ष तथा एक उपाध्यक्ष चुनते हैं। विधान परिषद् बहुमत से प्रस्ताव पारित कर इन पदाधिकारियों को इनके पदों से हटा सकती है, परन्तु ऐसा करने से पहले उन्हें इसकी 14 दिन पहले सूचना देनी पड़ती है। इस मामले में अध्यक्ष को वोट बराबर होने की स्थिति में निर्णायक मत देने का अधिकार है। अधिवेशन (Sessions)-विधान-परिषद् की बैठक राज्यपाल बुलाता है। एक वर्ष में इसकी कम-से-कम दो बैठकें अवश्य होनी चाहिएं और पहली तथा दूसरी बैठक के बीच 6 महीने से अधिक अन्तर नहीं होना चाहिए।

गणपूर्ति (Quorum):
विधान परिषद् की कुल गणपूर्ति इसके कुल सदस्यों का दसवाँ भाग या कम-से-कम दस सदस्य निश्चित की गई है। विधान परिषद् की बैठकें तभी जारी रह सकती हैं यदि सदस्यों की गणपूर्ति पूरी हो।

वेतन तथा भत्ते (Salary and Allowances)-राज्य विधान परिषद् के सदस्यों को विधान सभा के सदस्यों के समान वेतन व भत्ते मिलते हैं जो समय-समय पर दोनों सदनों के द्वारा निश्चित किए जाते हैं। यह वेतन तथा भत्ते सभी राज्यों में एक जैसे नहीं होते। विधान परिषद् के सदस्यों को मासिक वेतन के अतिरिक्त उन्हें अन्य प्रकार के भत्ते भी दिए जाते हैं। सदस्यों के विशेषाधिकार (Privileges of Members)

(1) संसद के सदस्यों की भाँति राज्य विधानमण्डल के सदस्यों को भी सदन में भाषण देने की स्वतन्त्रता दी गई है। उनके द्वारा सदन में कहे गए शब्दों के लिए उनके विरुद्ध कोई कार्रवाई नहीं की जा सकती।

(2) किसी भी सदस्य को अधिवेशन के आरम्भ होने से 40 दिन पहले तथा समाप्त होने के 40 दिन बाद तक किसी दीवानी मामले में गिरफ्तार नहीं किया जा सकता। यदि किसी अन्य मामले में उसे गिरफ्तार किया जाए, तो उसकी सूचना सदन के अध्यक्ष .. को भेजनी पड़ती है।

विधान परिषद् की स्थापना (Creation of the Legislative Council):
राज्य विधान सभा अपने कुल सदस्यों के बहुमत से तथा उपस्थित एवं मतदान करने वाले सदस्यों के दो-तिहाई (2/3) बहुमत से प्रस्ताव पारित करके विधान परिषद् की स्थापना अथवा समाप्ति के लिए केन्द्रीय संसद से प्रार्थना कर सकता है। केन्द्रीय संसद राज्य विधान सभा की ऐसी प्रार्थना के अनुसार कानून का निर्माण कर देती है।

विधान परिषद की स्थिति (Position of the Legislative Council):
विधान-परिषद राज्य विधानमण्डल का ऊपरी सदन है, परन्त वास्तव में इस सदन के पास कोई वास्तविक शक्ति या अधिकार नहीं है। यह न तो विधान सभा द्वारा बनाए गए कानून निरस्त कर सकती है और न ही विधान सभा की स्वीकृति के बिना कोई कानून बना सकती है। साधारण विधेयक को अधिक-से-अधिक 4 महीने तक रोक सकती है। वित्तीय मामलों में तो इसकी शक्ति ‘नहीं’ के बराबर है। वित्तीय विधेयक विधान परिषद् में प्रस्तावित नहीं किया जा सकता।

वित्तीय विधेयक को विधान परिषद् अधिक-से-अधिक 14 दिन तक रोक सकती है। मन्त्रि-परिषद् पर इसका कोई विशेष नियन्त्रण नहीं होता, परन्तु इस ऊपरी सदन का महत्त्व यह है कि इसमें सदस्य अधिक अनुभवी होते हैं। विधान सभा में जल्दबाजी में पास हुए विधेयक पर जब विधान परिषद् विचार करती है तो वह बहुत-सी त्रुटियों को विधान सभा के सामने रख सकती है और उसमें संशोधन के सुझाव भी दे सकती है। इस प्रकार दो सदनों में वाद-विवाद के बाद बने कानून में त्रुटियाँ होने की कम सम्भावना होती है।

विधान परिषद् की शक्तियाँ तथा कार्य (Powers and Functions of the Legislative Council)-विधान परिषद् को निम्नलिखित शक्तियाँ प्राप्त हैं

1. विधायी शक्तियाँ (Legislative Powers):
विधान सभा में प्रस्तावित किया जा सकने वाला कोई भी साधारण विधेयक विधान परिषद् में पेश किया जा सकता है। विधान परिषद् में पेश हुआ विधेयक पास होकर विधान सभा में भेज दिया जाता है। वह तब तक पास नहीं समझा जाता, जब तक कि विधान सभा उसे पास न कर दे। विधान सभा द्वारा निरस्त किए गए विधेयक . को विधान परिषद् दोबारा पास करने का अधिकार नहीं रखती। विधान सभा द्वारा पास किए गए किसी भी साधारण विधेयक को विधान परिषद् अधिक-से-अधिक 4 महीने तक रोक सकती है।

2. वित्तीय शक्तियाँ (Financial Powers):
विधान परिषद् को कुछ वित्तीय शक्तियाँ भी प्रदान की गई हैं, परन्तु उनका कोई विशेष महत्त्व नहीं है। संविधान के अनुसार वित्त विधेयक विधान परिषद् में प्रस्तुत नहीं किए जा सकते। विधान सभा में पारित होने के बाद वित्त विधेयक को विधान-परिषद् के सुझावों के लिए भेजा जाता है। यदि विधान परिषद् उस पर 14 दिन तक कोई कार्रवाई न करे अथवा उसको निरस्त कर दे अथवा विधेयक के प्रति कुछ ऐसे सुझाव दे जो विधान सभा को स्वीकार न हों तो इन स्थितियों में वित्त विधेयक दोनों सदनों द्वारा पारित समझा जाता है।

3. कार्यपालिका पर नियन्त्रण (Control over the Executive):
विधान परिषद् का कार्यपालिका पर कोई विशेष नियन्त्रण नहीं होता। विधान परिषद् के सदस्य मन्त्रियों से प्रश्न पूछ सकते हैं, परन्तु अविश्वास का प्रस्ताव पास कर मन्त्रि-परिषद् को पद से नहीं हटा सकते। यह अधिकार केवल विधान सभा को ही प्राप्त है।

4. संविधान में संशोधन (Constitutional Amendments):
जिन संविधान संशोधनों को राज्य के अनुमोदन के लिए भेजा जाता है, उनका अनुमोदन विधान परिषद् से विधान सभा के समान कराना होता है।

HBSE 11th Class Political Science Important Questions Chapter 5 विधायिका

प्रश्न 11.
राज्य विधानमण्डल में साधारण कानून पारित करने की प्रक्रिया का वर्णन करें।
उत्तर:
विधानमण्डल का सर्वाधिक महत्त्वपूर्ण कार्य कानुन बनाना है। साधारण विधेयक विधानमण्डल के किसी भी सदन में प्रस्तावित किया जा सकता है। साधारण विधेयक दो प्रकार के होते हैं-सरकारी तथा गैर-सरकारी। सरकारी विधेयक वे होते हैं जिन्हें कोई मन्त्री प्रस्तावित करता है और गैर-सरकारी विधेयक वे विधेयक होते हैं जिन्हें मन्त्रियों को छोड़कर अन्य विधायक प्रस्तावित करते हैं और गैर-सरकारी विधेयक प्रस्तावित करने के लिए प्रस्तावक को एक महीने का नोटिस तथा उसके साथ विधेयक की एक प्रति सदन के अध्यक्ष को देनी होती है। यदि अध्यक्ष चाहे तो कम समय के नोटिस पर भी विधेयक को प्रस्तावित करने की अनुमति दे सकता है, लेकिन सरकारी विधेयक के लिए पूर्व नोटिस की आवश्यकता नहीं होती।

साधारण विधेयक को पारित करने की प्रक्रिया (Procedure Regarding Ordinary Bills)-साधारण विधेयकों को पारित करने की प्रक्रिया का विस्तृत वर्णन संविधान में दिया गया है जो निम्नलिखित स्तरों (Stages) से गुजरता है

1. विधेयक प्रस्तुत करना और प्रथम वाचन (Introduction and First Reading of the Bill) साधारण विधेयक किसी भी सदन में प्रस्तुत किया जाता है, परन्तु प्रायः 85 से 90 प्रतिशत विधेयक विधान सभा में पेश होते हैं। जिस सदन में भी विधेयक पेश करना हो तो उस सदन को एक महीने का नोटिस देना पड़ता है।

निश्चित तिथि को विधेयक को प्रस्तुत करने वाला सदस्य प्रस्तावक अपने स्थान पर खड़ा होकर उस विधेयक को पेश करने की आज्ञा माँगता है और विधेयक के शीर्षक को पढ़ता है और विधेयक के सिद्धान्त व उद्देश्य बताता है। इस स्तर पर विधेयक का विरोध नहीं होता। इसके पश्चात् विधेयक सरकारी गजट में छाप दिया जाता है। सरकारी विधेयक के लिए नोटिस की भी आवश्यकता नहीं होती, सरकारी गजट में छाप दिया जाना ही काफी समझा जाता है। इस स्तर पर अधिक वाद-विवाद नहीं होता। यही प्रथम वाचन कहलाता है।

2. दूसरा वाचन (Second Reading)-प्रथम वाचन के पश्चात् विधेयक पेश करने वाला दूसरे वाचन के लिए प्रार्थना करता है। उसकी प्रार्थना पर दूसरे वाचन की तिथि सदन के सभापति द्वारा निश्चित कर दी जाती है। निश्चित तिथि को प्रस्तावक दिए गए प्रस्तावों में से एक प्रस्ताव पेश करता है कि-

  • विधेयक पर सदन में तुरन्त विचार किया जाए,
  • विधेयक को किसी प्रवरस भेज दिया जाए,
  • विधेयक दोनों सदनों की संयुक्त समिति को भेजा जाए,
  • विधेयक के विषय में लोगों के विचार जानने के लिए विधेयक को प्रस्तावित किया जाए।

बहुत-से महत्त्वपूर्ण विधेयकों पर तुरन्त विचार आरम्भ कर दिया जाता है अन्यथा विधेयक को किसी-न-किसी प्रवर समिति के पास भेजा जाता है। यदि सदन दोनों सदनों की संयुक्त समिति के पास विधेयक भेजने पर सहमत हो तो विधेयक को दूसरे सदन की सहमति के लिए भेजा जाता है तथा उसकी सहमति प्राप्त हो जाने के पश्चात् विधेयक दोनों सदनों की संयुक्त प्रवर समिति के पास भेजा जाता है। उसी विधेयक को जनमत जानने के लिए उसे प्रस्तावित किया जाता है जो सामाजिक, आर्थिक या राजनीतिक क्षेत्र में उथल-पुथल लाने वाला हो, वरना विधेयक सदा प्रवर समिति को ही भेजा जाता है।

प्रवर समिति में विधेयक के पक्ष तथा विपक्ष में तर्क व सुझाव प्रस्तुत किए जाते हैं। समिति विधेयक से सम्बन्धित लोगों को बुलाकर उनसे जानकारी प्राप्त कर सकती है तथा सरकार के रिकॉर्ड तथा पत्रों का निरीक्षण कर सकती है। इस समय विधेयक की एक-एक धारा पर विचार होता है और समिति विधेयक के शीर्षक को छोड़कर सम्पूर्ण विधेयक को बदल सकती है। विधेयक पर विचार करने के बाद समिति अपनी रिपोर्ट सहित विधेयक को वापस सदन में भेज देती है।

समिति की रिपोर्ट आ जाने पर सम्बन्धित विधेयक पर विस्तार से विचार किया जाता है और एक-एक धारा पर विचार करके उसे स्वीकृत या अस्वीकृत कर दिया जाता है। अन्त में विधेयक को सदन द्वारा मन्जूर करने पर उसका तीसरा वाचन होता है।

3. तीसरा वाचन (Third Reading):
जब विधेयक की रिपोर्ट अवस्था समाप्त हो जाती है तो इसके पश्चात् इसका तीसरा वाचन शुरू होता है। यह औपचारिक अवस्था होती है। इस स्तर पर विधेयक की शब्दावली की अशुद्धियों को ही दूर किया जाता है। इसके पश्चात् सम्पूर्ण मतदान करवाया जाता है और विधेयक पास कर दिया जाता है।

4. विधेयक दूसरे सदन में (Bill in the Second House):
यदि राज्य में एक ही सदन है तो विधेयक सदन में पास होने के पश्चात् स्वीकृति के लिए राज्यपाल के पास भेज दिया जाता है। दो सदन होने की दशा में विधान सभा में विधेयक पहले सदन की तरह भिन्न-भिन्न अवस्थाओं में से गुजरता है। इसके लिए उसे तीन महीने का समय दिया जाता है।

इस अवधि के दौरान वह अपने सुझावों सहित विधेयक विधान सभा को वापस भेज देता है। यदि विधान सभा विधान परिषद् के सुझावों को स्वीकार न करे तो विधान सभा पुनः विधेयक को पास करके विधान परिषद् के पास भेजती है। अब विधान परिषद् एक मास तक विधेयक को कानून बनाने से रोक सकती है। इस अवधि के पश्चात् विधेयक दोनों सदनों द्वारा पारित माना जाता है। इस प्रकार विधान परिषद् किसी विधेयक को कानून बनने से अधिक-से-अधिक चार महीने तक रोक सकती है।

5. राज्यपाल की स्वीकृति (Assent by the Governor):
जब विधेयक दोनों सदनों द्वारा पास कर दिया जाता है तो उसे राज्यपाल की स्वीकृति के लिए भेज दिया जाता है। राज्यपाल उसे स्वीकृत कर सकता है या उसे विधानमण्डल के पास पुनर्विचार के लिए भेज सकता है। यदि विधानमण्डल उस प्रस्ताव को दोबारा साधारण बहुमत से पास कर दे तो वह विधेयक बिना राज्यपाल की स्वीकृति के पास माना जाता है। राज्यपाल किसी विधेयक को स्वीकृति के लिए भी सुरक्षित रख सकता है। ऐसे विधेयक राष्ट्रपति द्वारा स्वीकृति देने पर ही कानून बनते हैं।

प्रश्न 12.
राज्य विधानमण्डल में धन विधेयक अथवा बजट पारित करने की प्रक्रिया का वर्णन करें।
उत्तर:
राज्य विधानमण्डल में धन विधेयक पारित करने की विधि संसद द्वारा धन विधेयक पारित करने की प्रक्रिया के समान है। धन विधेयक से तात्पर्य उस विधेयक से है जिसका सम्बन्ध किसी प्रकार के कर (Tax) लगाने, कर कम करने, बढ़ाने तथा राज्य कोष से धन व्यय करने से हो। वित्तीय विधेयक के सम्बन्ध में महत्त्वपूर्ण तथ्य इस प्रकार हैं-

  • धन विधेयक राज्यपाल की पूर्व-अनुमति से ही विधानमण्डल में प्रस्तावित किया जा सकता है,
  • धन विधेयक केवल विधान सभा में ही प्रस्तावित किया जा सकता है, विधान परिषद् में नहीं,
  • धन विधेयक मन्त्री द्वारा ही प्रस्तावित किया जा सकता है,
  • विधानमण्डल धन विधेयकों की माँगों को अस्वीकार कर सकता है, कम कर सकती है, लेकिन बढ़ा नहीं सकता,
  • धन विधेयक का निर्धारण विधान सभा का अध्यक्ष करता है। उसका निर्णय अन्तिम होता है,
  • विधान परिषद् धन विधेयक को अस्वीकार नहीं कर सकती,
  • विधान परिषद् धन विधेयक को ज्यादा-से-ज्यादा 14 दिन तक रोक सकती है। धन विधेयक या बजट को पारित करने की प्रक्रिया इस प्रकार से है बजट को पारित करने की प्रक्रिया (Procedure of Passing the Budget)-बजट को पारित करने की प्रक्रिया निम्नलिखित है

1. बजट की तैयारी (Preparation of Budget):
सर्वप्रथम वित्त मन्त्री अन्य मन्त्रियों से आगामी वर्ष के लिए उनके विभागों की आवश्यकताओं व माँगों की जानकारी लेता है। उसके बाद वित्त मन्त्री बजट तैयार करता है जिस पर पूरा मन्त्रिमण्डल विचार करता है और बजट को अन्तिम रूप देता है। .

2. बजट को प्रस्तावित करना (Introduction of Budget):
राज्यपाल की पूर्व अनुमति से वित्त मन्त्री बजट को विधान सभा में प्रस्तावित करता है। बजट के दो भाग होते हैं-

  • व्यय का भाग तथा
  • आय का भाग।

व्यय का भाग पुनः दो वर्गों में बँटा होता है-

  • राज्य की संचित निधि से होने वाले व्यय तथा
  • अन्य व्यय।

संचित निधि से किए जाने वाले खर्चों पर विचार तो किया जाता है, लेकिन मतदान नहीं होता। गों पर विधान सभा में विभागीय आधार पर विचार किया जाता है। विधान सभा में खर्चों में कटौती के प्रस्ताव रखे जा सकते हैं। विधान सभा खर्चों में बढ़ोतरी का प्रस्ताव पारित नहीं कर सकती। यदि विधान सभा कटौती का प्रस्ताव पारित कर देती है अथवा बजट को अस्वीकार कर देती है तो इसे सरकार की पराजय समझा जाता है और उसे त्याग-पत्र देना पड़ता है।

3. विनियोग विधेयक (Appropriation Bill):
जब अन्य खर्चों को विधान सभा मान लेती है तो इन खर्चों व संचित निधि से होने वाले खर्चों को मिलाकर एक विनियोग विधेयक विधान सभा में प्रस्तावित किया जाता है, जिसे सदन पारित करता है। इस विधेयक के पारित हो जाने पर ही सरकार को खर्च करने का अधिकार मिलता है।

4. वित्त विधेयक (Finance Bill):
सदन द्वारा विनियोग विधेयक पारित कर देने के बाद वित्तमन्त्री बजट के दूसरे भाग-आमदनी के भाग को एक विधेयक के रूप में सदन में प्रस्तावित करता है जिसमें आय के साधनों का वर्णन होता है। इस वित्त विधेयक के पारित हो जाने पर बजट विधान सभा द्वारा पारित माना जाता है।

5. बजट दूसरे सदन में (Budget in the Second House):
जिन राज्यों में विधानमण्डल का दूसरा सदन-विधान परिषद् होती है, उन राज्यों में विधान सभा द्वारा पारित बजट विधान परिषद् में भेजा जाता है। विधान परिषद् बजट को अस्वीकार नहीं कर सकती, अधिक-से-अधिक 14 दिन तक रोक सकती है। इस अवधि के पश्चात् बजट विधान-परिषद् द्वारा पारित माना जाता है।

6. राज्यपाल की स्वीकृति (AssentofGovernor):
विधानमण्डल द्वारा बजट पारित होने के बाद स्वीकृति के लिए राज्यपाल दिया जाता है। राज्यपाल इसको अस्वीकार नहीं कर सकता। वास्तव में राज्यपाल की स्वीकृति एक औपचारिकता ही होती है, क्योंकि धन विधेयक राज्यपाल की पूर्व-अनुमति के बिना प्रस्तावित ही नहीं किया जा सकता अर्थात् राज्यपाल अपनी स्वीकृति तो वास्तव में पहले ही दे चुका होता है। राज्यपाल की स्वीकृति के बाद बजट को सरकारी गजट में छाप दिया जाता है और वह कानून बन जाता है।

प्रश्न 13.
राज्य विधान सभा के अध्यक्ष की शक्तियों व कार्यों का वर्णन करें।
अथवा
राज्य विधान सभा के अध्यक्ष के चुनाव, शक्तियों, कार्यों व स्थिति का वर्णन करो।
उत्तर:
विधान सभा में सदन का सभापति अध्यक्ष कहलाता है जो विधान सभा की बैठकों की अध्यक्षता करता है।

चुनाव (Election)-विधान सभा के अध्यक्ष का चुनाव विधान सभा के सदस्य अपने में से ही करते हैं। अध्यक्ष बहुमत प्राप्त दल अथवा दलीय गठबन्धन की इच्छानुसार ही चुना जाता है, क्योंकि यदि चुनाव होता है तो बहुमत होने के कारण सत्तारूढ़ दल अथवा दलीय गठबन्धन को अपना उम्मीदवार जिताने में कोई कठिनाई नहीं आती।

कार्यकाल (Tenure)-विधान सभा के अध्यक्ष का कार्यकाल विधान सभा के कार्यकाल से जुड़ा होता है। यदि विधान सभा समय से पूर्व भंग हो जाती है तो अध्यक्ष अपने पद पर तब तक बना रहता है जब तक चुनावों के बाद नई विधान सभा नहीं बन जाती है और अपना अध्यक्ष नहीं चुन लेती है। अध्यक्ष स्वयं भी त्याग-पत्र दे सकता है। विधान सभा में अध्यक्ष को हटाए जाने सम्बन्धी प्रस्ताव भी प्रस्तावित किया जा सकता है। ऐसे प्रस्ताव की सूचना कम-से-कम 14 दिन पूर्व देनी आवश्यक है। यदि विधान सभा साधारण बहुमत से प्रस्ताव पारित कर देती है तो अध्यक्ष को त्याग-पत्र देना पड़ता है।

वेतन तथा भत्ते (Salary and Allowances)-विधान सभा के अध्यक्ष के वेतन व भत्ते विधानमण्डल द्वारा निर्धारित किए जाते हैं। अध्यक्ष को मासिक वेतन व भत्तों के अलावा अन्य सुविधाएँ भी प्राप्त होती हैं। विधानमण्डल अध्यक्ष के कार्यकाल के तों में कटौती नहीं कर सकती है। विधान सभा के अध्यक्ष के वेतन व भत्ते प्रत्येक राज्य में भिन्न-भिन्न होते हैं। अध्यक्ष की शक्तियाँ तथा कार्य (Powers and Functions of the Speaker)-अध्यक्ष को विभिन्न शक्तियाँ प्राप्त हैं जिनके आधार पर अध्यक्ष अनेक प्रकार के कार्य करता है जो इस प्रकार से हैं-

  • अध्यक्ष सदन के नेता की सलाह से सदन की कार्रवाई का कार्यक्रम बनाता है,
  • सदन में शान्ति और व्यवस्था बनाए रखना अध्यक्ष का अधिकार व उत्तरदायित्व है,
  • अध्यक्ष सदन की कार्रवाई के नियमों की व्याख्या करता है,
  • अध्यक्ष की आज्ञा से ही सदस्य सदन में बोल सकते हैं। अध्यक्ष सदस्यों को विचार प्रकट करने की आज्ञा देता है,
  • यदि कोई सदस्य अध्यक्ष की आज्ञा का उल्लंघन करता है, उसके बार-बार मना करने पर भी सभा में गड़बड़ करता है तो अध्यक्ष मार्शल को, उस सदस्य को सदन से बाहर निकालने का आदेश दे सकता है,
  • सदन में व्यवस्था स्थापित करने में असफल रहने पर वह सदन की कार्रवाई को स्थगित (Adjourn) भी कर सकता है,
  • काम रोको प्रस्ताव को स्वीकार अथवा अस्वीकार करता है,
  • धन विधेयक का निर्धारण करता है। उसका निर्णय अन्तिम होता है,
  • सदन में गणपूर्ति है अथवा नहीं, इसका निर्धारण करता है,
  • प्रस्तावों पर मतदान कराता है और परिणाम की घोषणा करता है,
  • यदि किसी प्रस्ताव पर पक्ष तथा विपक्ष में समान वोट आते हैं तो अपना निर्णायक वोट (Casting Vote) डालता है,
  • अध्यक्ष सदन के सदस्यों की जानकारी के लिए अथवा विशेष महत्त्व के मामलों पर सदन को सम्बोधित करता है,
  • सदस्यों द्वारा प्रयुक्त असंसदीय भाषा व शब्दों को सदन की कार्रवाई से निकालने का आदेश देता है,
  • यदि कोई सदस्य बार-बार अध्यक्ष की आज्ञा का उल्लंघन करता है तो वह सदस्य को निलम्बित (Suspend) भी कर सकता है,
  • यदि सदन किसी व्यक्ति को सदन के विशेषाधिकारों का उल्लंघन करने के आरोप में दण्ड दे तो उस दण्ड को लागू करवाना अध्यक्ष का कार्य है,
  • अध्यक्ष विधान सभा द्वारा पारित विधेयक को राज्यपाल के पास स्वीकृति के लिए अपने हस्ताक्षर करके भेजता है,
  • किसी सदस्य ने दल-बदल किया है अथवा नहीं, इसका फैसला अध्यक्ष करता है,
  • सदन में किसी दल का दल-विभाजन हुआ है अथवा नहीं, का फैसला भी अध्यक्ष ही करता है।

स्थिति (Position)-विधान सभा के अध्यक्ष की स्थिति लोकसभा के अध्यक्ष के समान है। उसका पद बड़ा ही गरिमामय, सम्मानजनक समझा जाता है। उससे निष्पक्षता की आशा की जाती है। जब से अध्यक्ष को दल-बदल के सम्बन्ध में अधिकार प्राप्त हुए हैं, उसका पद राजनीतिक रूप से भी अति महत्त्वपूर्ण बन गया है।

प्रश्न 14.
विधान सभा व विधान परिषद् की शक्तियों की तुलना करें।
उत्तर:
जिन राज्यों में एक सदनीय विधानमण्डल है, वहाँ सभी शक्तियाँ विधान सभा में निहित होती हैं और जिन राज्यों में दो सदन होते हैं। उन राज्यों में शक्तियाँ दोनों के पास होती हैं। साधारणतः दूसरे सदनों को कम शक्तियाँ प्राप्त होती हैं। दोनों सदनों के सम्बन्धों का वर्णन इस प्रकार है

1. कानून के क्षेत्र में सम्बन्ध (Relation in the field of Law Making) साधारण विधेयक दोनों सदनों में से किसी भी सदन में प्रस्तुत किया जा सकता है। फिर भी विधान परिषद् वैधानिक क्षेत्र में विधान सभा की अपेक्षा एक शक्तिहीन संस्था है। क्योंकि विधान सभा को वैधानिक क्षेत्र में प्रभुत्व प्राप्त है। साधारण विधेयक को विधान परिषद् अपने पास केवल 4 मास तक रख सकती है। विधान सभा के पास विधान परिषद् को समाप्त करने की शक्ति है।

2. वित्त सम्बन्धी क्षेत्र में सम्बन्ध (Relation in the field of Finance) वित्तीय क्षेत्र में भी विधान सभा सर्वोपरि है। धन विधेयक केवल विधान सभा में ही प्रस्तुत किया जा सकता है। धन विधेयक के विधान सभा द्वारा पास होने के बाद विधान परिषद् के पास भेजा जाता है। विधान परिषद् धन विधेयक को अपने पास केवल 14 दिन के लिए रोक सकती है। 14 दिन के अन्दर-अन्दर स्वीकृति न मिलने पर विधेयक को पास माना जाता है।

3. कार्यपालिका पर नियन्त्रण के क्षेत्र में (In the field of Control over Executive) मन्त्रिपरिषद् विधान सभा के प्रति उत्तरदायी है। विधान परिषद् का तो केवल नाममात्र का नियन्त्रण है। जहाँ विधान सभा अविश्वास का प्रस्ताव (Vote of Non-confidence) पास करके मन्त्रियों को हटा सकती है, वहाँ विधान-परिषद् प्रश्न व अनुपूरक प्रश्न पूछ सकती है तथा मन्त्रियों की आलोचना कर सकती है। इससे स्पष्ट है कि कार्यपालिका पर भी विधान सभा का ही नियन्त्रण है। इस क्षेत्र में विधान सभा के अधिकार विधान परिषद् से अधिक व्यापक है।

4. चुनाव सम्बन्धी क्षेत्र में (In the field of Electoral Powers)-चुनाव के क्षेत्र में केवल विधान सभा को अधिकार प्राप्त है, विधान परिषद् को नहीं। राष्ट्रपति के चुनाव में केवल विधान सभा के सदस्य ही भाग लेते हैं। विधान परिषद् के 1/3 सदस्य भी विधान सभा द्वारा ही निर्वाचित किए जाते हैं। इस प्रकार चुनाव के क्षेत्र में विधान सभा को अधिकार प्राप्त है। विधान सभा व विधान परिषद् के अधिकार के तुलनात्मक अध्ययन से यह स्पष्ट है कि विधान सभा एक शक्तिशाली व प्रभावशाली सदन है। विधान परिषद् एक दुर्बल सदन है इसीलिए विद्वान विधान परिषद् को समाप्त करने के पक्ष में हैं।

वस्तुनिष्ठ प्रश्न

निम्नलिखित प्रश्नों का उत्तर दिए गए विकल्पों में से उचित विकल्प छाँटकर लिखें

1. लोकसभा में संघीय क्षेत्रों (Union Territories) के प्रतिनिधियों की अधिक-से-अधिक संख्या हो सकती है
(A) 15
(B) 20
(C) 25
(D) 30
उत्तर:
(B) 20

2. राष्ट्रपति राज्यसभा में कितने सदस्य मनोनीत करता है?
(A)7
(B) 11
(C) 10
(D) 12
उत्तर:
(D) 12

3. संसद के दोनों सदनों में सबसे अधिक सदस्य कौन-से प्रांत के होते हैं?
(A) केरल
(B) हरियाणा
(C) मध्य प्रदेश
(D) उत्तर प्रदेश
उत्तर:
(D) उत्तर प्रदेश

4. एक सदन द्वारा पास किए गए साधारण विधेयक पर दूसरे सदन द्वारा कार्रवाई करनी आवश्यक है
(A) तीन महीने के अंदर
(B) छह महीने के अंदर
(C) चार महीने के अंदर
(D) आठ महीने के अंदर
उत्तर:
(B) छह महीने के अंदर

5. लोकसभा में अनुसूचित जातियों के लिए कितने स्थान आरक्षित हैं?
(A) 79
(B) 84
(C) 86
(D) 90
उत्तर:
(B) 84

HBSE 11th Class Political Science Important Questions Chapter 5 विधायिका

6. राज्यसभा में निर्वाचित सदस्य अधिक-से-अधिक कितने हो सकते हैं?
(A) 250
(B) 238
(C) 235
(D) 245
उत्तर:
(B) 238

7. भारत में संघीय विधानपालिका को नाम दिया गया है
(A) संसद
(B) लोकसभा
(C) राज्यसभा
(D) विधानसभा
उत्तर:
(A) संसद

8. लोकसभा का साधारण कार्यकाल कितना निश्चित किया गया है?
(A) 4 वर्ष
(B) 5 वर्ष
(C) 6 वर्ष
(D) 3 वर्ष
उत्तर:
(B) 5 वर्ष

9. लोकसभा में प्रांतों के प्रतिनिधियों की संख्या अधिक-से-अधिक कितनी हो सकती है?
(A) 520
(B) 525
(C) 530
(D) 545
उत्तर:
(C) 530

10. लोकसभा और राज्यसभा की गणपूर्ति (Quorum) उनके अपने कुल सदस्यों का कितना भाग निश्चित किया गया है?
(A) 1/3 भाग
(B) 1/5 भाग
(C) 1/10 भाग
(D) 1/15 भाग
उत्तर:
(C) 1/10 भाग

11. राज्यसभा के कितने सदस्य प्रत्येक दो वर्ष के बाद रिटायर हो जाते हैं?
(A) 1/2 भाग
(B) 1/3 भाग
(C) 1/4 भाग
(D) 1/6 भाग
उत्तर:
(B) 1/3 भाग

12. राज्यसभा के अधिकतम सदस्य हो सकते हैं
(A) 230
(B) 240
(C) 250
(D) 260
उत्तर:
(C) 250

13. संसद की स्थिति क्या है?
(A) सर्वोच्च है
(B) शक्तियाँ सीमित हैं
(C) शक्तियाँ असीमित हैं
(D) बहुत शक्तिहीन है
उत्तर:
(B) शक्तियाँ सीमित हैं

14. लोकसभा में निर्णायक मत (Casting Vote) देने का अधिकार है
(A) प्रधानमंत्री को
(B) उप-राष्ट्रपति को
(C) स्पीकर को
(D) राष्ट्रपति को
उत्तर:
(C) स्पीकर को

15. लोकसभा के अध्यक्ष का चुनाव करते हैं
(A) संसद के सदस्य
(B) लोकसभा तथा राज्यसभा के सदस्य
(C) लोकसभा के सदस्य
(D) राष्ट्रपति तथा प्रधानमंत्री
उत्तर:
(C) लोकसभा के सदस्य

16. लोकसभा में विधि-नियमों की व्याख्या करता है
(A) लोकसभा का सैक्रेटरी जनरल
(B) भारत का सर्वोच्च न्यायालय
(C) लोकसभा का स्पीकर
(D) भारत का प्रधानमंत्री
उत्तर:
(C) लोकसभा का स्पीकर

17. विधानसभा के अधिक-से-अधिक सदस्य हो सकते हैं
(A) 400
(B) 450
(C) 500
(D) 525
उत्तर:
(C) 500

18. निम्नलिखित संसद का कार्य है
(A) कानूनों का निर्माण करना
(B) शासन करना
(C) युद्ध की घोषणा करना
(D) नियुक्तियां करना
उत्तर:
(A) कानूनों का निर्माण करना

19. लोकसभा को कौन भंग कर सकता है?
(A) राष्ट्रपति
(B) अध्यक्ष
(C) प्रधानमंत्री
(D) मंत्रिमंडल
उत्तर:
(A) राष्ट्रपति

20. संसद तथा राष्ट्रपति में कड़ी है
(A) प्रधानमंत्री
(B) स्पीकर
(C) उप-राष्ट्रपति
(D) भारत का मुख्य न्यायाधीश
उत्तर:
(B) स्पीकर

21. लोकसभा में अध्यक्ष को मासिक वेतन मिलता है
(A) 80,000 रुपए
(B) 1,25,000 रुपए
(C) 4,00,000 रुपए
(D) इनमें से कोई नहीं
उत्तर:
(C) 4,00,000 रुपए

22. विधानसभा का कार्यकाल है
(A) 4 वर्ष
(B) 5 वर्ष
(C) 7 वर्ष
(D) 8 वर्ष
उत्तर:
(B) 5 वर्ष

23. लोकसभा के स्पीकर को पदच्युत किया जा सकता है
(A) राष्ट्रपति द्वारा
(B) संसद द्वारा
(C) सर्वोच्च न्यायालय द्वारा
(D) लोकसभा के बहुमत द्वारा
उत्तर:
(D) लोकसभा के बहुमत द्वारा

24. कार्यपालिका निम्नलिखित साधनों से उत्तरदायी बनाई जा सकती है
(A) प्रश्न पूछकर
(B) बहस द्वारा
(C) स्थगन प्रस्ताव
(D) उपर्युक्त सभी
उत्तर:
(D) उपर्युक्त सभी

25. विधानसभा की बैठकों की अध्यक्षता करता है
(A) राज्यपाल
(B) मुख्यमंत्री
(C) स्पीकर
(D) उप-राष्ट्रपति
उत्तर:
(C) स्पीकर

26. विधानसभा और विधान परिषद की कम-से-कम गणपूर्ति कितनी है?
(A) 5 सदस्य
(B) 10 सदस्य
(C) 20 सदस्य
(D) 15 सदस्य
उत्तर:
(B) 10 सदस्य

27. विधानसभा द्वारा पास किए गए धन विधेयक को विधान परिषद् अधिक से अधिक कितने समय के लिए अपने पास रख सकती है?
(A) 10 दिन
(B) 14 दिन
(C) 15 दिन
(D) 20 दिन
उत्तर:
(B) 14 दिन

28. राज्य विधान-परिषद् के कुल सदस्यों का 1/6 भाग नियुक्त किया जाता है
(A) राष्ट्रपति द्वारा
(B) प्रधानमंत्री द्वारा
(C) मुख्यमंत्री द्वारा
(D) राज्यपाल द्वारा
उत्तर:
(D) राज्यपाल द्वारा

29. विधानसभा में धन विधेयक पेश करने के लिए अंतिम स्वीकृति आवश्यक है
(A) मुख्यमंत्री की
(B) प्रधानमंत्री की
(C) राष्ट्रपति की
(D) राज्यपाल की
उत्तर:
(D) राज्यपाल की

30. विधानसभा का अध्यक्ष किस प्रकार निर्वाचित किया जाता है?
(A) राज्यपाल द्वारा
(B) मुख्यमंत्री द्वारा
(C) सदन के सदस्यों द्वारा
(D) जनता के द्वारा
उत्तर:
(C) सदन के सदस्यों द्वारा

31. निम्नलिखित में से किस राज्य में विधान परिषद् नहीं है?
(A) हरियाणा
(B) बिहार
(C) उत्तर प्रदेश
(D) कर्नाटक
उत्तर:
(A) हरियाणा

32. मंत्रिमंडल को निश्चित कार्यकाल समाप्त होने से पहले भंग किया जा सकता है
(A) प्रधानमंत्री द्वारा
(B) राज्यपाल द्वारा
(C) राष्ट्रपति द्वारा
(D) संसद द्वारा
उत्तर:
(B) राज्यपाल द्वारा

33. दल-बदल विरोधी कानून पारित किया गया
(A) वर्ष 1986 में
(B) वर्ष 1985 में
(C) वर्ष 1987 में
(D) वर्ष 1984 में
उत्तर:
(B) वर्ष 1985 में

34. दल-बदल कानून को प्रभावी बनाने हेतु सन् 2003 में कौन-सा संवैधानिक संशोधन पास किया गया?
(A) 85वाँ
(B) 91वाँ
(C) 92वाँ
(D) 95वाँ
उत्तर:
(B) 91वाँ

निम्नलिखित प्रश्नों का उत्तर एक शब्द में दें

1. संसद में कौन-कौन शामिल हैं?
उत्तर:
संसद में लोकसभा, राज्यसभा व राष्ट्रपति शामिल हैं।

2. लोकसभा का अधिवेशन कौन बुलाता है और इसकी अध्यक्षता कौन करता है?
उत्तर:
लोकसभा का अधिवेशन राष्ट्रपति बुलाता है और लोकसभा अध्यक्ष इसकी अध्यक्षता करता है।

3. लोकसभा को कब और कौन भंग कर सकता है?
उत्तर:
लोकसभा को प्रधानमंत्री की सलाह पर राष्ट्रपति भंग कर सकता है।

4. अध्यक्ष का चुनाव कौन करता है?
उत्तर:
अध्यक्ष का चुनाव केवल लोकसभा के सदस्य करते हैं।

5. राज्यसभा का पदेन अध्यक्ष कौन होता है?
उत्तर:
उप-राष्ट्रपति राज्यसभा का पदेन अध्यक्ष होता है।

6. संसद के दोनों सदनों के संयुक्त अधिवेशन की अध्यक्षता कौन करता है ?
उत्तर:
लोकसभा का अध्यक्ष।

7. वर्तमान में राज्यसभा के सभापति कौन हैं?
उत्तर:
श्री एम० वैंकेया नायडू।

8. अब तक कितनी लोकसभा के चुनाव सम्पन्न हो चुके हैं?
उत्तर:
17 लोकसभा के।

9. 17वीं लोकसभा में राष्ट्रीय जनतांत्रिक गठबन्धन सरकार के नेतृत्व में स्पीकर की भूमिका कौन निभा रहा है?
उत्तर:
श्री ओम बिड़ला।

10. संसद के संयुक्त अधिवेशन के समय यदि लोकसभा अध्यक्ष अनुपस्थित हो तो अध्यक्षता कौन करता है?
उत्तर:
लोकसभा का उपाध्यक्ष।

11. लोकसभा प्रत्याशी को जमानत की राशि के रूप में कितने रुपए जमा करवाने होते हैं?
उत्तर:
25,000 रुपए।

12. लोकसभा चुनाव में आरक्षित जाति के उम्मीदवार को जमानत की राशि के रूप में कितने रुपए जमा करवाने होते हैं?
उत्तर:
12,500 रुपए।

13. जमानत की राशि बचाने के लिए लोकसभा चुनाव में कितने मत प्राप्त करने आवश्यक हैं?
उत्तर:
कुल पड़े मतों का 1/6 प्रतिशत मत प्राप्त करने आवश्यक हैं।

14. सांसद को स्थानीय विकास योजना के अन्तर्गत कितना खर्च प्रतिवर्ष करने का अधिकार है?
उत्तर:
5 करोड़ रुपए।

15. राज्यसभा के सदस्य का कार्यकाल कितने वर्ष का होता है?
उत्तर:
6 वर्ष का।

HBSE 11th Class Political Science Important Questions Chapter 5 विधायिका

16. राज्यसभा के सभापति का मासिक वेतन कितना है?
उत्तर:
4 लाख रुपए प्रतिमाह।

17. विधान परिषद् के कितने सदस्य विधानसभा के सदस्यों द्वारा निर्वाचित किए जाते हैं ?
उत्तर:
विधान परिषद् के 1/3 सदस्य विधानसभा के सदस्यों द्वारा निर्वाचित किए जाते हैं।

रिक्त स्थान भरें

1. भारतीय संसद की व्यवस्था संविधान के अनुच्छेद …………. में की गई है।
उत्तर:
79

2. हरियाणा विधानसभा के स्पीकर ……………. हैं।
उत्तर:
श्री ज्ञानचन्द गुप्ता

3. हरियाणा में उप-मुख्यमन्त्री पद पर ……………. हैं।
उत्तर:
दुष्यन्त चौटाला

4. लोकसभा के अधिकतम निर्वाचन सदस्य ……………. हो सकते है।
उत्तर:
550

5. लोकसभा के वर्तमान अध्यक्ष ……………. हैं।
उत्तर:
श्री ओम बिड़ला

6. वर्तमान भारत में संघीय ……………. राज्य एवं …………… केन्द्र शासित प्रदेश हैं।
उत्तर:
28 एवं 8

7. ……………. राज्यसभा का पदेन अध्यक्ष होता है।
उत्तर:
उप-राष्ट्रपति

8. वर्तमान में राज्यसभा के उपसभापति ……………. हैं।
उत्तर:
श्री हरिवंश नारायण सिंह

9. लोकसभा अध्यक्ष अपना त्यागपत्र ……………. को देता है।
उत्तर:
उपाध्यक्ष

10. राज्यसभा को संसद का ……………. सदन कहा जाता है।
उत्तर:
ऊपरी

11. राज्यसभा के सदस्य हेतु न्यूनतम आयु ……………. वर्ष होनी चाहिए।
उत्तर:
30

12. ……………. संवैधानिक संशोधन दल-बदल विरोधी कानून, 1985 में लागू हुआ।
उत्तर:
52वां

13. राज्यसभा के ……………. सदस्य प्रत्येक 2 वर्ष के बाद सेवानिवृत हो जाते हैं।
उत्तर:
1/3

HBSE 11th Class Political Science Important Questions Chapter 4 कार्यपालिका

Haryana State Board HBSE 11th Class Political Science Important Questions Chapter 4 कार्यपालिका Important Questions and Answers.

Haryana Board 11th Class Political Science Important Questions Chapter 4 कार्यपालिका

अति लघूत्तरात्मक प्रश्न

प्रश्न 1.
कार्यपालिका का क्या अर्थ है?
उत्तर:
कार्यपालिका सरकार का वह अंग है जो विधानमंडल द्वारा बनाए गए कानूनों को लागू करती है।

प्रश्न 2.
ऐसे दो राज्यों के नाम बताइए जिनमें इकहरी कार्यपालिका है।
उत्तर:

  • भारत तथा
  • संयुक्त राज्य अमेरिका में इकहरी कार्यपालिका है।

प्रश्न 3.
ऐसे दो राज्यों के नाम बताइए जिनमें बहुल कार्यपालिका है।
उत्तर:

  • स्विट्ज़रलैंड तथा
  • रूस में बहुल कार्यपालिका है।

प्रश्न 4.
कार्यपालिका के कोई दो प्रकार बताएँ।
उत्तर:

  • एकल कार्यपालिका,
  • बहुल कार्यपालिका।

प्रश्न 5.
एकल कार्यपालिका से क्या अभिप्राय है? उदाहरण दीजिए।
उत्तर:
यदि कार्यकारी शक्तियाँ किसी एक व्यक्ति को अर्पित की जाएँ, तो वह एकल कार्यपालिका होती है; जैसे भारत तथा अमेरिका में एकल कार्यपालिका है।

प्रश्न 6.
बहुल कार्यपालिका से क्या अभिप्राय है? उदाहरण दीजिए।
उत्तर:
यदि कार्यकारी शक्तियाँ कुछ व्यक्तियों के समूह अथवा समिति को प्रदान की जाएँ तो उसे बहुल कार्यपालिका कहा जाता है; जैसे स्विट्ज़रलैंड तथा रूस में बहुल कार्यपालिका है।

प्रश्न 7.
पैतृक कार्यपालिका किसे कहा जाता है? दो उदाहरण दीजिए।
उत्तर:
पैतृक कार्यपालिका में राजा का पद पैतृक अथवा वंशज होता है; जैसे इंग्लैंड तथा जापान में पैतृक कार्यपालिका है।

HBSE 11th Class Political Science Important Questions Chapter 4 कार्यपालिका

प्रश्न 8.
मनोनीत कार्यपालिका किसे कहते हैं? कोई उदाहरण दीजिए।
उत्तर:
जब राज्याध्यक्ष किसी उच्च राज्याध्यक्ष द्वारा मनोनीत किया जाए तो उसे मनोनीत कार्यपालिका कहा जाता है; जैसे भारत में राज्यों के राज्यपाल।

प्रश्न 9.
वास्तविक कार्यपालिका किसे कहते हैं? उदाहरण दीजिए।
उत्तर:
वास्तविक कार्यपालिका वह है जो वास्तविक रूप में कार्यपालिका की शक्तियों का प्रयोग करती है। भारत में प्रधानमंत्री तथा मंत्रिमंडल और अमेरिका में राष्ट्रपति वास्तविक कार्यपालिका है।

प्रश्न 10.
नाममात्र की कार्यपालिका से क्या अभिप्राय है? उदाहरण दीजिए।
उत्तर:
जब संविधान द्वारा प्राप्त शक्तियों का प्रयोग स्वयं न करके उसके नाम पर अन्य व्यक्ति अथवा संस्था शक्तियों का प्रयोग करते हैं; जैसे भारत में राष्ट्रपति, इंग्लैण्ड में राजा अथवा रानी। उसे नाममात्र की कार्यपालिका कहा जाता है।

प्रश्न 11.
राजनीतिक कार्यपालिका किसे कहा जाता है?
उत्तर:
राजनीतिक कार्यपालिका ऐसी कार्यपालिका को कहा जाता है जो कुछ समय के लिए चुनाव अथवा अन्य साधन द्वारा नियुक्त की जाती है।

प्रश्न 12.
स्थाई कार्यपालिका से क्या अभिप्राय है?
उत्तर:
स्थाई कार्यपालिका कुछ शैक्षणिक अथवा तकनीकी योग्यता के आधार पर एक लंबी अवधि के लिए नियुक्त की जाती है। इसमें नौकरशाही तथा अन्य सरकारी कर्मचारी शामिल होते हैं।

प्रश्न 13.
भारत में संसदीय शासन प्रणाली के अपनाए जाने के दो कारण लिखें।
उत्तर:

  • उत्तरदायित्व तथा स्थिरता,
  • वास्तविक लोकतंत्र की स्थापना।

प्रश्न 14.
भारत में संसदीय शासन प्रणाली की दो मुख्य विशेषताएँ लिखें।
उत्तर:

  • विधानपालिका तथा कार्यपालिका में घनिष्ठ संबंध,
  • व्यक्तिगत उत्तरदायित्व।

प्रश्न 15.
भारत के राष्ट्रपति के निर्वाचक मण्डल में कौन-से सदस्य होते हैं?
उत्तर:
राष्ट्रपति के निर्वाचक-मंडल में संसद के दोनों सदनों के निर्वाचित सदस्य तथा राज्य विधान सभाओं के निर्वाचित सदस्य होते हैं।

प्रश्न 16.
राष्ट्रपति पद की दो अयोग्यताएँ बताएँ।
उत्तर:
राष्ट्रपति पद पर ऐसा व्यक्ति आसीन नहीं हो सकता, जिसमें निम्नलिखित दो अयोग्यताएँ हों-

  • वह दिवालिया या पागल घोषित किया जा चुका हो,
  • वह सरकार के किसी लाभ के पद पर कार्यरत हो।

प्रश्न 17.
राष्ट्रपति की चुनाव प्रणाली की दो कमियाँ बताइए।
उत्तर:
राष्ट्रपति की चुनाव प्रणाली की निम्नलिखित दो कमियाँ हैं-

  • यह प्रणाली अलोकतांत्रिक है क्योंकि इसमें जनता प्रत्यक्ष रूप से भाग नहीं लेती,
  • यह प्रणाली बड़ी जटिल है।।

प्रश्न 18.
राष्ट्रपति का कार्यकाल कितना होता है? क्या उसको समय से पूर्व हटाया जा सकता है?
उत्तर:
राष्ट्रपति का कार्यकाल 5 वर्ष होता है और उसे संसद महाभियोग द्वारा समय से पूर्व हटा सकती है।

प्रश्न 19.
राष्ट्रपति को कौन और किस आधार पर हटा सकता है?
उत्तर:
राष्ट्रपति को संविधान की रक्षा न कर पाने के आरोप में संसद महाभियोग द्वारा हटा सकती है।

प्रश्न 20.
राष्ट्रपति का पारिश्रमिक तथा अवकाश प्राप्ति पर पेन्शन बताइए।
उत्तर:
राष्ट्रपति को 5 लाख रुपए मासिक धनराशि तथा सेवानिवृत्त होने पर मूल वेतन का 50% पैंशन मिलती है।

प्रश्न 21.
क्या राष्ट्रपति को दोबारा चुना जा सकता है? क्या कोई दोबारा राष्ट्रपति चुना गया है?
उत्तर:
राष्ट्रपति को दोबारा चुना जा सकता है और अब तक केवल एक बार डॉ० राजेंद्र प्रसाद को दोबारा राष्ट्रपति चुना गया था।

प्रश्न 22.
राष्ट्रपति के दो विशेषाधिकार बताइए।
उत्तर:
राष्ट्रपति के निम्नलिखित दो विशेषाधिकार हैं-

  • राष्ट्रपति अपनी शक्तियों के प्रयोग के लिए न्यायपालिका के प्रति उत्तरदायी नहीं है,
  • राष्ट्रपति पर कार्यकाल के दौरान दीवानी मुकद्दमा नहीं चलाया जा सकता है।

प्रश्न 23.
राष्ट्रपति की दो कार्यपालिका शक्तियाँ बताएँ।
उत्तर:
राष्ट्रपति की दो कार्यपालिका शक्तियाँ हैं-

  • राष्ट्रपति प्रधानमंत्री व अन्य मंत्रियों की नियुक्ति करता है,
  • राष्ट्रपति तीनों सेनाओं का सर्वोच्च सेनापति है।

प्रश्न 24.
राष्ट्रपति की दो न्यायिक शक्तियाँ बताइए।
उत्तर:
राष्ट्रपति की दो न्यायिक शक्तियाँ हैं-

  • सर्वोच्च न्यायालय के मुख्य न्यायाधीश व अन्य न्यायाधीशों की नियुक्ति करना,
  • संवैधानिक मामलों पर सर्वोच्च न्यायालय से सलाह लेना।

प्रश्न 25.
राष्ट्रपति की दो विधायी शक्तियाँ बताइए।
उत्तर:
राष्ट्रपति के दो वैधानिक अधिकार या विधायी शक्तियाँ निम्नलिखित हैं-

  • संसद का अधिवेशन बुलाना.व सत्रावसान करना,
  • विधेयकों को स्वीकृति देना।

प्रश्न 26.
राष्ट्रपति को कितने प्रकार की संकटकालीन शक्तियाँ प्राप्त हैं? उनमें से किसी एक शक्ति को लिखें।
उत्तर:
राष्ट्रपति को तीन प्रकार की संकटकालीन शक्तियाँ प्राप्त हैं। जब देश में वित्तीय संकट उत्पन्न हो जाए तो राष्ट्रपति वित्तीय संकट की उद्घोषणा कर सकता है।

प्रश्न 27.
राष्ट्रपति राष्ट्रीय आपास्थिति कब लागू कर सकता है? अब तक कितनी बार ऐसी आपात स्थिति लागू की जा चुकी है?
उत्तर:
जब देश की सुरक्षा को युद्ध, बाह्य आक्रमण अथवा सशस्त्र विद्रोह से संकट उत्पन्न हो जाए तो राष्ट्रपति अनुच्छेद 352 के अधीन, राष्ट्रीय आपात स्थिति की उद्घोषणा कर सकता है। ऐसी आपास्थिति अब तक तीन बार 1962, 1971 तथा 1975 में लागू की जा चुकी है।

प्रश्न 28.
राष्ट्रपति अध्यादेश कब जारी कर सकता है? अध्यादेश कितनी देर प्रभावी रह सकता है?
उत्तर:
जब संसद का अधिवेशन न चल रहा हो और देश में संकटकालीन परिस्थिति उत्पन्न हो जाए, तो राष्ट्रपति अध्यादेश जारी कर सकता है। अध्यादेश अधिक-से-अधिक 6 महीने तक प्रभावी रह सकता है।

प्रश्न 29.
उप-राष्ट्रपति को हटाने संबंधी प्रस्ताव किस सदन में प्रस्तावित किया जा सकता है और उसे कैसे हटाया जा सकता है?
उत्तर:
उप-राष्ट्रपति को हटाने संबंधी प्रस्ताव केवल राज्य सभा में ही प्रस्तावित किया जा सकता है। जब संसद के दोनों सदन अलग-अलग सदन की कुल संख्या के बहुमत तथा उपस्थित व मतदान में भाग लेने वाले सदस्यों के 2/3 बहुमत से प्रस्ताव पारित कर दें तो उप-राष्ट्रपति को उसी समय पद छोड़ना पड़ता है।

प्रश्न 30.
राष्ट्रपति राज्य सभा के कितने सदस्य मनोनीत कर सकता है ?
उत्तर:
राष्ट्रपति राज्य सभा में बारह (12) सदस्य मनोनीत कर सकता है।

प्रश्न 31.
उप-राष्ट्रपति के पद की दो योग्यताएँ बताइए।
उत्तर:
उप-राष्ट्रपति के पद की दो आवश्यक योग्यताएँ हैं

  • वह भारत का नागरिक हो।
  • वह कम-से-कम 35 वर्ष की आयु पूरी कर चुका हो।

प्रश्न 32.
राष्ट्रपति पद के लिए कितने सदस्य नाम प्रस्तावित करते हैं और कितने अनुमोदित करते हैं?
उत्तर:
राष्ट्रपति पद की उम्मीदवारी के लिए निर्वाचक-मंडल के कम-से-कम पचास (50) सदस्य नाम प्रस्तावित करते हैं और पचास (50) सदस्य ही अनुमोदित करते हैं।

प्रश्न 33.
उप-राष्ट्रपति को कितने प्रकार के कार्य करने पड़ते हैं?
उत्तर:
उपराष्ट्रपति को दो प्रकार के कार्य करने पड़ते हैं

  • उप-राष्ट्रपति के रूप में,
  • राज्य सभा के सभापति के रूप में।

प्रश्न 34.
उप-राष्ट्रपति के राज्य सभा अध्यक्ष के रूप में दो कार्य बताइए।
उत्तर:

  • वह राज्य सभा के अधिवेशनों की अध्यक्षता करता है,
  • वह राज्य सभा की कार्रवाई का संचालन करता है।

प्रश्न 35.
राष्ट्रपति राज्य में आपातस्थिति कब और संविधान के किस अनुच्छेद के अधीन लगा सकता है? ।
उत्तर:
राष्ट्रपति संविधान के अनुच्छेद 356 के अधीन राज्य में आपास्थिति लागू कर सकता है, जब राज्यपाल की रिपोर्ट पर अथवा अन्य किसी सूत्र प्राप्त सूचना के आधार पर राष्ट्रपति को विश्वास हो जाए कि राज्य में संवैधानिक तन्त्र असफल हो गया है।

प्रश्न 36.
मंत्रिमंडल के दो कार्य लिखें।
उत्तर:

  • मंत्रिमंडल देश का प्रशासन चलाता है,
  • मंत्रिमंडल देश की आर्थिक, गृह व विदेश-नीति इत्यादि तय करता है।

HBSE 11th Class Political Science Important Questions Chapter 4 कार्यपालिका

प्रश्न 37.
भारतीय मंत्रिमंडलीय प्रणाली की दो विशेषताएँ बताइए।
उत्तर:

  • राष्ट्रपति नाममात्र का संवैधानिक अध्यक्ष है। वास्तविक शक्तियाँ प्रधानमंत्री के हाथों में हैं,
  • मंत्रिमंडल संसद के प्रति उत्तरदायी है।

प्रश्न 38.
प्रधानमन्त्री के मंत्रिमंडल के नेता के रूप में दो कार्य लिखें।
उत्तर:

  • राष्ट्रपति प्रधानमंत्री की सलाह से अन्य मंत्रियों की नियुक्ति करता है,
  • प्रधानमंत्री मंत्रियों के बीच विभागों का बँटवारा करता है।

प्रश्न 39.
राष्ट्रपति की दो वित्तीय शक्तियाँ लिखें।।
उत्तर:

  • राष्ट्रपति की पूर्व अनुमति के बिना संसद में कोई धन-विधेयक पेश नहीं किया जा सकता,
  • राष्ट्रपति वित्त आयोग की नियुक्ति करता है।

प्रश्न 40.
राष्ट्रपति की संकटकालीन शक्तियों के दो दोष बताइए।
उत्तर:

  • नागरिकों के मौलिक अधिकार स्थगित किए जा सकते हैं,
  • संघात्मक ढाँचा एकात्मक में बदल जाता है।

प्रश्न 41.
प्रधानमन्त्री के कार्यकाल की अवधि बताइए।
उत्तर:
प्रधानमंत्री के कार्यकाल की अवधि निश्चित नहीं होती है। प्रधानमंत्री तब तक अपने पद पर बना रहता है, जब तक उसे लोक सभा में बहुमत का समर्थन प्राप्त होता है।

प्रश्न 42.
भारत में वित्तीय आपात् स्थिति संविधान के किस अनुच्छेद के अंतर्गत लागू की जाती है? उसे अब तक कितनी बार लागू किया जा चुका है?
उत्तर:
संविधान के अनुच्छेद 360 के अन्तर्गत वित्तीय आपात स्थिति लागू की जाती है। भारत में अभी तक वित्तीय आपात स्थिति लागू नहीं की गई है।

प्रश्न 43.
उप-राष्ट्रपति को कितना वेतन मिलता है?
उत्तर:
उप-राष्ट्रपति को अपने उप-राष्ट्रपति पद के रूप में कोई वेतन नहीं मिलता। उन्हें केवल उप-सभापति के रूप में 4 लाख रुपए मासिक वेतन मिलता है।

प्रश्न 44.
राष्ट्रपति पद के प्रत्याशी बनने के लिए उसके नाम का प्रस्ताव कितने सदस्यों द्वारा प्रस्तावित, अनुमोदित होना आवश्यक है?
उत्तर:
राष्ट्रपति पद के लिए प्रस्तावित प्रत्याशी बनने के लिए उनके नाम का प्रस्ताव 50 सदस्यों द्वारा प्रस्तावित एवं 50 सदस्यों के द्वारा अनुमोदित होना चाहिए।

प्रश्न 45.
उप-राष्ट्रपति पद के प्रत्याशी बनने के लिए उसके नाम का प्रस्ताव कितने सदस्यों द्वारा प्रस्तावित एवं अनुमोदित होना चाहिए?
उत्तर:
उप-राष्ट्रपति पद का प्रत्याशी बनने के लिए उसके नाम का प्रस्ताव 20 सदस्यों द्वारा प्रस्तावित एवं 20 के द्वारा ही अनुमोदित होना चाहिए।

प्रश्न 46.
भारत में मन्त्रिमण्डल के दो लक्षण बताएँ।
उत्तर:

  • राष्ट्रपति मंत्रिमंडल का सदस्य नहीं होता। वह उसकी बैठकों में भाग नहीं लेता,
  • मंत्रिमंडल संसद के प्रति सामूहिक रूप से उत्तरदायी है।

प्रश्न 47.
मन्त्रियों की नियुक्ति कौन करता है?
उत्तर:
मंत्रियों की नियुक्ति राष्ट्रपति द्वारा प्रधानमंत्री के परामर्श के अनुसार की जाती है।

प्रश्न 48.
प्रधानमंत्री की नियुक्ति कैसे की जाती है?
उत्तर:
प्रधानमंत्री की नियुक्ति राष्ट्रपति द्वारा की जाती है। राष्ट्रपति द्वारा प्रायः उसी व्यक्ति को प्रधानमंत्री नियुक्त किया जाता है जिसे लोक सभा में बहुमत का समर्थन प्राप्त होता है।

प्रश्न 49.
प्रधानमन्त्री के कोई दो कार्य बताएँ।
उत्तर:

  • वह मंत्रिमंडल की बैठकें बुलाता है तथा उनकी अध्यक्षता करता है,
  • वह राष्ट्रपति का मुख्य सलाहकार होता है।

प्रश्न 50.
राज्यपाल बनने के लिए आवश्यक दो योग्यताएँ बताइए।
उत्तर:

  • वह भारत का नागरिक हो,
  • उसकी आयु 35 वर्ष से कम न हो।

प्रश्न 51.
सरकारिया आयोग द्वारा राज्यपाल की नियुक्ति के सम्बन्ध में दी गई दो सिफारिशें लिखें।
उत्तर:

  • राज्यपाल की नियुक्ति संबंधित राज्य सरकार के साथ सलाह-मशविरा करने के बाद ही की जाए,
  • निकट भूतकाल में सक्रिय दलीय राजनीति से जुड़े व्यक्ति को राज्यपाल न बनाया जाए।

प्रश्न 52.
राज्यपाल की दो कार्यपालिका शक्तियाँ लिखें।
उत्तर:

  • मुख्यमंत्री व मंत्रियों की नियुक्ति करना,
  • मंत्रि-परिषद् की सलाह से राज्य में उच्च अधिकारियों की नियुक्ति करना।

प्रश्न 53.
राज्यपाल की दो विधायी शक्तियाँ बताइए।
उत्तर:

  • राज्यपाल विधानमंडल का अधिवेशन बुला सकता है और स्थगित भी कर सकता है,
  • राज्यपाल को अध्यादेश जारी करने का अधिकार है।

प्रश्न 54.
राज्यपाल की दो न्यायिक शक्तियाँ बताइए।
उत्तर:

  • उच्च न्यायालय के मुख्य न्यायाधीश व अन्य न्यायाधीशों की नियुक्ति के बारे में सलाह देना,
  • जिला न्यायाधीशों की नियुक्ति करना।

प्रश्न 55.
राज्यपाल की दो न्यायिक उन्मक्तियाँ बताइए।
उत्तर:

  • राज्यपाल पर उसके कार्यकाल के दौरान फौजदारी मुकद्दमा नहीं चलाया जा सकता,
  • राज्यपाल के कार्यकाल के दौरान किसी भी न्यायालय द्वारा उसे नज़रबंद करने की आज्ञा नहीं दी जा सकती।

प्रश्न 56.
राज्यपाल की दो स्व-विवेकी शक्तियाँ बताइए।
उत्तर:

  • राज्यपाल अनुच्छेद 356 के अधीन राज्य में आपात स्थिति की सिफारिश कर सकता है,
  • वह किसी भी राज्य विधेयक को राष्ट्रपति के पास स्वीकृति के लिए भेज सकता है।

प्रश्न 57.
राज्यपाल किन दो परिस्थितियों में विधान सभा को समय से पहले भंग कर सकता है?
उत्तर:

  • जब ऐसा करने की सलाह मुख्यमंत्री दे,
  • जब विधान सभा में किसी भी दल को स्पष्ट बहुमत प्राप्त न हो और कोई भी दल सरकार बनाने की स्थिति में न हो।

प्रश्न 58.
राज्य कार्यपालिका का नाममात्र व वास्तविक अध्यक्ष कौन है?
उत्तर:
राज्य कार्यपालिका का नाममात्र का अध्यक्ष राज्यपाल होता है और वास्तविक अध्यक्ष मुख्यमंत्री होता है।

प्रश्न 59.
राज्यपाल का कार्यकाल कितना निश्चित किया गया है? क्या उसे पहले भी पद से हटाया जा सकता है?
उत्तर:
राज्यपाल का कार्यकाल पाँच वर्ष निश्चित किया गया है। राष्ट्रपति द्वारा राज्यपाल का कार्यकाल समाप्त होने से पहले भी उसे पद से हटाया जा सकता है।

प्रश्न 60.
राज्यपाल की नियुक्ति कौन करता है?
उत्तर:
राज्यपाल की नियुक्ति राष्ट्रपति द्वारा मुख्यमंत्री के परामर्श के अनुसार की जाती है।

प्रश्न 61.
मुख्यमंत्री की नियुक्ति किसके द्वारा की जाती है?
उत्तर:
मुख्यमंत्री की नियुक्ति राज्यपाल द्वारा की जाती है। राज्यपाल द्वारा उसी व्यक्ति को मुख्यमंत्री के पद पर नियुक्त किया जाता है जो राज्य विधान सभा में बहुमत प्राप्त दल का नेता होता है।

प्रश्न 62.
मुख्यमन्त्री का कार्यकाल कितना होता है?
उत्तर:
मुख्यमंत्री का कार्यकाल निश्चित नहीं होता। वह उतने समय तक अपने पद पर बना रहता है, जब तक उसे विधान सभा में बहुमत का समर्थन प्राप्त रहता है।

प्रश्न 63.
राजनीतिक कार्यपालिका तथा स्थाई कार्यपालिका में दो अंतर स्पष्ट करें।
उत्तर:

  • राजनीतिक कार्यपालिका की नियुक्ति राजनीतिक आधार पर तथा स्थाई कार्यपालिका की नियुक्ति कर्मचारियों की योग्यता के आधार पर की जाती है,
  • राजनीतिक कार्यपालिका नीतियाँ बनाती है, स्थाई कार्यपालिका उन्हें लागू करती है।

प्रश्न 64.
अच्छी प्रशासनिक सेवा के दो गुण बताएँ।
उत्तर:

  • लोक-सेवक कर्त्तव्यनिष्ठ होने चाहिएँ,
  • उन्हें जनता के हितों का ध्यान रखना चाहिएँ।

प्रश्न 65.
नौकरशाही से क्या अभिप्राय है?
उत्तर:
तकनीकी दृष्टि से कुशल कर्मचारियों का संगठन जो निष्पक्ष होकर राज्य का कार्य करता है, उसे नौकरशाही कहा जाता है।

प्रश्न 66.
नौकरशाही की दो विशेषताओं का वर्णन करें।
उत्तर:

  • नौकरशाही राजनीतिक रूप से तटस्थ होती है,
  • वह नियमानुसार कार्य करती है।

HBSE 11th Class Political Science Important Questions Chapter 4 कार्यपालिका

प्रश्न 67.
नौकरशाही के कोई दो कार्य बताएँ।
उत्तर:

  • मंत्रियों को परामर्श देना,
  • सरकार की नीतियों को लागू करना।

प्रश्न 68.
वचनबद्ध नौकरशाही का क्या अर्थ है?
उत्तर:
वचनबद्ध नौकरशाही का अर्थ है कि नौकरशाही किसी विशेष राजनीतिक दल के सिद्धान्तों एवं नीतियों से बँधी रहती है और उस दल के निर्देशों के अनुसार ही कार्य करती है।

प्रश्न 69.
नौकरशाही के दो दोष बताइए।
उत्तर:

  • लाल फीताशाही,
  • जन-साधारण की माँगों की उपेक्षा।

प्रश्न 70.
वचनबद्ध नौकरशाही के पक्ष में कोई तर्क दीजिए।
उत्तर:
वचनबद्ध नौकरशाही में सरकार की नीतियों तथा कार्यक्रमों को दृढ़ता से लागू किया जा सकता है।

प्रश्न 71.
वचनबद्ध नौकरशाही के विपक्ष में (विरुद्ध) कोई तर्क दीजिए।
उत्तर:
वचनबद्ध नौकरशाही अपना कार्य निष्पक्ष रूप से नहीं कर सकती।

लघूत्तरात्मक प्रश्न

प्रश्न 1.
कार्यपालिका के कोई पाँच कार्य बताएँ।
उत्तर:
कार्यपालिका के मुख्य पाँच कार्य इस प्रकार हैं-

  • कार्यपालिका का मुख्य कार्य कानूनों को लागू करना तथा देश में शान्ति और व्यवस्था बनाए रखना है,
  • कार्यपालिका अनेक उच्च अधिकारियों की नियुक्ति करती है,
  • कार्यपालिका देश की आन्तरिक तथा विदेश नीति का निर्माण करती है,
  • कार्यपालिका सरकार का वार्षिक बजट तैयार करती है और उसे विधानमण्डल से पास करवाती है,
  • कार्यपालिका अन्य देशों के साथ सम्बन्ध स्थापित करती है, वह अन्य देशों में जाने वाले राजदूतों की नियुक्ति करती है तथा अन्य देशों से आने वाले राजदूतों का स्वागत करती है। कार्यपालिका ही अन्य देशों के साथ सन्धियाँ एवं समझौते करती है।

प्रश्न 2.
एकल तथा बहुकार्यपालिका में भेद स्पष्ट कीजिए।
उत्तर:
यदि संविधान द्वारा समस्त शक्तियाँ एक ही व्यक्ति (अधिकारी) में निहित होती हैं, तो उसे एकल कार्यपालिका कहते हैं। भारत तथा अमेरिका के राष्ट्रपति एकल कार्यपालिका के उदाहरण हैं। इन दोनों देशों में कार्यपालिका की सारी शक्तियाँ राष्ट्रपति के हाथों में दी गई हैं। इसके दूसरी ओर जब कार्यपालिका शक्तियाँ एक व्यक्ति में निहित न होकर, कुछ व्यक्तियों अथवा किसी समिति में निहित होती हैं, तो उसे बहु-कार्यपालिका कहा जाता है। स्विट्ज़रलैण्ड की संघीय परिषद् (Federal Council), जिसमें 7 सदस्य हैं, इस प्रकार की कार्यपालिका का उदाहरण है।

प्रश्न 3.
कार्यपालिका के अध्यक्ष के प्रत्यक्ष निर्वाचन के गुण तथा अवगुण बताइए।
उत्तर:
कार्यपालिका का अध्यक्ष प्रत्यक्ष निर्वाचन द्वारा अर्थात् सीधा जनता द्वारा चुना जा सकता है। इसके गुण तथा अवगुण निम्नलिखित हैं
गुण-

  • यह विधि अधिक लोकतान्त्रिक है,
  • इससे जनता की सार्वजनिक मामलों में रुचि बढ़ती है,
  • प्रत्यक्ष चुनाव में जनता उसी व्यक्ति को चुनती है जिसकी योग्यता तथा ईमानदारी पर उसे पूरा विश्वास होता है।

अवगुण-

  • चुनाव क्षेत्र बहुत अधिक विस्तृत होने से जनता को उम्मीदवार के बारे में पूरी जानकारी नहीं होती,
  • इससे भ्रष्टाचार को बढ़ावा मिलता है।

प्रश्न 4.
संसदीय कार्यपालिका की पाँच विशेषताएँ बताएँ।
उत्तर:
संसदीय कार्यपालिका की पाँच विशेषताएँ इस प्रकार हैं-
(1) संसदीय सरकार में दो तरह की कार्यपालिका होती है। एक वास्तविक कार्यपालिका तथा दूसरी नाममात्र की कार्यपालिका। एक राज्याध्यक्ष तथा दूसरा सरकार का अध्यक्ष,

(2) संसदीय सरकार में मन्त्रिमण्डल विधानपालिका के प्रति उत्तरदायी होता है। मन्त्रिमण्डल का व्यक्तिगत तथा सामूहिक दोहरा उत्तरदायित्व होता है,

(3) संसदीय कार्यपालिका में विधानपालिका तथा कार्यपालिका में गहरा सम्बन्ध होता है। दोनों एक-दूसरे पर नियन्त्रण रखते हैं,

(4) संसदीय कार्यपालिका में मन्त्रिमण्डल के सदस्यों में राजनीतिक विचारों की एकरूपता तथा समानता होती है,

(5) संसदीय कार्यपालिका का कार्यकाल निश्चित नहीं होता। कार्यपालिका के सदस्य उतने समय तक अपने पद पर बने रहते हैं, जब तक उन्हें विधानमण्डल में बहमत का समर्थन प्राप्त रहता है। जब वे यह समर्थन खो बैठते हैं, तो उन्हें अपना त्यागपत्र देना पड़ता है।

HBSE 11th Class Political Science Important Questions Chapter 4 कार्यपालिका

प्रश्न 5.
कार्यपालिका के विधायी (कानून सम्बन्धी) कार्यों का वर्णन कीजिए।
उत्तर:
यद्यपि कानून बनाना विधानमण्डल का कार्य है, परन्तु कार्यपालिका को भी कुछ कानून-सम्बन्धी अधिकार प्राप्त होते हैं। ये इस प्रकार हैं-
(1) जिन देशों में संसदीय सरकार की स्थापना की गई है, उन देशों में कार्यपालिका का अध्यक्ष ही संसद का अधिवेशन बुलाता है तथा उसे स्थगित करता है,

(2) कार्यपालिका के अध्यक्ष को संसद के निचले सदन को उसका निश्चित कार्यकाल समाप्त होने से पहले भी भंग करने का अधिकार होता है,

(3) कार्यपालिका (मन्त्रिमण्डल) के सभी सदस्य विधानमण्डल के सदस्य होते हैं। वे विधानमण्डल की बैठकों में भाग लेते हैं, विधेयकों को पेश करते हैं तथा उन्हें पास करवाते हैं,

(4) जिस समय विधानमण्डल का अधिवेशन न चल रहा हो, कार्यपालिका के अध्यक्ष को अध्यादेश (Ordinance) जारी करने का अधिकार होता है,

(5) कई देशों में कार्यपालिका के अध्यक्ष को विधानमण्डल में कुछ सदस्य मनोनीत करने का अधिकार प्राप्त होता है। भारत का राष्ट्रपति राज्य सभा में 12 सदस्य मनोनीत करता है।

प्रश्न 6.
कार्यपालिका की शक्ति के विस्तार के कोई पाँच कारण बताएँ।
उत्तर:
कार्यपालिका की शक्तियों में वृद्धि के लिए मुख्य रूप से निम्नलिखित कारण उत्तरदायी हैं

1. दलीय पद्धति-आधुनिक लोकतन्त्रीय राज्यों में सरकार दलीय-पद्धति के आधार पर चलाई जाती है, जिस राजनीतिक दल को विधानमण्डल में बहुमत प्राप्त हो जाता है, वह सरकार का गठन करता है। दलीय अनुशासन के कारण कार्यपालिका को विधानमण्डल में बहुमत का समर्थन प्राप्त होने के कारण संसद में कुछ भी पास करवाना आसान होता है।

2. निम्न सदन को भंग करने का अधिकार-संसदीय शासन-प्रणाली में कार्यपालिका के अध्यक्ष को संसद के निम्न सदन को भंग करने का अधिकार होता है। विधानमण्डल इस भय के कारण कार्यपालिका की नीतियों एवं कानूनों को स्वीकृति दे देती है।

3. जन-कल्याणकारी राज्य की धारणा-वर्तमान युग में राज्य को जन-कल्याणकारी संस्था समझा जाता है। इसका अर्थ यह है कि राज्य जनता की भलाई के लिए अनेक कार्य करता है, जिससे कार्यपालिका के कार्यक्षेत्र में बहुत वृद्धि हो गई है।

4. प्रदत्त व्यवस्थापन-काम की अधिकता, विशेष ज्ञान की कमी व समय की कमी के कारण विधानमण्डल के लिए प्रत्येक कानून को पूरे विस्तृत रूप में पास करना सम्भव नहीं होता। इसके परिणामस्वरूप, विधानमण्डल कानून की रूपरेखा का निर्माण करके विस्तृत नियम तथा अधिनियम बनाने का कार्य कार्यपालिका पर छोड़ देती है।

इस सम्बन्ध में कार्यपालिका द्वारा बनाए गए नियम तथा अधिनियम भी उसी प्रकार से लागू होते हैं; जैसे संसद द्वारा पास किए गए कानून । इससे भी कार्यपालिका की शक्ति में वृद्धि हुई है। नियोजन आज का युग नियोजन का युग है। सभी देश अपने विकास के लिए योजनाएँ बनाते हैं। ये योजनाएँ तैयार करना, इन्हें लागू करना तथा इनका मूल्यांकन करना कार्यपालिका द्वारा ही सम्पन्न होता है। इससे कार्यपालिका की शक्ति का विस्तार हुआ है।

प्रश्न 7.
भारत के राष्ट्रपति का चुनाव कैसे किया जाता है?
उत्तर:
भारत के राष्ट्रपति का चुनाव एक निर्वाचक मण्डल द्वारा किया जाता है जिसमें संसद के दोनों सदनों के निर्वाचित सदस्य तथा राज्यों की विधानसभाओं के निर्वाचित सदस्य भाग लेते हैं। यदि निर्वाचित मण्डल में कुछ स्थान रिक्त भी हों, तो भी राष्ट्रपति का चुनाव निश्चित तिथि पर होता है। संसद के प्रत्येक निर्वाचित सदस्य तथा प्रत्येक राज्य के विधान सभा के सदस्य के मत का मूल्य अलग-अलग होता है। यह चुनाव एकल-संक्रमणीय मत-पद्धति के आधार पर आनुपातिक प्रतिनिधित्व प्रणाली द्वारा कराया जाता है। चुनाव जीतने के लिए एक उम्मीदवार को मतों की एक निश्चित संख्या, जिसे कोटा कहते हैं, प्राप्त करना होता है।

प्रश्न 8.
राष्ट्रपति पद के लिए कौन-सी योग्यताएँ अनिवार्य हैं?
उत्तर:
राष्ट्रपति पद के लिए आवश्यक योग्यताएँ इस प्रकार हैं-

  • वह भारत का नागरिक हो,
  • वह 35 वर्ष की आयु पूरी कर चुका हो,
  • वह किसी सरकारी लाभ के पद पर न हो,
  • वह लोक सभा का सदस्य बनने की योग्यता रखता हो,
  • उसके नाम का प्रस्ताव निर्वाचक मण्डल के 50 सदस्य करें तथा अन्य 50 ही अनुमोदन करें,
  • वह 15,000 रुपए जमानत राशि के रूप में जमा कराए।

प्रश्न 9.
भारत के निर्वाचित राष्ट्रपतियों के नाम बताइए।
उत्तर:
भारत में अब तक निम्नलिखित निर्वाचित राष्ट्रपति हुए हैं

  • डॉ० राजेन्द्र प्रसाद,
  • डॉ० राधाकृष्णन,
  • डॉ० जाकिर हुसैन,
  • श्री वी०वी० गिरि,
  • श्री फखरुद्दीन अली अहमद,
  • श्री नीलम संजीवा रेड्डी,
  • ज्ञानी जैल सिंह,
  • श्री आर० वेंकटरमन,
  • डॉ० शंकर दयाल शर्मा,
  • श्री के०आर० नारायणन,
  • डॉ० ए०पी०जे० अब्दुल कलाम,
  • श्रीमती प्रतिभा देवी सिंह पाटिल,
  • श्री प्रणब मुखर्जी,
  • श्री रामनाथ कोविंद।

प्रश्न 10.
राष्ट्रपति के वेतन तथा भत्ते लिखें।
उत्तर:
भारत के राष्ट्रपति को 5 लाख रुपए मासिक वेतन मिलता है। इसके अतिरिक्त उसे रहने के लिए बिना किराए के निवास-स्थान (राष्ट्रपति भवन) तथा कई अन्य भत्ते भी मिलते हैं। सेवानिवृत्त होने पर उसे मूल वेतन का 50 प्रतिशत पेंशन भी मिलती है। उसे निजी कार्यालय के लिए भत्ता तथा निःशुल्क बिजली, पानी, कार तथा टेलीफोन आदि की सुविधाएँ भी उपलब्ध होती हैं। राष्ट्रपति के वेतन तथा भत्ते भारत की संचित निधि में से दिए जाते हैं, जिन्हें उसके कार्यकाल के दौरान घटाया नहीं जा सकता।

प्रश्न 11.
राष्ट्रपति को पद से हटाने का क्या तरीका है?
उत्तर:
राष्ट्रपति को संविधान की रक्षा न कर पाने के कारण महाभियोग के द्वारा हटाया जा सकता है। महाभियोग का प्रस्ताव संसद के किसी भी सदन में प्रस्तावित किया जा सकता है। इसके लिए सदन के 1/4 सदस्य हस्ताक्षर सहित नोटिस दें। नोटिस कम-से-कम 14 दिन पहले दिया जाना चाहिए। इसके बाद सदन महाभियोग प्रस्ताव पर विचार करेगा।

यदि सदन, सदन की कुल संख्या के बहुमत तथा उपस्थित व मतदान में भाग लेने वाले सदस्यों के 2/3 बहुमत से प्रस्ताव पारित कर देता है तो वह प्रस्ताव सदन द्वारा पारित समझा जाता है। इसके बाद प्रस्ताव दूसरे सदन में भेज दिया जाता है। यदि दूसरा सदन भी सदन की कुल संख्या के बहुमत तथा उपस्थित व मतदान में भाग लेने वाले सदस्यों के 2/3 बहुमत से प्रस्ताव पारित कर देता है तो महाभियोग प्रस्ताव संसद द्वारा पारित माना जाएगा और राष्ट्रपति को अपने पद से हटना पड़ेगा।

प्रश्न 12.
राष्ट्रपति के निषेधाधिकार (Veto) का क्या अर्थ है?
उत्तर:
निषेधाधिकार (Veto Power) का अर्थ राष्ट्रपति के उस अधिकार से होता है, जिसके अन्तर्गत राष्ट्रपति संसद द्वारा पारित विधेयक पर हस्ताक्षर करने से इन्कार कर दे जैसा कि अमेरिकी राष्ट्रपति को अधिकार प्राप्त है। भारत के राष्ट्रपति को अमेरिकी राष्ट्रपति की भाँति निषेधाधिकार प्राप्त नहीं है। भारत का राष्ट्रपति हस्ताक्षर करने में देरी कर सकता है अथवा विधेयक को पुनर्विचार के लिए संसद में वापस भेज सकता है। यदि संसद विधेयक को उसी रूप में भी साधारण बहुमत से ही दोबारा पारित कर देती है तो राष्ट्रपति को स्वीकृति देनी ही पड़ती है।

प्रश्न 13.
राष्ट्रपति के पाँच कार्यपालिका अधिकार व कार्य बताइए।
उत्तर:
राष्ट्रपति के पाँच कार्यपालिका अधिकार व कार्य इस प्रकार हैं-

  • प्रधानमन्त्री की नियुक्ति करना,
  • प्रधानमन्त्री की सलाह से मन्त्रि-परिषद् के अन्य मन्त्रियों की नियुक्ति करना,
  • प्रधानमन्त्री की सलाह से मन्त्रियों में विभागों का बँटवारा करना,
  • प्रधानमन्त्री की सलाह से मन्त्रियों के विभागों में परिवर्तन करना, मन्त्रियों का त्यागपत्र स्वीकार करना अथवा मन्त्रियों को पद से हटाना,
  • राष्ट्रपति देश की सेनाओं (जल, थल व वायु सेना) का सर्वोच्च अधिकारी होता है। सेना में उच्च पदों पर नियुक्तियाँ राष्ट्रपति द्वारा की जाती हैं।

HBSE 11th Class Political Science Important Questions Chapter 4 कार्यपालिका

प्रश्न 14.
राष्ट्रपति की कोई पाँच विधायी शक्तियाँ लिखें।
उत्तर:
राष्ट्रपति की विधायी शक्तियाँ इस प्रकार हैं-

  • राष्ट्रपति को संसद के दोनों सदनों का अधिवेशन बुलाने तथा उसे स्थगित करने का अधिकार है,
  • राष्ट्रपति संसद के एक अथवा दोनों सदनों के संयुक्त अधिवेशन में भाषण दे सकता है,
  • राष्ट्रपति लोक सभा को उसका निश्चित कार्यकाल समाप्त होने से पहले भंग कर सकता है,
  • राष्ट्रपति को राज्य सभा में 12 सदस्य मनोनीत करने का अधिकार है,
  • राष्ट्रपति को संसद के दोनों सदनों द्वारा पास किए गए विधेयकों को स्वीकृति प्रदान करने का अधिकार है। वह ऐसे किसी विधेयक को वीटो (Veto) भी कर सकता है तथा उसे संसद के पास पुनर्विचार के लिए भेज सकता है।

प्रश्न 15.
अध्यादेश का अर्थ समझाइए।
उत्तर:
जब संसद का अधिवेशन न चल रहा हो और देश में ऐसी परिस्थिति अथवा आवश्यकता उत्पन्न हो जाए कि जिसके लिए तुरन्त कानून की आवश्यकता हो तो राष्ट्रपति अध्यादेश जारी कर सकता है, जिसका प्रभाव कानून के समान होता है। अध्यादेश की अधिकतम अवधि 6 महीने तक हो सकती है, लेकिन यदि संसद अध्यादेश को मंजूरी दे दे तो अध्यादेश कानून बन जाता है। अध्यादेश संसद का अधिवेशन शुरू होने पर शीघ्र ही मंजूरी के लिए संसद के सामने रखा जाता है। यदि अध्यादेश संसद में न रखा जाए अथवा संसद अध्यादेश पर कोई कार्रवाई न करे तो अध्यादेश अधिवेशन शुरू होने की तारीख से 6 सप्ताह बाद अपने आप समाप्त हो जाता है।

प्रश्न 16.
राष्ट्रपति किन परिस्थितियों में संकटकाल स्थिति की घोषणा कर सकता है? राष्ट्रपति द्वारा राष्ट्रीय संकटकाल की घोषणा अब तक कितनी बार की जा चकी है ?
उत्तर:
राष्ट्रपति निम्नलिखित तीन परिस्थितियों में संकटकाल की घोषणा कर सकता है
(1) युद्ध, बाहरी आक्रमण, सशस्त्र विद्रोह या आन्तरिक अशान्ति अथवा उनमें से किसी के भय की स्थिति में (अनुच्छेद 352) इस प्रकार की घोषणा राष्ट्रपति द्वारा अब तक तीन बार की जा चुकी है-

  • सन् 1962 में भारत पर चीन के आक्रमण के कारण,
  • सन् 1971 में भारत पर पाकिस्तान के आक्रमणों के कारण,
  • सन् 1975 में आन्तरिक अशान्ति के कारण।

(2) किसी राज्य में संवैधानिक तन्त्र के विफल हो जाने की स्थिति में। ऐसी घोषणा राष्ट्रपति द्वारा भारत के विभिन्न राज्यों में अब तक 100 से अधिक बार की जा चुकी है। (अनुच्छेद-356)

(3) वित्तीय संकट (अनुच्छेद 360) के कारण। जब राष्ट्रपति यह अनुभव करे कि देश में वित्तीय स्थिति बहुत खराब हो गई है और देश की वित्तीय साख खतरे में है, तो वह वित्तीय संकट की घोषणा कर सकता है। इस प्रकार की घोषणा राष्ट्रपति द्वारा अभी तक नहीं की गई है।

प्रश्न 17.
राष्ट्रपति की वित्तीय शक्तियाँ लिखें।
उत्तर:
राष्ट्रपति की वित्तीय शक्तियाँ निम्नलिखित हैं
1. बजट-संसद में बजट राष्ट्रपति के नाम पर ही पेश किया जाता है जिसमें संघ सरकार की आय-व्यय का ब्यौरा होता है।

2. धन विधेयक संसद में कोई भी धन विधेयक राष्ट्रपति की अनुमति के बिना पेश नहीं किया जा सकता। यह काम राष्ट्रपति के नाम पर किसी मन्त्री के द्वारा किया जाता है।

3. आकस्मिक निधि-भारत की आकस्मिक निधि राष्ट्रपति के अधीन है और उसमें से धन खर्च करने का अधिकार राष्ट्रपति को है।

4. वित्त आयोग की नियुक्ति हर पाँच वर्ष के पश्चात् राष्ट्रपति को वित्त आयोग नियुक्त करने का अधिकार है जो वित्तीय मामलों पर अपनी सिफारिश देता है।

प्रश्न 18.
भारत के उप-राष्ट्रपति का चुनाव कैसे किया जाता है?
उत्तर:
भारत के उप-राष्ट्रपति का चुनाव एक निर्वाचक मण्डल द्वारा किया जाता है जिसमें संसद के दोनों सदनों के सदस्य शामिल होते हैं। संसद के मनोनीत सदस्यों को भी उप-राष्ट्रपति के चुनाव में मतदान करने का अधिकार है, जबकि राष्ट्रपति के चुनाव में ऐसे सदस्यों को मतदान करने का अधिकार नहीं है। उसका चुनाव आनुपातिक प्रतिनिधित्व प्रणाली के आधार पर एकल संक्रमणीय मत प्रणाली द्वारा गुप्त मतदान की रीति से होता है।

प्रश्न 19.
संघीय मन्त्रिपरिषद् का निर्माण कैसे होता है?
उत्तर:
राष्ट्रपति लोक सभा में बहुमत प्राप्त दल अथवा दलीय गठबन्धन के नेता को प्रधानमन्त्री नियुक्त करता है। प्रधानमन्त्री की सलाह के अनुसार राष्ट्रपति अन्य मन्त्रियों की नियुक्ति करता है तथा उनमें विभागों का बँटवारा करता है। संघीय मन्त्रिपरिषद् के निर्माण में राष्ट्रपति की भूमिका औपचारिक मात्र है। हाँ, यदि लोक सभा में किसी दल अथवा दलीय गठबन्धन को स्पष्ट बहुमत प्राप्त न हो तो राष्ट्रपति अपने स्वविवेक से किसी को भी प्रधानमन्त्री पद के दावेदार को प्रधानमन्त्री नियुक्त कर सकता है और उसे. एक निश्चित अवधि में लोक सभा में बहुमत सिद्ध करने का आदेश दे सकता है।

प्रश्न 20.
मन्त्रिपरिषद् में कितने प्रकार के मन्त्री होते हैं ? बताइए।
उत्तर:
मन्त्रि-परिषद् में निम्नलिखित प्रकार के मन्त्री होते हैं
1. कैबिनेट मन्त्री-ये मन्त्रि-परिषद् के सर्वाधिक महत्त्वपूर्ण मन्त्री होते हैं। इनके पास शासन के महत्त्वपूर्ण विभाग होते हैं। ये मन्त्रिमण्डल के सदस्य होते हैं और शासन की नीतियों को बनाते हैं।

2. राज्य मन्त्री-ये मन्त्रिमण्डल के सदस्य नहीं होते। आमतौर पर ये केबिनेट मन्त्री के सहायक होते हैं, लेकिन इन्हें विभागों का स्वतन्त्र रूप से कार्यभार सौंपा जा सकता है।

3. उप-मन्त्री-ये तीसरी श्रेणी के मन्त्री होते हैं। ये मन्त्रिमण्डल के सदस्य नहीं होते। इन्हें विभागों का स्वतन्त्र रूप से कार्यभार नहीं सौंपा जाता।

प्रश्न 21.
मन्त्रिमण्डल तथा मन्त्रिपरिषद् में क्या अन्तर है?
उत्तर:
प्रायः मन्त्रिमण्डल तथा मन्त्रि-परिषद् को एक ही समझा जाता है, परन्तु इन दोनों में अन्तर है। यह निम्नलिखित बातों से स्पष्ट है
(1) मन्त्रि-परिषद् को संवैधानिक मान्यता प्राप्त है अर्थात् इसकी स्थापना संविधान द्वारा की गई है, परन्तु मन्त्रिमण्डल की रचना संवैधानिक सुविधा के लिए की जाती है,

(2) मन्त्रि-परिषद् में सभी प्रकार के मन्त्री मन्त्रिमण्डलीय मन्त्री, राज्य मन्त्री तथा उप-मन्त्री शामिल होते हैं। परन्तु मन्त्रिमण्डल में केवल पहली प्रकार के ही (मन्त्रिमण्डलीय) मन्त्री शामिल होते हैं। दूसरे शब्दों में परिषद् का एक भाग होता है। मन्त्रिमण्डल में प्रायः 15-20 तक मन्त्री होते हैं जबकि मन्त्रि-परिषद् में 50-60 कई ससे भी अधिक मन्त्री होते हैं,

(3) मन्त्रिमण्डल का प्रत्येक सदस्य मन्त्रि-परिषद् का सदस्य होता है, परन्तु कुछ महत्त्वपूर्ण विभागों के अध्यक्ष ही मन्त्रिमण्डल के सदस्य होते हैं।

प्रश्न 22.
भारतीय मन्त्रिमण्डल प्रणाली की पाँच विशेषताएँ बताइए।
उत्तर:
भारतीय मन्त्रिमण्डल प्रणाली की विशेषताएँ इस प्रकार हैं-

  • राष्ट्रपति नाममात्र का अध्यक्ष है,
  • मन्त्री संसद के दोनों सदनों में से लिए जाते हैं,
  • प्रधानमन्त्री मन्त्रिमण्डल का नेतृत्व करता है,
  • मन्त्रिमण्डल लोक सभा के प्रति प्रत्यक्ष रूप से उत्तरदायी है,
  • मन्त्रिमण्डल सामूहिक उत्तरदायित्व के सिद्धान्त के आधार पर कार्य करता है।

प्रश्न 23.
केन्द्रीय मन्त्रिपरिषद् के पाँच कार्य संक्षेप में बताइए।
उत्तर:
केन्द्रीय मन्त्रि-परिषद के कार्य इस प्रकार हैं–

  • देश में कानून व्यवस्था तथा शान्ति व्यवस्था बनाए रखना मन्त्रि-परिषद् का उत्तरदायित्व है,
  • देश की विदेश-नीति तय करना,
  • राष्ट्रपति को विभिन्न विषयों पर सलाह देना,
  • बजट तैयार करना,
  • मन्त्रि-परिषद् कानून-निर्माण में महत्त्वपूर्ण भूमिका निभाती है।

प्रश्न 24.
मन्त्रिमण्डल के सामूहिक उत्तरदायित्व पर एक नोट लिखें।
उत्तर:
भारत में संसदीय शासन-प्रणाली होने के कारण मन्त्रिमण्डल का सामूहिक उत्तरदायित्व रहता है। इसका अर्थ यह है कि मन्त्री अपने कार्यों के लिए संसद के प्रति सामूहिक रूप से उत्तरदायी हैं। मन्त्रिमण्डल में बहुमत द्वारा लिया गया निर्णय समस्त मन्त्रिमण्डल का निर्णय माना जाता है और संसद यदि किसी एक मन्त्री अथवा प्रधानमन्त्री के विरुद्ध भी अविश्वास का प्रस्ताव पास कर दे तो समस्त मन्त्रिमण्डल को अपना त्यागपत्र देना पड़ता है। सभी मन्त्री एक ही साथ तैरते हैं और एक ही साथ डूबते हैं।

प्रश्न 25.
‘मन्त्रियों के व्यक्तिगत उत्तरदायित्व’ पर संक्षिप्त नोट लिखिए।
उत्तर:
मन्त्रियों के सामूहिक उत्तरदायित्व के साथ-साथ उनका व्यक्तिगत उत्तरदायित्व भी होता है। प्रत्येक मन्त्री एक अथवा अधिक विभागों का अध्यक्ष होता है, जिनका प्रशासन चलाने के लिए वह व्यक्तिगत रूप से उत्तरदायी होता है अथवा मन्त्रालय के शासन का संचालन सुचारु रूप से नहीं होता, तो उसके लिए उस विभाग का मन्त्री व्यक्तिगत रूप से उत्तरदायी होता है। वह अपने विभाग के असैनिक अधिकारियों को उसके लिए उत्तरदायी नहीं ठहरा सकता। अपने विभाग के किसी गलत कार्य के लिए उसे त्यागपत्र भी देना पड़ सकता है। उदाहरणस्वरूप, श्री लाल बहादुर शास्त्री ने रेलवे मन्त्री के रूप में एक रेल दुर्घटना के लिए स्वयं को नैतिक रूप से उत्तरदायी ठहराते हुए अपना त्यागपत्र दे दिया था।

HBSE 11th Class Political Science Important Questions Chapter 4 कार्यपालिका

प्रश्न 26.
राष्ट्रपति तथा मन्त्रिमण्डल में क्या सम्बन्ध है?
उत्तर:
संविधान के अनुसार मन्त्रिमण्डल की नियुक्ति राष्ट्रपति को परामर्श देने के लिए की जाती है। राष्ट्रपति ही प्रधानमन्त्री की नियुक्ति करता है, प्रधानमन्त्री की सलाह से वह अन्य मन्त्रियों की नियुक्ति करता है। संविधान स्पष्ट रूप से कहता है कि मन्त्रिमण्डल या मन्त्रि-परिषद् के सदस्य तभी तक अपने पद पर रह सकते हैं जब तक राष्ट्रपति की इच्छा उन्हें उनके पद पर रखने की है।

वास्तव में राष्ट्रपति नाममात्र का अध्यक्ष है। वह राज्य का अध्यक्ष है, शासन का नहीं। परन्तु यह हमारे संविधान का सैद्धांतिक रूप है। व्यावहारिक परम्परा यह है कि राष्ट्रपति संसद में बहुसंख्यक दल के नेता को ही प्रधानमन्त्री पद सम्भालने के लिए कहता है। मन्त्रियों का चुनाव वास्तव में प्रधानमन्त्री ही करता है।

राष्ट्रपति मन्त्रिमण्डल की बैठकों में भाग नहीं लेता। उनकी अध्यक्षता प्रधानमन्त्री ही करता है। प्रधानमन्त्री के नेतृत्व में ही की गृह-नीति, विदेश-नीति, वित्तीय-नीति आदि को निर्धारित करता है। प्रधानमन्त्री का यह कर्तव्य है कि वह मन्त्रिमण्डल द्वारा किए गए सब फैसलों की सूचना राष्ट्रपति को दे। राष्ट्रपति यदि चाहे तो किसी भी मन्त्रिमण्डल के फैसले को फिर से विचार करने के लिए वापस भेज सकता है। राष्ट्रपति मन्त्रिमण्डल द्वारा दिए गए परामर्श को मानने के लिए बाध्य है।

प्रश्न 27.
प्रधानमन्त्री की नियुक्ति कैसे की जाती है?
उत्तर:
प्रधानमन्त्री की नियुक्ति राष्ट्रपति द्वारा की जाती है, परन्तु यह कार्य राष्ट्रपति अपनी इच्छानुसार नहीं करता। प्रधानमन्त्री के पद पर राष्ट्रपति द्वारा प्रायः उसी व्यक्ति को नियुक्त किया जाता है, जो लोक सभा में बहुमत प्राप्त दल का नेता होता है। यदि लोक सभा में किसी राजनीतिक दल को पूर्ण बहुमत का समर्थन प्राप्त न हो तो राष्ट्रपति किसी भी ऐसे व्यक्ति को प्रधानमन्त्री नियुक्त कर सकता है जो उसकी दृष्टि में स्थायी सरकार की स्थापना करने के योग्य हो।

प्रश्न 28.
किन परिस्थितियों में प्रधानमन्त्री की नियुक्ति राष्ट्रपति अपनी इच्छानुसार कर सकता है?
उत्तर:
साधारणतः प्रधानमन्त्री के पद पर राष्ट्रपति द्वारा उसी व्यक्ति को नियुक्त किया जाता है जो लोक सभा में बहुसंख्यक दल का नेता हो। परन्तु निम्नलिखित परिस्थितियों में राष्ट्रपति अपनी इच्छानुसार प्रधानमन्त्री की नियुक्ति कर सकता है

  • जब लोक सभा में किसी राजनीतिक दल को पूर्ण बहुमत प्राप्त न हो,
  • जब लोक सभा का बहुसंख्यक दल अपने नेता का निश्चय न कर सके,
  • जब कई दल मिलकर भी संयुक्त सरकार का निर्माण न कर सके।

इन परिस्थितियों में राष्ट्रपति किसी भी ऐसे व्यक्ति को प्रधानमन्त्री के पद पर नियुक्त कर सकता है जो उसकी दृष्टि में स्थायी सरकार की स्थापना करने के योग्य होता है।

प्रश्न 29.
प्रधानमन्त्री के कोई पाँच कार्य बताएँ।।
उत्तर:
प्रधानमन्त्री के पाँच कार्य इस प्रकार हैं-

  • प्रधानमन्त्री अपने मन्त्रि-परिषद् का निर्माण करता है,
  • वह विभिन्न मन्त्रियों के बीच विभागों का विभाजन करता है तथा उसमें अपनी इच्छानुसार किसी भी समय फेर-बदल कर सकता है,
  • वह मन्त्रिमण्डल की बैठकों की अध्यक्षता करता हैं,
  • वह शासन के विभिन्न विभागों में समन्वय बनाए रखता है,
  • वह राष्ट्रपति का मुख्य सलाहकार होता है।

निबंधात्मक प्रश्न

प्रश्न 1.
कार्यपालिका से क्या तात्पर्य है? इसके कार्यों का वर्णन करें।
उत्तर:
कार्यपालिका सरकार का महत्त्वपूर्ण अंग है। यह राज्य की उन इच्छाओं को कार्यान्वित करती है, जिन्हें कानून का रूप दिया जाता है। प्रायः कार्यपालिका सरकार का प्रत्यक्ष दिखाई देने वाला सबसे अधिक क्रियाशील अंग है। कार्यपालिका सरकार की वह शाखा है जो कानून द्वारा व्यक्त की गई लोगों की इच्छा को लागू करती है, इसलिए साधारण व्यक्ति इसे ही सम्पूर्ण सरकार मानता है। कार्यपालिका के दो रूप हैं-संकुचित तथा व्यापक।

संकुचित अर्थों में कार्यपालिका के अन्तर्गत राज्याध्यक्ष तथा मन्त्री ही आते हैं, परन्तु व्यापक अर्थ में कार्यपालिका के अन्तर्गत राज्याध्यक्ष, मन्त्रिमण्डल सभी प्रशासनिक कर्मचारी सम्मिलित हैं। इस प्रकार राष्ट्रपति (सीमित राजतन्त्र में राजा) से लेकर एक सिपाही तक तथा एक चपरासी तक कार्यपालिका के सदस्य हैं। राजनीति विज्ञान के विद्यार्थी का केवल कार्यपालिका के संकुचित अर्थ से ही सम्बन्ध है। कार्यपालिका का अर्थ अग्रलिखित परिभाषाओं से स्पष्ट हो जाता है

1. गिलक्राइस्ट (Gilchrist) के अनुसार, “कार्यपालिका शासन का वह अंग है जो विधियों के रूप में निर्मित जन इच्छा को क्रियान्वित करता है।”

2. सी०एच० हिडलन (C.H. Hidlon) के अनुसार, “कार्यपालिका से आशय शासन के उस प्राधिकारी से है जो विधियों को क्रियान्वित करता है, प्रशासित करता है या प्रभावी बनाता है।”

3. गार्नर (Garner) के मतानुसार, “व्यापक और सामूहिक अर्थ में कार्यपालिका में वे सभी राज्य कर्मचारी और एजेन्सियाँ शामिल हैं जिनका सम्बन्ध राज्य की उन इच्छाओं को क्रियान्वित करने से है जिन्हें कानून के रूप में सुनिश्चित करके अभिव्यक्त किया जा चुका है।” उपरोक्त परिभाषाओं से कार्यपालिका का अर्थ स्पष्ट हो जाता है।

कार्यपालिका के कार्य (Functions of the Executive)-आधुनिक युग में राज्य और सरकार का स्वरूप बदल चुका है और इसने कल्याणकारी रूप धारण कर लिया है। इसके फलस्वरूप सरकार के एक प्रमुख अंग कार्यपालिका का कार्यक्षेत्र विस्तृत हो गया है और कार्यक्षेत्र लगातार बढ़ता जा रहा है। आधुनिक समय में कार्यपालिका को निम्नलिखित कार्य करने पड़ते हैं

1. कूटनीतिक कार्य (Diplomatic Functions) कार्यपालिका को अन्य देशों से सम्बन्ध स्थापित करने पड़ते हैं। इस क्षेत्र में विदेशों में राजदूत तथा अन्य राजनीतिक प्रतिनिधि भेजना, विदेशों से आए राजदूत को अपने यहाँ मान्यता देना, अन्य राज्यों को मान्यता देना अथवा न देना, सन्धि तथा समझौते करना, युद्ध तथा शान्ति की घोषणा करना आदि कार्य शामिल हैं। यह ठीक है कि इन कार्यों पर विधानमण्डल का निरीक्षण तथा निर्देशन भी रहता है, लेकिन फिर भी राज्य के विदेशी सम्बन्धों में कार्यपालिका ही सबसे अधिक प्रभावशाली है।

2. प्रशासनिक कार्य (Administrative Functions) कार्यपालिका का मुख्य कार्य विधानमण्डल द्वारा बनाए गए कानूनों का पालन करवाना तथा शासन चलाना है। इस कार्य के लिए अलग-अलग प्रशासनिक विभागों का गठन किया जाता है।

3. सैनिक कार्य (Military Functions) कार्यपालिका को देश की रक्षा के लिए सैनिक कार्य करने पड़ते हैं। बाहरी आक्रमण तथा आन्तरिक उपद्रवों से राज्य की रक्षा करने के लिए जल, थल, वायु तीनों प्रकार की सेनाएँ संगठित की जाती हैं। कार्यपालिका युद्ध काल में सेना का सफलतापूर्वक संचालन करती है। यद्यपि तीनों सेनाओं के सेनापति होते हैं, परन्तु राज्याध्यक्ष ही सेनाओं का सर्वोच्च सेनापति होता है। वह ही युद्ध आरम्भ तथा समाप्त करने की घोषणा करता है। आन्तरिक विद्रोह दबाने के लिए भी वह सेना का प्रयोग कर सकता है।

4. न्यायिक कार्य (Judicial Functions) कार्यपालिका न्याय करने का अधिकार भी रखती है। न्यायालय द्वारा दण्डित व्यक्तियों को राज्याध्यक्ष क्षमा-दान दे सकता है, सजा को कम कर सकता है अथवा कुछ समय के लिए रुकवा सकता है। वह किसी अवसर पर आम माफी की घोषणा भी कर सकता है जिससे बड़ी संख्या में कैदियों की मुक्ति हो जाती है। कार्यपालिका न्यायालयों का संगठन करती है तथा न्यायाधीशों की नियुक्ति भी करती है।

5. विधायनी कार्य (Legislative Functions) कार्यपालिका विधायनी क्षेत्र में भी अधिकार रखती है। वही विधानमण्डल के अधिवेशन बुलाती, स्थगित करती तथा निम्न सदन को भंग करके नए चुनाव करवाती है। यद्यपि कानून विधानमण्डल ही बनाता है, परन्तु अधिकांश विधेयक कार्यपालिका द्वारा ही पेश किए जाते हैं तथा उन्हें पास करवाने में भी मन्त्रियों का विशेष प्रभाव रहता है। कार्यपालिका अध्यक्ष की स्वीकृति दिए बिना कोई विधेयक कानून नहीं बनता। राज्याध्यक्ष अध्यादेश भी जारी कर सकता है। इस प्रकार कार्यपालिका कानून बनाने में भी सहायक सिद्ध होती है।

6. वित्तीय कार्य (Financial Functions) कार्यपालिका ही सरकार का बजट तैयार करती है तथा उसे संसद में पेश करके पास करवाती है। कानून के अनुसार कर, ऋण तथा माल गुजारी एकत्रित करती है। विभिन्न प्रशासनिक विभागों के खर्च के लिए धन की स्वीकृति विधान-मण्डल से पास करवाती है।

7. विविध कार्य (Miscellaneous Functions)-प्रो० गार्नर ने कार्यपालिका अध्यक्ष के कुछ विभिन्न कार्यों का उल्लेख भी किया है। कुछ देशों के राज्याध्यक्षों को व्यावसायिक उपाधियाँ प्रदान करने, विदेशियों को नागरिकता देने, अधिकारियों की विधवाओं तथा अनाथ बच्चों को पेन्शन तथा भत्ते दिलाने, प्राइवेट नियमों को कानूनी करार देने अथवा निरस्त करने तथा राज्य की सुरक्षा के लिए यथोचित कदम उठाने आदि के भी अधिकार प्राप्त हैं।

निष्कर्ष (Conclusion)-उपर्युक्त वर्णित कार्यों से स्पष्ट होता है कि कार्यपालिका को विभिन्न कार्य करने पड़ते हैं, परन्तु कार्यपालिका का मुख्य कार्य शासन का प्रबन्ध करना और कानूनों को लागू करना है। शासन का क लिए उसको और भी काम सौंपे जाते हैं। प्रत्येक देश में एक जैसी कार्यपालिका नहीं है, अतः कार्य भी एक जैसे नहीं हैं।

प्रश्न 2.
कार्यपालिका के विभिन्न रूपों का वर्णन करो।
उत्तर:
कार्यपालिका एक तरह की नहीं कई प्रकार की होती है। आज विश्व में सरकार एक तरह की नहीं है। अलग-अलग देशों में अलग-अलग तरह की सरकारें हैं। इसलिए कार्यपालिकाएँ भी एक नहीं, भिन्न-भिन्न प्रकार की हैं जो निम्नलिखित हैं

1. नाममात्र की कार्यपालिका (Nominal Executive)-जब शासन का कार्य किसी व्यक्ति के नाम पर चलाया जाता है तथा वास्तविक शक्तियों का प्रयोग कोई और करता है तो उसे नाममात्र की कार्यपालिका कहा जाता है। इंग्लैण्ड की रानी और भारत का राष्ट्रपति नाममात्र के कार्यपालिका अध्यक्ष हैं। इनकी सभी संवैधानिक शक्तियों का प्रयोग मन्त्रिमण्डल करता है। अध्यक्ष का केवल नाम ही रहता है।

2. वास्तविक कार्यपालिका (Real Executive)-जब कोई व्यक्ति वास्तविक रूप से सभी कार्यपालिका की शक्तियों का प्रयोग करता है जो संविधान तथा कानून द्वारा उसको दी गई है तो उसे वास्तविक कार्यपालिका कहा जाता है। अमेरिका का राष्ट्रपति, भारत तथा इंग्लैण्ड के मन्त्रिमण्डल वास्तविक कार्यपालिका हैं।

3. एकल कार्यपालिका (Single Executive)-जब शासन की सारी शक्तियों का प्रयोग एक ही व्यक्ति करता है तो उसे एकल कार्यपालिका कहा जाता है। अमेरिका, चिल्ली, मैक्सिको, पीरू, ब्राजील आदि देशों में एकल कार्यपालिका है। इन देशों के तियों को ही सम्पूर्ण शासन सत्ता प्राप्त है। इनके विधानमण्डल भी हैं, परन्तु मन्त्रिमण्डल के सदस्य राष्ट्रपति के निजी सचिवों के रूप में कार्य करते हैं।

4. बहुसंख्यक कार्यपालिका (Plural Executive) बहुसंख्यक कार्यपालिका में कार्यपालिका शक्तियाँ एक से अधिक व्यक्तियों में निहित होती हैं। रूस तथा स्विट्ज़रलैण्ड में बहुसंख्यक कार्यकारिणी परिषद् है।

5. अध्यक्षात्मक कार्यपालिका (Presidential Executive) अध्यक्षात्मक कार्यपालिका में एक निर्वाचित राष्ट्रपति प्रमख शासनाध्यक्ष होता है। सारे शासन के लिए वही उत्तरदायी होता है। वह अपनी सहायता के लिए सचिव अथवा मन्त्री नियुक्त करता है, परन्तु इन मन्त्रियों का विधानमण्डल से कोई सम्बन्ध नहीं होता तथा न ही वे विधानमण्डल के प्रति उत्तरदायी होते हैं। संयुक्त राज्य अमेरिका तथा दक्षिणी अमेरिका के देशों में अध्यक्षात्मक कार्यपालिका है।

6. संसदीय कार्यपालिका (Parliamentary Executive) संसदीय कार्यपालिका उसे कहा जाता है जिसमें मुख्य कार्यपालिका अध्यक्ष नाममात्र की सत्ता रखता है तथा वास्तविक सत्ता का प्रयोग मन्त्रिमण्डल द्वारा किया जाता है। मन्त्रिमण्डल अपने सभी कार्यों – के लिए व्यक्तिगत तथा सामूहिक रूप से विधानमण्डल के प्रति उत्तरदायी होता है। दोनों एक दूसरे पर नियन्त्रण रखते हैं। भारत, इंग्लैण्ड, कनाडा, जापान आदि देशों में संसदीय कार्यपालिका की स्थापना की गई है।

7. वंशानुगत कार्यपालिका (Hereditary Executive)-जिस राज्य में राजतन्त्र स्थापित है, वहाँ वंशानुगत अथवा पैतृक कार्यपालिका होती है। एक राजा अथवा रानी के मरने के पश्चात् उसका बेटा अथवा बेटी गद्दी पर बैठती है तो समस्त कार्यपालिका शक्तियाँ उसको प्राप्त होती हैं।

8. निर्वाचित कार्यपालिका (Elected Executive)-जिस राज्य में कार्यपालिका अध्यक्ष जनता द्वारा प्रत्यक्ष अथवा अप्रत्यक्ष रूप से एक निश्चित अवधि के लिए निर्वाचित किया जाता है, उसे निर्वाचित कार्यपालिका कहा जाता है। भारत, अमेरिका आदि देशों में निर्वाचित कार्यपालिका है।

9. मनोनीत कार्यपालिका (Nominated Executive)-उपनिवेशों तथा संघ की इकाइयों में कार्यपालिका अध्यक्ष किसी उच्च सत्ता द्वारा मनोनीत किए जाते हैं। कनाडा, ऑस्ट्रेलिया, न्यूज़ीलैण्ड के गवर्नर राष्ट्रपति द्वारा मनोनीत किए जाते हैं। भारतीय संघ में भी राज्यों के गवर्नर राष्ट्रपति द्वारा मनोनीत किए जाते हैं।

10. राजनीतिक कार्यपालिका (Political Executive)-अस्थायी अथवा राजनीतिक कार्यपालिका मन्त्रिमण्डल के सदस्य हैं जो बहुमत तक अपने पदों पर बने रहते हैं। विधानमण्डल इन्हें पहले भी अपने पद से हटा सकते हैं। इनका पद अस्थायी होता है। इनकी नियुक्ति राजनीतिक दलों से होने के कारण इन्हें राजनीतिक कार्यपालिका भी कहा जाता है। इनका पद स्थायी नहीं होता, ये कभी भी स्वयं अपने पद से त्यागपत्र दे सकते हैं अथवा इन्हें एक निश्चित अवधि से पहले भी हटाया जा सकता है, इसलिए इन्हें अस्थायी कार्यपालिका कहा जाता है।

11. स्थायी कार्यपालिका (Permanent Executive)-प्रशासन के अधिकारी तथा अन्य कर्मचारी स्थायी कार्यपालिका कहलाते हैं। इन्हें एक निश्चित आयु पर नौकरी दी जाती है और निश्चित आयु तक वे अपने पद पर बने रहते हैं। पद से निवृत्त होने पर उन्हें पेन्शन दी जाती है। सरकार के बदलने पर भी ये अपने पद पर बने रहते हैं।

12. तानाशाही कार्यपालिका (Dictatorial Executive) यदि कोई सैनिक अधिकारी सैनिक क्रान्ति के बल पर शासन सत्ता अपने नियन्त्रण में ले लें तो वह तानाशाह कहलाता है। आज भी बहुत-से देशों में किसी-न-किसी रूप में तानाशाही सरकारें हैं। तानाशाह का वंशानुगत अथवा संवैधानिक आधार नहीं होता। – निष्कर्ष (Conclusion)-इस प्रकार कार्यपालिका के विविध रूपों का अध्ययन करने पर हम इस निष्कर्ष पर पहुँचते हैं कि वर्तमान युग में सैद्धान्तिक तौर पर कार्यपालिका के रूपों को स्थायी रूप में निश्चित करना सरल नहीं है। प्रत्येक देश की राजनीतिक, आर्थिक तथा सामाजिक स्थिति भिन्न है, इसलिए सब देशों में एक-सी कार्यपालिका नहीं है। आज कार्यपालिका विशुद्ध रूप में नहीं है, वह अनेक रूपों का मिश्रण है।

प्रश्न 3.
कार्यपालिका की सत्ता के विस्तार के लिए उत्तरदायी मुख्य कारकों की चर्चा कीजिए।
उत्तर:
सैद्धान्तिक दृष्टि से विधानपालिका सर्वोच्च है और कार्यपालिका उसके अधीन होती है। संसदीय शासन-प्रणाली में तो विधानमण्डल का कार्यपालिका पर पूरा नियन्त्रण होता है और विधानमण्डल को कार्यपालिका के विरुद्ध अविश्वास का प्रस्ताव पास करके उसे पद से हटाने का अधिकार होता है, परन्तु व्यवहार में आजकल कार्यपालिका की शक्तियों में निरन्तर वृद्धि हो रही है।

इंग्लैण्ड में तो संसद मन्त्रिमण्डल के हाथों का खिलौना मात्र बनकर रह गई है। रैम्जे म्यूर (Ramsay Muir) का कहना है कि इंग्लैण्ड में मन्त्रिमण्डल की शक्तियों का इतना विस्तार हो गया है कि वहाँ मन्त्रिमण्डल की तानाशाही स्थापित हो गई है। कार्यपालिका की शक्तियों में वृद्धि के लिए मुख्य रूप से निम्नलिखित कारण उत्तरदायी हैं

1. दलीय पद्धति (Party System)-आधुनिक लोकतन्त्रीय राज्यों में सरकार दलीय पद्धति के आधार पर चलाई जाती है। जिस राजनीतिक दल को विधानमण्डल में बहुमत प्राप्त हो जाता है, उसी दल की सरकार बनती है। दलीय अनुशासन के कारण कार्यपालिका को विधानमण्डल में बहुमत का समर्थन प्राप्त करने में कोई कठिनाई नहीं होती। उसके द्वारा संसद में कुछ भी पास करवाया जा सकता है।

2. निम्न सदन को भंग करने का अधिकार (Power of Dissolution of the Lower House)-संसदीय शासन-प्रणाली में कार्यपालिका के अध्यक्ष को संसद के निम्न सदन को भंग करने का अधिकार होता है। विधानमण्डल इस भय के कारण कार्यपालिका की नीतियों एवं काननों को स्वीकृति दे देती है।

3. जन-कल्याणकारी राज्य की धारणा (Concept of Welfare State)-वर्तमान युग में राज्य को जन-कल्याणकारी संस्था समझा जाता है। इसका अर्थ यह है कि राज्य जनता की भलाई के लिए अनेक कार्य करता है, जिससे कार्यपालिका के कार्यक्षेत्र में बहुत वृद्धि हो गई है।

4. प्रदत्त व्यवस्थापन (Delegated Legislation) काम की अधिकता, विशेष ज्ञान की कमी व समय की कमी के कारण विधानमण्डल के लिए प्रत्येक कानून को पूरे विस्तृत रूप में पास करना सम्भव नहीं होता। इसके परिणामस्वरूप, विधानमण्डल कानून .. की रूपरेखा का निर्माण करके विस्तृत नियम तथा अधिनियम बनाने का कार्य कार्यपालिका पर छोड़ देती है। इस सम्बन्ध में कार्यपालिका द्वारा बनाए गए नियम तथा अधिनियम भी उसी प्रकार से लागू होते हैं जैसे संसद द्वारा पास किए गए कानून। इससे भी कार्यपालिका की शक्ति में वृद्धि हुई है।

5. नियोजन (Planning)-आज का युग नियोजन का युग है। सभी देश अपने विकास के लिए योजनाएँ बनाते हैं। ये योजनाएँ तैयार करना, इन्हें लागू करना तथा इनका मूल्यांकन करना कार्यपालिका द्वारा ही सम्पन्न होता है। इससे कार्यपालिका की शक्ति का विस्तार हुआ है।

6. आधुनिक समस्याओं की जटिलता (Complicated Nature of Moderm Problems)- वर्तमान समय में राज्य को अनेक जटिल समस्याओं का सामना करना पड़ता है. जिसके लिए विशेष ज्ञान, योग्यता तथा अनुभव की आवश्यकता होती है। विधानमण्डल के सामान्य योग्यता के निर्वाचित सदस्य इन जटिल समस्याओं को समझने तथा उन्हें सुलझाने की योग्यता नहीं रखते। कार्यपालिका ही इन सभी समस्याओं को निपटाती है, जिससे उसके महत्व में काफी वृद्धि हुई है। उपरोक्त कारणों से कार्यपालिका की शक्तियों का विस्तार हुआ है।

HBSE 11th Class Political Science Important Questions Chapter 4 कार्यपालिका

प्रश्न 4.
भारत के राष्ट्रपति की चुनाव प्रक्रिया की संक्षिप्त व्याख्या कीजिए।
अथवा
अनुच्छेद 54 तथा 55 के अन्तर्गत भारत के राष्ट्रपति के चुनाव की क्या प्रक्रिया बताई गई है? अब तक राष्ट्रपति पद के लिए कितनी बार चुनाव हुए हैं?
अथवा
भारत के राष्ट्रपति का चुनाव कैसे होता है?
उत्तर:
भारत में संघीय सरकार की सभी कार्यपालिका शक्तियाँ राष्ट्रपति को प्रदान की गई हैं। अनुच्छेद 53 में कहा गया है कि केन्द्र की कार्यपालिका शक्ति राष्ट्रपति में निहित होगी, जिसका प्रयोग वह सीधे या अपने अधीनस्थ कर्मचारियों के माध्यम से करेगा। संघ का सारा शासन राष्ट्रपति के नाम पर चलाया जाता है। वह राज्य का अध्यक्ष है, परन्तु वह नाममात्र का अध्यक्ष है और अपनी कार्यपालिका शक्ति का प्रयोग मन्त्रिमण्डल के परामर्श से करता है। राष्ट्रपति अप्रत्यक्ष रूप से चुना जाता है।

योग्यताएँ (Qualifications)-राष्ट्रपति पद के लिए निश्चित की गई योग्यताएँ इस प्रकार हैं-

  • वह भारत का नागरिक हो,
  • वह 35 वर्ष की आयु पूरी कर चुका हो,
  • वह संसद का सदस्य बनने की योग्यता रखता हो,
  • वह भारत सरकार, राज्य सरकार या इनके नियन्त्रण में किसी स्थानीय सरकार के अधीन किसी लाभकारी पद पर आसीन न हो। राष्ट्रपति, उप-राष्ट्रपति, राज्यपाल, मन्त्री आदि के पद लाभकारी पद नहीं माने जाते,
  • वह राष्ट्रपति चुने जाने के पश्चात् संसद या राज्य विधान-मण्डल के किसी सदन का सदस्य नहीं रह सकता,
  • उसे नामांकन-पत्र के साथ 15000 रुपए की धनराशि सुरक्षा निधि के रूप में जमा करवानी पड़ती है,
  • उसके नाम का प्रस्ताव 50 निर्वाचकों द्वारा किया जाना चाहिए और 50 निर्वाचकों द्वारा अनुमोदित होना चाहिए।

राष्ट्रपति का चुनाव (Election of the President)-संविधान की धारा 54 के अनुसार, “राष्ट्रपति का चुनाव अप्रत्यक्ष रीति से एक निर्वाचक मण्डल द्वारा होगा जिसमें संसद के दोनों सदनों तथा राज्यों की विधानसभाओं के निर्वाचित सदस्य भाग लेंगे।” इस प्रकार संसद के दोनों सदनों तथा राज्य विधानसभाओं के मनोनीत (Nominated) सदस्यों को राष्ट्रपति के चुनाव में भाग लेने का अधिकार नहीं दिया गया है। यहाँ यह भी उल्लेखनीय है कि राज्यों में द्वितीय सदन विधानपरिषद् के सदस्यों को भी राष्ट्रपति चुनाव में भाग लेने का अधिकार नहीं है क्योंकि विधान परिषद् के साथ में दूसरा सदन सभी राज्यों में नहीं है।

संसद तथा राज्य विधानसभाओं के निर्वाचित सदस्य राष्ट्रपति का चुनाव एक विशेष चुनाव पद्धति, जिसे संविधान द्वारा ‘आनुपातिक प्रतिनिधित्व पद्धति के अनुसार एकल संक्रमणीय मत पद्धति’ (Proportional Representation and Single Transferable Vote System) का नाम दिया गया है, के अनुसार करेंगे। चुनाव गुप्त मतदान द्वारा होगा। इसके अतिरिक्त केन्द्र प्रशासित प्रदेशों, संघीय क्षेत्र राष्ट्रीय राजधानी दिल्ली एवं पाण्डिचेरी विधानसभा को सन् 1997 में कानून बनाकर भाग लेने का अधिकार दिया गया।

राष्ट्रपति को वस्तुतः राष्ट्र के प्रतिनिधि का रूप देने के लिए संविधान में यह व्यवस्था की गई है कि राष्ट्रपति के चुनाव में भाग लेने वाले, राज्य विधानसभाओं के सदस्यों तथा संसद के सदस्यों के मतों में समानता हो और जहाँ तक हो सके सभी राज्यों के प्रतिनिधित्व में एकरूपता हो। राज्य विधानसभाओं के निर्वाचित सदस्यों की संख्या समान न होने के कारण तथा विधानसभाओं के निर्वाचित सदस्यों की कुल संख्या संसद के निर्वाचित सदस्यों की संख्या से बहुत अधिक होने के कारण राज्यों में एकरूपता और संघ तथा राज्यों में समानता के उद्देश्य की पूर्ति एक व्यक्ति को एक मत देने के सिद्धान्त द्वारा नहीं हो सकती, इसलिए इसके स्थान पर एक मतदाता को कई मत देने के सिद्धान्त को अपनाया गया है।

एक मतदाता के मतों की संख्या को जानने के लिए एक नई विधि की व्यवस्था की गई है। इस विधि के अनुसार किसी राज्य की विधानसभा का प्रत्येक सदस्य उतने मत दे सकता है जितने राज्य की जनसंख्या को विधानसभा के निर्वाचित सदस्यों की संख्या से भाग करके और भागफल को 1000 से भाग करके जो शेषफल आए। यदि भाग करने के पश्चात् शेषफल 500 से कम न हो तो मतों की संख्या में एक और मत की वृद्धि की जाती है। साधारणतः एक राज्य की विधानसभा के सदस्यों के मतों की संख्या इस प्रकार निश्चित की जाती है
HBSE 11th Class Political Science Important Questions Chapter 4 Img 1
संविधान के मसौदे में मुम्बई राज्य का हवाला देकर इस विधि (Formula) की व्याख्या की गई है, जिसके अनुसार उस समय मुम्बई राज्य की जनसंख्या 2,08,49,840 तथा विधानसभा के सदस्यों की संख्या 208 थी।
HBSE 11th Class Political Science Important Questions Chapter 4 Img 2
अधिकार है, क्योंकि शेष 239 पाँच सौ से कम हैं, इसलिए उन्हें छोड़ दिया जाता है। इस प्रकार प्रत्येक सदस्य 100 मत दे सकता है। संघ तथा राज्यों में समानता लाने के लिए संसद के सभी निर्वाचित सदस्यों को उतने ही मत देने का अधिकार प्राप्त होता है जितने मत सभी राज्य की विधानसभाओं के निर्वाचित सदस्यों द्वारा दिए जाते हैं। इस प्रकार संसद का प्रत्येक निर्वाचित सदस्य राष्ट्रपति के चुनाव में उतने मत डाल सकता है जितने कि राज्यों की विधानसभाओं के सदस्यों के लिए नियत सम्पूर्ण मत संख्या को संसद के दोनों सदनों के निर्वाचित सदस्यों की सम्पूर्ण संख्या से भाग देने से आए।
HBSE 11th Class Political Science Important Questions Chapter 4 Img 3
यदि भाग देने पर शेष आधा या आधे से अधिक हो तो इसे एक मानकर प्रत्येक संसद सदस्य के मतों की संख्या में एक मत और जोड़ वधान के मसौदे में दिए गए उदाहरण के अनुसार यदि यह मान लिया जाए कि सभी विधानसभाओं के निर्वाचित सदस्यों के मतों की कुल संख्या 74,940 है तथा संसद के निर्वाचित सदस्यों की संख्या 750 है तो संसद के प्रत्येक सदस्य को \(\frac{74,940}{750}=99 \frac{23}{25}\) मत देने का अधिकार होगा, क्योंकि \(\frac{23}{25}\) आधे से अधिक है, इसलिए संसद का प्रत्येक सदस्य 100 मत दे सकता है।

राष्ट्रपति के चुनाव का प्रबन्ध चुनाव आयोग (Election Commission) द्वारा किया जाता है। विधानसभाओं के सदस्य अपने-अपने राज्य की राजधानी में मतदान करते हैं, जबकि संसद सदस्य नई दिल्ली में मतदान करते हैं।

राष्ट्रपति चुनाव में एक विधायक के मत का मूल्य निम्न विधि से निकालते हैं
HBSE 11th Class Political Science Important Questions Chapter 4 Img 5
राष्ट्रपति चुनाव में एक सांसद के मत का मूल्य
राज्य के कुल निर्वाचित विधायकों के मतों का मूल्य ………………………………………..5,49,474
लोकसभा के कुल निर्वाचित सदस्यों की संख्या …………………………………………543
राज्यसभा के कुल निर्वाचित सदस्यों की संख्या ………………………………………… 233
कुल सांसदों की संख्या …………………………………………543+233=776
HBSE 11th Class Political Science Important Questions Chapter 4 Img 6
=708.085 यानी 776 सांसदों के कुल मतों का मूल्य =708 × 776 = 5,49,408
चौदहवें राष्ट्रपति चुनाव में भाग लेने वाले कुल मतदाताओं की संख्या =
कुल विधायक (4120)+ कुल सांसद (776)=4,896
सभी मतदाताओं के कुल मतों का मूल्य =549474+549408=10,98,882

मतदान तथा मतगणना की विधि (The System of Polling and Counting of Votes)-राष्ट्रपति का निर्वाचन एकल संक्रमणीय मत द्वारा आनुपातिक प्रतिनिधित्व चुनाव प्रणाली (Proportional Representation and Single Transferable Vote System) के आधार पर होता है। साधारणतः इस प्रणाली का प्रयोग एक व्यक्ति के चुनाव के लिए नहीं किया जाता। भारत में राष्ट्रपति के चुनाव के लिए इस प्रणाली को इसलिए अपनाया गया है कि निर्वाचित होने वाले व्यक्ति को वास्तविक बहसंख्या मत प्राप्त हों।

इस प्रणाली के अन्तर्गत प्रत्येक मतदाता अपनी पसन्द (Preference) जाहिर करता है। इसका अर्थ यह हुआ कि प्रत्येक मतदाता जितने राष्ट्रपति पद के लिए उम्मीदवार खड़े हों, उतनी पसन्दें जाहिर कर सकता है। व्यवहार में प्रत्येक मतदाता को मतदान करने के लिए एक-एक पर्ची दी जाती है और उस पर्ची पर सभी उम्मीदवारों के नाम लिखे होते हैं।

प्रत्येक मतदाता क्रमानुसार इन उम्मीदवारों के नामों के सामने नम्बर डालकर अपनी पसन्द प्रकट करता है और जो उम्मीदवार आवश्यक मत प्राप्त कर ले उसे निर्वाचित घोषित कर दिया जाता है। इस प्रणाली में आवश्यक मतों की संख्या. इस प्रकार निश्चित की जाती है कि सबसे पहले अशुद्ध मतों को रद्द कर दिया जाता है और फिर शुद्ध मतों के आधार पर कोटा निश्चित किया जाता है जो निम्न प्रकार से है
HBSE 11th Class Political Science Important Questions Chapter 4 Img 7
इस प्रणाली की व्याख्या इस प्रकार की जा सकती है। यदि यह मान लिया जाए कि कुल मतों की संख्या 10,000 है तथा राष्ट्रपति पद के लिए चार उम्मीदवार ‘क’, ‘ख’, ‘ग’ तथा ‘घ’ हैं और उन्हें पहली पसन्द में इस प्रकार मत प्राप्त हुए हैं
क = 4,000
ख = 3,000
ग = 1,600
घ = 1,400

साधारणतः बहुमत प्रतिनिधित्व प्रणाली के आधार पर इनमें से ‘क’ को चुना जाना चाहिए था, परन्तु आनुपातिक प्रतिनिधित्व प्रणाली के अनुसार ऐसा नहीं होता। इसके अनुसार यदि आवश्यक मतों की संख्या (Quota) को देखा जाए तो जीतने वाले उम्मीदवार के लिए \(\frac{10000}{1+1}+1=5001\) मतों को प्राप्त करना आवश्यक है। इस स्थिति में इनमें से किसी भी उम्मीदवार को सफल घोषित नहीं किया जा सकता।

इस दशा में सबसे कम मत प्राप्त करने वाले उम्मीदवार को असफल समझकर चुनाव क्षेत्र से निकाल दिया जाता है तथा उसके मतों को मतदाताओं की दूसरी पसन्द के आधार पर दूसरे उम्मीदवारों को हस्तान्तरित कर दिया जाता है। वर्तमान स्थिति में ‘घ’ को पहली पसन्द में सबसे कम मत प्राप्त हुए हैं, इसलिए उसे निर्वाचन क्षेत्र से निकाल दिया जाएगा और उसके मतों को दूसरी पसन्द के आधार पर ‘क’, ‘ख’ तथा ‘ग’ में विभक्त कर दिया जाएगा। यदि ‘घ’ के 1400 मतदाताओं की दूसरी पसन्द इस प्रकार हो ‘क’ = 200, ‘ख’ = 800, ‘ग’ = 400 तो इन मतों को हस्तान्तरित करने के बाद तीनों उम्मीदवारों की स्थिति इस प्रकार होगी
क = 4000 + 200 = 4200
ख = 3000 + 800 = 3800
ग = 1600 + 400 = 2000

अभी भी किसी उम्मीदवार को आवश्यक संख्या में मत प्राप्त नहीं हुए। इस दशा में ‘ग’ को निर्वाचन क्षेत्र से निकाल दिया जाएगा और उसके 2000 मतों को तीसरे पसन्द के आधार पर ‘क’ तथा ‘ख’ में बाँट दिया जाएगा। यदि तीसरे पसन्द में ‘क’ को 700 तथा ‘ख’ को 1300 मत प्राप्त हों तो हस्तान्तरण करने के पश्चात् स्थिति इस प्रकार होगी
क = 4200 + 700 = 4900
ख = 3800 + 1300 = 5100

परिणामस्वरूप, क्योंकि ‘ख’ को आवश्यक संख्या से अधिक मत प्राप्त हो गए हैं, इसलिए उसे निर्वाचित घोषित किया जाएगा। इस विधि का विशेष लाभ यह है कि इससे अधिक-से-अधिक मतदाताओं की पसन्द का पता चलता है और कोई भी व्यक्ति संसद में बहुमत दल से सम्बन्धित न होने पर भी राष्ट्रपति चुना जा सकता है।

इससे यह भी स्पष्ट हो जाता है कि केवल पहली पसन्द में अधिक मत प्राप्त करने वाला उम्मीदवार ही नहीं चुना जा सकता, कोई दूसरा उम्मीदवार भी चुना जा सकता है; जैसे उपर्युक्त उदाहरण में पहली पसन्द में ‘क’ को सबसे अधिक मत मिले, परन्तु अन्त में ‘ख’ को चुना गया। इसके विपरीत यदि तीसरी पसन्द में ‘क’ को 801 मत प्राप्त होते और ‘ख’ को 1199 मत प्राप्त होते तो परिणाम ‘क’ के पक्ष में होता, क्योंकि इस दशा में ‘क’ को आवश्यक संख्या में मत प्राप्त हो जाते हैं। जिस प्रकार
क = 4200 + 801 = 5001
ख = 3800 + 1199 = 4999

इस प्रक्रिया को पहली बार अगस्त, 1969 में राष्ट्रपति डॉ० जाकिर हुसैन की मृत्यु के पश्चात् राष्ट्रपति के चुनाव के लिए अपनाया गया, क्योंकि इससे पहले चारों चुनावों में निर्णय पहली पसन्द के आधार पर ही होता रहा और निर्वाचित उम्मीदवारों को पहली पसन्द में आवश्यक संख्या में मत प्राप्त होते रहे, परन्तु सन् 1969 के चुनाव में पहली पसन्द के आधार पर किसी भी उम्मीदवार को आवश्यक मत प्राप्त न होने के कारण दूसरी पसन्द के आधार पर निर्णय किया गया जिसके अनुसार राष्ट्रपति श्री वी०वी० गिरि निर्वाचित हुए।

15वें राष्ट्रपति का चुनाव, (14वाँ कार्यकाल) जुलाई, 2017 (The 15th Presidential Election, (14th Tenure) July, 2017) भारत के 15वें राष्ट्रपति का चुनाव 17 जुलाई, 2017 को सम्पन्न हुआ। इस चुनाव में सतारूढ़ एन०डी०ए० प्रत्याशी श्री रामनाथ कोविंद तथा यू०पी०ए० समर्थित प्रत्याशी श्रीमती मीरा कुमार मुख्य प्रतिद्वन्द्वी थे। इस चुनाव में निर्वाचक मण्डल के कुल सदस्यों 4896 (सांसद 776 एवं विधायकों 4120) में से कुल 4851 (768 सांसद एवं 4083 विधायकों) सदस्यों ने अपने मत का प्रयोग किया, जिनके मतों का कुल मूल्य 10,90,300 था। इनमें से 77 मत पत्र जिनके मतों का मूल्य 20,942, था, अवैध घोषित किए गए।

इस प्रकार वैध मत पत्रों की संख्या 4774 रही. जिनका कुल मत मूल्य 10,69,358 था। इनमें से विजयी प्रत्याशी श्री रामनाथ कोविंद को 7,02,044 (65.65%) मत एवं श्रीमती मीरा कुमार को 3,67,314 (34.35%) मत प्राप्त हुए। इस प्रकार राष्ट्रपति के रूप में निर्वाचित होने के लिए 50% से अधिक मत प्राप्त होने की आवश्यक शर्त को 20 जुलाई, 2017 को हुई मतगणना में पूर्ण करने के पश्चात श्री रामनाथ कोविंद को राष्ट्रपति चुनाव में विजयी घोषित किया गया जिन्हें 25 जुलाई, 2017 को तात्कालिक मुख्य न्यायाधीश जे०एस० खेहर के द्वारा संसद के केन्द्रीय कक्ष में आयोजित समारोह में उन्हें देश के सर्वोच्च पद की शपथ दिलाई।

भारत के नवनियुक्त राष्ट्रपति श्री रामनाथ कोविंद का जन्म 1 अक्टूबर, 1945 को उत्तर प्रदेश के कानपुर के निकट परौंख में हुआ था, जो राष्ट्रपति का कार्यभार ग्रहण करने से पूर्व 16 अगस्त, 2015 से 20 जून, 2017 तक बिहार के राज्यपाल पद पर थे। चुनाव प्रक्रिया की आलोचना (Criticism of the Method of Election)-राष्ट्रपति के पद के लिए अपनाई गई चनाव-प्रणाली की निम्नलिखित बातों के आधार पर आलोचना की गई है-(1) राष्ट्रपति के चनाव के लिए अपनाई गई।

आनुपातिक प्रतिनिधित्व प्रणाली (Proportional Representation) कहना गलत है। यह प्रणाली केवल वहीं अपनाई जा सकती है जहाँ निर्वाचित होने वाले सदस्यों की संख्या कम-से-कम तीन हो । डॉ० एम०पी० शर्मा के अनुसार इस प्रणाली को ‘विकल्पनात्मक मत प्रणाली’ (Alternative Vote System) कहना अधिक उचित है, (2) राष्ट्रपति की चुनाव-प्रणाली बहुत जटिल है। साधारण व्यक्ति इसे आसानी से नहीं समझ सकता।

यद्यपि राष्ट्रपति की चुनाव-प्रणाली में कई त्रुटियाँ हैं, परन्तु ये मुख्यतः सैद्धान्तिक हैं, व्यावहारिक नहीं। राष्ट्रपति के पद के लिए अभी तक हुए चुनावों में ये कठिनाइयाँ सामने नहीं आई हैं। राष्ट्रपति का कार्यकाल (Tenure of President)-भारत के राष्ट्रपति का चुनाव पाँच वर्ष के लिए होता है। यह समय उस शुरू होता है जिस दिन राष्ट्रपति अपना पद सम्भालता है। एक व्यक्ति कितनी ही बार इस पद के लिए चुना जा सकता है। राष्ट्रपति चाहे तो वह पाँच वर्ष से पहले भी अपने पद से त्याग-पत्र दे सकता है।

राष्ट्रपति पर महाभियोग (Impeachment of President)-राष्ट्रपति का कार्यकाल 5 वर्ष है, लेकिन संसद केवल उसे महाभियोग के द्वारा ही उसके पद से हटा सकती है, जिसका उल्लेख संविधान के अनुच्छेद 61 में किया गया है। इस विधि के अंतर्गत संसद के दोनों सदनों में से जो सदन आरोप लगाना चाहता है, उसको 1/4 सदस्यों के हस्ताक्षरों सहित इस प्रस्ताव को राष्ट्रपति के पास 14 दिन पूर्व भेजना पड़ता है। जब सदन में महाभियोग विषय पर चर्चा चल रही होती है, तो राष्ट्रपति ऐसे समय पर स्वयं उपस्थित होकर या किसी अन्य व्यक्ति द्वारा आरोपों का जवाब सदन में दे सकता है।

यदि वह सदन 2/3 बहुमत से यह प्रस्ताव पारित कर दे तो दूसरा सदन उन आरोपों की जाँच-पड़ताल करता है। यदि दूसरा सदन भी.2/3 बहुमत से उन आरोपों को सही मान ले, तो राष्ट्रपति को अपना पद रिक्त करना पड़ता है। ऐसी स्थिति में उप-राष्ट्रपति, राष्ट्रपति का कार्यभार संभालता है, लेकिन 6 महीने के अंदर-अंदर नए राष्ट्रपति का निर्वाचन अनिवार्य है।

वेतन तथा भत्ते (Salary and Allowances) वर्तमान में राष्ट्रपति का मासिक वेतन 5 लाख रुपए प्रतिमाह निश्चित किया गया तथा मूल वेतन का 50 प्रतिशत सेवानिवृत्ति पर पेंशन के रूप में प्राप्त होगा जो कि 1 जनवरी, 2016 से प्रभावी किया गया है। इसके अतिरिक्त उसे कई प्रकार के भत्ते तथा सुविधाएँ भी दी जाती हैं, जिन्हें समय-समय पर संसद निश्चित करती है। उसके निवास के लिए बिना किराए का सरकारी भवन दिया जाता है, जिसे ‘राष्ट्रपति भवन’ कहते हैं।

इसके अतिरिक्त उसे जीवन-पर्यन्त सरकार की ओर से चिकित्सा की सहायता दी जाती है। राष्ट्रपति का वेतन तथा भत्ते भारत की संचित निधि (Consolidated Fund of India) में से दिए जाते हैं जिस पर संसद का मतदान नहीं होता। संसद राष्ट्रपति के वेतन तथा भत्तों में परिवर्तन कर सकती है, परन्तु इन्हें राष्ट्रपति के कार्यकाल में कम नहीं किया जा सकता। राष्ट्रपति अपनी इच्छा से ऐच्छिक वेतन समर्पण कानून, 1950 (Voluntary Surrender of Salaries Act, 1950) के अनुसार अपने वेतन का कुछ भाग त्याग सकता है जिस प्रकार डॉ० राजेन्द्र प्रसाद तथा डॉ० राधाकृष्णन केवल 2,500 तथा श्री नीलम संजीवा रेड्डी 3,000 रुपए मासिक वेतन के रूप में लेते थे। राष्ट्रपति के वेतन पर आयकर भी लगता है।

इसके अतिरिक्त यहाँ यह भी उल्लेखनीय है कि मई, 2000 में संसद द्वारा पारित विधेयक के अनुसार पूर्व राष्ट्रपति की मृत्यु के पश्चात् उसकी पत्नी को आजीवन पेन्शन और सरकारी मकान उपलब्ध करवाने की व्यवस्था की गई है। इससे पूर्व ऐसा प्रावधान नहीं था। विधेयक के अनुसार, पूर्व राष्ट्रपति को मिलने वाली पेन्शन का आधा भाग उसकी पत्नी को आजीवन प्राप्त होगा।

राष्ट्रपति द्वारा शपथ (Oath by the President)-प्रत्येक राष्ट्रपति और प्रत्येक उस व्यक्ति को, जो राष्ट्रपति के रूप में कार्य कर रहा हो अथवा उसके कृत्यों का निर्वाह करता हो, अपना पद ग्रहण करने से पहले भारत के मुख्य न्यायाधीश के समक्ष निम्नलिखित रूप में शपथ लेनी पड़ती है। शपथ का उल्लेख संविधान के अनुच्छेद 60 में किया गया है।

“मैं (नाम) ईश्वर की शपथ लेता हूँ/सत्यनिष्ठा से प्रतिज्ञा करता हूँ कि मैं श्रद्धापूर्वक भारत के राष्ट्रपति पद का कार्यपालन (अथवा राष्ट्रपति के कृत्यों का निर्वाहन) करूँगा तथा अपनी पूरी योग्यता से संविधान और विधि का परिरक्षण, संरक्षण तथा प्रतिरक्षण करूँगा और मैं भारत की जनता की सेवा और कल्याण में निरत रहूँगा।”

HBSE 11th Class Political Science Important Questions Chapter 4 कार्यपालिका

प्रश्न 5.
भारत के राष्ट्रपति की शक्तियों की विवेचना कीजिए। अथवा भारत के राष्ट्रपति की कार्यकारी शक्तियों की विवेचना कीजिए।
उत्तर:
राष्ट्रपति राज्य का अध्यक्ष है और राष्ट्र का शासन उसी के नाम पर चलाया जाता है। राष्ट्रपति एक संवैधानिक अध्यक्ष है और अपनी शक्तियों का प्रयोग मन्त्रिमण्डल की सलाह के अनुसार करता है।

राष्ट्रपति की शक्तियाँ (Powers of the President)-राष्ट्रपति की शक्तियों को हम दो भागों में बाँट सकते हैं

(क) शान्तिकालीन शक्तियाँ तथा
(ख) आपातकालीन शक्तियाँ।

(क) शान्तिकालीन शक्तियाँ (Powers in Peace Time)-राष्ट्रपति की शान्तिकालीन शक्तियाँ निम्नलिखित हैं

  • कार्यकारी शक्तियाँ (Executive Powers) राष्ट्रपति की कार्यकारी शक्तियाँ निम्नलिखित हैं
  • राज्य का अध्यक्ष (Head of the State)-राष्ट्रपति राज्य का अध्यक्ष है और भारत का समस्त शासन उसके नाम पर चलता है। सभी कानून उसके नाम से लागू होते हैं।

(1) मन्त्रिमण्डल की नियुक्ति (Appointment of Council of Ministers):
राष्ट्रपति प्रधानमन्त्री की नियुक्ति करता है और उसकी सलाह से अन्य मन्त्रियों की नियुक्ति करता है। प्रधानमन्त्री की सलाह से राष्ट्रपति मन्त्रियों में विभागों का बँटवारा करता है और बाद में भी उनके विभाग बदल सकता है। राष्ट्रपति मन्त्रियों को प्रधानमन्त्री की सिफारिश पर पदच्युत भी कर सकता है। प्रधानमन्त्री केवल उसी व्यक्ति को नियुक्त किया जाता है जो लोकसभा में बहुमत का नेता हो।

(2) उच्च अधिकारियों की निय क्ति (Appointment of High Officers):
राष्ट्रपति को कुछ महत्त्वपूर्ण नियुक्तियाँ करने का अधिकार है जैसे कि राज्यपाल, उप-राज्यपाल, महालेखा परीक्षक, संघीय लोक सेवा आयोग के अध्यक्ष तथा सदस्य, भारत का मुख्य न्यायाधीश तथा उच्चतम न्यायालय व उच्च न्यायालय के न्यायाधीश, चुनाव आयुक्त, महान्यायवादी (Advocate General), वित्त आयोग के सदस्य आदि, परन्तु ये नियुक्तियाँ भी राष्ट्रपति अपनी इच्छा से नहीं, बल्कि मन्त्रिमण्डल की सलाह से करता है।

(3) सैनिक शक्तियाँ (Military Powers):
राष्ट्रपति भारत की सशस्त्र सेनाओं का सर्वोच्च सेनापति (Supreme Commander of the Armed Forces):
है और सशस्त्र सेनाओं के तीनों भागों (Army, Navy and Air Force) के सेनाध्यक्षों की नियक्ति करता है तथा समस्त सेना उसी के अधीन मानी जाती है।

(4) विदेशी मामलों से सम्बन्धित शक्तियाँ (Powers Relating to Foreign Affairs):
राष्ट्रपति विदेशों में भारत का प्रतिनिधित्व करता है। दूसरे देशों में भेजे जाने वाले राजदूत राष्ट्रपति द्वारा नियुक्त किए जाते हैं और दूसरे देशों से आने वाले राजदूत राष्ट्रपति को ही अपने नियुक्ति-पत्र पेश करते हैं तथा वहीं उनको स्वीकृत करता है। राष्ट्रपति ही दूसरे देशों के साथ युद्ध तथा शान्ति की घोषणा करता है और सभी सन्धियाँ उसके नाम पर की जाती हैं।

(5) केन्द्र शासित प्रदेशों का प्रशासक (Administrator of Union Territories):
केन्द्र प्रशासित प्रदेशों; जैसे दिल्ली, पुद्दचेरी, अरुणाचल प्रदेश का प्रशासन राष्ट्रपति के नाम पर चलता है और उन प्रदेशों के उच्च अधिकारी राष्ट्रपति के द्वारा नियुक्त किए जाते हैं।

(6) राज्यों को निर्देश देना (To Issue Directions to the States):
राष्ट्रपति को राज्यपालों को निर्देश देने की शक्ति प्राप्त है। वह राज्य सरकार को संघीय कानूनों के पालन के लिए, उनके आपसी सम्बन्धों में समन्वय लाने के लिए तथा कुछ राज्यपालों को जन-जातियों के विकास के लिए आवश्यक निर्देश जारी कर सकता है और इनका पालन करते हुए राज्यपाल राष्ट्रपति के प्रति उत्तरदायी होता है।

(7) राष्ट्रपति कुछ आयोगों वित्त आयोग, चुनाव आयोग, राज्य भाषा आयोग तथा पिछड़े वर्गों की दशा को सुधारने से सम्बन्धित आयोग आदि-की भी नियुक्ति करता है।

(8) राष्ट्रपति को यह अधिकार है कि वह प्रधानमन्त्री से उसके मन्त्रिमण्डल द्वारा लिए गए निर्णयों के सम्बन्ध में सूचना प्राप्त कर सकता है। वह अपनी किसी बात को प्रधानमन्त्री द्वारा मन्त्रिमण्डल के पास पहुंचा सकता है।

2. विधायिनी शक्तियाँ (Legislative Powers)-राष्ट्रपति को कुछ विधायिनी शक्तियाँ भी प्राप्त हैं

(1) राष्ट्रपति संसद का एक अंग है (President is a part of the Parliament):
संघ की विधायिनी शक्तियाँ संसद को प्राप्त हैं और राष्ट्रपति उसका एक अभिन्न अंग है। अनुच्छेद 79 के अन्तर्गत यह स्पष्ट है कि राष्ट्रपति तथा लोकसभा व राज्यसभा नामक दोनों सदनों से मिलकर ही संसद बनती है।

(2) संसद का अधिवेशन बुलाना तथा स्थगित करना (To Summon and Prorogue the Session of Parliament):
राष्ट्रपति संसद के दोनों सदनों के अधिवेशन एक ही समय में या अलग-अलग समय में बुला सकता है। अधिवेशन को बढ़ा सकता है और उसे स्थगित भी कर सकता है। उसे 6 महीने के अन्दर संसद का दूसरा अधिवेशन भी अवश्य बुलाना पड़ता है। यह काम वह मन्त्रिमण्डल की सलाह से करता है।

(3) संसद को सम्बोधित करना (To Address the Parliament):
राष्ट्रपति संसद के दोनों सदनों को इकट्ठा या अलग-अलग भी सम्बोधित कर सकता है। संसद का पहला और वर्ष का पहला अधिवेशन राष्ट्रपति के अभिभाषण से ही आरम्भ होता है, जिसमें वह सरकार की नीति और आवश्यकताओं पर प्रकाश डालता है। राष्ट्रपति का भाषण सरकार ही तैयार करती है।

(4) ससद में कुछ सदस्यों को मनोनीत करना (Nomination of Some Members in the Parliament):
संसद में कुछ सदस्य मनोनीत करने का अधिकार है। इस अधिकार का प्रयोग वह मन्त्रिमण्डल की सलाह से करता है। वह राज्यसभा में 12 सदस्य ऐसे मनोनीत करता है, जिन्होंने विज्ञान, साहित्य, कला, समाज सेवा आदि के क्षेत्र में ख्याति प्राप्त की हो। राष्ट्रपति लोकसभा में भी एंग्लो-इण्डियन समुदाय के दो सदस्य मनोनीत कर सकता था। यहाँ यह उल्लेखनीय है कि दिसम्बर, 2019 में पारित 104वें संवैधानिक संशोधन द्वारा एंग्लो-इण्डियन जाति की मनोनयन प्रणाली को समाप्त करने का निर्णय किया गया।

(5) लोकसभा को भंग करना (Power to Dissolve the Lok Sabha):
राष्ट्रपति लोकसभा को इसकी अवधि पूरी होने से पहले भी भंग करके दोबारा चुनाव करवा सकता है, परन्तु इस शक्ति का प्रयोग भी राष्ट्रपति मन्त्रिमण्डल की सलाह से ही करता है, अपनी इच्छा से नहीं। 1991 में चन्द्रशेखर सरकार के परामर्श पर तत्कालीन राष्ट्रपति आर० वेंकटरमन ने लोकसभा भंग की थी। इसी प्रकार तत्कालीन राष्ट्रपति श्री० के०आर० नारायणन ने तत्कालीन प्रधानमन्त्री श्री अटल बिहारी वाजपेयी के परामर्श पर । वर्ष 1999 में लोकसभा को भंग किया था।

(6) विधेयकों पर स्वीकृति (Assent on the Bills):
संसद द्वारा पास किया गया कोई भी विधेयक उस समय तक कानून नहीं बन सकता, जब तक उस पर राष्ट्रपति की स्वीकृति न हो जाए। राष्ट्रपति चाहे तो उस पर अपनी स्वीकृति देने की बजाए उसे संसद के पास पुनर्विचार के लिए वापस भेज सकता है। संसद ऐसी स्थिति में उस विधेयक पर राष्ट्रपति की सिफारिशों के सम्बन्ध में पुनः विचार करती है और यदि संसद उस विधेयक को मूल रूप में ही दोबारा पास कर दे तो उस पर राष्ट्रपति को अपनी स्वीकृति देनी ही पड़ती है।

महत्त्वपूर्ण तथ्य यह है कि भारत का राष्ट्रपति अमेरिकी राष्ट्रपति की भाँति विधेयक को अस्वीकार नहीं कर सकता। वह ज्यादा-से-ज्यादा स्वीकृति देने में देरी कर सकता है। राज्यों के राज्यपाल भी कुछ विशेष विषयों से सम्बन्धित विधेयकों को राष्ट्रपति की स्वीकृति के लिए भेज सकते हैं। संविधान संशोधन विधेयक पर राष्ट्रपति को अपनी स्वीकृति देनी ही पड़ती है।

(7) कुछ विधेयकों को संसद में पेश करने के लिए राष्ट्रपति की पूर्व स्वीकृति (Prior approval of the President for Introducing Certain Bills in Parliament):
संसद में कुछ विधेयक राष्ट्रपति की पूर्व स्वीकृति के बिना पेश नहीं किए जा लिए नए राज्यों को बनाने वर्तमान राज्यों की सीमाओं में परिवर्तन करने, राज्यों के नाम बदलने वाले विधेयक राष्ट्रपति की पूर्व स्वीकति के बाद ही संसद में पेश किए जा सकते हैं। तत्कालीन राष्ट्रपति श्री आर० वेंकटरमन ने सांसदों के पेन्शन सम्बन्धी विधेयक को इसीलिए स्वीकार करने से मना कर दिया था, क्योंकि उस पर उनकी पूर्व स्वीकृति नहीं ली गई थी।

(8) अध्यादेश जारी करना (To Issue Ordinances):
अनुच्छेद 123 के अन्तर्गत यह व्यवस्था है कि जब संसद का अधिवेशन न हो रहा हो तो राष्ट्रपति अध्यादेश जारी कर सकता है। ये अध्यादेश कानून की तरह ही लागू होते हैं, परन्तु संसद का अधिवेशन आरम्भ होते ही इन्हें संसद के सामने रखा जाना आवश्यक है। ये अध्यादेश संसद का अधिवेशन आरम्भ होने की तिथि से 6 सप्ताह तक लागू रह सकते हैं। संसद इसे पहले भी अस्वीकार कर सकती है। संसद के द्वारा स्वीकृत होने पर यह कानून का रूप ले लेता है।

3. वित्तीय शक्तियाँ (Financial Powers) राष्ट्रपति की वित्तीय शक्तियाँ निम्नलिखित हैं
(1) बजट (Budget)-संसद में बजट राष्ट्रपति के नाम पर ही पेश किया जाता है, जिसमें संघ सरकार की आय-व्यय का ब्यौर होता है।
(2) धन विधेयक (Money Bill)-संसद में कोई भी धन विधेयक राष्ट्रपति की पूर्व अनुमति के बिना पेश नहीं किया जा सकता।
(3) आकस्मिक निधि (Contingency Fund) भारत की आकस्मिक निधि राष्ट्रपति के अधीन है और उसमें से धन खर्च करने का अधिकार राष्ट्रपति को है।
(4) वित्त आयोग की नियुक्ति (Appointment of Finance Commission) हर पाँच वर्ष के पश्चात् राष्ट्रपति को वित्त आयोग नियुक्त करने का अधिकार है जो वित्तीय मामलों पर अपनी सिफारिशें देता है।

4. न्यायिक शक्तियाँ (Judicial Powers):
राष्ट्रपति को बहुत-सी न्यायिक शक्तियाँ प्रदान की गई हैं जिनमें से कुछ प्रमुख न्यायिक शक्तियाँ निम्नलिखित हैं

(1) (न्यायाधीशों की नियुक्ति (Appointment of Judges):
राष्ट्रपति को उच्चतम न्यायालय तथा उच्च न्यायालयों के मुख्य न्यायाधीशों तथा अन्य न्यायाधीशों की नियुक्ति करने का अधिकार है। वह इसका प्रयोग मन्त्रिमण्डल की सलाह से ही करता है। .

(2) क्षमादान का अधिकार (Power of Pardon):
अनुच्छेद 72 के अधीन राष्ट्रपति किसी व्यक्ति को जिसे न्यायालय द्वारा दण्डित किया गया हो, क्षमादान दे सकता है। उसके दण्ड के स्वरूप को बदल सकता है, दण्ड को कुछ समय के लिए लागू होने से रोक सकता है।

(3) उच्चतम न्यायालय से सलाह लेना (To seek advice from the Supreme Court):
राष्ट्रपति को यह अधिकार है कि वह किसी मामले पर उच्चतम न्यायालय की सलाह ले सकता है और उच्चतम न्यायालय को अपना परामर्श देना पड़ता है, परन्तु इसके अनुसार चलने के लिए वह बाध्य नहीं है।

4) विशेषाधिकार (Privileges):
राष्ट्रपति को कुछ न्यायिक विशेषाधिकार भी प्राप्त हैं जैसे कि राष्ट्रपति अपने अधिकार और शक्तियों के प्रयोग के सम्बन्ध में किसी भी न्यायालय के सामने उत्तरदायी नहीं है। कोई न्यायालय उसके विरुद्ध उसके कार्यकाल में कोई फौजदारी कार्रवाई नहीं कर सकता और उसे बन्दी नहीं बनाया जा सकता। राष्ट्रपति के विरुद्ध दीवानी मुकद्दमा चलाने के लिए भी उसे कम-से-कम दो महीने की पूर्व सूचना दी जानी आवश्यक है। .

(ख) आपातकालीन शक्तियाँ (Emergency Powers)-संविधान के अनुच्छेद 352 के अन्तर्गत युद्ध, विदेशी आक्रमण अथवा सशस्त्र विद्रोह से उत्पन्न संकट का सामना करने के लिए राष्ट्रपति मन्त्रिमण्डल के लिखित परामर्श से राष्ट्रीय आपात्काल की घोषणा कर सकता है। अनुच्छेद 356 के अन्तर्गत राज्य में संवैधानिक तन्त्र विफल हो जाने पर राष्ट्रपति उस राज्य में आपातकाल लागू कर सकता है। अनुच्छेद 360 के अन्तर्गत राष्ट्रपति वित्तीय आपास्थिति की घोषणा मन्त्रिमण्डल के परामर्श से कर सकता है।

निष्कर्ष (Conclusion)-इस प्रकार हम देखते हैं कि भारतीय संविधान में राष्ट्रपति को व्यापक शक्तियाँ प्राप्त हैं, लेकिन संसदात्मक व्यवस्था होने के नाते राष्ट्रपति की इन शक्तियों का उपयोग प्रधानमन्त्री सहित मन्त्रिमण्डल करता है, लेकिन राष्ट्रपति देश का प्रथम नागरिक होने के कारण विशेष गरिमा रखता है।

प्रश्न 6.
भारत के राष्ट्रपति की संकटकालीन शक्तियों का वर्णन करो।
अथवा
राष्ट्रपति की आपात्कालीन शक्तियों का विवरण दीजिए।
अथवा
भारत के राष्ट्रपति की आपात्कालीन शक्तियों का आलोचनात्मक निरीक्षण कीजिए।
उत्तर:
संकटकालीन शक्तियाँ (Emergency Powers)-संकटकाल में प्रशासन संबंधी सारी शक्तियाँ राष्ट्रपति के हाथों में केन्द्रित हो जाती हैं। ये शक्तियाँ बहुत अधिक और व्यापक हैं। इन शक्तियों का प्रयोग भी अन्य शक्तियों की भांति वह प्रधानमंत्री और मंत्रिमंडल की सलाह से करता है। संकट तीन प्रकार का माना गया है-
(क) युद्ध, बाह्य आक्रमण या देश में सशस्त्र विद्रोह से पैदा हुआ संकट,
(ख) किसी राज्य में सवैधानिक व्यवस्था के विफल होने से उत्पन्न संकट,
(ग) वित्तीय स्थिति के बिगड़ने से पैदा हुआ संकट।

बाहरी आक्रमण, सशस्त्र विद्रोह या आन्तरिक अशान्ति से उत्पन्न संकट (EmergencyArising Out of War, External Invasion, Armed Rebellion or Internal Disturbance) यदि राष्ट्रपति को इस बात का विश्वास हो जाए कि भारत या उसके किसी भाग की सुरक्षा युद्ध, विदेशी आक्रमण अथवा आन्तरिक सशस्त्र विद्रोह के कारण खतरे में है तो वह संविधान के अनुच्छेद 352 के अन्तर्गत आपात्कालीन घोषणा कर सकता है।

सन् 1978 में पारित 44वें संशोधन के पश्चात् अनुच्छेद 352 में कहा गया है कि इस प्रकार की घोषणा मंत्रि-परिषद् द्वारा लिखित सिफारिश पर ही की जाएगी। राष्ट्रपति इस लिखित सिफारिश को दोबारा विचार करने के लिए मंत्रि-परिषद् के पास वापस भेज सकता है। अन्त में, मंत्रि-परिषद् की बात को राष्ट्रपति द्वारा स्वीकार करना पड़ता है।

संसद का अनुमोदन (Approval of the Parliament)-राष्ट्रपति की इस उद्घोषणा को संसद के दोनों सदनों द्वारा 30 दिन के अन्दर स्वीकृति प्रदान कर दी जानी चाहिए। यदि लोकसभा का अधिवेशन नहीं हो रहा है तो राज्यसभा स्वीकृति प्रदान कर सकती है। जैसे ही लोकसभा का अधिवेशर आरम्भ होता है तो 30 दिन के अन्दर राष्ट्रपति की घोषणा को स्वीकृति प्रदान की जा सकती है। एक समय में यह घोषणा 6 महीने तक लागू हो सकती है। यदि संसद 6 महीने के पश्चात् स्वीकृति प्रदान नहीं करती तो यह घोषणा समाप्त हो जाएगी।

44वें संशोधन द्वारा यह व्यवस्था भी की गई है कि यदि लोकसभा के 1/10 सदस्य लिखित रूप में घोषणा को समाप्त करने का नोटिस देते हैं तो इस पर विचार करने के लिए 14 दिन के अन्दर लोकसभा का अधिवेशन बुलाया जाना अनिवार्य है और यदि इस अधिवेशन में लोकसभा के उपस्थित एवं मतदान देने वाले सदस्यों के साधारण बहुमत से प्रस्ताव पारित कर दिया जाता है तो ऐसी उद्घोषणा की समाप्ति राष्ट्रपति द्वारा आदेश जारी करके कर दी जाती है।

उद्घोषणा का प्रभाव (Effects of the Proclamation)-राष्ट्रपति की इस आपात्कालीन घोषणा का निम्नलिखित प्रकार से बहुत व्यापक प्रभाव होता है-
(1) इस घोषणा से सम्पूर्ण देश का शासन राष्ट्रपति के हाथों में आ जाता है,

(2) राज्य-सूची में दिए गए विषयों पर संसद को कानून बनाने का अधिकार मिल जाता है,

(3) संघ सरकार किसी भी राज्य को आदेश दे सकती है कि वह अपनी कार्यपालिका शक्ति का प्रयोग किस प्रकार करे और राज्यपाल राष्ट्रपति की आज्ञानुसार कार्य करे,

(4) संविधान के 19वें अनुच्छेद द्वारा नागरिकों को दिए गए मौलिक अधिकार-विचार और भाषण की स्वतन्त्रता, शान्तिपूर्वक एकत्रित होने का अधिकार, संघ या समुदाय बनाने की स्वतन्त्रता, देश में कहीं भी निवास का अधिकार, आने-जाने की स्वतन्त्रता का अधिकार तथा आजीविका चलाने या व्यापार के अधिकार आदि को स्थगित किया जा सकता है,

(5) राष्ट्रपति को अधिकार होगा कि वह किसी अन्य मौलिक अधिकार को अमल में लाने के लिए नागरिकों को न्यायालय में जाने से रोक सकता है,

(6) वह संघ तथा राज्यों के बीच राजस्व के बँटवारे में इच्छानुसार परिवर्तन कर सकता है,

(7) इन सबका परिणाम यह होता है कि राज्यों की आन्तरिक स्वायत्तता नष्ट हो जाती है और केन्द्रीय सरकार का राज्यों की सरकारों पर नियन्त्रण स्थापित हो जाता है। देश में संघात्मक सरकार की जगह एकात्मक सरकार स्थापित हो जाती है।

युद्ध संबंधी आपात्कालीन घोषणा पहली बार चीन के आक्रमण के कारण 26 अक्तूबर, 1962 को राष्ट्रपति द्वारा लागू की गई थी। पाकिस्तान के सन् 1965 में आक्रमण के कारण यह घोषणा समाप्त नहीं की जा सकी। सन् 1967 के चुनाव के पश्चात् 10 जनवरी, 1968 को यह घोषणा समाप्त की गई। दिसम्बर, 1971 में पाकिस्तान के साथ युद्ध के कारण राष्ट्रपति को दोबारा राष्ट्रीय आपात्काल की घोषणा करनी पड़ी।

अभी बाह्य संकट की घोषणा समाप्त भी नहीं हुई थी कि जून, 1975 में आन्तरिक आपातकाल की घोषणा कर दी गई। सन् 1977 में लोकसभा के चुनावों में जनता पार्टी की सफलता के पश्चात दोनों तरह की आपात्कालीन स्थितियों को 21 मार्च, 1977 को समाप्त कर दिया गया।

(ख) राज्यों में संवैधानिक सरकार की विफलता (Failure of Constitutional Machinery in the States)-जब राष्ट्रपति को राज्यपाल द्वारा दी गई सूचना से अथवा किसी और सूत्र से यह विश्वास हो जाए कि राज्य का शासन संविधान के अनुसार नहीं चलाया जा रहा है अथवा नहीं चलाया जा सकता, तो राष्ट्रपति संविधान के अनुच्छेद 356 के अनुसार उस राज्य में संवैधानिक आपात स्थिति की घोषणा करके राष्ट्रपति शासन लागू कर सकता है।

संसद से स्वीकृति (Approval from Parliament)-राष्ट्रपति की इस घोषणा को संसद के दोनों सदनों द्वारा दो मास के अन्दर स्वीकृति प्रदान की जानी चाहिए अन्यथा दो मास के पश्चात् यह समाप्त हो जाएगी। 44वें संशोधन में यह व्यवस्था की गई है कि ऐसी घोषणा 6-6 मास करके एक वर्ष के लिए लागू की जा सकती है।

विशेष परिस्थितियों में 3 वर्ष तक इस घोषणा की अवधि को बढ़ाया जा सकता है। इसमें चुनाव आयोग द्वारा चुनाव न करा पाने की स्थिति भी शामिल है। पंजाब में इसी कारण से राष्ट्रपति शासन लगभग पाँच वर्ष तक चलता रहा जिसके लिए संविधान में सन् 1990 और 1991 में क्रमशः 67वाँ एवं 68वाँ संवैधानिक संशोधन करना पड़ा। अब तक भारतीय संघ के विभिन्न राज्यों में लगभग 130 से भी अधिक बार सवैधानिक आपात् स्थिति के अन्तर्गत राष्ट्रपति शासन लागू किया जा चुका है।

घोषणा के प्रभाव (Effects of the Proclamation)

(1) राष्ट्रपति सम्बन्धित राज्य के प्रशासन को पूर्ण या आंशिक रूप से अपने हाथ में ले लेता है। राज्य का मंत्रिमंडल भंग हो जाता है और राज्य के प्रशासनिक कार्यों को वह राज्यपाल या अन्य किसी
(2) राज्य के विधानमंडल को भंग करके या स्थगित करके उसकी शक्तियाँ संसद को दे सकता है,
(3) इस उद्घोषणा से राज्य अथवा राज्यों की आन्तरिक स्वायत्तता समाप्त हो जाती है,
(4) राष्ट्रपति संसद को अथवा अन्य किसी अधिकारी को उस राज्य के सम्बन्ध में कानून बनाने की शक्ति सौंप सकता है।

(ग) वित्तीय आपात स्थिति (Financial Emergency) यदि राष्ट्रपति को यह विश्वास हो जाए कि देश की आर्थिक स्थिति अथवा साख को खतरा है तो संविधान के अनुच्छेद 360 के अन्तर्गत वह वित्तीय आपात स्थिति की घोषणा कर सकता है। यह घोषणा भी दो महीने के लिए होती है और इस अवधि में संसद की स्वीकृति लेनी पड़ती है। संसद की स्वीकृति मिल जाने पर यह घोषणा तब तक लागू रहेगी जब तक कि राष्ट्रपति इसे दूसरी घोषणा द्वारा समाप्त नहीं कर देता।

उद्घोषणा का प्रभाव (Effects of the Proclamation)

(1) वित्तीय आपात स्थिति की घोषणा के समय राष्ट्रपति राज्यों और संघ के सभी कर्मचारियों के, जिनमें सर्वोच्च न्यायालय तथा उच्च न्यायालय के न्यायाधीश भी शामिल हैं, वेतन तथा भत्ते कम कर सकता है,

(2) राज्यों के धन-विधेयकों को अपनी स्वीकृति के लिए मंगवा सकता है,

(3) केन्द्र तथा राज्यों की साख को दोबारा स्थापित करने के लिए अन्य आवश्यक कदम उठा सकता है,

(4) राष्ट्रपति राज्यों के प्रति ऐसे कदम उठा सकता है जो उसकी दृष्टि में वित्तीय साख बनाए रखने के लिए आवश्यक हों। अभी तक भारत में वित्तीय आपात की घोषणा करने का अवसर नहीं आया है।

राष्ट्रपति की संकटकालीन शक्तियों का मूल्यांकन (Evaluation of the Emergency Powers of the President)- राष्ट्रपति की उपर्युक्त संकटकालीन शक्तियों को देखकर प्रायः यह विचार प्रकट किया जाता है कि इन शक्तियों के दुरुपयोग से देश में लोकतन्त्र समाप्त किया जा सकता है। इस विचारधारा का समर्थन करते हुए संविधान सभा के अनेक सदस्यों ने संविधान के अनुच्छेदों की कड़ी आलोचना की थी।

के०टी० शाह (K.T. Shah) ने राष्ट्रपति के इन अधिकारों को संविधान में “सबसे अधिक प्रतिक्रियात्मक अध्याय का आखिरी एवं शानदार आभूषण कहा है।” इसी प्रकार श्री एच०वी० कामथ (Sh. H.V. Kamath) ने कहा है, “विश्व के लोकतांत्रिक देशों के किसी भी संविधान में इस संकटकालीन अध्याय के समान अन्य कहीं कोई अध्याय नहीं मिलता।”

इन शक्तियों की संविधान सभा में आलोचना करते हुए उन्होंने कहा था, “मुझे डर है कि इस अकेले अध्याय से हम तानाशाही राज्य, पुलिस-राज्य अथवा एक ऐसे राज्य की स्थापना करने जा रहे हैं जो उन सब आदर्शों और सिद्धान्तों के विपरीत होगा जिन्हें हम पिछले कई वर्षों से समक्ष रखे हुए हैं, ऐसा राज्य जहाँ पर लाखों मासूम पुरुषों तथा स्त्रियों के अधिकार तथा स्वतन्त्रता पर निरन्तर आघात होगा; एक ऐसा राज्य जहाँ यदि शांति होगी तो यह शांति केवल श्मशान एवं मरुस्थल की शांति होगी।”

राष्ट्रपति की संकटकालीन शक्तियों की निम्नलिखित बातों के आधार पर आलोचना की गई है

1. मौलिक अधिकार अर्थहीन हो जाएंगे (Fundamental rights will become Meaningless):
राष्ट्रपति की संकटकालीन शक्तियों की सबसे बड़ी आलोचना यह है कि ये मौलिक अधिकारों को अर्थहीन बना देती हैं। इन शक्तियों के द्वारा राष्ट्रपति छेद 19 में वर्णित सातों स्वतन्त्रताओं और अनच्छेद 32 को भी निलंबित कर सकता है। राष्ट्रपति की इन शक्तियों को किसी न्यायालय में भी चुनौती नहीं दी जा सकती। अतः सत्तालोभी व्यक्ति इन शक्तियों का प्रयोग तानाशाह बनने के लिए कर सकता है।

2. राज्यों की आर्थिक स्वतन्त्रता समाप्त हो जाएगी (The Financial Autonomy of the States will be Nullified):
संकटकालीन समय में राष्ट्रपति राज्य सरकारों को वित्तीय मामलों में विभिन्न प्रकार के आदेश दे सकता है, जिनसे राज्य की आर्थिक स्वतन्त्रता नष्ट हो जाती है। इसी सम्बन्ध में हृदयनाथ कुंजरू (H.N. Kunzru) ने कहा था, “इनसे राज्य की वित्तीय स्वतन्त्रताओं को बड़ा धक्का पहुंचेगा।”

3. राज्यों में विरोधी दलों की सरकार का दमन हो सकता है (Opposition Governments in States can be Suppressed):
विरोधी दलों को प्रारम्भ से ही यह लगता था कि सत्ताधारी दल के द्वारा राज्यों में विरोधी दल की सरकार को दबाने के लिए राष्ट्रपति इन शक्तियों का प्रयोग कर सकता है। वह राज्य सरकार को पदच्युत करने की धमकी देकर उसे डरा सकता है या उसे पदच्युत भी कर सकता है। राष्ट्रपति ने इस शक्ति के प्रयोग द्वारा 1969 ई० में केरल की साम्यवादी दल की सरकार को भंग किया था।

4. न्यायपालिका के अधिकारों को सीमित किए जाने की व्यवस्था खतरनाक है (Provision for limitation of Judicial Power is Dangerous):
राष्ट्रपति द्वारा की गई संकटकालीन घोषणा को राज्य के किसी भी न्यायालय में चुनौती नहीं दी जा सकती। अगर राष्ट्रीय हित में संकटकाल सम्बन्धी अनुच्छेदों को उचित मान भी लिया जाए तो भी न्यायपालिका के अधिकारों को कम किए जाने की व्यवस्था नहीं होनी चाहिए थी।

संविधान के 38वें संशोधन के अन्तर्गत यह व्यवस्था की गई थीं कि अनुच्छेद 352 तथा 356 के अधीन राष्ट्रपति द्वारा की गई। आपात्काल की घोषणाओं के औचित्य को किसी भी न्यायालय में चुनौती नहीं दी जा सकती, लेकिन अब 44वें संशोधन के अन्तर्गत राष्ट्रपति द्वारा संकटकाल की घोषणा को न्यायालय में चुनौती दी जा सकती है।

5. संघीय विरोधी (Anti-Federal):
यह भी बड़े आश्चर्य का विषय है कि संकटकाल की घोषणा के समय संघीय ढाँचा . एकात्मक सरकार में बदल जाए और इकाइयों की सरकारें समाप्त कर दी जाएँ। इसी कारण संविधान सभा में टी०टी० कृष्णमाचारी (T.T.Krishnamachari) ने कहा था, “भारतीय संविधान साधारण काल में संघात्मक तथा युद्ध एवं अन्य संकटकालीन परिस्थितियों में एकात्मक रूप धारण कर लेता है।”

6. न्यायपालिका की स्वतन्त्रता के लिए नकारात्मक सिद्धान्त (Negative Principle in respect of Independence of Judiciary):
वित्तीय संकटकाल की अवस्था में राष्ट्रपति न्यायाधीशों के वेतन को कम करके उनकी स्वतन्त्रता को हानि पहुँचा सकता है। उपर्युक्त आलोचनाओं से यह स्पष्ट हो जाता है कि संविधान निर्माता राष्ट्रीय सुरक्षा को नागरिकों की स्वतन्त्रताओं और अधिकारों से अधिक महत्त्वपूर्ण मानते थे, इसलिए उन्होंने राष्ट्रपति को अधिक शक्तियाँ प्रदान की।

शक्तियों का औचित्य (Justification of Emergency Powers)-इसमें तनिक भी सन्देह नहीं है कि राष्ट्रपति की संकटकालीन शक्तियाँ बहुत अधिक विस्तृत एवं विशाल हैं, लेकिन इसके बावजूद भी इन शक्तियों को संविधान में स्थान देना अनिवार्य समझा गया है। इसके आधार अग्रलिखित हैं

1. ऐतिहासिक अनुभव (Historical Experience):
भारत का प्राचीन इतिहास इस बात का साक्षी है कि जब कभी भी भारत की केन्द्र सरकार कमज़ोर पड़ गई, तभी हमें हानि उठानी पड़ी, इसलिए केन्द्र सरकार को शक्तिशाली बनाना आवश्यक है।

2. राष्ट्रीय सुरक्षा का भार केन्द्र सरकार पर है (National Security depends upon the Federal Government):
भारत में संघीय प्रणाली को अपनाया गया है, लेकिन देश में संघीय रूप का इतना महत्त्व नहीं है, जितना कि राष्ट्रीय सुरक्षा का है। देश की सुरक्षा के लिए व्यक्तिगत हित का त्याग कर देना चाहिए। डॉ० अम्बेडकर (Dr. Ambedkar) के शब्दों में, “केवल केन्द्र ही समस्त देश की एक समान भलाई के लिए कार्य कर सकता है। इसलिए केन्द्र को संकटकाल में राज्य सरकारों की शक्तियाँ ग्रहण करने का अधिकार देना कोई अन्याय नहीं है।”

3. महाभियोग (Impeachment):
यदि राष्ट्रपति एक निरंकुश तथा वास्तविक शासक बनने का प्रयास करेगा, तब संसद उसे महाभियोग का दोष लगाकर हटा भी सकती है।

4. राष्ट्रपति एक संवैधानिक मुखिया है (President a Constitutional Head):
भारत में इंग्लैण्ड की भाँति संसदीय प्रणाली अपनाई गई है तथा भारत का राष्ट्रपति एक संवैधानिक मुखिया है। राष्ट्रपति को प्रधानमन्त्री एवं मन्त्रिमण्डल की सलाह से ही शासन के कार्य का संचालन करना होता है। व्यावहारिक (Practical) रूप में ऐसा कोई अवसर पैदा हो ही नहीं सकता, जब राष्ट्रपति मन्त्रिमण्डल की सलाह के बिना देश के शासन का संचालन कर सके।

5. संसद की स्वीकृति (Approval of the Parliament):
जब भारत के राष्ट्रपति के द्वारा संकटकालीन घोषणा की जाती है, तो उसके बाद निश्चित समय में संसद की स्वीकृति प्राप्त करना अनिवार्य है। अगर संसद राष्ट्रपति की संकटकाल की घोषणा को स्वीकृति प्रदान न करे, तो संकटकाल की घोषणा उसी समय समाप्त हो जाती है। इसके अलावा संविधान के 44वें संशोधन द्वारा यह भी व्यवस्था की गई है कि लोकसभा एक प्रस्ताव पास करके किसी भी समय आपात्कालीन घोषणा को समाप्त कर सकती है।

6. मौलिक अधिकारों से देश की सुरक्षा अधिक महत्त्वपूर्ण (Security of the Country is more important than ntal Rights):
जब हमारे देश में संकटकाल की घोषणा लागू होती है, तब संविधान के 19वें अनुच्छेद द्वारा दी गई स्वतन्त्रताएँ भंग की जा सकती हैं, लेकिन मौलिक अधिकारों के स्थगन का यह अभिप्राय नहीं है कि संकटकाल की घोषणा से मौलिक अधिकार स्वयंमेव रद्द हो जाते हैं। विपत्ति के समय मौलिक अधिकारों की अपेक्षा देश की सुरक्षा अधिक महत्त्वपूर्ण है।

7. वित्तीय संकटकालीन शक्तियों का होना आवश्यक है (Financial Powers are also very Essential):
संविधान में … वित्तीय संकट का प्रावधान करना भी उचित है। जब संविधान का निर्माण किया जा रहा था, उस समय भारत की आर्थिक स्थिति काफी खराब थी, तब हमारे संविधान बनाने वालों ने दूरदर्शिता का परिचय दिया।

क्या संकटकाल में राष्ट्रपति तानाशाह बन सकता है? (Can the President become a Dictator during Emergency?)-उपर्युक्त संकटकालीन शक्तियों के अध्ययन से यह मालूम पड़ता है कि भारतीय राष्ट्रपति संकटकाल में तानाशाह बन सकता है। वह सीज़र, जार या फ्यूरर का स्थान ले सकता है। संकटकालीन शक्तियों के उपबन्ध राष्ट्रपति को इतना शक्तिशाली बना देते हैं कि वह केन्द्रीय और प्रान्तीय सरकारों को अपने अधीन कर लेता है। भारतीय राष्ट्रपति संकटकाल की घोषणा केवल वास्तविक संकट आने पर ही नहीं करता, बल्कि संकटकाल की आशंका मात्र पर भी ऐसा कर सकता है। ऐसी स्थिति में राष्ट्रपति का मात्र संतुष्ट होना ही काफी है और वह मन्त्रिमण्डल की सलाह भी नहीं लेता।

वह लोकसभा को भंग करके 6-8 महीने तक मनमाने ढंग से शासन कर सकता है। वह नागरिकों के अधिकारों व स्वतन्त्रताओं का हनन कर सकता है। कार्यपालिका व विधानपालिका क्षेत्र में उसकी शक्तियाँ असीमित हो जाती हैं। केन्द्र और राज्यों की वित्तीय व्यवस्था पर उसका नियंत्रण काफी मात्रा में बढ़ जाता है।

इस प्रकार आलोचकों ने भारतीय राष्ट्रपति की आपातकालीन शक्तियों की तुलना जर्मनी के वाईमर संविधान से की है जिसके आधार पर हिटलर तानाशाह बन गया था। कुछ लोगों का कहना यह है कि राष्ट्रपति केन्द्रीय मन्त्रिमण्डल से गुप्त मंत्रणा (Confidential Meeting) करके लम्बी अवधि तक संकटकालीन समय को जारी रख सकता है और हर बार लोकसभा को भंग करके तानाशाह बन सकता है। इस प्रकार यह ठीक है कि इन्हीं संकटकालीन शक्तियों के आधार पर ही यह आशंका व्यक्त की गई है कि वह एक तानाशाह बन सकता है।

यह आशंका उचित नहीं है, यह तो मात्र बॉल की खाल उतारने वाली बात है। भारत में संसदीय प्रणाली की व्यवस्था की गई है। कोई भी राष्ट्रपति तानाशाह बनने की सोचेगा ही नहीं। इस प्रकार की कल्पना करना ही गलत है कि राष्ट्रपति तानाशाह बनेगा। यही नहीं, कोई भी प्रधानमन्त्री या मन्त्रिमण्डल इन उपबन्धों के आधार पर राष्ट्रपति को तानाशाह बनने में सहायता नहीं करेगा। राष्ट्रपति बिना संसद की सहायता के एक दिन भी प्रशासन नहीं चला सकता।

निष्कर्ष (Conclusion)-अतः यह कहा जा सकता है कि संकटकालीन शक्तियों के बारे में जो सन्देह प्रकट किए गए हैं, वे निराधार हैं। यह बिल्कुल सत्य है कि भारत के राष्ट्रपति को जितनी संकटकालीन शक्तियाँ प्रदान की गई हैं उतनी किसी भी प्रजातान्त्रिक देश के अध्यक्ष को प्रदान नहीं की गई हैं। इससे इन्कार नहीं किया जा सकता कि इन शक्तियों से अधिकारों एवं स्वतन्त्रताओं का हनन होता है। इनसे प्रजातन्त्र की आधारशिला को धक्का पहुँचता है। इससे राज्यों की स्वतन्त्रता को भी आघात पहुँचता है, लेकिन यह नहीं भूलना चाहिए कि देश रहेगा तो सब-कुछ रहेगा। अधिकार और स्वतन्त्रता की तुलना में राष्ट्रीय सुरक्षा अधिक महत्त्वपूर्ण है। किसी भी कीमत पर राष्ट्रीय एकता और अस्तित्व को बनाए रखना है।

ध्यान रहे इन आपात्कालीन शक्तियों का होना जरूरी है, लेकिन उतना ही आवश्यक है इन शक्तियों पर अंकुश लगाना, इसलिए 44वें संशोधन द्वारा अनुच्छेद 352 पर अंकुश लगाया है और उसके दुरुपयोग की सम्भावना तो समाप्त हो गई है, परन्तु अनुच्छेद 356 के दुरुपयोग को रोकने के लिए कोई-न-कोई प्रबन्ध करना पड़ेगा।

अगर ऐसा न किया गया तो किसी भी दल की केन्द्रीय सरकार अपने राजनीतिक लोगों के लिए और विपक्षी दलों को आघात पहुँचाने के लिए सक्रिय हो सकती है। अनुच्छेद 356 के दुरुपयोग को रोकने के लिए सरकारिया आयोग ने सुझाव दिए हैं कि इस अनुच्छेद का कम-से-कम प्रयोग हो। राज्य सरकारों को पहले चेतावनी देनी चाहिए। राज्यपाल की रिपोर्ट संसद के दोनों सदनों में रखी जानी चाहिए। वास्तविकता यह है कि यदि विरोधी दल अपनी भूमिका के प्रति सजग हैं और जनता में अपने राजनीतिक कर्तव्यों के प्रति जागरूक हैं तो इन संकटकालीन उपबन्धों का गलत प्रयोग हो ही नहीं सकता।

HBSE 11th Class Political Science Important Questions Chapter 4 कार्यपालिका

प्रश्न 7.
भारत के राष्ट्रपति की वास्तविक स्थिति का विश्लेषण कीजिए।
उत्तर:
भारतीय संविधान गणराज्यीय व्यवस्था स्थापित करता है, जिसमें राष्ट्रपति को व्यापक रूप से शक्तियाँ प्रदान की गई हैं। उसे संविधान के रक्षक की भूमिका दी गई है, वह राज्य का अध्यक्ष है और समस्त कार्यपालिका सम्बन्धी शक्तियाँ उसके अधीन हैं। संसद से अभिप्राय राष्ट्रपति सहित संसद है लेकिन भारतीय संविधान गणराज्य के साथ-साथ संसदात्मक व्यवस्था भी स्थापित करता है, जिसमें राज्याध्यक्ष तो नाममात्र का अध्यक्ष होता है, वास्तविक शक्तियाँ तो प्रधानमन्त्री सहित मन्त्रिमण्डल में पाई जाती हैं।

भारतीय राष्ट्रपति की वास्तविक स्थिति क्या है? यह प्रारम्भ से ही विवाद का विषय रहा है। संविधान सभा में इसके अध्यक्ष डॉ० राजेन्द्र प्रसाद ने आशा व्यक्त की थी कि भारतीय राष्ट्रपति संसदात्मक पद्धति वाले देशों की भाँति राज्याध्यक्ष की भूमिका निभाएगा। संविधान के अनुच्छेद 74 में कहा गया कि राष्ट्रपति को सहायता और सलाह देने के लिए एक मन्त्रि-परिषद् होगी जिसका प्रधान प्रधानमन्त्री होगा, लेकिन

इसमें स्पष्ट रूप से नहीं कहा गया था कि राष्ट्रपति मन्त्रिमण्डल की सलाह को मानने के लिए बाध्य होगा। डॉ० राजेन्द्र प्रसाद ने 1959 में राष्ट्रपति के रूप में विधि-विशेषज्ञों से कहा था कि वे देखें कि क्या भारतीय संविधान में राष्ट्रपति मन्त्रिमण्डल की सलाह मानने के लिए बाध्य है अथवा किन्हीं क्षेत्रों में स्व-विवेकी अधिकार भी रखता है?

इस विवाद को समाप्त करने तथा राष्ट्रपति तथा मन्त्रिमण्डल के बीच सम्बन्धों को पूर्णतया विवाद रहित बनाने के उद्देश्य से 42वें संविधान संशोधन 1976 के द्वारा अनुच्छेद 74 में संशोधन किया गया जिसमें कहा गया कि राष्ट्रपति मन्त्रिमण्डल द्वारा दिए गए परामर्श के अनुसार कार्य करेगा, लेकिन जनता पार्टी की सरकार ने 1978 में 44वें संविधान संशोधन द्वारा अनुच्छेद 74 को ज्यों-का-त्यों रखते हुए राष्ट्रपति को अधिकार दिया कि राष्ट्रपति मन्त्रिमण्डल को दी गई सलाह पर पुनर्विचार करने के लिए कह सकता है और राष्ट्रपति पुनर्विचार करने के बाद दी गई सलाह के अनुसार कार्य करेगा।

इस प्रकार 42वें व 44वें संविधान संशोधनों के बाद आशा थी कि राष्ट्रपति की वास्तविक स्थिति के बारे में विवाद समाप्त हो जाएगा, लेकिन राष्ट्रपति ज्ञानी जैलसिंह और प्रधानमन्त्री श्री राजीव गाँधी के मध्य रिश्तों ने इस विवाद को समाप्त नहीं होने दिया। सवैधानिक व व्यावहारिक राजनीति में अभी भी ऐसे अनेक मामले हैं जिनमें राष्ट्रपति स्व-विवेक का प्रयोग कर सकता है अथवा अपने प्रभाव का सफलतापूर्वक प्रयोग कर सकता है। राष्ट्रपति के स्व-विवेकी अधिकार (Discreationary Powers of the President)-राष्ट्रपति के स्व-विवेकी अधिकार निम्नलिखित हैं

1. लोकसभा में अस्पष्टता की स्थिति में प्रधानमन्त्री की नियुक्ति (Appointment of the Prime Minister in case of Lack of a Majority of any Party in Lok Sabha):
जब लोकसभा में किसी भी दल को स्पष्ट बहुमत नहीं मिलता है तो राष्ट्रपति अपने स्वविवेक का प्रयोग कर सकता है, जैसे 1989 में जब किसी भी दल को स्पष्ट बहुमत नहीं मिला था तो पहले सबसे बड़े दल के नेता होने के नाते राष्ट्रपति श्री वेंकटरमन ने श्री राजीव गाँधी को सरकार बनाने के लिए कहा। उनके मना करने पर वी०पी० सिंह को आमन्त्रित किया।

इसी प्रकार 1991 के चुनावों में भी श्री नरसिम्हा राव को सबसे बड़े दल का नेता होने के नाते ही प्रधानमन्त्री नियुक्त किया गया था लेकिन दोनों मामलों में राष्ट्रपति ने 30 दिन के भीतर लोकसभा में बहुमत सिद्ध करने के लिए कहा था। 1984 में श्रीमती इन्दिरा गाँधी की हत्या के बाद राष्ट्रपति ज्ञानी जैलसिंह ने श्री राजीव गाँधी को अपने स्व-विवेक के आधार पर ही प्रधानमन्त्री नियुक्त किया था। मार्च 1998 में हुए लोकसभा चुनावों के पश्चात् भी भूतपूर्व प्रधानमन्त्री अटल बिहारी वाजपेयी की नियुक्ति इसी प्रकार की गई।

2. लोकसभा को भंग करना (Dissolution of Lok Sabha):
सामान्यतया राष्ट्रपति प्रधानमन्त्री की सलाह पर लोकसभा को भंग करता है और इस मामले में राष्ट्रपति को कोई स्व-विवेकी अधिकार प्राप्त नहीं है, लेकिन जब ऐसा प्रधानमन्त्री लोकसभा भंग करने की सलाह देता है तो आवश्यक नहीं कि राष्ट्रपति उसकी सलाह को माने। हालांकि 1979 में चौधरी चरणसिंह तथा 1991 में चन्द्रशेखर की सलाह पर लोकसभा भंग तो की गई थी, लेकिन श्री नीलम संजीवा रेड्डी ने बाद में स्पष्ट कर दिया था कि उन्होंने चौधरी चरणसिंह की सलाह पर नहीं, बल्कि अपने स्व-विवेक का प्रयोग करते हुए ऐसा किया था।

इसी प्रकार से 1991 में राष्ट्रपति श्री आर० वेंकटरमन ने लोकसभा इसलिए भंग की थी, क्योंकि अन्य कोई दल सरकार बनाने की स्थिति में नहीं था। 2014 में प्रधानमन्त्री डॉ० मनमोहन सिंह के परामर्श पर राष्ट्रपति प्रणब मुखर्जी ने लोकसभा को भंग करके 16वीं लोकसभा के चुनाव घोषित किए थे।

3. विधेयक पर हस्ताक्षर करना अथवा पुनर्विचार करने के लिए वापस भेजना (To Assent the Bill or Return it for Re-consideration):
राष्ट्रपति संसद द्वारा पारित किसी भी विधेयक को पुनर्विचार करने के लिए संसद के पास वापस भेज सकता है। इस मामले में राष्ट्रपति मन्त्रिमण्डल की सलाह को मानने के लिए बाध्य नहीं है। इसके अतिरिक्त अनुच्छेद 111 में यद्यपि राष्ट्रपति को वीटो शक्ति प्राप्त नहीं है, लेकिन संसद अथवा मन्त्रिमण्डल किसी विधेयक पर एक निश्चित समयावधि में हस्ताक्षर करने के लिए बाध्य नहीं कर सकते।

यह पूर्णतया राष्ट्रपति के स्व-विवेक पर निर्भर करता है कि वो कब विधेयक को स्वीकृति दे। राष्ट्रपति ज्ञानी जैलसिंह तथा आर० वेंकटरमन ने 1986 में पारित पोस्टल संशोधन विधेयक पर हस्ताक्षर नहीं किए थे। इस विवाद का अन्त तभी हुआ कि जब चन्द्रशेखर सरकार के समय संसद ने इस विधेयक को वापस ले लिया।

4. उच्चतम न्यायालय से सलाह माँगना (To Seek Advice from Supreme Court):
राष्ट्रपति संविधान के तहत कानून से सम्बन्धित किसी भी विषय पर परामर्श माँग सकता है। सामान्यतया राष्ट्रपति मन्त्रिमण्डल की सलाह के अनुसार ही मामलों को उच्चतम न्यायालय के पास सलाह के लिए भेजता है, लेकिन अनेकों विधि-विशेषज्ञों का मानना है कि इस अधिकार का प्रयोग राष्ट्रपति अपनी इच्छानुसार कर सकता है और इस बारे में उसे मन्त्रिमण्डल की सलाह मानने की कोई आवश्यकता नहीं है।

5. महान्यायाधिवक्ता से परामर्श (To Seek Advice FromAttroney General):
संविधान व कानून से सम्बन्धित किसी भी प्रश्न पर राष्ट्रपति महान्यायाधिवक्ता से सलाह माँग सकता है। इस बारे में राष्ट्रपति को मन्त्रिमण्डल से सलाह करने की कोई आवश्यकता नहीं है। जैसे श्री आर० वेंकटरमन ने पोस्टल संशोधन विधेयक 1986 पर महान्यायाधिवक्ता से परामर्श करने के बाद ही हस्ताक्षर नहीं किए थे। इन स्व-विवेकी अधिकार क्षेत्रों के अतिरिक्त गणराज्य का अध्यक्ष व राष्ट्र का प्रथम नागरिक होने के नाते भी वह निम्नलिखित विशेषाधिकारों व अधिकारों का प्रयोग स्वतन्त्रतापूर्वक करता है

1. सलाह देना (To Advise):
राष्ट्रपति प्रधानमन्त्री को विभिन्न मामलों पर सलाह देता है। सामान्यतया प्रधानमन्त्री राष्ट्रपति की सलाह की अनदेखी नहीं करता है, क्योंकि राष्ट्रपति राजनीतिक दलबन्दी से ऊपर उठकर सलाह देता है।

2. प्रशासन पर नियन्त्रण (To Encourage):
राष्ट्रपति प्रधानमन्त्री सहित मन्त्रिमण्डल को प्रोत्साहित करता है। जैसे नेहरू जी अक्सर श्री राधाकृष्णन से परामर्श करते थे। उनका मार्गदर्शन लेते थे। फलस्वरूप डॉ० राधाकृष्णन उन्हें (नेहरू जी को प्रोत्साहित किया करते थे।

3. चेतावनी देना (To Warm):
यदि प्रधानमन्त्री राष्ट्रपति की सलाह की अनदेखी करता है तो राष्ट्रपति प्रधानमन्त्री को चेतावनी दे सकता है और इसके गम्भीर परिणामों की ओर संकेत कर सकता है, जैसे ज्ञानी जैलसिंह ने राजीव गाँधी को चेतावनी देते हुए 1986 व 1987 में बाकायदा पत्र तक लिख डाले थे, लेकिन चेतावनी का यह अर्थ कदापि नहीं कि वह प्रधानमन्त्री को हटाने की धमकी दे सकता है।

यही नहीं, वह सरकार को भी उसके कार्यों के लिए चेतावनी दे सकता है। जैसे वर्ष 2000 के प्रारम्भ में श्री अटल बिहारी वाजपेयी की सरकार ने संविधान पर पुनः विचार (Review)करने के लिए एक आयोग का गठन करने की घोषणा की। तत्कालीन राष्ट्रपति श्री के०आर० नारायणन ने संविधान की 50वीं वर्षगाँठ के उपलक्ष्य में बोलते हुए कहा कि सरकार को संविधान की आधारभूत बनावट (Basic Structure) में किसी प्रकार का परिवर्तन नहीं करना चाहिए।

4. सूचना पाने का अधिकार (To Get Information):
संविधान में प्रधानमन्त्री से अपेक्षा की गई है कि वह समय-समय पर राष्ट्रपति को प्रशासन के बारे में सूचित करता रहे। नेहरू जी 15 दिनों के पश्चात् राष्ट्रपति को सूचनाएँ देते और सलाह करते थे। इसलिए माना जाता है कि राष्ट्रपति को प्रधानमन्त्री व मन्त्रिमण्डल से सूचना पाने का अधिकार है, लेकिन ज्ञानी जैलसिंह के बार-बार कहने तथा लिखकर देने के बावजूद ठक्कर आयोग की रिपोर्ट राजीव गाँधी सरकार ने नहीं दिखाई थी।

इस प्रकार भारत का राष्ट्रपति संसदात्मक व्यवस्था होने के नाते तानाशाह बनने की स्थिति में नहीं है, क्योंकि 44वें संविधान संशोधन के बाद तो वह आपात्कालीन अधिकारों का दुरुपयोग भी नहीं कर सकता है, लेकिन वह निश्चित रूप से ब्रिटेन के राजा से अधिक अच्छी स्थिति में है, क्योंकि वह गणराज्य का निर्वाचित अध्यक्ष है और संविधान के

अन्तर्गत उसे ‘संविधान का रक्षक’ (Defender of the Constitution) …. बनाया गया है जिस कारण उसे कुछ स्व-विवेकी अधिकार भी मिल गए हैं। राष्ट्रपति की स्थिति को नेहरू जी ने ठीक ही इन शब्दों में व्यक्त किया है, “हमने अपने राष्ट्रपति को कोई वास्तविक शक्ति नहीं दी, बल्कि उसके पद को बड़ा शक्तिशाली और
सम्मानजनक बनाया है।”

प्रश्न 8.
भारत के उप-राष्ट्रपति की चुनाव-पद्धति, शक्तियों व कार्यों का वर्णन करो। अथवा उप-राष्ट्रपति की शक्तियों व स्थिति की व्याख्या करें।
उत्तर:
संविधान के अनुच्छेद 63 के अन्तर्गत राष्ट्रपति की सहायता के लिए एक उप-राष्ट्रपति की व्यवस्था की गई है। उप-राष्ट्रपति का निर्वाचन संसद के दोनों सदनों के सदस्यों द्वारा एक संयुक्त बैठक में आनुपातिक प्रतिनिधित्व के अनुसार एकल संक्रमणीय मत प्रणाली द्वारा किया जाता है।

राष्ट्रपति तथा उप-राष्ट्रपति की चुनाव पद्धति में भिन्नता होने का कारण यह है कि मुख्य कार्यपालक होता है तथा उसके निर्वाचन के लिए संघीय संसद एवं राज्य विधानपालिकाओं के निर्वाचित सदस्य भाग लेते हैं। इसके विपरीत उप-राष्ट्रपति साधारणतः केवल राज्यसभा के अध्यक्ष के रूप में कार्य करता है और उसे राष्ट्रपति की अनुपस्थिति में उसके स्थान पर काम करने के बहुत कम अवसर प्राप्त होते हैं।
HBSE 11th Class Political Science Important Questions Chapter 4 Img 8

1. योग्यताएँ (Qualifications):
उप-राष्ट्रपति के पद पर चुने जाने वाले व्यक्ति के लिए की गई योग्यताएँ इस प्रकार हैं निश्चित की गई हैं

  • वह भारत का नागरिक होना चाहिए,
  • उसकी आयु कम-से-कम 35 वर्ष हो,
  • उसके पास वे सभी योग्यताएँ हों जो राज्यसभा का सदस्य बनने के लिए आवश्यक हैं,
  • वह केन्द्रीय अथवा किसी राज्य सरकार के अधीन किसी लाभ के पद पर कार्य न कर रहा हो,
  • उप-राष्ट्रपति संसद अथवा राज्य विधानसभा का सदस्य नहीं होना चाहिए।यदि वह इनमें से किसी का सदस्य है तो उप-राष्ट्रपति चुने जाने पर उसे इसकी सदस्यता से त्याग-पत्र देना पड़ता है,
  • 5 जून, 1997 को राष्ट्रपति द्वारा जारी किए गए एक अध्यादेश के अनुसार उप-राष्ट्रपति पद के लिए चुनाव लड़ने वाले उम्मीदवार के लिए जमानत की राशि 15,000 रुपए कर दी गई है,
  • इसी अध्यादेश के द्वारा ही यह व्यवस्था की गई है कि उप-राष्ट्रपति पद के लिए चुनाव लड़ने वाले उम्मीदवार का नाम संसद के 20 सदस्यों द्वारा प्रस्तावित तथा अन्य 20 सदस्यों द्वारा अनुमोदित होना चाहिए।

2. उप-राष्ट्रपति का चुनाव (Election of Vice-President):
देश के वर्तमान उपराष्ट्रपति का चुनाव 5 अगस्त, 2017 को संपन्न हुआ। इस चुनाव में सतारूढ़ एन०डी०ए० (NDA) के प्रत्याशी श्री वेंकैया नायडू एवं यू०पी०ए० (UPA) गठबन्धन के प्रत्याशी श्री गोपाल कृष्ण गाँधी के बीच मुख्य मुकाबला था।

इस चुनाव में निर्वाचक मण्डल के कुल सदस्यों की संख्या 790 थी जिनमें से 771 सदस्यों ने मतदान में भाग लिया। इनमें से 11 सदस्यों के मत अवैध घोषित हुए और शेष पड़े 760 मतों में से 516 मत एन०डी०ए० प्रत्याशी श्री वेंकैया नायडू को प्राप्त हुए तथा 244 मत यू०पी०ए० गठबन्धन के प्रत्याशी श्री गोपाल कृष्ण गाँधी को प्राप्त हुए। इस प्रकार इस चुनाव में विजयी प्रत्याशी श्री वेंकैया नायडू को 11 अगस्त, 2017 को राष्ट्रपति श्री रामनाथ कोविंद ने राष्ट्रपति भवन में आयोजित समारोह में पद और गोपनीयता की शपथ दिलाई।

इस प्रकार श्री वेंकैया नायडू भारत के 15वें उपराष्ट्रपति तथा इस पद पर रहे व्यक्तियों की संख्या की दृष्टि से 13वें उपराष्ट्रपति बनें।

3. उप-राष्ट्रपति का कार्यकाल (Term of the Office of the Vice-President):
राष्ट्रपति की भांति उप-राष्ट्रपति का चुनाव भी 5 वर्ष के लिए होता है। यदि उप-राष्ट्रपति चाहे तो इस अवधि से पहले भी राष्ट्रपति को लिखकर अपने पद से त्याग-पत्र दे सकता है। उप-राष्ट्रपति को पद से हटाने के लिए राज्यसभा के उपस्थित सदस्यों के बहुमत से उसके हटाने का प्रस्ताव पास करना होता है और लोकसभा को उस प्रस्ताव का समर्थन करना होता है।

4. वेतन तथा भत्ते (Salary and Allowances):
फरवरी, 2018 में किए गए नवीन संशोधन के अनुसार उप-राष्ट्रपति को राज्य सभा के पदेन अध्यक्ष के रूप में 4 लाख रुपए प्रतिमाह प्राप्त होंगे और सेवानिवृत्ति पर उन्हें मूल वेतन का 50 प्रतिशत प्रतिमाह पेंशन के रूप में प्राप्त होंगे। इसके अतिरिक्त निःशुल्क सुसज्जित निवास-स्थान, मुफ्त डॉक्टरी सहायता, संसद सदस्य के समान निःशुल्क टेलीफोन कॉलें तथा निःशुल्क रेलवे एवं हवाई यात्राओं की सुविधा भी उप-राष्ट्रपति को प्रदान की गई है।

यहाँ यह उल्लेखनीय है कि भारतीय उप-राष्ट्रपति को उप-राष्ट्रपति के नाते वेतन नहीं दिया जाता है, बल्कि राज्यसभा के सभापति के रूप में वेतन दिया जाता है। जब उप-राष्ट्रपति किसी कारणवश राष्ट्रपति के पद पर कार्य करता है, तब उसे राष्ट्रपति के पद से सम्बन्धित वेतन-भत्ते और अन्य सुविधाएँ मिलती हैं।

शपथ (Oath) राष्ट्रपति की भांति उप-राष्ट्रपति को भी अपना पद ग्रहण करते समय राष्ट्रपति अथवा उस द्वारा नियुक्त किए गए अधिकारियों के सामने निम्नलिखित शपथ लेनी पड़ती है

“मै……….ईश्वर की शपथ लेता हूँ सत्यनिष्ठा से प्रतिज्ञा करता हूँ कि मैं विधि द्वारा स्थापित भारत के संविधान के प्रति सच्ची श्रद्धा और निष्ठा र गा तथा जिस पद को मैं ग्रहण करने वाला हूँ उसके कर्तव्यों का श्रद्धापूर्वक निर्वहन करूँगा।”-(धारा 69) उप-राष्ट्रपति की शक्तियाँ तथा कार्य (Powers and Functions of the Vice-President)-संवैधानिक रूप में उप-राष्ट्रपति की शक्तियों का स्पष्ट विवेचन नहीं किया गया है, फिर भी इसका स्थान राष्ट्रपति के बाद दूसरे क्रम पर निर्धारित किया गया है। सामान्य रूप में उप-राष्ट्रपति के कार्य निम्नलिखित हो सकते हैं

1. राष्ट्रपति के रूप में (As a President):
उप-राष्ट्रपति के रूप में उसे कोई विशेष अधिकार नहीं दिए गए। हाँ, जब राष्ट्रपति का पद किसी कारणवश खाली हो जाए तो वह अस्थायी रूप से राष्ट्रपति के सभी कार्यों को सम्भालता है। जब तक नए राष्ट्रपति का निर्वाचन नहीं हो जाता। यदि मृत्यु अथवा त्याग-पत्र अथवा महाभियोग के कारण जब राष्ट्रपति का पद खाली होता है तो वह उस पद पर अधिक-से-अधिक छह मास तक रहता है। इस बीच नए राष्ट्रपति का चुनाव हो जाना आवश्यक होता है।

यहाँ यह उल्लेखनीय है कि जब उप-राष्ट्रपति राष्ट्रपति के रूप में कार्य करता है तो उसे राष्ट्रपति की सभी शक्तियों, उन्मुक्तियों, उपबन्धियों, भत्तों और विशेष अधिकारों का अधिकार प्राप्त होगा। संसद का ‘प्रेसीडेन्ट सक्सेशन एक्ट, 1969’ यह व्यवस्था करता है कि यदि उप-राष्ट्रपति किसी कारणवश उपलब्ध नहीं है तो उच्चतम न्यायालय का मुख्य न्यायाधीश या उसके न रहने पर उसी न्यायालय का वरिष्ठतम न्यायाधीश जो उस समय उपलब्ध हो, राष्ट्रपति के कार्यों को सम्पादित करेगा।

2. राज्यसभा के अध्यक्ष के रूप में (AsChairman of the Rajya Sabha):
भारत का उप-राष्ट्रपति राज्यसभा का पदेन सभापति होता है। इस कारण वह राज्यसभा की बैठकों की अध्यक्षता करता है। अध्यक्ष के नाते उसके निम्नलिखित कार्य इस प्रकार हैं-

  • वह राज्यसभा के अधिवेशन में सभापतित्व करता है,
  • राज्यसभा में कानून व्यवस्था बनाए रखना उसकी जिम्मेदारी है, जो व्यक्ति सदन के अनुशासन को भंग करता है, उप-राष्ट्रपति उसे सदन से बाहर निकाल सकता है,
  • वह सदस्यों को बोलने की आज्ञा देता है,
  • उप-राष्ट्रपति को साधारणतः वोट देने का अधिकार नहीं है, लेकिन उसे निर्णायक मत (Casting Vote)का अधिकार है,
  • वह किसी भी बिल पर मतदान करा सकता है तथा गिनती के बाद परिणाम घोषित करता है,
  • जब बिल को राज्यसभा पास कर देती है, तब उप-राष्ट्रपति उस पर हस्ताक्षर करता है,
  • वह किसी भी सदस्य को असंसदीय भाषा का प्रयोग करने की मनाही कर सकता है।

3. सहयोगी कार्य (To provide co-operation to the President):
उप-राष्ट्रपति राष्ट्रपति के उन कार्यों को भी सम्पादित करता है जिनके बारे में राष्ट्रपति उप-राष्ट्रपति से सहयोग की अपेक्षा करता है। अतः इस आधार पर यह स्पष्ट होता है कि निःसन्देह उप-राष्ट्रपति की शक्तियाँ नगण्य मात्र हैं।

उप-राष्ट्रपति की स्थिति (Position of the Vice-President) संवैधानिक दृष्टि से उप-राष्ट्रपति को कोई विशेष अधिकार प्राप्त नहीं हैं, लेकिन मान-सम्मान की दृष्टि से उसका देश में दूसरा स्थान है। उसका पद गरिमा का पद है, अतः उप-राष्ट्रपति की स्थिति उसके व्यक्तित्व पर निर्भर करती है। उदाहरणस्वरूप, डॉ० राधाकृष्णन, डॉ० शंकरदयाल शर्मा तथा श्री के०आर० नारायणन ने सभी राजनीतिक दलों, राजनीतिक नेताओं तथा साधारण जनता से बहुत सम्मान प्राप्त किया है।

प्रश्न 9.
संघीय मन्त्रि-परिषद् की रचना तथा कार्यों का राष्ट्रपति तथा संसद के सन्दर्भ में विवेचन कीजिए। अथवा संघीय मन्त्रिपरिषद् की रचना, शक्तियों व कार्यों का वर्णन कीजिए।
उत्तर:
संघ की सभी कार्यपालिका शक्तियाँ राष्ट्रपति को दी गई हैं, परन्तु वह नाममात्र का अध्यक्ष है और इन शक्तियों का प्रयोग मन्त्रि-परिषद् की सलाह से करता है। संविधान के अनुच्छेद 74 में मन्त्रिपरिषद् की व्यवस्था की गई है जो राष्ट्रपति को उसके कार्यों में सलाह तथा सहयोग देती है। संविधान के 42वें तथा 44वें संशोधन ने स्पष्ट कर दिया है कि राष्ट्रपति मन्त्रिपरिषद् की सलाह मानने के लिए बाध्य है। अधिक-से-अधिक वह मन्त्रिमण्डल को अपनी सलाह पर पुनः विचार करने के लिए कह सकता है।

मन्त्रिपरिषद् का निर्माण (Composition of Council of Ministers) सर्वप्रथम राष्ट्रपति प्रधानमन्त्री की नियुक्ति करता है। नए चुनाव के बाद लोकसभा में जिस दल को अथवा दलीय गठबन्धन को बहुमत प्राप्त होता है, उसके नेता को ही प्रधानमन्त्री नियुक्त किया जाता है। जब किसी भी दल को स्पष्ट बहुमत प्राप्त न हो तो राष्ट्रपति अपने विवेक का प्रयोग कर सकता है, परन्तु उस समय भी ऐसे व्यक्ति को ही प्रधानमन्त्री नियुक्त किया जाना आवश्यक है जो बहुमत अपने पक्ष में कर सके। जैसे सितम्बर-अक्तूबर, 1999 के लोकसभा के चुनाव में किसी दल को बहुमत प्राप्त न होने की स्थिति में राष्ट्रपति ने 24 राजनीतिक दलों के गठबन्धन द्वारा निर्वाचित नेता श्री अटल बिहारी वाजपेयी को प्रधानमन्त्री नियुक्त किया था और उसे अपना मन्त्रिमण्डल निर्माण करने को कहा था।

अन्य मन्त्रियों की नियुक्ति राष्ट्रपति प्रधानमन्त्री की सलाह से करता है। वास्तव में प्रधानमन्त्री अन्य मन्त्रियों की सूची राष्ट्रपति को देता है और राष्ट्रपति उसी के अनुसार मन्त्रियों की नियुक्ति करता है। वह इसमें अपनी इच्छा से कोई परिवर्तन नहीं कर सकता। मन्त्री बनने के लिए यह योग्यता निश्चित है कि उसे 6 महीने के अन्दर-अन्दर संसद के किसी सदन का सदस्य बन जाना चाहिए नहीं तो वह अपने पद पर नहीं रह सकता। राष्ट्रपति मन्त्रियों में विभागों का बँटवारा प्रधानमन्त्री की सलाह के अनुसार ही करता है।

मन्त्रियों की विभिन्न श्रेणियाँ (Categories of the Ministers) मंत्रिपरिषद् में विभिन्न स्तर के मंत्री होते हैं। पहले स्तर के मंत्रियों को कैबिनेट स्तर के मंत्री कहा जाता है। ये मंत्री किसी-न-किसी विभाग के अध्यक्ष होते हैं और इनकी संख्या प्रायः 20 से 30 तक होती है। 17वीं लोकसभा के चुनावों के बाद श्री नरेन्द्र मोदी के नेतृत्व में गठित नई सरकार (30 मई, 2019) में 25 कैबिनेट मंत्री, 23 राज्य मंत्री तथा 11 स्वतंत्र प्रभार वाले मंत्री शामिल किए गए।

दूसरे स्तर के मंत्री राज्यमंत्री (Minister of State) होते हैं। ये विशेष विभागों से संबंधित होते हैं। ये कैबिनेट स्तर के मंत्रियों को सहायता पहुँचाते हैं। इनके नीचे तीसरे स्तर के उप-मंत्री तथा संसदीय सचिव होते हैं। ये विभागीय मंत्रियों को प्रशासन और संसदीय कार्यों के संबंध में सहायता पहुँचाते हैं।

इसके अतिरिक्त कुछ विभाग-रहित मंत्री (Ministers without Portfolio) भी होते हैं, जिनका काम प्रशासन चलाने में सहायता करना होता है। यहाँ यह भी उल्लेखनीय है कि जनवरी, 2004 में 91वें संविधान संशोधन अधिनियम के पारित होने के पश्चात् मंत्रिपरिषद् (संघ एवं राज्यों) के सदस्यों की अधिकतम संख्या निम्न सदन की कुल संख्या के 15 प्रतिशत से अधिक नहीं हो सकती। 30 मई, 2019 में 25 कैबिनेट, 23 राज्य मन्त्री तथा 11 स्वतन्त्र प्रभार वाले राज्य मंत्री शामिल किए गए।

मन्त्रिपरिषद् का कार्यकाल (Term of Office)-मन्त्रि-परिषद् का कोई निश्चित कार्यकाल नहीं है। संविधान में कहा गया है कि वे राष्ट्रपति के प्रसाद-पर्यन्त अपने पद पर रहते हैं, परन्तु वास्तव में वे उस समय तक अपने पद पर बने रहते हैं जब तक लोकसभा का बहुमत उनके साथ है। यदि लोकसभा मन्त्रिमण्डल के विरुद्ध अविश्वास प्रस्ताव पास कर दे तो मन्त्रिमण्डल को त्यागपत्र देना पड़ता है। जैसे 7 नवम्बर, 1990 को लोकसभा में वी०पी० सिंह मन्त्रिमण्डल के विरुद्ध अविश्वास प्रस्ताव पारित किया था।

प्रधानमन्त्री यदि त्यागपत्र दे दे तो समस्त मन्त्रि-परिषद् का त्यागपत्र माना जाता है जैसे मार्च, 1991 में श्री चन्द्रशेखर ने तथा 1997 में श्री इन्द्र कुमार गुजराल ने प्रधानमन्त्री पद से त्यागपत्र दे दिया था। वर्ष, 1999 में भी लोकसभा ने प्रधानमन्त्री श्री अटल बिहारी वाजपेयी की सरकार के विरुद्ध अविश्वास का पत्र पास किया और प्रधानमन्त्री ने त्यागपत्र दे दिया। साथ ही मन्त्रिपरिषद् का कार्यकाल भी समाप्त हो गया। राष्ट्रपति प्रधानमन्त्री की सलाह पर किसी भी मन्त्री को हटा सक का कार्यकाल अनिश्चित है।

मन्त्रिमण्डल की बैठकें (Meetings of the Cabinet)-मन्त्रिमण्डल की समय पर बैठकें होती हैं और इनकी अध्यक्षता प्रधानमन्त्री द्वारा की जाती है। राष्ट्रपति इनमें भाग नहीं लेता। मन्त्रि-परिषद् के कार्य तथा शक्तियाँ (Powers and Functions of the Council of Ministers)-मन्त्रि-परिषद को निम्नलिखित शक्तियाँ प्राप्त हैं

1. नीति-निर्धारण (Formation of National Policy) :
मन्त्रि-परिषद् का पहला कार्य राष्ट्र की नीति का निर्धारण करना है पर उसे देश की आर्थिक, सामाजिक, राजनीतिक तथा अन्य समस्याओं का समाधान करना और देश का विकास करना है।

2. प्रशासन पर नियन्त्रण (Control over the Administration):
संघ की समस्त शक्तियों का प्रयोग मन्त्रि-परिषद् करती है और इस प्रकार वह प्रशासन पर नियन्त्रण रखती है। प्रशासन का प्रत्येक विभाग किसी-न-किसी मन्त्री के अधीन होता है जो मन्त्रिमण्डल के निर्णयों तथा राष्ट्रीय नीति के अनुसार उसका संचालन करता है तथा उसके सुचारु रूप से संचालन के लिए संसद के प्रति उत्तरदायी होता है। सभी सरकारी कर्मचारी मन्त्रियों के अधीन कार्य करते हैं और कानूनों को लागू करते हैं।

3. विदेशों से सम्बन्ध स्थापित करना (To Maintain Foreign Relations):
मन्त्रिमण्डल ही अपनी विदेश नीति के अनुसार दूसरे देशों से सम्बन्ध स्थापित करता है, दूसरे देशों को भेजे जाने वाले राजदूतों का निर्णय करता है और दूसरे देशों से सन्धियाँ तथा समझौते करता है। युद्ध तथा शान्ति की घोषणा के बारे में निर्णय मन्त्रि-परिषद् द्वारा किया जाता है।

4. वैधानिक शक्तियाँ (Legislative Powers):
मन्त्रिमण्डल को वैधानिक शक्तियाँ भी प्राप्त हैं। संसद में अधिकतर विधेयक मन्त्रि-परिषद् द्वारा पेश किए जाते हैं और उसके दल अथवा दलीय गठबन्धन का बहुमत होने के कारण उसकी इच्छा के अनुसार ही कानून बनते हैं। मन्त्रि-परिषद् की इच्छा के विरुद्ध कोई विधेयक कानून नहीं बन सकता।

5. वित्तीय शक्तियाँ (Financial Powers):
मन्त्रिमण्डल को वित्तीय शक्तियाँ भी प्राप्त हैं। बजट के बारे में सबसे पहले मन्त्रिमण्डल में निर्णय होता है और संसद में बजट तथा अन्य धन विधेयक मन्त्री ही पेश कर सकते हैं, साधारण सदस्य नहीं। का प्रस्ताव, पराने करों में कमी अथवा बढोतरी या उन्हें समाप्त करने के प्रस्ताव आदि के विधेयक मन्त्रि-परिषद निश्चित करती है और लोकसभा में उसका बहुमत होने के कारण वह पास हो जाता है। इस प्रकार वित्तीय विधेयक पर वास्तविक नियन्त्रण मन्त्रिमण्डल का है, संसद का नहीं।।

6. नियुक्तियाँ (Appointments):
देश में की जाने वाली बड़ी-बड़ी नियुक्तियों के बारे में सर्वप्रथम निर्णय मन्त्रिमण्डल में ही होता है। इन पदों में मुख्य हैं-राज्यपाल, न्यायाधीश, राजदूत, संघ लोक सेवा आयोग के अध्यक्ष तथा सदस्य, चुनाव आयुक्त, भारत का महाविधिवेत्ता, महालेखा परीक्षक तथा नियन्त्रक, विभिन्न आयोगों के अध्यक्ष तथा सदस्य। ये सब नियुक्तियाँ राष्ट्रपति मन्त्रिमण्डल की सलाह के अनुसार ही करता है।

7. लोकसभा को भंग करने की शक्ति (Power to Dissolve the Lok Sabha):
मन्त्रिमण्डल को यह अधिकार है कि वह लोकसभा की अवधि समाप्त होने से पहले भी उसे भंग करने की सलाह राष्ट्रपति को दे सकती है। यह अधिकार इतना प्रभावशाली होता है कि लोकसभा आसानी से मन्त्रिमण्डल का विरोध नहीं कर सकती है।

8. संकटकालीन स्थिति का निर्णय (Decision Regarding the Proclamation of Emergency):
राष्ट्रपति संकटकाल की उद्घोषणा मन्त्रिमण्डल की सलाह से ही करता है। संकट की स्थिति पैदा हो चुकी है या होने की सम्भावना है, इसका निर्णय मन्त्रिमण्डल में ही होता है। इस प्रकार व्यवहार में राष्ट्रपति की आपात्कालीन शक्तियों का प्रयोग भी मन्त्रिमण्डल ही करता है।

9. उपाधियाँ देना (To Give Titles):
राष्ट्रपति नागरिकों तथा कर्मचारियों को विभिन्न उपाधियों-भारत रत्न, पद्म विभूषण, पदमश्री इत्यादि से विभूषित करता है। मन्त्रिपरिषद् की स्थिति (Position of Council of Ministers) संसदात्मक व्यवस्था में वास्तव में कार्यपालिका की समस्त शक्तियाँ मन्त्रि-परिषद् में ही निहित हैं। व्यावहारिक रूप में मन्त्रिपरिषद् की शक्तियों व कार्यों में वृद्धि हुई है, क्योंकि संसद के पास समय की कमी होती है।

संसद तो मुख्य रूप से नीतियों पर ही विचार करती है। कानून बनाने का उत्तरदायित्व व्यावहारिक रूप में मन्त्रि-परिषद् को ही प्रदान कर देती है। इसी प्रकार अधिकांश प्रस्ताव सरकारी प्रस्ताव होते हैं जिन्हें मन्त्री ही प्रस्तावित करते हैं। इन शक्तियों के अलावा आपास्थिति को लागू करवाने की शक्ति तथा लोकसभा भंग कराने की शक्ति मन्त्रि-परिषद् की स्थिति को मजबूत बना देती है।

लेकिन मन्त्रि-परिषद् भले ही कितनी भी शक्तिशाली क्यों न हो, उस पर संसदीय नियन्त्रण सदैव बना रहता है। कोई भी मन्त्रिपरिषद् संसद की अनदेखी नहीं कर सकती विशेषकर लोकसभा की। यदि कोई मन्त्रि-परिषद् संसद की अवहेलना करती है तो उसके विरुद्ध लोकसभा अविश्वास पारित करके मन्त्रि-परिषद् को हटा सकती है। इसके अलावा लोकतान्त्रिक व्यवस्था में मन्त्रि-परिषद् तानाशाह नहीं बन सकती, क्योंकि संसद के अतिरिक्त न्यायपालिका व जनता का दबाव भी रहता है जिसकी अनदेखी या उपेक्षा करना आत्महत्या के समान होता है।

प्रश्न 10.
भारत के प्रधानमन्त्री की नियुक्ति, शक्तियों, कार्यों तथा स्थिति का आलोचनात्मक परीक्षण कीजिए। अथवा भारत के प्रधानमन्त्री की भूमिका पर 150 शब्दों में एक नोट लिखें।
अथवा
“आधुनिक समय में भारत के प्रधानमन्त्री की शक्तियों में बढ़ोतरी हुई है।” इस कथन को ध्यान में रखकर भारत के प्रधानमन्त्री की शक्तियाँ और स्थिति बताइए।
उत्तर:
राष्ट्रपति राज्य का नाममात्र का अध्यक्ष है और वास्तविक कार्यपालिका मन्त्रि-परिषद् है। प्रधानमन्त्री शासनाध्यक्ष है और संसदीय व्यवस्था में शासन की वास्तविक शक्तियाँ तथा अधिकार प्रधानमन्त्री के पास होते हैं, न कि राष्ट्रपति के पास। भारत में प्रधानमन्त्री के सम्बन्ध में संविधान में निम्नलिखित प्रावधान किए गए हैं

प्रधानमन्त्री की नियुक्ति (Appointment of the Prime Minister) भारतीय संविधान के अनुच्छेद 75 के अंतर्गत यह व्यवस्था की गई है, “प्रधानमंत्री की नियुक्ति राष्ट्रपति करेगा और अन्य मंत्रियों की नियुक्ति राष्ट्रपति द्वारा प्रधानमंत्री की सलाह पर की जाएगी।”

इस प्रकार इस अनुच्छेद से स्पष्ट है कि प्रधानमंत्री की नियुक्ति राष्ट्रपति द्वारा की जाएगी, लेकिन यह सच नहीं है, क्योंकि राष्ट्रपति लोकसभा में बहुमत दल के नेता को ही प्रधानमंत्री नियुक्त करता है। जैसे 17वीं लोकसभा चुनाव, 2019 के बाद भाजपा नेतृत्व वाले एन.डी.ए. को 353 स्थानों में अकेले भाजपा को 303 वोटों के साथ

स्पष्ट बहुमत प्राप्त होने पर बहुमत दल के निर्वाचित नेता श्री नरेन्द्र मोदी को राष्ट्रपति श्री रामनाथ कोविंद द्वारा 30 मई, 2019 को प्रधानमन्त्री पद की शपथ दिलाई गई। राष्ट्रपति अपने विवेक और सूझ-बूझ का उस स्थिति में प्रयोग करता है जब लोकसभा में किसी भी दल को स्पष्ट बहुमत प्राप्त नहीं होता।

ऐसी स्थिति में राष्ट्रपति ऐसे व्यक्ति को प्रधानमंत्री नियुक्त करता है जो केंद्र में स्थाई सरकार बना सके। राष्ट्रपति ने ऐसा सन 1989 में किया जब लोकसभा में किसी भी दल को स्पष्ट बहुमत प्राप्त नहीं था। उस समय राष्ट्रीय मोर्चे के नेता श्री विश्वनाथ प्रताप सिंह को प्रधानमंत्री नियुक्त किया गया था। फिर मई-जून, 1991 में लोकसभा चुनाव हुए उसमें भी किसी दल को स्पष्ट बहुमत प्राप्त नहीं हुआ तो राष्ट्रपति ने सबसे बड़े दल के नेता श्री पामुलपति वेंकट नरसिम्हा राव को प्रधानमंत्री नियुक्त किया था।

सन् 1996 में संपन्न हुए 11वीं लोकसभा के चुनावों में किसी भी दल को स्पष्ट बहुमत प्राप्त नहीं हुआ। राष्ट्रपति ने पिछली परंपराओं के आधार पर लोकसभा में सबसे बड़े दल के नेता श्री अटल बिहारी वाजपेयी को सरकार बनाने के लिए आमंत्रित किया और उन्हें प्रधानमंत्री नियुक्त किया। साथ ही तत्कालीन राष्ट्रपति द्वारा वाजपेयी को 15 दिन के अंदर संसद में बहुमत सिद्ध करने को कहा। श्री वाजपेयी बहुमत सिद्ध नहीं कर पाए और उन्होंने 13 दिन बाद अपना त्याग-पत्र दे दिया। इसके बाद 1 जून, 1996 । हर्दनहल्ली डोडागौड़ा देवेगौड़ा, जो संयुक्त मोर्चे के नेता बनें, उन्हें राष्ट्रपति द्वारा प्रधानमंत्री नियुक्त किया गया। श्री देवेगौड़ा ।

जून, 1996 को लोकसभा में बहुमत सिद्ध कर प्रधानमत्री के पद पर आसीन रहे, लेकिन कुछ समय पश्चात् बहुमत के 11 अप्रैल, 1997 को देवगौड़ा द्वारा त्याग-पत्र देने के बाद 21 अप्रैल, 1997 से श्री इंद्रकुमार गुजराल इस पद पर विराजमान रहे। इन्हें भी कुछ समय पश्चात् बहुमत के अभाव में अपना त्याग-पत्र देना पड़ा।

फरवरी-मार्च, 1998 में हुए 12वीं लोकसभा के चुनावों में भी किसी राजनीतिक दल अथवा गठबंधन को लोकसभा में पूर्ण बहुमत प्राप्त नहीं हुआ, चूंकि भारतीय जनता पार्टी तथा उसके सहयोगी दलों को लोकसभा में अन्य दलों तथा गठबंधनों से अधिक स्थान प्राप्त थे, इसलिए राष्ट्रपति द्वारा भारतीय जनता पार्टी संसदीय दल के नेता श्री अटल बिहारी वाजपेयी को सरकार का गठन करने के लिए आमंत्रित किया गया।

श्री अटल बिहारी वाजपेयी ने अन्य सहयोगी दलों के सहयोग से केंद्र में मिली-जुली सरकार (Coalition Government) का गठन किया। सितंबर-अक्तूबर, 1999 में हुए 13वीं लोकसभा चुनावों में भाजपा के नेतृत्व वाला राष्ट्रीय जनतांत्रिक गठबंधन (N.D.A.) (जो 24 दलों से मिलकर बना था) को स्पष्ट बहुमत प्राप्त हुआ। राजग ने श्री वाजपेयी जी को अपने गठबंधन का नेता चुना। राष्ट्रपति द्वारा श्री वाजपेयी को सरकार बनाने के लिए आमंत्रित किया गया। श्री वाजपेयी ने 13 अक्तूबर, 1999 को शपथ ग्रहण की।

मई, 2004 में हुए 14वें लोकसभा चुनाव में फिर किसी भी राजनीतिक दल एवं गठबंधन को स्पष्ट बहुमत प्राप्त नहीं मिला था लेकिन काँग्रेस गठबंधन सबसे बड़े गठबंधन के रूप में उभरकर सामने आया। परिणामस्वरूप तत्कालिक राष्ट्रपति ऐ०पी०जे० अब्दुल कलाम ने गठबंधन दल के नेता डॉ० मनमोहन सिंह को सरकार बनाने के लिए आमंत्रित किया था।

यही स्थिति मई, 2009 के चुनाव में रही। डॉ० मनमोहन सिंह को पुनः देश के प्रधानमंत्री के रूप में नियुक्त किया गया। यद्यपि 16वीं लोकसभा चुनाव 2014 एवं 17वीं लोकसभा चुनाव, 2019 में क्रमशः 282 एवं 303 सीटों का स्पष्ट बहुमत प्राप्त करने पर तात्कालिक राष्ट्रपति की ओर से श्री नरेन्द्र मोदी को सरकार बनाने के लिए आमन्त्रित किया गया और राष्ट्रपति द्वारा श्री नरेन्द्र मोदी को प्रधानमंत्री पद के लिए शपथ दिलाई गई।

प्रधानमंत्री के लिए संसद का सदस्य होना अनिवार्य (To be a Member of Parliament is Essential for the Prime Minister)-संविधान के अनुच्छेद 75 (5) में यह व्यवस्था की गई है कि, “कोई गैर-सदस्य भी मंत्रिपरिषद् में शामिल किया जा सकता है, परंतु ऐसे सदस्य के लिए यह अनिवार्य है कि वह 6 महीने में संसद के किसी भी सदन का सदस्य अवश्य बने।” इस प्रकार स्पष्ट है कि प्रधानमंत्री सहित मंत्रिपरिषद् के सभी सदस्यों के लिए यह अनिवार्य है कि वह संसद के दोनों सदनों में से किसी एक सदन के सदस्य अवश्य हों।

21 जून, 1991 को जब काँग्रेस के नेता श्री पी०वी० नरसिम्हाराव को प्रधानमंत्री नियुक्त किया गया था तो वे किसी भी सदन के सदस्य नहीं थे लेकिन उन्होंने 6 महीने की शर्त के अंदर ही नवंबर, 1991 में नांदयाल लोकसभा क्षेत्र (आंध्रप्रदेश) से विजयी होकर संसद सदस्यता की शर्त को पूर्ण कर दिया। वैसे भारतीय संसदीय इतिहास में वे पहले ऐसे व्यक्ति थे जो प्रधानमंत्री की नियुक्ति के समय संसद के किसी सदन के सदस्य नहीं थे। 1 जून, 1996 को श्री एच०डी० देवगौड़ा को जब प्रधानमंत्री नियुक्त किया गया था, तब वे भी संसद के किसी सदन के नेता नहीं थे।

इन्होंने ने भी सदस्यता हेतु निश्चित अवधि के भीतर कर्नाटक राज्य विधानसभा में 20 सितंबर, 1996 को राज्यसभा की सदस्यता प्राप्त की। यहाँ यह उल्लेखनीय है कि भारतीय संसदीय लोकतंत्र के इतिहास में यह दूसरा अवसर था जब राज्यसभा का सदस्य प्रधानमंत्री पद पर विराजमान था। इससे पूर्व लाल बहादुर शास्त्री की मृत्यु के पश्चात् 1966 में जब इंदिरा गांधी प्रधानमंत्री बनी थीं तो वे भी राज्यसभा की ही सदस्या थीं।

तीसरा अवसर तब आया जब 21 अप्रैल, 1997 को संयुक्त मोर्चा के नेता श्री इंद्रकुमार गुजराल भारत के प्रधानमंत्री बनें। श्री गुजराल भी उस समय राज्यसभा के सदस्य थे। भारतीय संसदीय लोकतंत्र के इतिहास में चौथा एवं पाँचवां इस प्रकार का अवसर तब आया जब क्रमशः 22 मई, 2004 एवं 22 मई, 2009 को डॉ. मनमोहन सिंह भारत के प्रधानमंत्री बने। डॉ. मनमोहन सिंह भी प्रधानमंत्री की नियुक्ति के समय राज्य सभा के सदस्य थे। लेकिन 2014 एवं 2019 में प्रधानमंत्री बने श्री नरेन्द्र मोदी वाराणसी लोकसभा क्षेत्र से सांसद निर्वाचित हुए थे।

इस प्रकार स्पष्ट है कि प्रधानमंत्री पद पर नियुक्ति संसद के दोनों सदनों में से किसी की भी सदस्यता वाले व्यक्ति की हो सकती है। यद्यपि सन् 1966 तक हमारे देश में लोकसभा के सदस्य को ही प्रधानमन्त्री पद पर नियुक्त किया जाता रहा। कार्यकाल (Term) संविधान के अनुच्छेद 75 के अंतर्गत यह व्यवस्था की गई है, “मंत्री राष्ट्रपति के प्रसाद-पर्यंत अपने पद पर बने रहेंगे।”

लेकिन व्यवहार में स्थिति बिल्कुल भिन्न है। प्रधानमंत्री 5 वर्ष के लिए नियुक्त किया जाता है, उसे लोकसभा में बहुमत प्राप्त है, वह अपने पद पर बना रहेगा। अभिप्राय यह है कि प्रधानमंत्री का कार्यकाल निश्चित नहीं है। लोकसभा के अविश्वास प्रस्ताव के द्वारा प्रधानमंत्री को हटाया जा सकता है।

10 जुलाई, 1979 को लोकसभा के विरोधी दल के नेता यशवन्त राव चहाण ने प्रधानमंत्री देसाई के विरुद्ध अविश्वास प्रस्ताव पेश किया। अविश्वास प्रस्ताव पर मतदान होने से पूर्व ही प्रधानमंत्री देसाई ने 15 जुलाई, 1979 को त्याग-पत्र दे दिया, क्योंकि जनता पार्टी के अधिकतर सदस्यों ने जनता पार्टी को छोड़ दिया था। यह पहला अवसर था जब किसी प्रधानमंत्री को अविश्वास प्रस्ताव के कारण त्याग-पत्र देना पड़ा।

इसी प्रकार 13 अप्रैल, 1998 को राष्ट्रपति के०आर० नारायण ने प्रधानमंत्री श्री अटल बिहारी वाजपेयी को सुश्री जयललिता द्वारा समर्थन वापस लिए जाने की स्थिति में लोकसभा का विश्वास प्राप्त करने के लिए कहा। श्री वाजपेयी की सरकार एक वोट से विश्वास मत प्रस्ताव में हार गई। अंततः प्रधानमंत्री को अपनी 13 माह पुरानी सरकार से त्याग-पत्र देना पड़ा। अतः स्पष्ट है कि प्रधानमंत्री का कार्यकाल निश्चित नहीं है।

प्रधानमन्त्री की शक्तियाँ तथा कार्य (Powers and Functions of Prime Minister) इंग्लैंड के प्रधानमंत्री के बारे में, श्री ग्लैडस्टोन (Glaodstone) ने कहा था, “कहीं भी इतने छोटे पदार्थ की इतनी बड़ी छाया नहीं है।” इसी प्रकार से श्री ग्रीब्स ने कहा था, “उसकी शक्तियाँ एक तानाशाह जैसी दिखाई पड़ती हैं।” यदि इन दोनों उक्तियों को भारत के प्रधानमंत्री पर लागू किया जाए तो ये बिल्कुल ठीक बैठती हैं। भारत में ऐसा कोई भी व्यक्ति नहीं है, जिसे इतनी व्यापक शक्तियाँ प्राप्त हों। भारतीय प्रधानमंत्री की शक्तियों एवं कार्यों का विवरण निम्नलिखित है

1.मंत्रिपरिषद् का निर्माण (Formation of Council of Ministers):
संघीय मंत्रिपरिषद् में राष्ट्रपति द्वारा सर्वप्रथम प्रधानमंत्री की नियुक्ति होने के बाद प्रधानमंत्री का सर्वप्रमुख कार्य मंत्रिपरिषद् का निर्माण करना है। संविधान के अनुच्छेद 75 के अनुसार, “मंत्रिपरिषद् के मंत्रियों की नियुक्ति प्रधानमंत्री के परामर्श पर राष्ट्रपति के द्वारा की जाती है।”

इस अनुच्छेद के माध्यम से यह स्पष्ट है कि प्रधानमंत्री अपने सहयोगी मंत्रियों की सूची तैयार करके राष्ट्रपति को सौंपता है और राष्ट्रपति प्रधानमंत्री द्वारा सुझाए गए व्यक्तियों को मंत्री के रूप में शपथ दिलाने का कार्य करता है। इस प्रकार स्पष्ट है कि राष्ट्रपति के द्वारा मंत्रियों की नियुक्ति एक औपचारिक कार्रवाई है क्योंकि राष्ट्रपति न तो अपनी इच्छानुसार किसी.व्यक्ति को मंत्री नियुक्त कर सकता है और न ही किसी मंत्री की नियुक्ति के संबंध में प्रधानमंत्री की सिफारिश को ठुकरा सकता है।

अतः यह निर्णय करना भी प्रधानमंत्री का कार्य है कि मंत्रिपरिषद् में किसे किस स्तर (कैबिनेट, राज्य एवं उपमंत्री) का मंत्री बनाना है। प्रायः प्रधानमंत्री का मंत्रिपरिषद् का गठन जितना आसान माना जाता है, व्यवहार में यह बहुत कठिन होता है क्योंकि मंत्रिपरिषद् के गठन के समय उन्हें अनेक बातों जैसे लोकप्रिय एवं प्रभावशाली जन कार्यपालिका नेता की ध्वनि, विषय विशेषज्ञ, प्रशासनिक दक्षता, अनुभवी नेता, विभिन्न क्षेत्रों एवं वर्गों को प्रतिनिधित्व आदि बातों का ध्यान रखना पड़ता है। जैसे 30 मई, 2019 को श्री नरेन्द्र मोदी के नेतृत्व में बने एन०डी०ए० के मंत्रिपरिषद् का ..46 सदस्यीय 25 कैबिनेट एवं 34 राज्य-मंत्री मंत्रिपरिषद् में उपर्युक्त बातों का ध्यान रखा गया।

2. विभागों का बँटवारा (Distribution of Portfolios):
न केवल प्रधानमंत्री मंत्रियों को नियुक्त करता है, अपितु वह उनमें विभागों का बँटवारा भी करता है। यह निर्णय वही करता है कि कौन-सा मंत्री किस विभाग का अध्यक्ष होगा तथा कौन-सा मंत्री कैबिनेट मंत्री होगा और कौन-सा मंत्री राज्य-मंत्री अथवा उप-मंत्री। विभाग बाँटने की शक्ति को प्रधानमंत्री की स्वेच्छाचारी शक्ति कहा जा सकता है। यह ठीक है कि उसे पार्टी के नेताओं को संतुष्ट रखना होता है तथा देश के प्रत्येक क्षेत्र को प्रतिनिधित्व देना… होता है, परंतु एक दृढ़ तथा लोकप्रिय नेता प्रधानमंत्री के रूप में अपनी इच्छानुसार मंत्रिपरिषद् को रूप दे सकता है।

3. लोकसभा का नेतृत्व करता है (Leader of the Lok Sabha):
इंग्लैंड की भाँति भारत का प्रधानमंत्री लोकसभा का नेतृत्व करता है। वह सदन में सरकार की नीति से संबंधित महत्त्वपूर्ण घोषणाएँ करता है और प्रश्नों का उत्तर देता है। वह लोकसभा में वाद-विवाद को आरंभ करता है तथा मंत्रियों की सदन में आलोचना से सुरक्षा करता है। वह अपने दल के सदस्यों को सचेतक (Whips) द्वारा आदेश तथा निर्देश भेजता है तथा उन पर निगरानी और नियंत्रण रखता है। सदन के वैधानिक कार्यों पर उसका विशेष प्रभाव होता है। वह स्पीकर के साथ मिलकर सदन का कार्य करता है तथा सदन में अनुशासन बनाए रखने के लिए स्पीकर की सहायता करता है।

यहाँ यह उल्लेखनीय है कि यदि प्रधानमंत्री लोकसभा का सदस्य न हो तो वह अपने दल के किसी अन्य व्यक्ति को लोकसभा का नेता नियुक्त करता है। जैसे 14वीं लोकसभा चुनाव के बाद नियुक्त प्रधानमंत्री 22 मई, 2004 एवं 15वीं लोकसभा चुनाव के बाद पुनः 22 मई, 2009 को नियुक्त प्रधानमंत्री डॉ. मनमोहन सिंह ने अपने दल के अन्य सदस्य श्री प्रणब मुखर्जी को लोकसभा का नेता नियुक्त किया था, क्योंकि प्रधानमंत्री स्वयं लोकसभा के सदस्य नहीं थे, बल्कि राज्य सभा के सदस्य थे। श्री प्रणब मुखर्जी के राष्ट्रपति बन जाने पर गृहमन्त्री श्री सुशील कुमार शिंदे को लोकसभा का नेता नियुक्त किया गया था।

4. राष्ट्रपति तथा मंत्रिमंडल में कड़ी का काम करता है (Link between the President and the Cabinet):
प्रधानमंत्री राष्ट्रपति तथा मंत्रिमंडल के मध्य कड़ी का काम करता है। वह राष्ट्रपति को मंत्रिमंडल के द्वारा किए गए निर्णय की सूचना देता है तथा राष्ट्रपति के विचार मंत्रिमंडल के समक्ष रखता है। राष्ट्रपति उसे किसी एक मंत्री द्वारा व्यक्तिगत रूप से किए गए निर्णयों पर मंत्रिमंडल का निर्णय लेने के लिए कह सकता है। वह राष्ट्रपति का मुख्य सलाहकार होता है। यदि राष्ट्रपति प्रधानमंत्री के परामर्श से सहमत न हो तो भी उसे परामर्श मानना पड़ता है। .

5. विभिन्न विभागों में एक कड़ी (As a Link between Different Departments):
कैबिनेट का प्रधान होने के नाते वह एक अन्य महत्त्वपूर्ण कार्य करता है। वह विभिन्न विभागों में उत्पन्न होने वाली आपसी समस्याओं, झगड़ों तथा मतभेदों को इस तरह सुलझाता है, जिससे प्रशासनिक कुशलता बनी रहे। कुशल प्रशासन के लिए यह आवश्यक है कि सरकार के विभिन्न विभागों में आपसी सहयोग तथा साधनों का समन्वय हो। ऐसे उद्देश्य के लिए प्रधानमंत्री विभिन्न विभागों में कड़ी की तरह कार्य करता है। वह अंतर्विभागीय मतभेदों को दूर करने के लिए मध्यस्थ तथा निर्णायक के रूप में भी कार्य करता है।

6. राष्ट्रपति का मुख्य सलाहकार (Chief Advisor of the President):
प्रधानमंत्री राष्ट्रपति का मुख्य सलाहकार है। राष्ट्रपति प्रत्येक मामले पर प्रधानमंत्री की सलाह लेता है और उसके द्वारा दी गई सलाह के अनुसार ही कार्य करता है। वह उसकी सलाह को मानने के लिए बाध्य है। राष्ट्रपति को प्रशासन के बारे में किसी भी प्रकार की सूचना प्राप्त करनी हो, तो वह किसी अन्य मंत्री से सीधा बात न करके प्रधानमंत्री से ही बात करता है तथा सूचना प्राप्त करता है।

7. दल का नेता (Leader of the Party):
प्रधानमंत्री अपने दल का नेता होता है। दल की नीतियों तथा कार्यक्रमों को तैयार करने में उसका मुख्य हाथ होता है। आम चुनाव के समय दल के उम्मीदवारों को खड़ा करना, उन्हें टिकटें देना तथा उन्हें चुनाव जीताने में वह बहुत ही महत्त्वपूर्ण भूमिका निभाता है। उस समय वह समस्त देश का दौरा करके जनता से उसके दल के उम्मीदवारों के पक्ष में मत देने की अपील करता है।

8. सदन के नेता के रूप में (Leader of the House):
दल का नेता होने के साथ-साथ वह सदन का भी नेता होता है। प्रधानमंत्री ही संसद के अधिवेशन बुलाने की तिथि निश्चित करता है। लोकसभा की कार्रवाई चलाने का उत्तरदायित्व उस पर ही होता है। कौन-सा बिल कब प्रस्तुत किया जाएगा और कौन-सा बाद में, किस बिल पर कितना वाद-विवाद होगा, यदि विरोधी दल बिल पेश करना चाहता है तो उस पर वाद-विवाद कब होगा,

इन सब बातों का निर्णय स्पीकर प्रधानमंत्री और विपक्ष के नेता से सलाह करके है। प्रधानमंत्री ही सदन में महत्त्वपूर्ण नीतियों की घोषणा करता है। प्रधानमंत्री ही सरकार की नीतियों को सदन में पेश करता है। यदि विरोधी दल द्वारा इन नीतियों की आलोचना की जाती है, तो वह उन आलोचनाओं का उत्तर देता है। विशेषकर प्रधानमंत्री की महत्ता और भी बढ़ जाती है, जब वह अविश्वास के प्रस्ताव के नाजुक समय में अपने दल की रक्षा करता है।

9. प्रधानमंत्री राष्ट्र के नेता के रूप में (As the Leader of the Nation):
प्रधानमंत्री राष्ट्र का नेता है। सारा राष्ट्र प्रधानमंत्री की ओर अच्छे प्रशासन व पथ-प्रदर्शन के लिए निगाहें लगाए हुए होता है। साधारणतः चुनाव भी प्रधानमंत्री के नाम पर लडे जाते हैं; जैसे सन् 1980 में इंदिरा गांधी ने चुनाव जीता, ऐसे ही सन् 1989 में श्री वी०पी० सिंह ने जनता को विश्वास दिलाया था कि वह राष्ट्र को साफ-सुथरी सरकार देंगे। इसी प्रकार 13वीं लोकसभा चुनाव में यह नारा दिया गया है कि “अब की बारी अटल बिहारी” अर्थात् मतदाता दल को महत्त्व देते हैं, लेकिन साथ में इसके नेता को भी महत्त्व देते हैं।

इसी तरह 16वीं लोकसभा चुनाव, 2014 में भाजपा के नेता श्री नरेन्द्र मोदी ने ‘सबका साथ सबका विकास’ के नारे के साथ मतदाताओं को आकर्षित किया तो पुनः 2019 के 17वीं लोकसभा चुनाव में ‘सबका साथ सबका विकास एवं सबका विश्वास’ के साथ पुनः एक बार फिर मोदी सरकार के नारे मा को मोदी जी के नेतृत्व में पुनः स्पष्ट बहुमत प्राप्त हुआ। अतः मतदाताओं ने मोदी जी को राष्ट्रीय नेतृत्व के रूप में पूर्णतः आस्था व्यक्त की। प्रधानमंत्री का राष्ट्र के नेता के रूप में महत्त्व संकटकालीन समय में और भी बढ़ जाता है तथा सारा राष्ट्र देश के प्रधानमंत्री की ओर देखता है और जनता उसके विचारों को बड़े ध्यान से सुनती है।

10. लोकसभा को भंग कराने का अधिकार (Power to get Lok Sabha Dissolved):
इंग्लैंड के प्रधानमंत्री को यह विशेषाधिकार प्राप्त है कि वह कॉमंस सभा को भंग करवाकर देश में चुनाव करवा सकता है अर्थात उसे कॉमंस सभा को भंग करने का अधिकार प्राप्त है। इस मामले में उसे मंत्रिमंडल की सलाह लेने की आवश्यकता नहीं होती। इसी प्रकार का अधिकार भारत के प्रधानमंत्री को दिया गया है। वह राष्ट्रपति को सलाह देकर लोकसभा को भंग करवा सकता है।

जैसा कि सन् 1977 में राष्ट्रपति श्री फखरुद्दीन अली अहमद ने प्रधानमंत्री इंदिरा गांधी की सलाह पर लोकसभा को भंग किया था। राष्ट्रपति को प्रधानमंत्री की सलाह को मानना पड़ेगा, ऐसा प्रावधान संविधान के 42 संशोधन में किया गया था।

इसलिए वाजपेयी जी के नेतृत्व वाली गठबंधन सरकार ने 17 अप्रैल, 1999 को लोकसभा में विश्वास प्रस्ताव पास न करवा पाने के कारण उत्पन्न स्थिति में राष्ट्रपति को लोकसभा भंग करने की सलाह दे दी। परिणामस्वरूप 12वीं लोकसभा 26 अप्रैल, 1999 को राष्ट्रपति द्वारा भंग कर दी गई थी। इसी तरह के परामर्श द्वारा समय पूर्व अटल जी ने 13वीं लोकसभा को 6 फरवरी, 2004 को भंग करवा दिया गया था।

11. प्रधानमंत्री की आपातकालीन शक्तियाँ (Emergency Powers of the Prime Minister):
भारतीय संविधान के अंतर्गत अनुच्छेद 352, 356, 360 के द्वारा भारत के राष्ट्रपति को तीन प्रकार की संकटकालीन शक्तियाँ दी गई हैं, लेकिन वास्तविक रूप में राष्ट्रपति इन शक्तियों का प्रयोग प्रधानमंत्री की सलाह के अनुसार ही करता है; जैसे अक्तूबर, 1962 में चीन के आक्रमण के समय, 3 दिसंबर, 1971 को पाकिस्तान के आक्रमण के समय तथा 26 जून, 1975 को आंतरिक व्यवस्था के खराब होने पर अनुच्छेद 352 के अंतर्गत राष्ट्रपति ने प्रधानमंत्री की सलाह से ही आपातकालीन स्थिति की घोषणा की थी।

इसी प्रकार अनुच्छेद 356 के अंतर्गत राज्यों में राष्ट्रपति शासन भी प्रधानमंत्री की सलाह के अनुसार लगाया जाता है। 44वें संशोधन के अनुसार, राष्ट्रपति अनुच्छेद 352 के अंतर्गत संकटकाल की घोषणा तभी कर सकता है, यदि मंत्रिमंडल संकटकाल की घोषणा करने की लिखित सलाह दे। अप्रैल, 1977 में कार्यवाहक राष्ट्रपति श्री बी०डी० जत्ती ने प्रधानमंत्री की सलाह पर नौ विधानसभाओं को भंग किया था।

प्रधानमन्त्री की स्थिति (Position of Prime Minister) संविधान तथा व्यवहार में प्राप्त शक्तियों व अधिकारों के आधार पर प्रधानमन्त्री देश का सर्वाधिक शक्तिशाली अधिकारी है जो राष्ट्र, संसद व जनता का नेता है जिसकी इच्छा, विचारधारा सर्वाधिक महत्व रखती है इसलिए संविधान सभा में डॉ० अम्बेडकर ने कहा भी था, “यदि हमारे देश में किसी अधिकारी की तुलना अमेरिकी राष्ट्रपति से की जा सकती है तो वह प्रधानमन्त्री की है न कि राष्ट्रपति की।”

इसी प्रकार से संविधान सभा में प्रधानमन्त्री की शक्तियों व अधिकारों को देखते हुए श्री के०टी० शाह ने डर व आशंका व्यक्त की थी, “यदि वह चाहे तो किसी भी समय देश का तानाशाह बन सकता है।” केन्टी० शाह का यह डर 1975-77 की आन्तरिक आपास्थिति ने सत्य भी सिद्ध कर दिया था, लेकिन इसे अपवाद ही कहा जा सकता है अन्यथा प्रधानमन्त्री के पास कितने भी अधिकार क्यों न हों, उसमें तानाशाह बनने की सम्भावना कम ही है, क्योंकि उसकी स्थिति अनेक बातों पर निर्भर करती है। उस पर अनेक निम्नलिखित प्रतिबन्ध हैं

1. दल में स्थिति (Position in Party):
प्रधानमन्त्री की स्थिति मुख्यतया इस बात पर निर्भर करती है कि उसकी अपने दल में क्या स्थिति है? यदि उसका अपने दल पर नियन्त्रण है तो उसकी स्थिति बहुत मजबूत होती है जैसे नेहरू, इन्दिरा गाँधी तथा राजीव गाँधी की थी। यदि उसका अपने दल पर नियन्त्रण कम है तो उसकी स्थिति कमजोर होती है, जैसे मोरारजी देसाई, वी०पी० सिंह की थी।

उदाहरणार्थ जवाहरलाल नेहरू, इन्दिरा गाँधी या राजीव गाँधी ने जिसे चाहा अपने मन्त्रिमण्डल में लिया और जब चाहा निकाल दिया। इसके विपरीत मोरारजी देसाई को अपने मन्त्रिमण्डल में वाजपेयी को विदेश मन्त्री, चरणसिंह को गृहमन्त्री के रूप में लेने के लिए मजबूर होना पड़ा था। कार्यपालिका

2. दल की संसद में स्थिति (Position of Party in Parliament):
प्रधानमन्त्री की स्थिति इस बात पर भी निर्भर करती है कि उसके दल की संसद में क्या स्थिति है? यदि उसके दल को संसद में भारी भरकम बहुमत प्राप्त है तो उसकी स्थिति बहुत मजबूत हो जाती है जैसे नेहरू जी की थी या 1971 के बाद इन्दिरा गाँधी की हो गई थी और बाद में राजीव गाँधी की थी कि जब काँग्रेस को लोकसभा में दो तिहाई या उससे अधिक भी अधिक स्थान प्राप्त थे, लेकिन वी०पी० सिंह, चन्द्रशेखर की स्थिति कमजोर थी, क्योंकि इनके दलों को लोकसभा में स्पष्ट बहुमत भी प्राप्त नहीं था और अन्य दलों पर निर्भर करते थे।

3. जनता में लोकप्रियता (Popularity in Public):
प्रधानमन्त्री की स्थिति इस बात पर भी निर्भर करती है कि वह जनता में कितना लोकप्रिय है? यदि वह जनता में अति लोकप्रिय है तो उसकी स्थिति बहुत मजबूत होती है? जैसे नेहरू, इन्दिरा गाँधी अथवा राजीव गाँधी की थी। यदि वह जनता में बहुत लोकप्रिय नहीं है तो उसकी स्थिति कमजोर होती है, जैसे मोरारजी देसाई, चरणसिंह, वी०पी० सिंह या चन्द्रशेखर की थी।

4. व्यक्तित्व (Personality):
प्रधानमन्त्री का व्यक्तित्व भी उसकी स्थिति निर्धारित करता है। करिश्माई व्यक्तित्व वाले प्रधानमन्त्री जवाहरलाल नेहरू, इन्दिरा गाँधी, राजीव गाँधी अपने व्यक्तित्व के कारण मज़बूत प्रधानमन्त्री सिद्ध हुए, जबकि मोरारजी देसाई, चरणसिंह, वी०पी० सिंह, चन्द्रशेखर का व्यक्तित्व करिश्माई नहीं था, अतः उनकी स्थिति कभी भी मज़बूत नहीं रही।

5. निष्कर्ष (Conclusion):
इस प्रकार प्रधानमन्त्री की स्थिति अनेक बातों पर निर्भर करती है और कोई भी प्रधानमन्त्री अब तानाशाह नहीं बन सकता है। 1977 में इन्दिरा गाँधी की करारी हार ने सिद्ध कर दिया था कि भारतीय जनमत जागरूक है और वह किसी भी करिश्माई व्यक्तित्व वाले व्यक्ति की तानाशाही को सहन करने के लिए तैयार नहीं हैं। इस प्रकार प्रधानमन्त्री पर उसके दल, संसद व जनमत का दबाव रहता है। उसकी स्थिति ‘समकक्षों में प्रथम’ (First Among Equals) की है, तानाशाह की नहीं।

HBSE 11th Class Political Science Important Questions Chapter 4 कार्यपालिका

प्रश्न 11.
राष्ट्रपति के मन्त्रिपरिषद् तथा संसद के साथ सम्बन्धों की व्याख्या कीजिए।
उत्तर:
भारतीय संविधान संसदात्मक व्यवस्था की स्थापना करता है। इस कारण राष्ट्रपति का मन्त्रि-परिषद् व संसद से प्रत्यक्ष एवं घनिष्ठ सम्बन्ध है। राष्ट्रपति तथा मन्त्रिपरिषद् में सम्बन्ध (Relationship Between President and the Council of Ministers) राष्ट्रपति तथा मन्त्रि-परिषद् में निम्नलिखित प्रकार से सम्बन्ध पाए जाते हैं

1. राष्ट्रपति तथा मन्त्रिपरिषद् का गठन (President and Composition of Council of Ministers):
राष्ट्रपति प्रधानमन्त्री की नियक्ति करता है, लेकिन राष्ट्रपति का यह अधिकार औपचारिकता मात्र है, क्योंकि राष्ट्रपति को उसी व्यक्ति को प्रधानमन्त्री बनाना पड़ता है जिसे लोकसभा का विश्वास अर्थात् बहुमत का समर्थन प्राप्त होता है, लेकिन जब किसी भी प्रधानमन्त्री पद के दावेदार को स्पष्ट बहुमत प्राप्त न हो तो राष्ट्रपति अपने स्वविवेक का प्रयोग कर सकता है,

जैसे 1979 में मोरारजी देसाई और चरणसिंह में से राष्ट्रपति ने चरणसिंह को प्रधानमन्त्री बनाया था और 1990 में चन्द्रशेखर को प्रधानमन्त्री बनाया था। अन्य मन्त्रियों की नियुक्ति राष्ट्रपति प्रधानमन्त्री के परामर्श से करता है। राष्ट्रपति को उन व्यक्तियों को भी प्रधानमन्त्री की इच्छानुसार मन्त्री नियुक्त करना पड़ता है जिन्हें वो व्यक्तिगत रूप से भले ही नापसन्द करता हो। मन्त्रियों के विभागों का बंटवारा भी राष्ट्रपति प्रधानमन्त्री की सलाह के अनुरूप करता है।

2. राष्ट्रपति तथा मन्त्रि-परिषद् का विघटन (President and Dismissal of the Council of Ministers):
संविधान के अनुच्छेद 75 के अनुसार, “मन्त्री अपने पद पर राष्ट्रपति के प्रसाद-पर्यन्त रहते हैं” अर्थात् जब तक राष्ट्रपति चाहे तभी तक मन्त्री अपने पद पर रह सकते हैं, लेकिन संसदात्मक व्यवस्था होने के नाते राष्ट्रपति के इस अधिकार का प्रयोग वास्तव में प्रधानमन्त्री करता  हैं, न कि राष्ट्रपति । व्यवहार में राष्ट्रपति उस प्रधानमन्त्री को उसके पद से नहीं हटा सकता जिसे लोकसभा का विश्वास प्राप्त है।

इसी प्रकार से अपनी इच्छा से वह किसी मन्त्री को भी नहीं हटा सकता यदि उस मन्त्री को प्रधानमन्त्री का समर्थन प्राप्त है, लेकिन कभी-कभी ऐसी परिस्थिति भी उत्पन्न हो जाती है जबकि प्रधानमन्त्री को राष्ट्रपति की इच्छा को देखते हुए किसी मन्त्री से त्यागपत्र देने के लिए कहना पड़ता है, जैसे 1987 में प्रधानमन्त्री राजीव गाँधी को राष्ट्रपति ज्ञानी जैल सिंह की इच्छा को देखते हुए श्री के०के० तिवारी से मन्त्री पद त्यागने के लिए कहना पड़ा था।

3. राष्ट्रपति तथा मन्त्रि-परिषद् की सलाह (President and Advice of the Council of Ministers):
संविधान के अनुच्छेद 74 में कहा गया है कि राष्ट्रपति की सहायता एवं सलाह के लिए एक मन्त्रि-परिषद् होगी। संसदात्मक व्यवस्था होने के कारण राष्ट्रपति मन्त्रि-परिषद् द्वारा दी गई सलाह के अनुसार कार्य करता है। 42वें तथा 44वें संविधान संशोधनों के द्वारा यह स्पष्ट कर दिया गया है कि राष्ट्रपति मन्त्रि-परिषद् को दी गई सलाह पर पुनर्विचार करने के लिए कह सकता है और पुनर्विचार के बाद दी गई सलाह को मानने के लिए राष्ट्रपति बाध्य है।

4. प्रशासन के लिए राष्ट्रपति नहीं, मन्त्रिपरिषद् उत्तरदायी (The Council of Ministers, not the President is Responsible for Administration):
यद्यपि संविधान में समस्त कार्यपालिका शक्तियाँ राष्ट्रपति में निहित हैं, लेकिन देश के प्रशासन के लिए राष्ट्रपति उत्तरदायी नहीं है। देश के प्रशासन के लिए संसदात्मक व्यवस्था होने के नाते मन्त्रि-परिषद् उत्तरदायी है जिस कारण राष्ट्रपति की कार्यपालिका शक्तियों का प्रयोग वास्तव में मन्त्रि-परिषद् करती है।

5. राष्ट्रपति का मन्त्रि-परिषद् को सलाह देने, चेतावनी देने व उत्साहित करने का अधिकार (Presidential Right to Advice, Encourage and Warn the Council of Ministers):
राष्ट्रपति ब्रिटिश राज्य की भांति मन्त्रि-परिषद् को सलाह दे सकता है, उत्साहित कर सकता है और जो मन्त्रि-परिषद् उसकी सलाह की अनदेखी करे, उसे चेतावनी दे सकता है, लेकिन महत्त्वपूर्ण तथ्य यह है कि वह मन्त्रिपरिषद् को भंग नहीं कर सकता।

6. राष्ट्रपति का सूचना पाने का अधिकार (Presidential Right to be Informed):
राष्ट्रपति प्रधानन्त्री से देश के शासन व प्रशासन से सम्बन्धित सूचनाएँ माँग सकता है। वास्तव में राष्ट्रपति का सूचना पाने का अधिकार ही ऐसा अधिकार है जो राष्ट्रपति को उसके कर्तव्यों का वहन करने में सक्षम बनाता है।

राष्ट्रपति तथा संसद में सम्बन्ध (Relationship between the President and the Parliament):
संसदात्मक व्यवस्था के अन्तर्गत राष्ट्रपति तथा संसद में घनिष्ठ सम्बन्ध पाया जाता है। संविधान के अनुच्छेद 79 के अनुसार, “संघ की एक संसद होगी जो राष्ट्रपति तथा दोनों सदनों लोकसभा व राज्य सभा से मिलकर बनेगी।” इस प्रकार संसद से अभिप्राय राष्ट्रपति सहित संसद से है। यद्यपि राष्ट्रपति लोकसभा अथवा राज्यसभा का सदस्य नहीं हो सकता, लेकिन राष्ट्रपति का संसद से निम्नलिखित प्रकार से घनिष्ठ सम्बन्ध है ।

1. राष्ट्रपति का निर्वाचन (Election of the President):
राष्ट्रपति का चुनाव एक निर्वाचन मण्डल करता है जिसमें लोकसभा तथा राज्यसभा के निर्वाचित सदस्य भाग लेते हैं।

2. राष्ट्रपति को महाभियोग द्वारा हटाना (Impeachment of the President):
राष्ट्रपति को संविधान की रक्षा न कर पाने के आरोप में महाभियोग द्वारा हटाने का एकमात्र अधिकार संसद को प्राप्त है।

3. संसद में सदस्यों को मनोनीत करना (Nomination of Members in the Parliament):
भारतीय संविधान राष्ट्रपति को राज्यसभा में ऐसे 12 सदस्यों को मनोनीत करने का अधिकार देता है जिनका समाज सेवा या साहित्य के किसी क्षेत्र में नाम हो। राष्ट्रपति लोकसभा में भी स्वेच्छा से दो एंग्लो-इण्डियन जाति के सदस्यों को मनोनीत कर सकता था। यहाँ यह उल्लेखनीय है कि दिसम्बर, 2019 में पारित 104वें संवैधानिक संशोधन द्वारा एंग्लो-इण्डियन जाति की मनोनयन प्रणाली को समाप्त करने का निर्णय किया गया।

4. संसद का सत्र बुलाना और सत्रावसान करना (To Call the Session and Adjournment Sine Die):
राष्ट्रपति ही संसद के सत्रों को समय-समय पर बुलाता है। वह संसद के अधिवेशनों का समय बढ़ा सकता है, स्थगित कर सकता है और सत्रावसान करता है। राष्ट्रपति ही संसद के दोनों सदनों का संयुक्त अधिवेशन बुलाता है।

5. संसद को सम्बोधित करना (ToAddress the Parliament):
राष्ट्रपति संसद के दोनों सदनों को अलग-अलग अथवा संयुक्त रूप से सम्बोधित कर सकता है। नई संसद का पहला तथा वर्ष का पहला अधिवेशन राष्ट्रपति के अभिभाषण से ही प्रारम्भ होता है।

6. सन्देश भेजना (To Send Message):
संविधान के अन्तर्गत राष्ट्रपति संसद में किसी भी सदन को सन्देश भेज सकता है और सदन उस सन्देश पर शीघ्र ही विचार करता है।

7. अनेक आयोगों की रिपोर्ट संसद में पेश करना (To Put the Reports of the Various Commissions Before the Parliament):
राष्ट्रपति अनेक आयोगों जैसे वित्त आयोग की रिपोर्ट संसद में पेश करता है। ये आयोग अपनी रिपोर्ट राष्ट्रपति को . प्रस्तुत करते हैं और राष्ट्रपति इन रिपोर्टों को संसद में मन्त्रियों के द्वारा प्रस्तुत करता है।

8. राष्ट्रपति की पूर्व अनुमति (Prior permission of the President):
कुछ विधेयक जैसे धन विधेयक, नए राज्यों को बनाने, वर्तमान राज्यों के नाम व सीमा में परिवर्तन सम्बन्धी विधेयक राष्ट्रपति की पूर्व अनुमति से ही संसद में प्रस्तावित किए जा सकते हैं।

9. विधेयकों पर स्वीकृति (Assent on the Bills):
संसद द्वारा पारित विधेयक स्वीकृति के लिए राष्ट्रपति के पास भेजे जाते हैं। जब तक राष्ट्रपति हस्ताक्षर नहीं कर देता, तब तक विधेयक कानून का रूप नहीं ले पाता है।

10. अध्यादेश जारी करना (To Issue Ordinances):
जब संसद का अधिवेशन न चल रहा हो और किसी कानून की तुरन्त जरूरत हो तो राष्ट्रपति अध्यादेश जारी करता है। यह अस्थायी प्रबन्ध है।

11. लोकसभा भंग करना (To Dissolve the Lok Sabha):
राष्ट्रपति प्रधानमन्त्री की सलाह पर लोकसभा को भंग भी कर सकता है। इस प्रकार राष्ट्रपति के मन्त्रिपरिषद् व संसद से घनिष्ठ सम्बन्ध होते हैं।

वस्तुनिष्ठ प्रश्न

निम्नलिखित प्रश्नों का उत्तर दिए गए विकल्पों में से उचित विकल्प छाँटकर लिखें

1. अध्यक्षात्मक कार्यपालिका में वास्तविक शक्तियाँ किसके पास हैं?
(A) राष्ट्रपति
(B) प्रधानमंत्री
(C) मंत्रिमंडल
(D) जनता
उत्तर:
(A) राष्ट्रपति

2. भारत में संसदीय शासन प्रणाली में निम्नलिखित दोष है
(A) संगठित विरोधी दल का अभाव
(B) संसद सदस्यों की निष्क्रिय भूमिका
(C) प्रधानमंत्री की प्रमुख स्थिति
(D) उपर्युक्त सभी
उत्तर:
(D) उपर्युक्त सभी

3. भारत की समस्त सेनाओं का सर्वोच्च सेनापति है
(A) प्रधानमंत्री
(B) रक्षामंत्री
(C) राष्ट्रपति
(D) उप-राष्ट्रपति
उत्तर:
(C) राष्ट्रपति

4. निम्नलिखित में से किस राज्य में इकहरी (Singular) कार्यपालिका है?
(A) स्विट्जरलैंड
(B) रूस
(C) अमेरिका
(D) इनमें से कोई नहीं
उत्तर:
(C) अमेरिका

5. राष्ट्रपति का चुनाव लड़ने के लिए उम्मीदवार का नाम कितने निर्वाचकों द्वारा प्रस्तावित होना आवश्यक है?
(A) 40
(B) 50
(C) 30
(D) 60
उत्तर:
(B) 50

6. राष्ट्रपति को उसके पद से हटाया जा सकता है?
(A) साधारण मुकद्दमे द्वारा
(B) महाभियोग द्वारा
(C) सर्वोच्च न्यायालय द्वारा
(D) प्रधानमंत्री द्वारा
उत्तर:
(B) महाभियोग द्वारा

7. उप-राष्ट्रपति का चुनाव किया जाता है
(A) जनता द्वारा
(B) राष्ट्रपति द्वारा
(C) संसद के दोनों सदनों के सदस्यों द्वारा
(D) राज्य विधानसभाओं के सदस्यों द्वारा
उत्तर:
(C) संसद के दोनों सदनों के सदस्यों द्वारा

8. भारत का राष्ट्रपति चुना जाता है
(A) निर्वाचक मंडल द्वारा
(B) संसद द्वारा
(C) जनता द्वारा
(D) सभी राज्य विधानसभाओं द्वारा
उत्तर:
(A) निर्वाचक मंडल द्वारा

9. भारत के वर्तमान राष्ट्रपति हैं
(A) श्री प्रणब मुखर्जी
(B) श्री रामनाथ कोविंद
(C) श्रीमती प्रतिभा पाटिल
(D) इनमें से कोई नहीं
उत्तर:
(B) श्री रामनाथ कोविंद

HBSE 11th Class Political Science Important Questions Chapter 4 कार्यपालिका

10. राष्ट्रपति को पद की शपथ दिलाता है
(A) उप-राष्ट्रपति
(B) प्रधानमन्त्री
(C) संसद
(D) सर्वोच्च न्यायालय का मुख्य न्यायाधीश
उत्तर:
(D) सर्वोच्च न्यायालय का मुख्य न्यायाधीश

11. यदि राष्ट्रपति और उप-राष्ट्रपति दोनों के पद रिक्त हों तो कार्यवाहक राष्ट्रपति के रूप में कार्य करेगा
(A) दिल्ली उच्च न्यायालय का मुख्य न्यायाधीश
(B) लोकसभा का अध्यक्ष
(C) भारत के सर्वोच्च न्यायालय का मुख्य न्यायाधीश
(D) इनमें से कोई नहीं
उत्तर:
(C) भारत के सर्वोच्च न्यायालय का मुख्य न्यायाधीश

12. राष्ट्रपति के पास निम्नलिखित परिस्थिति में संकटकालीन घोषणा करने का अधिकार है
(A) विदेशी आक्रमण अथवा सशस्त्र क्रान्ति के समय
(B) जब देश की वित्तीय साख को खतरा हो
(C) किसी राज्य में संवैधानिक मशीनरी के विफल होने
(D) उपर्युक्त सभी की स्थिति में
उत्तर:
(D) उपर्युक्त सभी

13. राष्ट्रपति का पद खाली होने पर कार्यवाहक राष्ट्रपति (Acting President) के रूप में कौन कार्य करता है?
(A) सर्वोच्च न्यायालय का मुख्य न्यायाधीश
(B) लोकसभा का अध्यक्ष
(C) उप-राष्ट्रपति
(D) प्रधानमन्त्री
उत्तर:
(C) उप-राष्ट्रपति

14. निम्नलिखित में से उप-राष्ट्रपति का कार्य है
(A) लोकसभा की बैठकों की अध्यक्षता करना
(B) राज्यसभा की बैठकों की अध्यक्षता करना
(C) प्रधानमन्त्री को पद से हटाना
(D) संसद को भंग करना
उत्तर:
(B) राज्यसभा की बैठकों की अध्यक्षता करना

15. भारत के उप-राष्ट्रपति के चुनाव हेतु न्यूनतम आयु होनी चाहिए.
(A) 32 वर्ष
(B) 35 वर्ष
(C) 25 वर्ष
(D) 18 वर्ष
उत्तर:
(B) 35 वर्ष

16. भारत का राष्ट्रपति अपना त्याग-पत्र निम्नलिखित में से किसे सौंपता है?
(A) लोकसभा अध्यक्ष
(B) भारत के मुख्य न्यायाधीश
(C) उप-राष्ट्रपति
(D) प्रधानमन्त्री
उत्तर:
(C) उप-राष्ट्रपति

17. वित्त आयोग की नियुक्ति की जाती है?
(A) प्रधानमंत्री द्वारा
(B) उप-राष्ट्रपति द्वारा
(C) राष्ट्रपति द्वारा
(D) मंत्रिपरिषद् द्वारा
उत्तर:
(C) राष्ट्रपति द्वारा

18. मंत्रि-परिषद् अपने कार्यों व कार्यकाल के लिए सामूहिक रूप से निम्नलिखित में से किस एक के प्रति उत्तरदायी है?
(A) राष्ट्रपति
(B) प्रधानमंत्री
(C) लोकसभा
(D) राज्यसभा
उत्तर:
(C) लोकसभा

19. निम्नलिखित संशोधन के अनुसार यह व्यवस्था की गई है कि राष्ट्रपति मंत्रिमंडल के सुझाव को पुनर्विचार के लिए भेज सकता है
(A) 45वें
(B) 43वें
(C) 42वें
(D) 44वें
उत्तर:
(D) 44वें

20. मंत्रिमंडल उत्तरदायी है
(A) संसद के प्रति
(B) प्रधानमंत्री के प्रति
(C) राष्ट्रपति के प्रति
(D) स्पीकर के प्रति
उत्तर:
(A) संसद के प्रति

21. भारत की संघीय मंत्रि-परिषद् में निम्नलिखित में से कौन शामिल होते हैं?
(A) राष्ट्रपति तथा उप-राष्ट्रपति
(B) प्रधानमंत्री तथा अन्य मंत्री
(C) राष्ट्रपति, उपराष्ट्रपति तथा प्रधानमंत्री
(D) इनमें से कोई नहीं
उत्तर:
(B) प्रधानमंत्री तथा अन्य मंत्री

22. संघीय मंत्रिमंडल का नेता है
(A) राष्ट्रपति
(B) प्रधानमंत्री
(C) उप-राष्ट्रपति
(D) लोकसभा अध्यक्ष
उत्तर:
(B) प्रधानमंत्री

23. प्रधानमंत्री को बहुमत का विश्वास प्राप्त होना चाहिए
(A) लोकसभा में
(B) राज्यसभा में
(C) राज्य विधान परिषद् में
(D) इनमें से कोई नहीं
उत्तर:
(A) लोकसभा में

24. प्रधानमंत्री का कार्यकाल कितना है?
(A) 5 वर्ष
(B) 6 वर्ष
(C) 4 वर्ष
(D) लोकसभा के बहुमत के समर्थन पर निर्भर है
उत्तर:
(D) लोकसभा के बहुमत के समर्थन पर निर्भर है

25. वर्तमान में भारत के प्रधानमंत्री निम्नलिखित में से कौन हैं?
(A) श्री अमित शाह
(B) श्री राजनाथ सिंह
(C) श्री नरेन्द्र मोदी
(D) डॉ० मनमोहन सिंह
उत्तर:
(C) श्री नरेन्द्र मोदी

26. निम्नलिखित में से राज्य की कार्यपालिका का कौन अध्यक्ष होता है?
(A) राष्ट्रपति
(B) उप-राष्ट्रपति
(C) प्रधानमंत्री
(D) राज्यपाल
उत्तर:
(D) राज्यपाल

27. राज्य के राज्यपाल को मासिक वेतन मिलता है
(A) 80,000 रुपए
(B) 1,10,000 रुपए
(C) 1,25,000 रुपए
(D) 3,50,000 रुपए
उत्तर:
(D) 3,50,000 रुपए

28. वर्तमान में हरियाणा के राज्यपाल निम्नलिखित में से कौन हैं?
(A) श्री जगन्नाथ पहाड़िया
(B) श्री सत्यदेव नारायण आर्य
(C) श्री कप्तान सिंह सोलंकी
(D) इनमें से कोई नहीं
उत्तर:
(B) श्री सत्यदेव नारायण आर्य

निम्नलिखित प्रश्नों का उत्तर एक शब्द में दें

1. कार्यपालिका अध्यक्ष की नियुक्ति के कोई दो तरीके बताइए।
उत्तर:

  • जनता द्वारा प्रत्यक्ष चुनाव,
  • संसद अथवा निर्वाचन-मंडल द्वारा चुनाव।

2. 17वीं लोकसभा में विरोधी दल के नेता कौन हैं?
उत्तर:
कोई भी नहीं।

3. भारत में कौन-सी शासन प्रणाली अपनाई गई है?
उत्तर:
संसदीय शासन प्रणाली।

4. भारत के प्रथम व वर्तमान राष्ट्रपति का नाम लिखें।
उत्तर:
भारत के प्रथम राष्ट्रपति डॉ० राजेन्द्र प्रसाद थे तथा वर्तमान राष्ट्रपति श्री रामनाथ कोविंद हैं।

5. भारत के उप-राष्ट्रपति का चुनाव कौन करता है?
उत्तर:
भारत के उप-राष्ट्रपति का चुनाव संसद के दोनों सदनों द्वारा किया जाता है।

6. भारत के वर्तमान उप-राष्ट्रपति का नाम लिखें।
उत्तर:
भारत के वर्तमान उप-राष्ट्रपति श्री एम० बैंकेया नायडू हैं।

रिक्त स्थान भरें

1. संघीय कार्यपालिका का वास्तविक अध्यक्ष ……… है।
उत्तर:
प्रधानमन्त्री

2. संघीय कार्यपालिका का नाममात्र संवैधानिक अध्यक्ष ………… है।
उत्तर:
राष्ट्रपति

3. राज्य का संवैधानिक कार्यपालिका अध्यक्ष ………….. होता है।
उत्तर:
राज्यपाल

4. वर्तमान में हरियाणा के राज्यपाल …………… एवं मुख्यमंत्री ……………. है।
उत्तर:
श्री सत्यदेव नारायण आर्य एवं श्री मनोहर लाल खट्टर

5. वर्तमान में भारत के राष्ट्रपति ………….. हैं।
उत्तर:
श्री रामनाथ कोविंद

6. वर्तमान में भारत के उप-राष्ट्रपति ………….. हैं।
उत्तर:
श्री वैंकेया नायडू

7. राष्ट्रपति का मासिक वेतन ………….. रुपए हैं।
उत्तर:
5 लाख

8. उप-राष्ट्रपति को मासिक वेतन के रूप में ……………. मिलता है।
उत्तर:
कोई वेतन नहीं

9. राज्यपाल का मासिक वेतन …………. रुपए है।
उत्तर:
3.50 लाख

HBSE 11th Class Political Science Important Questions Chapter 4 कार्यपालिका

10. राष्ट्रपति को ………….. के द्वारा शपथ दिलाई जाती है।
उत्तर:
मुख्य न्यायाधीश

11. राष्ट्रपति को अनुच्छेद ………… के अन्तर्गत राष्ट्रीय आपातकाल की घोषणा की शक्ति प्राप्त है।
उत्तर:
352

12. राष्ट्रपति को भारत की ………. में से वेतन दिया जाता है।
उत्तर:
संचित निधि

13. उप-राष्ट्रपति ………… की कार्यवाही का संचालन करता है।
उत्तर:
राज्यसभा

14. केन्द्रीय मंत्री परिषद् अपने कार्यों के लिए सामूहिक रूप से …………. के प्रति उत्तरदायी होती है।
उत्तर:
लोकसभा

15. राष्ट्रपति के चुनाव हेतु न्यूनतम आयु ………. होनी चाहिए।
उत्तर:
35 वर्ष

16. भारत में ………… शासन-प्रणाली अपनाई गई है।
उत्तर:
संसदीय

HBSE 11th Class Political Science Important Questions Chapter 3 चुनाव और प्रतिनिधित्व

Haryana State Board HBSE 11th Class Political Science Important Questions Chapter 3 चुनाव और प्रतिनिधित्व Important Questions and Answers.

Haryana Board 11th Class Political Science Important Questions Chapter 3 चुनाव और प्रतिनिधित्व

अति लघूत्तरात्मक प्रश्न

प्रश्न 1.
मताधिकार का क्या अर्थ है?
उत्तर:
एक देश के नागरिकों को अपने प्रतिनिधियों को चुनने के अधिकार को मताधिकार कहा जाता है। भारत में प्रत्येक वयस्क नागरिक को मतदान करने का अधिकार दिया गया है।

प्रश्न 2.
निर्वाचकगण (Electorate) किसे कहते हैं?
उत्तर:
वे व्यक्ति जिन्हें मतदान करने का अधिकार होता है, उनके सामूहिक रूप को निर्वाचकगण कहा जाता है।

प्रश्न 3.
प्रत्यक्ष चुनाव-प्रणाली का क्या अर्थ है?
उत्तर:
जिस चुनाव-प्रणाली में साधारण मतदाता स्वयं अपने प्रतिनिधियों का चुनाव करते हैं, उसे प्रत्यक्ष चुनाव-प्रणाली कहा जाता है। लोकसभा के सदस्यों का चुनाव इसी प्रणाली द्वारा किया जाता है।

प्रश्न 4.
अप्रत्यक्ष चुनाव-प्रणाली का क्या अर्थ है?
उत्तर:
ऐसे प्रतिनिधि जो जनता द्वारा प्रत्यक्ष रूप से न चुने जाकर जनता के चुने हुए प्रतिनिधियों द्वारा चुने जाते हों, उस प्रणाली को अप्रत्यक्ष चुनाव-प्रणाली कहा जाता है। राष्ट्रपति तथा उप-राष्ट्रपति के चुनाव इस प्रणाली द्वारा कराए जाते हैं।

प्रश्न 5.
प्रत्यक्ष चुनाव-प्रणाली के दो गुण लिखिए।
उत्तर:

  • यह लोकतान्त्रिक सिद्धांतों के अनुसार है,
  • लोगों को राजनीतिक शिक्षा प्राप्त होती है।

प्रश्न 6.
अप्रत्यक्ष चुनाव-प्रणाली के दो दोष बताइए।
उत्तर:

  • यह अलोकतांत्रिक है,
  • इसमें भ्रष्टाचार की संभावना होती है।

प्रश्न 7.
वयस्क मताधिकार से क्या तात्पर्य है?
उत्तर:
एक विशेष आयु प्राप्त करने वाले व्यक्तियों को यदि वोट का अधिकार दिया जाए तो उसे वयस्क मताधिकार कहते हैं। वोट देने की आयु विभिन्न देशों में भिन्न-भिन्न होती है। भारत में मतदान की आयु 18 वर्ष निश्चित की गई है।

HBSE 11th Class Political Science Important Questions Chapter 3 चुनाव और प्रतिनिधित्व

प्रश्न 8.
वयस्क मताधिकार के कोई दो लाभ बताइए।
उत्तर:

  • यह समानता पर आधारित है,
  • इसमें सभी को राजनीतिक शिक्षा प्राप्त होती है।

प्रश्न 9.
वयस्क मताधिकार के दो दोष बताइए।
उत्तर:

  • वयस्क मताधिकार गुणों की अपेक्षा संख्या को महत्त्व देता है और अशिक्षित तथा अज्ञानियों का, जिनकी संख्या अधिक है, शासन स्थापित हो जाता है,
  • यह अप्राकृतिक है क्योंकि प्रकृति ने सबको समान न बनाकर कुछ को बुद्धिमान तथा कुछ को कम बुद्धिमान बनाया है।

प्रश्न 10.
एक-सदस्यीय निर्वाचन क्षेत्र से क्या तात्पर्य है?
उत्तर:
एक-सदस्यीय निर्वाचन-क्षेत्र का अर्थ है कि एक निर्वाचन-क्षेत्र से एक ही प्रतिनिधि चुना जाएगा।

प्रश्न 11.
एक-सदस्यीय निर्वाचन-क्षेत्र के दो गुण बताइए।
उत्तर:

  • यह तरीका बड़ा सरल है और आम व्यक्ति इसे समझ सकता है,
  • प्रतिनिधि और मतदाता के बीच सीधा संपर्क होता है क्योंकि चुनाव-क्षेत्र छोटा होता है।

प्रश्न 12.
बहु-सदस्यीय निर्वाचन क्षेत्र से क्या तात्पर्य है?
उत्तर:
बह-सदस्यीय निर्वाचन-क्षेत्र से तात्पर्य उस निर्वाचन-क्षेत्र से है, जहाँ से एक से अधिक सदस्य निर्वाचित होते हैं।

प्रश्न 13.
बहु-सदस्यीय निर्वाचन क्षेत्र के दो अवगुण बताइए।
उत्तर:

  • यह खर्चीली प्रणाली है,
  • प्रतिनिधियों व मतदाताओं में संपर्क का अभाव रहता है।

प्रश्न 14.
बहु-सदस्यीय निर्वाचन-क्षेत्र के दो गुण बताइए।
उत्तर:

  • मतदाता की पसंद सीमित नहीं होती और वह अपनी पसंद के प्रतिनिधि का चुनाव कर सकता है,
  • इस प्रकार से एक योग्य प्रतिनिधि का चयन होता है।

प्रश्न 15.
प्रादेशिक चुनाव-प्रणाली का क्या अर्थ है?
उत्तर:
प्रादेशिक चुनाव-प्रणाली में सारे देश को समान प्रादेशिक क्षेत्रों में विभाजित कर दिया जाता है और प्रत्येक चुनाव-क्षेत्र विधानमंडल में अपने प्रतिनिधि को चुनकर भेजता है।

प्रश्न 16.
प्रादेशिक चुनाव-प्रणाली के कोई दो गुण लिखिए।
उत्तर:

  • यह साधारण प्रणाली है,
  • स्थानीय आवश्यकताओं की पूर्ति हो जाती है।

प्रश्न 17.
प्रादेशिक चुनाव क्षेत्र के दो अवगुण लिखिए।
अथवा
प्रादेशिक प्रतिनिधित्व की दो मूलभूत सीमा लिखिए।
उत्तर:

  • क्षेत्रीयवाद को बढ़ावा मिलता है,
  • विभिन्न हितों का प्रतिनिधित्व नहीं होता।

प्रश्न 18.
आनुपातिक प्रतिनिधित्व का अर्थ बताइए।
उत्तर:
आनुपातिक प्रतिनिधित्व से तात्पर्य है कि प्रत्येक वर्ग, राजनीतिक दल व अल्पसंख्यक वर्ग आदि को उनके मतों की संख्या अनुपात में प्रतिनिधित्व प्रदान करना।

प्रश्न 19.
आनुपातिक प्रतिनिधित्व प्रणाली की दो विधियों के नाम बताइए।
उत्तर:

  • इकहरी परिवर्तनीय मत प्रणाली तथा
  • सूची प्रणाली।

प्रश्न 20.
आनुपातिक प्रतिनिधित्व प्रणाली के दो गुण लिखिए।
उत्तर:

  • प्रत्येक वर्ग को उचित प्रतिनिधित्व का मिलना,
  • लोकतांत्रिक सिद्धांतों पर आधारित।

प्रश्न 21.
आनुपातिक प्रतिनिधित्व प्रणाली के दो दोष बताइए।
उत्तर:

  • यह जटिल प्रणाली है,
  • बड़े देशों में लागू नहीं किया जा सकता।

प्रश्न 22.
संग्रहीत मत-प्रणाली (Cumulative Vote System) से क्या तात्पर्य है?
उत्तर:
इस प्रणाली के लिए बहु-सदस्यीय निर्वाचन-क्षेत्र अनिवार्य है। एक मतदाता को उतने वोट दिए जाते हैं, जितने स्थान भरे जाने हैं। मतदाता अपने सारे मत एक उम्मीदवार को दे सकता है और यदि वह चाहे तो अपने मत अलग-अलग उम्मीदवारों को भी दे सकता है।

प्रश्न 23.
क्षेत्रीय (Territorial) और कार्यात्मक प्रतिनिधित्व (Functional Representation) में अन्तर स्पष्ट कीजिए।
उत्तर:

  • क्षेत्रीय प्रतिनिधित्व में क्षेत्रीय आधार पर प्रतिनिधित्व दिया जाता है जबकि कार्यात्मक प्रतिनिधित्व का आधार व्यवसाय होता है,
  • क्षेत्रीय प्रतिनिधित्व में प्रतिनिधि अपने निर्वाचन-क्षेत्र के सभी लोगों का प्रतिनिधित्व करता है, जबकि कार्यात्मक प्रतिनिधित्व व्यवसाय के लोगों का ही प्रतिनिधित्व करता है।

प्रश्न 24.
उप-चुनाव (Bye-Election) से क्या तात्पर्य है?
उत्तर:
विधानपालिका के रिक्त स्थान को भरने के लिए करवाए गए चुनाव को उप-चुनाव कहा जाता है।

प्रश्न 25.
मध्यावधि चुनाव (Mid-Term Election) से क्या तात्पर्य है?
उत्तर:
यदि लोकसभा व विधानसभा को उसके निश्चित कार्यकाल से पहले ही भंग कर दिया जाए तो उस स्थिति में जो चुनाव करवाने पड़ते हैं, उसे मध्यावधि चुनाव कहा जाता है।

प्रश्न 26.
भारत में मतदाता कौन हो सकता है?
उत्तर:
भारत में प्रत्येक नागरिक को, जिसकी आयु 18 वर्ष या इससे अधिक हो, मताधिकार प्राप्त है। चुनाव में उसी नागरिक को मत डालने दिया जाता है जिसका नाम मतदाता सूची में हो।।

प्रश्न 27.
भारतीय मतदाता में कौन-कौन-सी दो योग्यताएँ होनी चाहिएँ?
उत्तर:

  • वह भारत का नागरिक होना चाहिए,
  • उसकी आयु कम-से-कम 18 वर्ष होनी चाहिए।

प्रश्न 28.
चुनाव आयोग क्या है?
उत्तर:
भारत एक लोकतंत्रीय राज्य है जिसमें समय-समय पर चुनाव होते रहते हैं। यह चुनाव निष्पक्ष रूप से हों, इसके लिए संविधान के अनुच्छेद 324 के अंतर्गत चुनाव आयोग के गठन की व्यवस्था की गई है जिसमें एक मुख्य चुनाव आयुक्त तथा कुछ अन्य चुनाव आयुक्त हो सकते हैं।

प्रश्न 29.
चुनाव आयोग के सदस्यों की नियुक्ति कैसे की जाती है?
उत्तर:
संविधान में की गई व्यवस्था के अनुसार मुख्य चुनाव आयुक्त तथा अन्य चुनाव आयुक्तों (वर्तमान स्थिति में दो) की नियुक्ति भारत के राष्ट्रपति द्वारा की जाती है।

प्रश्न 30.
चुनाव आयुक्त को अथवा मुख्य चुनाव आयुक्त को पद से कैसे हटाया जा सकता है?
उत्तर:
चुनाव आयुक्त को संसद में महाभियोग चलाकर उसके पद से हटाया जा सकता है।

प्रश्न 31.
चुनाव आयोग के सदस्यों का कार्यकाल कितना है?
उत्तर:
चुनाव आयोग के आयुक्तों (सदस्यों) का कार्यकाल साधारणतः 6 वर्ष होता है, लेकिन राष्ट्रपति द्वारा इस कार्यकाल को बढ़ाया भी जा सकता है।

प्रश्न 32.
भारतीय चुनाव आयोग के सदस्यों के नाम लिखें।
उत्तर:
भारतीय चुनाव आयोग बहु-सदस्यीय है। इसके मुख्य चुनाव आयुक्त श्री सुशील चंद्रा तथा अन्य दो चुनाव आयुक्त हैं।

प्रश्न 33.
भारतीय चुनाव-प्रणाली की तीन मुख्य विशेषताएँ बताएँ।
उत्तर:

  • वयस्क मताधिकार,
  • एक-सदस्यीय चुनाव-प्रणाली,
  • गुप्त मतदान।

प्रश्न 34.
गुप्त मतदान से क्या तात्पर्य है?
उत्तर:
गुप्त मतदान का अर्थ है कि चुनाव अधिकारियों द्वारा मतदान के समय ऐसा प्रबंध किया जाता है कि यह मालूम न पड़े कि मतदाता ने अपने मताधिकार का प्रयोग किसके पक्ष में किया है या नहीं किया है।

प्रश्न 35.
चुनाव आयोग के कोई दो कार्य लिखिए।
उत्तर:

  • चुनाव आयोग संसद तथा राज्य विधानसभाओं के चुनावों की व्यवस्था करता है,
  • चुनाव आयोग को चुनाव संबंधी सभी मामलों पर निरीक्षण तथा निर्देशन का अधिकार है।

प्रश्न 36.
चुनाव याचिका का वर्णन करें। अथवा भारत में चुनाव-याचिका की सुनवाई कौन करता है?
उत्तर:
चुनावों के समय यदि किसी उम्मीदवार के द्वारा कोई अनियमितता बरती गई है, या कोई उम्मीदवार कानून के विरुद्ध कार्य करके चुनाव जीत गया है, तो उसका चुनाव रद्द करवाने के लिए कोई भी अन्य उम्मीदवार या कोई भी मतदाता याचिका दायर कर सकता है। यह याचिका सीधे उच्च न्यायालय में दी जाती है और वह स्वयं इसे सुनता है। यदि निर्वाचित उम्मीदवार के विरुद्ध लगाए गए आरोप सही सिद्ध हो जाएँ तो उच्च न्यायालय उसका चुनाव रद्द कर सकता है तथा संबंधित आरोपों के आधार पर उसे चुनाव के अयोग्य भी ठहरा सकता है।

प्रश्न 37.
भारतीय संविधान में निर्वाचन क्षेत्रों के आरक्षण के प्रावधान पर टिप्पणी लिखें।
उत्तर:
भारत के संविधान में सांप्रदायिक चुनाव-प्रणाली को समाप्त करके संयुक्त चुनाव-प्रणाली की व्यवस्था की गई है परंतु अनुच्छेद 330 के द्वारा समाज के पिछड़े वर्गों अनुसूचित जातियों (SC) तथा अनुसूचित जनजातियों (ST) के सदस्यों के लिए आरक्षण की व्यवस्था की गई है। इस समय लोकसभा में 84 सीटें अनुसूचित जातियों तथा 47 सीटें अनुसूचित जनजातियों के लिए आरक्षित हैं। 104वें संशोधन द्वारा आरक्षित स्थानों की अवधि सन् 2030 तक बढ़ा दी गई है।

HBSE 11th Class Political Science Important Questions Chapter 3 चुनाव और प्रतिनिधित्व

प्रश्न 38.
भारतीय चुनाव-प्रणाली की चार विशेषताएँ लिखिए।
उत्तर:

  • वयस्क मताधिकार,
  • एक सदस्य निर्वाचन-क्षेत्र,
  • गुप्त मतदान,
  • अनुसूचित जातियों, अनुसूचित जनजातियों तथा अन्य पिछड़े वर्गों के लिए स्थान सुरक्षित रखना।

प्रश्न 39.
भारतीय चुनाव-प्रणाली के दो दोष लिखें।
उत्तर:

  • चुनावों में धन की बढ़ती हुई भूमिका,
  • चुनाव में बाहुबल तथा हिंसा का प्रयोग।

प्रश्न 40.
भारतीय चुनाव-प्रणाली में सुधार के दो सुझाव दीजिए।
उत्तर:

  • चुनाव आयोग को अधिक शक्तिशाली तथा प्रभावी बनाया जाना चाहिए। उन्हें चुनावों में होने वाले भ्रष्टाचार को रोकने के लिए अधिक शक्तियाँ दी जाएँ,
  • सभी मतदाताओं को परिचय-पत्र (Identity Cards) दिए जाने चाहिएँ ताकि गलत (Bogus) मतदान को रोका जा सके।

लघूत्तरात्मक प्रश्न

प्रश्न 1.
प्रत्यक्ष चुनाव-प्रणाली तथा अप्रत्यक्ष चुनाव-प्रणाली में अंतर स्पष्ट कीजिए।
उत्तर:
प्रत्यक्ष चुनाव-प्रणाली साधारण शब्दों में, प्रत्यक्ष चुनाव-प्रणाली उस चुनाव व्यवस्था को कहा जाता है जिसमें साधारण मतदाता प्रत्यक्ष रूप से (स्वयं) अपने प्रतिनिधियों को चुनते हैं। प्रत्येक मतदाता चुनाव-स्थान पर जाकर स्वयं अपनी पसन्द के उम्मीदवार के पक्ष में, अपने मत का प्रयोग करता है और जिस उम्मीदवार को सबसे अधिक मत प्राप्त होते हैं, उसे निर्वाचित घोषित कर दिया जाता है।

भारत में लोकसभा तथा राज्य विधानसभा के सदस्यों का चुनाव प्रत्यक्ष प्रणाली द्वारा किया जाता है। अप्रत्यक्ष चुनाव-प्रणाली-अप्रत्यक्ष चुनाव वह चुनाव व्यवस्था है जिसमें साधारण मतदाता कुछ प्रतिनिधियों (निर्वाचकों) का चुनाव करते हैं और वे निर्वाचक प्रतिनिधि का चुनाव करते हैं। प्रतिनिधि के चुनाव में साधारण मतदाता प्रत्यक्ष रूप से अपने मत का प्रयोग नहीं करते। भारत में राष्ट्रपति तथा उप-राष्ट्रपति के चुनाव के लिए इस चुनाव-प्रणाली को अपनाया जाता है।

प्रश्न 2.
प्रत्यक्ष चुनाव प्रणाली के पक्ष में चार तर्क (लाभ) लिखें।
उत्तर:
प्रत्यक्ष चुनाव प्रणाली के पक्ष में चार तर्क निम्नलिखित हैं

1. अधिक लोकतन्त्रीय प्रणाली-इस प्रणाली में जनता को स्वयं प्रत्यक्ष रूप से मतदान करके अपने प्रतिनिधि चुनने का अवसर मिलता है। अतः यह प्रणाली अधिक लोकतान्त्रिक है।

2. मतदाताओं तथा प्रतिनिधियों में सीधा सम्पर्क-इस प्रणाली में मतदाता तथा उम्मीदवार सीधे रूप से सम्पर्क में आते हैं तथा मतदाता उम्मीदवारों को भली-भांति जान सकते हैं। उम्मीदवार भी अपनी नीतियाँ तथा कार्यक्रम जनता के सामने रख सकते हैं।

3. राजनीतिक शिक्षा-इस प्रणाली में मतदाताओं तथा उम्मीदवारों में सीधा सम्पर्क होता है। इससे मतदाताओं को राजनीतिक शिक्षा मिलती है और उनमें राजनीतिक जागरूकता की भावना का भी उदय होता है।

4. अधिकारों और कर्तव्यों का ज्ञान-इस प्रणाली के अन्तर्गत सामान्य जनता को मताधिकार तथा अन्य अधिकारों का ज्ञान प्राप्त होता है तथा कर्तव्यों की भी जानकारी मिलती है।

प्रश्न 3.
प्रादेशिक प्रतिनिधित्व किसे कहते हैं?
उत्तर:
लोकतन्त्रीय राज्यों में चुनाव के लिए निर्वाचन-क्षेत्रों का गठन भौगोलिक आधार पर किया जाता है। समस्त राज्य को एक-सदस्य अथवा बहु-सदस्य निर्वाचन-क्षेत्रों में बाँट दिया जाता है। एक चुनाव-क्षेत्र में रहने वाले नागरिकों को उस चुनाव-क्षेत्र का निवासी होने के नाते अपने प्रतिनिधि चुनने के अधिकार को ही प्रादेशिक प्रतिनिधित्व कहा जाता है। संक्षेप में, सामान्य प्रतिनिधियों का निर्वाचन जब प्रादेशिक आधार पर हो, तो उस प्रणाली को प्रादेशिक प्रतिनिधित्व कहा जाता है। भारत, इंग्लैण्ड तथा अमेरिका आदि राज्यों में इसी प्रणाली को लागू किया गया है।

प्रश्न 4.
आनुपातिक प्रतिनिधित्व का क्या अर्थ है? इसकी दो पद्धतियों के नाम लिखें।
उत्तर:
आनुपातिक प्रतिनिधित्व का अर्थ है प्रत्येक जाति या वर्ग को उसकी जनसंख्या के अनुपात में संसद या प्रतिनिधि सभा में प्रतिनिधित्व का मिलना। जैकी का कहना है, “अल्पसंख्यकों को उचित प्रतिनिधित्व देने का महत्त्व अतिशय महान है। यदि किसी निर्वाचन-क्षेत्र के दो तिहाई मतदाता एक दल को मत दें और शेष मतदाता किसी दूसरे दल को, तो स्पष्ट है कि बहुसंख्यक वर्ग को दो-तिहाई और अल्पसंख्यक वर्ग को एक-तिहाई प्रतिनिधित्व प्राप्त होना चाहिए।” अर्थात प्रत्येक वर्ग, जाति या दल को उसके समर्थकों के अनुपात के अनुसार प्रतिनिधित्व का मिलना ही आनुपातिक प्रतिनिधित्व कहलाता है।

प्रश्न 5.
आनुपातिक प्रतिनिधित्व के चार गुण बताइए। उत्तर-आनुपातिक प्रतिनिधित्व के चार गुण निम्नलिखित हैं
1. सभी वर्गों को प्रतिनिधित्व इस प्रणाली का यह गुण है कि समाज के सभी वर्गों को उनकी जनसंख्या के अनुपात में विधानमण्डल में प्रतिनिधित्व मिल जाता है। कोई वर्ग प्रतिनिधित्व से वंचित नहीं रहता और सबको संतुष्टि मिलती है।

2. अल्पसंख्यकों में सुरक्षा की भावना-इस प्रणाली का यह गुण है कि सभी वर्गों को उचित प्रतिनिधित्व मिलने से उनके हितों की रक्षा होती है और अल्पसंख्यक वर्ग के लोग स्वयं को सुरक्षित समझते हैं। वे स्वयं को बहुसंख्यक वर्ग की तानाशाही का शिकार नहीं समझते।

3. मतदाताओं को मतदान में अधिक सुविधा इस प्रणाली में मतदाता को अपनी पसन्द के कई उम्मीदवारों के पक्ष में मत डालने का अवसर मिलता है। उसे एक ही व्यक्ति के पक्ष में मत डालने के लिए मजबूर नहीं होना पड़ता। इससे उसे अपनी पसन्द के उम्मीदवार को मत देने में सुविधा हो जाती है।

4. निर्वाचन के लिए निर्धारित मत प्राप्त करना आवश्यक है इस प्रणाली में यह सम्भावना नहीं रहती कि कोई उम्मीदवार थोड़े-से प्रतिशत मत लेकर भी चुन लिया जाएगा। इस प्रणाली में चुने जाने के लिए उम्मीदवार को एक निश्चित संख्या में मत प्राप्त करना आवश्यक होता है अर्थात् चुने जाने के लिए उम्मीदवार को एक निश्चित संख्या के मतदाताओं का समर्थन प्राप्त करना आवश्यक है।

प्रश्न 6.
आनुपातिक प्रतिनिधित्व के चार अवगुण लिखें।
उत्तर:
आनुपातिक प्रतिनिधित्व के चार अवगुण निम्नलिखित हैं

1. जटिल प्रणाली आनुपातिक प्रतिनिधित्व प्रणाली बहुत जटिल है। साधारण मतदाता इसे समझ नहीं सकता।

2. राष्ट्रीय एकता के विरुद्ध आनुपातिक प्रतिनिधित्व प्रणाली के अन्तर्गत छोटे-छोटे दलों को प्रोत्साहन मिलता है। अल्पसंख्यक जातियाँ भी अपनी भिन्नता बनाए रखती हैं और दूसरी जातियों के साथ अपने हितों को मिलाना नहीं चाहतीं। इसके परिणामस्वरूप राष्ट्र छोटे-छोटे वर्गों और गुटों में बँट जाता है और राष्ट्रीय एकता पनप नहीं पाती।

3. राजनीतिक दलों को अधिक महत्त्व आनुपातिक प्रतिनिधित्व की सूची प्रणाली के अन्तर्गत राजनीतिक दलों का महत्त्व बहुत अधिक होता है। मतदाता को किसी-न-किसी दल के पक्ष में वोट डालना होता है क्योंकि इस प्रणाली के अन्तर्गत निर्दलीय उम्मीदवार चुनाव नहीं लड़ सकते।

4. उत्तरदायित्व का अभाव इस प्रणाली के अन्तर्गत निर्वाचन-क्षेत्र बहु-सदस्यीय होते हैं और एक क्षेत्र में कई प्रतिनिधि होते हैं। चूंकि एक क्षेत्र का प्रतिनिधि निश्चित नहीं होता, इसलिए प्रतिनिधियों में उत्तरदायित्व की भावना पैदा नहीं होती।

प्रश्न 7.
अल्पसंख्यक प्रतिनिधित्व समस्या का क्या अर्थ है?
उत्तर:
आधुनिक युग प्रजातान्त्रिक युग है और वास्तविक प्रजातन्त्र वही होता है जिसमें समाज के प्रत्येक वर्ग को समुचित प्रतिनिधित्व मिले। समुचित प्रतिनिधित्व से हमारा तात्पर्य यह है कि प्रत्येक वर्ग, धर्म या जाति के प्रतिनिधि विधानमण्डल में होने चाहिएँ, ताकि वे भी अपना पक्ष रख सकें। यदि इस प्रकार का प्रतिनिधित्व नहीं होता तो विधानमण्डल को ‘जनमत का दर्पण’ नहीं कहा जा सकेगा। परन्तु आधुनिक लोकतन्त्र प्रणाली इस प्रकार की है कि उसमें अल्पसंख्यकों को प्रतिनिधित्व नहीं मिलता।

साधारणतः आजकल प्रायः सभी देशों में एक-सदस्यीय चुनाव क्षेत्रों के आधार पर होने वाले चुनावों में बहुमत प्राप्त वर्ग को अधिकतर क्षेत्रों में प्रतिनिधित्व मिलता है और अल्पसंख्यक वर्गों को अपने प्रतिनिधि चुनने का अवसर प्राप्त नहीं होता। परिणामस्वरूप बहुसंख्यक वर्ग को अपनी जनसंख्या के अनुपात में काफी अधिक स्थान विधानमण्डन में मिल जाते हैं और अल्पसंख्यक बिना प्रतिनिधित्व के रह जाते हैं; जैसे एक दल को 60% मत प्राप्त होते हैं तो उस दल को 60% स्थान प्राप्त हो जाते हैं और उनकी सरकार बन जाती है परन्तु 40% लोग बिना किसी प्रतिनिधित्व के रह जाते हैं और उनके हितों की ओर ध्यान नहीं दिया जाता। इसी को अल्पसंख्यकों के प्रतिनिधित्व की समस्या कहते हैं।

प्रश्न 8.
सीमित मत प्रणाली पर नोट लिखें।
उत्तर:
सीमित मत-प्रणाली (Limited Vote System) के अन्तर्गत सारा देश बहुत-से निर्वाचन-क्षेत्रों में विभाजित होता है। प्रत्येक निर्वाचन-क्षेत्र में से कम-से-कम 3 प्रतिनिधियों का निर्वाचन हो सकता है। इस प्रणाली में मतदाताओं को उम्मीदवारों की निश्चित संख्या से कम वोट देने का अधिकार होता है। उदाहरण के लिए, यदि हिसार निर्वाचन-क्षेत्र में से 5 उम्मीदवार चुने जाने हैं तो प्रत्येक मतदाता को 3 या 4 वोट देने का अधिकार होगा, परन्तु एक मतदाता एक उम्मीदवार को एक से अधिक मत नहीं दे सकता।

मतों की संख्या सीमित होने के कारण सभी स्थान बहुसंख्यक दल द्वारा पूरित नहीं होंगे, परिणामस्वरूप अल्पसंख्यकों को भी प्रतिनिधित्व मिल सकेगा। लेकिन जनसंख्या के अनुपात में उन्हें प्रतिनिधित्व प्राप्त नहीं हो पाता। इसके द्वारा बड़े तथा सुसंगठित अल्पमत वर्गों को ही प्रतिनिधित्व मिल सकता है।

प्रश्न 9.
वयस्क मताधिकार के पक्ष में चार तर्क (गुण) दीजिए।
उत्तर:
व्यस्क मताधिकार के पक्ष में चार तर्क निम्नलिखित हैं

1. लोक प्रभुसत्ता के सिद्धान्त के अनुकूल लोक प्रभुसत्ता का अर्थ है कि सर्वोच्च शक्ति जनता में निहित है। लोकतन्त्र तब तक वास्तविक लोकतन्त्र नहीं हो सकता जब तक कि प्रतिनिधियों के चुनाव में प्रत्येक नागरिक का योगदान न हो। अतः प्रतिनिधियों का चुनाव सामान्य जनता द्वारा किया जाना चाहिए।

2. यह समानता पर आधारित है लोकतन्त्र का मुख्य आधार है-समानता। सभी व्यक्ति समान हैं और विकास के लिए सभी को मताधिकार देना भी आवश्यक है। जिन नागरिकों को मतदान का अधिकार नहीं होता, उनके हितों तथा अधिकारों की सरकार तनिक भी परवाह नहीं करती। इसलिए प्रत्येक वयस्क को मत देने का अधिकार होना चाहिए। नों का प्रभाव सभी पर पड़ता है-राज्य के कानूनों तथा नीतियों का प्रभाव सभी व्यक्तियों पर पड़ता है। उसे निश्चित करने में भी सबका भाग होना चाहिए।

4. नागरिकों को राजनीतिक शिक्षा मिलती है वयस्क मताधिकार होने से सभी नागरिक समय-समय पर देश में होने वाले चुनावों में भाग लेते रहते हैं। विभिन्न राजनीतिक दलों के नेता अपने दल की नीति का लोगों में प्रचार करते हैं और देश की समस्याओं के बारे में उनको जानकारी देते रहते हैं। इससे नागरिकों को राजनीतिक शिक्षा मिलती है।

प्रश्न 10.
वयस्क मताधिकार के चार अवगुण लिखें।
उत्तर:
व्यस्क मताधिकार के चार अवगुण निम्नलिखित हैं

1. अशिक्षित व्यक्तियों को मताधिकार देना अनुचित है-प्रत्येक देश में अधिकतर जनता अशिक्षित तथा अज्ञानी होती है। वे उम्मीदवार के गुणों को न देखकर जाति, धर्म तथा मित्रता आदि के आधार पर अपने मत का प्रयोग करते हैं। ऐसे व्यक्ति राजनीतिक नेताओं के जोशीले भाषणों से भी शीघ्र प्रभावित हो जाते हैं। अतः अशिक्षित व्यक्तियों को मताधिकार देना उचित नहीं है।

2. भ्रष्टाचार को बढ़ावा-वयस्क मताधिकार प्रणाली में भ्रष्टाचार को बढ़ावा मिलता है। निर्धन व्यक्ति थोड़े-से लालच में पड़कर अपना मत स्वार्थी तथा भ्रष्टाचारी उम्मीदवारों के हाथों में बेच देते हैं।

3. प्रशासन तथा देश की समस्याएँ जटिल- आधुनिक युग में शासन संबंधी प्रश्न तथा समस्याएँ दिन-प्रतिदिन जटिल होती जा रही हैं, जिन्हें समझ पाना साधारण व्यक्ति के बस की बात नहीं है। प्रायः साधारण मतदाता अयोग्य व्यक्ति को चुन लेते हैं क्योंकि उनके पास देश की समस्याओं पर विचार करने तथा उन्हें समझने के लिए समय ही नहीं होता।

4. साधारण जनता रूढ़िवादी होती है वयस्क मताधिकार के विरुद्ध एक तर्क यह प्रस्तुत किया जाता है कि साधारण जनता रूढ़िवादी होती है। उनके द्वारा आर्थिक तथा सामाजिक क्षेत्र में प्रगतिशील नीतियों का विरोध किया जाता है। अतः मताधिकार केवल उन्हीं व्यक्तियों को ही मिलना चाहिए जो इसका उचित प्रयोग करने की योग्यता रखते हों।

प्रश्न 11.
चुनाव आयोग के कोई चार कार्य लिखें।
उत्तर:
चुनाव आयोग के चार कार्य निम्नलिखित हैं

1. चुनाव प्रबन्धन, निर्देशन व नियन्त्रण चुनाव आयोग का प्रमुख कार्य निष्पक्ष चुनाव करवाना है, इसलिए सम्पूर्ण चुनाव व्यवस्था चुनाव आयोग के अधीन है। यह चुनावों का प्रबन्ध, निर्देशन व नियन्त्रण करता है तथा चुनावों से सम्बन्धित समस्याओं का समाधान करता है।

2. मतदाता सूचियाँ तैयार करना चुनाव आयोग चुनाव से पूर्व चुनाव क्षेत्र के आधार पर मतदाता सूचियाँ तैयार करवाता है, जिसके लिए यथासम्भव उन सभी वयस्क नागरिकों को मतदाता सूची में सम्मिलित करने का प्रयास किया जाता है जो मतदाता बनने की योग्यता रखते हैं।

3. राजनीतिक दलों को निर्वाचन में ठीक व्यवहार रखने के निर्देश-चुनाव आयोग चुनाव के समय उचित वातावरण बनाए रखने के लिए सभी राजनीतिक दलों और साधारण जनता के लिए आचार संहिता बना सकता है; जैसे मत प्राप्त करने के लिए राजनीतिक दल जातिवाद या सांप्रदायिकता की भावना को नहीं भड़काएँगे तथा भ्रष्ट तरीकों को नहीं अपनाएँगे।

4. चुनाव की तिथि की घोषणा करना-चुनाव आयोग उम्मीदवारों के लिए नामांकन-पत्र भरने, नाम वापस लेने तथा नामांकन-पत्रों की जांच करने की तिथि निश्चित करता है। यह आयोग उस तिथि की भी घोषणा करता है, जिस दिन आम चुनाव होने होते हैं और नागरिकों को अपने मतदान के अधिकार का प्रयोग करना होता है।

प्रश्न 12.
चुनाव आयोग का क्या महत्त्व है?
उत्तर:
भारतीय चुनाव आयोग की महत्ता का विवरण निम्नलिखित आधारों पर किया जा सकता है

1. भारतीय लोकतन्त्र के लिए आवश्यक-भारत में लोकतन्त्र की स्थापना की गई है। लोकतन्त्र स्वतन्त्र व निष्पक्ष चुनावों पर आधारित है। भारत के संविधान निर्माताओं ने भारत में स्वतन्त्र व निष्पक्ष चुनाव करवाने के लिए चुनाव आयोग का गठन किया। अतः चुनाव आयोग स्वतन्त्र व निष्पक्ष चुनाव करवाकर भारतीय लोकतन्त्र की सुरक्षा करता है।

2. राजनीतिक दलों पर नियन्त्रण के लिए आवश्यक-चुनाव आयोग चुनाव के दिनों में राजनीतिक दलों के कार्यों को निर्देशित व नियंत्रित करने की महत्त्वपूर्ण भूमिका निभाता है। चुनाव आयोग ही विभिन्न राजनीतिक दलों को मान्यता प्रदान करता है एवं उनकी मान्यता को रद्द भी करता है। इसके साथ ही चुनाव आयोग राजनीतिक दलों के लिए आचार संहिता को निश्चित करता है। आचार संहिता की अवहेलना करने वाले दलों के या उनके सदस्यों के विरुद्ध कार्रवाई भी करता है।

3. भारतीय चुनाव राजनीति को प्रदूषित होने से बचाने वाली संस्था के रूप में चुनाव आयोग भारतीय चुनाव राजनीति को प्रदूषित होने से बचाने वाली संस्था के रूप में अहम भूमिका निभाता है। चुनाव आयोग समय-समय पर सन् 1951 के भारतीय प्रतिनिधित्व अधिनियम के अन्तर्गत भिन्न-भिन्न आदेश देता है जिससे चुनाव प्रक्रिया को भ्रष्ट होने से बचाया जा सकता है; जैसे प्रत्येक उम्मीदवार को चुनाव के बाद चुनाव में हुए खर्च का ब्यौरा देना होता है, शासक दल सरकारी मशीनरी का प्रयोग नहीं कर सकता, चुनाव के दौरान कोई घोषणा नहीं की जा सकती आदि।

4. सरकारों पर नियन्त्रण की भूमिका चुनाव आयोग केंद्र सरकार व राज्य सरकारों पर नियन्त्रण रखने की महत्त्वपूर्ण भूमिका निभाता है। भारत में बहुमत प्राप्त दल सरकार का निर्माण करता है। सत्तारूढ़ दल अपनी शक्ति का प्रयोग करके चुनाव के लिए धन एकत्रित कर सकता है, जबकि विरोधी दल ऐसा नहीं कर सकता। इसलिए आयोग आदर्श आचार संहिता के अन्तर्गत सरकारों को निर्देश देता है और उन पर नियन्त्रण रखता है।

HBSE 11th Class Political Science Important Questions Chapter 3 चुनाव और प्रतिनिधित्व

प्रश्न 13.
भारतीय चुनाव प्रणाली की कोई चार विशेषताएँ लिखें।
उत्तर:
भारतीय चुनाव प्रणाली की चार विशेषताएँ निम्नलिखित हैं

1. वयस्क मताधिकार-भारतीय चुनाव व्यवस्था की प्रमुख विशेषता वयस्क मताधिकार है। इसका अर्थ यह है कि देश के प्रत्येक नागरिक, जिसकी आयु 18 वर्ष अथवा उससे अधिक है, को मतदान में भाग लेने का अधिकार है। यहाँ यह उल्लेखनीय है कि 61वें संवैधानिक संशोधन द्वारा मताधिकार की आयु 21 वर्ष से घटाकर 18 वर्ष कर दी गई थी। इसके अतिरिक्त भारत में जाति, धर्म, वर्ण, लिंग, शिक्षा आदि के आधार पर किसी भी व्यक्ति को मताधिकार से वंचित नहीं किया जा सकता।

2. अनुसूचित जातियों, अनुसूचित जनजातियों तथा एंग्लो-इंडियन जाति के सदस्यों के लिए स्थान सुरक्षित रखना भारतीय संविधान के अनुसार संसद, राज्यों के विधानमण्डलों तथा स्थानीय स्वशासन की इकाइयों में पिछड़ी हुई जातियों, अनुसूचित जातियों तथा एंग्लो-इंडियन जाति के सदस्यों के लिए स्थान सुरक्षित रखने की व्यवस्था की गई है। आरंभ में यह व्यवस्था केवल 10 वर्ष के लिए अर्थात् 1960 तक थी, परन्तु दस-दस वर्ष के लिए बढ़ाकर इसे लागू रखा गया है। अब यह व्यवस्था 104वें संवैधानिक संशोधन द्वारा सन् 2030 तक कर दी गई है।

3. एक-सदस्यीय चनाव क्षेत्र भारतीय चनाव व्यवस्था की एक अन्य विशेषता एक-सदस्यीय निर्वाचन क्षेत्र का होना है। चुनाव के समय प्रत्येक राज्य को लगभग बराबर जनसंख्या वाले क्षेत्रों में बाँट दिया जाता है और एक निर्वाचन क्षेत्र से एक ही प्रतिनिधि चुना जाता है। स्वतन्त्रता-प्राप्ति से पूर्व चुनाव में कुछ क्षेत्र दो सदस्यों वाले भी होते थे-एक साधारण जनता के लिए और दूसरा आरक्षित स्थान से । परन्तु अब इस व्यवस्था को समाप्त कर दिया गया है।

4. गुप्त मतदान स्वतन्त्र तथा निष्पक्ष चुनाव के लिए गुप्त मतदान आवश्यक है। भारत में भी लोकसभा, विधानसभा आदि के चुनाव के लिए गुप्त मतदान प्रणाली को अपनाया गया है। मत डालने वाले के अतिरिक्त अन्य किसी व्यक्ति को इस बात का पता नहीं लगता कि उसने अपना मतदान किस उम्मीदवार को दिया है। इससे भ्रष्टाचार में भी कमी होती है।

प्रश्न 14.
भारतीय चुनाव-प्रणाली की कोई चार त्रुटियाँ लिखें।
उत्तर:
भारतीय चुनाव-प्रणाली की चार त्रुटियाँ निम्नलिखित हैं

1. मतदाता सूचियों के बनाने में लापरवाही यह भी देखा गया है कि भारत में मतदाता सूचियों के बनाने में बड़ी लापरवाही से काम लिया जाता है और कई बार जान-बूझकर तथा कई बार अनजाने में पूरे-के-पूरे मोहल्ले सूचियों से गायब हो जाते हैं। मतदाता सूचियाँ अधिकतर राज्य सरकार के कर्मचारियों द्वारा बनाई जाती हैं और वे इसे फिजूल का काम समझते हैं।

विभिन्न विभागों के सामान्य कर्मचारियों विशेषतः पटवारियों तथा स्कूल के अध्यापकों से यह काम करवाया जाता है। प्रायः यह भी देखने में आता है कि नई मतदाता सूची बनाते समय नए मतदाताओं के नाम तो जोड़ दिए जाते हैं परन्तु स्वर्गवासी हो चुके मतदाताओं के नाम मतदाता सूची में ज्यों के त्यों बने रहते हैं। इस प्रकार मतदाता सूचियों में अंकित नामों एवं वास्तविक नामों में पर्याप्त अन्तर हो जाता है। अतः मतदाता सूचियों को समय-समय पर संशोधित करने एवं चुनावी प्रक्रिया के प्रथम कार्य को सावधानीपूर्वक करने की आवश्यकता है।

2. सरकारी तन्त्र का दुरुपयोग भारतीय चुनाव व्यवस्था की एक और गम्भीर त्रुटि सामने आई है। मन्त्रियों द्वारा दलीय लाभ के लिए सरकारी तंत्र का प्रयोग किया जाता है। वोट बटोरने के लिए मन्त्रियों द्वारा लोगों को तरह-तरह के आश्वासन दि हैं। विभिन्न वर्गों के लिए अनेकानेक रियायतें और सुविधाओं की घोषणा की जाती है।

अनेक प्रकार की विकास योजनाओं की घोषणा की जाती है; जैसे कारखानों, स्कूलों, कॉलेजों, अस्पतालों व पुलों के शिलान्यास आदि की घोषणा करना। सरकारी कर्मचारियों के वेतन-भत्ते आदि में वृद्धि की जाती है। कर्जे माफ किए जाते हैं। इसके अतिरिक्त सरकारी गाड़ियों तथा अन्य सुविधाओं का प्रयोग किया जाता है। यद्यपि चुनाव सम्बन्धी अधिनियम के अनुसार ऐसा करना भ्रष्ट व्यवहार में सम्मिलित है परन्तु फिर भी चुनावी प्रक्रिया के दौरान यह सब देखने को मिलता है।

3. राजनीतिक दलों को मतों के अनुपात से स्थानों की प्राप्ति न होना यह भी देखा गया है कि चुनावों में राजनीतिक दलों को उस अनुपात में विधानमण्डलों में स्थान प्राप्त नहीं होते, जिस अनुपात में उन्हें मत प्राप्त होते हैं। जैसे भारत में 13वीं लोकसभा के चुनाव के समय भारतीय जनता पार्टी को 182 स्थान मिले और मतों का प्रतिशत 23.70 प्रतिशत रहा, जबकि काँग्रेस को 28.42 प्रतिशत मत मिलने के पश्चात् भी 114 स्थान ही प्राप्त हुए, फिर भी काँग्रेस का मत-प्रतिशत भाजपा से 5 प्रतिशत अधिक है। इससे दलों में मायूसी का पैदा होना स्वाभाविक है। इस प्रकार एक लोकतान्त्रिक व्यवस्था के अन्तर्गत इस स्थिति को न्यायपूर्ण नहीं कहा जा सकता।

4. चुनाव नियमों में दोष यह भी देखा गया है कि चुनाव सम्बन्धी नियमों में बड़ी कमियाँ हैं और इसी कारण चुनाव में बरती गई अनियमितताओं को न्यायालय के सामने सिद्ध करना बड़ा कठिन हो जाता है। लगभग सभी दल धर्म और जाति के आधार पर अपने उम्मीदवार चुनते हैं तथा इसी के नाम पर वोट माँगते हैं, परन्तु इसको सिद्ध करना कठिन है। सभी उम्मीदवार मतदाताओं को लाने व ले जाने के लिए गाड़ियों का प्रयोग करते हैं, परन्तु यह बात चुनाव-याचिका की सुनवाई के समय सिद्ध नहीं हो पाती।

प्रश्न 15.
भारतीय चुनाव प्रणाली में सुधार के लिए कोई तीन सुझाव दें।
उत्तर:
भारतीय चुनाव प्रणाली में सुधार के लिए तीन सुझाव निम्नलिखित हैं

1. आनुपातिक प्रतिनिधित्व प्रणाली शुरू करना-भारत में चुनाव-प्रणाली का सबसे बड़ा दोष यह है कि कई बार राजनीतिक दल कम मत प्राप्त करते हैं, परन्तु उन्हें अधिक स्थान प्राप्त हो जाते हैं। उदाहरणतः सन् 1980 में काँग्रेस को लोकसभा के लिए केवल 42.6% मत मिले, परन्तु संसद में 67% स्थान मिले। इस बुराई को समाप्त करने के लिए आनुपातिक प्रतिनिधित्व प्रणाली का सुझाव दिया गया है। यद्यपि भारत में इसे लागू करना कठिन कार्य है।

2. चुनाव याचिकाओं के बारे में सुझाव-गत अनेक वर्षों से चुनाव याचिकाएँ उच्च न्यायालयों के समक्ष आई हैं, जिनका निपटारा करने में कई वर्ष लगे हैं। चुनाव याचिकाएँ चुनावों से भी अधिक खर्चीली तथा कष्टमय बन गई हैं। इसलिए इन याचिकाओं का निपटारा शीघ्र होना चाहिए, ताकि इसके दोनों पक्षों को बेकार की मुसीबत तथा खर्च से छुटकारा मिल सके।

3. निर्दलीय उम्मीदवारों पर रोक-निर्दलीय उम्मीदवार निष्पक्ष चुनावों के लिए एक समस्या हैं। इसलिए निर्दलीय उम्मीदवारों पर रोक लगनी चाहिए। यद्यपि कानून द्वारा स्वतन्त्र उम्मीदवारों को रोका नहीं जा सकता, लेकिन फिर भी ऐसा कुछ अवश्य होना चाहिए कि मजाक के लिए चुनाव लड़ने वालों पर रोक लगे। इस संबंध में यह सुझाव दिया है कि एक तो जमानत की राशि को बढ़ा देना चाहिए। दूसरे यह व्यवस्था होनी चाहिए कि जिस निर्दलीय उम्मीदवार को निश्चित प्रतिशत से कम वोट प्राप्त होते हैं उसे अगले चुनाव में भाग लेने का अधिकार नहीं होगा। इस प्रकार की व्यवस्था से निश्चित रूप से निर्दलीय उम्मीदवारों पर रोक लगेगी।

प्रश्न 16.
प्रादेशिक प्रतिनिधित्व के चार गुण लिखें।
उत्तर:
प्रादेशिक प्रतिनिधित्व के चार गुण निम्नलिखित हैं
1. प्रतिनिधियों का उत्तरदायित्व प्रादेशिक प्रतिनिधित्व प्रणाली में प्रतिनिधि अपने क्षेत्र के मतदाताओं के प्रति उत्तरदायी होता है। एक क्षेत्र के सभी मतदाताओं का प्रतिनिधि होने के कारण वह अपने उत्तरदायित्व से इन्कार नहीं कर सकता।

2. क्षेत्रीय हितों की रक्षा प्रादेशिक प्रतिनिधित्व प्रणाली में क्षेत्रीय हितों की अच्छी तरह पूर्ति होती है। प्रतिनिधि अपने क्षेत्र की आवश्यकताओं को अच्छी तरह समझते हैं क्योंकि उनका अपने मतदाताओं व क्षेत्र से समीप का संबंध होता है। अतः प्रतिनिधि अपने निर्वाचन-क्षेत्र की आवश्यकताओं की पूर्ति के लिए भरसक प्रयत्न करते हैं। यदि प्रतिनिधि अपने मतदाताओं के हितों की रक्षा नहीं करता, तो ऐसे प्रतिनिधि को मतदाता दोबारा नहीं चुनते। इसलिए प्रतिनिधि अपने क्षेत्र के हितों की उपेक्षा नहीं कर सकता।

3. अधिक विकास की सम्भावना प्रादेशिक प्रतिनिधित्व प्रणाली में अधिक विकास की सम्भावना बनी रहती है। इसका कारण यह है कि इस प्रणाली के अन्तर्गत प्रत्येक प्रतिनिधि अपने क्षेत्र का अधिक-से-अधिक विकास करना चाहता है क्योंकि भावी चुनाव में वह अपनी सीट को सुनिश्चित कर लेना चाहता है। सभी क्षेत्रों के विकास से देश का विकास होना स्वाभाविक है।

4. कम खर्चीली इस प्रणाली में चुनाव कम खर्चीला होता है और उम्मीदवार को भी चुनाव में कम खर्च करना पड़ता है।

प्रश्न 17.
प्रादेशिक प्रतिनिधित्व के चार अवगुण लिखें।
उत्तर:
प्रादेशिक प्रतिनिधित्व के चार अवगुण निम्नलिखित हैं

1. क्षेत्रीयवाद की भावना-इस प्रणाली द्वारा चुने गए प्रतिनिधियों का दृष्टिकोण संकुचित हो जाता है। प्रतिनिधि अपने को एक क्षेत्र विशेष का प्रतिनिधि समझने लगते हैं और उसी क्षेत्र के विकास की बात सोचते तथा करते हैं और उसके लिए प्रयत्नशील रहते हैं। इससे राष्ट्रीय हितों की अवहेलना होने लगती है।

2. सीमित पसन्द-कई बार मतदाताओं की पसन्द सीमित हो जाती है क्योंकि चुनाव लड़ने वाले प्रायः उसी क्षेत्र के निवासी होते हैं और यदि उस क्षेत्र में अच्छे उम्मीदवार न हों तो मतदाताओं को इच्छा न होते हुए भी किसी-न-किसी उम्मीदवार के पक्ष में मत डालना ही पड़ता है।

3. भ्रष्ट होना मतदाताओं को भ्रष्ट किए जाने की सम्भावना रहती है क्योंकि मतदाता कम होते हैं और धनी उम्मीदवार धन के बल पर वोट खरीदने का प्रयत्न करने लगते हैं।

4. अल्पसंख्यकों को उचित प्रतिनिधित्व न मिलना-इस प्रणाली में अल्पसंख्यक वर्गों को प्रतिनिधित्व आसानी से नहीं मिलता। एक क्षेत्र से एक उम्मीदवार चुना जाता है और स्वाभाविक है कि बहुमत वर्ग का उम्मीदवार ही चुना जाता है। इस प्रकार अल्पसंख्यक वर्ग के प्रतिनिधि लगभग सभी चुनाव क्षेत्रों में हार जाते हैं।

निबंधात्मक प्रश्न

प्रश्न 1.
प्रत्यक्ष चुनाव प्रणाली से आप क्या समझते हैं? इसके पक्ष तथा विपक्ष में तर्क दीजिए।
उत्तर:
साधारण शब्दों में, प्रत्यक्ष चुनाव-प्रणाली उस चुनाव व्यवस्था को कहा जाता है जिसमें साधारण मतदाता प्रत्यक्ष रूप से अपने प्रतिनिधियों को चुनते हैं। प्रत्येक मतदाता चुनाव-स्थान पर जाकर स्वयं अपनी पसन्द के उम्मीदवार के पक्ष में, अपने मत का प्रयोग करता है और जिस उम्मीदवार को सबसे अधिक मत प्राप्त होते हैं, उसे निर्वाचित घोषित कर दिया जाता है।

भारत में लोकसभा तथा राज्य विधानसभा के सदस्यों का चुनाव प्रत्यक्ष प्रणाली द्वारा किया जाता है। इस प्रणाली के पक्ष तथा विपक्ष में निम्नलिखित तर्क दिए जाते हैं प्रत्यक्ष चुनाव प्रणाली के पक्ष में तर्क (Arguments in Favour of Direct Method of Election)-प्रत्यक्ष चुनाव-प्रणाली .. की पक्ष में निम्नलिखित तर्क दिए जा सकते हैं

1. अधिक लोकतन्त्रीय प्रणाली (More Democratic System):
इस प्रणाली में जनता को स्वयं प्रत्यक्ष रूप से मतदान करके अपने प्रतिनिधि चुनने का अवसर मिलता है। अतः यह प्रणाली अधिक लोकतान्त्रिक है।

2. मतदाताओं तथा प्रतिनिधियों में सीधा सम्पर्क (Direct link between the voters and their Representatives):
इस प्रणाली में मतदाता तथा उम्मीदवार सीधे रूप से सम्पर्क में आते हैं तथा मतदाता उम्मीदवारों को भली-भांति जान सकते हैं। उम्मीदवार भी अपनी नीतियाँ तथा कार्यक्रम जनता के सामने रख सकते हैं।

3. राजनीतिक शिक्षा (Political Education):
इस प्रणाली में मतदाताओं तथा उम्मीदवारों में सीधा सम्पर्क होता है। इससे मतदाताओं को राजनीतिक शिक्षा मिलती है और उनमें राजनीतिक जागरूकता की भावना का भी उदय होता है।

4. अधिकारों और कर्तव्यों का ज्ञान (Knowledge of Rights and Duties):
इस प्रणाली के अन्तर्गत सामान्य जनता को मताधिकार तथा अन्य अधिकारों का ज्ञान प्राप्त होता है तथा कर्तव्यों की भी जानकारी मिलती है।

5. राजनीतिक दलों के लिए आसानी (Convenient for Political Parties):
इस प्रणाली में राजनीतिक दलों के लिए भी आसानी होती है और जो राजनीतिक दल अधिक प्रभावशाली होता है उसके लिए अधिक मत प्राप्त करना तथा बहुमत प्राप्त करना सरल हो जाता है।

6. मतदाताओं में आत्म-सम्मान की भावना आती है (It brings sense of Self-respect among the Voters):
प्रत्येक चुनाव-प्रणाली के अनुसार प्रत्येक मतदाता को निर्वाचन में भाग प्राप्त होता है, जिसके फलस्वरूप मतदाताओं में आत्म-सम्मान की भावना जन्म लेती है। वे अपने-आप को शासन-तन्त्र का अंश अनुभव करते हैं और राजनीतिक गतिविधियों में बढ़-चढ़कर भाग लेते हैं।

7. प्रतिनिधि सभी लोगों के सामने उत्तरदायित्व निभाते हैं (Representatives owe Responsibility to All):
वयस्क मताधिकार के आधार पर प्रत्यक्ष चुनाव-प्रणाली द्वारा चुने गए प्रतिनिधि अपने-आपको जनता के सामने अधिक उत्तरदायी महसूस करते हैं क्योंकि वे कुछ मतदाताओं द्वारा नहीं, बल्कि सभी मतदाताओं द्वारा चुने जाते हैं।

प्रत्यक्ष चुनाव प्रणाली के विपक्ष में तर्क (Arguments Against of Direct Method of Election) प्रत्यक्ष चुनाव-प्रणाली . के विपक्ष में निम्नलिखित तर्क दिए जाते हैं

1. साधारण मतदाताओं में मतं का उचित प्रयोग करने की क्षमता का अभाव (Ordinary Voters cannot exercise the Right to Vote Properly): प्रत्यक्ष चुनाव-प्रणाली के आलोचकों का कहना है कि साधारण मतदाताओं में अपने मत का उचित प्रयोग करने की क्षमता नहीं होती। वे झूठे प्रचारों और जोशीले भाषणों के प्रभाव में बह जाते हैं और अयोग्य उम्मीदवारों को अपना वोट डाल देते हैं।

2. अनुचित प्रचार (False Propaganda):
इस प्रणाली के अन्तर्गत निर्वाचन अभियान में झूठे प्रचार का सहारा लिया जाता है। उम्मीदवार एक-दूसरे की निन्दा करते हैं और एक-दूसरे पर झूठे आरोप लगाते हैं जिसके परिणामस्वरूप मतदाता पथ-भ्रष्ट हो सके।

3. अधिक खर्चीली प्रणाली (More Expensive System):
प्रत्यक्ष चुनाव-प्रणाली अधिक खर्चीली मानी गई है। इसके अन्तर्गत चुनाव कराने वाले अभिकरण को भी बहत खर्चा करना पड़ता है और उम्मीदवारों को भी बहत खर्चा करना पड़ता है।

4. अव्यवस्थाजनक (Causes Disruption):
यह चुनाव-प्रणाली एक ऐसी चुनाव-प्रणाली है जिसमें अव्यवस्था हो जाती है। अधिक जोश-खरोश के कारण दंगे-फसाद और अन्य प्रकार की अव्यवस्थाजनक स्थितियाँ सामने आ जाती हैं जिनका समाज पर बुरा प्रभाव पड़ता है।

5. दलगत भावना को बढ़ावा (Encourages Partisan Spirit):
प्रत्यक्ष चुनाव-प्रणाली राजनीतिक दलों का अखाड़ा बन जाती है, जिसमें राजनीतिक दल एक-दूसरे के विरुद्ध आरोप-प्रत्यारोप लगाते रहते हैं।

प्रश्न 2.
आनुपातिक प्रतिनिधित्व प्रणाली के विभिन्न तरीकों या रूपों का वर्णन करें।
उत्तर:
अल्पसंख्यकों को उचित प्रतिनिधित्व देने के लिए यह प्रणाली सबसे अधिक महत्त्वपूर्ण है। इसकी विशेषता यह है कि इसका उद्देश्य प्रत्येक दल को कुछ प्रतिनिधित्व दिलाना है। इसके अन्तर्गत प्रत्येक दल को उसके मतदान की शक्ति के अनुपात में ही प्रतिनिधित्व दिया जाता है। आनुपातिक प्रतिनिधित्व के मुख्य दो रूप हैं-

  • एकल संक्रमणीय मत-प्रणाली (Single Transferable Vote System) और
  • सूची-प्रणाली (List System)। ये दोनों प्रणालियाँ अल्पसंख्यकों को उनकी जनसंख्या के अनुपात के अनुसार प्रतिनिधित्व दिलाती हैं।

1. एकल संक्रमणीय मत प्रणाली (Single Transferable Vote System) इस प्रणाली को हेयर प्रणाली (Hare System) भी कहा जाता है, क्योंकि सर्वप्रथम इसका वर्णन एक अंग्रेज़ विद्वान थॉमस हेयर (Thomas Hare) ने सन् 1851 में अपनी पुस्तक ‘प्रतिनिधियों का चुनाव’ (Election of Representatives) में किया था।

सन् 1855 में डेनमार्क के एक मन्त्री एंड्रे (Andrae) द्वारा इसे पहली बार लागू किया गया। इसे अधिमान्य प्रणाली (Preferential System) भी कहा जाता है, क्योंकि इसमें मतदाता मत-पत्र (Ballot-Paper) पर अपने अधिमानों (Preferences) का संकेत दे सकता है। भारतवर्ष में राज्यसभा के सदस्यों का चुनाव इसी प्रणाली के द्वारा किया जाता है। इस प्रणाली के अनुसार चुनाव जीतने के लिए जितने मत प्राप्त करने आवश्यक हैं, उनकी संख्या एक सूत्र (Formula) के अनुसार निश्चित की जाती है। इसे कोटा (Quota) कहा जाता है। इसकी गणना निम्नलिखित प्रकार से की जाती है
HBSE 11th Class Political Science Important Questions Chapter 2 Img 1
मतदान-पत्र (Ballot Paper) में निर्वाचन के लिए खड़े सभी उम्मीदवारों के नाम लिखे हुए होते हैं। निर्वाचन के लिए जितने उम्मीदवार खड़े होते हैं, मतदाता को उतने ही अधिमान या पूर्वाधिकार (Preferences) देने का अधिकार होता है। उसे अपने अधिमानों (Preferences) को 1, 2, 3, 4, 5, 6 आदि के रूप में लिखना पड़ता है। . जब मतदान की प्रक्रिया (Polling) समाप्त हो जाती है तो सभी उम्मीदवारों को प्राप्त हुए प्रथम अधिमानों (First Preferences) को गिन लिया जाता है।

जो उम्मीदवार निश्चित कोटा प्राप्त कर लेते हैं, उन्हें शीघ्र ही निर्वाचित घोषित कर दिया जाता है। ऐसे उम्मीदवार निश्चित कोटे से कुछ अधिक वोट भी प्राप्त कर सकते हैं। इन अतिरिक्त (Additional) वोटों का उनके लिए कुछ भी महत्त्व नहीं होता। इसीलिए इन अतिरिक्त (Additional) वोटों को मत-पत्र (Ballot Paper) में लिखे गए अधिमानों (Preferences) के क्रम के अनुसार दूसरे उम्मीदवारों को दे दिया जाता है अर्थात् हस्तांतरित कर दिया जाता है।

इस प्रकार का हस्तांतरण उस समय तक जारी रहता है, जब तक कोटा प्राप्त करने वाले उम्मीदवारों की संख्या निर्वाचन-क्षेत्र से चुने जाने वाले उम्मीदवारों की संख्या के बराबर न हो जाए। जिस समय यह संख्या समान हो जाती है तो उस समय हस्तांतरण की प्रक्रिया बन्द कर दी जाती है और निर्वाचन परिणाम घोषित कर दिया जाता है।

2. सूची प्रणाली (List System)-आनुपातिक प्रतिनिधित्व का दूसरा रूप सूची प्रणाली है। यह प्रणाली बहु-सदस्यीय निर्वाचन-क्षेत्र में, जिसमें कम-से-कम तीन सीटें हों, लागू की जा सकती है। इस प्रणाली के अन्तर्गत चुनाव में जो उम्मीदवार खड़े होते हैं, उनकी अपने-अपने दलों के अनुसार अनुसूचियाँ बना ली जाती हैं। प्रत्येक मतदाता को उतने मत देने का अधिकार होता है जितने कि सदस्य उस निर्वाचन-क्षेत्र से चुने जाते हैं, परन्तु वह एक उम्मीदवार को एक से अधिक मत नहीं दे सकता।

उदाहरणस्वरूप, यदि किसी निर्वाचन-क्षेत्र में 4 सीटें हैं तो प्रत्येक मतदाता को 4 मत देने का अधिकार होगा। इस प्रणाली में चुनाव का परिणाम अलग-अलग उम्मीदवारों को प्राप्त हुए मतों के अनुसार नहीं निकाला जाता, बल्कि प्रत्येक सूची के उम्मीदवारों के मत इकट्ठे जोड़ लिए जाते हैं। इस प्रकार हम देखते हैं कि मतदाता अपना मत उम्मीदवार को नहीं देते, बल्कि सूची को देते हैं।

इसके अन्तर्गत भी उपर्युक्त एकल संक्रमणीय प्रणाली में दिए गए फार्मूले के अनुसार कोटा (Quota) निकाला जाता है और एक स्थान से चुनाव जीतने के लिए न्यूनतम मतों की संख्या को निश्चित किया जाता है, इसके पश्चात प्रत्येक सूची को कितने स्थान मिलने चाहिएँ, यह उस सूची को दिए गए मतों की कुल संख्या को कोटा से विभाजित करके निकाल लिया जाता है। उदाहरणस्वरूप, एक आएगा।
HBSE 11th Class Political Science Important Questions Chapter 2 Img 2
यदि चुनाव में तीन दल भाग ले रहे हैं और उनको मत इस प्रकार मिले हैं जनता दल 4,200, काँग्रेस 3,000 और साम्यवादी दल 2,800 तो उस समय जनता दल को दो स्थान, काँग्रेस को एक स्थान तथा साम्यवादी दल को एक स्थान प्राप्त हो जाएगा। यदि इसमें किसी दल को कोटे से कम मत प्राप्त होते हैं तो बची हुई सीट उस दल को दी जाएगी, जिसके शेष मतों की संख्या सबसे अधिक है। इस प्रकार प्रत्येक दल को उसकी मत-संख्या के अनुपात से प्रतिनिधित्व मिल जाता है। यह प्रणाली नार्वे, स्वीडन तथा बेल्जियम आदि देशों में अपनाई गई है।

HBSE 11th Class Political Science Important Questions Chapter 3 चुनाव और प्रतिनिधित्व

प्रश्न 3.
आनुपातिक प्रतिनिधित्व प्रणाली के पक्ष तथा विपक्ष में तर्क दीजिए।
उत्तर:
आनुपातिक प्रतिनिधित्व प्रणाली के पक्ष में तर्क (Arguments in Favour of System of Proportional Representation)-आधुनिक प्रतिनिधित्व प्रणाली के पक्ष में तर्क निम्नलिखित हैं

1. सभी वर्गों को प्रतिनिधित्व (Representation to All Classes):
इस प्रणाली का यह गुण है कि समाज के सभी वर्गों को उनकी जनसंख्या के अनुपात में विधानमण्डल में प्रतिनिधित्व मिल जाता है। कोई वर्ग प्रतिनिधित्व से वंचित नहीं रहता और सबको संतुष्टि मिलती है।

2. अल्पसंख्यकों में सुरक्षा की भावना (Feeling of Protection among Minorities):
इस प्रणाली का यह गुण है कि सभी वर्गों को उचित प्रतिनिधित्व मिलने से उनके हितों की रक्षा होती है और अल्पसंख्यक वर्ग के लोग स्वयं को सुरक्षित समझते हैं। वे स्वयं को बहुसंख्यक वर्ग की तानाशाही का शिकार नहीं समझते।

3. मतदाताओं को मतदान में अधिक सुविधा (More Scope of Choice for Voters):
इस प्रणाली में मतदाता को अपनी पसन्द के कई उम्मीदवारों के पक्ष में मत डालने का अवसर मिलता है। उसे एक ही व्यक्ति के पक्ष में मत डालने के लिए मजबूर नहीं होना पड़ता। इससे उसे अपनी पसन्द के उम्मीदवार को मत देने में सुविधा हो जाती है।

4. निर्वाचन के लिए निर्धारित मत प्राप्त करना आवश्यक है (Quota is Necessary to Win the Election):
इस प्रणाली भावना नहीं रहती कि कोई उम्मीदवार थोड़े-से प्रतिशत मत लेकर भी चुन लिया जाएगा। इस प्रणाली में चुने जाने के लिए उम्मीदवार को एक निश्चित संख्या में मत प्राप्त करना आवश्यक होता है, अर्थात् चुने जाने के लिए उम्मीदवार को एक निश्चित संख्या के मतदाताओं का समर्थन प्राप्त करना आवश्यक है।

5. मत व्यर्थ नहीं जाते (Votes do not go Waste):
इस प्रणाली का यह भी गुण है कि इसमें कोई भी मत व्यर्थ नहीं जाता। मतदाता का मत यदि पहली पसन्द के अनुसार काम नहीं आता है, तो वह दूसरी या तीसरी या चौथी पसन्द के पक्ष में अवश्य काम आता है। मतदाता को भी यह तसल्ली रहती है कि. उसके मत का मूल्य है।

6. विधानमण्डल जनमत का सही प्रतिनिधित्व करता है (Legislature becomes Real Mirror of Public Opinion):
इस प्रणाली से विधानमण्डल जनमत का सही प्रतिनिधित्व करता है क्योंकि इसमें सभी वर्गों और जातियों के लोग अपने अनुपात से विधानमण्डल में स्थान प्राप्त कर लेते हैं।

7. बहु-दलीय प्रणाली के लिए उपयोगी (Useful in case of Multi-party System):
जिस देश में कई राजनीतिक दल हैं, उसमें यह प्रणाली बड़ी उपयोगी सिद्ध होती है क्योंकि इसके अन्तर्गत सभी राजनीतिक दलों को प्रतिनिधित्व मिल जाता है और किसी एक दल की तानाशाही स्थापित नहीं हो पाती।

8. लोगों को अधिक राजनीतिक शिक्षा (More Political Education to People):
इस प्रणाली का यह भी गुण है कि इससे लोगों को राजनीतिक शिक्षा अधिक मिलती है। लोगों को मत डालते समय काफी सोच-विचार के बाद अपनी पसन्द का प्रयोग करना पड़ता है और मतगणना के समय भी काफी सोच-समझ से काम लेना पड़ता है, इसलिए यह प्रणाली लोगों को अधिक राजनीतिक शिक्षा देती है।

आनुपातिक प्रतिनिधित्व प्रणाली के विपक्ष में तर्क (Arguments Against the System of Proportional Representation) आधुनिक प्रतिनिधित्व प्रणाली के विपक्ष में तर्क निम्नलिखित हैं

1. जटिल प्रणाली (Complicated System):
आनुपातिक प्रतिनिधित्व प्रणाली बहुत जटिल है। साधारण मतदाता इसे समझ नहीं सकता। वोटों पर पसन्द अंकित करना, कोटा निश्चित करना, वोटों की गिनती करना और वोटों को पसन्द के अनुसार हस्तांतरित करना आदि ये सब बातें एक साधारण पढ़े-लिखे व्यक्ति समझ नहीं पाते।

2. राष्ट्रीय एकता के विरुद्ध (Against National Unity):
आनुपातिक प्रतिनिधित्व प्रणाली के अन्तर्गत छोटे-छोटे दलों को प्रोत्साहन मिलता है। अल्पसंख्यक जातियाँ भी अपनी भिन्नता बनाए रखती हैं और दूसरी जातियों के साथ अपने हितों को मिलाना नहीं चाहतीं। इसके परिणामस्वरूप राष्ट्र छोटे-छोटे वर्गों और गुटों में बँट जाता है और राष्ट्रीय एकता पनप नहीं पाती।

3. राजनीतिक दलों को अधिक महत्त्व (More Importance to Political Parties):
आनुपातिक प्रतिनिधित्व की सूची प्रणाली के अन्तर्गत राजनीतिक दलों का महत्त्व बहुत अधिक होता है। मतदाता को किसी-न-किसी दल के पक्ष में वोट डालना होता है क्योंकि इस प्रणाली के अन्तर्गत निर्दलीय उम्मीदवार चुनाव नहीं लड़ सकते।

4. उत्तरदायित्व का अभाव (Lack of Responsibility):
इस प्रणाली के अन्तर्गत निर्वाचन-क्षेत्र बहु-सदस्यीय होते हैं और एक क्षेत्र में कई प्रतिनिधि होते हैं। चूंकि एक क्षेत्र का प्रतिनिधि निश्चित नहीं होता, इसलिए प्रतिनिधियों में उत्तरदायित्व की भावना पैदा नहीं होती।

5. मतदाता और प्रतिनिधि में सम्पर्क का अभाव (Lack of Contact between Voters and Representatives):
इस प्रणाली में चुनाव-क्षेत्र बड़े-बड़े होते हैं, जिस कारण से मतदाताओं को उम्मीदवारों की पूरी जानकारी प्राप्त नहीं होती और उनमें घनिष्ठ संबंध होना असम्भव-सा हो जाता है। परिणाम यह निकलता है कि प्रतिनिधियों का लोगों के साथ सम्पर्क नहीं हो पाता।

6. मतदाताओं की पसन्द सीमित हो जाती है (Voter’s Choice is Limited):
कई बार मतदाता की पसन्द का व्यक्ति नहीं चुना जाता। मान लो कि एक मतदाता ने किसी सूची में चौथे नंबर पर लिखे हुए उम्मीदवार के कारण अपना मत उस सूची के पक्ष में डाला, परन्तु उस सूची के हिस्से में केवल दो सीटें आईं तो ऐसी दशा में उस मतदाता की पसन्द अर्थहीन हो गई।

7. सरकार की अस्थिरता (Government is Unstable):
इस प्रणाली के अन्तर्गत देश में बहुत-से दल बन जाते हैं और सभी दल विधानमण्डल में कुछ-न-कुछ सीटें प्राप्त करने में सफल हो जाते हैं। जिन देशों में इस चुनाव-प्रणाली को अपनाया गया है, उनके विधानमण्डलों में किसी एक राजनीतिक दल को स्पष्ट बहुमत प्राप्त नहीं हो पाता और प्रायः मिली-जुली सरकार बनती है जो अधिक देर तक स्थिर नहीं रहती।

8. उप-चुनाव करवाने में कठिनाई (Difficulty in holding By-election):
इस प्रणाली के अधीन निर्वाचन-क्षेत्र का बहु-सदस्यीय होना बहुत आवश्यक है, परन्तु यदि आम चुनाव के बाद किसी क्षेत्र में एक सीट खाली हो जाए तो उसका उप-चुनाव कैसे किया जाए, यह एक ऐसी समस्या है, जिसे आसानी से सुलझाया नहीं जा सकता।

निष्कर्ष (Conclusion)-आनुपातिक प्रतिनिधित्व के पक्ष और विपक्ष का अध्ययन करने के पश्चात् यह निष्कर्ष निकालना आसान है कि यह प्रणाली केवल ऐसे देश में लागू हो सकती है जहाँ लोग अधिक पढ़े-लिखे हों, साथ ही यह प्रणाली ऐसे राज्य के लिए कदाचित उचित नहीं ठहराई जा सकती, जहाँ पर संसदीय शासन लागू हो। इस चुनाव-प्रणाली का एक गुण तो सराहनीय है कि यह अल्पसंख्यक जातियों को प्रतिनिधित्व दिलाने में बहुत सहायक सिद्ध हो सकती है। भारत में विधानपालिका के ऊपरी सदनों के चुनाव इसी प्रणाली के अनुसार होते हैं।

प्रश्न 4.
भारतीय चुनाव-प्रणाली की मुख्य विशेषताओं का वर्णन करें।
उत्तर:
भारतीय चुनाव-प्रणाली की मुख्य विशेषताओं का विवरण निम्नलिखित है

1. वयस्क मताधिकार (Adult Franchise):
भारतीय चुनाव-प्रणाली की प्रमुख विशेषता यह है कि यहाँ वयस्क मत्तधिकार प्रणाली को लागू किया गया है। 18 वर्ष की आयु पूरी करने वाले प्रत्येक नागरिक को संसद, विधानमण्डल, नगरपालिका, पंचायत आदि के चुनावों में मत डालने का अधिकार दिया गया है।

मताधिकार के लिए जाति, धर्म, वंश, रंग, लिंग आदि के आधार पर किसी भी प्रकार का भेदभाव नहीं किया जाता। हाँ, मतदान क्षेत्र (Constituency) में एक निश्चित अवधि तक रहने, मतदाता-सूची में नाम होने आदि की शर्ते निश्चित की गईं हैं, परन्तु अमीर-गरीब, छूत-अछूत, हिन्दू-मुसलमान आदि के आधार पर किसी को मताधिकार से वंचित नहीं किया गया है।

वयस्क मताधिकार में लोकतान्त्रिक सहभागिता के सिद्धान्त को लागू किया गया है और लगभग 50% से अधिक जनसंख्या को मत डालने का अधिकार मिला हुआ है। सितम्बर-अक्तूबर, 1999 के लोकसभा चुनाव के समय भारत में कुल मतदाताओं की संख्या लगभग 62.04 करोड़ थी, जो 14वीं लोकसभा चुनाव (अप्रैल-मई, 2004) के समय बढ़कर लगभग 67.5 करोड़ हो गई थी।

15वीं लोकसभा चुनाव (अप्रैल-मई, 2009) में मतदाताओं की कुल संख्या लगभग 71 करोड़ 40 लाख, 16वीं लोकसभा चुनाव (अप्रैल-मई, 2014) में लगभग 83.41 करोड़ थी जो अप्रैल-मई, 2019 में हुए 17वीं लोकसभा में बढ़कर लगभग 89 करोड़ 78 लाख हो गई। इस प्रकार स्पष्ट है कि भारत में जनता के एक बहुत बड़े भाग को चुनावों में अपनी सहभागिता निभाने का अधिकार प्राप्त है।

2. प्रत्यक्ष चुनाव (Direct Election):
भारतीय चुनाव-प्रणाली की एक विशेषता यह भी है कि यहाँ जनता के प्रतिनिधि जनता द्वारा प्रत्यक्ष रूप से चुने जाते हैं। केवल राज्यसभा तथा विधान परिषदों के सदस्य ही अप्रत्यक्ष रूप से चुने जाते हैं और ये संस्थाएँ इतनी शक्तिशाली नहीं हैं। वास्तविक शक्ति-प्राप्त संस्थाएँ लोकसभा, राज्य विधानसभा, पंचायत, नगरपालिका हैं और इन सबके प्रतिनिधि जनता द्वारा प्रत्यक्ष रूप से चुने जाते हैं। इनमें मनोनीत किए जाने की व्यवस्था भी नाममात्र है।

3. संयुक्त निर्वाचन (Joint Election):
भारत में चुनाव अब पृथक् निर्वाचन के आधार पर नहीं होते, बल्कि संयुक्त निर्वाचन के आधार पर होते हैं। एक चुनाव-क्षेत्र में रहने वाले सभी मतदाता, चाहे वे किसी भी जाति या धर्म से सम्बन्ध रखते हैं, अपना एक प्रतिनिधि चुनते हैं। संविधान के अनुच्छेद 325 के अनुसार प्रत्येक प्रादेशिक चुनाव के लिए संसद के सदस्य चुनने के लिए एक सामान्य निर्वाचक सूची होगी और कोई भी भारतीय धर्म, जाति तथा लिंग के आधार पर सूची में नाम लिखवाने के अयोग्य नहीं ठहराया जाएगा। इस प्रकार स्पष्ट है कि एक निर्वाचन क्षेत्र के समस्त मतदाता मिलकर संयुक्त रूप से अपने एक प्रतिनिधि का निर्वाचन करते हैं।

4. गुप्त मतदान (Secret Voting):
भारत में गुप्त मतदान की व्यवस्था की गई है। निष्पक्ष और स्वतन्त्र चुनाव के लिए गुप्त मतदान की व्यवस्था आवश्यक है। इस व्यवस्था में मत डालने वाले व्यक्ति के अतिरिक्त अन्य किसी व्यक्ति को इस बात का पता नहीं चलता कि उसने अपना मत किसको दिया है। इस प्रकार आपसी झगड़े और शत्रुता की सम्भावना कम हो जाती है तथा देश में शान्ति और व्यवस्था बनी रहती है।

5. एक-सदस्यीय चुनाव-क्षेत्र (Single-Member Constituencies):
भारत में एक-सदस्यीय चुनाव-क्षेत्र बनाए जाते हैं। इसका अर्थ यह है कि चुनाव के समय सारे भारत या उस राज्य को, जिसमें कि चुनाव होना हो, बराबर जनसंख्या वाले चुनाव-क्षेत्रों में बाँट दिया जाता है और प्रत्येक क्षेत्र में से एक प्रतिनिधि चुना जाता है। पहले चुनावों में कुछ क्षेत्र दो सदस्यों वाले भी होते थे एक सदस्य साधारण जनता में से और दूसरा सुरक्षित स्थान से, परन्तु अब जिन क्षेत्रों में अनुसूचित जातियों अथवा जनजातियों की संख्या अधिक होती है, उस इलाके से उस जाति का एक ही सदस्य चुना जाता है और उसे सभी मतदाता चुनते हैं। इस प्रकार अब सभी चुनाव-क्षेत्र एक-सदस्यीय हैं। वर्तमान में भारतीय संघ को 543 संसदीय क्षेत्रों में बाँटा गया है।

6. आनुपातिक प्रतिनिधित्व और एकल संक्रमणीय मत प्रणाली (Proportional Representation and Single Transferable Vote System):
भारत में इस प्रकार की चुनाव-प्रणाली का प्रयोग भारतीय गणराज्य के राष्ट्रपति एवं उप-राष्ट्रपति के निर्वाचन के लिए किया जाता है। आनुपातिक प्रतिनिधित्व के अनुसार संसद व राज्य विधानसभाओं के सदस्यों को अनुपात के अनुसार समान मूल्य के मत देने का अधिकार होता है।

एकल संक्रमणीय प्रणाली के अनुसार मतदाता को उम्मीदवारों के नाम के आगे अपनी पसन्द लिखनी होती है। मतगणना के समय यदि प्रथम पसन्द वाला कोई व्यक्ति नहीं चुना जाता तो द्वितीय या उससे अगली पसन्द वालों के नाम मतों का संक्रमण हो जाता है। इस प्रकार किसी मतदाता का कोई मत व्यर्थ नहीं जाता।

7. चुनाव आयोग (Election Commission):
भारतीय संविधान द्वारा चुनावों के सुचारू रूप से संचालन के लिए चुनाव आयोग की व्यवस्था की गई है, जिसमें एक मुख्य चुनाव आयुक्त तथा दो अन्य चुनाव आयुक्त होते हैं। इनकी नियुक्ति राष्ट्रपति करता है। वर्तमान में श्री सुशील चंद्रा मुख्य निर्वाचन आयुक्त हैं एवं अन्य दो चुनाव आयुक्त बहु-सदस्यीय आयोग में कार्यरत हैं। इसी तरह भारत में बहु-सदस्यीय चुनाव आयोग कार्य कर रहा है।

8. स्थानों का आरक्षण (Reservation of Seats):
भारत में सभी नागरिकों को समानता का अधिकार दिया गया है। इसके साथ ही पिछड़े वर्गों, अनुसूचित जातियों तथा जनजातियों के लिए संसद, विधानसभाओं, नगरपालिकाओं तथा पंचायतों में स्थान आरक्षित किए गए हैं। कुछ निर्वाचन-क्षेत्र ऐसे रखे जाते हैं, जिनसे किसी पिछड़े वर्ग या अनुसूचित जाति अथवा जनजाति का सदस्य ही चुनाव लड़ सकता है।

आरम्भ में यह व्यवस्था केवल 10 वर्ष के लिए अर्थात् 1960 तक थी, परन्तु दस-दस वर्ष बढ़ाकर इसे लागू रखा गया। वर्तमान में 104वें संशोधन के द्वारा इसे बढ़ाकर 2030 तक कर दिया गया। यह व्यवस्था समाज के पिछड़े वर्ग को सुविधा प्रदान करके उन्हें शेष वर्गों के समान स्तर पर लाने के अभिप्राय से की गई है। यह व्यवस्था लोकतन्त्र के सिद्धान्तों के विरुद्ध नहीं, बल्कि उसे वास्तविक व व्यावहारिक बनाने के लिए की गई है।

9. चनाव याचिका (Election Petition):
भारत में चनाव सम्बन्धी झगडों को निपटाने के लिए चनाव याचिका की व्यवस्था की गई है। इसके अनुसार कोई भी उम्मीदवार या मतदाता यदि किसी चुनाव से सन्तुष्ट नहीं है या वह महसूस करता है कि किसी चुनाव-विशेष में भ्रष्ट साधन अपनाए गए हैं, तो वह अपनी याचिका सीधे उच्च न्यायालय के पास भेज देता है। यदि यह सिद्ध हो जाए कि किसी प्रतिनिधि ने चुनाव के समय भ्रष्ट साधन अपनाए हैं, तो न्यायालय उसके चुनाव को रद्द घोषित कर सकता है।

10. परिणाम साधारण बहुमत के आधार पर (Result on the basis of Simple Majority):
भारत के संविधान निर्माताओं , ने स्वतन्त्रता प्राप्ति के बाद भारत के लिए ब्रिटेन में 16वीं शताब्दी में प्रचलित चुनाव प्रणाली, जिसे ‘फर्स्ट पास्ट द पोस्ट सिस्टम’ (First Past the Post System) या एकल बहुमत प्रणाली (Single Plurality System) कहा जाता है, को अपनाना अधिक उचित समझा। इस प्रणाली के अन्तर्गत प्रत्येक निर्वाचन क्षेत्र से चुनाव लड़ने वाले सभी उम्मीदवारों में से सबसे अधिक मत पाने वाला उम्मीदवार लोकसभा/राज्यसभा विधानसभा विधान-परिषद्/स्थानीय नगर निकायों/पंचायत राज संस्थाओं के लिए निर्वाचित हो जाता है।

11. ऐच्छिक मतदान (Optional Voting):
चुनावों में प्रत्येक मतदाता द्वारा अपने मत का प्रयोग करना एवं न करना उसकी इच्छा पर निर्भर करता है। वह कानून द्वारा वोट डालने के लिए बाध्य नहीं है। यद्यपि गुजरात सरकार द्वारा स्थानीय स्तर (पंचायती राज संस्थाओं) पर मतदान के अनिवार्य सम्बन्धी कानून पास करके भारत में एक बार पुनः चर्चा का विषय बना दिया गया है कि . मतदान की अनिवार्यता के कानून को लागू करना कितना व्यावहारिक एवं सार्थक है। यहाँ यह उल्लेखनीय है कि सन् 2014 में लागू किए गए मतदान की अनिवार्यता सम्बन्धी कानून को लागू करने वाला गुजरात राज्य भारत का प्रथम राज्य बन गया है।

12. नोटा का प्रावधान (Provision of NOTA):
भारतीय चुनाव प्रणाली में नोटा (None of the Above) के प्रावधान को लागू करके चुनाव में एक नए विकल्प को भी जनता के लिए प्रदान किया गया है। इस विकल्प द्वारा मतदाता चुनाव में खड़े हुए सभी प्रत्याशियों को नकार सकता है। इस तरह यदि मतदाता चुनाव में खड़े हुए सभी प्रत्याशियों को अयोग्य या भ्रष्ट प्रवृत्ति का समझते हैं तो वे अपनी भावना को व्यक्त करने का पूर्ण अधिकार रखते हैं।

भारत में हुए 16वीं लोकसभा चुनाव, 2014 में प्रथम बार नकारात्मक मतदान के लिए नोटा (NOTA) विकल्प मतदाताओं को उपलब्ध करवाया गया। 16वीं लोकसभा चुनाव में कुल 59,97,054 मतदाताओं ने इस विकल्प का प्रयोग किया। यह सभी 543 लोकसभा सीटों के लिए पड़े कुल मतों का लगभग 1.1% है। यह व्यवस्था निश्चित जन-प्रतिनिधियों को जहाँ उत्तरदायित्वपूर्ण बना सकती है, वहाँ उन्हें इस बात के लिए भी प्रेरित कर सकती है कि वे वास्तव में समाज में एक सच्चे जन-सेवक के आदर्श के रूप में अपने-आपको प्रस्तुत करें, अन्यथा जनता उन्हें नकार भी सकती है।

प्रश्न 5.
भारतीय चुनाव-प्रणाली की मुख्य त्रुटियों (दोषों) का वर्णन कीजिए।
उत्तर:
भारत संसार का सबसे बड़ा लोकतन्त्र है। अब यहाँ लगभग 89 करोड़ 78 लाख से अधिक मतदाता अपने मत का प्रयोग करने का अधिकार रखते हैं और आने वाली सरकार का निर्णय करते हैं। भारत में अब तक लोकसभा के 17 चुनाव हो चुके हैं तथा विभिन्न राज्य विधानसभाओं के चुनाव हो चुके हैं। इन चुनावों की आम प्रशंसा भी हुई है, परन्तु भारतीय चुनाव-प्रणाली में कुछ उभरे दोषों सम्बन्धी तथ्य अब छिपे हुए नहीं हैं।

सामान्य तौर पर उम्मीदवारों की अपराधिक पृष्ठभूमि, अवैध तरीके से जमा की गई प्रत्याशियों की अचूक सम्पत्ति, अपराधिक व्यक्तियों से प्रत्याशियों की साँठ-गाँठ, मतदान केन्द्रों पर जबरन कब्जा कर लेने, फायरिंग करके या डरा-धमका कर वैध मतदाताओं को मतदान करने से रोकने पर फर्जी मतदान करने जैसी घटनाएँ एवं दोष भारतीय चुनाव प्रणाली में प्रायः देखी जा सकती है।

इसलिए भारत में चुनाव सुधार का मुद्दा सबसे अधिक चर्चित विषय रहता है। यद्यपि इस सम्बन्ध में भारतीय चुनाव आयोग, सरकार एवं न्यायपालिका ने समय-समय पर परिस्थितियों के अनुसार अपना-अपना सहयोग दिया है, परन्तु अब भी चुनाव-सुधार के लिए वास्तविक क्रिया चयन हेतु महत्त्वपूर्ण कदम उठाए जाने की आवश्यकता है। यहाँ हम चुनाव-सुधार सम्बन्धी सुझावों पर चर्चा करने से पूर्व भारतीय चुनाव प्रणाली के दोषों पर दृष्टिपात कर रहे हैं जो निम्नलिखित हैं

1. चुनावों में धन की बढ़ती हुई भूमिका (The Increasing Role of Money in Elections):
भारतीय चुनाव-प्रणाली का सबसे बड़ा दोष चुनावों में धन की बढ़ती हुई भूमिका है। भारतीय चुनावों में धन के अन्धाधुन्ध प्रयोग और दुरुपयोग ने भारत की राजनीति को काफी भ्रष्ट किया है। भारत में काले धन का बड़ा बोलबाला है और उसका चुनावों में दिल खोलकर प्रयोग किया जाता है।

मतदाताओं के लिए शराब के दौर चलाए जाते हैं, मत खरीदे जाते हैं, उम्मीदवारों को धनी लोगों द्वारा खड़ा किया जाता है और पैसे के बल पर उम्मीदवारों को बैठाया जाता है तथा मतदाताओं को लाने व ले जाने के लिए गाड़ियों का प्रयोग किया जाता है। आज का चुनाव पैसे के बल पर ही जीता जा सकता है और इस धन ने मतदाताओं, राजनीतिक दलों तथा प्रतिनिधियों आदि सबको भ्रष्ट बना दिया है।

सार्वजनिक जीवन से सम्बन्ध रखने वाले व्यक्तियों का कहना है कि लोकसभा के एक उम्मीदवार ने 2 से 10 करोड़ रुपए तक खर्च किए हैं अर्थात् इतना चुनाव खर्च राजनीतिक व आर्थिक स्थिति को दूषित कर रहा है। एक अनुमान के अनुसार, समय के साथ भारत में लोकसभा एवं विधानसभाओं के चुनावों की लागत (राजकोष पर भारित) तथा उम्मीदवारों एवं राजनीतिक दलों द्वारा किए जाने वाले खर्च का परिमाण सन् 1952 के खर्चों की तुलना में सन् 2014 में बढ़कर लगभग 328 गुना हो गया।

यहाँ तक कि राजनीतिक दलों एवं उम्मीदवारों द्वारा किए जाने वाले व्यय में तो 500 गुना से भी अधिक वृद्धि हो गई है। एक तथ्य के अनुसार सन् 1952 के लोकसभा चुनावों पर मात्र 10.45 करोड़ रुपए खर्च किए गए थे जिससे प्रति व्यक्ति चुनाव खर्च लगभग 0.60 रुपए था तथा सन् 2019 के 17वीं लोकसभा चुनाव में कुल खर्च लगभग 70 हजार करोड़ रुपए अनुमानित है जिससे प्रति व्यक्ति खर्च बढ़कर लगभग 60 रुपए हो गया है।

भारतीय लोकतन्त्र धीरे-धीरे ऐसे दौर में पहुंच गया है जहाँ किसी भी सामान्य पृष्ठभूमि के कार्यकर्ता के लिए किसी राजनीतिक दल का टिकट प्राप्त करना तथा चुनाव लड़ना लगभग असम्भव हो गया है। एक तथ्य के अनुसार, 17वीं लोकसभा में 88 प्रतिशत सांसदों की सम्पत्ति औसतन एक-एक करोड़ रुपए से अधिक थी जबकि सन् 2004, 2009 एवं 2014 में करोड़पति सांसदों का औसत क्रमशः लगभग 30 प्रतिशत एवं 82 प्रतिशत था।

इस तरह स्पष्ट है कि राजनीति में धनाढ्य लोगों का ही प्रभुत्व बनता जा रहा है जो भारतीय लोकतन्त्र एवं चुनाव प्रणाली के लिए शुभ संकेत नहीं कहा जा सकता। निश्चित ही चुनाव प्रबंधन में काले धन के उपयोग से भारतीय चुनाव परिदृश्य अत्यन्त भ्रष्ट, कलुषित, फिजूलखर्ची एवं हिंसक होता जा रहा है।

2. जाति और धर्म के नाम पर वोट (Voting on the basis of Caste and Religion):
भारत में साम्प्रदायिकता का बड़ा प्रभाव है। जाति और धर्म के नाम पर खुले रूप से मत माँगे और डाले जाते हैं। राजनीतिक दल भी अपने उम्मीदवार खड़े करते समय इस बात को ध्यान में रखते हैं और उसी जाति और धर्म का उम्मीदवार खड़ा करने का प्रयत्न करते हैं, जिस जाति का उस निर्वाचन-क्षेत्र में बहुमत हो। भारत में अब तक जो चुनाव हुए हैं, उनके आँकड़े भी इस बात का समर्थन करते हैं।

यद्यपि 16वीं एवं 17वीं लोकसभा चुनाव में प्रधानमन्त्री श्री नरेन्द्र मोदी के चमत्कारिक नेतृत्व को मिला अपार जनसमर्थन यह दर्शा रहा है कि भारत में भ्रष्टाचार को समाप्त करने एवं विकास तथा सुशासन के लिए आम जनता ने जाति एवं धर्म के आधार से ऊपर उठकर मतदान किया है। यही कारण है कि 16वीं लोकसभा चुनाव के बाद लगभग 30 वर्षों के बाद भारत में एक दल वाली स्पष्ट बहुमत की सरकार बनी। वहाँ 17वीं लोकसभा चुनाव के बाद मोदी नेतृत्व वाली भाजपा सरकार को पुनः पूर्णतः स्पष्ट जनसमर्थन मिला।

3. राजनीतिक दलों को मतों के अनुपात से स्थानों की प्राप्ति न होना (Political Parties not get the seats in Proportion to Votes):
यह भी देखा गया है कि चुनावों में राजनीतिक दलों को उस अनुपात में विधानमण्डलों में स्थान प्राप्त नहीं होते, जिस अनुपात में उन्हें मत प्राप्त होते हैं। जैसे भारत में 13वीं लोकसभा के चुनाव के समय भारतीय जनता पार्टी को 182 स्थान मिले और मतों का प्रतिशत 23.70 प्रतिशत रहा, जबकि काँग्रेस को 28.42 प्रतिशत मत मिलने के पश्चात् भी 114 स्थान ही प्राप्त हुए, फिर भी काँग्रेस का मत-प्रतिशत भाजपा से 5 प्रतिशत अधिक है। इससे दलों में मायूसी का पैदा होना स्वाभाविक है। इस प्रकार एक लोकतान्त्रिक व्यवस्था के अन्तर्गत इस स्थिति को न्यायपूर्ण नहीं कहा जा सकता।

4. मतदाताओं की अनुपस्थिति (Absence of the Voters):
चुनावों में बहुत-से मतदाता चुनावों में रुचि लेते ही नहीं। उनके लिए वोट डालना एक समस्या बन गई है। वे मतपत्र का प्रयोग करते ही नहीं। मत का प्रयोग न करना एक प्रकार से लोकतन्त्र को धोखा देना ही है। अक्सर देखने में आता है कि मतदान लगभग 60% होता है। मतदान का प्रतिशत कई चुनावों में तो 60% से भी कम रहता है।

इससे यह होता है कि कम वोट प्राप्त करने वाले दल को अधिक सीटें मिल जाती हैं; जैसे 13वीं लोकसभा के चुनाव, जो सन् 1999 में हुए, में 59.30 प्रतिशत मतदाताओं ने 14वीं लोकसभा चुनाव में 58 प्रतिशत मतदाताओं ने, 15वीं लोकसभा चुनाव, 2009 में भी 71 करोड़ 40 लाख मतदाताओं में से केवल 42 करोड़ 80 लाख मतदाताओं ने, 16वीं लोकसभा चुनाव में कुल पंजीकृत 83.41 करोड़ मतदाताओं में से केवल 66.4 प्रतिशत मतदाताओं ने तथा 17वीं लोकसभा चुनाव, 2019 में 89.78 करोड़ मतदाताओं में से 67.11 प्रतिशत मतदाताओं ने ही भाग लिया।

यानि चुनाव आयोग एवं विभिन्न सामाजिक संगठनों के प्रयास के बावजूद आज भी लगभग 33 प्रतिशत मतदाता अपने पवित्र एवं अमूल्य मताधिकार के प्रति जागरूक नहीं हैं। ऐसी स्थिति में यह कहना बहत कठिन है कि जीतने वाला प्रत्याशी लोकतान्त्रिक भावना के अनुरूप बहमत

5. चुनाव नियमों में दोष (Defective Election Rules):
यह भी देखा गया है कि चुनाव सम्बन्धी नियमों में बड़ी कमियाँ हैं और इसी कारण चुनाव में बरती गई अनियमितताओं को न्यायालय के सामने सिद्ध करना बड़ा कठिन हो जाता है। लगभग सभी दल धर्म और जाति के आधार पर अपने उम्मीदवार चुनते हैं तथा इसी के नाम पर वोट माँगते हैं, परन्तु इसको सिद्ध करना कठिन है। सभी उम्मीदवार मतदाताओं को लाने व ले जाने के लिए गाड़ियों का प्रयोग करते हैं, परन्तु यह बात चुनाव-याचिका की सुनवाई के समय सिद्ध नहीं हो पाती।

6. राजनीति का अपराधीकरण (Criminalization of Politics):
पिछले कुछ वर्षों में भारतीय चुनाव-प्रणाली में एक और दोषपूर्ण मोड़ आया है। प्रायः सभी राजनीतिक दलों ने ऐसे बहुत-से उम्मीदवार चुनाव में खड़े किए, जिनका अपराधों की दनिया में नाम था। ऐसे व्यक्तियों ने राजनीति में अपराधीकरण को बढ़ावा देने का काम किया और लोगों को भय दिखाकर वोट माँगे। जब अपराधी, तस्कर और लुटेरे पहले किसी दल के सक्रिय सदस्य तथा बाद में विधायक बन जाएँ तो उस देश के भविष्य के उज्ज्वल होने की आशा नहीं की जा सकती।

यदि एक तथ्य पर नजर डाली जाए तो राजनीति में अपराधीकरण की प्रवृत्ति स्वतः ही स्पष्ट हो जाती है; जैसे 14वीं लोकसभा में 128 सांसद ऐसे थे जिन पर विभिन्न मामलों पर अपराधिक मामले चल रहे थे जबकि 15वीं लोकसभा में यह संख्या बढ़कर 162 हो गई। जो कुल सांसदों का 30 प्रतिशत थी। एक तथ्य के अनुसार 162 में से 78 सांसदों के विरुद्ध गम्भीर अपराधिक मामले दर्ज थे।

इसके अतिरिक्त 16वीं लोकसभा, (2014) के निर्वाचित सांसदों पर ‘एसोसिएशन फॉर डेमोक्रेटिक रिफार्स’ के द्वारा निर्वाचित सांसदों के दिए गए शपथ पत्रों के विश्लेषण के आधार पर निकाले गए निष्कर्ष के अनुसार 34 प्रतिशत सांसदों (185) के विरुद्ध विभिन्न न्यायालयों में अपराधिक मामले लम्बित हैं। इनमें से 112 सांसदों के विरुद्ध तो हत्या, हत्या का प्रयास, आगजनी, अपहरण जैसे गम्भीर प्रवृत्ति के मामले दर्ज हैं।

सन् 2019 में हुए 17वीं लोकसभा चुनाव में लगभग 233 सांसदों के विरुद्ध आपराधिक मामले लम्बित हैं जिनमें बीजेपी के 116, कांग्रेस के 29, जनता दल के 13, डी.एम.के. के 10 एवं टी०एम०सी० के 9 सांसद सम्मिलित हैं जो कुल सांसदों का 43 प्रतिशत हैं। इनमें से 29 प्रतिशत सांसदों पर गम्भीर जुर्म (हत्या, घर में घुसना, डकैती, फिरौती की मांग, धमकाने आदि) के आरोप हैं। इस प्रकार स्पष्ट है कि भारतीय राजनीति में अपराधीकरण की प्रवृत्ति निरन्तर बढ़ती जा रही है जो कि चिन्तनीय है।

7. जाली वोट की समस्या (Problem of Impersonation):
भारतीय चुनाव-प्रणाली का एक और महत्त्वपूर्ण दोष है जाली मतदान। चुनावों में जाली मतदान किया जाता है। यहाँ तक कि मृत व्यक्तियों के वोटों का भुगतान भी होता है। वास्तविकता यह है कि जाली मतदान भारतीय लोकतन्त्र के लिए खतरा बनता जा रहा है। इससे बड़ा खतरा तो यह है कि जाली मतदान विभिन्न राजनीतिक दलों द्वारा संगठित तौर पर करवाया जाता है।

8. बहुत अधिक चुनाव प्रत्याशी (Too many Candidates for Election):
भारतीय चुनाव-प्रणाली में एक और दोष है उम्मीदवारों की बढ़ती हुई संख्या। सन् 1996 के लोकसभा के चुनावों में उम्मीदवारों की संख्या 13,952 थी, जो फरवरी-मार्च, 1998 के आम चुनावों में बढ़कर 47,501 हो गई।

1999 में हुए 13वीं लोकसभा के चुनावों में भी कुल प्रत्याशियों की संख्या 4,648 थी जोकि 15वीं लोकसभा चुनाव में कुल प्रत्याशियों की संख्या बढ़कर 8070 हो गई और अप्रैल-मई, 2014 में हुए 16वीं लोकसभा चुनाव में कुल प्रत्याशियों की संख्या बढ़कर 8251 हो गई। यद्यपि 17वीं लोकसभा चुनाव में प्रत्याशियों की संख्या 8040 ही रही जो कि 16वीं लोकसभा से कम है।

17वीं लोकसभा चुनाव के समय 7 राष्ट्रीय एवं 59 राज्य स्तरीय मान्यता प्राप्त दल थे परन्तु अत्यधिक प्रत्याशियों के कारण जनमत का ठीक प्रदर्शन नहीं हो पाता। वोटें अधिक भागों में बंट जाती हैं। बहुत कम वोट प्राप्त करने वाले उम्मीदवार की जीतने की संभावना रहती है। इस समस्या को निर्दलीय उम्मीदवारों की बढ़ती हुई संख्या ने और भी जटिल बना दिया है। यद्यपि सन् 1999, 2004, 2009, 2014 एवं 2019 के लोकसभा चुनावों में निर्दलीय प्रत्याशियों की संख्या में कमी अवश्य हुई है फिर भी निर्दलीय प्रत्याशियों का अस्तित्व जनमत विभाजन की दृष्टि से निश्चित ही हानिकारक माना जाता है।

9. प्रतिनिधियों को वापस बुलाने की व्यवस्था का न होना (No Provision for Recall of Representatives):
प्रतिनिधियों को अपने उत्तरदायित्व का आभास करवाने के लिए उन्हें वापस बुलाने की व्यवस्था होनी चाहिए, जिससे प्रतिनिधि वापस बुलाए जाने के भय से अपने कर्तव्यों का ठीक ढंग से पालन करेंगे। इस प्रकार की व्यवस्था स्विट्ज़रलैण्ड में है, जबकि भारत में .इस व्यवस्था का अभाव है।

10. अत्यधिक राजनीतिक दल (Too many Political Parties):
भारत एक बहुदलीय प्रणाली वाला देश है। भारत की बहुदलीय प्रणाली के कारण हुए जनमत विभाजन के परिणामस्वरूप सन् 2014 के 16वीं लोकसभा से पूर्व तक पिछले तीन दशकों से भारत में गठबन्धन सरकार का युग रहा ।

यद्यपि 17वीं लोकसभा चुनाव में गठबन्धन सरकार के बावजूद एन०डी०ए० गठबन्धन का नेतृत्व कर रही भारतीय जनता पार्टी को अकेले. ही 303 लोकसभा सीटों पर स्पष्ट बहुमत प्राप्त हुआ, परन्तु फिर भी भारतीय राजनीति में अत्यधिक राजनीतिक दलों का होना जिनमें से अधिकांश राजनीतिक दृष्टि से सक्रिय भी नहीं होते कोई अधिक शुभ संकेत नहीं है।

एक तथ्य के अनुसार 17वीं लोकसभा चुनाव, 2019 के समय चुनाव आयोग के पास लगभग 2293 राजनीतिक दल पंजीकृत थे, जिसमें केवल 7 राष्ट्रीय स्तर एवं 59 राज्य स्तरीय राजनीतिक दल थे। शेष राजनीतिक दलों को चुनाव आयोग की मान्यता प्राप्त नहीं थी। इससे स्पष्ट है कि भारत में बने अधिकांश राजनीतिक दल ऐसे हैं जो राजनीतिक सक्रियता नहीं रखते हैं। अतः भारतीय चुनाव प्रणाली की यह अपनी ही ऐसी व्यवस्था बन गई है जिसमें गैर-मान्यता प्राप्त दल भी चुनावों में भाग लेते हैं।

11. चुनावों में हिंसा का प्रयोग (Use of Violence in Elections):
भारतीय चुनाव प्रणाली का एक अन्य महत्त्वपूर्ण दोष . यह भी है कि चुनावी प्रक्रिया में हिंसा की घटनाएँ होती रहती हैं। हिंसा एवं शारीरिक बल से कई जगह मतदान केन्द्रों पर जबरन कब्जे की कोशिश की जाती है जिसके कारण कई बार कुछ मतदान केन्द्रों पर पुनर्मतदान भी करवाना पड़ता है। इस तरह हिंसक घटनाओं से जहाँ राजनीतिक वातावरण दूषित होता है वहाँ आम जनता का भी लोकतन्त्रीय प्रणाली से मोह भंग होता है।

HBSE 11th Class Political Science Important Questions Chapter 3 चुनाव और प्रतिनिधित्व

प्रश्न 6.
भारतीय चुनाव प्रणाली की त्रुटियों को दूर करने के लिए सुझाव दीजिए।
उत्तर:
भारत में वर्तमान चुनाव पद्धति 1952 से कार्य करती चली आ रही है। इस चुनाव पद्धति में अनेक दोष देखने को मिले हैं। समय-समय पर राजनीतिक विद्वानों व राजनीतिज्ञों ने विभिन्न सुधार एवं सुझाव प्रस्तुत किए हैं। भारतीय चुनावों के दोषों को दूर करने के लिए दिए गए सुझावों का ब्यौरा अग्रलिखित है

1. चुनाव आयोग को अधिक शक्तिशाली और प्रभावी बनाया जाना (Making Election Commission more Powerful and Effective):
चुनावी अनियमितताओं को ध्यान में रखते हुए यह आवश्यक है कि चुनाव आयोग को अधिक प्रभावी और शक्तिशाली बनाया जाए। सरकार इस सुझाव पर विचार कर रही है कि चुनाव आयोग को दीवानी न्यायालयों (Civil Courts) द्वारा प्रयुक्त निम्नलिखित शक्तियाँ प्रदान की जाएँ

(1) ऐसी शक्ति जिससे वह यह निर्णय दे सके कि चुनावी भ्रष्टाचार के कारण कोई व्यक्ति संसद या विधानसभा का सदस्य बना रहने के योग्य नहीं है।

(2) चुनावी अनियमितताओं की जांच-पड़ताल करने का अधिकार ।

(3) चुनावी अपराध सिद्ध हो जाने पर लोगों को दंडित करने का अधिकार, जिसमें जुर्माना वसूल करना या हर्जाना दिलाना शामिल है। एक सुझाव यह दिया जा सकता है कि ‘चुनाव आयोग’ के पास अपनी ‘स्वतन्त्र निधि’ होनी चाहिए जिससे हर छोटी-बड़ी बात के लिए उसे राज्य सरकारों का मुंह न ताकना पड़े।

2. चुनाव आयोग का पुनर्गठन (Reorganization of Election Commission):
भारतीय संविधान के अनुच्छेद 324 के अनुसार चुनाव आयोग बहु-सदस्यीय हो सकता है; परन्तु काफी समय तक चुनाव आयोग एक-सदस्यीय रहा। इसे बहु-सदस्यीय बनाने की माँग जोर पकड़ती गई और इसे अब बहु-सदस्यीय बना दिया गया है। वर्तमान चुनाव आयोग के सदस्य मुख्य निर्वाचन आयुक्त सहित तीन हैं। मुख्य चुनाव आयुक्त की नियुक्ति राष्ट्रपति द्वारा प्रधानमन्त्री के परामर्श पर की जाती है।

इसके लिए सुझाव दिया गया है कि चुनाव आयोग का गठन वैसे ही किया जाए, जैसे संघ लोक सेवा आयोग का होता है। मुख्य चुनाव आयुक्त की नियुक्ति के लिए एक समिति बनाई जाए, जिसमें प्रधानमन्त्री, मुख्य न्यायाधीश व संसद में प्रतिपक्ष का नेता हो। मुख्य चुनाव आयुक्त व अन्य आयुक्तों की योग्यताएँ संविधान में निर्धारित की जाएँ। मुख्य चुनाव आयुक्त व अन्य आयुक्तों का कार्यकाल पांच वर्ष निर्धारित किया गया है।

इनकी पुनः नियुक्ति राष्ट्रपति व प्रधानमन्त्री पर आधारित है। वे जितनी बार चाहें, उन्हें पुनः नियुक्त कर सकते हैं। संविधान में यह धारा चुनाव आयोग की स्वतन्त्रता के विरुद्ध है। इसके कारण मुख्य चुनाव आयुक्त व अन्य आयुक्तों की ईमानदारी पर सन्देह किया जा सकता है। इस कारण सुझाव दिया गया है कि मुख्य चुनाव आयुक्त व अन्य आयुक्तों का निश्चित कार्यकाल हो।

3. आनुपातिक प्रतिनिधित्व प्रणाली शुरू करना (To Start Proportional Representation System):
भारत में चुनाव प्रणाली का सबसे बड़ा दोष यह है कि कई बार राजनीतिक दल कम मत प्राप्त करते हैं, परन्तु उन्हें अधिक स्थान प्राप्त हो जाते हैं। उदाहरणतः सन् 1980 में काँग्रेस को लोकसभा के लिए केवल 42.6% मत मिले, परन्तु संसद में 67% स्थान मिले। इस बुराई को समाप्त करने के लिए आनुपातिक प्रतिनिधित्व प्रणाली का सुझाव दिया गया है। यद्यपि भारत में इसे लागू करना कठिन कार्य है।

4. मतदाता पहचान-पत्र (Identity Cards for the Voters):
चुनाव आयोग के द्वारा 28 अगस्त, 1993 को मतदान हेतु अनिवार्य पहचान-पत्र बनाने का निर्णय लिया गया था। इस सम्बन्ध में मुख्य चुनाव आयुक्त ने राज्य सरकारों से कहा है कि मतदाताओं को पहचान-पत्र जारी कर दें। इस सुझाव के सम्बन्ध में सभी पार्टियाँ सहमत हैं जैसे कि पूर्व चुनावों की भाँति 17वीं लोकसभा चुनाव, 2019 में भी पहचान-पत्र मतदाता के लिए अनिवार्य कर दिया गया, हालांकि चुनाव आयोग द्वारा पहचान-पत्र सभी मतदाताओं को उपलब्ध न कराए जाने के कारण पहचान के लिए नए प्रमाण-पत्र; जैसे राशन-कार्ड, बस-पास, विद्यार्थी पहचान पत्र, अंक तालिका आदि को भी मान्य कर दिया गया है।

17वीं लोकसभा चुनाव में इसका आशातीत परिणाम सामने आया और फर्जी मतदान रोकने में विशेष सहायता मिली। पहचान-पत्र की उपयोगिता मतदान केन्द्रों तक ही सीमित नहीं। बैंक, अदालत, सरकारी सेवा और सरकारी कार्य व्यवहार में भी इससे अनुकूल सहायता मिल सकती है और नागरिकों की कई कठिनाइयाँ दूर हो सकती हैं। अतः भारतीय लोकतन्त्र की सुरक्षा एवं निष्पक्ष चुनाव करवाने की दृष्टि से हमें सभी मतदाताओं के फोटोयुक्त पहचान-पत्र बनाने के लिए निरन्तर सरकारी तन्त्र की ओर से सार्थक प्रयास करना बहुत आवश्यक है जिससे प्रत्येक मतदाता की चुनाव में निष्पक्ष एवं सही सहभागिता सुनिश्चित की जा सके।

5. सरकारी तन्त्र का दुरुपयोग नहीं होना चाहिए (There should not be misuse of the Official Machinery):
चुनावों के अवसर पर प्रायः शासन-तन्त्र का दुरुपयोग किया जाता है। वोट बटोरने के लिए मन्त्रियों द्वारा लोगों को तरह-तरह के आश्वासन दिए जाते हैं तथा उद्घाटन और शिलान्यास के बहाने सरकारी गाड़ियों और दूरदर्शन सुविधाओं का दुरुपयोग किया जाता है।

सन् 1968 में उच्चतम न्यायालय ने ‘घासी बनाम दलसिंह’ नामक केस में यह कहा था कि चुनाव से पूर्व पानी, बिजली, सड़कों की सफाई आदि पर राजकोष से व्यय करके चुनावों को प्रभावित करना ‘बुरा आचरण’ (Bad Practice) है। सरकारी खर्च से विज्ञापन छपवाना या सरकारी तन्त्र का चुनाव-कार्य के लिए प्रयोग पूरी तरह से निषिद्ध किया जाए।

6. चुनावों के व्यय के बारे में सुझाव (Suggestions about the Expenditure of Elections):
चुनाव आयोग ने समय-समय पर होने वाले चुनाव पर होने वाले व्यय को निर्धारित किया है। पहले दो चुनावों में विधानसभा के लिए अधिकतम सात हजार व लोकसभा के लिए 25 हजार रुपए निर्धारित किए गए थे।

जबकि समय-समय किए गए संशोधनों के पश्चात् वर्तमान में 1 मार्च, 2014 से प्रभावी नवीन संशोधन के अनुसार व्यय राशि को बढ़ाकर लोकसभा चुनाव हेतु इन राज्यों; जैसे आंध्र प्रदेश, असम बिहार, छत्तीसगढ़, गुजरात, हरियाणा, पंजाब, मध्यप्रदेश, महाराष्ट्र, पश्चिम बंगाल, ओडिशा, कर्नाटक, केरल, तमिलनाडु तथा दिल्ली आदि के लिए 70 लाख रुपए जबकि राज्य विधानसभा चुनाव के लिए उपर्युक्त राज्यों के अतिरिक्त उत्तराखण्ड एवं हिमाचल प्रदेश के लिए 28 लाख रुपए की व्यय राशि निर्धारित की गई।

शेष राज्यों के लोकसभा चुनावों के लिए 54 लाख रुपए, जबकि शेष राज्यों के विधानसभा चुनाव के लिए 20 लाख रुपए की व्यय राशि निर्धारित की गई इसके अतिरिक्त इस चुनाव खर्च को रोकने के लिए विभिन्न सुझाव दिए गए हैं

(1) राजनीतिक दलों द्वारा चुनाव पर किया गया खर्च उम्मीदवार के खर्चे की सीमा में शामिल किया जाना चाहिए।

(2) राजनीतिक दलों के आय-व्यय विवरण की विधिवत जांच की जानी चाहिए। प्रत्येक राजनीतिक दल के लिए आय-व्यय का समस्त विवरण रखा जाना अनिवार्य किया जाए और उसकी जांच मुख्य चुनाव आयुक्त द्वारा नियुक्ति किए गए लेखा परीक्षक द्वारा होनी चाहिए।

(3) चुनाव प्रचार की अवधि में कमी करनी चाहिए, ताकि चुनाव खर्च कम हो सके। सुझाव है कि अवधि 10 दिन से भी कम होनी चाहिए।

(4) संसद और विधानसभा के चुनाव साथ-साथ होने चाहिएँ। सन् 1996 के चुनावों में यह प्रयत्न किया गया था कि लोकसभा व विधानसभा के चुनाव साथ-साथ हों।

(5) चुनाव अवधि में सार्वजनिक संस्थाओं को अनुदान देने पर रोक लगानी चाहिए।

(6) चुनाव खर्च का कुछ भाग राज्य द्वारा वहन किया जाना चाहिए। वर्तमान समय में विश्व के कुछ देशों के राज्य चुनाव के सारे खर्च वहन करते हैं; जैसे स्वीडन, फ्राँस आदि में। कुछ देशों में मिश्रित चुनाव व्यय की व्यवस्था है जिसमें सरकार खर्च का कुछ भाग देती है।

(7) पार्टियों को चन्दा मात्र चैक द्वारा दिया जाना चाहिए।

7. चुनाव याचिकाओं के बारे में सुझाव (Suggestions about the Election Petitions):
गत अनेक वर्षों से चुनाव याचिकाएँ उच्च न्यायालयों के समक्ष आई हैं, जिनका निपटारा करने में कई वर्ष लगे हैं। चुनाव याचिकाएँ चुनावों से भी अधिक खर्चीली तथा कष्टमय बन गई हैं। इसलिए इन याचिकाओं का निपटारा शीघ्र होना चाहिए, ताकि इसके दोनों पक्षों को बेकार की मुसीबत तथा खर्च से छुटकारा मिल सके।

8. निर्दलीय उम्मीदवारों पर रोक (Restrictions on Independent Candidates):
निर्दलीय उम्मीदवार निष्पक्ष चुनावों के लिए एक समस्या हैं, इसलिए निर्दलीय उम्मीदवारों पर रोक लगनी चाहिए। यद्यपि कानून द्वारा स्वतन्त्र उम्मीदवारों को रोका नहीं जा सकता, लेकिन फिर भी ऐसा कुछ अवश्य होना चाहिए कि मज़ाक के लिए चुनाव लड़ने वालों पर रोक लगे।

इस सम्बन्ध में यह सुझाव दिया है कि एक तो जमानत की राशि को बढ़ा देना चाहिए। जुलाई, 1996 में चुनाव आयोग की एक बैठक हुई, जिसमें जमानत की राशि बढ़ाने का फैसला लिया गया और लोकसभा के लिए यह राशि बढ़ाकर 25,000 रुपए और विधानसभा के लिए 10,000 रुपए कर दी गई है।

दूसरे यह व्यवस्था होनी चाहिए कि जिस निर्दलीय उम्मीदवार को निश्चित प्रतिशत से कम वोट प्राप्त होते हैं उसे अगले चुनाव में भाग लेने का अधिकार नहीं होगा। इस प्रकार की व्यवस्था से निश्चित रूप से निर्दलीय उम्मीदवारों पर रोक लगेगी। जैसे 15वीं लोकसभा चुनाव में केवल 9 सदस्य, 16वीं एवं 17वीं लोकसभा के चुनावों में भी केवल 3-3 प्रत्याशी ही निर्दलीय तौर पर विजयी रहे। स्पष्ट है कि निर्दलीय प्रत्याशियों के प्रति जनता का रुझान कम हुआ है।

9. वोटिंग मशीनों का प्रयोग (Use of Voting Machines):
सामान्यतः देखा गया है कि तस्कर, माफ़िया और गुंडा तत्त्व को तथाकथित कुछ जन-प्रतिनिधियों का संरक्षण प्राप्त होता है और वे शासन पर हावी हो जाते हैं। इस स्थिति को दूर करने के लिए सुझाव दिया गया है कि संवेदनशील केंद्रों पर सुरक्षा को बढ़ा दिया जाना चाहिए और चुनाव में वोटिंग मशीनों का प्रयोग किया ।

जा सकता है। सन् 1999 में 13वीं लोकसभा के चुनावों में राज्यों के 46 निर्वाचन क्षेत्रों में वोटिंग मशीन का प्रयोग कर सफल मतदान सम्भव करवाया गया। 22 फरवरी, 2000 को हुए हरियाणा राज्य के विधानसभा चुनावों में भी 90 निर्वाचन क्षेत्रों में से 45 निर्वाचन क्षेत्रों में वोटिंग मशीन का प्रयोग किया गया। ऐसे में हरियाणा भारत का पहला राज्य बन गया, जहाँ वोटिंग मशीन द्वारा चुनाव सम्पन्न हुआ। 14वीं, 15वीं, 16वीं एवं 17वीं लोकसभा के चुनाव भी पूर्णतः वोटिंग मशीन द्वारा सम्पन्न हुए थे।

10. अपराधीकरण पर अंकुश (Check on Criminalization):
भारतीय राजनीति में बढ़ती अपराधीकरण की प्रवृत्ति ने भारतीय लोकतन्त्र को सबसे अधिक प्रभावित किया है। इसलिए लोकतन्त्र की रक्षा हेतु आवश्यक है कि ऐसी प्रवृत्ति पर अंकुश हेतु तुरन्त कानून बनाया जाए। पूर्व राष्ट्रपति डॉ० के०आर० नारायणन ने भी कहा था कि, “आपराधिक पृष्ठभूमि और माफिया से संबंध रखने वाले लोगों को चुनाव प्रक्रिया से बाहर रखने के लिए कानून बनाया जाए।” अतः ऐसी प्रवृत्ति पर अंकुश आवश्यक है।

11. अन्य प्रयास (Other Efforts):
भारतीय चुनाव-प्रणाली में सुधार किए जाने का सिलसिला जारी है। इस संबंध में तारकुंडे समिति, गोस्वामी समिति तथा चुनाव आयुक्त आर०के० द्विवेदी ने कुछ सुझाव दिए। इसके अतिरिक्त 23 जुलाई, 1996 को चनाव आयोग और राजनीतिक दलों की एक बैठक हुई, जिसमें कछ सधारों पर विचार किया गया तथा कछ सुधारों पर सहमा हो गई।

इस बैठक में कुछ निर्णय लिए गए और चुनाव संशोधन किए गए। जन-प्रतिनिधि अधिनियम (The Representation of the People Act) में संशोधन किया गया है। इस संशोधन को राष्ट्रपति ने 1 अगस्त, 1996 को अपनी स्वीकृति प्रदान कर दी है। कुछ मुख्य संशोधन निम्नलिखित प्रकार से हैं-

(1) इस चुनाव सुधार संशोधन द्वारा चुनाव प्रचार का समय 21 दिन से घटाकर 14 दिन कर दिया गया है। उम्मीदवारों के लिए जमानत की राशि बढ़ा दी गई है,

(2) लोकसभा के उम्मीदवार के लिए जमानत की राशि बढ़ाकर 25,000 रुपए और विधानसभा के उम्मीदवार के लिए 10,000 रुपए कर दी गई है। आरक्षित लोकसभा चुनाव क्षेत्र के लिए जमानत की राशि 12,500 रुपए तथा विधानसभा चुनाव क्षेत्र के लिए 5,000 रुपए निश्चित की गई है,

(3) इसके साथ ही अरुचिकर उम्मीदवारों (Non-serious Candidates) की संख्या कम करने के लिए नाम पेश करने वालों की संख्या बढ़ाकर 10 कर दी गई है,

(4) चुनाव अधिकारियों या चुनाव पर्यवेक्षकों को मतदान केंद्र के कब्जे के मामले में गिनती रोकने के अधिकार दे दिए गए हैं,

(5) नए कानून के अधीन चुनाव से 48 घंटे पहले शराब के बेचने पर प्रतिबन्ध लगा दिया गया है तथा मत के पास शस्त्र लेकर घूमना अपराध घोषित कर दिया गया है।

12. चुनाव आयोग द्वारा जारी किए गए नए दिशा-निर्देश (New Guidelines issued by the Election Commission):
सर्वोच्च न्यायालय के निर्णय के आधार पर 1 अप्रैल, 2003 को चुनाव आयोग ने एक आदेश जारी किया है जिसके अनुसार संसद तथा विधानसभाओं के चुनावों में उम्मीदवार को नामांकन-पत्र के साथ एक हलफ़नामा (Affidavit) देना जरूरी होगा जिसमें उसे अपने क्रिमिनल बैकग्राऊंड (Criminal Background) का भी उल्लेख करना होगा।

नामांकन-पत्र के साथ ही उसे अपनी सम्पत्ति और देनदारी तथा शैक्षिक योग्यता की जानकारी भी हलफनामे में देनी होगी। आयोग ने यह भी स्पष्ट किया है कि यदि कोई उम्मीदवार नामांकन के समय यह जानकारी हलफनामे में नहीं देगा तो यह सर्वोच्च न्यायालय के आदेश का उल्लंघन माना जाएगा और चुनाव अधिकारी को जांच के समय उस नामांकन-पत्र को रद्द करने का अधिकार होगा।

आयोग ने यह भी कहा कि प्रत्येक उम्मीदवार को यह शपथ-पत्र प्रथम श्रेणी मजिस्ट्रेट, नोटरी पब्लिक (Notary Public) अथवा शपथ आयुक्त (Oath Commissioner) के समक्ष विधिवत शपथ लेकर प्रस्तुत करना होगा।

13. निर्वाचन एवं अन्य सम्बन्धित अधिनियम (संशोधन) विधेयक, 2003 (Election and Other Related Laws Amendment Bill, 2003)-राजनीतिक दलों द्वारा चन्दा लिए जाने की प्रक्रिया के साथ-साथ चुनाव प्रक्रिया के विभिन्न पहलुओं को भी अधिक पारदर्शी बनाने के उद्देश्य से निर्वाचन एवं अन्य सम्बन्धित अधिनियम संशोधन विधेयक, 2003 (Election and Other Related laws Amendment Bill, 2003) को संसद के दोनों सदनों ने पारित कर दिया है। वरिष्ठ काँग्रेसी नेता प्रणब मुखर्जी की अध्यक्षता वाली स्थायी समिति की सिफारिशों को विधेयक में समाहित कर लिया गया। विधेयक के प्रमुख प्रावधान इस प्रकार हैं-

(1) राजनीतिक दलों के लिए 20 हजार रुपए या अधिक राशि के चन्दों की रिपोर्टिंग अनिवार्य (अभी यह सीमा 10 हजार रुपए थी),

(2) रिपोर्टिंग न करने वाले दलों को इन राशियों पर आयकर में छूट नहीं होगी,

(3) राजनीतिक दलों को प्रदत्त चन्दे पर सम्बन्धित कंपनी को आयकर अधिनियम के तहत छूट। लाभांश का अधिकतम 5 प्रतिशत ही चन्दे के रूप में देने की अनुमति होगी,

(4) मतदाताओं व मान्यता प्राप्त राजनीतिक दलों के प्रत्याशियों को कुछेक सामग्री चुनाव आयोग द्वारा उपलब्ध कराने का प्रावधान। दूसरे शब्दों में, चुनावों के आंशिक सरकारी वित्तीय सहायता की शुरुआत।

प्रश्न 7.
वयस्क मताधिकार प्रणाली के पक्ष तथा विपक्ष में तर्क दीजिए।
उत्तर:
वयस्क मताधिकार से तात्पर्य मतदान करने की ऐसी व्यवस्था से हैं जिसमें प्रत्येक नागरिक को, बिना किसी प्रकार के भेदभाव के, एक निश्चित आयु पूरी करने पर मतदान का अधिकार प्राप्त हो जाता है। इस प्रणाली में शिक्षा, लिंग, सम्पत्ति, जन्म-स्थान अथवा अन्य आधार पर नागरिकों में भेदभाव नहीं किया जा सकता। केवल पागल, दिवालिए तथा कुछ विशेष प्रकार के अपराधी व विदेशियों को ही मतदान के अधिकार से वंचित किया जाता है। संसार के विभिन्न देशों में वयस्क होने की आयु भिन्न-भिन्न है। भारत, अमेरिका तथा इंग्लैण्ड में यह 18 वर्ष, नार्वे में 23 वर्ष तथा जापान में यह 25 वर्ष है। वयस्क मताधिकार के पक्ष तथा विपक्ष में निम्नलिखित तर्क दिए जाते हैं

पक्ष में तर्क (Arguments in Favour)-वयस्क मताधिकार के पक्ष में प्रायः निम्नलिखित तर्क पेश किए जाते हैं

1. लोक प्रभुसत्ता के सिद्धान्त के अनुकूल (In Harmony with the Principle of Popular Sovereignty):
लोक प्रभुसत्ता का अर्थ है कि सर्वोच्च शक्ति जनता में निहित है। लोकतन्त्र तब तक वास्तविक लोकतन्त्र नहीं हो सकता जब तक कि प्रतिनिधियों के चुनाव में प्रत्येक नागरिक का योगदान न हो। अतः प्रतिनिधियों का चुनाव सामान्य जनता द्वारा किया जाना चाहिए।

2. यह समानता पर आधारित है (It is based on Equality):
लोकतन्त्र का मुख्य आधार है-समानता। सभी व्यक्ति समान हैं और विकास के लिए सभी को मताधिकार देना भी आवश्यक है। जिन नागरिकों को मतदान का अधिकार नहीं होता, उनके हितों तथा अधिकारों की सरकार तनिक भी परवाह नहीं करती। इसलिए प्रत्येक वयस्क को मत देने का अधिकार होना चाहिए।

3. कानूनों का प्रभाव सभी पर पड़ता है (Laws AffectAll):
राज्य के कानूनों तथा नीतियों का प्रभाव सभी व्यक्तियों पर पड़ता है। उसे निश्चित करने में भी सबका भाग होना चाहिए।

4. नागरिकों को राजनीतिक शिक्षा मिलती है (Citizens get Political Education):
वयस्क मताधिकार होने से सभी नागरिक समय-समय पर देश में होने वाले चुनावों में भाग लेते रहते हैं। विभिन्न राजनीतिक दलों के नेता अपने दल की नीति का लोगों में प्रचार करते हैं और देश की समस्याओं के बारे में उनको जानकारी देते रहते हैं। इससे नागरिकों को राजनीतिक शिक्षा मिलती है।

5. सभी के अधिकार सुरक्षित रहते हैं (Rights of all are Safeguarded):
वयस्क मताधिकार प्रणाली में देश के सभी नागरिकों के लिए अपने अधिकारों की रक्षा करना सम्भव होता है। मतदाता अपने प्रतिनिधियों का चुनाव करते हैं तथा उन पर नियन्त्रण बनाए रखते हैं। सरकार नागरिकों के भय से उनके अधिकारों को छीन नहीं सकती।

6. यह स्वाभिमान जागृत करती है (It awakens Self-respect):
वयस्क मताधिकार नागरिकों में स्वाभिमान की भावना को जागृत करता है। जब बड़े-बड़े नेता उनके पास वोट माँगने के लिए आते हैं तो उन्हें अपनी वास्तविक शक्ति का ज्ञान होता है और उनकी सोई हुई आत्मा जागती है। उन्हें इस बात का अनुभव होता है कि देश के शासन में उनका भी योगदान है। इससे उनके मन में स्वाभिमान की भावना जागृत होती है।

7. सार्वजनिक कार्यों में रुचि उत्पन्न करती है (It creates Interest in Public Matters):
वयस्क मताधिकार प्रणाली नागरिकों में सार्वजनिक कार्यों में रुचि उत्पन्न करती है। उन्हें यह अनुभव होता है कि शासन में उनका भी कुछ हाथ है और वे भी देश के कानून तथा नीतियों को प्रभावित कर सकते हैं। वे राष्ट्रहितों के कार्यों का समर्थन करते हैं तथा राष्ट्र-विरोधी कार्यों का विरोध करते हैं। इससे नागरिकों में राष्ट्रीय एकता तथा राष्ट्र-प्रेम की भावना जागृत होती है।

8. अल्पसंख्यकों के हितों की रक्षा (Protection of the Interests of the Minorities):
यह एक ऐसी प्रणाली है जिसमें अल्पसंख्यकों को भी अपने प्रतिनिधि भेजने का अवसर मिल जाता है। उनके द्वारा वे अपने हितों की रक्षा कर सकते हैं।

9. धन का प्रभाव बहुत कम होता है (Less Influence of Money):
इस प्रणाली में सभी वयस्क नागरिकों को मतदान करने का अधिकार होता है, मतों का खरीदना सम्भव नहीं होता। इस प्रकार यह प्रणाली चुनाव में धन के प्रभाव को बहुत कम कर देती है।

10. क्रांति की कम सम्भावना (Less Chance of Revolution):
वयस्क मताधिकार प्रणाली में शांति तथा व्यवस्था की स्थापना की सम्भावना अधिक रहती है क्योंकि नागरिक अपने द्वारा चुनी गई सरकार के विरुद्ध क्रांति नहीं करते।

11. सार्वजनिक हित की दृष्टि से वांछनीय (Necessary for Welfare of All):
लोकतन्त्र का शुद्ध रूप वही होता है जिसमें सार्वजनिक हित की कामना की जाती है। सब वर्गों के हितों और उनके अधिकारों का उचित संरक्षण तभी हो सकता है, जब प्रत्येक वर्ग के लोगों को अपने मत द्वारा शासन की नीतियों और उसके कार्यों को प्रभावित करने का अवसर प्राप्त हो। ऐसा अवसर अधिक-से-अधिक वर्गों को केवल वयस्क मताधिकार द्वारा ही प्राप्त करवाया जा सकता है। अतः वयस्क मताधिकार वर्ग हित की दृष्टि से वांछनीय है।

12. इससे राष्ट्रीय एकता में वृद्धि होती है (It Increases National Unity):”
प्रत्येक व्यक्ति को बिना किसी भेदभाव के मताधिकार देने से राष्ट्रीय एकता में वृद्धि होना स्वाभाविक है। ऐसी स्थिति में कोई भी व्यक्ति अपने-आप को सर्वश्रेष्ठ नहीं समझता, क्योंकि सब व्यक्तियों को जीवन का विकास करने के लिए समान अवसर प्राप्त होते हैं।

यदि किसी एक विशेष वर्ग या जाति को मताधिकार दिया जाए तो राष्ट्रीय एकता के नष्ट होने की सम्भावना हो सकती है। राष्ट्रीय एकता के साथ-साथ यह नागरिकों में देश-प्रेम की भावना को भी जागृत करता है क्योंकि प्रत्येक नागरिक स्वयं को सरकार का अंग समझता है और उसके मन में अपने देश के प्रति सम्मान पैदा होता है।

विपक्ष में तर्क (Arguments Against)-वयस्क मताधिकार के विपक्ष में निम्नलिखित तर्क दिए गए हैं

1. अशिक्षित व्यक्तियों को मताधिकार देना अनुचित है (It is not proper to give Right to Vote to the Ignorants):
प्रत्येक देश में अधिकतर जनता अशिक्षित तथा अज्ञानी होती है। वे उम्मीदवार के गुणों को न देखकर जाति, धर्म तथा मित्रता आदि के आधार पर अपने मत का प्रयोग करते हैं। ऐसे व्यक्ति राजनीतिक नेताओं के जोशीले भाषणों से भी शीघ्र प्रभावित हो जाते हैं। अतः अशिक्षित व्यक्तियों को मताधिकार देना उचित नहीं है।

2. भ्रष्टाचार को बढ़ावा (Encourages Corruption):
वयस्क मताधिकार प्रणाली में भ्रष्टाचार को बढ़ावा मिलता है। कुछ निर्धन व्यक्ति थोड़े-से लालच में पड़कर अपना मत स्वार्थी तथा भ्रष्टाचारी उम्मीदवारों के हाथों में बेच देते हैं।

3. प्रशासन तथा देश की समस्याएँ जटिल (Administration and Problems of the Country are Complicated):
आधुनिक युग में शासन संबंधी प्रश्न तथा समस्याएँ दिन-प्रतिदिन जटिल होती जा रही हैं, जिन्हें समझ पाना साधारण व्यक्ति के बस की बात नहीं है। प्रायः साधारण मतदाता अयोग्य व्यक्ति को चुन लेते हैं क्योंकि उनके पास देश की समस्याओं पर विचार करने तथा उन्हें समझने के लिए समय ही नहीं होता।

4. साधारण जनता रूढ़िवादी होती है (Masses are Conservative):
वयस्क मताधिकार के विरुद्ध एक तर्क यह प्रस्तुत किया जाता है कि साधारण जनता रूढ़िवादी होती है। उनके द्वारा आर्थिक तथा सामाजिक क्षेत्र में प्रगतिशील नीतियों का विरोध किया जाता है। अतः मताधिकार केवल उन्हीं व्यक्तियों को ही मिलना चाहिए जो इसका उचित प्रयोग करने की योग्यता रखते हों।

5. मताधिकार प्राकृतिक अधिकार नहीं है (Right to Vote is not a Natural Right):”
वयस्क मताधिकार के आलोचकों द्वारा यह तर्क पेश किया जाता है कि मताधिकार कोई प्राकृतिक अधिकार नहीं है जो प्रत्येक नागरिक को प्राप्त होना चाहिए। उनके अनुसार मताधिकार केवल उन्हीं व्यक्तियों को ही दिए जाने चाहिएँ जो उनका उचित प्रयोग करने की योग्यता रखते हैं। मताधिकार के बारे में तो यह बात और भी अधिक महत्त्वपूर्ण है क्योंकि यह एक ऐसा अधिकार है जो देश के भविष्य को उज्ज्वल भी बना सकता है और मिट्टी में भी मिला सकता है। अतः मताधिकार केवल उन्हीं व्यक्तियों को दिया जाना चाहिए, जो इसके योग्य हों।

6. सभी नागरिक समान नहीं होते (AIICitizens are not Equal):
वयस्क मताधिकार के विरुद्ध एक तर्क यह दिया जाता है कि सभी नागरिक किसी भी दृष्टि से समान नहीं होते, इसलिए उन सभी को समानता के आधार पर मतदान का अधिकार देना उचित नहीं है।

प्रश्न 8.
भारत के चुनाव आयोग (Election Commission) पर नोट लिखें।
उत्तर:
भारतीय संविधान के अनुच्छेद 324 में चुनाव व्यवस्था के अधीक्षण, निर्देशन एवं नियन्त्रण का कार्य भारत में संवैधानिक मान्यता प्राप्त, स्वतन्त्र एवं निष्पक्ष चुनाव आयोग को सौंपा है जिसमें एक मुख्य चुनाव आयुक्त व अन्य चुनाव आयुक्त हो सकते हैं, जिनकी नियुक्ति राष्ट्रपति करता है। संविधान के अनुच्छेद 324 में यह भी प्रावधान है कि मुख्य चुनाव आयुक्त की सलाह के लिए राष्ट्रपति द्वारा अन्य आयुक्तों की नियुक्ति की जा सकती है।

भारत में 1951 में पहली बार संविधान के अन्तर्गत एक सदस्यीय निर्वाचन आयोग का गठन किया गया, परन्तु केन्द्र की कांग्रेस सरकार ने 16 अक्तूबर, 1989 को राष्ट्रपति वेंकटरमन द्वारा अन्य चुनाव और प्रतिनिधित्व दो निर्वाचन आयुक्तों की नियुक्ति करवाते हुए निर्वाचन आयोग को पहली बार बहु-सदस्यीय आयोग बनाते हुए इसे व्यापक स्वरूप प्रदान किया, लेकिन 2 जनवरी, 1990 को राष्ट्रीय मोर्चे की सरकार ने चुनाव आयोग को फिर से एक-सदस्यीय बना दिया।

2 अक्तूबर, 1993. में भारत के राष्ट्रपति ने अध्यादेश जारी करके चुनाव आयोग में दो अन्य सदस्यों (एम०एस० गिल एवं जी०वी०जी० कृष्णामूर्ति) की नियुक्ति कर दी। 20 दिसम्बर, 1993 में संसद ने एक विधेयक पास करके चुनाव आयोग को बहु-सदस्यीय बना दिया और दो अन्य आयुक्तों को मुख्य निर्वाचन आयुक्त के बराबर दर्जा प्रदान किया गया।

14 जुलाई, 1995 को सर्वोच्च न्यायालय ने अपने निर्णय में अन्य दो आयुक्तों की स्थिति को मुख्य निर्वाचन आयुक्त की स्थिति के बराबर दर्जा देने को वैध ठहराया। अतः इस समय चुनाव आयोग बहु-सदस्यीय है और मुख्य चुनाव आयुक्त एवं अन्य दो चुनाव आयुक्तों की स्थिति बराबर है। इस समय चुनाव आयोग के मुख्य चुनाव आयुक्त श्री सुशील चंद्रा हैं। इसके अतिरिक्त दो अन्य चुनाव आयुक्त हैं।

नियुक्ति (Appointment)-संविधान के अनुच्छेद 324 (2) के अनुसार चुनाव आयोग के मुख्य चुनाव आयुक्त एवं अन्य आयुक्तों की नियुक्ति भारत के राष्ट्रपति एवं संसद द्वारा निर्धारित विधि के अनुसार की जाती है। प्रान्तीय व क्षेत्रीय चुनाव आयुक्तों की नियुक्ति भी राष्ट्रपति द्वारा ही की जाती है।

योग्यताएँ (Qualifications) भारतीय संविधान में मुख्य निर्वाचन आयुक्त एवं अन्य आयुक्तों की नियुक्ति सम्बन्धी योग्यताओं का संविधान में कोई उल्लेख नहीं किया गया है अर्थात् इस सम्बन्ध में हमारा संविधान मौन है।

कार्यकाल (Term)-भारतीय संसद ने सन् 1995 में चुनाव आयोग के कार्यकाल सम्बन्धी एक कानून पास किया है, जिसके अनुसार चुनाव आयुक्त का कार्यकाल 6 वर्ष होगा। इस कानून में यह भी व्यवस्था की गई है कि यदि चुनाव आयुक्त की आयु 6 वर्ष की अवधि से पहले 65 वर्ष की हो जाती है, तो वह अपने पद से अवकाश ग्रहण कर लेता है अर्थात् 6 वर्ष की अवधि या 65 वर्ष की आयु दोनों में से जो पहले पूरी हो, तब तक चुनाव आयुक्त अपने पद पर बने रहते हैं। विशेष परिस्थितियों में मुख्य चुनाव आयुक्तों की अवधि को बढ़ाया जा सकता है, जैसा कि श्री सुकुमार सेन व श्री एस०पी० सेन वर्मा का कार्यकाल 6 वर्ष से अधिक बढ़ाया गया था।

चुनाव आयुक्तों को पदच्युत करना (Removal of Election Commissioners) भारतीय संविधान में चुनाव आयुक्तों को पद से हटाने की व्यवस्था भी की गई है। संविधान के अनुच्छेद 324 (5) के अनुसार चुनाव आयुक्तों को सर्वोच्च न्यायालय के न्यायाधीशों को हटाने की विधि द्वारा ही हटाया जा सकता है अर्थात् चुनाव आयुक्तों को उनके पद से हटाने के लिए सर्वोच्च न्यायालय के न्यायाधीश को हटाए जाने की विधि को ही अपनाया गया है।

इसका अभिप्राय यह है कि मुख्य चुनाव आयुक्तों को तभी हटाया जा सकता है, जब संसद के दोनों सदन अलग-अलग अपने कुल सदस्यों के बहुमत से तथा उपस्थित एवं मतदान करने वाले सदस्यों के 2/3 बहुमत से महाभियोग को पारित कर दें। अन्य चुनाव आयुक्तों या क्षेत्रीय चुनाव आयुक्तों को तो राष्ट्रपति मुख्य चुनाव आयुक्त की सलाह से ही उनके पद से हटाता है अर्थात् मुख्य चुनाव की मंजूरी के बिना चुनाव आयुक्तों या क्षेत्रीय चुनाव आयुक्तों को उनके पद से हटाया नहीं जा सकता।

सन् 2009 में मुख्य चुनाव आयुक्त श्री एन० गोपालास्वामी द्वारा चुनाव आयुक्त नवीन चावला को पक्षपात करने के आधार पर पद से हटाने की सिफारिश की गई थी, परन्तु डॉ. मनमोहन सिंह की सरकार द्वारा इसे स्वीकार नहीं किया गया और उसने राष्ट्रपति को इसके लिए कोई सलाह नहीं भेजी। इसके बावजूद श्री नवीन चावला को नया मुख्य चुनाव आयुक्त नियुक्त कर दिया गया था।

वेतन तथा सेवा शर्ते (Salary and Terms of Service)-चुनाव आयुक्तों के वेतन, भत्ते व सेवा शर्ते समय-समय पर संसद द्वारा निश्चित की जाती हैं। इनके वेतन व भत्ते भारत सरकार की संचित निधि में से दिए जाते हैं। निर्वाचन आयुक्तों के वेतन, भत्तों व सेवा शर्तों में इनके कार्यकाल में कटौती नहीं की जा सकती। 1 अक्तूबर, 1993 में संसद द्वारा पास किए गए एक विधेयक द्वारा चुनाव आयुक्तों को सर्वोच्च न्यायालय के न्यायाधीश के वेतन के समान वेतन दिए जाने की व्यवस्था कर दी गई अर्थात् वर्तमान में मुख्य चुनाव आयुक्त एवं अन्य आयुक्तों को 2,50,000 रुपए मासिक वेतन दिया जाता है।

चुनाव आयोग के लिए कर्मचारी (Staff for Election Commission) संविधान के अनुच्छेद 324 (6) के अनुसार चुनाव आयोग अपने कार्यों को पूरा करने के लिए राष्ट्रपति एवं राज्यों के मुखिया राज्यपालों से आवश्यकतानुसार कर्मचारियों की माँग कर सकता है। ऐसी स्थिति में राष्ट्रपति तथा राज्यपालों का यह कर्त्तव्य है कि वे चुनाव आयुक्तों व क्षेत्रीय आयुक्तों की प्रार्थना पर

उचित स्टाफ का प्रबन्ध करें। चुनाव आयोग को अपने दायित्व का पालन करने के लिए राज्य व जिला स्तर पर बहुत-से कर्मचारियों की आवश्यकता होती है। मतदान केन्द्रों के लिए प्रधान पदाधिकारी (Presiding Officers) व पोलिंग अधिकारी (Polling Officers) चाहिए। इसके अतिरिक्त मतदान केन्द्रों की सुरक्षा एवं व्यवस्थित रूप से संचालन के लिए आवश्यक पुलिस बल चाहिए। अतः केन्द्र में राष्ट्रपति तथा राज्यों में राज्यपाल का कर्त्तव्य है कि चुनाव आयोग की सिफारिश पर आवश्यकतानुसार सुविधाएँ जुटाई जाएँ।

HBSE 11th Class Political Science Important Questions Chapter 3 चुनाव और प्रतिनिधित्व

प्रश्न 9.
प्रादेशिक प्रतिनिधित्व प्रणाली से आप क्या समझते हैं? इसके पक्ष तथा विपक्ष में तर्क दीजिए।
उत्तर:
प्रादेशिक प्रतिनिधित्व का अर्थ (Meaning of Territorial Representation)-लोकतन्त्रीय राज्यों में चुनाव के लिए निर्वाचन-क्षेत्रों का गठन भौगोलिक आधार पर किया जाता है। समस्त राज्य को एक-सदस्य अथवा बहु-सदस्य निर्वाचन-क्षेत्रों में बाँट दिया जाता है। एक चुनाव-क्षेत्र में रहने वाले नागरिकों को उस चुनाव-क्षेत्र का निवासी होने के नाते अपने प्रतिनिधि चुनने के अधिकार को ही प्रादेशिक प्रतिनिधित्व कहा जाता है।

संक्षेप में, सामान्य प्रतिनिधियों का निर्वाचन जब प्रादेशिक आधार पर हो, तो उस प्रणाली को प्रादेशिक प्रतिनिधित्व कहा जाता है। भारत, इंग्लैण्ड तथा अमेरिका आदि राज्यों में इसी प्रणाली को लागू किया गया है। प्रादेशिक प्रतिनिधित्व प्रणाली के गुण अथवा पक्ष में तर्क (Arguments in Favour of Territorial Representation) प्रादेशिक प्रतिनिधित्व प्रणाली के पक्ष में निम्नलिखित तर्क दिए गए हैं

1. सरल चुनाव-प्रणाली (Simple Electoral System):
प्रादेशिक प्रतिनिधित्व प्रणाली का सबसे महत्त्वपूर्ण गुण यह है कि यह प्रणाली अति सरल है। इस प्रणाली में मतदाताओं की सूचियाँ बड़ी सरलता से तैयार की जाती हैं और प्रत्येक निर्वाचन-क्षेत्र की एक मतदाता सूची होती है। एक चुनाव-क्षेत्र में कई उम्मीदवार खड़े होते हैं, मतदाता को किसी एक उम्मीदवार को मत डालना होता है और जिस उम्मीदवार को सबसे अधिक मत प्राप्त होते हैं, उसे निर्वाचित घोषित कर दिया जाता है। साधारण जनता के लिए भी इस प्रणाली को समझना आसान होता है।

2. लोकतन्त्रीय सिद्धान्तों पर आधारित (Based on Democratic Principles):
प्रादेशिक प्रतिनिधित्व प्रणाली लोकतन्त्रीय सिद्धान्तों पर आधारित है। लोकतन्त्र का मुख्य सिद्धान्त यह है कि सभी व्यक्ति समान हैं तथा उनमें जाति, धर्म, रंग तथा लिंग आदि किसी भी आधार पर कोई भेदभाव न किया जाए। इस प्रणाली में एक निर्वाचन-क्षेत्र में रहने वाले सभी मतदाता समानता के आधार पर अपने प्रतिनिधि का चुनाव करते हैं। वह प्रतिनिधि अपने निर्वाचन-क्षेत्र का प्रतिनिधित्व करता है।

3. राष्ट्रीय एकता को बढ़ावा (Promotes National Unity):
प्रादेशिक प्रतिनिधित्व प्रणाली से राष्ट्रीय एकता को बढ़ावा मिलता है। देश के विधानमण्डल में समस्त राष्ट्र के प्रतिनिधि होते हैं न कि विभिन्न वर्गों या व्यवसायों के। ऐसे प्रतिनिधि अपने-आपको किसी वर्ग अथवा व्यवसाय का प्रतिनिधि न मानकर समस्त राष्ट्र का प्रतिनिधि मानते हैं और राष्ट्रीय हितों को प्रधानता देते हैं। इसका प्रभाव जनता पर भी पड़ता है। उनका दृष्टिकोण भी राष्ट्रीय बनता है तथा राष्ट्रीय एकता में वृद्धि होती है।

4. प्रतिनिधियों तथा मतदाताओं में निकटता का संबंध (Close Relation between Representatives and Voters):
प्रादेशिक निर्वाचन-क्षेत्र प्रायः छोटे होते हैं और एक निर्वाचन-क्षेत्र से प्रायः एक ही सदस्य चुना जाता है, जिससे मतदाताओं और प्रतिनिधियों में निकटता का संबंध स्थापित होने की अधिक सम्भावना रहती है। प्रतिनिधि प्रायः उसी निर्वाचन-क्षेत्र का निवासी होता है, जिस क्षेत्र का वह प्रतिनिधित्व कर रहा होता है।

मतदाता और प्रतिनिधि प्रायः एक-दूसरे को अच्छी तरह जानते होते हैं। अतः प्रतिनिधि अपने क्षेत्र के मतदाताओं की इच्छाओं और आवश्यकताओं को अच्छी तरह समझता है और उन्हें पूरा करने के लिए प्रयास करता है।

5. प्रतिनिधियों का उत्तरदायित्व (Fixed Responsibility):
प्रादेशिक प्रतिनिधित्व प्रणाली में प्रतिनिधि अपने क्षेत्र के मतदाताओं के प्रति उत्तरदायी होता है। एक क्षेत्र के सभी मतदाताओं का प्रतिनिधि होने के कारण वह अपने उत्तरदायित्व से इन्कार नहीं कर सकता।

6. क्षेत्रीय हितों की रक्षा (Safeguards Territorial Interests):
प्रादेशिक प्रतिनिधित्व प्रणाली में क्षेत्रीय हितों की अच्छी तरह पूर्ति होती है। प्रतिनिधि अपने क्षेत्र की आवश्यकताओं को अच्छी तरह समझते हैं क्योंकि उनका अपने मतदाताओं व क्षेत्र से समीप का संबंध होता है। अतः प्रतिनिधि अपने निर्वाचन-क्षेत्र की आवश्यकताओं की पूर्ति के लिए भरसक प्रयत्न करते हैं। यदि प्रतिनिधि अपने मतदाताओं के हितों की रक्षा नहीं करता, तो ऐसे प्रतिनिधि को मतदाता दोबारा नहीं चुनते। इसलिए प्रतिनिधि अपने क्षेत्र के हितों की उपेक्षा नहीं कर सकता।

7. अधिक विकास की सम्भावना (Possibility of Greater Development):
प्रादेशिक प्रतिनिधित्व प्रणाली में अधिक विकास की सम्भावना बनी रहती है। इसका कारण यह है कि इस प्रणाली के अन्तर्गत प्रत्येक प्रतिनिधि अपने क्षेत्र का अधिक-से-अधिक विकास करना चाहता है क्योंकि भावी चुनाव में वह अपनी सीट को सुनिश्चित कर लेना चाहता है। सभी क्षेत्रों के विकास से देश का विकास होना स्वाभाविक है।

8. कम खर्चीली (Less Expensive):
इस प्रणाली में चुनाव कम खर्चीला होता है और उम्मीदवार को भी चुनाव में कम खर्च करना पड़ता है। क्षेत्रीय प्रतिनिधित्व प्रणाली के दोष अथवा विपक्ष में तर्क (DemeritsArguments against of Territorial Representation) कई विद्वानों ने क्षेत्रीय प्रतिनिधित्व प्रणाली की कड़ी आलोचना की है। ड्यूगी (Duguit), जी०डी०एच० कोल (GD.H. Cole) और ग्राहम वालास (Graham Wallas) ने इस प्रणाली की कटु आलोचना की है। इस प्रणाली में निम्नलिखित दोष पाए जाते हैं

1. क्षेत्रीयवाद की भावना (Feeling of Regionalism):
इस प्रणाली द्वारा चुने गए प्रतिनिधियों का दृष्टिकोण संकुचित हो जाता है। प्रतिनिधि अपने को एक क्षेत्र विशेष का प्रतिनिधि समझने लगते हैं और उसी क्षेत्र के विकास की बात सोचते तथा करते हैं और उसके लिए प्रयत्नशील रहते हैं। इससे राष्ट्रीय हितों की अवहेलना होने लगती है।

2. सीमित पसन्द (Limited Choice):
कई बार मतदाताओं की पसन्द सीमित हो जाती है क्योंकि चुनाव लड़ने वाले प्रायः उसी क्षेत्र के निवासी होते हैं और यदि उस क्षेत्र में अच्छे उम्मीदवार न हों तो मतदाताओं को इच्छा न होते हुए भी किसी-न-किसी उम्मीदवार के पक्ष में मत डालना ही पड़ता है।

3. भ्रष्ट होना (Corruption):
मतदाताओं को भ्रष्ट किए जाने की सम्भावना रहती है क्योंकि मतदाता कम होते हैं और धनी उम्मीदवार धन के बल पर वोट खरीदने का प्रयत्न करने लगते हैं।

4. अल्पसंख्यकों को उचित प्रतिनिधित्व न मिलना (Minorities fail to get proper Representation):
इस प्रणाली में अल्पसंख्यक वर्गों को प्रतिनिधित्व आसानी से नहीं मिलता। एक क्षेत्र से एक उम्मीदवार चना जाता है और स्वाभाविक है कि बहमत वर्ग का उम्मीदवार ही चुना जाता है। इस प्रकार अल्पसंख्यक वर्ग के प्रतिनिधि लगभग सभी चुनाव क्षेत्रों में हार जाते हैं।

वस्तु निष्ठ प्रश्न

निम्नलिखित प्रश्नों का उत्तर दिए गए विकल्पों में से उचित विकल्प छाँटकर लिखें

1. निम्न में से कौन-सा अप्रत्यक्ष चुनाव-प्रणाली का गुण है?
(A) यह अधिक लोकतांत्रिक है
(B) खर्च अधिक होता है।
(C) केवल योग्य तथा बुद्धिमान व्यक्तियों को चुना जाता है
(D) इनमें से कोई नहीं
उत्तर:
(C) केवल योग्य तथा बुद्धिमान व्यक्तियों को चुना जाता है

2. भारत में अपनाई गई है
(A) वयस्क मताधिकार प्रणाली
(B) धर्म पर आधारित मत प्रणाली
(C) शिक्षा पर आधारित मत प्रणाली
(D) जाति के आधार पर मत प्रणाली
उत्तर:
(A) वयस्क मताधिकार प्रणाली

3. भारत में निम्न का चुनाव प्रत्यक्ष चुनाव-प्रणाली द्वारा किया जाता है
(A) राष्ट्रपति
(B) उप-राष्ट्रपति
(C) राज्यपाल
(D) लोकसभा के सदस्य
उत्तर:
(D) लोकसभा के सदस्य

4. वह चुनाव-प्रणाली जिसमें साधारण मतदाता स्वयं प्रत्यक्ष रूप से अपने प्रतिनिधि का चुनाव करते हैं, कहलाती है
(A) प्रत्यक्ष चुनाव-प्रणाली
(B) अप्रत्यक्ष चुनाव-प्रणाली
(C) सीमित मत प्रणाली
(D) सूची प्रणाली
उत्तर:
(A) प्रत्यक्ष चुनाव-प्रणाली

5. वयस्क मताधिकार प्रणाली में मतदान के अधिकार का आधार होता है
(A) शिक्षा
(B) संपत्ति
(C) आयु
(D) व्यवसाय
उत्तर:
(C) आयु

6. भारत की चुनाव-प्रणाली की विशेषता है
(A) वयस्क मताधिकार
(B) एक-सदस्य निर्वाचन क्षेत्र
(C) सीटों का आरक्षण
(D) उपर्युक्त सभी
उत्तर:
(D) उपर्युक्त सभी

7. निम्न वयस्क मताधिकार का अवगुण है
(A) राजनीतिक शिक्षा मिलती है
(B) भ्रष्टाचार को बढ़ावा मिलता है
(C) इस प्रणाली में महिलाओं को भी मतदान का
(D) इस प्रणाली में गरीबों को भी मतदान का अधिकार अधिकार होता है होता है।
उत्तर:
(B) भ्रष्टाचार को बढ़ावा मिलता है

8. भारत में मतदान के लिए न्यूनतम आयु निश्चित की गई है
(A) 18 वर्ष
(B) 21 वर्ष
(C) 25 वर्ष
(D) 20 वर्ष
उत्तर:
(A) 18 वर्ष

9. निम्न वयस्क मताधिकार का गुण है
(A) यह समानता पर आधारित है ।
(B) नागरिकों को राजनीतिक शिक्षा मिलती है
(C) सभी के अधिकार सुरक्षित रहते हैं
(D) उपर्युक्त सभी
उत्तर:
(D) उपर्युक्त सभी

10. भारत में लोकसभा का चुनाव लड़ने के लिए न्यूनतम आयु होनी चाहिए
(A) 21 वर्ष
(B) 30 वर्ष
(C) 18 वर्ष
(D) 25 वर्ष
उत्तर:
(D) 25 वर्ष

11. निम्न में से कौन-सा प्रत्यक्ष चुनाव-प्रणाली का गुण है?
(A) अधिक लोकतांत्रिक है
(B) योग्य तथा बुद्धिमान व्यक्तियों का चुनाव
(C) खर्च कम होता है
(D) राजनीतिक दलों का प्रभाव कम होता है।
उत्तर:
(A) अधिक लोकतांत्रिक है

12. भारत में निम्न का चुनाव अप्रत्यक्ष चुनाव-प्रणाली से किया जाता है
(A) लोकसभा के सदस्य
(B) राज्य विधानसभा के सदस्य
(C) राष्ट्रपति
(D) इनमें से कोई नहीं
उत्तर:
(C) राष्ट्रपति

13. भारत में पहला आम चनाव निम्न वर्ष में हआ था
(A) सन् 1947
(B) सन् 1950
(C) सन् 1952
(D) सन् 1955
उत्तर:
(C) सन् 1952

14. निम्न प्रादेशिक प्रतिनिधित्व का गुण है
(A) सरल चुनावे-प्रणाली
(B) मतदाताओं की सीमित पसंद
(C) अल्पसंख्यकों को प्रतिनिधित्व
(D) इनमें से कोई नहीं
उत्तर:
(A) सरल चुनाव-प्रणाली

15. देश में चुनाव करवाने संबंधी निर्णय लेने का अधिकार है
(A) राष्ट्रपति के पास
(B) प्रधानमंत्री के पास
(C) चुनाव आयोग के पास
(D) उप-राष्ट्रपति के पास
उत्तर:
(C) चुनाव आयोग के पास

16. चुनाव आयोग में आजकल कितने सदस्य (मुख्य चुनाव आयुक्त सहित) हैं
(A) 4
(B) 3
(C) 2
(D) 5
उत्तर:
(B) 3

17. निम्न चुनाव आयोग का कार्य नहीं है
(A) चुनाव कराना
(B) राजनीतिक दलों को मान्यता प्रदान करना
(C) मतदाता सूचियाँ तैयार करना
(D) चुनाव के लिए उम्मीदवार खड़े करना
उत्तर:
(D) चुनाव के लिए उम्मीदवार खड़े करना

HBSE 11th Class Political Science Important Questions Chapter 3 चुनाव और प्रतिनिधित्व

18. भारत में सांप्रदायिक चुनाव-प्रणाली निम्न वर्ष में लागू हुई
(A) सन् 1906
(B) सन् 1909
(C) सन् 1935
(D) सन् 1952
उत्तर:
(B) सन् 1909

19. मुख्य निर्वाचन आयुक्त को मासिक वेतन मिलता है
(A) 1,50,000
(B) 2,00,000
(C) 1,00,000
(D) 2,50,000
उत्तर:
(D) 2,50,000

20. आनुपातिक प्रतिनिधित्व की एक शर्त है
(A) एक-सदस्य निर्वाचन-क्षेत्र
(B) बहु-सदस्य निर्वाचन-क्षेत्र
(C) शहरी निर्वाचन-क्षेत्र
(D) ग्रामीण निर्वाचन-क्षेत्र
उत्तर:
(B) बहु-सदस्य निर्वाचन-क्षेत्र

21. भारत में अब तक लोकसभा के कितने चुनाव हो चुके हैं?
(A) 10
(B) 12
(C) 15
(D) 17
उत्तर:
(D) 17

22. भारत में मुख्य चुनाव आयुक्त हैं
(A) श्री नवीन चावला
(B) हरिशंकर ब्रह्मा
(C) श्री सुशील चंद्रा
(D) श्री ओमप्रकाश रावत
उत्तर:
(C) श्री सुशील चंद्रा

23. निम्न भारतीय चुनाव-प्रणाली का दोष है
(A) चुनावों में धन की बढ़ती हुई भूमिका
(B) सरकारी तंत्र का दुरुपयोग
(C) जाति तथा धर्म के नाम पर मतदान
(D) उपर्युक्त सभी
उत्तर:
(D) उपर्युक्त सभी

24. निम्न निर्वाचन आयोग का कार्य है
(A) चुनावों का प्रबंध करना
(B) मतदाता सूचियां तैयार करना
(C) चुनाव तिथि की घोषणा करना
(D) उपर्युक्त सभी
उत्तर:
(D) उपर्युक्त सभी

निम्नलिखित प्रश्नों का उत्तर एक शब्द में दें

1. 17वीं लोकसभा के चुनाव कब हुए?
उत्तर:
अप्रैल-मई, 2019 में।

2. 17वीं लोकसभा चुनाव में कितनी महिला सांसद निर्वाचित हुईं?
उत्तर:
78 महिलाएँ।

3. 17वीं लोकसभा चुनाव में मतदाताओं की कुल संख्या कितनी थी?
उत्तर:
लगभग 89.78 करोड़।

4. 17वीं लोकसभा चुनाव में कुल मतदान प्रतिशत कितना रहा?
उत्तर:
67.11 प्रतिशत।

5. 17वीं लोकसभा चुनाव के बाद महिला सांसदों का प्रतिशत कितना हो गया?
उत्तर:
17वीं लोकसभा चुनाव के बाद महिला सांसदों का 14.4 प्रतिशत हो गया है।

6. 17वीं लोकसभा में सर्वाधिक महिला सांसद किस राजनीतिक दल से हैं एवं कितनी हैं?
उत्तर:
सर्वाधिक महिला सांसद भाजपा से हैं जिनकी संख्या 42 है।

7. वर्तमान में भारत के मुख्य निर्वाचन आयुक्त कौन हैं?
उत्तर:
श्री सुशील चंद्रा।

8. मुख्य निर्वाचन आयुक्त एवं अन्य आयुक्तों को कितना मासिक वेतन मिलता है?
उत्तर:
2,50,000 रुपए मासिक।

9. भारत में मतदान के अधिकार के लिए न्यूनतम आयु कितनी निश्चित की गई है?
उत्तर:
18 वर्ष।

रिक्त स्थान भरें

1. भारत में अब तक ……………. लोकसभा के चुनाव सम्पन्न हो चुके है।
उत्तर:
17

2. ……… को भारत में 17वीं लोकसभा के चुनाव सम्पन्न हुए।
उत्तर:
अप्रैल-मई, 2019

3. ………… भारत के मुख्य निर्वाचन आयुक्त हैं।
उत्तर:
श्री सुशील चंद्रा

4. भारत में 61वें संवैधानिक संशोधन के अनुसार मताधिकार की आयु वर्ष निचित की गई।
उत्तर:
18

HBSE 11th Class Political Science Important Questions Chapter 3 चुनाव और प्रतिनिधित्व

5. भारत में प्रथम आम चुनाव सन् ……………. में हुआ।
उत्तर:
1952

6. भारतीय चुनाव आयोग में कुल …………… सदस्य हैं।
उत्तर:
3

7. 17वीं लोकसभा चुनाव में …………… प्रतिशत मतदान हुआ।
उत्तर:
67.11

8. 16वीं लोकसभा चुनाव में मतदाताओं के नए विकल्प के रूप ……………. का प्रयोग प्रारम्भ हुआ था।
उत्तर:
नोटा (Nota)

HBSE 11th Class Political Science Important Questions Chapter 2 भारतीय संविधान में अधिकार

Haryana State Board HBSE 11th Class Political Science Important Questions Chapter 2 भारतीय संविधान में अधिकार Important Questions and Answers.

Haryana Board 11th Class Political Science Important Questions Chapter 2 भारतीय संविधान में अधिकार

अति लघूत्तरात्मक प्रश्न

प्रश्न 1.
सर्वप्रथम मानवाधिकारों की घोषणा कब और कहाँ हुई थी?
उत्तर:
सर्वप्रथम मानवाधिकारों की घोषणा 1789 ई० में फ्रांस की राष्ट्रीय सभा में हुई थी।

प्रश्न 2.
भारत में सर्वप्रथम कब और किसके द्वारा मौलिक अधिकारों की माँग की गई?
उत्तर:
भारत में सर्वप्रथम 1895 ई० में बाल गंगाधर तिलक के द्वारा मौलिक अधिकारों की माँग की गई।

प्रश्न 3.
भारतीय संविधान द्वारा दिए अधिकारों को मौलिक कहने के कोई दो कारण लिखिए।
उत्तर:
(1) अधिकारों को मौलिक इसलिए कहा जाता है क्योंकि इन्हें देश की मौलिक विधि अर्थात् संविधान में स्थान दिया गया है और इनमें विशेष संशोधन प्रक्रिया के अतिरिक्त किसी अन्य प्रकार से परिवर्तन नहीं किया जा सकता,

(2) ये अधिकार – व्यक्ति के प्रत्येक पक्ष के विकास हेतु मूल रूप में आवश्यक हैं, जिनके अभाव में उनके व्यक्तित्व का विकास अवरुद्ध हो जाएगा।

प्रश्न 4.
दक्षिण अफ्रीका के संविधान में वर्णित किन्हीं दो मूल अधिकारों का उल्लेख कीजिए।
उत्तर:

  • गरिमा का अधिकार,
  • स्वास्थ्य की रक्षा, रोटी, पानी तथा सामाजिक सुरक्षा का अधिकार।

प्रश्न 5.
मूल अधिकारों को संविधान में रखने के किन्हीं दो उद्देश्यों का उल्लेख कीजिए।
उत्तर:

  • सत्तारूढ़ दल के अत्याचार से सुरक्षा करने हेतु तथा,
  • अल्पसंख्यकों के हितों की रक्षा करने के लिए।

प्रश्न 6.
भारतीय संविधान में वर्णित मौलिक अधिकारों की कोई दो विशेषताएँ लिखिए।
उत्तर:

  • मौलिक अधिकार पूर्ण तथा निरंकुश नहीं हैं,
  • संसद मूल अधिकारों को निलम्बित कर सकती है।

प्रश्न 7.
‘कानून के समक्ष समानता’ से क्या तात्पर्य है?
उत्तर:
कानून के समक्ष समानता का तात्पर्य है कि किसी व्यक्ति को कोई विशेषाधिकार प्राप्त नहीं होता और समस्त व्यक्ति कानून के सामने बराबर होते हैं।

HBSE 11th Class Political Science Important Questions Chapter 2 भारतीय संविधान में अधिकार

प्रश्न 8.
‘कानून के समक्ष संरक्षण’ से क्या तात्पर्य है?
उत्तर:
कानून के समक्ष संरक्षण से तात्पर्य है कि समान परिस्थितियों में सभी व्यक्तियों के साथ कानून का एक-समान व्यवहार होना।

प्रश्न 9.
भारतीय संविधान में वर्णित ‘समानता के अधिकार’ के कोई दो अपवाद लिखिए।
उत्तर:

  • विदेशी राज्यों के मुखियाओं और राजनायिकों के विरुद्ध भारतीय कानून के अंतर्गत कार्रवाई नहीं की जा सकती,
  • राष्ट्रपति तथा राज्यपालों के विरुद्ध उनके कार्यकाल के दौरान कोई फौजदारी मुकद्दमा नहीं चलाया जा सकता।

प्रश्न 10.
भारतीय संविधान में उल्लेखित स्वतंत्रता के अधिकार के किन्हीं दो अपवादों का उल्लेख कीजिए।
उत्तर:

  • ये अधिकार शत्रु-देश के नागरिकों को प्राप्त नहीं होंगे,
  • निवारक नजरबन्दी के अधीन की गई गिरफ्तारी के सन्दर्भ में भी स्वतंत्रता संबंधी अधिकार की व्यवस्थाएँ लागू नहीं होंगी।

प्रश्न 11.
‘बन्दी प्रत्यक्षीकरण’ (Habeas Corpus) का क्या अर्थ है?
उत्तर:
हैबियस कॉरपस’ लेटिन भाषा का शब्द है जिसका अर्थ है ‘शरीर हमारे सामने पेश करो।’ इस लेख द्वारा न्यायालय बन्दी बनाने वाले अधिकारी को यह आदेश देता है कि बन्दी बनाए गए व्यक्ति को एक निश्चित तिथि और स्थान पर न्यायालय में उपस्थित किया जाए, जिससे न्यायालय यह निर्णय कर सके कि किसी व्यक्ति को बन्दी बनाए जाने के कारण वैध हैं या अवैध । यदि बन्दी बनाने के कारण अवैध हैं तो उसे मुक्त करने संबंधी लेख जारी किया जाता है।

प्रश्न 12.
‘परमादेश लेख (Writ of Mandamus) से क्या तात्पर्य है?
उत्तर:
‘मेण्डेमस’ शब्द लेटिन भाषा का है जिसका अर्थ है ‘हम आज्ञा देते हैं। यह लेख न्यायालय उस समय जारी करता है, जब कोई व्यक्ति या संस्था अपने सार्वजनिक दायित्व का निर्वाह न कर रही हो और जिसके फलस्वरूप व्यक्तियों के मूल अधिकारों का उल्लंघन हो रहा हो।

प्रश्न 13.
भारतीय संविधान में वर्णित मौलिक अधिकारों की आलोचना के कोई दो आधार लिखिए।
उत्तर:

  • संविधान में वर्णित मौलिक अधिकारों पर बहुत अधिक प्रतिबंध हैं, जिसके फलस्वरूप साधारण व्यक्ति इन्हें समझने में प्रायः असमर्थ-सा रहता है,
  • मौलिक अधिकारों की भाषा कठिन एवं अस्पष्ट है जिसे साधारण एवं अशिक्षित व्यक्ति समझ नहीं पाता।

प्रश्न 14.
मौलिक अधिकारों की कोई दो उपयोगिताएँ लिखिए।
उत्तर:

  • मौलिक अधिकार कानून का शासन स्थापित करते हैं,
  • मौलिक अधिकारों द्वारा समाज में सामाजिक समानता की स्थापना होती है।

प्रश्न 15.
भारतीय संविधान में मौलिक कर्तव्यों को कब और कौन-से भाग में जोड़ा गया?
उत्तर:
भारतीय संविधान में सन् 1976 में 42वें संवैधानिक संशोधन द्वारा संविधान के भाग चार में अनुच्छेद 51-A में जोड़ा गया।

प्रश्न 16.
भारतीय संविधान में दिए गए किन्हीं दो मौलिक कर्तव्यों का उल्लेख कीजिए।
उत्तर:

  • संविधान का पालन करना तथा इसके आदर्शों, इसकी संस्थाओं, राष्ट्रीय झण्डे तथा गान का सम्मान करना,
  • भारत की प्रभुसत्ता, एकता तथा अखण्डता को बनाए रखना और सुरक्षित रखना।।

प्रश्न 17.
भारतीय संविधान में सम्मिलित मौलिक कर्तव्यों के कोई दो महत्त्व लिखिए।
उत्तर:

  • मौलिक कर्तव्य व्यक्ति के आदर्श एवं पथ-प्रदर्शक हैं,
  • मौलिक कर्त्तव्य मूल अधिकारों की प्राप्ति में सहायक हैं।

प्रश्न 18.
राष्ट्रीय मानवाधिकार आयोग के कोई दो कार्य लिखिए।
उत्तर:

  • यह मानवाधिकारों के उल्लंघन की शिकायतों की जाँच करता है,
  • यह मानवाधिकारों के क्षेत्र में अनुसन्धान को प्रोत्साहित करता है।

प्रश्न 19.
राज्य-नीति के निदेशक सिद्धान्तों से क्या तात्पर्य है?
उत्तर:
राज्य-नीति के निदेशक सिद्धान्तों का तात्पर्य यह है कि ये सिद्धान्त संविधान के द्वारा आगामी सरकारों के लिए कुछ नैतिक निर्देश हैं। ये सिद्धान्त इस प्रकार के आदेश हैं जिन पर आगामी सरकारों को जनता के हित में कार्य करने के लिए प्रेरणा दी गई है।

प्रश्न 20.
राज्य-नीति के निदेशक सिद्धान्तों के कोई दो लक्षण लिखिए।
उत्तर:

  • ये राज्य की शासन-व्यवस्था के आधारभूत सिद्धान्त हैं,
  • ये सिद्धान्त न्याय योग्य नहीं हैं।

प्रश्न 21.
राज्य-नीति के कोई दो समाजवादी निदेशक सिद्धान्तों का उल्लेख कीजिए।
उत्तर:

  • राज्य ऐसा प्रयास करें कि सम्पत्ति और उत्पादन के साधनों का इस प्रकार केंद्रीयकरण न हो कि सार्वजनिक हित को किसी प्रकार की बाधा पहुँचे,
  • स्त्री और पुरुष दोनों को समान कार्य के लिए समान वेतन मिले।

प्रश्न 22.
राज्य-नीति के कोई दो गाँधीवादी निदेशक सिद्धान्तों का उल्लेख कीजिए।
उत्तर:
(1) अनुच्छेद 40 के अनुसार राज्य ग्राम पंचायतों के गठन के लिए प्रयत्न करेगा और उन्हें ऐसी शक्तियाँ तथा प्राधिकार प्रदान करेगा जिनसे कि वे स्वायत्त शासन की इकाइयों के रूप में कार्य कर सकें,

(2) अनुच्छेद 46 के अनुसार राज्य समाज के दुर्बल वर्गों विशेषकर अनुसूचित जातियों तथा कबीलों की शिक्षा और आर्थिक उन्नति के लिए प्रयत्न करेगा तथा उन्हें सामाजिक अन्याय एवं शोषण से बचाएगा।

प्रश्न 23.
राज्य-नीति के कोई दो उदारवादी निदेशक सिद्धान्त लिखिए।
उत्तर:

  • राज्य समस्त भारत में समान आचार-संहिता लागू करने का प्रयत्न करेगा,
  • राज्य ऐतिहासिक एवं कलात्मक महत्त्व रखने वाले स्मारकों, स्थानों तथा वस्तुओं की रक्षा करेगा और उनको नष्ट होने से बचाएगा।

प्रश्न 24.
राज्य-नीति के किन्हीं दो अंतर्राष्ट्रीय निदेशक सिद्धान्तों का उल्लेख कीजिए।
उत्तर:

  • राज्य द्वारा अंतर्राष्ट्रीय शान्ति व सुरक्षा को बढ़ावा देने का प्रयास करना,
  • राष्ट्रों के मध्य उचित व सम्मानपूर्वक संबंध बनाए रखने का प्रयास करना।

प्रश्न 25.
राज्य-नीति के निदेशक सिद्धान्तों की आलोचना के कोई दो आधार बताइए।
उत्तर:

  • निदेशक सिद्धान्तों के पीछे कानूनी शक्ति का अभाव है,
  • 20वीं शताब्दी में लागू होने वाले निदेशक सिद्धान्त 21वीं शताब्दी में भी उपयोगी होंगे, यह आवश्यक नहीं है।

प्रश्न 26.
राज्य-नीति के निदेशक सिद्धान्तों के कोई दो महत्त्व लिखिए।
उत्तर:

  • ये सिद्धान्त कल्याणकारी राज्य की स्थापना में सहायक सिद्ध होंगे,
  • इन सिद्धान्तों से सरकार की नीतियों में निरन्तरता तथा स्थिरता सम्भव होगी।

लघूत्तरात्मक प्रश्न

प्रश्न 1.
मौलिक अधिकार का क्या अर्थ है? भारतीय संविधान द्वारा नागरिकों को कौन-से मौलिक अधिकार दिए गए हैं?
उत्तर:
मौलिक अधिकार उन सुविधाओं, स्वतन्त्रताओं तथा अधिकारों को कहते हैं जो एक व्यक्ति के पूर्ण विकास के लिए अति आवश्यक हैं। ये वे मूल अधिकार हैं जिनका प्रयोग किए बिना व्यक्ति अपना शारीरिक, मानसिक तथा बौद्धिक विकास नहीं कर सकता। प्रत्येक लोकतन्त्रीय राज्य अपने सभी नागरिकों को बिना किसी भेद-भाव के कुछ मौलिक अधिकार देता है। भारतीय संबिधान द्वारा भी नागरिकों को कुछ मौलिक अधिकार दिए गए हैं जो इस प्रकार हैं-

  • समानता का अधिकार,
  • स्वतन्त्रता का अधिकार,
  • शोषण के विरुद्ध अधिकार,
  • धार्मिक स्वतन्त्रता का अधिकार,
  • सांस्कृतिक तथा शिक्षा सम्बन्धी अधिकार,
  • संवैधानिक उपचारों का अधिकार

प्रश्न 2.
भारतीय संविधान में दिए गए मूल अधिकारों का महत्त्व बताइए।
उत्तर:
भारतीय संविधान में दिए गए मौलिक अधिकारों का बहुत महत्त्व है। ये देश के सभी नागरिकों को समान रूप से विकास के अवसर प्रदान करते हैं। ये सामाजिक, आर्थिक, राजनीतिक तथा कानून के सामने समानता पर बल देते हैं। ये नागरिकों को अनेक प्रकार की स्वतन्त्रताएँ प्रदान करते हैं जिनके प्रयोग से वे अपने जीवन की उन्नति तथा विकास कर सकते हैं।

इन अधिकारों से भारत के एक धर्मनिरपेक्ष राज्य होने का भी संकेत मिलता है। मौलिक अधिकार सरकार की निरंकुशता पर रोक लगाते हैं और उसे मनमानी करने से रोकते हैं। ये अधिकार सामाजिक-आर्थिक कार्यक्रमों को बढ़ावा देते हुए भारत में सामाजिक-आर्थिक लोकतन्त्र के विकास में सहायता करते हैं। मौलिक अधिकार अल्पसंख्यकों को संरक्षण प्रदान करते हैं।

HBSE 11th Class Political Science Important Questions Chapter 2 भारतीय संविधान में अधिकार

प्रश्न 3.
भारतीय संविधान में दिए गए मौलिक अधिकारों की कोई पाँच विशेषताएँ बताएँ।।
उत्तर:
भारतीय संविधान में दिए गए मौलिक अधिकारों की पाँच विशेषताएँ निम्नलिखित हैं-

  • भारतीय संविधान में दिए गए मौलिक अधिकार बहुत ही व्यापक तथा विस्तृत हैं,
  • मौलिक अधिकार न्याय-संगत हैं। इसका अर्थ यह है कि इनका उल्लंघन होने पर नागरिकों को न्यायालय में जाकर न्याय माँगने का अधिकार है,
  • मौलिक अधिकार सीमित हैं। इनके प्रयोग पर कुछ सीमाएँ लगी हुई हैं,
  • मौलिक अधिकार देश के सभी नागरिकों को समान रूप से प्राप्त हैं,
  • संकटकाल में मौलिक अधिकारों को निलम्बित (Suspend) किया जा सकता है।

प्रश्न 4.
भारतीय संविधान में दिए गए समानता के अधिकार की व्याख्या करो।
अथवा
भारतीय संविधान के अनच्छेद 14-18 में दिए गए समानता के अधिकार का वर्णन करो।
उत्तर:
‘समानता का अधिकार’ का वर्णन संविधान की धारा 14-18 में किया गया है। अनुच्छेद 14 के अनुसार भारत के सभी नागरिकों को कानून के सामने समानता प्रदान की गई है।
अनुच्छेद 15 के अनुसार नागरिकों में जाति, धर्म, रंग, लिंग तथा जन्म-स्थान आदि के आधार पर सभी प्रकार के भेद-भावों को समाप्त कर दिया गया है।
अनुच्छेद 16 के अनुसार सभी नागरिकों को सरकारी नौकरी पाने के क्षेत्र में समानता प्रदान की गई है।
अनुच्छेद 17 के अनुसार देश में छुआछूत को समाप्त कर दिया गया है और इसके प्रयोग को कानून द्वारा अवैध घोषित कर दिया गया है।
अनुच्छेद 18 द्वारा सैनिक तथा शिक्षा-सम्बन्धी उपाधियों को छोड़कर अन्य सभी प्रकार की उपाधियों को समाप्त कर दिया गया है।

प्रश्न 5.
संविधान के अनुच्छेद 19 के अन्तर्गत नागरिकों को कौन-कौन-सी स्वतन्त्रताएँ प्रदान की गई हैं? अथवा भारतीय संविधान में दी गई विभिन्न मूल स्वतन्त्रताओं की व्याख्या कीजिए।
उत्तर:
संविधान के अनुच्छेद 19 के अन्तर्गत नागरिकों को दी गई स्वतन्त्रताएँ इस प्रकार हैं-

  • भाषण देने तथा विचार प्रकट करने की स्वतन्त्रता,
  • शान्तिपूर्ण तथा बिना हथियारों के सभा करने की स्वतन्त्रता,
  • समुदाय अथवा संघ बनाने की स्वतन्त्रता,
  • देश के किसी भी भाग में घूमने-फिरने की स्वतन्त्रता,
  • देश में किसी भी स्थान पर बसने की स्वतन्त्रता,
  • कोई भी व्यवसाय अपनाने की स्वतन्त्रता।

प्रश्न 6.
मौलिक स्वतन्त्रताओं पर किन आधारों पर तार्किक प्रतिबन्ध (Reasonable Restrictions) लगाए जा सकते हैं?
उत्तर:
अनुच्छेद 19 के अन्तर्गत दी गई स्वतन्त्रताओं पर निम्नलिखित तार्किक प्रतिबन्ध लगाए जा सकते हैं-

(1) संसद भारत की प्रभुसत्ता, अखण्डता तथा सुरक्षा को बनाए रखने के लिए इन स्वतन्त्रताओं पर प्रतिबन्ध लगा सकती है,
(2) संसद विदेशी राज्यों से मैत्रीपूर्ण सम्बन्धों को बनाए रखने के लिए इन स्वतन्त्रताओं पर प्रतिबन्ध लगा सकती है,
(3) संसद सार्वजनिक व्यवस्था, शिष्टता अथवा नैतिकता, न्यायालय का अपमान, मान-हानि व हिंसा के लिए उत्तेजित करना आदि के आधारों पर इन स्वतन्त्रताओं पर प्रतिबन्ध लगा सकती है,
(4) राज्य जनता के हितों और अनुसूचित कबीलों के हितों की सुरक्षा के लिए इन स्वतन्त्रताओं पर उचित प्रतिबन्ध लगा सकते हैं।

प्रश्न 7.
शोषण के विरुद्ध अधिकार (Right against Exploitation) से आप क्या समझते हैं?
उत्तर:
संविधान के अनुच्छेद 23 और 24 के अनुसार नागरिकों को शोषण के विरुद्ध अधिकार दिया गया है। इस अधिकार के अनुसार व्यक्तियों को बेचा या खरीदा नहीं जा सकता है। किसी भी व्यक्ति से बेगार नहीं ली जा सकती। किसी भी व्यक्ति की आर्थिक दशा से अनुचित लाभ नहीं उठाया जा सकता और उसे कोई भी काम उसकी इच्छा के विरुद्ध करने के लिए बाध्य नहीं किया जा सकता। 14 वर्ष से कम आयु के बच्चों को किसी ऐसे कारखाने या खान में नौकर नहीं रखा जा सकता, जहाँ उसके स्वास्थ्य पर बुरा प्रभाव पड़ने की सम्भावना हो।

प्रश्न 8.
मौलिक अधिकारों की सुरक्षा के लिए भारतीय संविधान में की गई व्यवस्थाओं का वर्णन कीजिए।
उत्तर:
(1) संविधान के अनुच्छेद 13 में यह व्यवस्था की गई है कि राज्य कोई ऐसा कानून नहीं बना सकता जो नागरिकों के मौलिक अधिकारों को सीमित अथवा समाप्त करता हो।

(2) मौलिक अधिकार न्यायसंगत हैं और हम उनकी रक्षा के लिए न्यायालयों में जा सकते हैं। न्यायालय अधिकारों की रक्षा के लिए आदेश जारी कर सकते हैं।

(3) सर्वोच्च न्यायालय तथा उच्च न्यायालयों के पास पुनर्निरीक्षण की शक्ति है। इस शक्ति के द्वारा सर्वोच्च न्यायालय व उच्च न्यायालय संसद तथा राज्य विधानमण्डल के कानूनों को और राष्ट्रपति तथा राज्यपालों के आदेश को रद्द कर सकते हैं, यदि वे कानून या आदेश मौलिक अधिकारों का उल्लंघन करते हों।

प्रश्न 9.
संवैधानिक उपचारों के अधिकार से आप क्या समझते हैं?
उत्तर:
सवैधानिक उपचारों का अधिकार नागरिकों के मौलिक अधिकारों की प्राप्ति की रक्षा का अधिकार है। अनुच्छेद 32 के अनुसार प्रत्येक नागरिक अपने मौलिक अधिकारों की प्राप्ति और रक्षा के लिए उच्च न्यायालय या सर्वोच्च न्यायालय के पास जा सकता है।

यदि सरकार हमारे किसी मौलिक अधिकार को लागू न करे या उसके विरुद्ध कोई काम करे तो उसके विरुद्ध न्यायालय में प्रार्थना-पत्र दिया जा सकता है और न्यायालय द्वारा उस अधिकार को लागू करवाया जा सकता है या उस कानन को रद्द कराया जा सकता है। उच्च न्यायालयों तथा सर्वोच्च न्यायालय को इस सम्बन्ध में कई प्रकार के लेख (Writs) जारी करने का अधिकार है।

प्रश्न 10.
सवैधानिक उपचारों के अधिकार के अन्तर्गत न्यायपालिका किस प्रकार के आदेशों को जारी कर सकती है?
उत्तर:
मौलिक अधिकारों को लागू कराने के लिए न्यायपालिका निम्नलिखित आदेश जारी कर सकती है-

  • बन्दी प्रत्यक्षीकरण लेख,
  • परमादेश का आज्ञा-पत्र,
  • मनाही आज्ञा-पत्र,
  • उत्प्रेषण लेख तथा
  • अधिकार-पृच्छा लेख।

प्रश्न 11.
‘बन्दी प्रत्यक्षीकरण लेख’ (Writ of Habeas Corpus) तथा ‘परमादेश लेख’ (Writ of Mandamus) पर नोट लिखिए।
उत्तर:
1. बन्दी प्रत्यक्षीकरण लेख-‘बन्दी प्रत्यक्षीकरण’ शब्द लेटिन भाषा के शब्द ‘हेबियस कॉर्पस’ से लिया गया है, जिसका अर्थ है-‘हमारे सम्मुख शरीर को प्रस्तुत करो’ (Let us have the body.) इस आदेश के अनुसार न्यायालय किसी अधिकारी को जिसने किसी व्यक्ति को गैर-कानूनी ढंग से बन्दी बना रखा हो, आज्ञा दे सकता है कि कैदी को समीप के न्यायालय में उपस्थित किया जाए, ताकि उसकी गिरफ्तारी के कानून के औचित्य या अनौचित्य का निर्णय किया जा सके। अनियमित गिरफ्तारी की दशा में न्यायालय उसको स्वतन्त्र करने का आदेश दे सकता है।

2. ‘परमादेश’ लेख-‘परमादेश लेख’ लेटिन भाषा के शब्द ‘मैण्डमस’ से लिया गया है। जिसका अर्थ है-‘हम आदेश देते हैं। (We Command.) इस आदेश द्वारा न्यायालय किसी अधिकारी, संस्था अथवा निम्न न्यायालय को अपने कर्तव्यों का पालन करने के लिए बाधित कर सकता है। इस आदेश द्वारा न्यायालय राज्य के कर्मचारियों से ऐसे कार्य करवा सकता है जिनको वे किसी कारण से न कर रहे हों तथा जिनके न किए जाने से किसी नागरिक के मौलिक अधिकारों का उल्लंघन हो रहा हो।

प्रश्न 12.
अधिकार-पृच्छा लेख (Writ of Quo-Warranto) से आप क्या समझते हैं?
उत्तर:
अधिकार-पृच्छा लेख का अर्थ है ‘किसके आदेश से’ अथवा ‘किस अधिकार से’। यह आदेश उस समय जारी किया जाता है, जब कोई व्यक्ति ऐसे कार्य को करने का दावा करता हो, जिसको करने का उसको अधिकार न हो। इस आदेश के अनुसार उच्च न्यायालय या सर्वोच्च न्यायालय किसी व्यक्ति को कोई पद ग्रहण करने से रोकने के लिए निषेधाज्ञा जारी कर सकता है और उक्त पद के रिक्त होने की तब तक के लिए घोषणा कर सकता है, जब तक कि न्यायालय द्वारा कोई निर्णय न दिया जाए।

प्रश्न 13.
संकटकाल में मौलिक अधिकारों के स्थगन पर एक नोट लिखें।
उत्तर:
संविधान राष्ट्रपति को अधिकार देता है कि जब अनुच्छेद 352 तथा 356 के अन्तर्गत संकटकालीन व्यवस्था की जाए तो संविधान के अनुच्छेद 19 में दिए गए मौलिक अधिकारों को स्थगित किया जा सकता है। मौलिक अधिकार तभी स्थगित किए जा सकते हैं, जब संकटकाल की घोषणा युद्ध या बाहरी आक्रमण के कारण हो, न कि शस्त्र-विद्रोह के आधार पर।

राष्ट्रपति संकटकाल में मौलिक अधिकारों को लागू करवाने के लिए न्यायालय का सहारा लेने के अधिकार को समस्त भारत या उसके किसी भाग में स्थगित कर सकता है, परन्तु अनुच्छेद 21 के अधीन जीवन या व्यक्तिगत स्वतन्त्रता के अधिकार को लागू करवा अधिकार को स्थगित नहीं किया जा सकता।

प्रश्न 14.
‘निवारक नजरबन्दी’ (Preventive Detention) पर संक्षिप्त टिप्पणी लिखिए।
उत्तर:
किसी भी व्यक्ति को राज्य की सुरक्षा को भंग किए जाने के सन्देह पर बन्दी बनाया जा सकता है। ऐसे व्यक्ति को किसी भी प्रकार की ‘निजी स्वतन्त्रता’ प्राप्त नहीं होती, परन्तु यदि उसे दो महीने से अधिक नजरबन्द रखना हो, तो ऐसा सलाहकार बोर्ड के परामर्श पर ही किया जा सकता है। निवारक नजरबन्दी में बन्दी बनाए गए व्यक्ति को उसके बंदी बनाए जाने का कारण बताया जाना आवश्यक है।

HBSE 11th Class Political Science Important Questions Chapter 2 भारतीय संविधान में अधिकार

प्रश्न 15.
सांस्कृतिक तथा शिक्षा सम्बन्धी अधिकारों का वर्णन कीजिए।
उत्तर:
इस अधिकार का वर्णन संविधान के अनुच्छेद 29 तथा 30 में किया गया है। इसके अन्तर्गत-
(1) सभी नागरिकों को अपनी भाषा, धर्म व संस्कृति को सुरक्षित रखने तथा उसका विकास करने का पूरा अधिकार है,

(2) भाषा अथवा धर्म के आधार पर अल्पसंख्यक वर्गों को अपनी इच्छानुसार शिक्षा-संस्थाएँ स्थापित करने का अधिकार है। इस प्रकार की संस्थाओं को अनुदान देने में राज्य किसी प्रकार का भेद-भाव नहीं करेगा,

(3) किसी भी नागरिक को राज्य द्वारा स्थापित अथवा सहायता से चलाए जाने वाले शिक्षा-संस्थान में प्रवेश देने में जाति, धर्म, वंश, भाषा अथवा इनमें से किसी एक के आधार पर मनाही नहीं की जा सकती।

प्रश्न 16.
भारतीय संविधान में दिए गए पाँच मौलिक कर्तव्यों की व्याख्या कीजिए। उत्तर भारतीय संविधान में दिए गए पाँच मौलिक कर्तव्य निम्नलिखित हैं

  • प्रत्येक नागरिक का कर्तव्य है कि वह संविधान, राष्ट्रीय झण्डे तथा राष्ट्रीय गीत का सम्मान करे।
  • नागरिकों का कर्तव्य है कि वे राष्ट्रीय स्वतन्त्रता के आन्दोलन के उद्देश्यों को स्मरण तथा प्रफुल्लित करें।
  • प्रत्येक नागरिक का कर्त्तव्य है कि वह भारत की प्रभुसत्ता, एकता व अखण्डता का समर्थन और रक्षा करे।
  • नागरिकों का कर्तव्य है कि वे देश की रक्षा करें तथा राष्ट्रीय सेवाओं में आवश्यकता के समय भाग लें।
  • लोगों में वैज्ञानिक दृष्टिकोण फैलाना।

प्रश्न 17.
मौलिक कर्तव्यों का क्या महत्त्व है?
उत्तर:
मौलिक कर्तव्यों का विशेष महत्त्व है। मौलिक कर्त्तव्य नागरिक को आदर्श बनाते हैं तथा उनमें जागरूकता उत्पन्न करते हैं। मौलिक कर्त्तव्य नागरिकों का दृष्टिकोण व्यापक बनाते हैं और नागरिकों में संविधान का पालन, देश की रक्षा, एकता तथा अखण्डता को बनाए रखने की भावना पैदा करते हैं। मौलिक कर्तव्यों का पालन करके लोकतन्त्र को सफल बनाया जा सकता है।

इसके फलस्वरूप व्यक्ति की व्यक्तिगत उन्नति तथा विकास के साथ-साथ समाज और देश भी प्रगति के पथ पर अग्रसर होगा। इन मौलिक कर्तव्यों के बारे में श्रीमती इन्दिरा गाँधी ने कहा था कि यदि लोग मौलिक कर्तव्यों को अपने दिमाग में रख लेंगे तो हम शीघ्र ही एक शान्तिपूर्ण एवं मैत्रीपूर्ण क्रान्ति देख सकेंगे। अतः मौलिक कर्तव्यों को संविधान में अंकित किए जाने से नागरिकों को यह ध्यान में रहेगा कि अधिकारों के साथ-साथ उनके कुछ कर्त्तव्य भी हैं।

प्रश्न 18.
संविधान में दिए गए मौलिक कर्तव्यों की आलोचना कीजिए।
उत्तर:
संविधान में दिए गए मौलिक कर्तव्यों की आलोचना निम्नलिखित तथ्यों के अन्तर्गत की जा सकती है

1. कुछ मौलिक कर्त्तव्य अस्पष्ट हैं-मौलिक कर्तव्यों का विवरण देते हुए संविधान में कुछ अस्पष्ट शब्दों का प्रयोग किया गया है, जिनकी मनमाने ढंग से व्याख्या की जा सकती है; जैसे मिली-जुली संस्कृति, वैज्ञानिक दृष्टिकोण का अन्वेषण व सुधार की भावना।

2. दण्ड की भावना का अभाव मौलिक कर्त्तव्यों के पीछे दण्ड की भावना का अभाव है। इसी संदर्भ में स्वर्ण सिंह समिति ने यह सुझाव दिया था कि मौलिक कर्तव्यों की अवहेलना करने वालों को दण्ड दिया जाना चाहिए और इसके लिए संसद द्वारा उचित कानून का निर्माण करना चाहिए।

3. केवल उच्च आदर्श मौलिक कर्तव्यों की आलोचना तीसरे स्थान पर की जा सकती है कि ये मात्र उच्च आदर्श प्रस्तुत करते. हैं। भारत की अंधिक जनसंख्या गाँवों में निवास करती है जो इन उच्च आदर्शों को समझने में असमर्थ है।

4. संविधान के तीसरे अध्याय में सम्मिलित होने चाहिएँ मौलिक कर्तव्यों को भारतीय संविधान के अध्याय चार में शामिल किया गया है, जबकि इन्हें मौलिक अधिकारों वाले अध्याय तीन में ही रखा जाना चाहिए, व कर्तव्य अच्छे लगते हैं।

प्रश्न 19.
भारतीय संविधान में दिए गए राज्य-नीति के निदेशक सिद्धान्तों का स्वरूप (प्रकृति) क्या है?
रतीय संविधान के निर्माता भारत को एक आदर्श कल्याणकारी राज्य बनाना चाहते थे। इस उद्देश्य से उन्होंने सरकार की नीति तथा शासन को सही दिशा देने के लिए संविधान में राज्य-नीति के निदेशक सिद्धान्तों को शामिल किया। ये सिद्धान्त सरकार का मार्गदर्शन करने के लिए हैं और ये न्यायसंगत नहीं हैं।

यदि सरकार इनमें से किसी भी सिद्धान्त को लागू नहीं करती, तो भी नागरिकों को न्यायालय में जाकर सरकार के विरुद्ध न्याय माँगने का अधिकार नहीं है। ये सरकार को सकारात्मक निर्देश हैं और यदि सरकार इन्हें लागू नहीं करती तो जनमत उस सरकार के विरुद्ध हो जाएगा। ऐसी सरकार के अगले चुनावों में जीतने की सम्भावना नहीं होगी।

प्रश्न 20.
हमारे संविधान में राज्य-नीति के निदेशक सिद्धान्तों से सम्बन्धित अध्याय का क्या महत्त्व है? अथवा राज्य-नीति के निदेशक सिद्धान्तों का महत्त्व बताएँ।
उत्तर:
राज्य-नीति के निदेशक सिद्धान्तों का महत्त्व इस प्रकार है-
(1) संघीय सरकार तथा राज्य सरकारों द्वारा इन सिद्धान्तों के लागू करने से भारत एक कल्याणकारी राज्य बन सकता है,

(2) संविधान की प्रस्तावना में सभी नागरिकों को राजनीतिक, आर्थिक तथा सामाजिक न्याय प्रदान करने की जो घोषणा की गई है, ये सिद्धान्त उस उद्देश्य की प्राप्ति में सहायता करते हैं,

(3) ये सिद्धान्त अन्तर्राष्ट्रीय शान्ति व सुरक्षा की स्थापना में सहायता करते हैं,

(4) सरकारें जब इन सिद्धान्तों को ध्यान में रखकर अपनी नीति का निर्माण करेंगी, तो समान कानूनों का निर्माण होगा, जिससे राष्ट्रीय एकता की स्थापना होगी,

(5) ये सिद्धान्त जनता के पास सरकार की सफलताओं को जाँचने की कसौटी है।

प्रश्न 21.
42वें संशोधन द्वारा निदेशक सिद्धान्तों में कौन-से नए सिद्धान्त जोड़े गए हैं ?
उत्तर:

  • राज्य अपनी नीति का संचालन इस प्रकार करेगा कि बच्चों को स्वतंत्र और प्रतिष्ठापूर्ण वातावरण में अपने विकास के लिए अवसर और सुविधाएँ प्राप्त हों,
  • राज्य ऐसी कानून प्रणाली के प्रचलन की व्यवस्था करेगा जो समान अवसर के आधार पर न्याय का विकास करे,
  • राज्य कानून या अन्य ढंग से श्रमिकों को उद्योगों के प्रबन्ध में भागीदारी बनाने के लिए पग उठाएगा,
  • राज्य वातावरण की सुरक्षा और विकास करने के लिए देश के वन तथा वन्य जीवन को सुरक्षित रखने का प्रयत्न करेगा।

प्रश्न 22.
कोई ऐसे पाँच निदेशक सिद्धान्त बताएँ जिनका सम्बन्ध कल्याणकारी राज्य की स्थापना से है?
उत्तर:
(1) राज्य अपनी नीति का संचालन इस प्रकार करेगा कि राज्य के सभी नागरिकों, पुरुषों तथा स्त्रियों को समान रूप से जीविका के पर्याप्त साधन प्राप्त हों,

(2) बेकारी, बुढ़ापे, बीमारी तथा अंगहीन होने की अवस्था में ओर से आर्थिक सहायता पाने का अधिकार हो,

(3) राज्य काम करने वालों के लिए न्यायपूर्ण मानवीय परिस्थितियों में काम करने की व्यवस्था का प्रबन्ध करेगा और स्त्रियों के लिए प्रसूति सहायता देने का प्रबन्ध करेगा,

(4) संविधान के लागू होने से दस वर्ष के भीतर राज्य 14 वर्ष तक की आयु के सभी बच्चों के लिए निःशुल्क तथा अनिवार्य शिक्षा का प्रबन्ध करेगा,

(5) राज्य अनुसूचित जातियों तथा जनजातियों के हितों की रक्षा की व्यवस्था करेगा।

प्रश्न 23.
राज्य-नीति के नीति निदेशक सिद्धान्तों के पीछे कौन-सी शक्ति कार्य कर रही है?
उत्तर:
राज्य-नीति के नीति निदेशक सिद्धान्त न्यायसंगत नहीं हैं अर्थात् इनके पीछे कानून की शक्ति नहीं है, परन्तु इन सिद्धान्तों के पीछे जनमत (Public Opinion) की शक्ति है। लोकतन्त्र में जनमत का बहुत महत्त्व होता है और कोई भी सरकार जनमत की अवहेलना करके अधिक समय तक पद पर बनी नहीं रह सकती।

चूँकि ये सिद्धान्त कल्याणकारी राज्य की स्थापना करने में सहायता करते हैं, अतः कोई भी सरकार अपनी नीति का निर्माण करते समय इन्हें अपनी आँखों से ओझल नहीं कर सकती। इनकी अवहेलना करने वाली सरकार के लिए अगले चुनावों में जीतने की सम्भावना नहीं होती। अतः निदेशक सिद्धान्तों के पीछे जनमत की शक्ति है।

प्रश्न 24.
भारतीय संविधान में दिए गए निदेशक सिद्धान्तों में से कोई पाँच सिद्धान्त लिखें।
उत्तर:
भारतीय संविधान में दिए गए पाँच निदेशक सिद्धान्त इस प्रकार हैं-

  • राज्य ऐसे समाज का निर्माण करेगा, जिसमें लोगों को सामाजिक, आर्थिक एवं राजनीतिक न्याय प्राप्त होगा,
  • स्त्रियों और पुरुषों को आजीविका कमाने के समान अवसर दिए जाएँगे,
  • देश के सभी नागरिकों के लिए समान कानून तथा समान न्याय संहिता की व्यवस्था होनी चाहिए,
  • बच्चों और नवयुवकों की नैतिक पतन तथा आर्थिक शोषण से रक्षा हो,
  • राज्य ग्राम पंचायतों का संगठन करेगा।

प्रश्न 25.
नीति निदेशक सिद्धान्त भारत में किस प्रकार एक धर्म-निरपेक्ष और समाजवादी समाज की नींव डालते हैं?
उत्तर:
नीति निदेशक सिद्धान्त धर्म-निरपेक्ष और समाजवादी समाज की आधारशिला रखते हैं। निदेशक सिद्धान्त राज्य सरकार को भारत में एक समान व्यवहार संहिता लागू करने का निर्देश देते हैं। निदेशक सिद्धान्तों के अनुसार राज्य एक ऐसी सामाजिक व्यवस्था का निर्माण करेगा, जिसमें सभी नागरिकों को राष्ट्रीय जीवन के प्रत्येक क्षेत्र में सामाजिक, आर्थिक तथा राजनीतिक न्याय प्राप्त होगा।

प्रश्न 26.
भारत की विदेश नीति से सम्बन्धित निदेशक सिद्धान्त लिखें। अथवा अन्तर्राष्ट्रीय नीति सम्बन्धी सिद्धान्तों का वर्णन करें।
उत्तर:
राज्य-नीति के निदेशक सिद्धान्त राष्ट्रीय नीति के साथ-साथ अन्तर्राष्ट्रीय नीति से भी सम्बन्धित हैं। अनुच्छेद 51 में कहा गया है कि राज्य-

  • अन्तर्राष्ट्रीय शान्ति व सुरक्षा को बढ़ावा देगा,
  • राष्ट्रों के मध्य उचित व सम्मानपूर्वक सम्बन्ध बनाए रखेगा,
  • अन्तर्राष्ट्रीय सन्धि व कानूनों को सम्मान देगा,
  • अन्तर्राष्ट्रीय विवादों का मध्यस्थता द्वारा निपटारा करने का प्रयास करेगा।

प्रश्न 27.
किन बातों के आधार पर राज्य-नीति के निदेशक सिद्धान्तों की आलोचना की गई है?
उत्तर:
राज्य-नीति के निदेशक सिद्धान्तों की आलोचना दी गई बातों के आधार पर की गई है

  • राज्य-नीति के निदेशक सिद्धान्त न्याय-संगत नहीं हैं,
  • राज्य-नीति के निदेशक सिद्धान्तों में उचित वर्गीकरण तथा स्पष्टता का अभाव है,
  • निदेशक सिद्धान्त केवल आश्वासन मात्र हैं,
  • राज्य-नीति के निदेशक सिद्धान्त अव्यावहारिक हैं,
  • निदेशक सिद्धान्तों में स्थायित्व की कमी है।

प्रश्न 28.
मौलिक अधिकारों तथा राज्य-नीति के निदेशक सिद्धान्तों में कोई पाँच अन्तर बताएँ।
उत्तर:
मौलिक अधिकारों तथा राज्य-नीति के निदेशक सिद्धान्तों में अन्तर इस प्रकार हैं-

  • मौलिक अधिकार न्याय-योग्य हैं, जबकि राज्य-नीति के निदेशक सिद्धान्त न्याय-योग्य नहीं हैं,
  • मौलिक अधिकारों का स्वरूप निषेधात्मक है, जबकि निदेशक सिद्धान्त अधिकतर सकारात्मक हैं,
  • मौलिक अधिकार नागरिकों के लिए हैं, जबकि निदेशक सिद्धान्त सरकार के लिए हैं,
  • मौलिक अधिकार राजनीतिक लोकतन्त्र का आधार हैं, जबकि निदेशक सिद्धान्त सामाजिक तथा आर्थिक लोकतन्त्र की स्थापना में सहायक हैं,
  • मौलिक अधिकारों को स्थगित किया जा सकता है, जबकि निदेशक सिद्धान्तों को स्थगित नहीं किया जा सकता।

प्रश्न 29.
मौलिक अधिकारों व राज्य-नीति के निदेशक तत्त्वों में टकराव की स्थिति में प्राथमिकता का क्या प्रश्न है?
उत्तर:
संविधान में स्पष्ट रूप से इस प्रश्न का उत्तर नहीं दिया गया है। इस प्रश्न का उत्तर सर्वोच्च न्यायालय ने अनेक निर्णयों में दिया है। सर्वप्रथम चम्पाकम दोराय राजन बनाम चेन्नई राज्य, 1951 के मुकद्दमे में सर्वोच्च न्यायालय ने मौलिक अधिकारों में वाद-योग्य होने के कारण माना कि यदि मौलिक अधिकारों व राज्य-नीति के निदेशक तत्त्वों में टकराव पाया जाता है तो मौलिक अधिकारों को प्राथमिकता दी जानी चाहिए, लेकिन शीघ्र ही सर्वोच्च न्यायालय ने अपनी गलती को महसूस किया और इनरी केरल एजुकेशन विधेयक, चन्द्रभवन बोर्डिंग केस, विजय कॉटन मिल्स केस इत्यादि अनेक मामलों में दोनों को समान धरातल पर माना और निदेशक तत्त्वों को मौलिक अधिकारों पर युक्ति-युक्त प्रतिबन्ध के रूप में स्वीकार किया।

संसद ने सन् 1971 में संविधान के 25वें संशोधन के द्वारा अनुच्छेद 39 (b) व (c) में निहित निदेशक तत्त्वों की अनुच्छेद 14, 19 व 31 में निहित मौलिक अधिकारों पर प्राथमिकता स्थापित की। इस संशोधन को सर्वोच्च न्यायालय ने केशवानन्द भारती केस, सन् 1973 में संविधान के मूलभूत ढाँचे के अन्तर्गत नहीं माना, लेकिन इसके बाद सन् 1976 में संसद ने जब 42वें संविधान संशोधन के द्वारा सभी नीति-निदेशक तत्त्वों की अनुच्छेद 14, 19 व 31 में निहित समानता, स्वतन्त्रता व सम्पत्ति के मौलिक अधिकारों पर प्राथमिकता स्थापित की तो सर्वोच्च न्यायालय ने सन् 1980 में मिनर्वा मिल्स केस में इस संशोधन को संविधान के मूलभूत ढाँचे के विरुद्ध मानते हुए अवैध घोषित कर दिया और प्रतिपादित किया कि संविधान का मूलभूत ढाँचा मौलिक अधिकारों व नीति-निदेशक तत्त्वों के मध्य सन्तुलन पर आधारित है।

दोनों एक-दूसरे के पूरक व सम्पूरक हैं। दोनों में से किसी एक को प्राथमिकता देना संविधान के मूलभूत ढाँचे को विकृत तथा नष्ट करना होगा। … इस प्रकार सर्वोच्च न्यायालय ने दोनों को समान माना है जिसमें केवल एक अपवाद अनुच्छेद 39 (b) व (c) की अनुच्छेद 14 व 19 में निहित समानता व स्वतन्त्रता के मौलिक अधिकारों पर प्राथमिकता है, क्योंकि अनुच्छेद 31 को सन् 1978 के 44वें समाप्त कर दिया गया है। इस प्रकार मौलिक अधिकार और राज्य-नीति के निदेशक सिद्धान्त एक-दूसरे के पूरक हैं।

HBSE 11th Class Political Science Important Questions Chapter 2 भारतीय संविधान में अधिकार

प्रश्न 30.
राष्ट्रीय आपात का मौलिक अधिकारों पर प्रभाव पर नोट लिखिए।
उत्तर:
भारत के संविधान में ऐसी व्यवस्था है कि राष्ट्रीय आपात् का भारत के नागरिकों के मौलिक अधिकारों पर सीधा प्रभाव पड़ता है। भारत का संविधान भारत के राष्ट्रपति को यह अधिकार प्रदान करता है कि जब देश पर किसी विदेशी शक्ति का आक्रमण हुआ हो या आक्रमण होने की सम्भावना हो या देश में सशस्त्र विद्रोह फैल गया हो या ऐसा विद्रोह फैलने की सम्भावना हो तो वह देश में राष्ट्रीय आपात् की घोषणा कर सकता है।

राष्ट्रीय आपात् काल की घोषणा करके नागरिकों के मौलिक अधिकारों को स्थगित कर सकता है। यहाँ तक कि संवैधानिक उपचारों के अधिकार को भी स्थगित किया जा सकता है। नागरिक अपने अधिकारों की रक्षा के लिए न्यायालय में भी शरण नहीं ले सकते। ऐसी स्थिति में कार्यपालिका अपनी शक्तियों का दुरुपयोग कर सकती है, जैसा कि सन् 1975 में आपातकालीन घोषणा के बाद प्रशासन ने मनमाने तरीके से शक्ति का प्रयोग किया। इस प्रकार से मौलिक अधिकारों का स्थगित किया जाना प्रजातन्त्र की भावना के विपरीत है।

निबंधात्मक प्रश्न

प्रश्न 1.
मौलिक अधिकारों से क्या अभिप्राय है? मौलिक अधिकारों की प्रकृति अथवा विशेषताओं का वर्णन कीजिए। अथवा मौलिक अधिकारों से आप क्या समझते हैं? भारतीय नागरिकों के मौलिक अधिकारों की विशेषताएँ लिखिए।
उत्तर:
प्रो० लास्की के अनुसार, “मूल अधिकार जीवन की उन आवश्यक अवस्थाओं को कहते हैं, जिनके बिना व्यक्ति के जीवन का विकास असम्भव है।” (Rights are those conditions of Social Life without which no man can seek in general, to be himself at his best.), अर्थात् प्रो० लास्की के अनुसार ये आवश्यक अवस्थाएँ व्यक्ति के चरित्र के विकास के लिए अनिवार्य हैं, क्योंकि व्यक्ति और राज्य की उन्नति एक-दूसरे के साथ जुड़ी हुई है,

अतः अधिकार एक प्रकार से राज्य के विकास और उन्नति के लिए भी अनिवार्य हैं। लगभग सभी राज्य अपने नागरिकों के लिए इन आवश्यक अवस्थाओं अथवा सुविधाओं की संविधान में व्यवस्था करते हैं। इंग्लैण्ड जैसे अलिखित संविधान में मौलिक अधिकार अलिखित हैं, परन्तु अमेरिका जैसे राज्यों में, जहां लिखित संविधान है, मौलिक अधिकार लिखित संविधान का भाग हैं।

भारतीय संविधान लिखित संविधान है, अतः भारत में मूल अधिकार लिखित संविधान का भाग हैं। संविधान के तीसरे भाग के 12 से 35 अनुच्छेदों में मूल अधिकार दिए गए हैं। भारत में मौलिक अधिकार केवल लिखित ही नहीं, बल्कि न्याय-योग्य भी हैं। मौलिक अधिकारों को न्याय-योग्य इसलिए ठहराया गया है कि विधानमण्डल और कार्यकारिणी के सदस्य नागरिकों को सताने न लगें।

कहने का तात्पर्य यह है कि न्यायपालिका पर लोगों के अधिकारों की रक्षा का उत्तरदायित्व है अर्थात् न्यायपालिका का यह कर्तव्य है कि वह देखे कि मौलिक अधिकारों का अतिक्रमण न होने पाए, अतएव न्यायपालिका के पास यह शक्ति है कि वह विधानमण्डल द्वारा बनाए गए उन सब कानूनों को असंवैधानिक घोषित कर सकती है जो मौलिक अधिकारों का हनन् करते हैं। मौलिक अधिकारों के तत्त्व या विशेषताएँ (Characteristics or Features of Fundamental Rights)-मौलिक अधिकारों के कुछ विशेष तत्त्व निम्नलिखित हैं

1. मौलिक अधिकार न्यायसंगत हैं (Fundamental Rights are Justiciable):
मौलिक अधिकार केवल नाममात्र के ही नहीं हैं, बल्कि इनको अदालत के द्वारा संरक्षित किया गया है। यदि किसी मनुष्य का अधिकार सरकार द्वारा या किसी और द्वारा छीना जाए तो वह अदालत की शरण ले सकता है और उसको न्याय मिलेगा। हम मौलिक अधिकारों की रक्षा के लिए सीधे ही उच्चतम तथा उच्च न्यायालय में जा सकते हैं।

2. मौलिक अधिकार सीमित हैं (Fundamental Rights are Limited):
ये मौलिक अधिकार सीमित हैं। नागरिक इनका प्रयोग मनमानी से नहीं कर सकता। इनके ऊपर उचित प्रतिबन्ध लगाए हुए हैं। सुरक्षा, शान्ति बनाए रखने के लिए इन पर प्रतिबन्ध लगाए गए हैं।

3. अति विस्तृत (Most Elaborate):
संविधान के तृतीय भाग में 24 अनुच्छेद (अनुच्छेद 12 से 35) नागरिक के मौलिक अधिकारों से सम्बन्धित हैं। अनुच्छेद 12, 13, 33, 34 तथा 35 में ऐसे साधारण उपबन्ध हैं, जिनका सम्बन्ध सभी अधिकारों से है। अनुच्छेद 14 से 30 और अनुच्छेद 32 में नागरिकों को छः प्रकार के मौलिक अधिकार दिए गए हैं। मौलिक रूप में अनुच्छेद 31 द्वारा नागरिकों को ‘सम्पत्ति का अधिकार’ दिया गया था, परन्तु 44वें संशोधन द्वारा इस अनुच्छेद को संविधान में से निकाल दिया गया है।

4. मौलिक अधिकार निलम्बित किए जा सकते हैं (Fundamental Rights can be Suspended):
अनुच्छेद 352, 356 तथा 360 द्वारा राष्ट्रपति को आपात्काल की स्थिति की घोषणा करने का अधिकार दिया गया है। राष्ट्रपति आपात्काल की घोषणा. द्वारा इन अधिकारों को निलम्बित कर सकता है। इसके अतिरिक्त अनुच्छेद 32 में वर्णित सवैधानिक उपचारों के अधिकार पर भी राष्ट्रपति आपातकालीन स्थिति में प्रतिबन्ध लगा सकता है।

यहाँ यह वर्णन करने योग्य है कि नागरिकों के जीवन और व्यक्तिगत स्वतन्त्रता के अधिकार को लागू करवाने के लिए किसी आपात्कालीन अवस्था में भी नागरिकों के अदालत में शरण लेने के अधिकार को समाप्त नहीं किया जा सकता। यह व्यवस्था भारतीय संविधान में 44वें संवैधानिक संशोधन द्वारा की गई है।

5. संसद मौलिक अधिकारों को सीमित कर सकती है (Parliament can Curtail the Fundamental Rights):
संसद मौलिक अधिकारों में हर प्रकार का परिवर्तन कर सकती है। यह व्यवस्था 1971 में 24वें संशोधन द्वारा संविधान में की गई थी। 42वें संविधान संशोधन द्वारा भी यह व्यवस्था की गई है कि संसद मौलिक अधिकारों वाले प्रकरण सहित सम्पूर्ण संविधान में परिवर्तन कर सकती है तथा संसद द्वारा किए गए संशोधन को किसी भी न्यायालय में किसी भी आधार पर चुनौती नहीं दी जा सकती। इसका अभिप्रायः यह हुआ कि संसद संविधान संशोधनों द्वारा मौलिक अधिकारों को कम या सीमित कर सकती है।

6. सकारात्मक व नकारात्मक प्रकृति (Positive and Negative Nature):
मौलिक अधिकारों की सकारात्मक प्रकृति से तात्पर्य है कि मौलिक अधिकार राज्य को कुछ कार्य करने का अधिकार प्रदान करते हैं और नकारात्मक प्रकृति से अर्थ है कि मौलिक अधिकार राज्य को कुछ कार्य करने से रोकते हैं।

भारतीय मौलिक अधिकारों में ये दोनों ही प्रवृतियाँ पाई जाती हैं। इसका सर्वोत्तम उदाहरण समानता का अधिकार है, जिसमें नागरिकों को ‘कानून के समक्ष समानता’ (Equality before Law) के साथ-साथ ‘कानून का समान संरक्षण’ (Equal Protection of Law) का अधिकार दिया गया है।

‘कानून के समक्ष समानता’ की प्रकृति नकारात्मक है, क्योंकि यह राज्य को नागरिकों में भेद-भाव करने से रोकती है और इसके विपरीत ‘कानून का समान संरक्षण’ की प्रकृति सकारात्मक है, क्योंकि इसके अन्तर्गत राज्य नागरिकों में भेद-भाव कर सकता है, ताकि गरीब व पिछड़े वर्गों का समान संरक्षण मिल सके।

7. मौलिक अधिकार संविधान द्वारा दिए गए हैं, प्राकृतिक अधिकार नहीं (Fundamental Rights are conferrred by the Constitution, not Natural Rights):
भारत में केवल वे ही मौलिक अधिकार हैं जो संविधान में दिए गए हैं। इसके अतिरिक्त अन्य किसी अधिकार को मौलिक अधिकार के रूप में मान्यता प्राप्त नहीं है। अमेरिका की भाँति भारत में भी प्राकृतिक अधिकारों को मौलिक अधिकारों के रूप में मान्यता प्राप्त नहीं है। न्यायपालिका का संरक्षण संविधान में दिए गए मौलिक अधिकारों को ही प्राप्त है।

8. भारतीय पृष्ठभूमि (Indian Background):
भारत में मौलिक अधिकार किसी अन्य देश की नकल नहीं हैं, बल्कि विशिष्ट भारतीय पृष्ठभूमि के अनुसार प्रदान किए गए हैं, जैसे छुआछूत का निषेध । छुआछूत भारतीय समाज में ही पाया जाता था, इसलिए इसका निषेध एक मौलिक अधिकार के रूप में प्रदान किया गया है। इसी प्रकार से धार्मिक स्वतन्त्रता का अधिकार भारतीय समाज के बहुधर्मी रूप को देखते हुए दिया गया है।

9. सभी पर लागू होते हैं (Binding upon Everybody):
मौलिक अधिकार राज्य की सभी संस्थाओं, जिनमें विधायिका, कार्यपालिका, न्यायपालिका, सरकारी संस्थाएँ और यहाँ तक कि भारतीय नागरिक भी शामिल हैं, पर समान रूप से लागू होते हैं। कोई भी इनको मानने से इन्कार नहीं कर सकता और न ही किसी को इनके उल्लंघन का अधिकार है।

10. मुख्यतया राजनीतिक प्रकृति के (Primarily of Political Nature):
भारत में मौलिक अधिकार मुख्य रूप से राजनीतिक प्रकृति के हैं, सामाजिक और आर्थिक अधिकार राज्य-नीति के निदेशक तत्त्वों में दिए गए हैं। संविधान सभा का विचार था कि चूंकि अभी भारत के पास इतने अधिक साधन नहीं हैं कि नागरिकों को सामाजिक व आर्थिक अधिकार भी सुलभ कराए जा सकें। अतः राजनीतिक अधिकार ही प्रदान कर दिए गए, ताकि राजनीतिक दृष्टि से लोकतन्त्र स्थापित किया जा सके।

11. नागरिक व व्यक्ति में अन्तर किया गया है (Difference between Citizen and People):
भारत में कुछ मौलिक अधिकार केवल नागरिकों को प्राप्त हैं तो कुछ अधिकार व्यक्तियों को प्राप्त हैं। व्यक्तियों को दिए गए अधिकार नागरिकों व विदेशियों दोनों को प्राप्त हैं, जब कि नागरिकों को दिए गए अधिकार विदेशियों को प्राप्त नहीं हैं। उदाहरणार्थ, स्वतन्त्रता का अधिकार केवल नागरिकों को प्राप्त है, जबकि वैयक्तिक स्वतन्त्रता का अधिकार व्यक्तियों को प्राप्त है। समानता का अधिकार यदि नागरिकों को प्राप्त है तो शोषण के विरुद्ध अधिकार व्यक्तियों को प्राप्त है।

12. केन्द्र तथा राज्य सरकारों की शक्तियों पर प्रतिबन्ध (Limitations on the Powers of the Centre and the State Governments):
मौलिक अधिकार केन्द्र तथा राज्य सरकारों की शक्तियों पर प्रतिबन्ध लगाते हैं अर्थात् केन्द्रीय सरकार, राज्य सरकारों तथा स्थानीय सरकारों को मौलिक अधिकारों के अनुसार ही कानून बनाने पड़ते हैं और ये सरकारें कोई ऐसा कानून नहीं बना सकतीं जो मौलिक अधिकारों की प्रकृति के विरुद्ध हो। यदि ये सरकारें मौलिक अधिकारों के विरुद्ध कोई कानून बना दें तो न्यायालय उन्हें अवैध घोषित कर सकता है।

प्रश्न 2.
भारतीय संविधान द्वारा भारत के नागरिकों को प्रदान किए गए मौलिक अधिकारों का उन पर लगी सीमाओं सहित विवेचन कीजिए।
अथवा
हमारे संविधान में निहित मौलिक अधिकारों पर एक लेख लिखिए।
अथवा
निम्नलिखित पर नोट लिखिए
(1) समानता का अधिकार, अनुच्छेद 14 से 18 तक,
(2) स्वतन्त्रता का अधिकार, अनुच्छेद 19 से 22 तक,
(3) शोषण के विरुद्ध अधिकार, अनुच्छेद 23 से 24 तक,
(4) संस्कृति एवं शिक्षा सम्बन्धी अधिकार, अनुच्छेद 29 से 30 तक।
उत्तर:
सन् 1979 से पहले भारतीय नागरिकों को 7 प्रकार के मौलिक अधिकार प्राप्त थे, परन्तु इसके पश्चात् इन अधिकारों की संख्या 6 हो गई है। 30 अप्रैल, 1979 को 44वाँ संविधान संशोधन राष्ट्रपति द्वारा स्वीकृत किया गया था। इस संशोधन के अन्तर्गत नागरिकों का ‘सम्पत्ति का अधिकार’ (Right to Property) मौलिक अधिकारों की सूची में से निकाल दिया गया और अब यह अधिकार एक साधारण अधिकार (Ordinary Right) बन गया है। 44वें संशोधन के द्वारा सम्पत्ति के अधिकार सम्बन्धी यह व्यवस्था 19 जून, 1979 को लागू की गई थी। भारतीयों के शेष छः अधिकारों का विस्तारपूर्वक वर्णन निम्नलिखित है

1. समानता का अधिकार, अनुच्छेद 14 से 18 तक [Right to Equality, Articles 14 to 18|-अनुच्छेद 14 से 18 में वर्णित अधिकारों द्वारा भारतीयों को समानता का अधिकार दिया गया है। समानता के अधिकार के कई पक्ष हैं, जिनका संक्षिप्त वर्णन निम्नलिखित है

(i) कानून के सामने समानता (Equality before Law):
अनुच्छेद 14 के अनुसार सभी व्यक्ति कानून के सामने समान हैं। कानून की दुनिया में ऊँच-नीच, अमीर-गरीब, रंग या नस्ल, जाति, जन्म, धर्म आदि के आधार पर कोई मतभेद नहीं है। राष्ट्रपति से लेकर साधारण नागरिक तक सभी कानून की दृष्टि में समान समझे जाते हैं तथा कानून समान रूप से ही सबकी रक्षा करता है।

(ii) कोई भेदभाव नहीं (No Discrimination) संविधान के अनुच्छेद 15 के अनुसार राज्य किसी नागरिक के साथ जाति, धर्म, नस्ल, लिंग, रंग आदि के आधार पर भेदभाव नहीं करेगा। सार्वजनिक स्थानों, जैसे होटल, तालाब, कुएँ, सिनेमा घर, दुकानों आदि, के प्रयोग के लिए किसी को मनाही नहीं होगी। अपवाद (Exceptions) अनुच्छेद 15 के दो अपवाद हैं-(क) राज्य बच्चों व महिलाओं के लिए विशेष सुविधाएँ प्रदान कर सकता है, (ख) पिछड़े वर्गों, अनुसूचित जातियों तथा जनजातियों के उत्थान के लिए राज्य विशेष प्रकार की व्यवस्था कर सकता है।

(iii) अवसर की समानता (Equality of Opportunity):
अनुच्छेद 16 के अनुसार सरकारी पदों पर नियुक्ति योग्यता के आधार पर होगी। नियुक्ति करते समय सरकार किसी व्यक्ति के साथ रंग, नस्ल, जाति, जन्म, धर्म के आधार पर भेदभाव नहीं कर सकती। किसी व्यक्ति को इन बातों के आधार पर उसके पद से पदच्युत नहीं किया जा सकता, परन्तु सरकार को संविधान की ओर से अनुसूचित जातियों तथा जनजातियों के लिए स्थान सुरक्षित रखने की छूट है।

अपवाद (Exceptions) इस व्यवस्था के तीन अपवाद हैं-प्रथम, कुछ विशेष पदों के लिए निवास स्थान सम्बन्धी आवश्यक शर्ते लगाई जा सकती हैं। द्वितीय, पिछड़े वर्गों के लिए आरक्षण की व्यवस्था की जा सकती है। तृतीय, किसी धार्मिक अथवा साम्प्रदायिक संस्था से सम्बन्धित पदों पर एक विशेष धर्म अथवा सम्प्रदाय के लोगों की नियुक्ति की जा सकती है।।

(iv) अस्पृश्यता का अन्त (Abolition of Untouchability):
अनुच्छेद 17 के अनुसार अस्पृश्यता को अवैध घोषित किया गया है। जो व्यक्ति अस्पृश्यता को किसी भी तरीके से लागू करने का यत्न करता है, अथवा प्रोत्साहित करता है उसको कानून द्वारा दण्ड दिया जा सकता है। इसके अतिरिक्त सभी सार्वजनिक स्थान मन्दिर, होटल, कुएँ, स्कूल, कॉलेज आदि हरिजन लोगों के लिए खुले हैं। उनको इनके प्रयोग से रोकना कानूनी अपराध है।

(v) उपाधियों की समाप्ति (Abolition of Titles):
अंग्रेज़ सरकार भारत में नागरिकों को कई प्रकार की उपाधियाँ प्रदान करती थी। ये उपाधियाँ हमारे भारतीय समाज में भेदभाव की भावना पैदा करती थीं। ऐसी स्थिति की समाप्ति के लिए भारतीय संविधान के अनुच्छेद 18 द्वारा शैक्षणिक अथवा सैनिक उपाधियों के अलावा राज्य को अन्य किसी प्रकार की उपाधियाँ देना वर्जित किया गया है।

2. स्वतन्त्रता का अधिकार, अनुच्छेद 19 से 22 तक (Right to Freedom,Articles 19 to 22):
संविधान में स्वतन्त्रता के अधिकार का वर्णन अनुच्छेद 19 से 22 तक किया गया है। यह अधिकार ‘मौलिक अधिकारों की आत्मा’ है, क्योंकि इस अधिकार के बिना अन्य अधिकारों का कोई महत्त्व नहीं रहता। इस अधिकार के आधार पर ही प्रजातन्त्रीय समाज की कल्पना की जा सकती है। इस विषय में पायली महोदय का कथन महत्त्वपूर्ण है।

उन्होंने कहा है, “संविधान निर्माताओं ने इन अधिकारों को मौलिक अधिकारों के अध्याय में शामिल करके ठीक ही किया है तथा इस प्रकार प्रजातन्त्रीय समाज के विकास में सहायता की है।”

(1) अनुच्छेद 19 के द्वारा निम्नलिखित स्वतन्त्रताएँ प्रदान की गई हैं…
(क) भाषण तथा लेखन की स्वतन्त्रता,
(ख) शान्तिपूर्वक तथा बिना शस्त्रों के इकट्ठा होने की स्वतन्त्रता,
(ग) संघ तथा समुदाय बनाने की स्वतन्त्रता,
(घ) भारत के किसी भी क्षेत्र में आने-जाने की स्वतन्त्रता,
(ङ) भारत के किसी भाग में रहने या निवास करने की स्वतन्त्रता,
(च) कोई भी व्यवसाय करने, पेशा अपनाने या व्यापार करने की स्वतन्त्रता।

इनका संक्षिप्त वर्णन इस प्रकार है
(क) भाषण एवं लेखन की स्वतन्त्रता (Freedom of Speech and Expression)-सभी नागरिकों को भाषण देने और अपने विचार प्रकट करने की स्वतन्त्रता प्रदान की गई है। वे बोलकर या लिखकर और छपवाकर अपने विचार प्रकट कर सकते हैं। लोकतन्त्र में इस अधिकार का बड़ा महत्त्व होता है, क्योंकि इसी के द्वारा जनमत का निर्माण और अभिव्यक्ति हो सकती है।

परन्तु इस अधिकार पर राज्य न्यायालय के अपमान, सदाचार तथा नैतिकता, राज्य की सुरक्षा आदि के आधार पर उचित प्रतिबन्ध लगा सकता है। कोई भी नागरिक इस अधिकार का प्रयोग दूसरे का अपमान करने के लिए नहीं कर सकता।

(ख) शान्तिपूर्वक तथा बिना शस्त्रों के इकट्ठा होने की स्वतन्त्रता (Freedom to Assemble Peacefully and without Arms) नागरिकों को बिना हथियार और शान्तिपूर्ण तरीके से इकट्ठा होने, सभा करने तथा जुलूस निकालने की स्वतन्त्रता प्रदान की गई है। इन सभाओं और जुलूसों में नागरिक अपने विचार प्रकट कर सकते हैं तथा अपने उद्देश्यों की पूर्ति के प्रयत्न कर सकते हैं।

इस स्वतन्त्रता पर भी राज्य उचित प्रतिबन्ध लगा सकता है। सार्वजनिक शांति और व्यवस्था, भारत की अखण्डता व सुरक्षा की दृष्टि से इस पर प्रतिबन्ध लगाया जा सकता है। अनुच्छेद 144 का लगाया जाना इसी प्रतिबन्ध का एक उदाहरण है।

(ग) संघ तथा समुदाय बनाने की स्वतन्त्रता (Freedom to form Associations and Unions)-नागरिकों को अपने विभिन्न लक्ष्यों की पूर्ति के लिए संगठित होने और संघ तथा समुदाय बनाने की स्वतन्त्रता दी गई है। इन संघों और समुदायों को भी अपना कार्य स्वतन्त्रता-पूर्वक करने का अधिकार है। परन्तु कोई समुदाय या संघ ऐसा कार्य नहीं कर सकता, जिससे देश की अखण्डता व सुरक्षा को खतरा पैदा हो, जो अनैतिक हो अथवा शान्ति व व्यवस्था में बाधक बने।

(घ) भारत के किसी भी क्षेत्र में आने-जाने की स्वतन्त्रता (Freedom to Move freely Throughout the Territory of India)- सभी नागरिकों को भारत के समस्त क्षेत्र में घूमने-फिरने और आने-जाने की स्वतन्त्रता प्रदान की गई है। एक स्थान से दूसरे स्थान पर आने-जाने के लिए किसी भी तरह का आज्ञा-पत्र लेने की आवश्यकता नहीं है।

नागरिक भारत के एक कोने से दूसरे कोने तक बिना रोक-टोक के आ-जा सकते हैं, परन्तु इस स्वतन्त्रता पर भी सार्वजनिक शांति, सुरक्षा, व्यवस्था तथा अनुसूचित कबीलों के हितों की दृष्टि से उचित सीमा लगाई जा सकती है और नागरिकों के घूमने-फिरने की स्वतन्त्रता पर प्रतिबन्ध लगाए जाते रहे हैं।

(ङ) भारत के किसी भाग में रहने और निवास करने की स्वतन्त्रता (Freedom to Reside and Settle in any part of the Territory of India)-भारतीय नागरिकों को भारत के किसी भी भाग में निवास करने और बस जाने की स्वतन्त्रता दी गई है। एक राज्य से दूसरे राज्य में जाकर रहने और निवास करने पर कोई अंकुश नहीं है, नागरिक जहाँ उचित समझे रह सकता है, परन्तु राज्य इस पर भी उचित प्रतिबन्ध लगा सकता है।

(च) कोई भी व्यवसाय करने, पेशा अपनाने या व्यापार करने की स्वतन्त्रता (Freedom to practise any Profession or carry on any Occupation, Trade or Business)- सरकार किसी नागरिक को कोई कार्य विशेष करने या न करने के लिए बाध्य नहीं कर सकती। अपनी आजीविका कमाने के लिए नागरिकों को कोई भी व्यवसाय, पेशा या व्यापार करने की स्वतन्त्रता है, जिसे वे उचित समझें।

इस स्वतन्त्रता पर भी उचित प्रतिबन्ध है। सरकार जन-हित में किसी भी व्यापार, काम-धन्धे और व्यवसाय पर प्रतिबन्ध लगा सकती है और अनैतिक व्यापार को रोक सकती है। सरकार किसी व्यवसाय के लिए व्यावसायिक योग्यताएँ भी निश्चित कर सकती है, जैसे चिकित्सा, व्यवसाय, वकालत आदि के लिए योग्यताएँ। सरकार कानून द्वारा किसी भी व्यापार को अपने स्वामित्व में ले सकती है।

इस प्रकार स्पष्ट है कि सभी स्वतन्त्रताओं को असीमित रूप में नहीं दिया गया, बल्कि उन पर उचित प्रतिबन्ध लगाए गए हैं और लगाए जा सकते हैं। अधिकतर सार्वजनिक शांति और व्यवस्था, राज्य की सुरक्षा और अखण्डता, सार्वजनिक नैतिकता, लोक-हित, अनुसूचित जातियों और कबीलों के हितों आदि के आधार पर ही ये प्रतिबन्ध लगे हुए हैं और लगाए जा सकते हैं। जब नगर में अशांति हो तो कयूं भी लगाया जाता है और घर से निकलने पर भी प्रतिबन्ध लगाया जा सकता है।

(2) अनुच्छेद 20 के अनुसार-

(क) व्यक्ति को किसी ऐसे कानून का उल्लंघन करने पर दण्ड नहीं दिया जा सकता जो उसके अपराध करते समय लागू नहीं था।
(ख) किसी व्यक्ति को एक ही अपराध की एक से अधिक बार सजा नहीं दी जा सकती।
(ग) किसी अपराधी को स्वयं अपने विरुद्ध गवाही देने के लिए विवश नहीं किया जा सकता।

(3) अनुच्छेद 21 के अनुसार, किसी व्यक्ति को कानून द्वारा स्थापित विधि के अतिरिक्त जीवन और व्यक्तिगत स्वतन्त्रता से वंचित नहीं किया जा सकता। यहाँ यह उल्लेखनीय है कि 44वें संशोधन द्वारा संविधान में यह व्यवस्था की गई है कि नागरिकों के इस अधिकार को आपातकाल के समय भी समाप्त नहीं किया जा सकता। इसका भाव यह हुआ कि आपात्कालीन स्थिति के दौरान अन्य स्वतन्त्रताएँ तो समाप्त की जा सकती हैं, परन्तु नागरिकों की ‘जीवन या व्यक्तिगत स्वतन्त्रता’ को ऐसी स्थिति में भी समाप्त नहीं किया जा सकता।

(4) अनुच्छेद 22 विशेष रूप से बन्दियों के अधिकारों की घोषणा करता है। इस अनुच्छेद के अनुसार,
(क) किसी भी व्यक्ति को उसके अपराध से अवगत कराए बिना बन्दी नहीं बनाया जा सकता।
(ख) अपराधी को उसकी इच्छानुसार किसी वकील से परामर्श लेने की छूट है।
(ग) अपराधी को गिरफ्तार करने के 24 घण्टे के अन्दर-अन्दर किसी निकटतम मैजिस्ट्रेट के सामने पेश करना आवश्यक है।
(घ) न्यायालय की अनुमति के बिना किसी दोषी को 24 घण्टे से अधिक बन्दी नहीं रखा जा सकता।

अपवाद इस अधिकार के निम्नलिखित अपवाद भी हैं-
(क) ये अधिकार शत्रु-देश के नागरिकों को प्राप्त नहीं होंगे,

(ख) निवारक नजरबन्दी (Preventive Detention) के अधीन की गई गिरफ्तारी के सन्दर्भ में उपर्युक्त व्यवस्थाएँ लागू नहीं होंगी, निवारक नजरबन्दी के सम्बन्ध में 44वें संविधान संशोधन द्वारा व्यवस्थाएँ की गई हैं कि-
(क) नजरबन्दी का मामला दो महीने के अन्दर सलाहकार मण्डल (Advisory Board) के पास जाना आवश्यक है,
(ख) सलाहकार मण्डल का गठन उच्च न्यायालय के मुख्य न्यायाधीश द्वारा किया जाएगा,
(ग) सलाहकार मण्डल का अध्यक्ष उच्च न्यायालय का वर्तमान न्यायाधीश होगा, लेकिन उसके अन्य सदस्य वर्तमान अथवा सेवानिवृत्त न्यायाधीश हो सकते हैं,
(घ) नजरबन्द किए गए व्यक्ति को शीघ्र-से-शीघ्र उसकी नजरबन्दी का कारण बताया जाएगा।

उपर्युक्त व्यवस्थाएँ 1975-77 की आपात स्थिति के कटु अनुभवों को ध्यान में रखकर ही की गई थीं। वर्तमान में राष्ट्रीय सुरक्षा अधिनियम (N.S.A.) इसी अनुच्छेद के अन्तर्गत बनाया गया है। आलोचना (Criticism)-स्वतन्त्रता के मौलिक अधिकार की निम्नलिखित अनुच्छेदों पर आलोचना की जाती है

(क) नागरिकों की स्वतन्त्रताओं पर अनेक सीमाएँ लगा दी गई हैं, जिसके परिणामस्वरूप राज्य-सत्ता के सम उपर्युक्त स्वतन्त्रताएं अर्थहीन होकर रह जाती हैं। ये स्वतन्त्रताएं यदि एक हाथ से दी गई हैं तो दूसरे हाथ से छीन ली गई हैं।

(ख) सीमाएँ अत्यधिक व्यापक होने के कारण अस्पष्टता से ग्रसित हो जाती हैं। परिणामस्वरूप विधायिका व न्यायपालिका में टकराव की सम्भावना बनी रहती है।

(ग) निवारक नजरबन्दी का अधिकार राज्य को प्राप्त है जिसके कारण शान्ति काल में भी जीवन तथा निजी स्वतन्त्रता का अधिकार अर्थहीन हो जाता है। न्यायाधीश मुखर्जी के शब्दों में, “जहाँ तक मुझे मालूम है संसार के किसी भी देश में निवारक नज़रबन्दी को संविधान का अटूट भाग नहीं बनाया गया है, जैसा कि भारत में किया गया है, यह वास्तव में दुर्भाग्यपूर्ण है।”

यद्यपि. उपर्युक्त आलोचनाएँ सही हैं और लोकतन्त्र पर प्रश्न-चिह्न लगाती हैं, नागरिक स्वतन्त्रताओं को दुष्प्रभावित करती हैं, लेकिन हमें यह भी ध्यान में रखना होगा कि भारतीय गणराज्य का जन्म साम्प्रदायिक हिंसा, हत्या तथा लूट-पाट के वातावरण में हुआ है। लोकतन्त्र की सफलता के लिए प्राथमिक अनिवार्यता राष्ट्र व गणराज्य की राष्ट्र-विरोधी असामाजिक तत्त्वों से सुरक्षा है।

3. शोषण के विरुद्ध अधिकार, अनुच्छेद 23 से 24 तक (RightAgainst Exploitation,Articles 23 to 24)-शोषण के विरुद्ध अधिकार का उद्देश्य है-समाज के निर्बल वर्गों को शक्तिशाली वर्ग के अन्याय से बचाना। इस मौलिक अधिकार के अन्तर्गत निम्नलिखित व्यवस्थाएँ हैं

(1) मनुष्यों के क्रय-विक्रय और उनके शोषण पर प्रतिबन्ध (Prohibition of Sale and Purchase of Human beings and their Exploitation): हजारों वर्ष गुलाम रहने के बाद भारतीय समाज में बहुत-सी कुरीतियाँ उत्पन्न हो गई थीं जिनमें से एक थी-स्त्रियों व बच्चों का क्रय-विक्रय। मनुष्यों का पशुओं के समान क्रय-विक्रय किया जाता था और उन्हें दास बनाकर मनमाने तरीके से उनका प्रयोग किया जाता था। भारतीय संविधान के निर्माताओं ने अनुच्छेद 23 के अनुसार, मानव के इस शोषण के विरुद्ध प्रतिबन्ध लगाया है और इस प्रकार अब भारत में स्त्रियों, पुरुषों के क्रय-विक्रय पर प्रतिबन्ध लगा दिया गया है। इस व्यवस्था का उल्लंघन करना एक दण्डनीय अपराध घोषित कर दिया गया है।

(2) बेगार लेने पर प्रतिबन्ध (Prohibition on Forced Labour)-भारत के मध्य काल में जमींदार लोग तथा राजा और नवाब अपने अधीनस्थ लोगों से बेगार लेते थे। अपने निजी कार्य उनसे कराकर उनके बदले में उन्हें कुछ नहीं देते थे, परन्तु अब भारतीय संविधान के अनुच्छेद 23 के अनुसार कोई व्यक्ति किसी से बेगार नहीं ले सकता अर्थात् बिना मजदूरी दिए किसी व्यक्ति से कोई काम नहीं लिया जा सकता और न ही किसी व्यक्ति से उसकी इच्छा के विपरीत कोई काम कराया जा सकता है। अब ये दोनों ही बातें एक दण्डनीय अपराध घोषित हो चुकी हैं।

अपवाद (Exceptions)-संविधान के अनुच्छेद 23 में दिए गए अधिकारों पर एक प्रतिबन्ध लगा दिया गया है और वह यह है कि सरकार को जनता के हितों के लिए अपने नागरिकों से आवश्यक सेवा करवाने का अधिकार है। उदाहरणस्वरूप, सरकार नागरिकों को अनिवार्य सैनिक-सेवा तथा अनिवार्य सामाजिक सेवा करने के लिए कानून बना सकती है, परन्तु ऐसा करते हुए सरकार धर्म, वंश, जाति, वर्ग अथवा इनमें से किसी के आधार पर किसी भी प्रकार का भेदभाव नहीं कर सकती।

(3) कारखानों आदि में छोटी आयु के बच्चों को काम करने की मनाही (Prohibition of Employment of Children in Factories etc.)-कारखानों व खानों के मालिक छोटी आयु के बच्चों को काम पर लगाना अति लाभदायक समझते थे क्योंकि उन्हें कम मजदूरी देनी पड़ती थी, परन्तु अब भारतीय संविधान के अनुच्छेद 24 में स्पष्ट उल्लेख कर दिया गया है कि 14 वर्ष से कम आयु के बच्चों को कारखानों या खानों में कार्य करने के लिए नहीं लगाया जा सकता। ऐसा करना अब एक दण्डनीय अपराध है। यह व्यवस्था इसलिए की गई है, ताकि बच्चों के स्वास्थ्य पर विपरीत प्रभाव न पड़े।

4. साँस्कृतिक एवं शैक्षणिक अधिकार, अनुच्छेद 29 से 30 तक (Cultural and Educational Rights, Articles 29 to 30)
(1)अनुच्छेद 29 तथा 30 के अन्तर्गत नागरिकों को, विशेषतया अल्पसंख्यकों को, सांस्कृतिक तथा शैक्षणिक अधिकार प्रदान किए गए हैं। ये अधिकार निम्नलिखित हैं अनुच्छेद 29 के अनुसार, भारत के किसी भी क्षेत्र में रहने वाले नागरिकों के किसी भी वर्ग या उसके किसी भाग को, जिसकी अपनी भाषा, लिपि अथवा संस्कृति हो, यह अधिकार है कि वह अपने संस्कृति व शिक्षा सम्बन्धी अधिकारों की रक्षा करे। अनुच्छेद 29 के अनुसार, केवल अल्पसंख्यकों को ही अपनी भाषा, संस्कृति इत्यादि को सुरक्षित रखने का अधिकार नहीं है, बल्कि यह अधिकार नागरिकों के प्रत्येक वर्ग को प्राप्त है।

(2) किसी भी नागरिक को राज्य द्वारा या उसकी सहायता से चलाई जाने वाली शिक्षा संस्था में प्रवेश देने से धर्म, जाति, वंश, भाषा या इनमें से किसी भी आधार पर इन्कार नहीं किया जा सकता। 1951 में मद्रास (चेन्नई) सरकार ने एक मैडिकल कॉलेज में सीटों का विभाजन भिन्न-भिन्न जातियों के आधार पर कर दिया था जिसके कारण चम्पाकम नामक एक ब्राह्मण लड़की को उस कॉलेज में दाखिला न मिल सका, क्योंकि उस जाति को दिए गए

सभी स्थान पूर्ण हो गए थे। चम्पाकम ने अपने अधिकार की प्राप्ति के लिए न्यायालय में रिट (Writ) की। उच्च न्यायालय ने सरकार के आदेश को असंवैधानिक घोषित कर दिया।

(3) अनुच्छेद 30 के अनुसार, सभी अल्पसंख्यकों को, चाहे वे धर्म पर आधारित हों या भाषा पर, यह अधिकार प्राप्त है कि वे अपनी इच्छानुसार शिक्षा संस्थाओं की स्थापना करें तथा उनका प्रबन्ध करें।

(4) अनुच्छेद 30 के अनुसार, राज्य द्वारा शिक्षा संस्थाओं को सहायता देते समय शिक्षा संस्था के प्रति इस आधार पर भेदभाव नहीं होगा कि वह अल्पसंख्यकों के प्रबन्ध के अधीन है, चाहे वे अल्पसंख्यक भाषा के आधार पर हों या धर्म के आधार पर। 44वें संशोधन द्वारा अनुच्छेद 30 में संशोधन करके यह व्यवस्था की गई है कि राज्य अल्पसंख्यकों द्वारा स्थापित की गई व चलाई जा रही शिक्षा संस्थाओं की सम्पत्ति को अनिवार्य रूप से लेने के लिए कानून का निर्माण करते समय इस बात का ध्यान रखेगा कि कानून के अन्तर्गत निर्धारित की गई रकम से अल्पसंख्यकों के अधिकार पर कोई प्रभाव नहीं पड़ेगा। इस प्रकार अनुच्छेद 29 तथा 30 द्वारा अल्पसंख्यकों के हितों तथा शिक्षा सम्बन्धी अधिकारों को सुरक्षा प्रदान की गई है। भारत में इस अधिकार का बहुत महत्त्व है, क्योंकि भारत में विभिन्न जातियों, धर्मों तथा भाषाओं वाले लोग रहते हैं।

HBSE 11th Class Political Science Important Questions Chapter 2 भारतीय संविधान में अधिकार

प्रश्न 3.
धार्मिक स्वतन्त्रता पर एक नोट लिखिए। अथवा भारतीय संविधान में दिए गए धार्मिक स्वतन्त्रता के अधिकार की व्याख्या करें।
उत्तर:
भारतीय संविधान की प्रस्तावना में भारत को एक धर्म-निरपेक्ष राज्य घोषित किया गया है। इसके अनुसार प्रत्येक भारतीय को धार्मिक स्वतन्त्रता प्रदान की गई है। इस उद्देश्य की पूर्ति के लिए मौलिक अधिकारों में अनुच्छेद 25 से अनुच्छेद 28 तक धार्मिक स्वतन्त्रता के अधिकार का वर्णन किया गया है, जिसकी व्याख्या निम्नलिखित है

1. अन्तःकरण की स्वतन्त्रता तथा किसी भी धर्म को मानने व उसका प्रचार करने की स्वतन्त्रता (Freedom of Conscience and Freedom to Profess and Propagate any Religion)-अनुच्छेद 25 के द्वारा सभी को अन्तःकरण की स्वतन्त्रता है। इसका अभिप्रायः है कि प्रत्येक व्यक्ति जैसी चाहे पूजा-पद्धति को अपना सकता है।

प्रत्येक व्यक्ति को किसी भी धर्म को मानने की स्वतन्त्रता है। राज्य किसी धर्म विशेष को मानने के लिए किसी भी व्यक्ति को बाध्य नहीं कर सकता। इसी प्रकार प्रत्येक व्यक्ति किसी भी धर्म का प्रचार कर सकता है, परन्तु उच्चतम न्यायालय के एक निर्णय के अनुसार, “छल, कपट, प्रलोभन या बल-प्रयोग द्वारा किसी भी व्यक्ति का धर्म-परिवर्तन कराना संविधान के विरुद्ध है।”

प्रतिबन्ध (Limitations) अनुच्छेद 25 में दी गई धार्मिक स्वतन्त्रता पर जिस आधार पर प्रतिबन्ध लगाए जा सकते हैं वे इस प्रकार हैं-
(1) यदि धर्म-प्रचार या धर्म-परिवर्तन लोक व्यवस्था, सदाचार और जन स्वास्थ्य के विरुद्ध है तो राज्य द्वारा उसमें हस्तक्षेप किया जा सकता है,

(2) अनुच्छेद 25 में दी गई धार्मिक स्वतन्त्रता के अन्तर्गत किसी भी धार्मिक स्थल का प्रयोग राष्ट्र विरोधी गतिविधियों के लिए नहीं किया जा सकता है,

(3) प्रचार के नाम पर धर्म-परिवर्तन की मनाही है,

(4) समाज सुधार एवं कल्याण के लिए विभिन्न धार्मिक परम्पराओं और अन्धविश्वासों को दूर किया जा सकता है अर्थात् धर्म के नाम पर सामाजिक कुरीतियों और अन्धविश्वासों को बढ़ावा नहीं दिया जा सकता,

(5) हिन्दुओं के धार्मिक स्थलों को सभी वर्गों के लिए खोल दिया गया है।

2. धार्मिक मामलों के प्रबन्ध की स्वतन्त्रता (Freedom to Manage Religious Affairs):
संविधान के अनुच्छेद 26 के द्वारा धार्मिक मामलों के प्रबन्ध की स्वतन्त्रता प्रदान की गई है, जिसके अन्तर्गत निम्नलिखित स्वतन्त्रताएँ हैं

  • धार्मिक व लोकोपकारी संस्थाएँ चलाना,
  • अपने धार्मिक मामलों का स्वयं प्रबन्ध करना,
  • चल व अचल सम्पत्ति प्राप्त करना,
  • कानून के अनुसार उपर्युक्त सम्पत्ति का प्रबन्ध करना।

धार्मिक मामलों के प्रबन्ध की स्वतन्त्रता का यह अर्थ बिल्कुल नहीं है कि धार्मिक संस्थाओं का दुरुपयोग राष्ट्र-विरोधी गतिविधियों के लिए किया जा सके। उदाहरणतः जून 1984 में ऑप्रेशन ब्लू स्टार के द्वारा स्वर्ण मन्दिर अमृतसर में सेना को इसलिए प्रवेश करना पड़ा था क्योंकि उसमें राष्ट्र-विरोधी असामाजिक तत्त्वों का जमाव हो चुका था।

3. किसी धर्म विशेष को बढ़ाने के लिए कर की अदायगी से छूट (Freedom from Payment of Taxes for Promotion of any Particular Religion) संविधान के अनुच्छेद 27 के अनुसार किसी व्यक्ति को कोई ऐसा कर देने के लिए बाध्य नहीं किया जा सकता जिसको किसी धर्म को बढ़ावा देने के लिए व्यय किया जाना हो। राज्य कर के रूप में लिए गए धन को किसी धर्म विशेष की उन्नति के लिए प्रयोग नहीं करेगा, परन्तु यदि राज्य बिना किसी भेदभाव के धार्मिक एवं अन्य संस्थाओं को समान रूप से सहायता प्रदान करता है तो उस स्थिति में अनुच्छेद लागू नहीं होगा।

4. राजकीय शिक्षा संस्थाओं में धार्मिक शिक्षा पर प्रतिबन्ध (No Religious Teachings in Educational Institutions maintained by State Funds) अनुच्छेद 28 के अन्तर्गत उन राजकीय शिक्षण संस्थाओं में धार्मिक शिक्षा नहीं दी जा सकती, जिनका सारा खर्च राज्य करता हो, लेकिन यह प्रतिबन्ध उन शिक्षण संस्थाओं पर लागू नहीं होता जिन्हें राज्य की ओर से मान्यता प्राप्त है तथा आर्थिक सहायता भी मिलती है, लेकिन सारा खर्च राज्य न करता हो। किन्तु ऐसी शिक्षा संस्थाओं में भी किसी व्यक्ति को उसकी व उसके अभिभावकों की इच्छा के विरुद्ध धार्मिक शिक्षा प्राप्त करने के लिए बाध्य नहीं किया जा सकता। राज्य द्वारा प्रशासित तथा धर्मस्व व न्यास के अधीन शिक्षण संस्थाओं में धार्मिक शिक्षा देने के बारे में कोई प्रतिबन्ध नहीं है।

इस प्रकार धार्मिक स्वतन्त्रता का अधिकार मौलिक अधिकारों की प्रस्तावना में घोषित उद्देश्यों-धर्म-निरपेक्षता व अंतःकरण की स्वतन्त्रता को निश्चित बनाता है जिसके अन्तर्गत राज्य का कोई सरकारी धर्म नहीं है और न ही राज्य किसी धर्म विशेष को बढ़ावा देने के लिए कार्य कर सकता है। धार्मिक स्वतन्त्रता के अधिकार पर निम्नलिखित तीन सीमाएँ हैं

(1) राज्य शान्ति और व्यवस्था, नैतिकता तथा जन-स्वास्थ्य के आधार पर इस अधिकार पर उचित प्रतिबन्ध लगा सकता है। सती-प्रथा तथा देवदासी प्रथा पर रोक इसी आधार पर लगा दी गई है।

(2) धार्मिक समुदायों की आर्थिक तथा राजनीतिक गतिविधियों पर नियन्त्रण रखने वाले कानून बनाए जा सकते हैं।

(3) हिन्दुओं की धार्मिक संस्थाओं को हिन्दू समाज के सभी वर्गों के लिए खोला जा सकता है। हिन्दू संस्थाओं के अन्तर्गत सिक्ख, जैन तथा बौद्ध समुदायों की संस्थाएँ भी सम्मिलित हैं।

प्रश्न 4.
भारतीय संविधान में लिखित संवैधानिक उपचारों के मौलिक अधिकार का विवेचन कीजिए। अथवा संवैधानिक उपचार के अधिकार का विश्लेषण करें।
उत्तर:
सवैधानिक उपचारों का अधिकार (Right to Constitutional Remedies)-भारतीय संविधान के द्वारा भारत के नागरिकों को जो मौलिक अधिकार प्रदान किए गए हैं, संविधान में ही उन अधिकारों की रक्षा भी की गई है। मौ की रक्षा की इस व्यवस्था के अभाव में सरकार अथवा कोई अन्य नागरिक इन अधिकारों के उपयोग में बाधा पैदा कर सकता था और इस प्रकार ये मौलिक अधिकार नागरिकों के लिए महत्त्वहीन हो जाते।

संवैधानिक उपचारों के अधिकार के अनुसार यदि सरकार या कोई नागरिक मौलिक अधिकारों से किसी को वंचित करता है तो वह व्यक्ति अपने मौलिक अधिकारों की सुरक्षा की में न्यायालय अथवा उच्च न्यायालयों में कर सकता है। संविधान के अनुच्छेद 32 के अनुसार-

  • भारत के उच्चतम न्यायालय को नागरिकों के मौलिक अधिकारों की रक्षा के लिए भिन्न-भिन्न आदेश जारी करने के अधिकार दिए गए हैं,
  • उच्चतम न्यायालय को विभिन्न आदेश या निर्देश जारी करने का अधिकार है,
  • संसद कानून बनाकर किसी भी न्यायालय को मौलिक अधिकारों की सुरक्षा के लिए आदेश (writ) जारी करने की शक्ति दे सकती है,
  • उन परिस्थितियों को छोड़कर, जिनका संविधान में वर्णन किया गया है, संवैधानिक उपचारों के मौलिक अधिकार को स्थगित नहीं किया जा सकता।

इसीलिए डॉ० अम्बेडकर ने संविधान सभा में बोलते हुए इस अधिकार को संविधान की आत्मा कहा था। उन्होंने इस अनुच्छेद के सन्दर्भ में लिखा था, “यदि मुझे कोई पूछे कि संविधान का कौन-सा महत्त्वपूर्ण अनुच्छेद है जिसके बिना संविधान प्रभाव शून्य हो जाएगा तो मैं इस अनुच्छेद के अतिरिक्त किसी और अनुच्छेद की ओर संकेत नहीं कर सकता। यह संविधान की आत्मा है। यह संविधान का हृदय है।” मौलिक अधिकारों के संरक्षण हेतु निम्नलिखित आदेश जारी करने का अधिकार उच्चतम न्यायालय तथा उच्च न्यायालय को प्राप्त है

(1) बन्दी प्रत्यक्षीकरण लेख (Writ of Habeas Corpus):
लैटिन भाषा के इस शब्द का अर्थ है-‘हमें शरीर दो’ (Let us have the body), अर्थात् शरीर हमारे सामने पेश करो। इस आदेश के अन्तर्गत न्यायपालिका को अधिकार है कि वह सरकार को बन्दी बनाए गए किसी भी व्यक्ति को अपने सामने प्रस्तुत करने का आदेश दे सकती है।

ऐसे लेख का प्रार्थना-पत्र बन्दी स्वयं या उसका कोई रिश्तेदार न्यायालय के सामने प्रस्तुत कर सकता है, यदि वह महसूस करे कि उसे गैर-कानूनी ढंग से बन्दी बनाया गया है।

बन्दी जब न्यायालय के सामने प्रस्तुत होता है तो न्यायालय उसके मामले पर विचार करता है और यदि न्यायालय यह समझे कि बन्दी को वास्तव में ही गैर-कानूनी ढंग से बन्दी रखा गया है तो वह उसके मुक्त किए जाने का आदेश जारी कर सकता है। इस प्रकार पुलिस किसी व्यक्ति को मनमाने ढंग से बन्दी नहीं बनाए रख सकती। इस अधिकार को आपातकाल में भी निलम्बित नहीं किया जा सकता। न्यायपालिका ने इस अधिकार का प्रयोग करके नागरिकों को पुलिस के अत्याचार से बचाया है। राजनीतिक कैदियों को भी कई बार न्यायालय ने इसका प्रयोग करके मुक्त किया है। इस प्रकार यह लेख नागरिक की दैहिक स्वतन्त्रता की रक्षा के लिए बड़ा महत्त्वपूर्ण साबित हुआ है।

(2) परमादेश लेख (Writ of Mandamus):
इस आदेश द्वारा न्यायालय किसी व्यक्ति या अधिकारी या संस्था को अपना कर्त्तव्य-पालन करने के आदेश दे सकता है। लैटिन भाषा के इन शब्दों का अर्थ है “हम आदेश देते हैं” (We Command)। यदि कोई व्यक्ति यह महसूस करे कि कोई अधिकारी या संस्था अपने कर्तव्यों का पालन नहीं करती तो वह न्यायालय को ऐसा आदेश जारी करने का प्रार्थना-पत्र दे सकता है।

इस प्रार्थना पर विचार करने के बाद न्यायालय यदि यह अनुभव करे कि वास्तव में ही उस अधिकारी या संस्था द्वारा अपने कर्त्तव्य का पालन नहीं हो रहा है तो न्यायालय उसे आदेश दे सकता है और उस आदेश को मानना उस अधिकारी या संस्था का कर्तव्य है।

(3) प्रतिषेध लेख (Writ of Prohibition):
इस आदेश या लेख का अर्थ है-‘रोकना’ या ‘मनाही करना’ । यदि कोई कर्मचारी या संस्थान कोई ऐसा कार्य कर रहा हो जिसका उसे अधिकार नहीं है और इससे किसी के मौलिक अधिकार का हनन होता हो तो वह व्यक्ति न्यायालय में प्रार्थना-पत्र दे सकता है और यदि न्यायालय यह अनुभव करे कि कर्मचारी या अधिकारी या संस्था अपने अधिकार-क्षेत्र से बाहर जा रहा है या कानून की प्रक्रिया के विरुद्ध जा रहा है तो वह प्रतिषेध लेख जारी करके उसे ऐसा करने से रोक सकता है।

(4) अधिकार पृच्छा लेख (Writ of Quo-Warranto):
इन शब्दों का अर्थ है-“किस अधिकार से” (Under What Authority)। यदि कोई व्यक्ति कोई ऐसा कार्य करने का दावा करता है, जिसे करने का उसे अधिकार नहीं या किसी व्यक्ति ने कानून के विरुद्ध कोई पद-ग्रहण कर लिया हो या किसी के पास पद की योग्यता न हो तो कोई भी नागरिक न्यायालय में प्रार्थना-पत्र देकर उसे ऐसा करने से रोकने की प्रार्थना कर सकता है।

न्यायालय यह आदेश जारी करके उस अधिकारी या कर्मचारी को अपनी स्थिति स्पष्ट करने के लिए कह सकता है और यदि वह गैर-कानूनी ढंग से पद ग्रहण किए हुए है तो उसे पदच्युत भी कर सकता है।

(5) उत्प्रेषण लेख (Writof Certiorari):
इसका अर्थ है-“पूर्णतः सूचित करो।” (Be More Fully Informed)। यह आदेश उच्च न्यायालय द्वारा निम्न न्यायालय को दिया जाता है, जिसके द्वारा उच्च न्यायालय निम्न न्यायालय में चल रहे किसी भी मुकद्दमे का पूर्ण ब्यौरा तथा रिकार्ड अपने पास मॅगवा सकता है और यदि उच्च न्यायालय अनुभव करे कि निम्न न्यायालय ने अपने अधि का उल्लंघन किया है

या कानन की प्रक्रिया का समचित पालन नहीं किया है तो वह उस मकद्दमे को स्वयं भी सन सकता है और उसे कुछ निर्देश सहित निम्न न्यायालय को वापस भेज सकता है। इसके द्वारा नागरिकों को न्यायपालिका के अतिक्रमण से मुक्ति दिलाने की व्यवस्था की गई है।

संवैधानिक उपचारों का अधिकार सभी व्यक्तियों को प्रदान किया गया है। प्रभावित व्यक्ति, अर्थात् जिसके अधिकार का उल्लंघन हुआ हो या जिसके साथ अत्याचार हुआ हो, उसके अतिरिक्त अन्य व्यक्ति तथा संस्थाएँ भी इन उपचारों का प्रयोग कर सकते हैं। न्यायपालिका ने लोगों द्वारा लिखे गए साधारण पत्रों, समाचार-पत्रों में छपी खबरों आदि को भी प्रार्थना-पत्र (Writ Petition) मानकर कार्रवाई की है और लोगों के अधिकारों की रक्षा करने का प्रयास किया है जो कि अत्यन्त सराहनीय है।

निष्कर्ष (Conclusion)-निःसन्देह भारतीय संविधान में दिए गए मौलिक अधिकारों की कई पक्षों से आलोचना की गई है। कुछ विद्वानों का कहना है कि मौलिक अधिकारों के अध्याय में सामाजिक और आर्थिक अधिकार सम्मिलित न करना एक बहुत बड़ी भूल है और इससे मौलिक अधिकार खोखले बनकर रह गए हैं। इन अधिकारों की आलोचना इसलिए भी की जाती है कि सरकार को मौलिक अधिकारों पर प्रतिबन्ध लगाने की शक्तियाँ बहुत दी गई हैं, परन्तु इन सब बातों के बावजूद भी हमें मानना पड़ता है कि मौलिक अधिकार लोकतन्त्र की नींव हैं। अधिकार कभी असीमित नहीं होते।

संसार में कोई भी देश ऐसा नहीं है जहाँ मूल अधिकारों पर बन्धन न लगाए गए हों। मौलिक अधिकारों द्वारा नागरिकों की व्यक्तिगत स्वतन्त्रता की रक्षा की गई है और कार्यपालिका और संसद की स्वेच्छाचारिता पर अंकुश लगा दिया गया है। हम एम०वी० पायली (M.V. Paylee) के इस कथन से ‘सहमत हैं, “सम्पूर्ण दृष्टि में संविधान में अंकित मौलिक अधिकार भारतीय प्रजातन्त्र को दृढ़ तथा जीवित रखने का अधिकार हैं।”

प्रश्न 5.
किन अनुच्छेदों पर मौलिक अधिकारों की आलोचना की गई है? व्याख्या करें। अथवा भारतीय संविधान में दिए गए मौलिक अधिकारों की आलोचना किन-किन आधारों पर की जाती है?
उत्तर:
हमारे संविधान-निर्माताओं ने भारत को प्रभुतासम्पन्न प्रजातन्त्रीय गणराज्य घोषित किया है। प्रजातन्त्रीय व्यवस्थाओं के अनुरूप ही भारतीय संविधान के भाग III में मौलिक अधिकारों का विस्तार से वर्णन किया गया है। इनके अध्ययन से स्पष्ट है कि व्यवस्था में कछ दोष पाए जाते हैं। विद्वानों ने इन दोषों को देखते हए यहाँ तक कह दिया है कि मौलिक अधिकार नामक भाग को ‘मौलिक अधिकार तथा उनकी सीमाएँ’ नाम दे दिया जाए। मौलिक अधिकारों की अग्रलिखित अनुच्छेदों पर आलोचना की गई है

1. आर्थिक अधिकारों का न होना (Omission of Economic Rights):
आलोचकों का कहना है कि यद्यपि भारतीय संविधान में मौलिक अधिकारों का विस्तार से वर्णन किया गया है, परन्तु इसमें कुछ महत्त्वपूर्ण आर्थिक तथा सामाजिक अधिकारों, जैसे कार्य पाने का अधिकार (Right to Work), ‘आराम तथा विश्राम का अधिकार’, सामाजिक सुरक्षा का अधिकार (Right to Social Securities) भारतीयों को प्रदान नहीं किए गए। साम्यवादी देशों; जैसे रूस आदि में इन अधिकारों को प्रमुख स्थान दिया गया है।

2. भारतीयों को केवल वे ही अधिकार प्राप्त हैं, जो संविधान में दिए गए हैं (Only Enumerated Rights are granted to Indians):
भारतीय संविधान में दिए गए मौलिक अधिकारों की इस आधार पर भी आलोचना की गई है कि भारतीय नागरिकों को केवल वही अधिकार दिए गए हैं, जिनका कि संविधान में उल्लेख किया गया है।

इसके अलावा उन्हें कोई अधिकार प्राप्त नहीं हैं, परन्तु अमेरिका के नागरिकों को संविधान में वर्णित अधिकारों के अलावा वे अधिकार भी प्राप्त हैं, जो साधारण कानून (Common Law) तथा प्राकृतिक न्याय (Natural Justice) पर आधारित हैं। भारत में ऐसी व्यवस्था नहीं है।

3. मौलिक अधिकारों पर प्रतिबन्धों का होना (Limitations on Fundamental Rights):
मौलिक अधिकारों की इस आधार पर भी आलोचना की गई है कि संविधान ने एक हाथ से मौलिक अधिकार देकर उन पर प्रतिबन्धों तथा अपवादों का घेरा लगाकर दूसरे हाथ से उन्हें वापस ले लिया है। इस व्यवस्था से मौलिक अधिकारों की वास्तविकता ही समाप्त हो जाती है।

परन्तु यह आलोचना पूरी तरह से ठीक नहीं है। कोई भी मौलिक अधिकार असीमित नहीं हो सकता तथा देश की बदलती हई राजनीतिक, सामाजिक, आर्थिक परिस्थितियों के अनुसार उनमें संशोधन भी करना पड़ता है। इस व्यवस्था के लिए आवश्यक है कि मौलिक अधिकारों के बारे में अन्तिम सत्ता संसद के हाथों में होनी चाहिए। यही व्यवस्था भारतीय संविधान में अपनाई गई है।

4. निवारक नजरबन्दी व्यवस्था (Preventive Detention Provision):
अनुच्छेद 22 के अन्तर्गत व्यक्तिगत स्वतन्त्रता की व्यवस्था की गई है, इसके साथ ही संविधान में निवारक नजरबन्दी की भी व्यवस्था की गई है। जिन व्यक्तियों को निवारक नज़रबन्दी कानून के अन्तर्गत गिरफ्तार किया गया हो, उनको अनुच्छेद 22 में दिए गए अधिकार प्राप्त नहीं होते। निवारक नज़रबन्दी के अधीन सरकार कानून बनाकर किसी भी व्यक्ति को बिना मुकद्दमा चलाए अनिश्चित काल के लिए जेल में बन्द कर सकती है तथा उसकी स्वतन्त्रता का हनन कर सकती है।

5. मौलिक अधिकारों पर संसद का नियन्त्रण (Control of Parliament Over Fundamental Rights):
भारतीय संविधान के अनुसार, कानून द्वारा निर्धारित ढंग (Procedure Established by Law) की व्यवस्था की गई है। इसके अनुसार मौलिक अधिकारों के बारे में अन्तिम निर्णय संसद के हाथों में है।

संसद जो भी कानून बनाती है, यदि वह संविधान के अनुकूल है, तो उच्चतम न्यायालय उसे वैध मानेगा। इस व्यवस्था की बजाय ‘उचित कानूनी प्रक्रिया’ (Due Process of Law) को अपनाया जाना चाहिए था, जिससे कि मौलिक अधिकारों के बारे में अन्तिम सत्ता उच्चतम न्यायालय के हाथों में होती, परन्तु भारत में कानून द्वारा निर्धारित ढंग अपनाने का मुख्य कारण मुकद्दमेबाजी को कम करना था।

6. न्यायपालिका के निर्णय संसद के कानूनों द्वारा निरस्त (Decisions of Judiciary struck by Parliament):
उच्चतम न्यायालय ने अनेक बार संसद के कानूनों को इस आधार पर निरस्त किया है कि वे कानून नागरिकों के मौलिक अधिकारों का उल्लंघन करते हैं, परन्तु संसद ने संविधान में संशोधन करके न्यायपालिका द्वारा निरस्त घोषित किए गए कानूनों को वैध तथा सवैधानिक घोषित कर दिया।

7. कठिन भाषा (Difficult Language):
सर आइवर जेनिंग्स (Sir Ivor Jennings) के अनुसार, भारतीय संविधान में मौलिक अधिकारों का उल्लेख बहुत कठिन भाषा में किया गया है। इसको साधारण व्यक्ति समझ नहीं सकता। उनका कहना है कि भारतीय मौलिक अधिकारों की भाषा अमेरिका के संविधान की तरह सरल तथा स्पष्ट होनी चाहिए थी।

8. न्याय का महँगा होना (Costly Judicial Remedies):
भारत में न्याय-व्यवस्था वैसे ही महँगी है। इधर मौलिक अधिकारों की रक्षा के लिए व्यक्ति या तो उच्च न्यायालय में या उच्चतम न्यायालय में प्रार्थना-पत्र दे। इसका अर्थ है-बहुत अधिक धन खर्च करना, जो कि साधारण नागरिक के लिए असहनीय है। न्याय-व्यवस्था सरल तथा सस्ती होनी चाहिए। उपर्युक्त मौलिक अधिकारों की आलोचना तथा उसके उत्तर से स्पष्ट है कि जो प्रतिबन्ध इन अधिकारों पर लगाए गए हैं, वे उचित हैं।

प्रश्न 6.
सम्पत्ति के अधिकार पर संक्षिप्त नोट लिखिए। अथवा भारतीय संविधान के अनुसार राज्य किन शर्तों पर व्यक्तिगत सम्पत्ति को ग्रहण कर सकता है?
उत्तर:
भारतीय संविधान के अनुच्छेद 31 के अनुसार किसी व्यक्ति को कानून के प्राधिकार (Authority of Law) के बिना उसकी सम्पत्ति से वंचित नहीं किया जा सकता। इस अनुच्छेद के अनुसार ही सार्वजनिक हित के अतिरिक्त किसी सम्पत्ति पर अधिकार नहीं किया जा सकता और ऐसे कानून द्वारा उस सम्पत्ति के प्रति मुआवज़ा देने की व्यवस्था होनी चाहिए अथवा उन सिद्धान्तों का वर्णन होना चाहिए, जिनके आधार पर मुआवजा दिया जाता हो।

प्रथम संशोधन (1951) द्वारा अनुच्छेद 31-A तथा 31-B को अनुच्छेद 31 में जोड़ा गया। अनुच्छेद 31-A द्वारा यह निर्धारित किया गया कि यदि किसी राज्य का कोई कानून (पिछला या भविष्य में) किसी भी सम्पत्ति या जमींदारी प्रथा के मौलिक अथवा मध्यस्थ अधिकार पर प्रभाव डाले या कुछ समय के लिए किसी के अधिकारों को नियन्त्रित करे या उन अधिकारों को समाप्त करे या सार्वजनिक हित में उस सम्पत्ति का उचित प्रबन्ध करने के लिए, कुछ काल के लिए किसी की सम्पत्ति पर कब्जा करे तो ऐसी किसी भी अवस्था में न्यायालय उस राज्य के अधिकार को केवल इस आधार पर अवैध घोषित नहीं करेंगे कि यह अधिकार या कब्जा संविधान के उन अधिकारों के विरुद्ध है जो अनुच्छेद 14, 19 तथा 31 में प्रदान किए गए हैं।

अनुच्छेद 31-B ने संविधान के साथ 9वीं अनुसूची जोड़ी जिसमें जमींदारी प्रथा समाप्त करने सम्बन्धी 13 जमींदारी उन्मूलन कानून दर्ज किए, जिन्हें 31-A उपबन्ध के अभाव में अनुच्छेद 31 के अन्तर्गत न्यायालयों में चुनौती दी जा सकती थी। इस सूची के लिए व्यवस्था हुई कि ये कानून इस आधार पर अवैध घोषित नहीं किए जाएँगे कि इनका कोई उपबन्ध संविधान के भाग 3 में दिए गए मूल अधिकारों में से किसी के उलट है। अनुच्छेद 31 (B) द्वारा विधानमण्डल (Competent Legislature) की 9वीं सूची में दर्ज किसी भी अधिनियम को समाप्त अथवा संशोधित करने की शक्ति भी मिली।

25वें संशोधन द्वारा ‘मुआवज़ा’ (Compensation) शब्द के स्थान पर ‘रांशि’ (Amount) शब्द का प्रयोग किया गया। ऐसा इसलिए किया गया, ताकि सार्वजनिक हित के लिए प्राप्त सम्पत्ति के बदले दी जाने वाली राशि को इस न्यायालय में चुनौती न दी जा सके कि राशि अपर्याप्त है। 25वें संशोधन द्वारा अनुच्छेद 31-C को जोड़ा गया।

44वें संशेधन द्वारा अनुच्छेद 31 को संविधान में से निकाल दिया गया है परन्तु 31-A, 31-B, 31-C को वही रहने दिया गया है। अतः इस संशोधन द्वारा सम्पत्ति के अधिकार को मौलिक अधिकारों के अध्याय में से निकाल दिया गया है और सम्पत्ति का अधिकार केवल कानूनी अधिकार बन गया है। परन्तु इस बात का ध्यान रखा गया है कि सम्पत्ति के अधिकार को मौलिक अधिकारों से निकालने का प्रभाव अल्पसंख्यकों की संस्थाओं की स्थापना तथा उनके संचालन के अधिकार पर नहीं पड़ना चाहिए। 44वें संशोधन द्वारा संविधान में एक अनुच्छेद 300-A शामिल किया गया है जो यह घोषणा करता है कि कानून के आदेश के बिना किसी को भी उसकी सम्पत्ति से वंचित नहीं किया जाएगा।

प्रश्न 7.
भारतीय संविधान में दिए गए मौलिक अधिकारों का महत्त्व बताइए।
अथवा
नागरिकों की उन्नति और विकास के लिए मौलिक अधिकार क्यों आवश्यक हैं? व्याख्या कीजिए।
उत्तर:
मौलिक अधिकारों का महत्त्व (Importance of Fundamental Rights)-व्यक्ति की उन्नति और विकास के लिए मौलिक अधिकारों का बहुत महत्त्व है। यदि व्यक्ति को मौलिक अधिकार प्रदान न किए जाएँ, तो उसके जीवन, सम्पत्ति और स्वतन्त्रता की रक्षा का कोई उपाय न रहे। मौलिक अधिकार सरकार तथा विधानमण्डल को तानाशाह बनने से रोकते हैं. और व्यक्ति को आत्म-विकास का अवसर प्रदान करते हैं। मौलिक अधिकारों का व्यक्तिगत स्वतन्त्रता तथा अल्पसंख्यकों के हितों की रक्षा के सम्बन्ध में विशेष महत्त्व है। मौलिक अधिकार वास्तव में लोकतन्त्र की आधारशिला हैं। मौलिक अधिकारों के महत्त्व का वर्णन निम्नलिखित है

1. मौलिक अधिकारों द्वारा सामाजिक समानता की स्थापना होती है (Social Equality is Established by the Fundamental Rights):
मौलिक अधिकारों द्वारा सामाजिक समानता की स्थापना होती है। मौलिक अधिकार देश के सभी नागरिकों को बिना किसी भेदभाव के प्रदान किए गए हैं। धर्म, जाति, भाषा, रंग-लिंग आदि के आधार पर सबको सामाजिक समानता प्राप्त हो तथा जीवन और सम्पत्ति की सुरक्षा हो।

मौलिक अधिकार विभिन्न प्रकार की स्वतन्त्रता प्रदान करते हैं तथा जीवन और सम्पत्ति की सुरक्षा की व्यवस्था करते हैं। समानता, स्वतन्त्रता तथा भ्रातृत्व लोकतन्त्र की नींव हैं। भारतीय संविधान द्वारा ये तीनों प्रकार के मौलिक अधिकार नागरिकों को प्रदान किए गए हैं। अतः इस तरह से भारत में मौलिक अधिकार यहाँ के लोकतन्त्र की आधारशिला हैं।

2. मौलिक अधिकार कानून का शासन स्थापित करते हैं (Fundamental Rights Establish Rule of Law):
भारत में मौलिक अधिकार कानून के शासन की स्थापना करते हैं। मौलिक अधिकारों में यह स्पष्ट कर दिया गया है कि सभी व्यक्ति कानून के समक्ष समान हैं और सभी को कानून का समान संरक्षण प्राप्त है।

जो व्यक्ति कानून को तोड़ता है, उसे कानून के अनुसार दण्ड दिया जाता है। कानून जाति, धर्म, रंग, लिंग आदि के आधार पर कोई मतभेद नहीं करता। किसी भी व्यक्ति को कानून द्वारा स्थापित प्रक्रिया के बिना जीवन और निजी स्वतन्त्रता से वंचित नहीं किया जा सकता। शासन कानून के अनुसार चलाया जाता है, न कि किसी व्यक्ति की इच्छानुसार।

3. मौलिक अधिकार सरकार की निरंकुशता को रोकते हैं (Fundamental Rights check the Despotism of the Government):
मौलिक अधिकारों का महत्त्व इस बात में भी निहित है कि ये अधिकार एक ओर तो कार्यकारिणी तथा व्यवस्थापिका को उनके निश्चित अधिकार क्षेत्रों में रहने का निर्देश देते हैं और इस प्रकार अधिकारों को उनके अनुचित हस्तक्षेप से सुरक्षित रखते हैं।

दूसरी ओर ये अधिकार नागरिकों को सरकार के निरंकुश शासन के विरुद्ध जनमत को संगठित करने का अवसर प्रदान करते हैं। श्री ए०एन० पालकीवाला के अनुसार, “मौलिक अधिकार राज्य के निरंकुश स्वरूप से साधारण नागरिकों की रक्षा करने वाले कवच होते हैं।” हमारे देश में केन्द्रीय सरकार व राज्य सरकारें शासन चलाने के लिए अपनी इच्छानुसार कानून नहीं बना सकतीं, बल्कि उन्हें संविधान के अनुसार कार्य करना पड़ता है।

4. मौलिक अधिकार व्यक्तिगत हितों तथा सामाजिक हितों में उचित सामञ्जस्य स्थापित करते हैं (Co-ordinate Individual and Social Interests) मौलिक अधिकारों द्वारा व्यक्तिगत हितों तथा सामाजिक हितों में उचित सामंजस्य स्थापित करने के लिए काफी सीमा तक सफल प्रयास किया गया है।

5. मौलिक अधिकार कानून का शासन स्थापित करते हैं (Fundamental Rights Establish Rule of Law):
मौलिक अधिकारों की एक महत्त्वपूर्ण विशेषता यह है कि ये कानून के शासन की स्थापना करते हैं, जिससे सबको समान न्याय प्राप्त होता है।

6. मौलिक अधिकार अल्पसंख्यकों के हितों की रक्षा करते हैं (Fundamental Rights Protect the interests of Minorities):
अल्पसंख्यकों को अपनी भाषा, लिपि तथा संस्कृति को सुरक्षित रखने का अधिकार दिया गया है। अल्पसंख्यक अपनी पसन्द की शिक्षा संस्थाओं की स्थापना कर सकते हैं और उनका संचालन करने का अधिकार भी उनको प्राप्त है।

सरकार अल्पसंख्यकों के साथ किसी प्रकार का भेदभाव नहीं करेगी। निष्कर्ष (Conclusion) यह कहना कि मौलिक अधिकारों का कोई महत्त्व नहीं है, एक बड़ी मूर्खता के अतिरिक्त और कुछ नहीं है। सन् 1950 में लीनर नामक पत्रिका ने मौलिक अधिकारों के विषय में कहा था, “व्यक्तिगत अधिकारों पर लेख जनता को कार्यपालिका की स्वेच्छाचारिता की सम्भावना के विरुद्ध आश्वासन देता है। इन मौलिक अधिकारों जिसकी कोई भी बुद्धिमान राजनीतिज्ञ उपेक्षा नहीं कर सकता।” मौलिक अधिकारों का व्यक्तिगत स्वतन्त्रता तथा अल्पसंख्यकों के हितों की रक्षा के सम्बन्ध में विशेष महत्त्व है। मौलिक अधिकार वास्तव में लोकतन्त्र की आधारशिला हैं।

HBSE 11th Class Political Science Important Questions Chapter 2 भारतीय संविधान में अधिकार

प्रश्न 8.
भारतीय संविधान में लिखित मौलिक कर्तव्यों पर एक संक्षिप्त टिप्पणी लिखिए।
अथवा
मौलिक कर्तव्यों से आप क्या समझते हैं? भारतीय संविधान में 42वें संशोधन द्वारा सम्मिलित किए गए मौलिक कर्तव्य कौन-कौन से हैं? इनकी उपयोगिता पर प्रकाश डालिए। आप किन अनुच्छेदों पर इनकी आलोचना कर सकते हैं?
उत्तर:
मौलिक रूप में भारतीय संविधान में अधिकारों सम्बन्धी अनुच्छेद 14 से 32 तक सात प्रकार के मौलिक अधिकार अंकित किए गए थे, परन्तु इन अधिकारों के साथ भारतीय नागरिकों के किसी भी प्रकार के कर्तव्य निश्चित नहीं किए गए थे। काँग्रेस दल के प्रधान श्री डी०के० बरुआ द्वारा सवैधानिक परिवर्तनों के सम्बन्ध में विचार करने के लिए फरवरी, 1976 में एक 9 सदस्यीय समिति नियुक्त की गई थी। इस समिति के अध्यक्ष भारत के भूतपूर्व रक्षा मन्त्री सरदार स्वर्ण सिंह थे।

काफी तर्को के बाद इस समिति ने अपनी रिपोर्ट मई, 1976 में पेश की। अपनी रिपोर्ट में स्वर्ण समिति ने यह सिफारिश की कि भारतीय संविधान में मौलिक अधिकारों के साथ-साथ मौलिक कर्तव्यों का भी एक अध्याय शामिल किया जाए। इस समिति की रिपोर्ट के आधार पर ही संविधान के 42वें संशोधन द्वारा भारतीय संविधान में एक नया भाग 4-A मौलिक कर्तव्य (Part IV-A, Fundamental Duties) अंकित किया गया है। इस नए भाग में भारतीय नागरिकों के दस प्रकार के मौलिक कर्त्तव्य अंकित किए गए हैं। इन दस प्रकार के कर्तव्यों का वर्णन निम्नलिखित है

1. संविधान का पालन तथा राष्ट्र-थ्वज व राष्ट्र-गान का आदर करना (To abide by the Constitution and Respect its Ideals and Institutions like National Flag and National Anthem):
भारत के प्रत्येक नागरिक का यह कर्तव्य है कि वह संविधान में निहित आदर्शों का पालन करे और देश की सर्वोच्च संस्थाओं, राष्ट्र-ध्वज तथा राष्ट्र-गान का आदर करे। संविधान देश का सर्वोच्च कानून है और इसका पालन करना सरकार का ही नहीं, नागरिकों का भी कर्तव्य है। इसी प्रकार राष्ट्र-गान तथा राष्ट्र-ध्वज का आदर करना भी प्रत्येक नागरिक का कर्तव्य है।

2. भारतीय प्रभुसत्ता, एकता व अखण्डता का समर्थन तथा रक्षा करना (To Uphold and protect the Sovereignty unity and Integrity of India): प्रत्येक नागरिक के लिए यह कर्त्तव्य बड़ा महत्त्वपूर्ण है क्योंकि यदि हम इस पर नहीं चलेंगे तो कड़े संघर्ष और बलिदान के बाद प्राप्त हुई स्वतन्त्रता और प्रभुसत्ता खतरे में पड़ सकती है। राष्ट्रीय एकता, देश की अखण्डता और राज्य की प्रभुसत्ता की रक्षा करना प्रत्येक नागरिक का कर्तव्य है।

3. देश की रक्षा करना तथा आवश्यकता पड़ने पर राष्ट्रीय सेवाओं में भाग लेना (To defend the Country and render National Service when called upon to do so):
जिस देश के नागरिक देश की रक्षा के लिए सदैव तैयार रहेंगे, वह देश कभी गुलाम नहीं बन सकता। सबका कर्त्तव्य है कि देश की रक्षा करें और समय आने पर अनिवार्य सेवा के लिए सबको तैयार रहना चाहिए।

4. भारत में सब नागरिकों में भ्रातृत्व की भावना विकसित करना (To promote Spirit of Brotherhood amongest all citizens):
राष्ट्रीय एकता को बनाए रखने के लिए यह लिखा गया है, “प्रत्येक नागरिक का कर्तव्य है कि वह धार्मिक, भाषायी तथा क्षेत्रीय या वर्गीय भिन्नताओं से ऊपर उठकर भारत के सब लोगों में समानता तथा भ्रातृत्व की भावना विकसित करे।” नारियों की स्थिति में सुधार लाने के लिए मौलिक कर्तव्यों के अध्याय में अंकित किया गया है कि प्रत्येक नागरिक का यह कर्त्तव्य है कि वह उन प्रथाओं का त्याग करे जिनसे नारियों का अनादर होता है।

5. स्वतन्त्रता के लिए किए गए राष्ट्रीय आन्दोलन को प्रेरित करने वाले उच्च आदर्शों को मानना तथा उनका प्रसार करना (To accept and follow the Noble Ideals which Inspired our National Struggle for the freedom):
प्रत्येक नागरिक का परम कर्त्तव्य है कि जिन आदर्शों, जैसे स्वतन्त्रता, धर्म-निरपेक्षता, लोकतन्त्र, अहिंसा, राष्ट्रीय एकता, विश्व-बन्धुत्व आदि, के लिए स्वतन्त्रता संग्राम लड़ा गया और जिनके लिए शहीदों ने अपना बलिदान किया, उन्हें अपनाएँ और उन पर चलते हुए राष्ट्र का विकास करें।

6. लोगों में वैज्ञानिक दृष्टिकोण फैलाना (To Develop Scientific Attitude in the People):
आधुनिक युग विज्ञान का युग है, परन्तु भारत की अधिकांश जनता आज भी अन्धविश्वासों के चक्कर में फंसी हुई है। उनमें वैज्ञानिक दृष्टिकोण की कमी है जिस कारण वे अपने व्यक्तित्त्व तथा अपने जीवन का ठीक प्रकार से विकास नहीं कर पाते। इसलिए अब व्यवस्था की है, “प्रत्येक नागरिक का यह कर्त्तव्य है कि वैज्ञानिक स्वभाव, मानववाद तथा जाँच करने और सुधार करने की भावना विकसित करे।”

7. प्राचीन संस्कृति की देन को सुरक्षित रखना (To Preserve the Rich Heritage of Composite Culture):
आज आवश्यकता इस बात की है कि युवकों को भारतीय संस्कृति की महानता के बारे में बताया जाए ताकि युवक अपनी संस्कृति पर गर्व अनुभव कर सकें। इसलिए मौलिक कर्तव्यों के अध्याय में यह अंकित किया गया है, “प्रत्येक भारतीय नागरिक का यह कर्तव्य है कि वह सम्पूर्ण संस्कृति तथा शानदार विरासत का सम्मान करे और उसको स्थिर रखे।”

8. व्यक्तिगत तथा सामूहिक यत्नों के द्वारा उच्च राष्ट्रीय लक्ष्यों की प्राप्ति के यत्न करना (To Strive towards excellence in spheres of Individual and Collective Activities):
कोई भी समाज तथा देश तब तक उन्नति नहीं कर सकता, जब तक कि उसके नागरिकों में प्रत्येक कार्य को करने की लगन तथा श्रेष्ठता प्राप्त करने की इच्छा न हो। अतः प्रत्येक भारतीय नागरिक का कर्तव्य है कि वह व्यक्तिगत तथा सामूहिक गतिविधियों के प्रत्येक क्षेत्र में सर्वश्रेष्ठता प्राप्त करने का यत्न करे, ताकि उसका उच्च स्तरों के प्रति ज्ञान निरन्तर बढ़ता रहे और राष्ट्र उन्नति के पथ पर अग्रसर हो।

9. वनों, झीलों, नदियों तथा जंगली जानवरों की रक्षा करना तथा उनकी उन्नति के लिए प्रयत्न करना (To protect and improve the National Environment including Forests, Lakes, Rivers and Wildlife and to have compossion for living creatures):
प्रत्येक भारतीय नागरिक का कर्तव्य है कि वनों, झीलों, नदियों तथा वन्य-जीवन सहित प्राकृतिक वातावरण की रक्षा और सुधार करें तथा जीव-जन्तुओं के प्रति दया की भावना रखें।

10. हिंसा को रोकना तथा राष्ट्रीय सम्पत्ति की रक्षा करना (To safeguard Public Property and Adjure Vio lence) सार्वजनिक सम्पत्ति देश के धन, शक्ति और सम्पन्नता का स्रोत होती है। इसको हानि पहुँचाना एक प्रकार से अपनी सम्पत्ति को हानि पहुँचाना है। हिंसा से नैतिक और मानवीय मूल्यों का पतन होता है तथा देश की प्रगति में बाधा पड़ती है। इसलिए सभी भारतीय नागरिकों का कर्तव्य है कि वे सार्वजनिक सम्पत्ति की रक्षा करें तथा हिंसा का त्याग करें।

संविधान में मौलिक कर्तव्यों का अंकित किया जाना एक प्रगतिशील कदम है। संविधान में मौलिक कर्त्तव्यों की व्याख्या न होना संविधान की महत्त्वपूर्ण कमी थी, जिसे 42वें संशोधन ने मौलिक कर्तव्यों के अध्याय को शामिल करके दूर किया। कोई देश तब तक उन्नति नहीं कर सकता जब तक उसके नागरिक अपने अधिकार की अपेक्षा अपने कर्तव्यों के प्रति अधिक जागृत न हों।

महात्मा गाँधी अधिकारों की अपेक्षा कर्त्तव्यों पर अधिक जोर देते थे। उनका कहना था कि अधिकार कर्तव्यों का पालन करने से प्राप्त होते हैं। मौलिक कर्तव्यों की उपयोगिता एवं महत्त्व (Utility and Importance of the Fundamental Duties) मौलिक कर्त्तव्यों की उपयोगिता एवं महत्त्व निम्नलिखित है

1. मौलिक कर्त्तव्य व्यक्ति के आदर्श व पथ-प्रदर्शक हैं (The Fundamental Duties are the Ideals and Guidelines for the Individual)-भारत के संविधान में सम्मिलित किए गए मौलिक कर्त्तव्य आदर्शात्मक हैं। इन कर्त्तव्यों का उद्देश्य कोई स्वार्थ न होकर नागरिकों के दिलों में देश-हित की भावना जागृत करना है। इसके साथ ही ये कर्तव्य नागरिकों का पथ-प्रदर्शन करते हैं।

आज समाज के चारों तरफ स्वार्थ और भ्रष्टाचार का वातावरण फैला हुआ है तथा व्यक्ति व समाज के हितों में उचित सामंजस्य नहीं है। इसके अतिरिक्त जहाँ व्यक्ति स्वहित को प्राथमिक और समाज के हितों को गौण मानता है तो ऐसे समाज के लिए ये मौलिक कर्तव्य जनता का मार्गदर्शन करते हैं और उनके व्यवहार के लिए आदर्श उपस्थित करते हैं, ताकि वे निजी स्वार्थ की संकीर्ण भावनाओं से ऊपर उठकर सामूहिक हित के लिए अपने कर्तव्यों का पालन करें।

2. मौलिक कर्त्तव्य नागरिकों में चेतना उत्पन्न करेंगे (The Fundamental Duties will Create Consciousness Among the People):
मौलिक कर्तव्यों को संविधान में सम्मिलित करने से नागरिकों में अपने कर्तव्यों के प्रति चेतना जागृत होगी और लोगों का श्रेष्ठ आचरण सम्भव हो सकेगा। इसके फलस्वरूप व्यक्तिगत उन्नति तथा विकास के साथ-साथ समाज और देश भी प्रगति के पथ पर अग्रसर होंगे।

मौलिक कर्तव्यों को संविधान में सम्मिलित करते समय हमारी स्वर्गीय प्रधानमन्त्री श्रीमती इन्दिरा गाँधी ने कहा था कि अगर लोग मौलिक कर्तव्यों को अपने दिमाग में रख लेंगे तो हम तुरन्त एक शान्तिपूर्ण तथा मैत्रीपूर्ण क्रान्ति देख सकेंगे। अतः इस प्रकार हम देखते हैं कि संविधान में मौलिक कर्त्तव्यों के अंकित किए जाने से ये नागरिकों को सदैव याद दिलाते रहेंगे कि उनके अधिकारों के साथ-साथ कुछ कर्त्तव्य भी हैं।

3. मौलिक अधिकारों की प्राप्ति में सहायक (Helpful in Attaining the Fundamental Rights):
कर्त्तव्यों की तीसरी महत्ता है कि ये भारतीय संविधान में दिए गए मौलिक अधिकारों को प्राप्त करने में सहायक सिद्ध होंगे क्योंकि कई ऐसे अधिकार हैं जो कर्त्तव्यों को निभाने मात्र से ही प्राप्त हो जाएँगे।

4. मौलिक कर्त्तव्य का नैतिक महत्त्व (Moral Importance of Fundamental Duties):
यद्यपि यह ठीक है कि इन कर्तव्यों के पीछे किसी प्रकार की कानूनी शक्ति नहीं है, फिर भी इनकी नैतिक महत्ता है। कर्तव्यों का नैतिक स्वरूप अपना विशेष महत्त्व रखता है।

5. कमी को पूरा करते हैं (Remove Deficiency):
भारतीय संविधान में मूल रूप से मौलिक कर्तव्यों को शामिल नहीं किया गया था, जिसके कारण भारतीय नागरिक केवल अपने अधिकारों के प्रति ही जागरूक थे तथा अपने कर्तव्यों को भूल गए थे। अतः इन मौलिक कर्त्तव्यों को संविधान में 42वें संशोधन द्वारा शामिल करके इस कमी को पूरा कर दिया गया है। मौलिक कर्त्तव्य प्रारम्भिक कमी को पूरा करते हैं।

6. कर्त्तव्य विवाद रहित हैं (Duties are Non-controversial):
भारतीय संविधान में सम्मिलित किए गए मौलिक कर्त्तव्य विवाद रहित हैं। इस पर विभिन्न विद्वानों में कोई मतभेद नहीं है। सभी विद्वानों ने इन कर्त्तव्यों को भारतीय संस्कृति के अनुकूल बताया है। सभी विद्वान इस बात पर सहमत हैं कि इन कर्तव्यों का पालन भारतीय विकास में सहायक होगा। मौलिक कर्तव्यों की आलोचना (Criticism of the Fundamental Duties) मौलिक कर्तव्यों की आलोचना इस प्रकार हैं

1. कुछ मौलिक कर्त्तव्य अस्पष्ट हैं (Some of the Fundamental Duties are not Clearly Defined):
आलोचकों का कहना है कि संविधान में ऐसे शब्दों का प्रयोग करना चाहिए, जिनका अर्थ एकदम स्पष्ट हो। परन्तु ‘कर्तव्यों’ वाले भाग में कुछ ऐसे शब्दों का प्रयोग किया गया है जिनका मनमाना अर्थ लगाया जा सकता है, जैसे मिली-जुली संस्कृति (Composite Culture), वैज्ञानिक दृष्टिकोण (Scientific Temper), अन्वेषण और सुधार की भावना (Spirit of Enquiry and Reform) तथा मानववाद आदि।

कर्तव्यों को लागू करने के लिए कोई दण्डात्मक व्यवस्था नहीं है (There is no Coercive Machinery for the Enforcement of the Duties): स्वर्णसिंह समिति ने यह सुझाव दिया था कि मौलिक कर्तव्यों की अवहेलना करने वालों को दण्ड दिया जाए और उसके लिए संसद उचित कानूनों का निर्माण करे, परन्तु अभी तक ऐसा कुछ नहीं किया गया है। वास्तव में कर्तव्यों के वर्तमान रूप को देखते हुए दण्ड की व्यवस्था की ही नहीं जा सकती। जैसा कि पहले ही स्पष्ट किया जा चुका है, कर्तव्यों के स्वरूप नितान्त अस्पष्ट हैं, अतः नागरिकों को किस आधार पर दण्ड दिया जा सकता है।।

3. केवल उच्च आदर्श (High Ideals Only):
मौलिक कर्तव्यों की आलोचना तीसरे स्थान पर की जा सकती है कि ये केवल मात्र उच्च आदर्श प्रस्तुत करते हैं। भारत की अधिक जनसंख्या गाँवों में निवास करती है जो इन उच्च आदर्शों को समझने में असमर्थ है।

4. संविधान के तीसरे अध्याय में सम्मिलित होने चाहिएँ (Should have been Included in Chapter No. Three):
मौलिक कर्तव्यों को भारतीय संविधान के अध्याय चार में शामिल किया गया है, जबकि इन्हें मौलिक अधिकारों वाले अध्याय तीन में ही रखा जाना चाहिए क्योंकि अधिकारों के साथ ही कर्तव्य अच्छे लगते हैं।

5. महत्त्वपूर्ण कर्त्तव्यों का छोड़ा जाना (Some Important Duties have been Left):
मौलिक कर्तव्यों की आलोचना आलोचकों के द्वारा इस आधार पर की गई है कि कई महत्त्वपूर्ण कर्त्तव्यों को कर्तव्यों की सूची में लिखा नहीं गया है, जैसे अनिवार्य मतदान, अनिवार्य सैनिक सेवा, दूसरों के अधिकारों का सम्मान करना आदि को संविधान में शामिल किया जाना चाहिए था। इन कर्तव्यों को सूची से बाहर रखा जाना विचित्र-सा लगता है।

प्रश्न 9.
राज्य-नीति के निदेशक सिद्धान्तों की विशेषताओं का वर्णन कीजिए। अथवा राज्य-नीति के निदेशक सिद्धान्तों से क्या अभिप्राय है? निदेशक सिद्धान्तों के स्वरूप का विवेचन कीजिए। अथवा राज्य-नीति के निदेशक सिद्धान्तों का अर्थ लिखकर उनके स्वरूप की व्याख्या कीजिए। अथवा राज्य के नीति-निदेशक सिद्धान्तों की प्रकृति का वर्णन करें।
उत्तर:
हमारे संविधान-निर्माताओं ने इस बात को पूरी तरह से ध्यान में रखा और संविधान के चौथे अध्याय (Chapter IV) में कुछ ऐसे सिद्धान्तों का वर्णन किया जो राज्य के पथ-प्रदर्शन का कार्य करते रहें। इन सिद्धान्तों के पीछे कानून की शक्ति नहीं है अर्थात् इन सिद्धान्तों को न्यायालय द्वारा लागू नहीं करवाया जा सकता, परन्तु यह बात हमारे संविधान में स्पष्ट शब्दों में कह दी गई है कि राज्य की नीति के इन निदेशक सिद्धान्तों का शासन-व्यवस्था में मौलिक रूप से पालन किया जाएगा। भले ही ये सिद्धान्त कानूनी रूप में लागू न किए जा सकते हों, परन्तु इन्हें व्यवहार में लागू करने के लिए जनमत (Public Opinion) का हाथ अधिक मात्रा में होगा।

हमारे संविधान में इन सिद्धान्तों को राज्य-नीति के निदेशक सिद्धान्तों (Directive Principles of State Policy) का नाम दिया गया है। इनके नाम से ही यह स्पष्ट हो जाता है कि ये सिद्धान्त राज्य का पथ-प्रदर्शन करने के लिए बनाए गए हैं। भारतीय संविधान के निर्माताओं का कथन था कि देश में राजनीतिक प्रजातन्त्र के साथ आर्थिक तथा सामाजिक प्रजातन्त्र का होना भी आवश्यक है, तभी राष्ट्र उन्नति कर सकता है।

राज्य-नीति निदेशक सिद्धान्त उन साधनों तथा नीतियों को बताते हैं, जिनका पालन करके भविष्य में भारत में एक क-कल्याणकारी राज्य (Welfare State) की स्थापना की जा सके। एम०बी० पायली के शब्दों में, “सामूहिक रूप से सिद्धान्त लोकतन्त्रात्मक भारत का शिलान्यास करते हैं। ये भारतीय जनता के आदर्शों एवं आकांक्षाओं का वह भाग है जिन्हें वह एक सीमित अवधि के भीतर प्राप्त करना चाहती है।”

दूसरे शब्दों में, राज्य-नीति के निदेशक सिद्धान्त वे आदेश-पत्र हैं, जिनको राज्य की व्यवस्थापिका तथा कार्यपालिका को कोई भी कानून या नीति बनाते समय ध्यान में रखना पड़ता है। डॉ० अम्बेडकर (Dr. Ambedkar) ने संविधान सभा में कहा था, “संविधान के इस भाग को अधिनियमित कर भविष्य में सभी व्यवस्थापिका तथा कार्यपालिका को यह निर्देश दिया गया है कि वे किस प्रकार से अपनी शक्तियों का प्रयोग करें।”
निदेशक सिद्धान्तों की प्रमुख विशेषताएँ निम्नलिखित हैं

1. आयरलैण्ड के संविधान से प्रेरित (Inspired from Irish Constitution):
राज्य-नीति के निदेशक सिद्धान्त आयरलैण्ड के संविधान से प्रेरित कहे जा सकते हैं। आयरलैण्ड के संविधान में भी राज्य-नीति के निदेशक सिद्धान्त पाए जाते हैं, परन्तु भारतीय व्यवस्था आयरिश व्यवस्था से कुछ भिन्न; जैसे भारत और आयरलैण्ड में निदेशक तत्त्वों की प्रकृति समान नहीं है तथा भारत में निदेशक तत्त्व राज्य के लिए निर्देश हैं, जबकि आयरलैंड में केवल विधायकों के लिए निर्देश हैं।

2. शासन-व्यवस्था के आधारभूत सिद्धान्त (Fundamental Principles for Governance of the State):
राज्य के निदेशक तत्त्व शासन-व्यवस्था के आधारभूत सिद्धान्त हैं। संविधान की अनुच्छेद 37 में स्पष्ट कहा गया है कि यद्यपि ये सिद्धान्तवाद योग्य नहीं हैं तो भी राज्य के प्रशासन हेतु आधारभूत हैं, जिनके आधार पर राज्य को कानून बनाने चाहिएँ।।

3. समाजवादी व्यवस्था के आधार (Basis of Socialist System):
राज्य-नीति के निदेशक सिद्धान्त समता पर आधारित समाजवादी व्यवस्था की स्थापना करते हैं जिसमें आर्थिक विषमता व असमानता को दूर करने की बात कही गई है। इसलिए आइवर जेनिंग्स (Ivor Jennings) ने नीति-निदेशक सिद्धान्तों के बारे में टिप्पणी करते हुए कहा, “संविधान के ये पन्ने समाजवाद शब्द का प्रयोग किए बिना ही लोकतान्त्रिक समाजवाद की पृष्ठभूमि प्रस्तुत करते हैं।”

4. भारतीय सामाजिक पृष्ठभूमि के अनुरूप (In accordance with Indian Social Background):
राज्य-नीति के निदेशक सिद्धान्त भारतीय सामाजिक पृष्ठभूमि के अनुरूप हैं, जैसे इनमें बहुधर्मी समाज होने के नाते समान नागरिक आचार संहिता. गरीबी के दृष्टिकोण से मुफ्त कानूनी सहायता, अनुसूचित जातियों व जनजातियों के विकास के लिए मुफ्त शिक्षा का प्रबन्ध इत्यादि को लागू करने के लिए कहा गया है।

5. व्यापक क्षेत्र (Wide Scope):
निदेशक सिद्धान्तों का क्षेत्र व्यापक है, जिसमें सामाजिक-आर्थिक न्याय से सम्बन्धित तत्त्वों के साथ अन्तर्राष्ट्रीय, गाँधीवादी पंचायत व्यवस्था, मद्य-निषेध, जंगली जीवों व पर्यावरण की रक्षा सम्बन्धी तत्त्व भी पाए जाते हैं। इस प्रकार निदेशक सिद्धान्त समाज के सभी पक्षों सामाजिक, आर्थिक, राजनीतिक, प्रशासनिक, पर्यावरण तथा इतिहास से सम्बन्धित हैं।

6. सकारात्मक प्रकृति (Positive Nature):
निदेशक सिद्धान्त की प्रकृति सकारात्मक है जो राज्य को निदेशक तत्त्वों के अनुसार कार्य करने का निर्देश देती है, जैसे राज्य गरीबों को मुफ्त कानूनी सहायता प्रदान करे, राज्य ऐसी नीतियाँ बनाए, जिनसे सभी नागरिकों को, स्त्री व पुरुषों को समान रूप से जीवनयापन के उचित साधन उपलब्ध हो सकें, इत्यादि।

7. मौलिक अधिकारों के पूरक (Complementary of Fundamental Rights):
नागरिकों को मौलिक अधिकार यथार्थ में तभी प्राप्त हो सकते हैं, जबकि नीति-निदेशक तत्त्वों को लागू किया जाए। राज्य-नीति के निदेशक तत्त्वों में निहित व्यवस्था के अन्तर्गत ही मौलिक अधिकारों की प्राप्ति सम्भव है। इसलिए न्यायपालिका ने बार-बार अनेक निर्णयों में कहा है कि नीति-निदेशक सिद्धान्त मौलिक अधिकारों के पूरक हैं।

8. आदर्शवादी (Idealistic):
नीति-निदेशक सिद्धान्त यथार्थवादी ही नहीं हैं, बल्कि उनमें आदर्श भी निहित हैं, जैसे विश्व-शान्ति का आदर्श, वन्य जीवों की रक्षा का आदर्श इत्यादि।

9. लोक-कल्याणकारी राज्य के आधार (Basis of Welfare State):
अनुच्छेद 38 में प्रतिपादित नीति-निदेशक सिद्धान्त में स्पष्ट रूप से कहा गया है कि राज्य लोगों के कल्याण हेतु ऐसी सामाजिक व्यवस्था करे, जिसमें लोगों को सामाजिक, आर्थिक व राजनीतिक न्याय मिल सके। अनुच्छेद 39 का सम्बन्ध उन नीतियों से है जो लोक-कल्याणकारी राज्य की प्राप्ति के लिए आवश्यक हैं।

10. न्याय-योग्य नहीं (Non-Justiceable):
अनुच्छेद 37 में कहा गया है कि ये निदेशक सिद्धान्त वाद-योग्य नहीं हैं अर्थात् नागरिक इन तत्त्वों को लागू करवाने के लिए न्यायालय में नहीं जा सकते। न्यायालय सरकार को इन सिद्धान्त को लागू करने लिए निर्देश जारी नहीं कर सकता, लेकिन वाद-योग्य न होने का यह अर्थ बिल्कुल नहीं है कि नागरिक इन सिद्धान्त के आधार पर किसी कानून तथा शासकीय कार्य को चुनौती नहीं दे सकते और न्यायालय इन सिद्धान्त के आधार पर निर्णय नहीं दे सकते।

नागरिक इन सिद्धान्त के आधार पर कानूनों तथा प्रशासकीय कार्यों को न्यायालय में चुनौती दे सकते हैं और न्यायालय इन सिद्धान्त के आधार पर निर्णय दे सकते हैं, केवल इन सिद्धान्त को लागू करने के लिए निर्देश नहीं दे सकते।

11. ये सिद्धान्त किसी विशेष राजनीतिक विचारअनुच्छेद से सम्बन्धित नहीं हैं (These Principles are not connected with any particular theory):
नीति-निदेशक सिद्धान्तों के अध्ययन से स्पष्ट होता है कि ये किसी विशेष विचारअनुच्छेद से बँधे हुए नहीं हैं। ये सिद्धान्त बहुत लचीले हैं और किसी भी विचारअनुच्छेद के माध्यम से इनकी पूर्ति हो सकती है। वास्तव में राजनीतिक लोकतन्त्र के माध्यम से कल्याणकारी राज्य की स्थापना के प्रयास के कारण इन्हें विचारअनुच्छेदों के बन्धन से मुक्त रखा गया है।

प्रश्न 10.
राज्य-नीति के निदेशक सिद्धान्त क्या हैं? हमारे संविधान में दिए गए राज्य-नीति के निदेशक सिद्धान्तों की विवेचना कीजिए। इन्हें कैसे मनवाया जा सकता है?
अथवा
भारतीय संविधान में दिए गए किन्हीं पाँच राज्य-नीति के निदेशक सिद्धान्तों का उल्लेख कीजिए।
उत्तर:
संविधान के अध्याय IV में अनुच्छेद 36 से लेकर 51 तक कुल 16 अनुच्छेदों में निदेशक सिद्धान्तों का वर्णन किया गया है। अध्ययन के लिए इन्हें प्रायः तीन वर्गों में विभाजित किया जाता है-

  • समाजवादी सिद्धान्त (Socialist Principles),
  • गाँधीवादी सिद्धान्त (Gandhian Principles),
  • उदारवादी लोकतन्त्रीय सिद्धान्त (Liberal Democratic Principles)। इन तीन वर्गों में एक और वर्ग भी जोड़ा जा सकता है-
  • अन्तर्राष्ट्रीय सम्बन्धी सिद्धान्त (Principles Relating to International Relations) स्मरण रहे कि संविधान में इनका वर्णन इन विभिन्न रूपों में विभाजित करके नहीं किया गया है

1. समाजवादी सिद्धान्त (Socialist Principles)- इस वर्ग के अधीन ऐसे सिद्धान्त रखे जा सकते हैं जिनका उद्देश्य भारत में समाजवादी कल्याणकारी राज्य की स्थापना करना है। ऐसे सिद्धान्त हैं-

(1) राज्य अपनी नीति का इस प्रकार संचालन करेगा जिससे
(क) सभी व्यक्तियों को समान रूप से जीविका उपार्जन के पर्याप्त साधन प्राप्त हो सकें,
(ख) समान काम के लिए सभी को समान वेतन मिले,
(ग) देश के भौतिक तथा उत्पादन के साधनों का विभाजन इस प्रकार हो कि सार्वजनिक हित का पालन हो,
(घ) धन का उचित वितरण हो तथा केवल कुछ ही व्यक्तियों के हाथों में धन का केन्द्रीयकरण न हो,
(ङ) मजदूरों, स्त्रियों तथा बालकों की परिस्थितियों का दुरुपयोग न हो और वे अपनी आर्थिक आवश्यकताओं से मजबूर होकर कोई ऐसा कार्य न करें जो उनकी शक्ति से बाहर हो तथा जिससे उनका स्वास्थ्य खराब हो,

(2) मजदूरों को काम करने की न्यायपूर्ण तथा मानवीय परिस्थितियाँ प्राप्त हों तथा स्त्रियों को प्रसति सहायता मिले। राज्य हर प्रकार से यह प्रयत्न करे कि राज्य से बेकारी और बीमारी दूर हो। बूढ़ों और दिव्यांगों को सार्वजनिक सहायता दी जाए,

(3) राज्य में अधिक-से-अधिक नागरिकों के लिए शिक्षा प्राप्त करने की व्यवस्था की जाए,

(4) राज्य यह प्रयत्न करेगा कि सभी कर्मचारियों, जो किसी भी उद्योग, कृषि अथवा धन्धे में लगे हों, को काम का उचित वेतन तथा काम की उचित व्यवस्थाएँ उपलब्ध हों, जिनसे वे अपना जीवन-स्तर ऊँचा कर सकें।

2. गाँधीवादी सिद्धान्त (Gandhian Principles)-संविधान-निर्माताओं ने गाँधी जी के विचारों को व्यवहार में लाने के लिए निम्नलिखित सिद्धान्त निदेशक सिद्धान्तों में शामिल किए हैं-
(1) राज्य गाँवों में पंचायतों को संगठित करे तथा उनको इतनी शक्तियाँ दे जिनसे कि वे प्रशासनिक इकाइयों के रूप में सफलतापूर्वक कार्य कर सकें,

(2) राज्य समाज के दुर्बल वर्गों, विशेषकर अनुसूचित जातियों तथा कबीलों की शिक्षा और आर्थिक उन्नति के लिए प्रयत्न करेगा तथा उन्हें सामाजिक अन्याय एवं शोषप

(3) गाँवों में घरेलू दस्तकारियों की उन्नति के लिए प्रयत्न करेगा,

(4) राज्य नशीली वस्तुओं के सेवन को रोकने का प्रयत्न करेगा,

(5) राज्य कृषि तथा पशुपालन उद्योग का संगठन वैज्ञानिक अनुच्छेदों पर करने का प्रयत्न करेगा। दूध देने वाले पशुओं को मारने पर रोक लगाएगा और पशुओं की नस्ल सुधारने का प्रयत्न करेगा।

3. उदारवादी लोकतन्त्रीय सिद्धान्त (Liberal Democratic Principles)

  • राज्य न्यायपालिका को कार्यपालिका से अलग करने के लिए उचित कदम उठाएगा,
  • संविधान के लागू होने के 10 वर्ष के अन्दर-अन्दर राज्य 14 वर्ष के बालकों के लिए निःशुल्क तथा अनिवार्य शिक्षा का प्रबन्ध करेगा,
  • राज्य समस्त भारत में सामान्य व्यवहार नियम (Uniform Civil Code) लागू करने का प्रयत्न करेगा,
  • राज्य लोगों के जीवन-स्तर तथा आहार-स्तर को ऊँचा उठाने तथा उनके स्वास्थ्य में सुधार करने का यत्न करेगा,
  • राज्य ऐतिहासिक अथवा कलात्मक दृष्टि से महत्त्व रखने वाले स्मारकों, स्थानों तथा वस्तुओं की रक्षा करेगा और उनको नष्ट होने अथवा कुरूप होने से बचाएगा।

4. अन्तर्राष्ट्रीय सम्बन्धों से सम्बन्धित सिद्धान्त (Principles Relating to International Relations)- नीति-निदेशक सिद्धान्त राष्ट्रीय नीति के साथ-साथ अन्तर्राष्ट्रीय नीति से भी सम्बन्धित हैं। अनुच्छेद 51 में कहा गया है कि राज्य-

  • अन्तर्राष्ट्रीय शान्ति व सुरक्षा को बढ़ावा देगा,
  • राष्ट्रों के मध्य उचित व सम्मानपूर्वक सम्बन्ध बनाए रखेगा,
  • अन्तर्राष्ट्रीय सन्धि व कानून को सम्मान देगा,
  • अन्तर्राष्ट्रीय विवादों का मध्यस्थता द्वारा निपटारा करने का प्रयास करेगा।

42वें संशोधन द्वारा राज्य नीति के निदेशक सिद्धान्तों में वृद्धि (Accretion in Directive Principles of State Policy through 42nd Amendment)42वें संशोधन द्वारा राज्य-नीति के निदेशक सिद्धान्तों में निम्नलिखित नए सिद्धान्त शामिल किए गए हैं

(1) राज्य अपनी नीति का संचालन इस प्रकार से करेगा जिससे कि बच्चों को स्वस्थ, स्वतन्त्र और प्रतिष्ठापूर्ण वातावरण में अपने विकास के लिए अवसर और सुविधाएँ प्राप्त हों,

(2) राज्य ऐसी कानून प्रणाली के प्रचलन की व्यवस्था करेगा जो समान अवसरों के आधार पर न्याय का विकास करे। राज्य आर्थिक दृष्टि से कमजोर व्यक्तियों के लिए मुफ्त कानूनी सहायता की व्यवस्था करने का प्रयत्न करेगा,

(3) राज्य कानून द्वारा या अन्य ढंग से श्रमिकों को उद्योगों के प्रबन्ध में भागीदार बनाने के लिए पग उठाएगा,

(4) राज्य पर्यावरण की सुरक्षा और विकास करने तथा देश के वन और वन्य जीवन को सुरक्षित रखने का प्रयत्न करेगा।

44वें संशोधन द्वारा राज्य-नीति के निदेशक सिद्धान्तों का विस्तार किया गया है। 44वें संशोधन के अन्तर्गत अनुच्छेद 38 के अन्तर्गत एक और निदेशक सिद्धान्त जोड़ा गया है। 44वें संशोधन के अनुसार राज्य, विशेषकर आय की असमानता को न्यूनतम करने और न केवल व्यक्तियों में, बल्कि विभिन्न क्षेत्रों अथवा व्यवसायों में लगे लोगों के समूहों में स्तर, सुविधाओं और अवसरों की असमानता को दूर करने का प्रयास करेगा।

इस तरह राज्य-नीति के निदेशक सिद्धान्त जीवन के पहलू के साथ सम्बन्धित हैं, क्योंकि ये सिद्धान्त कई विषयों के साथ सम्बन्ध रखते हैं, इसलिए स्वाभाविक ही है कि इनको परस्पर किसी विशेष तर्कशास्त्र के साथ नहीं जोड़ा गया है। यह तो एक तरफ का प्रयत्न था कि इन सिद्धान्तों द्वारा सरकार को निर्देश दिए जाएँ, ताकि सरकार उन कठिनाइयों को दूर कर सके जो उस समय समाज में विद्यमान थीं।

प्रश्न 11.
राज्य-नीति के निदेशक सिद्धान्तों की आलोचना किस आधार पर की जाती है?
उत्तर:
भारतीय संविधान के अध्याय IV में दिए गए निदेशक सिद्धान्तों की आलोचकों द्वारा कटु आलोचना की गई है। इन्हें व्यर्थ व अनावश्यक कहा गया है। संविधान में केवल उन्हीं बातों का वर्णन होता है जिनको व्यवहार में लाया जा सके, परन्तु निदेशक सिद्धान्त ऐसे तत्त्व हैं जिनको तुरन्त व्यवहार में नहीं लाया जा सकता। सरकार को इन पर चलने के लिए बाध्य नहीं किया जा सकता।

ये कोरे वायदे हैं जो जनता को धोखा देने के लिए संविधान में रखे गए हैं तथा इनकी वास्तविक उपयोगिता कुछ नहीं है। इनका महत्त्व राजनीतिक घोषणाओं के अतिरिक्त कुछ भी नहीं है। प्रो० व्हीयर (Prof. Wheare) ने इन्हें केवल उद्देश्यों व आकांक्षाओं का घोषणा-पत्र कहा है।

श्री के०टी० शाह (K.T. Shah) के अनुसार, “निदेशक सिद्धान्त उस चैक के समान हैं जिसका बैंक ने अपनी सुविधा के अनुसार भुगतान करना है।” (Directive Principles are like a cheque payable by the bank at its convenience.)

श्री नसीरुद्दीन (Sh. Nasiruddin) ने इनकी तुलना “नए वर्ष के उन प्रस्तावों से की है जो जनवरी के दूसरे दिन ही तोड़े जा सकते हैं।” (New years resolutions which can be broken on the second of January.) आलोचना निम्नलिखित अनुच्छेदों पर की गई है

1. इनके पीछे कानूनी शक्ति नहीं है (They are not backed by Legal Sanctions):
निदेशक सिद्धान्तों के पीछे कोई कानूनी शक्ति नहीं है तथा सरकार को इन पर चलने के लिए बाध्य नहीं किया जा सकता। इन्हें मौलिक अधिकारों की तरह न्यायालयों द्वारा लागू नहीं करवाया जा सकता। ये न्यायसंगत नहीं हैं। इनको व्यावहारिक रूप देना सरकार की इच्छा पर निर्भर करता है।

2. ये अप्राकृतिक हैं (They are Unnatural):
संविधान में इन सिद्धान्तों का सम्मिलित किया जाना अप्राकृतिक प्रतीत होता है। कोई भी प्रभुत्व-सम्पन्न राज्य अपने आपको इस प्रकार के निर्देश नहीं दे सकता। निर्देश सदा सर्वोच्च सत्ता द्वारा अधीनस्थ सत्ता को दिए जाते हैं, न कि अपने आपको।

3. ये व्यर्थ हैं (They are Superfluous):
राज्य-नीति के निदेशक सिद्धान्त संविधान की प्रस्तावना में निहित हैं। इनका पृथक् वर्णन करने की आवश्यकता नहीं है। इससे संविधान की जटिलता बढ़ती है।

4. ये स्थाई नहीं हैं (They are not Permanent):
इस प्रकार के सिद्धान्त स्थाई नहीं हो सकते। देश की आर्थिक व सामाजिक परिस्थितियाँ बदलती रहती हैं तथा समय और स्थिति की आवश्यकता के अनुसार सिद्धान्तों को बदलना पड़ता है। यह निश्चयपूर्वक नहीं कहा जा सकता कि जो सिद्धान्त 20वीं शताब्दी में उपयोगी समझे गए हैं, वे 21वीं शताब्दी में भी उपयोगी सिद्ध होंगे।

5. कुछ सिद्धान्त व्यावहारिक नहीं हैं (Some Principles are not Practicable):
निदेशक तत्त्वों में कुछ सिद्धान्त ऐसे हैं, जिनको व्यावहारिक रूप देना कठिन है। उदाहरणतया नशाबन्दी के सिद्धान्त को लागू करने से कई कठिनाइयाँ उत्पन्न होती हैं। एक ओर सरकार का राजस्व बहुत कम हो जाता है और दूसरी ओर अवैध शराब का निकालना बढ़ जाता है तथा कई कानूनी समस्याएँ उत्पन्न हो जाती हैं। आलोचकों का कहना है कि राज्य द्वारा लागू की गई नैतिकता कोई नैतिकता नहीं होती।

6. ये प्रकृति में विदेशी हैं (They are Foreigner in Nature):
इन सिद्धान्तों पर विदेशी विचारअनुच्छेद का प्रभाव है। सर आइवर जेनिंग्स (Sir Ivor Jennings) के अनुसार, “इन पर इंग्लैण्ड के 19वीं शताबी के फेबियन समाजवाद (Fabian Socialism) का प्रभाव स्पष्ट दिखलाई पड़ता है। जिन सिद्धान्तों को 19वीं शताब्दी में इंग्लैण्ड में ठीक माना जाता था, उन्हें 20वीं शताब्दी के भारतीय संविधान में स्थान देना प्रगतिशील व्यवस्था नहीं थी। ये सिद्धान्त भारतीय संस्कृति व परम्परा के अनुकूल नहीं हैं।”

इनका सही ढंग से वर्गीकरण नहीं किया गया है (They are not Properly Classified):
डॉ० श्रीनिवासन (Dr. Srinivasan) के अनुसार, “निदेशक सिद्धान्तों का उचित ढंग से वर्गीकरण नहीं किया गया है और न ही उन्हें क्रमबद्ध रखा गया है। इस घोषणा में अपेक्षाकृत कम महत्त्व वाले विषयों को अत्यन्त महत्त्वपूर्ण आर्थिक व सामाजिक प्रश्नों के साथ जोड़ दिया गया है। इसमें आधुनिकता का प्राचीनता के साथ बेमेल मिश्रण किया गया है। इसमें तर्कसंगत और वैज्ञानिक व्यवस्थाओं को भावनापूर्ण और द्वेषपूर्ण समस्याओं के साथ जोड़ा गया है।”

8. वे अस्पष्ट व दोहराए गए हैं (They are Vague and Repetitive):
ये सिद्धान्त अस्पष्ट व अनिश्चित हैं। इन्हें बार-बार दोहराया गया है। ये सिद्धान्त संविधान की प्रस्तावना में निहित हैं। डॉ० श्रीनिवासन के अनुसार, “इन्हें उत्साहपूर्ण नहीं कहा जा सकता। ये अस्पष्ट व दोहराए गए सिद्धान्त हैं।” ।

9. ये कोरे वायदे हैं (They are Unused Promises):
आलोचकों का कहना है कि निदेशक सिद्धान्त जनता को धोखा देने का साधन मात्र हैं। ये कोरे आश्वासन हैं जिनसे जनसाधारण को सन्तुष्ट रखने का प्रयास किया गया है। इनके द्वारा भोली-भाली जनता को झुठलाने की कोशिश की गई है।

10. इनमें राजनीतिक दार्शनिकता अधिक है व व्यावहारिक राजनीति कम (They are more a Political Philoso phy than a Practical Politics) ये सिद्धान्त शुद्ध आदर्शवाद प्रस्तुत करते हैं। इनका व्यावहारिक राजनीति से बहुत कम सम्बन्ध है।

प्रश्न 12.
राज्य-नीति के निदेशक सिद्धान्तों की उपयोगिता तथा महत्त्व का वर्णन करें।
उत्तर:
राज्य-नीति के निदेशक सिद्धान्तों की उपयोगिता तथा महत्त्व (Utility and Importance of Directive Principles of State Policy)-राज्य-नीति के निदेशक सिद्धान्तों की काफी आलोचना हुई है और बहुत-से लोगों ने इन्हें केवल नव वर्ष की शुभ कामनाएँ मात्र कहा है, क्योंकि इन्हें लागू करवाने के लिए न्यायालय का द्वार नहीं खटखटाया जा सकता।

ये सरकार की इच्छा पर निर्भर करते हैं, परन्तु इनकी उपयोगिता से इन्कार नहीं किया जा सकता। निदेशक सिद्धान्तों में निहित सामाजिक व आर्थिक लोकतन्त्र के उद्देश्यों को प्राप्त किए बिना भारत में लोकतन्त्र सफल हो ही नहीं सकता। निदेशक सिद्धान्तों की उपयोगिता निम्नलिखित बातों से स्पष्ट होती है

1. ये मौलिक अधिकारों के पूरक हैं (These are Supplement of the Fundamental Rights):
मौलिक अधिकार देश में केवल राजनीतिक लोकतन्त्र (Political Democracy) को स्थापित करते हैं, परन्तु राजनीतिक लोकतन्त्र की सफलता के लिए देश में आर्थिक लोकतन्त्र की व्यवस्था करनी आवश्यक है। आर्थिक लोकतन्त्र के बिना राजनीतिक लोकतन्त्र अधूरा रह जाता है। राजनीतिक लोकतन्त्र में लोगों को राजनीतिक स्वतन्त्रताएँ तो प्राप्त होती हैं, परन्तु उन्हें आर्थिक चिन्ताओं से छुटकारा नहीं मिल पाता।

उन्हें वे सुविधाएँ प्राप्त नहीं होतीं, जिनसे उनका जीवन-स्तर ऊपर उठ सके और वे आर्थिक दृष्टि से समृद्ध हो सकें। निदेशक सिद्धान्त भारतीय संविधान में दिए गए मौलिक अधिकारों की कमी को दूर करते हैं और उन तत्त्वों को अपनाने के लिए कहते हैं जिन पर चलते हुए इस देश में राजनीतिक लोकतन्त्र के साथ-साथ आर्थिक लोकतन्त्र की भी स्थापना की जा सके।

2. ये सरकारों के लिए निर्देश हैं (They are Directives to Governments):
ये सिद्धान्त केन्द्रीय, राज्य व स्थानीय सरकारों का मागदर्शन करते हैं और उन्हें बतलाते हैं कि संविधान में निश्चित किए गए उद्देश्यों की प्राप्ति के लिए उन्हें क्या-क्या कार्य करने हैं। इन सिद्धान्तों की तुलना 1935 के अधिनियम के अन्तर्गत गवर्नर जनरल व गवर्नरों को जारी किए गए उन निर्देश-पत्रों से की जा सकती है जिनमें उन्हें अपनी शक्तियों का प्रयोग करने के बारे में निर्देश दिए जाते थे।

अन्तर केवल इतना है कि इन निर्देश-पत्रों में निर्देश विधानपालिका व कार्यपालिका दोनों को दिए गए हैं, जबकि नीति निदेशक सिद्धान्तों के अन्तर्गत निर्देश केवल कार्यपालिका को ही दिए जाते हैं। श्री एम०सी० सीतलवाद (M.C. Setalvad) के अनुसार, “वे संघ में समस्त अधिकारी वर्ग को दिए गए निर्देश-पत्र या सामान्य सिफारिशों के समान प्रतीत होते हैं जो उन्हें उन आधारभूत सिद्धान्तों की याद दिलाते हैं जिनके आधार पर संविधान का उद्देश्य नई सामाजिक व आर्थिक व्यवस्था का निर्माण करना है।”

3. ये राज्य के सकारात्मक दायित्व हैं (They are Positive Obligations of State):
राज्य-नीति के निदेशक सिद्धान्त राज्य की सकारात्मक ज़िम्मेदारियाँ बतलाते हैं। ग्रेनविल ऑस्टिन (Granville Austin) के शब्दों में, “राज्य के सकारात्मक दायित्व निश्चित करते हुए संविधान सभा के सदस्यों ने भारत की भावी सरकारों को यह ज़िम्मेदारी सौंपी कि वे व्यक्तिगत स्वतन्त्रता और सार्वजनिक भलाई के बीच तथा कुछ व्यक्तियों की सम्पत्ति व विशेषाधिकार और सभी लोगों को लाभ पहुँचाने के बीच का मार्ग निकालें, ताकि सभी लोगों की शक्तियों को समान रूप से सभी लोगों की भलाई के लिए प्रयोग किया जा सके।”

4. ये सामाजिक क्रांति का आधार हैं (These are basis of Social Revolution):
राज्य-नीति के निदेशक सिद्धान्त नई सामाजिक दशा के सूचक हैं। वे सामाजिक क्रांति का आधार हैं। ग्रेनविल ऑस्टिन (Granville Austin) के अनुसार, “निदेशक सिद्धान्तों में सामाजिक क्रांति का स्पष्ट विवरण मिलता है। वे भारतीय जनता को सकारात्मक अर्थ में स्वतन्त्र करते हैं। वे उन्हें समाज और प्रकृति द्वारा उत्पन्न की गई शताब्दियों की निष्क्रियता से मुक्ति दिलाते हैं। वे उन्हें उन घृणित शारीरिक परिस्थितियों से मुक्त कराते हैं जिन्होंने उन्हें अपने जीवन का सर्वोत्तम विकास करने से रोके रखा था।”

5. इन सिद्धान्तों के पीछे जनमत की शक्ति है (These Principles are backed by Public Opinion):
ये सिद्धान्त न्याय-संगत नहीं हैं, परन्तु इनके पीछे लोकमत की शक्ति है। जैसा कि पहले बताया जा चुका है, कोई भी सरकार लोकमत की अवहेलना नहीं कर सकती। यदि वह ऐसा करेगी तो वह लोगों का विश्वास खो बैठेगी और अगले आम चुनाव में उसकी हार अवश्य होगी।

प्रो० पायली के अनुसार, “ये निर्देश राष्ट्र की आत्मा का आधारभूत स्तर हैं तथा जो इनका उल्लंघन करेंगे, वे अपने आप को जिम्मेदार स्थान (Position of Responsibility) से हटाने का खतरा मोल लेंगे जिसके लिए उन्हें चुना गया है।”

6. ये सरकार की नीतियों में स्थिरता बनाए रखते हैं (These maintain stability in the Policies of Govt.):
संसदीय शासन-प्रणाली में सरकारें बदलती रहती हैं, परन्तु जो भी सरकार सत्तारूढ़ होगी, वह इन सिद्धान्तों पर चलने के लिए बाध्य होगी। इस प्रकार सरकार की मूल नीतियों में निरन्तरता व स्थिरता बनी रहेगी।

7. ये सिद्धान्त शैक्षणिक महत्त्व रखते हैं (These Principles have an Educative Value):
ये सिद्धान्त आने वाली पीढ़ियों के नवयुवकों को इस बात की शिक्षा देंगे कि हमारे संविधान-निर्माता देश में किस प्रकार की सामाजिक व आर्थिक व्यवस्था का निर्माण करना चाहते थे तथा इस बारे में क्या कछ किया जा चका है और क्या कछ करना बाकी है। प्रो० एम०वी० पायली (Prof. M.V. Pylee) के अनुसार, “वे भविष्य के नवयुवकों के मन व विचारों में स्थिर व गतिशील राजनीतिक व्यवस्था के मूल तत्त्वों को स्थान देंगे।” .

8. कल्याणकारी राज्य की स्थापना में सहायक (Helpful for Establishment of a Welfare State):
आज कल्याणकारी राज्य का युग है और इन सिद्धान्तों में ही कल्याणकारी राज्य के आदर्शों की घोषणा की गई है। इन्हें लागू करके ही भारत को एक वास्तविक कल्याणकारी राज्य बनाया जा सकता है। न्यायमूर्ति संपू (Justice Sapru) ने कहा है कि राज्य-नीति के निदेशक सिद्धान्तों में वे सभी बातें विद्यमान हैं जिनके आधार पर किसी भी आधुनिक जाति में कल्याणकारी राज्य की स्थापना की. जा सकती है।

9. आर्थिक लोकतन्त्र हेत (For Economic Democracy):
आर्थिक लोकतन्त्र के अभाव में राजनीतिक लोकतन्त्र अर्थहीन है। आर्थिक लोकतन्त्र को निदेशक तत्त्वों के द्वारा ही प्राप्त किया जा सकता है। डॉ० अम्बेडकर के शब्दों में, “प्रत्येक सरकार आर्थिक प्रजातन्त्र की स्थापना के लिए प्रयास करेगी। इसी उद्देश्य से संविधान के चौथे भाग में कुछ सिद्धान्तों का उल्लेख किया गया है।”

10. संविधान का पालन (Enforcement of Constitution):
संविधान का पालन निदेशक तत्त्वों के पालन में ही हेत है। डॉ० अल्लादि कृष्णास्वामी का कथन है, “यदि राज्य इन आदर्शों की उपेक्षा करता है तो व्यावहारिक रूप से यह स्वयं संविधान की उपेक्षा करने के समान होगा।”

11. न्यायपालिका के लिए मार्गदर्शक (Guide for Judiciary):
भारतीय न्यायपालिका ही कानूनों तथा संविधानों की व्यवस्था का अधिकार रखती है। बेशक ये सिद्धान्त न्यायालयों के माध्यम से लागू नहीं करवाए जा सकते, परन्तु इन्होंने कानूनों तथा संविधान की व्याख्या में न्यायपालिका का मार्गदर्शन करने में महत्त्वपूर्ण भूमिका निभाई है तथा उच्चतम न्यायालय ने अपने निर्णयों में इनका उल्लेख भी कर दिया है।

12. सरकार की नीति में निरन्तरता तथा स्थिरता (Continuity and Stability in Policies of the Government):
वैसे तो जब भी सत्ता में परिवर्तन होता है और दूसरे दल के हाथ में सत्ता आती है तो वह अपनी नीति निश्चित करता है, परन्तु इन सिद्धान्तों के कारण सरकार की नीति में निरन्तरता काफी मात्रा में बनी रहती है और उसे स्थिरता मिलती है क्योंकि प्रत्येक दल जन-कल्याण की नीति अवश्य अपनाता है और ऐसा करते समय उसे इनका सहारा लेना ही पड़ता है।

13. शासकीय कार्यों के मूल्याँकन हेतु मापदण्ड (Measurement for Evaluation of Government’s Functions):
नीति-निदेशक तत्त्व जनता को शासकीय कार्यों के मूल्यांकन के लिए मापदण्ड प्रदान करते हैं जिनके आधार पर जनता शासकीय कार्यों का मूल्यांकन कर सकती है। डॉ० अम्बेडकर के शब्दों में, “यदि कोई सरकार इनकी अवहेलना करती है तो उसे निर्वाचन के समय निर्वाचकों को निश्चित रूप से जवाब देना होगा।”

निष्कर्ष (Conclusion)-निष्कर्ष रूप में हम कह सकते हैं कि भारतीय संविधान में लिखित उद्देश्यों-लोक-कल्याण, लोकतन्त्र, सामाजिक, आर्थिक व राजनीतिक न्याय, व्यक्ति की गरिमा इत्यादि को नीति-निदेशक तत्त्वों को लागू करके ही प्राप्त किया जा सकता है। वाद-योग्य न होने से इनका महत्त्व कम नहीं हो जाता।

वर्तमान में तो न्यायपालिका भी अपने निर्णय इन सिद्धान्तों के आधार पर देने लगी है। इन्हें उसी प्रकार से भारतीय जनता का समर्थन प्राप्त है, जिस प्रकार इंग्लैण्ड में प्रथाओं को प्राप्त है। भारत का भविष्य इन निदेशक सिद्धान्तों पर ही निर्भर है। एम०सी० छागला (M.C. Chagla) के अनुसार, “यदि इन सिद्धान्तों को ठीक प्रकार से अपनाया जाए तो हमारा देश वास्तव में धरती पर स्वर्ग बन जाएगा।

भारतीयों के लिए लोकतन्त्र तब केवल राजनीतिक अर्थ में ही नहीं होगा, बल्कि यह भारत के नागरिकों के कल्याण के लिए बना एक ऐसा कल्याणकारी राज्य होगा जिसमें कि प्रत्येक व्यक्ति को कार्य करने के लिए शिक्षा प्राप्त करने और अपने श्रम का उचित प्रतिफल पाने का समान अवसर प्राप्त होगा।”

वस्तु निष्ठ प्रश्न

निम्नलिखित प्रश्नों का उत्तर दिए गए विकल्पों में से उचित विकल्प छाँटकर लिखें

1. मौलिक अधिकारों में संशोधन किया जा सकता है-
(A) राष्ट्रपति के द्वारा
(B) संसद के द्वारा
(C) विधानमंडलों के द्वारा
(D) प्रधानमंत्री के द्वारा
उत्तर:
(B) संसद के द्वारा

2. संविधान में धार्मिक स्वतंत्रता के अधिकार की व्यवस्था है
(A) अनुच्छेद 14 से 18 के अंतर्गत
(B) अनुच्छेद 19 से 22 के अंतर्गत
(C) अनुच्छेद 23 से 25 के अंतर्गत
(D) अनुच्छेद 25 से 28 के अंतर्गत
उत्तर:
(D) अनुच्छेद 25 से 28 के अंतर्गत

3. कौन-से संवैधानिक संशोधन द्वारा मौलिक कर्त्तव्य संविधान में अंकित किए गए हैं
(A) 42वें संशोधन द्वारा
(B) 44वें संशोधन द्वारा
(C) 45वें संशोधन द्वारा
(D) 47वें संशोधन द्वारा
उत्तर:
(A) 42वें संशोधन द्वारा

HBSE 11th Class Political Science Important Questions Chapter 2 भारतीय संविधान में अधिकार

4. संविधान के कौन-से भाग में मौलिक कर्त्तव्य अंकित हैं
(A) भाग तृतीय में
(B) भाग चतुर्थ में
(C) भाग चतुर्थ-ए में
(D) भाग पांचवें में
उत्तर:
(C) भाग चतुर्थ-ए में

5. भारतीय नागरिकों को 6 प्रकार के मौलिक अधिकार प्रदान किए गए हैं
(A) अनुच्छेद 14 से 31
(B) अनुच्छेद 12 से 35
(C) अनुच्छेद 12 से 35
(D) अनुच्छेद 1 से 32
उत्तर:
(C) अनुच्छेद 12 से 35

6. ‘समानता का अधिकार’ (Right to Equality) में सम्मिलित नहीं है
(A) कानून के समक्ष समानता
(B) अवसर की समानता
(C) सामाजिक समानता
(D) आर्थिक समानता
उत्तर:
(D) आर्थिक समानता

7. कौन-से अनुच्छेद द्वारा संसद को मौलिक अधिकारों को सीमित करने की मनाही की गई है?
(A) अनुच्छेद 13 द्वारा
(B) अनुच्छेद 33 द्वारा
(C) अनुच्छेद 35 द्वारा
(D) अनुच्छेद 18 द्वारा
उत्तर:
(A) अनुच्छेद 13 द्वारा

8. निम्नलिखित आदेशों में से कौन-सा आदेश न्यायालय द्वारा गैर-कानूनी ढंग से नजरबन्द किए गए व्यक्ति को छोड़ने के लिए दिया जाता है?
(A) बंदी प्रत्यक्षीकरण
(B) परमादेश
(C) उत्प्रेषण लेख
(D) अधिकार पृच्छा
उत्तर:
(A) बंदी प्रत्यक्षीकरण

9. ‘समानता के अधिकार’ (Right to Equality) की व्यवस्था है
(A) अनुच्छेद 14 से 18 के अंतर्गत
(B) अनुच्छेद 19 से 22 के अंतर्गत
(C) अनुच्छेद 25 से 28 के अंतर्गत
(D) अनुच्छेद 29 से 32 के अंतर्गत
उत्तर:
(A) अनुच्छेद 14 से 18 के अंतर्गत

10. मौलिक अधिकारों वाले अध्याय में स्पष्ट वर्णन नहीं है
(A) विचार प्रकट करने की स्वतंत्रता का
(B) धार्मिक स्वतंत्रता का
(C) कानून के समक्ष समानता का
(D) प्रेस की स्वतंत्रता का
उत्तर:
(D) प्रेस की स्वतंत्रता का

11. सांस्कृतिक और शिक्षा संबंधी अधिकारों की भारतीय संविधान में व्यवस्था है
(A) अनुच्छेद 31 व 32 के अंतर्गत.
(B) अनुच्छेद 23 व 24 के अंतर्गत
(C) अनुच्छेद 29 व 30 के अंतर्गत
(D) अनुच्छेद 33 व 34 के अंतर्गत
उत्तर:
(C) अनुच्छेद 29 व 30 के अंतर्गत

12. ‘स्वतंत्रता के अधिकार’ (Right to Freedom) में सम्मिलित नहीं है
(A) भाषण देने और विचार प्रकट करने की स्वतंत्रता
(B) समुदाय बनाने की स्वतंत्रता
(C) कोई कारोबार करने की स्वतंत्रता
(D) हथियारों सहित एकत्र होने और सभा करने की स्वतंत्रता
उत्तर:
(D) हथियारों सहित एकत्र होने और सभा करने की स्वतंत्रता

13. निम्नलिखित में से कौन-सा मौलिक कर्त्तव्य नहीं है?
(A) संविधान का पालन करना
(B) भारत की प्रभुसत्ता, एकता और अखंडता का समर्थन व रक्षा करना
(C) सार्वजनिक संपत्ति की रक्षा करना
(D) माता-पिता की सेवा करना
उत्तर:
(D) माता-पिता की सेवा करना

14. निम्नलिखित में से किस आदेश के द्वारा किसी व्यक्ति को उस कार्रवाई को करने से रोक दिया जाता है जिसके लिए वह कानूनी रूप से उपयुक्त नहीं है?
(A) परमादेश
(B) प्रतिषेध
(C) उत्प्रेषण
(D) अधिकार पृच्छा
उत्तर:
(D) अधिकार पृच्छा

15. किसके सुझावानुसार निदेशक सिद्धांत संविधान में अंकित किए गए थे?
(A) डॉ० बी०आर० अंबेडकर
(B) डॉ० राजेंद्र प्रसाद
(C) पं० जवाहरलाल नेहरू
(D) सर वी०एन० राव
उत्तर:
(D) सर वी०एन० राव

16. यह किसने कहा था कि “राज्य-नीति के निदेशक सिद्धांत एक चैक की तरह हैं जिसका भुगतान बैंक की सुविधा पर छोड़ दिया है।”
(A) केन्टी० शाह ने
(B) नसीरुद्दीन ने
(C) डॉ० राजेंद्र प्रसाद ने
(D) श्रीमती इंदिरा गांधी ने
उत्तर:
(A) के०टी० शाह ने

17. संविधान में निदेशक सिद्धांत अंकित करने की प्रेरणा मिली थी
(A) 1935 के भारत सरकार कानून से
(B) ऑस्ट्रेलिया के संविधान से
(C) आयरलैंड के संविधान से
(D) अमेरिका के संविधान से
उत्तर:
(C) आयरलैंड के संविधान से

18. निम्नलिखित में से नीति-निदेशक सिद्धांतों का उद्देश्य कौन-सा है?
(A) पुलिस-राज्य की स्थापना करना
(B) सामाजिक तथा आर्थिक लोकतंत्र की स्थापना करना
(C) राजनीतिक लोकतंत्र की स्थापना करना
(D) समाजवादी राज्य की स्थापना करना
उत्तर:
(B) सामाजिक तथा आर्थिक लोकतंत्र की स्थापना करना

19. निम्नलिखित निदेशक सिद्धांतों में से कौन-सा उदारवादी है?
(A) काम देने का अधिकार
(B) विश्व-शांति एवं सुरक्षा
(C) समान व्यवहार संहिता
(D) धन केंद्रीकरण पर रोक
उत्तर:
(C) समान व्यवहार संहिता

20. निम्नलिखित निदेशक सिद्धांत सरकार द्वारा अभी तक लागू नहीं किया गया है
(A) पुरुषों तथा स्त्रियों के लिए समान
(B) ग्राम पंचायतों की स्थापना करना कार्य के लिए समान वेतन की व्यवस्था
(C) शराब-बंदी लागू करना
(D) कृषि तथा उद्योग को बढ़ावा देने के लिए प्रयत्न करना
उत्तर:
(C) शराब-बंदी लागू करना

21. निम्नलिखित में से किसने नीति-निदेशक सिद्धांतों को संविधान की अनोखी विशेषता कहा है?
(A) श्री वी०एन० राय ने
(B) पंडित जवाहरलाल नेहरू ने
(C) डॉ० बी० आर० अंबेडकर ने
(D) इनमें से कोई नहीं
उत्तर:
(C) डॉ० बी० आर० अंबेडकर ने

22. कौन-से निदेशक सिद्धांतों में भारतीय विदेश नीति के कुछ मूल आधार मिलते हैं?
(A) अनुच्छेद 36 में अंकित निदेशक सिद्धांत में
(B) अनुच्छेद 48 में अंकित निदेशक सिद्धांत में
(C) अनुच्छेद 51 में अंकित निदेशक सिद्धांत में
(D) उपर्युक्त सभी में
उत्तर:
(C) अनुच्छेद 51 में अंकित निदेशक सिद्धांत में

23. “निदेशक सिद्धांतों का कोई महत्त्व नहीं है” यह कथन है
(A) डॉ० बी०आर० अंबेडकर
(B) जैनिंग्स
(C) डॉ० श्रीनिवासन
(D) इनमें से कोई नहीं
उत्तर:
(D) इनमें से कोई नहीं ।

24. निम्नलिखित में से कौन-सा सिद्धांत गांधीवादी सिद्धांत है ?
(A) ग्राम पंचायतों और स्वशासन की समाप्ति
(B) स्त्री तथा पुरुषों को समान कार्य के लिए समान वेतन
(C) देश के सभी नागरिकों के लिए आचार संहिता
(D) इनमें से कोई नहीं
उत्तर:
(A) ग्राम पंचायतों और स्वशासन की समाप्ति

25. “राज्य-नीति के निदेशक सिद्धांत नौ वर्षों के प्रस्तावों का संग्रह है।” यह कथन किस विद्वान् का है?
(A) के०सी० ह्वीयर का
(B) नसीरुद्दीन
(C) डॉ० अंबेडकर का
(D) इनमें से कोई नहीं
उत्तर:
(B) नसीरुद्दीन

26. निदेशक सिद्धांतों की आलोचना के संबंध में निम्नलिखित ठीक है
(A) अनिश्चित तथा अस्पष्ट
(B) वैधानिक शक्ति का अभाव
(C) पर्याप्त साधनों का अभाव
(D) उपर्युक्त सभी
उत्तर:
(D) उपर्युक्त सभी

27. निम्नलिखित निदेशक सिद्धांतों में एक अंतर्राष्ट्रीयवाद संबंधी सिद्धांत है
(A) विश्व-शांति एवं सुरक्षा
(B) ग्राम पंचायतों का संगठन
(C) समान व्यवहार संहिता
(D) समान कार्य, समान वेतन
उत्तर:
(A) विश्व-शांति एवं सुरक्षा

निम्नलिखित प्रश्नों के उत्तर एक वाक्य या एक शब्द में दीजिए-

1. भारतीय संविधान के कौन-से भाग या अध्याय में मौलिक अधिकारों की व्यवस्था की गई है?
उत्तर:
अध्याय तीन में।

2. मूल अधिकारों का उल्लेख संविधान के किन अनुच्छेदों में किया गया है?
उत्तर:
अनुच्छेद 12 से 35 तक।

3. भारतीय संविधान के किन अनुच्छेदों में ‘स्वतंत्रता के अधिकार’ का उल्लेख है?
उत्तर:
अनुच्छेद 19 से 22 तक।

4. भारतीय संविधान में वर्णित संवैधानिक उपचारों के अधिकार का उल्लेख कौन-से अनुच्छेद में किया गया है?
उत्तर:
अनुच्छेद 32 में।

5. भारत में राष्ट्रीय मानवाधिकार आयोग की स्थापना कब की गई?
उत्तर:
अक्तूबर, 1993 में।

6. कौन-से सवैधानिक संशोधन के द्वारा सम्पत्ति के अधिकार को मौलिक अधिकारों से निकाला गया?
उत्तर:
44वें संवैधानिक संशोधन के द्वारा।

7. राष्ट्रीय मानवाधिकार आयोग के वर्तमान अध्यक्ष कौन हैं?
उत्तर:
पूर्व मुख्य न्यायाधीश श्री एच०एल० दत्तू।

8. राष्ट्रीय मानवाधिकार आयोग के वर्तमान महासचिव कौन हैं?
उत्तर:
श्री जयदीप गोविंद।

9. राष्ट्रीय मानवाधिकार आयोग के अध्यक्ष का कार्यकाल कितना है?
उत्तर:
5 वर्ष या 70 वर्ष की आयु जो भी पहले पूर्ण हो।

10. राष्ट्रीय मानवाधिकार आयोग के अध्यक्ष या सदस्य अपना त्यागपत्र किसे सौंपते हैं?
उत्तर:
राष्ट्रपति को।

11. राज्य नीति के निर्देशक सिद्धांतों का उल्लेख संविधान के कौन-से अध्याय में किया गया है?
उत्तर:
अध्याय IV में।

रिक्त स्थान भरें

1. भारतीय संविधान के अध्याय ………… में मौलिक अधिकारों का उल्लेख किया गया है।
उत्तर:
III

2. भारतीय संविधान का अध्याय …………. राज्य नीति के निर्देशक सिद्धान्तों से सम्बन्धित है।
उत्तर:
IV

3. भारतीय संविधान में राज्यनीति के निर्देशक सिद्धान्त ………….. के संविधान से प्रेरित होकर सम्माहित किए गए है।
उत्तर:
आयरलैण्ड

4. वर्तमान में ………….. राष्ट्रीय मानवाधिकार आयोग के अध्यक्ष हैं।
उत्तर:
श्री एच०एल० दत्तू

5. राष्ट्रीय मानवाधिकार आयोग के अध्यक्ष का कार्यकाल ……………. या अधिकतर ……………. तक की आयु तक है।
उत्तर:
5 वर्ष, 70 वर्ष

6. वर्तमान में संविधान में अंकित मौलिक कर्त्तव्यों की संख्या …………. है।
उत्तर:
11

7. भारत में राष्ट्रीय मानवाधिकार आयोग का गठन ………….. में हुआ।
उत्तर:
1993

8. दक्षिण अफ्रीका का संविधान सन् …………. में लागू हुआ।
उत्तर:
1996

9. भारत में शिक्षा के मूल अधिकार के रूप में …………. संवैधानिक संशोधन द्वारा संवैधानिक रूप दिया गया।
उत्तर:
86वें

HBSE 11th Class Political Science Important Questions Chapter 2 भारतीय संविधान में अधिकार

10. संविधान के भाग ……………. में मौलिक कर्तव्यों का उल्लेख किया गया।
उत्तर:
IV क (अनुच्छेद 51क)

11. वर्तमान में ……….. राष्ट्रीय मानवाधिकार आयोग के महासचिव हैं।
उत्तर:
जयदीप गोविंद

12. …………. ने सवैधानिक उपचारों के अधिकार को संविधान की आत्मा एवं हृदय कहा है।
उत्तर:
डॉ० बी.आर. अम्बेडकर

13. भारतीय संविधान का अनुच्छेद ………….. किसी भी रूप में अस्पृश्यता का निषेध करता है।
उत्तर:
अनुच्छेद 17

14. भारतीय संविधान का अनुच्छेद ………… सामाजिक, शैक्षणिक रूप से पिछड़े वर्गों हेतु आरक्षण का प्रावधान करता है।
उत्तर:
अनुच्छेद 151

HBSE 11th Class Political Science Important Questions Chapter 1 संविधान : क्यों और कैसे?

Haryana State Board HBSE 11th Class Political Science Important Questions Chapter 1 संविधान : क्यों और कैसे? Important Questions and Answers.

Haryana Board 11th Class Political Science Important Questions Chapter 1 संविधान : क्यों और कैसे?

अति लघूत्तरात्मक प्रश्न

प्रश्न 1.
संविधान से क्या अभिप्राय है?
उत्तर:
संविधान उन मौलिक नियमों, सिद्धांतों तथा परम्पराओं का संग्रह होता है, जिनके अनुसार राज्य की सरकार का गठन, सरकार के कार्य, नागरिकों के अधिकार तथा नागरिकों और सरकार के बीच संबंधों को निश्चित किया जाता है।

प्रश्न 2.
लिखित संविधान किसे कहा जाता है?
उत्तर:
लिखित संविधान वह संविधान होता है जो एक गठित संविधान सभा द्वारा बनाया जाए और एक निश्चित तिथि को लागू हो; जैसे अमेरिका, भारत, जापान, आयरलैण्ड आदि देशों के संविधान लिखित संविधान कहलाते हैं।

प्रश्न 3.
अलिखित संविधान किसे कहा जाता है?
उत्तर:
अलिखित संविधान ऐसा संविधान होता है जो प्रायः रीति-रिवाजों, परम्पराओं तथा समय-समय पर दिए गए न्यायिक निर्णयों पर आधारित होता है जिनका धीरे-धीरे विकास होता है। ऐसे संविधान के लिए न संविधान सभा निर्मित होती है और ही उन्हें लागू करने की निश्चित शर्त को पूर्ण करना होता है। इंग्लैण्ड का संविधान अलिखित संविधान है।

प्रश्न 4.
संविधान हमारे लिए क्यों आवश्यक है? कोई दो कारण लिखिए।
उत्तर:

  • संविधान ऐसे मौलिक नियमों को निश्चित करता है जो समाज में रहने वाले लोगों में समन्वय तथा आपसी विश्वास की स्थापना करते हैं।
  • संविधान समाज में सरकार के वर्तमान एवं भविष्य में सरकार के संचालन के मूल सिद्धांतों एवं आदर्शों पर प्रकाश डालता है।

प्रश्न 5.
भारतीय संविधान सभा में प्रारम्भ में कुल सदस्यों की संख्या कितनी थी?
उत्तर:
भारतीय संविधान सभा में प्रारम्भ में कुल सदस्यों की संख्या 389 थी जिसमें 292 ब्रिटिश प्रांतों के प्रतिनिधि, 4 चीफ कमिश्नर प्रांतों के तथा 93 देशी रियासतों के प्रतिनिधि थे।

प्रश्न 6.
संविधान सभा के प्रथम स्थायी अध्यक्ष कौन थे? उन्हें कब स्थायी अध्यक्ष चुना गया?
उत्तर:
संविधान सभा के प्रथम स्थायी अध्यक्ष डॉ० राजेन्द्र प्रसाद थे, जिन्हें 11 दिसम्बर, 1946 को संविधान सभा द्वारा निर्विरोध चुना गया था।

प्रश्न 7.
भारत की स्वाधीनता एवं विभाजन के बाद संविधान सभा के सदस्यों की संख्या कितनी रह गई?
उत्तर:
भारत की स्वाधीनता एवं विभाजन के बाद संविधान सभा के कुल सदस्यों की संख्या 324 रह गई जिसमें 235 प्रांतों के प्रतिनिधि तथा 89 देशी रियासतों के प्रतिनिधि थे।

प्रश्न 8.
भारतीय संविधान सभा के किन्हीं चार सदस्यों के नाम बताइए।
उत्तर:

  • डॉ० राजेन्द्र प्रसाद,
  • जवाहरलाल नेहरू,
  • सरदार पटेल,
  • डॉ० बी०आर० अम्बेडकर।

प्रश्न 9.
भारतीय संविधान सभा में उद्देश्य संबंधी प्रस्ताव कब और किसके द्वारा प्रस्तुत किया गया?
उत्तर:
भारतीय संविधान सभा में 13 दिसम्बर, 1946 को पं० जवाहरलाल नेहरू ने उद्देश्य संबंधी प्रस्ताव प्रस्तुत किया।

प्रश्न 10.
भारतीय संविधान सभा में प्रस्तुत उद्देश्य प्रस्ताव संबंधी किन्हीं दो बातों का उल्लेख कीजिए।
उत्तर:

  • संविधान सभा भारत को एक प्रभुसत्ता सम्पन्न प्रजातंत्रीय गणराज्य घोषित करने के दृढ़-निश्चय की घोषणा करती है।
  • यह प्राचीन भूमि विश्व में अपना उचित तथा सम्मानित स्थान ग्रहण करती है और मानव-कल्याण व विश्व शांति के विस्तार में अपना पूर्ण तथा ऐच्छिक योगदान करती है।

HBSE 11th Class Political Science Important Questions Chapter 1 संविधान : क्यों और कैसे?

प्रश्न 11.
भारतीय संविधान सभा द्वारा निर्मित विभिन्न समितियों में से किन्हीं चार समितियों के नाम लिखिए।
उत्तर:

  • संघीय संविधान समिति,
  • संघीय शक्तियाँ समिति,
  • प्रांतीय संविधान समिति,
  • अल्पसंख्यकों तथा मौलिक अधिकारों से सम्बन्धित समिति।

प्रश्न 12.
भारतीय संविधान सभा की मसौदा समिति में कुल कितने सदस्य थे?
उत्तर:
भारतीय संविधान सभा की मसौदा समिति के अध्यक्ष सहित कुल सात सदस्य थे, जिनमें दो सदस्य गोपाल स्वामी आयंगर एवं के०एम० मुन्शी थे।

प्रश्न 13.
भारतीय संविधान सभा द्वारा पारित संविधान पर कब और किसके अन्तिम हस्ताक्षर हुए थे?
उत्तर:
भारतीय संविधान सभा द्वारा पारित संविधान पर 26 नवम्बर, 1949 को संविधान सभा के अध्यक्ष डॉ० राजेन्द्र प्रसाद के अन्तिम हस्ताक्षर हुए थे।

प्रश्न 14.
भारतीय संविधान सभा के द्वारा संविधान के निर्माण हेतु कितने अधिवेशन हुए एवं कुल कितने दिन विचार-विमर्श हुआ?
उत्तर:
भारतीय संविधान सभा के द्वारा संविधान निर्माण हेतु कुल 11 अधिवेशन हुए और उन्होंने 165 दिनों तक विचार-विमर्श किया।

प्रश्न 15.
भारतीय संविधान सभा के किन सदस्यों को संविधान सभा का आन्तरिक वर्ग (Inner Circle) कहा जाता है?
उत्तर:
पं० जवाहरलाल नेहरू, सरदार वल्लभ भाई पटेल, डॉ० अब्दुल कलाम आज़ाद, डॉ० राजेन्द्र प्रसाद, सीता रमैया, गोविंद बल्लभ पन्त, डॉ० अम्बेडकर, एन०सी० आयंगर, के०एम० मुन्शी, आयर एवं सत्यनारायण सिन्हा सहित कुल 11 सदस्यों को संविधान सभा के आन्तरिक-वर्ग (Inner-circle) में शामिल किया जाता था।

प्रश्न 16.
संविधान सभा के प्रतिनिधित्व के स्वरूप की आलोचना किन आधारों पर की जाती है? कोई दो आधार लिखें।
उत्तर:

  • संविधान के प्रतिनिधियों का चुनाव वयस्क मताधिकार के आधार पर नहीं हुआ।
  • संविधान सभा में एक विशेष जाति हिन्दुओं को प्रतिनिधित्व दिया गया था।

प्रश्न 17.
संविधान सभा में निर्णय लेने की प्रक्रिया का संक्षेप में उल्लेख कीजिए।
उत्तर:
संविधान सभा में निर्णय की प्रक्रिया सहमति एवं समायोजन के सिद्धांत पर आधारित थी जो कि पूर्णतः लोकतांत्रिक प्रक्रिया थी।

प्रश्न 18.
भारतीय संविधान के किन्हीं दो स्रोतों का उल्लेख कीजिए।
उत्तर:

  • भारत सरकार अधिनियम, 1935 तथा,
  • विदेशी संविधानों का प्रभाव।

प्रश्न 19.
भारतीय संविधान पर कनाडा के संविधान के पड़ने वाले किन्हीं दो प्रभावों का उल्लेख कीजिए।
उत्तर:

  • भारतीय संविधान में कनाडा की भाँति भारत को ‘राज्यों का संघ’ कहा है।
  • संघात्मक ढाँचे के साथ-साथ केन्द्र को शक्तिशाली बनाना भी कनाडा के संविधान की देन है।

प्रश्न 20.
भारतीय संविधान में राज्य-नीति के निर्देशक सिद्धांतों संबंधी प्रावधान किस देश के संविधान से प्रभावित होकर किया गया है?
उत्तर:
आयरलैण्ड के संविधान से प्रभावित होकर किया।

प्रश्न 21.
भारतीय संविधान के संचालन में भूमिका निभाने वाली किन्हीं दो राजनीतिक प्रथाओं का उल्लेख कीजिए।
उत्तर:

  • राष्ट्रपति के पद पर दो बार से अधिक एक ही व्यक्ति का न रहना भी प्रथा पर आधारित है क्योंकि संविधान कोई ऐसी पाबन्दी नहीं लगाता है।
  • राज्यपाल के पद पर नियुक्त होने वाला व्यक्ति उस राज्य का निवासी नहीं होना चाहिए। यह भी प्रथा पर आधारित है।

प्रश्न 22.
भारतीय संविधान 26 जनवरी को ही क्यों लागू किया गया?
उत्तर:
स्वतंत्रता आंदोलन के दौरान 26 जनवरी को पूर्ण स्वराज्य दिवस के रूप में मनाने का निर्णय किया गया था। इस तिथि को विशेष महत्त्व देने एवं यादगार बनाने हेतु नया संविधान 26 जनवरी, 1950 को ही लागू किया गया।

लघूत्तरात्मक प्रश्न

प्रश्न 1.
संविधान की कोई दो परिभाषाएँ दीजिए। उत्तर-संविधान की दो परिभाषाएँ निम्नलिखित हैं

1. गिलक्राइस्ट (Gilchrist) के अनुसार, “संविधान लिखित अथवा अलिखित नियमों अथवा कानूनों का वह समूह होता है जिनके द्वारा सरकार का संगठन, सरकार की शक्तियों का विभिन्न अंगों में वितरण और इन शक्तियों के प्रयोग के सामान्य सिद्धांत निश्चित किए जाते हैं।”

2. लार्ड ब्राईस (Lord Bryce) के अनुसार, “संविधान ऐसे निश्चित नियमों का संग्रह होता है, जिसमें सरकार की कार्यविधि निहित होती है और जिनके द्वारा उसका संचालन होता है।”

प्रश्न 2.
लिखित एवं अलिखित संविधान में अंतर स्पष्ट कीजिए।
उत्तर:
लिखित संविधान-लिखित संविधान वह संविधान है जो पूर्ण रूप से या जिसका अधिकांश भाग लिखित होता है। इसके अंतर्गत सरकार के रूप और उसके संगठन के बारे में तथा सरकार के तीनों अंगों के पारस्परिक संबंधों को स्पष्ट रूप में लिख दिया जाता है। इसके अतिरिक्त नागरिकों के अधिकारों तथा उनके सरकार से संबंधों के बारे में लिख दिया जाता है।

लिखित संविधान में उस प्रणाली का भी वर्णन किया जा सकता है जिसके द्वारा संविधान में संशोधन किया जा सकता है। ऐसा संविधान प्रायः एक संविधान सभा द्वारा बनाया जाता है जो कुछ समय लगाकर उसे एक निश्चित लेख के रूप में तैयार करती है और ऊपर दिए गए सभी विषयों पर निर्णय लेकर उन्हें लिखित रूप देती है। कई बार कुछ उदार शासकों द्वारा स्वयं ही कुछ लोगों को यह प्रदान किया जाता है. या लोग स्वयं राजा की निरंकुश शक्तियों का बलपूर्वक विरोध करके उसे ऐसा करने पर विवश कर देते हैं।

अलिखित संविधान इसके विपरीत अलिखित संविधान वह संविधान है जिसका अधिकतर भाग अलिखित होता है। ऐसा संविधान प्रायः रीति-रिवाजों, परंपराओं तथा समय-समय पर दिए गए न्यायिक निर्णयों पर आधारित होता है। इसमें शासन का रूप, सरकार की शक्तियाँ तथा नागरिकों के अधिकार आदि मुख्यतः रीति-रिवाजों पर ही आधारित होते हैं जिनका धीरे-धीरे विकास हुआ है।

वहाँ पर संविधान में संशोधन के लिए भी कोई विशेष प्रणाली नहीं अपनाई जाती, बल्कि बदलती हुई परिस्थितियों के अनुसार उसमें आसानी से परिवर्तन कर लिया जाता है। ऐसा संविधान न तो किसी संविधान सभा द्वारा एक निश्चित समय पर बनाया जाता है और न ही किसी सम्राट् द्वारा प्रदान किया जाता है, बल्कि इसका आवश्यकता के अनुसार विकास होता है। इसका मुख्य उदाहरण इंग्लैण्ड का संविधान है जिसका कभी निर्माण नहीं किया गया और जो मुख्यतः रीति-रिवाज़ों पर आधारित है।

प्रश्न 3.
संविधान सभा में निर्णय लेने की प्रक्रिया का संक्षेप में वर्णन कीजिए।
उत्तर:
संविधान सभा में काँग्रेस दल के सदस्यों का एक बहुत बड़ा बहुमत था, लेकिन इस दल ने कभी किसी भी फैसले को बहुमत के आधार पर लादने का प्रयास नहीं किया। अगर किसी भी संवैधानिक प्रश्न पर नेहरू एवं पटेल दोनों ही सहमत होते थे, तो उस विषय में निर्णय बहुत सुगमता के साथ ले लिया जाता था। अगर उन दोनों में आपस में मतभेद होता था तो उनके समर्थक एक-दूसरे का समर्थन करते थे।

संविधान सभा में लंबी-लंबी बहसें होती थीं तथा जब तक नेहरू एवं पटेल अपने मतभेदों को समाप्त नहीं कर पाते थे, तब तक कोई निर्णय नहीं हो पाता था। लेकिन ये दोनों विभूतियाँ जल्दी ही विषय पर सहमति प्रकट कर देती थीं। इसके अलावा संविधान सभा में काँग्रेस दल के सदस्यों की महत्त्वपूर्ण विषयों पर बैठक होती थी, खुलकर वाद-विवाद होता था, मतदान भी हो जाता था एवं अंत में विषय पर निर्णय ले लिया जाता था।

प्रत्येक महत्त्वपूर्ण निर्णय लेने में अल्पतंत्र वर्ग के नेताओं का विशेष महत्त्व होता था। इस प्रकार यह स्पष्ट विदित हो जाता है कि इन नेताओं ने प्रजातंत्रीय सिद्धांतों के आधार पर संविधान सभा में निर्णय सहमति के द्वारा ही लिए थे। यही कारण है कि हमारा संविधान इतना व्यावहारिक बन सका है।

HBSE 11th Class Political Science Important Questions Chapter 1 संविधान : क्यों और कैसे?

प्रश्न 4.
भारतीय संविधान पर पड़े किन्हीं चार अमेरिकी संविधान के प्रावधानों का उल्लेख कीजिए।
उत्तर:
भारत के संविधान पर संयुक्त राज्य अमेरिका (U.S.A.) के संविधान का भी काफी प्रभाव है जैसे

(1) हमारे संविधान के आरम्भ से पूर्व एक प्रस्तावना है। इसमें अमेरिका के संविधान की प्रस्तावना की भान्ति यह लिखा गया है कि संविधान का निर्माण करने वाले ‘हम भारत के लोग’ (‘We the People of India’) हैं।

(2) हमारे मौलिक अधिकार संयुक्त राज्य अमेरिका के बिल ऑफ राईट्स (Bill of Rights) से मिलते-जुलते हैं।

(3) भारत की सुप्रीम कोर्ट (Supreme Court of India) के कार्य और दर्जा अमेरिका की सुप्रीम कोर्ट जैसे हैं।

(4) न्यायपालिका की स्वतंत्रता का सिद्धांत।

प्रश्न 5.
भारतीय संविधान पर पड़े किन्हीं चार ब्रिटिश संविधान के प्रभावों का उल्लेख कीजिए।
उत्तर:
भारतीय संविधान में सबसे अधिक प्रभाव ब्रिटिश संविधान का है। यह शायद इस कारण है कि अंग्रेजों ने स्वतंत्रता-प्राप्ति से पूर्व लगभग 200 वर्ष तक भारत पर शासन किया और इस बीच भारतीयों को उनकी राजनीतिक संस्थाओं का काफी अनुभव हुआ। ब्रिटिश संविधान से हमने निम्नलिखित बातें अपनाई हैं

  • इंग्लैण्ड के सम्राट की भान्ति भारत का राष्ट्रपति नाममात्र का तथा संवैधानिक मुखिया है।
  • संसदीय प्रणाली इंग्लैण्ड की नकल है।
  • मंत्रिमंडल में प्रधानमंत्री का श्रेष्ठ स्थान है तथा मंत्रिमंडल वास्तविक कार्यपालिका है जैसे कि ब्रिटेन में है।
  • संसद का द्विसदनीय विधानमंडल होना और लोकसभा इंग्लैण्ड के कॉमन सदन की भान्ति अधिक शक्तिशाली है।

प्रश्न 6.
भारतीय संविधान के विकास में भूमिका निभाने वाले संसद द्वारा पारित किन्हीं पाँच अधिनियमों का उल्लेख कीजिए।
उत्तर:
संविधान के अनुसार अनेक शासकीय व्यवस्थाओं को पूरा करने के लिए संसद को कानून बनाने का अधिकार दिया गया है। इसके अधीन संसद द्वारा बनाए गए निम्नलिखित कानून उल्लेखनीय हैं

  • निरोधक नजरबन्दी अधिनियम, 1950,
  • 1950, 1951 का जन-प्रतिनिधित्व अधिनियम,
  • 1951 का वित्त आयोग का अधिनियम,
  • 1951 का राष्ट्रपति तथा उप-राष्ट्रपति चुनाव अधिनियम,
  • भारतीय नागरिकता अधिनियम, 1955 ।

प्रश्न 7.
भारतीय संविधान के निर्माण पर पड़े ‘भारत सरकार अधिनियम, 1935’ के किन्हीं पाँच प्रभावों का उल्लेख कीजिए।
उत्तर:
भारत के वर्तमान संविधान का अधिकांश भाग भारत सरकार अधिनियम, 1935 पर आधारित है। यह निम्नलिखित बातों से स्पष्ट है

  • वर्तमान संविधान में संघीय शासन की व्यवस्था सन् 1935 के अधिनियम पर आधारित है।
  • केन्द्र तथा राज्यों के बीच शक्तियों का विभाजन भी इसी अधिनियम पर आधारित है। उस एक्ट की भान्ति नए संविधान में शासन-शक्तियों का तीन सूचियों
    (a) संघीय सूची,
    (b) राज्य सूची तथा
    (c) समवर्ती सूची में विभाजन किया गया है।
  • सन् 1935 के एक्ट की भान्ति नए संविधान में भी केन्द्रीय सरकार को अधिक शक्तिशाली बनाया गया है।
  • सन् 1935 के एक्ट की भान्ति नए संविधान द्वारा भी केन्द्र में द्विसदनीय विधानमंडल की स्थापना की गई है।
  • नए संविधान की धारा 356 के अंतर्गत राज्यों में राष्ट्रपति शासन लागू करने की व्यवस्था का आधार सन 1935 के एक्ट का सैक्शन 93 है।

प्रश्न 8.
भारतीय संविधान के विकास के स्रोत के रूप में भूमिका निभाने वाले किन्हीं पाँच सवैधानिक विशेषज्ञों की टिप्पणियों का उल्लेख कीजिए।
उत्तर:
सवैधानिक विशेषज्ञों के विचार और टिप्पणियाँ भी हमारे संविधान का महत्त्वपूर्ण स्रोत हैं। इन विशेषज्ञों ने संविधान के बारे में अपने-अपने विचार अपने संवैधानिक लेखों में व्यक्त किए हैं। चाहे इन विचारों को कानूनी मान्यता प्राप्त नहीं फिर भी इनकी उपेक्षा नहीं की जा सकती। इन विचारकों का प्रभाव हमारे कानून निर्माताओं तथा न्यायाधीशों पर अवश्य पड़ सकता है। इस श्रेणी के विशेषज्ञों व उनके संवैधानिक लेखों के नाम निम्नलिखित हैं

  • D.D. Basu : Commentary on the Constitution of India.
  • V.N. Rao : The Constitution of India.
  • K.V. Rao : Parliamentary Democracy of India.
  • N.A. Palkivala : Our Constitution : Defaced and Defiled.
  • M.C. Setelvad : The Indian Constitution.

निबंधात्मक प्रश्न

प्रश्न 1.
‘संविधान’ शब्द से क्या अभिप्राय है? हमें संविधान की आवश्यकता क्यों है?
उत्तर:
प्रत्येक राज्य का प्रायः एक संविधान होता है। साधारण शब्दों में, संविधान उन मौलिक नियमों, सिद्धांतों तथा परंपराओं का संग्रह होता है, जिनके अनुसार राज्य की सरकार का गठन, सरकार के कार्य, नागरिकों के अधिकार तथा नागरिकों और सरकार के बीच संबंध को निश्चित किया जाता है। शासन का स्वरूप लोकतांत्रिक हो या अधिनायकवादी, कुछ ऐसे नियमों के अस्तित्व से इन्कार नहीं किया जा सकता जो राज्य में विभिन्न राजनीतिक संस्थाओं तथा शासकों की भूमिका को निश्चित करते हैं।

इन नियमों के संग्रह को ही संविधान कहा जाता है। संविधान में शासन के विभिन्न अंगों तथा उनके पारस्परिक संबंधों का विवरण होता है। इन संबंधों को निश्चित करने हेतु कुछ नियम बनाए जाते हैं, जिनके आधार पर शासन का संचालन सुचारू रूप से संभव हो जाता है तथा शासन के विभिन्न अंगों में टकराव की संभावनाएँ कम हो जाती हैं। संविधान के अभाव में शासन के सभी कार्य निरंकुश शासकों की इच्छानुसार ही चलाए जाएँगे जिससे नागरिकों पर अत्याचार होने की संभावना बनी रहेगी।

ऐसे शासक से छुटकारा पाने के लिए नागरिकों को अवश्य ही विद्रोह का सहारा लेना पड़ेगा जिससे राज्य में अशांति तथा अव्यवस्था फैल जाएगी। इस प्रकार एक देश के नागरिकों हेतु एक सभ्य समाज एवं कुशल तथा मर्यादित सरकार का अस्तित्व एक संविधान की व्यवस्थाओं पर ही निर्भर करता है।

संविधान की दो परिभाषाएँ निम्नलिखित हैं
1. गिलक्राइस्ट (Gilchrist) के अनुसार, “संविधान लिखित अथवा अलिखित नियमों अथवा कानूनों का वह समूह होता है जिनके द्वारा सरकार का संगठन, सरकार की शक्तियों का विभिन्न अंगों में वितरण और इन शक्तियों के प्रयोग के सामान्य सिद्धांत निश्चित किए जाते हैं।”

2. लार्ड ब्राईस (Lord Bryce) के अनुसार, “संविधान ऐसे निश्चित नियमों का संग्रह होता है, जिसमें सरकार की कार्यविधि निहित होती है और जिनके द्वारा उसका संचालन होता है।”

हमारे लिए संविधान क्यों आवश्यक है? (Why do we need a Constitution?)-किसी भी देश के लिए उसका संविधान बहुत ही महत्त्वपूर्ण होता है। यह सरकार की शक्तियों को निश्चित करता है तथा उन पर अंकुश लगाता है। संविधान सरकार के । विभिन्न अंगों की शक्तियों को भी निश्चित करता है जिससे उनमें झगड़े की संभावना नहीं रहती। यह नागरिकों के अधिकारों तथा सरकार के साथ नागरिकों के संबंध भी निश्चित करता है।

संविधान के द्वारा लोग अपने अधिकारों की रक्षा कर सकते हैं तथा सरकार पर अंकुश लगा सकते हैं। संविधान के अभाव में शासन के सभी कार्य शासकों की इच्छानुसार ही चलाए जाएँगे, जिससे नागरिकों पर अत्याचार होने की संभावना बनी रहेगी। ऐसे शासक से छुटकारा पाने के लिए नागरिकों को विद्रोह का सहारा लेना पड़ेगा जिससे देश में अशांति व अव्यवस्था का वातावरण बना रहेगा। प्रो० जैलीनेक (Prof. Jellineck) ने लिखा है, “संविधान के बिना राज्य नहीं रहेगा, बल्कि अराजकता होगी।”

संविधान समाज के लिए निम्नलिखित कार्य करता है
(1) सर्वप्रथम, संविधान कुछ ऐसे मौलिक नियम निश्चित करता है जो समाज में रहने वाले लोगों में समन्वय तथा आपसी विश्वास की स्थापना करते हैं। संविधान के द्वारा ही किसी राज्य के स्वरूप को निश्चित किया जा सकता है।

(2) संविधान ही सरकार के विभिन्न अंगों पर नियंत्रण स्थापित करता है और उन्हें तानाशाह होने से बचाता है।

(3) संविधान ही नागरिकों के मौलिक अधिकारों और कर्तव्यों की रक्षा करता है। यह नागरिक स्वतंत्रता (Civil Liberties) की धरोहर है। व्यवहार में इन अधिकारों पर कुछ पाबंदियाँ लगानी आवश्यक होती हैं। संविधान ही उन परिस्थितियों को निश्चित करता हैं, जिनमें सरकार द्वारा नागरिकों के अधिकारों को छीना जा सकता है। भारत में भी राष्ट्रीय संकट के समय सरकार को नागरिकों पर पाबंदी लगाने का अधिकार दिया गया है।

(4) संविधान एक ध्रुव तारे के समान है, जो शासक को हमेशा दिशा-निर्देश देता है और उसका मार्गदर्शन करता है।

(5) संविधान ही सरकार के विभिन्न अंगों के बीच संबंध बनाए रखता है और उनमें जो मनमुटाव पैदा होता है, उसे स्पष्ट करता है।

(6) संविधान एक ऐसा आईना (Mirror) है जिसमें उस देश के भूत, वर्तमान और भविष्य की झलक मिलती है।

(7) विश्व के अधिकतर पुराने संविधान ऐसे हैं जिनमें केवल सरकार के गठन तथा शक्तियों और उन पर लगे प्रतिबंधों की ही व्यवस्था की गई है, परंतु, 20वीं शताब्दी में बने अनेक ऐसे संविधान हैं, जिनमें इन बातों के अतिरिक्त सरकार से नागरिकों की भलाई के लिए कुछ सकारात्मक कार्य करने के लिए भी कहा गया है। भारतीय संविधान भी एक ऐसा ही संविधान है।

इसके द्वारा भारतीय समाज में मौजूदं सामाजिक असमानता को समाप्त करने के लिए व्यवस्था की गई है। सरकार द्वारा कानून पास करके छुआछूत को समाप्त कर दिया गया है। इस प्रकार संविधान द्वारा सरकार को बच्चों की शिक्षा तथा नागरिकों के स्वास्थ्य की देखभाल करने की जिम्मेवारी भी सौंपी गई है।

सरकार को नागरिकों के लिए रोज़गार की व्यवस्था करने तथा उनको इतने आर्थिक साधन जुटाने के लिए भी कहा गया है, जिससे नागरिक एक सम्मानपूर्वक जीवन व्यतीत कर सकें। इसी प्रकार दक्षिण अफ्रीका का संविधान सरकार को यह निर्देश देता है कि वह देश में लंबे समय से चली आ रही जातीय भेदभाव की नीति को समाप्त करे तथा सभी के लिए मकान तथा स्वास्थ्य सेवाएँ उपलब्ध कराए। इंडोनेशिया का संविधान भी सरकार को राष्ट्रीय शिक्षा व्यवस्था (National Education System) की व्यवस्था करने का निर्देश देता है।

प्रश्न 2.
भारत की संविधान सभा के गठन का वर्णन कीजिए। भारतीय संविधान के उद्देश्य-प्रस्ताव की व्याख्या कीजिए।
उत्तर:
भारत के वर्तमान संविधान का निर्माण एक संविधान सभा ने किया, जिसकी स्थापना 1946 में मंत्रिमंडल मिशन योजना के अंतर्गत की गई थी। इस संविधान सभा के सदस्यों की कुल संख्या 389 निश्चित की गई, जिसमें से 292 ब्रिटिश प्रांतों के प्रतिनिधि, 4 चीफ कमिश्नर वाले प्रांतों के तथा 93 देशी रियासतों के प्रतिनिधि होने थे।

प्रांतों के 296 सदस्यों के चुनाव जुलाई, 1946 में करवाए गए। इनमें से 212 स्थान काँग्रेस को, 73 मुस्लिम लीग को एवं 11 स्थान अन्य दलों को प्राप्त हुए। काँग्रेस की इस शानदार सफलता को देखकर मुस्लिम लीग को बड़ी निराशा हुई और उसने संविधान सभा का बहिष्कार करने का निर्णय किया। 9 दिसम्बर, 1946 को संविधान सभा का विधिवत् उद्घाटन हुआ और भारत के भविष्य के संविधान का निर्माण करने के लिए प्रतिनिधि पहली बार इकट्ठे हुए, लेकिन मुस्लिम लीग के किसी भी सदस्य ने इसमें भाग नहीं लिया और उसने पाकिस्तान के लिए अलग संविधान सभा की माँग शुरू कर दी।

संविधान सभा का अधिवेशन सच्चिदानन्द सिन्हा, जो संविधान सभा के सबसे वयोवृद्ध सदस्य थे, की अध्यक्षता में शुरू हुआ। 11 दिसम्बर, 1946 को डॉ० राजेन्द्र प्रसाद को संविधान सभा का निर्विरोध रूप से स्थायी अध्यक्ष चुन लिया गया। संविधान सभा के अधिवेशन चलते रहे, लेकिन मुस्लिम लीग ने उनमें भाग नहीं लिया। राजनीतिक गतिविधियों के कारण माऊंट बैटन योजना 3 जून, 1947 के अनुसार भारत स्वतंत्रता अधिनियम, 1947 पारित किया गया।

इस कानून के अनुसार भारत दो डोमिनियन राज्यों भारत और पाकिस्तान में विभाजित हो गया। इस विभाजन के कारण जहाँ संविधान सभा के गठन में भी परिवर्तन हुआ, वहाँ संविधान सभा के स्तर में भी परिवर्तन हो गया। स्वतंत्रता-प्राप्ति और विभाजन के बाद संविधान सभा के कुल सदस्यों की संख्या 324 रह गई, जिनमें से 235 प्रांतों के प्रतिनिधि और 89 रियासतों के प्रतिनिधि थे।

पंजाब और बंगाल के दो भाग, जो भारत में रह गए थे, उनके लिए फिर से चुनाव हुआ। इन सदस्यों ने 14 जुलाई, 1947 को संविधान सभा में स्थान ग्रहण किया। स्वाधीनता-प्राप्ति के बाद संविधान सभा पूरी तरह से प्रभुता-संपन्न हो गई थी, क्योंकि कैबिनेट मिशन योजना 1946 के द्वारा उस पर जो प्रतिबंध लगाए गए थे, वे समाप्त हो गए थे। इस संविधान सभा ने संविधान बनाने का कार्य 26 नवम्बर, 1949 को पूरा कर लिया था।

26 जनवरी की यादगार को हमेशा बनाए रखने के लिए संविधान निर्माताओं ने जान-बूझकर संविधान 26 जनवरी, 1950 को लागू किया। 26 जनवरी का दिन ‘भारत में गणतंत्र दिवस’ के रूप में मनाया जाता है। इस तरह से भारत का संविधान भारतीयों द्वारा निर्वाचित प्रतिनिधियों के द्वारा निर्मित किया गया है। इस संविधान सभा में देश के प्रसिद्ध नेता, वकील तथा राजनीतिज्ञ शामिल थे।

इनमें मुख्य-मुख्य के नाम इस प्रकार थे-डॉ० राजेन्द्र प्रसाद, जवाहरलाल नेहरू, मौलाना आज़ाद, सरदार पटेल, गोविंद बल्लभ पंत, अल्लादी कृष्ण स्वामी अय्यर, डॉ० बी०आर० अम्बेडकर, आचार्य कृपलानी, सच्चिदानन्द सिन्हा, गोपाल स्वामी आयंगर, सर फजरूल्ला खां, सर फिरोज शाहनून आदि।

उद्देश्य संबंधी प्रस्ताव (Objective Resolution):
13 दिसम्बर, 1946 को पं० जवाहरलाल नेहरू ने संविधान सभा में उद्देश्य संबंधी प्रस्ताव पेश किए, जो इस प्रकार थे

(1) “संविधान सभा भारत को एक प्रभुसत्ता सम्पन्न प्रजातंत्रीय गणराज्य (Sovereign Democratic Republic) घोषित करने और उसके भविष्य के शासन के लिए संविधान बनाने के अपने दृढ़ और पवित्र निश्चय की घोषणा करती है, तथा

(2) जो क्षेत्र इस समय ब्रिटिश भारत में या भारतीय रियासतों के अंतर्गत हैं एवं भारत के ऐसे अन्य भाग जो ब्रिटिश भारत तथा रियासतों के बाहर हैं, वे सभी अगर प्रभुसत्ता सम्पन्न भारत में मिलना चाहते हैं, तो सभी मिलकर एक संघ का निर्माण करेंगे, तथा

(3) जिसमें उपरोक्त क्षेत्रों का अपनी वर्तमान सीमाओं सहित या ऐसी सीमाओं सहित जो संविधान सभा द्वारा और उसके बाद संविधान की विधि द्वारा निश्चित की जाएगी, स्वायत्त इकाइयों का पद मिलेगा और वह सरकार शासन की सभी शक्तियों का प्रयोग करेगी, सिवाय उन अधिकारों के जो संघ को दिए गए हैं तथा

(4) जिसमें प्रभुसत्ता सम्पन्न स्वतंत्र भारत, इसके संगठित भागों और सरकार के अंगों की समस्त शक्ति तथा अधिकार जनता से प्राप्त किए गए हैं, तथा

(5) जिसमें भारत के सभी लोगों की सामाजिक, आर्थिक और राजनीतिक न्याय, पद तथा अवसर व कानून के समक्ष समानता, विचार, विश्वास, धर्म, पूजा, व्यवसाय, समुदाय बनाने की, कानून तथा सार्वजनिक नैतिकता के अनुसार स्वतंत्रता मिली हुई हो और सुरक्षित हो, तथा

(6) जिसमें अल्पसंख्यक वर्गों, पिछड़े हुए कबीलों और जातियों को काफी सुरक्षा की व्यवस्था होगी, तथा

(7) जिसके द्वारा सभ्य राष्ट्रों के कानून तथा न्याय के अनुसार गणतंत्र के स्थायित्व जल, थल व वायु पर अधिकार होगा।

(8) यह प्राचीन भूमि विश्व में अपना उचित तथा सम्मानित स्थान ग्रहण करती है और मानव-कल्याण व विश्व-शांति के विस्तार में अपना पूर्ण तथा ऐच्छिक योगदान करती है।”

इस प्रस्ताव के अध्ययन से यह स्पष्ट होता है कि इनमें उन आधारभूत उद्देश्यों की घोषणा की गई थी जिनके आधार पर भारत के नए संविधान का निर्माण किया जाना था। इसमें भारत को सम्पूर्ण प्रभुसत्ता सम्पन्न लोकतंत्रीय गणराज्य घोषित करना, जनता को सामाजिक, आर्थिक व राजनीतिक न्याय, समानता तथा अन्य स्वतंत्रताएँ मिलें, पिछड़ी हुई तथा अनुसूचित जातियों के विकास के लिए कुछ संरक्षण हों तथा भारत में ऐसा संघ स्थापित हो जिसमें ब्रिटिश प्रांत, देशी रियासतें तथा अन्य भारतीय क्षेत्र शामिल हों, जो विश्व-शांति को बढ़ावा दें, आदि मुख्य बातें थीं।

इसके महत्त्व की चर्चा करते हुए के०एम० मुन्शी ने कहा था, “नेहरू का उद्देश्य संबंधी यह प्रस्ताव ही हमारे स्वतंत्र गणराज्य की जन्म कुंडली है।” (“Objective resolution cast the horoscope of our Sovereign Democratic Republic.”) इस उद्देश्य-प्रस्ताव को संविधान सभा द्वारा 22 जनवरी, 1947 को सर्वसम्मति से पास कर दिया गया।

HBSE 11th Class Political Science Important Questions Chapter 1 संविधान : क्यों और कैसे?

प्रश्न 3.
भारतीय संविधान के स्रोतों का वर्णन कीजिए।
उत्तर:
भारतीय संविधान एक ऐसा संविधान है जिसमें संसार के अनेक संविधानों के अच्छे तत्त्वों को अपनाया गया है। हमारे संविधान के निर्माताओं का उद्देश्य किसी आदर्श अथवा मौलिक संविधान का निर्माण करना नहीं था, वे तो देश के लिए एक व्यावहारिक तथा कामचलाऊ संविधान का निर्माण करना चाहते थे। अतः भारतीय संविधान के निर्माण में अनेक स्रोतों की सहायता ली गई। भारतीय संविधान के मुख्य स्रोत निम्नलिखित हैं

1. भारत सरकार अधिनियम, 1935 (Government of India Act, 1935):
भारत के वर्तमान संविधान का अधिकांश भाग भारत सरकार अधिनियम, 1935 पर आधारित है। यह निम्नलिखित बातों से स्पष्ट है

  • वर्तमान संविधान में संघीय शासन की व्यवस्था सन् 1935 के अधिनियम पर आधारित है।
  • केन्द्र तथा राज्यों के बीच शक्तियों का विभाजन भी इसी अधिनियम पर आधारित है। उस एक्ट की भान्ति नए संविधान में शासन-शक्तियों का तीन सूचियों
    (a) संघीय सूची,
    (b) राज्य सूची तथा
    (c) समवर्ती सूची में विभाजन किया गया है।
  • सन 1935 के एक्ट की भान्ति नए संविधान में भी केन्द्रीय सरकार को अधिक शक्तिशाली बनाया गया है।
  • सन 1935 के एक्ट की भान्ति नए संविधान द्वारा भी केन्द्र में द्विसदनीय विधानमंडल की स्थापना की गई है।
  • नए संविधान की धारा 356 के अंतर्गत राज्यों में राष्ट्रपति शासन लागू करने की व्यवस्था का आधार सन 1935 के एक्ट का सैक्शन 93 है।
  • नए संविधान की धारा 352 के अंतर्गत राष्ट्रपति को प्राप्त संकटकालीन शक्तियाँ सन 1935 के एक्ट के सैक्शन 10 की नकल है।

2. विदेशी संविधानों का प्रभाव (Influence of Foreign Constitutions) भारत के संविधान पर विश्व के विभिन्न देशों के संविधानों का प्रभाव स्पष्ट दिखाई देता है। भारतीय संविधान के निर्माताओं ने संसार के अनेक देशों के संविधानों का अध्ययन किया और अपने देश का संविधान बनाते समय उन देशों में प्रचलित सवैधानिक प्रणालियों में से वे बातें अपनाईं जो हमारे देश की परिस्थितियों के अनुकूल थीं. मुख्य विदेशी संविधान जिनका प्रभाव विशेष रूप से पड़ा है, निम्नलिखित हैं

(क) ब्रिटिश संविधान (British Constitution)-भारतीय संविधान में सबसे अधिक प्रभाव ब्रिटिश संविधान का है। यह शायद इस कारण है कि अंग्रेजों ने स्वतंत्रता-प्राप्ति से पूर्व लगभग 200 वर्ष तक भारत पर शासन किया और इस बीच भारतीयों को उनकी राजनीतिक संस्थाओं का काफी अनुभव हुआ। ब्रिटिश संविधान से हमने निम्नलिखित बातें अपनाई हैं

  • इंग्लैण्ड के सम्राट की भान्ति भारत का राष्ट्रपति नाममात्र का तथा संवैधानिक मुखिया है।
  • संसदीय प्रणाली इंग्लैण्ड की नकल है।
  • मंत्रिमंडल में प्रधानमंत्री का श्रेष्ठ स्थान है तथा मंत्रिमंडल वास्तविक कार्यपालिका है जैसे कि ब्रिटेन में है।
  • मंत्रिमंडल सामूहिक तौर पर लोकसभा के प्रति उत्तरदायी है।
  • संसद का द्विसदनीय विधानमंडल होना और लोकसभा इंग्लैण्ड के कॉमन सदन की भान्ति अधिक शक्तिशाली है।।
  •  कानून के शासन का होना (Rule of Law)।

परंतु भारत और ब्रिटेन के संविधानों में इन सब बातों में एकरूपता होते हुए भी मूल अंतर इस बात का है कि इंग्लैण्ड में इन सबके बारे में कोई लिखित व्यवस्था नहीं की गई जबकि भारत में उन्हें लिखित संविधान द्वारा अपनाया गया है।

(ख) संयुक्त राज्य अमेरिका का संविधान (Constitution of the U.S.A.):
भारत के संविधान पर संयुक्त राज्य अमेरिका (U.S.A.) के संविधान का भी काफी प्रभाव है; जैसे

  • हमारे संविधान के आरम्भ से पूर्व एक प्रस्तावना है। इसमें अमेरिका के संविधान की प्रस्तावना की भान्ति यह लिखा गया है कि संविधान का निर्माण करने वाले ‘हम भारत के लोग’ ‘We the People of India’ हैं।
  • हमारे मौलिक अधिकार संयुक्त राज्य अमेरिका के बिल ऑफ राईट्स (Bill of Rights) से मिलते-जुलते हैं।
  • भारत की सुप्रीम कोर्ट (Supreme Court of India) के कार्य और दर्जा अमेरिका की सुप्रीम कोर्ट जैसे हैं।
  • न्यायपालिका की स्वतंत्रता का सिद्धांत।
  • उप-राष्ट्रपति के कार्य और दर्जा।

(ग) कनाडा का संविधान (Canadian. Constitution) कुछ बातें हमने कनाडा के संविधान से भी अपनाई हैं; जैसे

  • कनाडा की भान्ति हमने भारत को राज्यों का संघ (Union of States) माना है।
  • संघात्मक ढाँचा अपनाने के साथ-साथ केन्द्र को अधिक शक्तिशाली बनाया है।
  • शेष शक्तियाँ केन्द्र को दी गई हैं।

(घ) ऑस्ट्रेलिया का संविधान (Australian Constitution)-ऑस्ट्रेलिया की भान्ति हमारे संविधान में समवर्ती-सूची की व्यवस्था और केन्द्र तथा राज्यों में झगड़ों का निपटारा करने की विधि (अनुच्छेद 251) अपनाई गई है।

(ङ) आयरलैण्ड का संविधान (The Irish Constitution)-राज्य नीति के निदेशक सिद्धांत, राष्ट्रपति के एक विशेष निर्वाचन मंडल द्वारा चुनाव की व्यवस्था, राज्यसभा के सदस्यों में मनोनीत सदस्यों की व्यवस्था आदि आयरलैण्ड के संविधान पर आधारित है।

(च) अन्य संविधान (Other Constitutions)-उपरोक्त संविधानों के अतिरिक्त कई अन्य संविधानों ने भी हमारे संविधान को प्रभावित किया है; जैसे राष्ट्रपति की संकटकालीन शक्तियों का स्रोत जर्मनी का संविधान है। इसी प्रकार संवैधानिक संशोधन की प्रक्रिया तथा राज्यसभा के सदस्यों की निर्वाचन विधि दक्षिणी अफ्रीका के संविधान की देन है।

3. संसद द्वारा पास किए गए अधिनियम (Statutes)-संविधान के अनुसार अनेक शासकीय व्यवस्थाओं को पूरा करने के लिए संसद को कानून बनाने का अधिकार दिया गया है। इसके अधीन संसद द्वारा बनाए गए निम्नलिखित कानून उल्लेखनीय हैं–

  • निरोधक नजरबन्दी अधिनियम, 1950,
  • 1950, 1951 का जन-प्रतिनिधित्व अधिनियम,
  • 1951 का वित्त आयोग का अधिनियम,
  • 1951 का राष्ट्रपति तथा उप-राष्ट्रपति चुनाव अधिनियम,
  • भारतीय नागरिकता अधिनियम, 1955,
  • सर्वोच्च न्यायालय (न्यायाधीश संख्या) अधिनियम, 1956,
  • सरकारी भाषा अधिनियम, 1963,
  • पंजाब पुनर्गठन अधिनियम, 1966,
  • 1971 का आन्तरिक सुरक्षा स्थापित रखने संबंधी एक्ट,
  • 1977 का प्रधानमंत्री तथा स्पीकर के चुनाव-विवादों के संबंध में अधिनियम,
  • 1977 का राष्ट्रपति और उप-राष्ट्रपति के चुनाव के विवाद के संबंध में अधिनियम,
  • 1981 का राष्ट्रीय सुरक्षा संबंधी कानून,
  • राष्ट्रीय राजधानी क्षेत्र सरकार कानून, 1992,
  • पोटा, 2002

4. मसौदा संविधान, 1948 (Draft Constitution of 1948)-भारतीय संविधान का एक महत्त्वपूर्ण स्रोत सन 1948 का मसौदा संविधान है, जिसे डॉ० अम्बेडकर की अध्यक्षता में स्थापित की गई मसौदा समिति ने तैयार किया। इसमें 315 अनुच्छेद तथा 8 सूचियाँ थीं। इस पर संविधान सभा में खूब वाद-विवाद हुआ। सदस्यों द्वारा इसमें 7635 संशोधन पेश किए गए जिनमें से 2473 पर विचार किया गया। नए संविधान के अधिकांश अनुच्छेद इसी मसौदा संविधान में से ही लिए गए हैं।

5. संविधान सभा के विवाद (Debates of Constituent Assembly) भारत के वर्तमान संविधान की रूप-रेखा निर्धारित करने में संविधान सभा में हुए वाद-विवाद भी विशेष महत्त्व रखते हैं। इन विवादों में उच्चकोटि के विद्वानों, कानून शास्त्रियों तथा राजनीतिज्ञों ने भाग लिया जो संविधान सभा के सदस्य थे। गोपालन बनाम मद्रास राज्य (Gopalan V/s State of Madras) के मुकद्दमें में उच्चतम न्यायालय में संविधान सभा की रिपोर्ट का उदाहरण दिया गया था।

6. न्यायिक निर्णय (Judicial Decisions) भारतीय संविधान के विकास में सर्वोच्च न्यायालय तथा उच्च न्यायालयों के निर्णय भी विशेष महत्त्व रखते हैं और ये भी हमारे संविधान का एक महत्त्वपूर्ण स्रोत हैं। इन न्यायालयों ने संविधान के रक्षक और व्याख्याता होने के नाते कई प्रसिद्ध फैसले दिए हैं जो अब संविधान का भाग बन गए हैं; जैसे (Gopalan V/s State of Madras के मुकद्दमे में सुप्रीम कोर्ट ने निजी स्वतंत्रता के क्षेत्र की व्याख्या की।

इसी प्रकार Madras V/s Champkan के मुकद्दमे में न्यायालय ने यह फैसला दिया कि राज्य-नीति के निदेशक सिद्धांत मौलिक अधिकारों से ऊपर नहीं। I.C.,Golaknath V/s State of Punjab के मुकद्दमे में यह कहा गया कि संसद मौलिक अधिकारों को नहीं बदल सकती। सर्वोच्च न्यायालय ने सन् 1973 के प्रसिद्ध Fundamental Rights Case में अपने सन 1967 के फैसले को बदल दिया और संसद के संविधान के किसी भी भाग में संशोधन । करने के अधिकार को मान्यता दी।

7. सवैधानिक विशेषज्ञों के विचार (Views of Constitutional Experts)-संवैधानिक विशेषज्ञों के विचार और टिप्पणियाँ भी हमारे संविधान का महत्त्वपूर्ण स्रोत हैं। इन विशेषज्ञों ने संविधान के बारे में अपने-अपने विचार अपने संवैधानिक लेखों में व्यक्त किए हैं। चाहे इन विचारों को कानूनी मान्यता प्राप्त नहीं फिर भी इनकी उपेक्षा नहीं की जा सकती। इन विचारकों का प्रभाव हमारे कानून निर्माताओं तथा न्यायाधीशों पर अवश्य पड़ सकता है।

8. संवैधानिक संशोधन (Constitutional Amendments)-संविधान के लागू होने के समय से लेकर अब तक इसमें लगभग 104 संशोधन (दिसम्बर, 2019 तक) हो चुके हैं, जो अब संविधान के अभिन्न अंग हैं तथा एक मुख्य स्रोत के रूप में कार्य करते हैं। इस बात से इंकार नहीं किया जा सकता कि इन संशोधनों द्वारा मूल संविधान में बहुत परिवर्तन आए हैं।

इनसे बहुत-सी बातों को संविधान से निकाल दिया गया है। प्रथम संशोधन द्वारा नागरिकों के स्वतन्त्रता के अधिकार पर प्रतिबन्ध लगाने की व्यवस्था की गई है। 15वें संशोधन द्वारा उच्च न्यायालयों के न्यायाधीशों की पदावधि 60 वर्ष से बढ़ाकर 62 वर्ष कर दी गई। 22वें संशोधन द्वारा सिन्धी भाषा को भी भारतीय भाषाओं की सूची में जोड़ दिया गया है। 24वें संशोधन द्वारा संसद को यह अधिकार दिया गया कि वह संविधान के किसी भी भाग में, जिसमें मौलिक अधिकारों वाला भाग भी शामिल है, संशोधन कर सकती है।

26वें संशोधन के अनुसार राजा-महाराजाओं के प्रिवी पर्सी (Privy Purses) को समाप्त कर दिया गया। 27वें संशोधन द्वारा मणिपुर तथा मेघालय को पूर्ण राज्य का स्तर दे दिया गया। 42वें संशोधन द्वारा संविधान की प्रस्तावना (Preamble) में संशोधन करके भारत को प्रभुसत्ता सम्पन्न, समाजवादी, धर्म-निरपेक्ष, लोकतांत्रिक गणराज्य (Sovereign, Socialist, Secular, Democratic Republic) घोषित किया गया और संविधान में नागरिकों के मौलिक कर्तव्य से जोड़ दिया गया।

44वें संशोधन के अनुसार, ‘सम्पत्ति के अधिकार’ को मौलिक अधिकारों की सूची से निकाल दिया गया। केन्द्र एवं राज्यों में प्रधानमन्त्री/मुख्यमन्त्री सहित मन्त्रियों की कुल संख्या निचले सदन में सदस्यों की संख्या के 15 प्रतिशत से अधिक नहीं होनी चाहिए। 95वें संशोधन द्वारा अनुसूचित जातियों, जनजातियों तथा एंग्लो इंडियन समुदाय के लिए लोकसभा तथा विधानसभाओं आदि में आरक्षण की अवधि बढ़ाकर 25 जनवरी, 2020 तक कर दी गई।

संविधान में किए गए 96 व 97वें संशोधन सहकारी समितियों (Co-operative Societies) के संबंध में हैं। इनके द्वारा सहकारी समितियों की आर्थिक गतिविधियों को प्रोत्साहन देना है जिससे वे ग्रामीण भारत के विकास में सहायता कर सकें। 98वें (2013) संशोधन द्वारा कर्नाटक के राज्यपाल को यह शक्ति दी गई है कि हैदराबाद-कर्नाटक खण्ड (Hyderabad- Karnataka Region) के विकास के लिए कदम उठा सकें।

सन् 2015 में हुए 99वें संशोधन द्वारा भारत तथा बांग्लादेश के बीच हुई सन्धि के अन्तर्गत दोनों देशों के बीच भू-भाग परिवर्तन के परिणामस्वरूप उन क्षेत्रों में रहने वाले व्यक्तियों को नागरिकता देने के अधिकार से संबंधित है। . ‘वस्तु एवं सेवा कर’ सम्बन्धी 101वां संशोधन अधिनियम राज्यसभा एवं लोकसभा में क्रमशः 3 अगस्त एवं 8 अगस्त, 2016 – को पारित करने के उपरान्त 8 सितम्बर, 2016 को राष्ट्रपति ने अपनी स्वीकृति प्रदान की।

यद्यपि उक्त संशोधन अधिनियम संख्या की दृष्टि से 122वां था, परन्तु पूर्व में संसद द्वारा 100 संशोधन विधेयक पारित किए जा चुके थे अतः संशोधन की दृष्टि से 101वां संशोधन विधेयक था। राष्ट्रीय पिछड़ा वर्ग आयोग (NCBC) को संवैधानिक दर्जा देने सम्बन्धी 123वाँ संविधान संशोधन विधेयक संसद से पारित होने के पश्चात् राष्ट्रपति की स्वीकृति मिलने पर 11 अगस्त, 2018 को लागू हुआ जो संशोधन की संख्या दृष्टि से 102वां संशोधन विधेयक था।

इसी क्रम में 124वां संविधान संशोधन विधेयक के रूप में 9 जनवरी, 2019 को संसद द्वारा पारित करने के पश्चात् 12 जनवरी, 2019 को राष्ट्रपति की स्वीकृति मिलने पर सामान्य वर्ग के आर्थिक रूप से कमजोर लोगों को शिक्षा एवं रोजगार में 10 प्रतिशत आरक्षण देने सम्बन्धी संशोधन विधेयक लागू किया गया जो संवैधानिक संशोधन की संख्या दृष्टि से 103वाँ संशोधन विधेयक है। इसी क्रम में 104वें सवैधानिक संशोधन (संशोधन प्रस्ताव संख्या 126) द्वारा दिसम्बर, 2019 में अनुसूचित जाति एवं अनुसूचित जनजाति के लोकसभा एवं राज्य विधानसभाओं के साथ-साथ सरकारी सेवाओं में भी आरक्षण की अवधि को दस वर्ष बढ़ाकर 25 जनवरी, 2030 तक कर दिया गया है।

इस प्रकार संविधान के लागू होने के समय से लेकर अब तक (दिसम्बर, 2019) इसमें कुल 104 संशोधन हो चुके हैं। इस प्रकार इन संशोधनों द्वारा बहुत परिवर्तन हुए हैं और इनका अध्ययन किए बिना संविधान की पूरी जानकारी प्राप्त नहीं हो सकती।

9. प्रथाएँ (Conventions) यद्यपि भारतीय संविधान एक लिखित संविधान है, फिर भी इस देश में महत्त्वपूर्ण प्रथाएँ विकसित हुई हैं, जो संविधान के संचालन में महत्त्वपूर्ण भूमिका निभाती हैं। इनमें से कुछ प्रथाएँ इस प्रकार हैं

(1) राज्यपाल की नियुक्ति करने का अधिकार राष्ट्रपति के पास है, परंतु प्रथा के आधार पर संघीय सरकार किसी राज्यपाल की नियुक्ति करते समय सम्बन्धित राज्य के मुख्यमंत्री की सलाह लेती है।

(2) अब यह भी प्रथा बन गई है कि राज्यपाल के पद पर नियुक्त होने वाला व्यक्ति उस राज्य का निवासी नहीं होना चाहिए।

(3) लोकसभा तथा विधानमंडलों के अध्यक्ष (स्पीकर) निष्पक्षता के साथ अपना कार्य करते हैं चाहे वे किसी भी दल से सम्बन्धित क्यों न हों।

(4) प्रधानमंत्री की नियुक्ति राष्ट्रपति के द्वारा की जाती है, परंतु प्रथा के आधार पर राष्ट्रपति उसी व्यक्ति को प्रधानमंत्री के पद पर नियुक्त करता है जो लोकसभा में बहुसंख्यक दल का नेता होता है। राष्ट्रपति उसकी नियुक्ति करते समय स्वेच्छा से काम नहीं करता।

(5) राष्ट्रपति का दो बार से अधिक एक ही व्यक्ति का न रहना भी प्रथा पर आधारित है। संविधान के द्वारा ऐसी कोई पाबन्दी नहीं लगाई गई है। निष्कर्ष (Conclusion) ऊपर दिए गए विवरण से यह बात स्पष्ट हो जाती है कि भारतीय संविधान के अनेक स्रोत हैं। हमारे संविधान के निर्माताओं ने विदेशी संविधानों से बहुत-सी बातें ली हैं जिसके परिणामस्वरूप कई बार इसे ‘उधार ली का थैला’ (Bag of Borrowings) अथवा विविध संविधानों की खिचड़ी (Hotch-Potch) कहकर पुकारा जाता है, परंतु यह आलोचना न्यायसंगत नहीं है।

वास्तव में, संविधान निर्माताओं द्वारा विदेशी संविधानों की केवल उन धाराओं तथा व्यवस्थाओं को अपने संविधान में अपना लिया गया जो उन देशों में सफलतापूर्वक कार्य कर चुकी थीं और भारत की परिस्थितियों के अनुकूल थीं।

वस्तु निष्ठ प्रश्न

निम्नलिखित प्रश्नों का उत्तर दिए गए विकल्पों में से उचित विकल्प छाँटकर लिखें

1. निम्नलिखित देश का संविधान अलिखित है
(A) चीन
(B) फ्रांस
(C) स्विटज़रलैंड
(D) इंग्लैंड
उत्तर:
(D) इंग्लैंड

2. भारतीय संविधान का निर्माण करने वाली संविधान सभा की स्थापना हुई
(A) 1947 ई०
(B) 1946 ई०
(C) 1949 ई०
(D) 1950 ई०
उत्तर:
(B) 1946 ई०

3. मूल रूप में संविधान सभा के सदस्यों की संख्या निश्चित की गई थी
(A) 389
(B) 360
(C) 272
(D) 420
उत्तर:
(A) 389

HBSE 11th Class Political Science Important Questions Chapter 1 संविधान : क्यों और कैसे?

4. भारतीय संविधान का निर्माण किया गया
(A) संविधान सभा द्वारा
(B) ब्रिटिश संसद द्वारा
(C) भारतीय संसद द्वारा
(D) ब्रिटिश सम्राट द्वारा
उत्तर:
(A) संविधान सभा द्वारा

5. भारतीय संविधान
(A) लचीला है
(B) कठोर है
(C) अंशतः लचीला तथा अंशतः कठोर है
(D) इनमें से कोई नहीं
उत्तर:
(C) अंशतः लचीला तथा अंशतः कठोर है

6. संविधान सभा की प्रथम बैठक हुई
(A) 7 दिसंबर, 1947
(B) 9 दिसंबर, 1946
(C) 3 जून, 1947
(D) 14 जुलाई, 1947
उत्तर:
(B) 9 दिसंबर, 1946

7. निम्नलिखित देश का संविधान लिखित है
(A) भारत
(B) संयुक्त राज्य अमेरिका
(C) चीन
(D) उपर्युक्त सभी
उत्तर:
(D) उपर्युक्त सभी

8. निम्नलिखित देश का संविधान लचीला है
(A) भारत
(B) संयुक्त राज्य अमेरिका
(C) इंग्लैंड
(D) स्विटजरलैंड
उत्तर:
(C) इंग्लैंड

9. भारतीय संविधान
(A) सरकार के विभिन्न अंगों के गठन तथा
(B) सरकार के विभिन्न अंगों पर नियंत्रण स्थापित करता है शक्तियों को निश्चित करता है। और उन्हें तानाशाह बनने से रोकता है।
(C) नागरिकों के अधिकारों की रक्षा करता है
(D) उपर्युक्त तीनों कार्य करता है।
उत्तर:
(D) उपर्युक्त तीनों कार्य करता है।

10. निम्नलिखित देश का संविधान कठोर है
(A) चीन
(B) इंग्लैंड
(C) संयुक्त राज्य अमेरिका
(D) इनमें से कोई नहीं
उत्तर:
(C) संयुक्त राज्य अमेरिका

11. निम्नलिखित राजनीतिक दल के सदस्यों द्वारा संविधान सभा का बहिष्कार किया गया था
(A) कांग्रेस
(B) अकाली दल
(C) मुस्लिम लीग
(D) भारतीय जनता पार्टी
उत्तर:
(C) मुस्लिम लीग

12. संविधान सभा के सदस्यों का चुनाव किया गया था
(A) वयस्क मताधिकार के आधार पर
(B) जातीय आधार पर
(C) सांप्रदायिक आधार पर
(D) ब्रिटिश सम्राट द्वारा
उत्तर:
(C) सांप्रदायिक आधार पर

13. भारतीय संविधान में अनुसूचियाँ हैं
(A) 7
(B) 8
(C) 10
(D) 12
उत्तर:
(D) 12

14. संविधान सभा में उद्देश्य प्रस्ताव (Objective Resolution) निम्न सदस्य द्वारा पेश किया गया था
(A) जवाहरलाल नेहरू
(B) सरदार पटेल
(C) आचार्य कृपलानी
(D) मौलाना आज़ाद
उत्तर:
(A) जवाहरलाल नेहरू

15. निम्नलिखित भारतीय संविधान का स्रोत नहीं है
(A) भारत सरकार अधिनियम, 1935
(B) इंग्लैंड का संविधान
(C) चीन का संविधान
(D) संयुक्त राज्य अमेरिका का संविधान
उत्तर:
(C) चीन का संविधान

16. भारतीय संविधान बनकर तैयार हो गया था
(A) 26 जनवरी, 1950
(B) 14 नवंबर, 1949
(C) 26 नवंबर, 1949
(D) 30 नवंबर, 1950
उत्तर:
(C) 26 नवंबर, 1949

17. भारत का संविधान लागू हुआ
(A) 26 नवंबर, 1949
(B) 26 जनवरी, 1950
(C) 26 जनवरी, 1949
(D) 15 अगस्त, 1947
उत्तर:
(B) 26 जनवरी, 1950

18. स्वतंत्रता प्राप्ति तथा देश के विभाजन के पश्चात् संविधान सभा के सदस्यों की संख्या रह गई थी
(A) 280
(B) 324
(C) 213
(D) 235
उत्तर:
(B) 324

19. भारतीय संविधान सभा में निम्नलिखित राजनीतिक दल को भारी बहुमत प्राप्त था
(A) कांग्रेस
(B) मुस्लिम लीग
(C) अकाली दल
(D) हिंदू महासभा
उत्तर:
(A) कांग्रेस

20. निम्नलिखित संविधान सभा की मसौदा समिति (Drafting Committee) के अध्यक्ष थे
(A) डॉ० राजेंद्र प्रसाद
(B) डॉ० अंबेडकर
(C) जवाहरलाल नेहरू
(D) के०एम० मुंशी
उत्तर:
(B) डॉ० अंबेडकर

21. भारतीय संविधान सभा के अध्यक्ष थे
(A) जवाहरलाल नेहरू
(B) डॉ० अंबेडकर
(C) डॉ० राजेंद्र प्रसाद
(D) सरदार पटेल
उत्तर:
(C) डॉ० राजेंद्र प्रसाद

22. भारतीय संविधान सभा के निर्माण में लगे
(A) 2 वर्ष 11 महीने 18 दिन
(B) पूरे 3 वर्ष
(C) 2 वर्ष तथा 4 महीने
(D) 2 वर्ष 6 महीने तथा 15 दिन
उत्तर:
(A) 2 वर्ष 11 महीने 18 दिन

23. भारतीय संविधान में संशोधन की प्रक्रिया का वर्णन संविधान के निम्नलिखित अनुच्छेद में किया गया है
(A) अनुच्छेद 324
(B) अनुच्छेद 368
(C) अनुच्छेद 370
(D) अनुच्छेद 326
उत्तर:
(B) अनुच्छेद 368

24. भारतीय संविधान में मौलिक अधिकारों को जोड़ने की प्रेरणा निम्नलिखित संविधान से मिली
(A) संयुक्त राज्य अमेरिका के संविधान से
(B) इंग्लैंड के संविधान से
(C) जर्मनी के संविधान से
(D) कनाडा के संविधान से
उत्तर:
(A) संयुक्त राज्य अमेरिका के संविधान से

25. भारतीय संविधान में दिए गए राज्यनीति के निदेशक सिद्धांतों पर निम्नलिखित देश के संविधान की छाप है
(A) इंग्लैंड के संविधान की
(B) आयरलैंड के संविधान की
(C) जर्मनी के संविधान की
(D) कनाडा के संविधान की
उत्तर:
(B) आयरलैंड के संविधान की

26. निम्नलिखित भारतीय संविधान का स्रोत है
(A) इंग्लैंड का संविधान
(B) संवैधानिक संशोधन
(C) संसद द्वारा पारित अधिनियम
(D) उपर्युक्त तीनों संविधान
उत्तर:
(D) उपर्युक्त तीनों संविधान

निम्नलिखित प्रश्नों का उत्तर एक शब्द में –

1. भारतीय संविधान सभा का गठन कब किया गया?
उत्तर:
सन् 1946 में।

2. भारतीय संविधान सभा की प्रथम बैठक कब हुई?
उत्तर:
9 दिसम्बर, 1946 को।

3. संविधान सभा के प्रथम अस्थायी अध्यक्ष कौन थे?
उत्तर:
डॉ० सच्चिदानन्द

4. प्रारम्भ से ही किस राजनीतिक दल ने संविधान सभा की बैठकों का बहिष्कार किया?
उत्तर:
मुस्लिम लीग ने।

HBSE 11th Class Political Science Important Questions Chapter 1 संविधान : क्यों और कैसे?

5. भारतीय संविधान सभा ने संविधान बनाने का कार्य कब पूर्ण किया?
उत्तर:
26 नवम्बर, 1949 को।

6. भारतीय संविधान कब लागू हुआ?
उत्तर:
26 जनवरी, 1950 को।

7. भारतीय संविधान सभा के किसी एक प्रमुख संवैधानिक परामर्शदाता का नाम लिखिए।
उत्तर:
बी०एन० राव।

8. भारतीय संविधान सभा के मसौदा समिति (Drafting Committee) के अध्यक्ष कौन थे?
उत्तर:
डॉ० बी०आर० अम्बेडकर।

9. भारतीय संविधान सभा की मसौदा समिति में कुल कितने सदस्य थे?
उत्तर:
अध्यक्ष सहित कुल सात सदस्य थे।

10. भारतीय संविधान का निर्माण करने में कितना समय लगा?
उत्तर:
2 वर्ष, 11 महीने एवं 18 दिन।

11. ‘The Indian Constitution-Corner stone of a Nation’ नामक पुस्तक के लेखक कौन हैं?
उत्तर:
जी० ऑस्टिन।

12. वर्तमान संविधान में कितने अनुच्छेद एवं कितनी अनुसूचियाँ हैं?
उत्तर:
वर्तमान संविधान में 395 अनुच्छेद एवं 12 अनुसूचियाँ हैं।

13. भारतीय संविधान में राष्ट्रपति की संकटकालीन शक्तियों का प्रावधान किस देश के संविधान से प्रभावित होकर किया है?
उत्तर:
जर्मनी के संविधान से।

14. भारतीय संविधान में राज्य-नीति के निदेशक सिद्धांतों संबंधी प्रावधान किस देश के संविधान से प्रभावित होकर किया गया है?
उत्तर:
आयरलैण्ड के संविधान से।

15. उद्देश्य प्रस्ताव को संविधान सभा द्वारा कब स्वीकृति प्रदान की गई थी?
उत्तर:
22 जनवरी, 1947 को।

16. डॉ० अम्बडेकर की अध्यक्षता में प्रारूप समिति का गठन कब किया गया?
उत्तर:
29 अगस्त, 1947 को।

17. संविधान सभा की अन्तिम बैठक कब हुई?
उत्तर:
24 जनवरी, 1950 को।

18. मूल भारतीय संविधान में कुल कितने अनुच्छेद, अनुसूचियाँ एवं भाग थे?
उत्तर:
395 अनुच्छेद, 8 अनुसूचियाँ एवं 22 भाग थे।

19. संविधान सभा के सदस्यों ने संविधान पर अन्तिम रूप में हस्ताक्षर कब किए?
उत्तर:
24 जनवरी, 1950 को।

20. संविधान सभा के अध्यक्ष कौन थे?
उत्तर:
डॉ० राजेन्द्र प्रसाद

रिक्त स्थान भरें

1. भारतीय संविधान में कुल ………….. अनुच्छेद हैं।
उत्तर:
395

2. भारतीय संविधान सभा के कुल ………….. अधिवेशन हुए हैं।
उत्तर:
12

3. भारतीय संविधान सभा का गठन ……………. में हुआ।
उत्तर:
1946

HBSE 11th Class Political Science Important Questions Chapter 1 संविधान : क्यों और कैसे?

4. ……………. भारतीय संविधान सभा के अस्थायी अध्यक्ष थे।
उत्तर:
डॉ० सच्चिदानन्द

5. संविधान सभा की प्रथम बैठक ……………. को हुई।
उत्तर:
9 दिसम्बर, 1946

6. ………….. मसौदा समिति के अध्यक्ष थे।
उत्तर:
डॉ० अम्बेडकर

7. मूल संविधान में कुल ………….. अनुसूचियाँ थी।
उत्तर:
8

8. भारतीय संविधान …………… को लागू हुआ।
उत्तर:
26 जनवरी, 1950

9. ………….. संविधान सभा के प्रथम स्थायी अध्यक्ष थे।
उत्तर:
डॉ० राजेन्द्र प्रसाद

10. संविधान सभा के कुल सदस्यों की संख्या ………….. निर्धारित की गई थी।
उत्तर:
389

11. संविधान में …………. सदस्य अनुसूचित जाति के थे।
उत्तर:
26

12. संविधान में ……………. सवैधानिक सलाहकार थे।
उत्तर:
बी.एन. राव

13. संविधान सभा की प्रान्तीय संविधान समिति के अध्यक्ष ………….. थे।
उत्तर:
सरदार पटेल

14. भारतीय संविधान को ………….. द्वारा अपनाया गया।
उत्तर:
संविधान सभा

15. संविधान सभा की अन्तिम एवं 12वीं बैठक ……………. को हुई थी।
उत्तर:
24 जनवरी, 1950